Master the SSAT & ISEE...Peterson’s Master the SSAT & ISEE was designed to be as user-friendly as...

Post on 11-Mar-2020

39 views 4 download

Transcript of Master the SSAT & ISEE...Peterson’s Master the SSAT & ISEE was designed to be as user-friendly as...

Peterson’s

MastertheSSAT®&ISEE®

AboutPeterson’s

Peterson’sprovidestheaccurate,dependable,high-qualityeducationcontentandguidanceyouneedtosucceed.Nomatterwhereyouareonyouracademicorprofessionalpath,youcanrelyonPeterson’sprintanddigitalpublicationsforthemostup-to-dateeducationexplorationdata,experttest-preptools,andtop-notchcareersuccessresources—everythingyouneedtoachieveyourgoals.

Formoreinformation,contactPeterson’s,3ColumbiaCircle,Suite205,Albany,NY12203-5158;800-338-3282Ext.54229;orfindusontheWorldWideWebatwww.petersonsbooks.com.

©2014Peterson’s,aNelnetcompany

Previouseditions©2001,2002,2003,2004,2005,2007,2008,2009

SSATisaregisteredtrademarkoftheSecondarySchoolAdmissionTestBoard,whichdoesnotendorsethisbook.

ISEEisaregisteredtrademarkoftheEducationalRecordsBureau,whichdoesnotendorsethisbook.

ALLRIGHTSRESERVED.Nopartofthisworkcoveredbythecopyrighthereinmaybereproducedorusedinanyformorbyanymeans—graphic,electronic,ormechanical,includingphotocopying,recording,taping,Webdistribution,orinformationstorageandretrievalsystems—withoutthepriorwrittenpermissionofthepublisher.

Forpermissiontousematerialfromthistextorproduct,completethePermissionRequestFormathttp://www.petersonspublishing.com/spa/permissions.aspx.

e-ISBN:978-0-7689-3946-0

NinthEdition

PUBLISHINGUPDATES

Checkoutourwebsiteatwww.petersonspublishing.com/publishingupdatestoseeifthereisanynewinformationregardingthetestandanyrevisionsorcorrectionstothecontentofthisbook.We’vemadesuretheinformationinthisbookisaccurateandup-to-date;however,thetestformatorcontentmayhavechangedsincethetimeofpublication.

Contents

BeforeYouBeginHowtoUseThisBookWhattoStudySpecialStudyFeaturesAnImportantNoteforEighth-andNinth-GradersParents’GuidetoPrivateSecondarySchoolYou’reWellonYourWaytoSuccessGiveUsYourFeedbackTop10StrategiestoRaiseYourScore

PARTI:HIGHSCHOOLENTRANCEEXAMBASICS

1AllAbouttheTestsWhatAretheHighSchoolEntranceExams?HowDoIFindOutWhichExamIMustTake?IfIHaveaChoiceofExams,HowDoIChoose?WhatKindsofQuestionsAreAskedonHighSchoolEntranceExams?HowAretheExamsStructured?WhatDotheAnswerSheetsLookLike?HowAretheExamsScored?

2SSATQuestionsWhatCanYouExpectontheTest?HowDoestheSSATMeasureVerbalAbility?HowDoestheSSATMeasureQuantitativeAbility?HowDoestheSSATMeasureReadingAbility?WhatIstheWritingSample?

3ISEEQuestionsWhatCanYouExpectontheTest?HowDoestheISEEMeasureVerbalAbility?

HowDoestheISEEMeasureQuantitativeAbility?HowDoestheISEEMeasureReadingComprehension?HowDoestheISEEMeasureMathematicsAchievement?WhatIstheEssayQuestion?

PARTII:DIAGNOSINGSTRENGTHSANDWEAKNESSES

4PracticeTest1:DiagnosticTestSection1:Synonyms(SSATandISEE)Section2:VerbalAnalogies(SSATOnly)Section3:SentenceCompletions(ISEEOnly)Section4:ReadingComprehension(SSATandISEE)Section5:QuantitativeAbility(SSATandISEE)Section6:QuantitativeComparisons(ISEEOnly)AnswersandExplanationsEvaluatingYourScoreandPlanningYourStudyTime

PARTIII:VOCABULARYREVIEW

5WordArithmetic(SSATandISEE)WhyDoTheyTestMyVocabulary?HowAreWordsBuilt?HowDoWordPartsWork?ListofCommonWordPartsWordListExercises:WordArithmeticAnswersandExplanations

PARTIV:VERBALABILITYREVIEW

6Synonyms(SSATandISEE)WhatDoSynonymQuestionsLookLike?HowDoYouAnswerSynonymQuestions?TestYourselfQuizzesAnswersandExplanationsExercises:SynonymsAnswersandExplanations

7VerbalAnalogies(SSATOnly)WhatMakesaVerbalAnalogy?WhatDoVerbalAnalogyQuestionsLookLike?HowDoYouSolveVerbalAnalogies?WhatDoSmartTest-TakersKnow?AnswersandExplanationsExercises:MixedRelationshipsAnswersandExplanations

8SentenceCompletions(ISEEOnly)WhatMakesaSentenceCompletion?HowDoYouAnswerSentenceCompletionQuestions?WhatDoSmartTest-TakersKnow?TestYourselfQuizzesAnswersandExplanationsExercises:SentenceCompletionsAnswersandExplanations

PARTV:READINGREVIEW

9ReadingComprehension(SSATandISEE)WhyIsMyReadingAbilityBeingTested?WhatKindsofQuestionsWillBeAsked?HowDoYouAnswerReadingComprehensionQuestions?WhatDoSmartTest-TakersKnow?TestYourselfQuizzesAnswersandExplanationsExercises:MainIdeaAnswersandExplanationsExercises:DetailsAnswersandExplanationsExercises:VocabularyAnswersandExplanationsExercises:InferenceAnswersandExplanations

PARTVI:MATHEMATICSREVIEW

10Mathematics(SSATandISEE)WhatMathematicsMustIKnow?HowDoIEstimatetheAnswer?WhenMustICalculate?TestYourselfQuizzesAnswersandExplanationsExercises:MathematicsAnswersandExplanations

11QuantitativeAbility(SSATandISEE)WhatMakesMultiple-ChoiceMathEasier?HowDoYouSolveMultiple-ChoiceQuantitativeAbilityQuestions?WhatDoSmartTest-TakersKnow?TestYourselfQuizzesAnswersandExplanationsExercises:QuantitativeAbilityAnswersandExplanations

12QuantitativeComparisons(ISEEOnly)WhatAreTheseStrange-LookingQuestions?HowDoYouSolveQuantitativeComparisons?WhatDoSmartTest-TakersKnow?TestYourselfQuizzesAnswersandExplanationsExercises:QuantitativeComparisonsAnswersandExplanations

PARTVII:WRITINGSAMPLEREVIEW

13WritingMechanics(SSATandISEE)WhatAreWritingMechanics?WhatAretheRulesofSpelling?SpellingDemonsWhatAretheRulesofPunctuation?

WhatAretheRulesforCapitalization?WhatAretheRulesofGrammar?WhatIsCorrectEnglishUsage?HowCanIImproveMyWriting?Exercises:SpellingAnswersandExplanationsExercises:PrinciplesofGrammarAnswersandExplanations

14TheEssay(SSATandISEE)WhatIsthePurposeoftheEssay?HowDoYouWriteanEssayUnderTimePressure?WhatDoSmartTest-TakersKnow?Exercises:EssayWriting1Exercises:EssayWriting2SampleResponses1SampleResponses2

PARTVIII:FOURPRACTICETESTS

PracticeTest2:SSAT(UpperLevel)PartI:WritingSamplePartII:MultipleChoiceAnswersandExplanationsScoreYourself

PracticeTest3:SSAT(UpperLevel)PartI:WritingSamplePartII:MultipleChoiceAnswersandExplanationsScoreYourself

PracticeTest4:ISEE(UpperLevel)Section1:VerbalReasoningSection2:QuantitativeReasoningSection3:ReadingComprehension

Section4:MathematicsAchievementSection5:EssayAnswersandExplanationsScoreYourself

PracticeTest5:ISEE(UpperLevel)Section1:VerbalReasoningSection2:QuantitativeReasoningSection3:ReadingComprehensionSection4:MathematicsAchievementSection5:EssayAnswersandExplanationsScoreYourself

APPENDIXES

AppendixA:ListofSynonyms

AppendixB:Parents’GuidetoPrivateSchoolsWhyChooseanIndependentSchool?AnotherOption:IndependentDaySchoolsWhyBoardingSchool:ReasonsIChoseThisPathandWhyYouMight,TooWhyaTherapeuticorSpecialNeedsSchool?FindingthePerfectMatchPlanaSuccessfulSchoolSearchUnderstandingtheAdmissionApplicationFormAboutAdmissionTestsandtheSSATPayingforaPrivateEducation

AppendixC:PrivateSchoolsAt-a-Glance

Credits

ExcerptsfromAChristmasCarol,byCharlesDickens

ExcerptfromTheGoldenFleece,byNathanielHawthorne

ExcerptfromRipVanWinkle,byWashingtonIrving

ExcerptfromTheNightingale,byHansChristianAndersen

TheRhodora,byRalphWaldoEmerson

BeforeYouBegin

HOWTOUSETHISBOOKCongratulations!You’vejustpickedupthebesthighschoolentranceexaminationpreparationguideyoucanbuy.ThisbookhasalltheanswerstoyourquestionsabouttheSSATandtheISEE.Itcontainsup-to-dateinformation,hundredsofpracticequestions,andsolidtest-takingadvice.Here’showyoucanuseittogetyourbesthighschoolentranceexamscore...andgetintothesecondaryschoolofyourchoice.

•Top10StrategiestoRaiseYourScoregivesyoutried-and-truetest-takingstrategies.•PartIcontainsanswerstoallyourquestionsabouttheSSATandtheISEE.You’lllearnwhatkindsofquestionstoexpect,howthetestsarescored,whatthequestionslooklike,andhowyoucankeepyourcoolontestday.

•PartIIgivesyouyourfirstchancetotryyourhandatsamplequestionsfrombothexams.PracticeTest1:Diagnosticcanshowyouwhereyourskillsarestrong—andwheretheyneedsomeimprovement.

•PartsIII,IV,V,andVIleadyouthroughthesubjectsoftheexams.Hereyouwillreviewwordanalysisandbasicmathematics.Skim,scan,orstudythesereviews,dependingonyourownneeds.You’llalsolearnabouteachquestiontypeyouwillseeonyourexam,andyou’llfindthedetailed,step-by-stepmethodsthattaketheguessworkoutofansweringthosequestions.Thesechaptersletyouinonthesecretssmarttest-takersknow—secretsthatcanaddvaluablepointstoyourtestscore.

•PartVIItakesyourhandasyoufacethepartoftheexamthatsomestudentsfindterrifying—theessay.You’lllearnhowtoorganizeandexpressyourideasundertimepressure.

•PartVIIIcontainsfourpracticetests.Takeatleastoneofyourexams.Takebothiftimeallows.Remember:practicemakesperfect!

•AppendixAgivesyoualistofhelpfulsynonyms.KnowingthewordsonthislistwillprepareyounotonlyforthesynonymsquestionsontheSSATandISEE,butwillalsoimproveyouroverallverbalskills.

•AppendixB:Parents’GuidetoPrivateSchoolsincludesusefularticlesonchoosingprivateschools,understandingadmissionapplicationforms,andpayingforprivateschools.Thearticlesarewrittenbyteachers,administrators,deans,andothereducationalprofessionals.

•AppendixC:PrivateSchoolsAt-a-GlancegivesyouvaluabledataonprivatesecondaryschoolsfromPeterson’sPrivateSecondarySchools.These“snapshots”willhelpguideyoursearchandalsoprovideinformationaboutAdvancedPlacement(AP)subjectareas

andsports.

WHATTOSTUDYPartsIIIthroughVIIofthisbookprovidecontentforyoutoreview.Usethetablebelowtodeterminewhichchapterstostudyforyourtest.

No. Chapter SSAT ISEE

PartIII:VocabularyReview

5 WordArithmetic X X

PartIV:VerbalAbilityReview

6 Synonyms X X

7 VerbalAnalogies X

8 SentenceCompletions X

PartV:ReadingReview

9 ReadingComprehension X X

PartVI:MathematicsReview

10 Mathematics X X

11 QuantitativeAbility X X

12 QuantitativeComparisons X

PartVII:WritingSampleReview

13 WritingMechanics X X

14 TheEssay X X

SPECIALSTUDYFEATURESPeterson’sMastertheSSAT&ISEEwasdesignedtobeasuser-friendlyasitiscomplete.Itincludesseveralfeaturestomakeyourpreparationeasier.

Overview

Eachreviewchapterbeginswithabulletedoverviewlistingthetopicsthatwillbecoveredinthechapter.Youknowimmediatelywheretolookforatopicthatyouneedtoworkon.

SummingItUp

Eachchapterendswithapoint-by-pointsummarythatcapturesthemostimportantpoints.

TestYourselfQuizzesandExercises

EachchapteroffersTestYourselfQuizzesand/orExercisesattheendofthechapter.Takeasmanyquizzesanddoasmanyexercisesasyoucan.Usetheresultstodeterminewhereyoustill

needwork.

BonusInformation

Inaddition,besuretolookinthepagemarginsofyourbookforthefollowingtest-preptools:

Note

NoteshighlightcriticalinformationabouttheformatoftheSSATandtheISEE.

Tip

Tipsdrawyourattentiontovaluableconcepts,advice,andshortcutsfortacklingthetests.

Alert!

Wheneveryouneedtobecarefulofacommonpitfallortest-takertrap,you’llfindanAlert!Thisinformationrevealsandhelpseliminatethewrongturnsmanypeopletakeontheexam.

ANIMPORTANTNOTEFOREIGHTH-ANDNINTH-GRADERSYoumayfindthatsomeofthetestquestionsthatappearontheSSATorISEEareextremelydifficultorcovermaterialthatyouhavenotyetbeenexposedto.Thisisintentional.Keepinmindthatthesameupper-levelexamisadministeredtostudentsingrades8,9,10,and11.However,yourfinalscorewillonlybecomparedtothescoresofotherstudentsinyourgrade.WhentheSecondarySchoolAdmissionTestBoardandtheEducationalRecordsBureausendadmissionofficersyourscores,they’llincludeinformationthatallowsyourscoretobecomparedtoallstudentsinyourgradewhohavetakenthetestinthepastthreeyears.Nooneexpectsyoutocompeteagainstolderstudents,sodon’tworryifyouencountervocabularyquestionsthatseemtooadvancedormathconceptsthatyouhaven’tmasteredyetinschool.Itwon’tbeheldagainstyou.Besides,becauseyou’reworkingwiththisbook,you’llbebetterpreparedtodealwiththosetoughquestionswhenyoutaketherealtest!

PARENTS’GUIDETOPRIVATESECONDARYSCHOOLParents—here’stheinformationyouneedtonavigatethroughtheprivatesecondaryschoolchoiceandadmissionprocess.We’veputtogethervaluableadvicefromadmissionexpertsaboutchoosingandapplyingtotheschoolthat’srightforyourchildandyourfamily.You’llbeabletogetanswerstoyourquestionsaboutindependentdayschools,findingtheperfectmatch,theadmissionapplicationform,andhowtopayforyourchild’sprivateschooleducation.

YOU’REWELLONYOURWAYTOSUCCESS

Rememberthatknowledgeispower.ByusingPeterson’sMastertheSSAT&ISEE,you’llbestudyingwiththemostcomprehensivepreparationguideavailableandyou’llbecomeextremelyknowledgeableaboutthetestyou’retaking.Welookforwardtohelpingyouscorehigher.Goodluck!

GIVEUSYOURFEEDBACKPeterson’spublishesafulllineofbooks—testprep,educationexploration,financialaid,andcareerpreparation.Peterson’spublicationscanbefoundatyourlocalbookstoreandlibrary,highschoolguidanceoffices,collegelibrariesandcareercenters,andatwww.petersonsbooks.com.Peterson’sbooksarenowalsoavailableasebooks.

Wewelcomeanycommentsorsuggestionsyoumayhaveaboutthispublication.Yourfeedbackwillhelpusmakeeducationaldreamspossibleforyou—andotherslikeyou.

TOP10STRATEGIESTORAISEYOURSCORE

1.Youdon’treallyneedtoreadthedirectionswhenyoutakeyourtest.Bythetimeyouactuallysitdowntotakeyourexam,you’vereadthisbook,you’vetakenallofthepracticetestsyoucouldfind,andyou’vereadenoughtestdirectionstofillalibrary.Sowhentheexamclockstartsticking,don’twastetimerereadingdirectionsyoualreadyknow.Instead,godirectlytoQuestion1.If,however,youdon’tfeelasconfidentaswedoaboutyourabilities,thenreadthedirections.

2.Questionsetsusuallygofromeasiesttomostdifficult—youshould,too!Exceptforthereadingcomprehensionquestions,testquestionsfollowthispattern.So,workyourwaythroughtheearlier,easierquestionsasquicklyasyoucan.

3.Theeasyanswerisn’talwaysthebest.Remember,thehardestquestionsareusuallyattheendofasectionandthatalsomeanstheanswersaremorecomplex.Lookcarefullyatthechoicesandreallythinkaboutwhatthequestionisasking.

4.Aneducatedguessisalwaysbest.Theprocessofeliminationisthebestwaytoimproveyourguessingodds.Checkouttheanswerchoicesandtrytocrossoutanythatyoudefinitelyknowarewrong.Ifyou’reansweringaquestionwithfiveanswerchoicesandyou’reableto

knockouttwochoices,theoddsgoupto33¹⁄3percentthatyouwillbecorrect.

5.It’ssmarttokeepmoving.It’shardtoletgo,butsometimesyouhaveto.Don’tspendtoomuchtimeonanyonequestionbeforeyou’vetriedallofthequestionsinasection.

6.FortheISEE,donotleaveanyblanksattheendofatest.Keeptrackofyourtimeandifyouhavequestionsremainingwhentimeisalmostup,trytoanswertherestofthequestions.

7.You’regoingtoneedawatch.Ifyou’regoingtopaceyourself,youneedtokeeptrackofthetime—andwhatifthereisnoclockintheroomoriftheonlyclockisoutofyourlineofvision?Awordofwarning:Don’tuseawatchalarmoryourwatchwillendupontheproctor’sdesk!

8.Keepingyourplaceontheanswersheetsavestimeandtestpoints.Keepyourplaceontheanswersheetbylayingyourtestbookletontopofitsoitactsasalinemarker,movingthetwoasyoumarkananswer.Thiswayyouwon’tlosepointsbymarkingtherightanswerinthewrongrow.

9.Checkandrecheckifyouhavetime.Ifyoufinishapartbeforetimeisup,usetheremainingtimetocheckthateachquestionisansweredintherightspaceandthatthereisonlyoneanswerforeachquestion.

10.Relax,andgoodluck!

PARTIHIGHSCHOOLENTRANCE

EXAMBASICS

CHAPTER1:AllAbouttheTests

CHAPTER2:SSATQuestions

CHAPTER3:ISEEQuestions

Chapter1

AllAbouttheTests

OVERVIEW

•Whatarethehighschoolentranceexams?•HowdoIfindoutwhichexamImusttake?•IfIhaveachoiceofexams,howdoIchoose?•Whatkindsofquestionsareaskedonhighschoolentranceexams?•Howaretheexamsstructured?•Whatdotheanswersheetslooklike?•Howaretheexamsscored?•Summingitup

WHATARETHEHIGHSCHOOLENTRANCEEXAMS?Thehighschoolentranceexamsarestandardizedtests.Independent,parochial,church-affiliated,andspecializedpublichighschoolsusescoresontheseexamstohelpthemmaketheiradmissionsdecisions.

Thereareanumberofwidelyusedstandardizedhighschoolexams.Thebestknownofthesearethefollowing:

•SSAT,theSecondarySchoolAdmissionTest,whichisadministeredbytheSecondarySchoolAdmissionTestBoardlocatedinPrinceton,NewJersey.Scoresareacceptedbymorethan900schools.SchoolsthatacceptSSATscoresincludeindependentunaffiliatedprivatedayandboardingschools,non-diocesanCatholicschoolsorCatholicschoolsoperatedbyreligiousorders,andnon-Catholicreligious-affiliatedschools.TheSSATisofferedatthreelevels—ElementaryLevelforstudentsenteringgrades4and5,MiddleLevelforstudentsenteringgrades6through8,andUpperLevelforstudentsentering

grades9andabove.ThisbookconcentratesonpreparingyoufortheUpper-Levelexam.•ISEE,theIndependentSchoolEntranceExamination,whichisadministeredbyEducationalRecordsBureauofNewYorkCity.TheISEEisacceptedbyallEducationalRecordsBureaumemberindependentschoolsintheUnitedStatesandabroad.TheISEEisofferedatfourlevels—PrimaryLevelforstudentsenteringgrades2through

4,LowerLevelforstudentsenteringgrades5and6,MiddleLevelforstudentsenteringgrades7and8,andUpperLevelforstudentsenteringgrades9andabove.ThisbookislimitedtopreparationfortheUpper-Levelexam.

•HSPT®,theHighSchoolPlacementTest,oneofthetwomostcommonlyusedCatholichighschoolentranceexams(theotheristheCOOP).Itisafive-part,multiple-choicetestofverbal,quantitative,reading,mathematics,andlanguageskills,andittakesapproximately hourstocomplete.Theexamcontains298questionsthataredesignedtoindicatehowwellastudentperformstasksthatcanbeexpectedofaneighth-grader.TherearetwodifferentHSPT®exams:theClosedHSPT®andtheOpenHSPT®.TheClosedHSPT®isadministeredbytheschool,butitisscoredbytheScholasticTestingService(STS).ClosedHSPT®scoresarecomparedtonationalstandarddistributionnorms,sonationwidepercentilescanbecompared.Scoringinformationissenttothehighschoolsindicatedonthestudent’sanswersheet,andit’suptotheschooltodeterminetheacceptablescoreforadmissiontoitsninthgradeclass.TheOpenHSPT®,whichusesoldversionsoftheClosedHSPT®,isadministeredandscoredbytheschool,soSTSdoesnotdealdirectlywithstudentresults.YoucanfindfullpreparationfortheHSPT®inPeterson’sMastertheCatholicHighSchoolEntranceExams.

•COOP,theCooperativeEntranceExam,publishedbyCTB/McGraw-HillofMonterey,California.TheCOOPisadministeredonlytostudentsplanningtoenterninthgrade.ItistheexclusiveentranceexaminationoftheArchdioceseofNewark,NewJerseyandtheDioceseofPaterson,NewJersey.Useinothersystemsorinindependentschoolsisscattered.BecausetheCOOPisusedalmostexclusivelybyCatholichighschools,fullpreparationisofferedinPeterson’sMastertheCatholicHighSchoolEntranceExams.

•SHSAT,theNewYorkCitySpecializedHighSchoolsAdmissionsTestadministeredbytheNewYorkCityDepartmentofEducation.ThisexamisonlyforeighthandninthgraderswholiveinNewYorkCityandwhowishtoattendBronxHighSchoolofScience,BrooklynTechnicalHighSchool,StuyvesantHighSchool,HighSchoolforAmericanStudiesatLehmanCollege,QueensHighSchoolforSciencesatYorkCollege,orHighSchoolforMath,ScienceandEngineeringatCityCollege.BecausetheuseofSHSATissolimited,thisbookoffersnopreparationforit.IfyouwillbetakingtheSHSAT,purchasePeterson’sMastertheNewYorkCitySpecializedHighSchoolsAdmissionsTest.

Thereareanumberoflesser-knownorindividuallyconstructedexams,assomeschoolsconstructtheirownexamsorpurchasestandardizedexamsfromsmallcompanies.Thesubjectsandquestionstylesoftheseexamscoverawiderangeofpossibilities,butverbal,reading,and

mathematicsskillsaresuretobeincluded.Youshouldfindthisbookhelpfulforanyentranceexam.

HOWDOIFINDOUTWHICHEXAMIMUSTTAKE?Calltheadmissionsofficesofallschoolstowhichyouareapplyingandaskwhichexameachschoolrequiresorwhichexamresultstheschoolwillaccept.Askalsoforthecutoffdatesbywhichyourscoresmustbereceivedbytheschool.Findoutiftheschoolhasmadespecialarrangementsfortestingitsapplicantsonaspecificdateataconvenientlocation.

IFIHAVEACHOICEOFEXAMS,HOWDOICHOOSE?IfallyourschoolswillacceptscoresfromeithertheSSATortheISEE,youcanchooseonthebasisofconvenienceoftestingdateandlocation.Or,ifyoubeganyourpreparationsearlyenoughandhavetriedasampleofeachexaminthisbook,youcanchoosetheexamwithwhichyoufeelmorecomfortable.

Youcangetlistsoftestinglocations,dates,registrationdeadlines,andfeesalongwithofficialtestdescriptions,officialsamplequestions,andregistrationformsbywritingorcallingorbyvisitingthefollowingwebsites:

SSATBCN5399Princeton,NJ08543609-683-4440info@ssat.orgwww.ssat.org

EducationalRecordsBureau470ParkAvenueSouthSecondFloor,SouthTowerNewYork,NY10016800-989-3721(toll-free)212-672-9800isee@erblearn.orgwww.erblearn.org

WHATKINDSOFQUESTIONSAREASKEDONHIGHSCHOOLENTRANCEEXAMS?

ThescopeofquestionsisprettylimitedforboththeSSATandISEE.Youwillnotfindsubject-specificquestions—nogrammarorspelling,nogeographyorscience.Butyouwillfindquestionstestingyourverbal,reading,andmathskills—skillsyou’vebeenworkingonsinceyouwere6yearsold.

Asidefromtheessay,whichisnotscored,allofthequestionsaremultiple-choice—foreachquestion,youhavefivechoicesontheSSATandfourchoicesontheISEE.Multiple-choicequestionsmakethetestseemeasierbecauseyou’llalwayshavethecorrectanswerinfrontofyou—youjusthavetofindit!

NOTEOnboththeSSATandtheISEEallquestiontypescountthesame.Youdon’tneedtoworryaboutsomequestionsbeingmoreimportantthanothers.

TheVerbalAbilitySections

Theverbalabilitysectionscontainquestionsthattestvocabularyandverbalreasoning.BoththeSSATandISEEusesynonymquestionstotestyourvocabulary.TheSSATtestsyourverbalreasoningwithanalogyquestionswhiletheISEEtestsverbalreasoningwithsentencecompletions.

Synonyms

Synonymquestionspresentasinglewordandaskyoutochoosethewordwiththesameormostsimilarmeaning.Thisisapuretestofvocabulary,thoughword-buildingskillsmayhelpyoufigureoutmeaningsofsomeunfamiliarwords.

Analogies(SSATonly)

Analogiespresentapairofwordsthathavesomelogicalrelationshiptooneanother.Thecorrectanswerconsistsofapairofwordswiththesamekindofrelationshipasthefirstpair.

SentenceCompletions(ISEEonly)

Sentencecompletionquestionsaskyoutochooseawordorwordsthatfillintheblanksinagivensentence.Theytesthowwellyoucanusecontextcluesandwordmeaningstocompleteasentence.

ReadingComprehension

Readingcomprehensionquestionsrelatetoapassagethatisprovidedforyoutoread.SSAT

passagescanbeaboutalmostanything.ISEEpassagesarebasedonsocialstudies,humanities,orsciencereadings,buttheydonottestyourknowledgeinthesesubjects.Regardlessofthesubjectmatterofthereadingpassage,thequestionsafterittesthowwellyouunderstoodthepassageandtheinformationinit.

MathematicsSections

ThemathsectionsontheSSATandtheISEEarestructuredsomewhatdifferently,butbothexamsseektomeasureyourunderstandingandapplicationofquantitativeconcepts—thatis,arithmetic,algebraic,andgeometricconcepts.It’simportanttoknowthatthesequestionsrequireeithersimplecalculationsornoneatall.OntheSSAT,youwillseeonlystandardmultiple-choicequestions.TheISEEusesacombinationofmultiple-choicequestionsandquantitativecomparisons.

NOTECalculatorsarenotallowedoneitherexam,butyoureallydon’tneedone.Leaveyourcalculatorandscratchpaperathomeanddoanyfiguringinyourheadorinthemarginsofyourtestbooklet.

StandardMultiple-ChoiceQuestions

Standardmultiple-choicequestionsgiveyouaprobleminarithmetic,algebra,orgeometry.Thenyouchoosethecorrectanswerfromthechoicesoffered.Manyoftheseproblemsinvolvenumberpatterns,asmallbodyofeasilymemorizedinformation,andresponsechoicesthatmakeitpossibleforyoutoestimatethecorrectanswer.Thesequestionscanoftenbesolvedwithoutpencilandpaper.

QuantitativeComparisons(ISEEonly)

Quantitativecomparisonquestionstestyourskillsincomparinginformationandinestimating.You’llseetwoquantities,oneinColumnAandoneinColumnB.Yourjobistocomparethetwoquantitiesanddecideifoneisgreaterthantheother,iftheyareequal,orifnocomparisonispossible.These,too,canoftenbedeterminedwithoutpencilandpaper,particularlyafteryouhavereviewedthebasicconceptsinthemathsectionshere.

HOWARETHEEXAMSSTRUCTURED?SSAT

TheSSATisadministeredinsixseparatelytimedsections.Thefirstsectionisthewritingsample.AmongtheotherfivesectionsyouwillalwaysfindtwoQuantitativesections,oneVerbal(synonymsandanalogies)section,andoneReadingComprehensionsection.ThereisalsoanExperimentalsection,whichisnotscored.ThissectioncontainssixVerbal,fiveQuantitative,andfiveReadingComprehensionquestions.Everyquestiononthemultiple-choicepartoftheSSAToffersfiveanswerchoiceslettered(A),(B),(C),(D),and(E).

HereistheSSATformatandtimetable:

TIPSetyourwatchtoexamtime.Atthebeginningofeachtestsection,setyourwatchto12o’clock,andlayitonthedeskinfrontofyou.Thenallyouhavetodoisglancequicklyatthehandstoseehowmuchtimehaspassed.

FormatofaTypicalSSAT

SectionNumberofQuestions TimeAllowed

WritingSample 25minutes

Multiple-Choice

Section1: Quantitative(Math) 25questions 30minutesSection2: ReadingComprehension 40questionsbased

onapproximately8–9readingpassages

40minutes

Section3: Verbal 60questions 30minutes30Synonymquestions30Analogyquestions

Section4: Quantitative(Math) 25questions 30minutesSection5: Experimental 16questions 15minutes

NOTEManytestproctorswillnotallowyoutowearadigitalwatchduringthetest.

ISEE

TheISEEisadministeredinfiveseparatelytimedsections.Thefirstfoursectionsofthetestaremultiple-choiceandcontainsomescoredandsomeunscoredquestions.Atotalof21multiple-choicequestionsonthetestarenotscored.Theessay,whichisthefinalsectionoftheISEE,isalsonotscored.Aphotocopyofyouressayisprovidedtoeachschoolasawritingsample.

Eachmultiple-choicequestionontheISEEoffersfouranswerchoices.Thesearelettered(A),(B),(C),and(D).

HereistheISEEformatandtimetable:

TIPKeepyourplaceontheanswersheetbylayingyourtestbookletontopofitsoitactsasalinemarker,movingthetwoasyoumarkananswer.Thiswayyouwon’tlosepointsbymarkingtherightanswerinthewrongrow.

FormatofaTypicalISEE

SectionNumberofQuestions TimeAllowed

Section1: VerbalReasoning 40questions 20minutesSynonymsSentenceCompletions

Section2: QuantitativeReasoning 37questions 35minutesNumbersandOperationsAlgebraicConceptsGeometryMeasurementDataAnalysisandProbabilityConcepts/UnderstandingApplications/HigherOrderThinkingQuantitativeComparisons

Section3: ReadingComprehension 36questionsbasedon6readingpassages

40minutes

HumanitiesPassagesSciencePassagesSocialStudiesPassages

Section4: MathematicsAchievement 47questions 40minutesNumbersandOperationsAlgebraicConceptsGeometricConceptsMeasurementDataAnalysisandProbability

Section5: Essay 30minutes

TIPMovequicklyontheeasyones.Thequestionsinasetusuallygofromeasiesttohardest.Trytogothroughtheeasyonesquickly.

WHATDOTHEANSWERSHEETSLOOKLIKE?Whenyougetthetestbooklet,you’llalsogetaseparatesheetonwhichyou’llmarkyouranswers.

ALERT!WhentakingtheSSAT,leavequestionsblankifyourunoutoftimeorhavenoideawhattheansweris.Takeaguessifyoucanruleout2ofthe5answerchoicesorifyouhaveaprettystrongfeelingaboutonechoice.Neverfillintheanswerbubblesrandomly—itwillcostyoupoints!

Foreachmultiple-choicequestion,you’llseeacorrespondingsetofanswerbubbles.Thebubblesarelabeledfrom(A)to(D)ontheISEEandfrom(A)to(E)ontheSSAT.Answersheetsarereadbymachines.Fillinyourchosenanswerbubblescompletelyandboldlysotherecanbenomistakeaboutwhichoneyouhavechosen.

HOWARETHEEXAMSSCORED?OK,you’veansweredallthequestions,timeisup(andnotamomenttoosoon),andyouturninyouranswersheet.Whatnext?Offitgoestothemachineatthecentralofficesofthetestingorganization.Themachinescansthesheetinsecondsandcalculatesascore.Howitcalculatesyourscoredependsonwhichexamyouweretaking.

SSAT

ThescoringoftheSSATisasfollows:Yougetonepoint(+1point)foreachcorrectanswer,andyouloseonequarterofapoint foreachincorrectanswer.Omittedanswershave

noeffect(0points)onyourscore.Calculationofrightanswersminusonefourthofwronganswersyieldsyourrawscore.Donotworrythatyourstandingontheexammaysufferincomparisontostudentsinothergradestakingthesameupper-levelexam.SSATscoresarescaledandreportedinpercentilesthatcompareonlystudentswithinthesameageandgradegroup.

ISEE

ScoringoftheISEEisuncomplicated.Youreceiveonepointforeveryquestionthatyouanswercorrectly.Thereisnopenaltyforawronganswer.Itreceivesnocredit,butthereisnodeductionfortheerror.AswiththeSSAT,scoringandpercentilerankingaredoneseparatelyformembersofeachgradegrouptakingthesameexam.Youwillbecomparedonlywithyourpeers.

SUMMINGITUP

•TheSSATisa3-hourtestofverbal,quantitative,andreadingability.•TheISEEisa3-hourtestofverbalandquantitativeability,readingcomprehension,andmathematicsachievement.

•TheSSATdeductsonefourthofapointfromyourscoreforeachincorrectanswer.Randomguessingwilldonogood.Acalculatedguessisalwaysworthwhile.

•TheISEEgivesyouonepointforeachcorrectanswer.Wronganswersdonotaffectyourscore.Leavenoblanks.

•AllSSATquantitativequestionsarestandardmultiple-choicequestions.•SomeISEEmathematicalquestionsarestandardmultiple-choice;somearequantitativecomparisons.

•Theessayisonlyawritingsample.Itisnotscored.•Onbothexamsallquestionswithineachset,exceptreadingcomprehension,arearrangedfromeasytohard.

•Useaprocessofeliminationtomakeeducatedguesseswhenyouarenotsure.•Yourpercentilerankingisbasedoncomparisonofyourscorewithstudentsinyourgradeonly.

•Youcangetaverygoodscoreevenifyoudon’tanswereveryquestion.•Don’tspinyourwheels.Don’tspendtoomuchtimeonanyonequestion.Giveitsomethought,takeyourbestshot,andmovealong.

•Guessingisalwayspermissible,butisitalwayswise?Thatdependsonwhetherornotyouhaveaclueastothepossibleanswer,anditdependsonwhichexamyouaretaking.Ifyou’retakingtheSSAT,youwillgetpenalized forincorrectanswers.Guessonlyifyoucaneliminatebetween2or3choices.Youwon’tbepenalizedforguessingontheISEE,sogoaheadandtakethechance.Youhavenothingtolose!

Chapter2

SSATQuestions

OVERVIEW

•Whatcanyouexpectonthetest?•HowdoestheSSATmeasureverbalability?•HowdoestheSSATmeasurequantitativeability?•HowdoestheSSATmeasurereadingability?•Whatisthewritingsample?•Summingitup

WHATCANYOUEXPECTONTHETEST?TheSSATusesfourquestiontypestotestyourverbalandquantitativeabilitiesandyourreadingcomprehension.Thischapterwilldescribeeachoftheminturnandshowyousamples.LearningthesequestiontypesinadvanceisthebestwaytopreparefortheSSAT.Theyneverchange,soyou’llknowwhattoexpectwhenyoutakethetest.TheSSATalsoasksyoutowriteashortessayonanassignedtopic.Thischapterwillintroduceyoutotheessayrequirement.

HOWDOESTHESSATMEASUREVERBALABILITY?TheSSATmeasuresyourverbalabilitywithtwoquestiontypes:

SynonymsAnalogies

Averbalsectionconsistsof60questions;30ofthesequestionsaresynonymsand30areanalogies.

SynonymQuestions

Asynonymisawordwiththesamemeaningornearlythesamemeaningasanotherword.SSATsynonymquestionsaskyoutochoosethebestsynonymforaquestionwordthatiswrittenincapitalletters.

ThedirectionsforSSATsynonymquestionslooksomethinglikethis:

Directions:ChoosethewordorphrasewhosemeaningismostsimilartothemeaningofthewordinCAPITALletters.

HerearetwosampleSSATsynonymquestions.Tryeachoneonyourown;thenreadtheexplanationthataccompaniesit.

1.NOVICE

(A)competitive

(B)clumsy

(C)aged

(D)beginner

(E)impulsiveThecorrectansweris(D).ANOVICEisabeginner,someonewithoutexperience.Youmayrecognizetherootofnovel,meaningnew,acluetothedefinition.

2.CONVOY

(A)hearse

(B)thunderstorm

(C)group

(D)jeep

(E)journeyThecorrectansweris(C).ACONVOYisagrouptravelingtogetherforprotectionorconvenience.Youhaveprobablyseenconvoysofmilitaryvehiclestravelingsinglefilealongthehighwaytowardsummerreservecamp.

AnalogyQuestions

Analogyquestionsaskyoutomatchuppairsofwordsthatarerelatedinthesameway.Eachquestionstartswithawordpair.Yourjobistofindorcreateanotherpairofwordsthatisrelatedinthesamewayasthefirstpair.

ThedirectionsforSSATanalogyquestionslooksimilartothis:

Directions:Findtherelationshipbetweenthewords.Readeachquestionandthenchoosetheanswerthatbestcompletesthemeaningofthesentence.

HerearetwosampleSSATanalogyquestions.Tryeachoneonyourownbeforereadingtheexplanationthataccompaniesit.

3.Lidistoboxascorkisto

(A)float

(B)bottle

(C)wine

(D)blacken

(E)stopperThecorrectansweris(B).Therelationshipisoneofpurpose.Thepurposeofalidistocloseabox;thepurposeofacorkistocloseabottle.Corkiseasilyassociatedwithallthechoices,soyoumustrecognizethepurposefulrelationshipoftheinitialpairtochoosethecorrectanswer.

4.Poisonistodeathas

(A)bookistopages

(B)musicistoviolin

(C)kindnessistocooperation

(D)lifeistofamine

(E)nothingistosomethingThecorrectansweris(C).Thisisacause-and-effectrelationship.Poisonmayleadtodeath;kindnessmayleadtocooperation.Neitheroutcomeisaforegoneconclusion,butbothareequallylikely,sotheparallelismaintained.Choice(B)offersareversedrelationship.

ALERT!Notethatthedirectionsaskyoutochoosethebest

answer.Theremaybeseveralanswersthatarepartiallycorrect,butonlyonethatiscompletelycorrect.That’swhyyoushouldalwaysreadalltheanswerchoicesbeforeyoumakeyourfinalselection.

HOWDOESTHESSATMEASUREQUANTITATIVEABILITY?TheSSATtestsyourquantitativeabilityintwo25-questionmathematicssections,thatis,with50mathematicsquestionsinall.Thequestionsineachsectionmeasureyourknowledgeofnumberconceptsandoperations,algebra,geometry/measurement,anddataanalysis/probability.

Thedirectionsarethesameforbothquantitativesections.Theyaresimilartothis:

Directions:Calculateeachprobleminyourheadorinthescratchareaofthetestbookletandchoosethebestanswer.

HerearefoursampleSSATquantitativeabilityquestionsshowingtherangeofmathematicalquestions.Tryeachoftheseonyourownbeforeyoureadtheexplanationthataccompaniesit.

5. %of1500=

(A)60

(B)15

(C)7.50

(D)3.75

(E)1.50

Thecorrectansweris(D). %writtenasadecimalis0.0025.(1500)(0.0025)=3.75.

Youcouldhavedonethisprobleminyourheadbythinking:10%of1500is150;1%of

1500is15; of1%=15÷4=3.75.

6.IfpsychologicalstudiesofjuveniledelinquentsshowKpercenttobeemotionallyunstable,thenumberofjuveniledelinquentsnotemotionallyunstableper100juveniledelinquentsis

(A)100–K

(B)1–K

(C)K–100

(D)100÷K

(E)K÷100Thecorrectansweris(A).“Percent”meansoutof100.IfKpercentareemotionallyunstable,thenKoutof100areemotionallyunstable.Theremainder,100–K,arenotunstable.

NOTELeaveyourcalculatorathome.Youwon’tbeabletouseitonanyofthetests.Instead,usethespaceprovidedinthetestbookletforscratchwork.

7.Apieceofwood35feet,6incheslongwasusedtomake4shelvesofequallength.Thelengthofeachshelfwas

(A)9feet,1 inches.

(B)8feet,10 inches.

(C)8feet,1 inches.

(D)7feet,10 inches.

(E)7feet,1 inches.

Thecorrectansweris(B).Firstconvertthefeettoinches.35feet,6inches=420inches+6

inches=426inches.426÷4=106.5inchespershelf=8feet,10 inchespershelf.

8.AngleABDisa(n)

(A)straightangleandcontains180º.

(B)acuteangleandcontains35º.

(C)obtuseangleandcontains360º.

(D)rightangleandcontains45º.

(E)rightangleandcontains90º.Thecorrectansweris(E).AngleABCandangleABDaresupplementaryangles.SinceangleABC=90º,angleABDmustalsoequal90º(180º–90º=90º).Arightanglecontains90º.

Youmayhavenotedthatonlynumber7ofthesamplemathquestionsrequiredapencil-and-papercalculationbecausethenumberswereabitdifficulttomanipulatementally.Theothersrequiredapplicationofamathematicalconceptandverylittlecalculation.

NOTETheSSATreadingcomprehensionsectionisanopen-booktest.Theanswerswillalwaysbedirectlystatedorimpliedinthegivenpassage.

HOWDOESTHESSATMEASUREREADINGABILITY?TheSSATmeasuresyourabilitytoreadquicklyandtounderstandwhatyoureadbyaskingyouquestionsaboutpassagesyoumustread.The40questionsinareadingcomprehensionsectionarebasedonabouteightorninereadingpassages.

ThedirectionsforSSATreadingcomprehensionquestionslooksimilartothis:

Directions:Carefullyreadeachpassageandthequestionsthatfollowit.Choosethebestanswertoeachquestiononthebasisoftheinformationinthe

passage.

HereisasampleSSATreadingpassagefollowedbyfourquestions.Readthepassageandtryansweringthequestionsonyourownbeforereadingtheexplanations.

CottonfabricstreatedwiththeXYZProcesshavefeaturesthatmakethemfarsuperiortoanypreviouslyknownflame-retardant-treatedcottonfabrics.XYZProcess-treatedfabricsaredurabletorepeatedlaunderinganddrycleaning;areglowresistantaswellasflameresistant;whenexposedtoflamesorintenseheatformtough,pliable,andprotectivechars;areinertphysiologicallytopeoplehandlingorexposedtothefabric;areonlyslightlyheavierthanuntreatedfabrics;andaresusceptibletofurtherwetanddryfinishingtreatments.Inaddition,thetreatedfabricsexhibitlittleornoadversechangeinfeel,texture,andappearanceandareshrink-,rot-,andmildew-resistant.Thetreatmentreducesstrengthonlyslightly.Finishedfabricshave“easycare”propertiesinthattheyarewrinkle-resistantanddryrapidly.

9.Itismostaccuratetostatethattheauthorintheprecedingselectionpresents

(A)factsbutreachesnoconclusionconcerningthevalueoftheprocess.

(B)aconclusionconcerningthevalueoftheprocessandfactstosupportthatconclusion.

(C)aconclusionconcerningthevalueoftheprocessunsupportedbyfacts.

(D)neitherfactsnorconclusions,butmerelydescribestheprocess.

(E)thecaseformakingallfabricsflame-retardant.Thecorrectansweris(B).Thisisaquestionthatrequiresyoutoevaluatetheauthor’sapproachtothepassage.Ifyoucannotanswerthisquestionreadily,rereadtheselection.TheauthorclearlythinksthattheXYZProcessisterrificandsayssointhefirstsentence.Therestoftheselectionpresentsawealthoffactstosupporttheinitialclaim.

10.ForwhichoneofthefollowingarticleswouldtheXYZProcessbemostsuitable?

(A)Nylonstockings

(B)Woolenshirt

(C)Silktie

(D)Cottonbedsheet

(E)PolyesterslacksThecorrectansweris(D).Atfirstglanceyoumightthinkthatthisisaninferencequestionrequiringyoutomakeajudgmentbaseduponthefewdrawbacksoftheprocess.Closerreading,however,showsyouthatthereisnocontestforthecorrectanswerhere.

Thisisasimplequestionoffact.TheXYZProcessisatreatmentforcottonfabrics.

11.ThemainreasonfortreatingafabricwiththeXYZProcessisto

(A)preparethefabricforotherwetanddryfinishingtreatment.

(B)renderitshrink-,rot-,andmildew-resistant.

(C)increaseitsweightandstrength.

(D)reducethechancethatitwillcatchfire.

(E)justifyapriceincrease.Thecorrectansweris(D).Thisisadetailquestion.Youmustidentifythefact,giveninthefirstsentence,thatXYZProcesscreatesflame-retardantfabrics.

12.WhichoneofthefollowingwouldbeconsideredaminordrawbackoftheXYZProcess?

(A)Formscharswhenexposedtoflame

(B)Makesfabricsmildew-resistant

(C)Addstotheweightoffabrics

(D)Iscompatiblewithotherfinishingtreatments

(E)DoesnotwashoutofthefabricThecorrectansweris(C).Obviouslyadrawbackisanegativefeature.Theselectionmentionsonlytwonegativefeatures.Thetreatmentreducesstrengthslightly,anditmakesfabricsslightlyheavierthanuntreatedfabrics.Onlyoneofthesenegativefeaturesisofferedamongtheanswerchoices.

NOTEForthewritingsample,givespecificexamplestosupportyourposition.

WHATISTHEWRITINGSAMPLE?AtthebeginningofeachSSATtestingsession,youmustwritea25-minuteessayonagiventopic.Youwillhaveachoiceoftwotopics,acreativewritingtopicandanessay-typetopic.Thisessayisnotscored.Itisduplicatedandsenttoeachschoolasasampleofyourabilitytoexpressyourselfinwritingunderthesameconditionsasallothercandidatesforadmissiontotheschool.

ThedirectionsfortheSSATwritingsamplelooksimilartothis:

Directions:Readthetopics,choosetheoneyoufindmostinteresting,andorganizeyouressaybeforewriting.Writealegibleessayonthepaperprovided.

HereisasampleSSATessaytopic.Trytoorganizeandwriteanessayonthistopic.

Topic:Badthingsalwayshappeninthrees.

Doyouagreeordisagree?Supportyourpositionwithexamplesfromyourownexperience,theexperienceofothers,currentevents,oryourreading.

SUMMINGITUP

•TheSSATmeasuresverbalabilitywithtwoquestiontypes:synonymsandanalogies.•TheSSATmeasuresquantitativeabilityintwosectionscoveringnumberconceptsandoperations,algebra,geometry/measurement,anddataanalysis/probability.

•Calculatorsarenotpermitted.Instead,usethespaceprovidedinthetestbookletforscratchwork.

•Theanswerstothereadingcomprehensionquestionsarebasedoninformationeitherdirectlystatedorimpliedinthepassages.

•AtthebeginningofeachSSATtestingsession,youmustwritea25-minuteessayononeoftwogiventopics.

Chapter3

ISEEQuestions

OVERVIEW

•Whatcanyouexpectonthetest?•HowdoestheISEEmeasureverbalability?•HowdoestheISEEmeasurequantitativeability?•HowdoestheISEEmeasurereadingcomprehension?•HowdoestheISEEmeasuremathematicsachievement?•Whatistheessayquestion?•Summingitup

WHATCANYOUEXPECTONTHETEST?TheISEEusesfivequestiontypestotestyourverbalandquantitativeabilitiesandyourachievementinmathematicsandreadingcomprehension.Thischapterwilldescribeeachoftheminturnandshowyousamples.LearningthesequestiontypesinadvanceisthebestwaytopreparefortheISEE.Theyneverchange,soyou’llknowwhattoexpectandyouwon’thaveanyunpleasantsurpriseswhenyoushowuptotakethetest.TheISEEalsoasksyoutowriteashortessayonanassignedtopic.Thischapterwillintroduceyoutotheessayrequirement.

HOWDOESTHEISEEMEASUREVERBALABILITY?TheISEEmeasuresyourverbalabilitywithtwoquestiontypes:

SynonymsSentencecompletions

Theverbalabilitytestconsistsof40questions.Atotalof19ofthesequestionsaresynonyms

and21aresentencecompletions.Fiveofthequestionsareunscored.

SynonymQuestions

Asynonymisawordwiththesamemeaningornearlythesamemeaningasanotherword.ISEEsynonymquestionsaskyoutochoosethebestsynonymforacapitalizedword.

ThedirectionsforISEEsynonymquestionslooklikesomethingthis:

Directions:ChoosethewordthatismostnearlythesameinmeaningasthewordinCAPITALletters.

HerearetwosampleISEEsynonymquestions.Tryeachoneonyourown;thenreadtheexplanationthataccompaniesit.

1.TENANT

(A)occupant

(B)landlord

(C)owner

(D)farmerThecorrectansweris(A).ThemostcommonsenseofthewordTENANTisrenter.Assuch,thetenantisneverthelandlord.Theownermaywellbeanoccupant,butunlessheorsheoccupiesonaverytemporarybasis,heorsheisnotconsideredatenant.Atenantfarmerlivesonandcultivatesthelandofanother.

2.CALCULATED

(A)multiplied

(B)added

(C)answered

(D)figuredoutThecorrectansweris(D).CALCULATINGmaywellincludemultiplyingoraddinginordertoarriveattheanswer,butnotallcalculationsneedbemathematical.Itisthefiguringoutthatisthecalculating.

ALERT!Notethatthedirectionsaskyoutochoosethebestanswer.Theremaybeseveralanswerchoicesthatarealmostright,butonlyonethatiscompletelycorrect.That’swhyyoushouldalwaysreadalltheanswerchoicesbeforeyoumakeyourfinalselection.

SentenceCompletionQuestions

Justasthenameimplies,sentencecompletionsare“fill-in-the-blank”questions.ISEEsentencecompletionquestionsmayhaveoneortwoblanks.Yourjobistochoosefromamongtheanswerchoicesthewordorwordsthatbestfiteachblank.

ThedirectionsforISEEsentencecompletionquestionslooklikethis:

Directions:Theblanksinthefollowingsentencesindicatethatwordsaremissing.Ifthereisoneblank,onlyasinglewordismissing.Iftherearetwoblanks,apairofwordsismissing.Choosetheonewordorpairofwordsthatwillbestcompletethemeaningofthesentenceasawhole.

HerearetwosampleISEEsentencecompletionquestions:aone-blankquestionandatwo-blankquestion.Tryeachoneonyourownbeforereadingtheexplanationthataccompaniesit.

3.Utilityisnot__________,fortheusefulnessofanobjectchangeswithtimeandplace.

(A)planned

(B)practical

(C)permanent

(D)understandableThecorrectansweris(C).Iftheusefulnessofanobjectchanges,thenthatusefulnessisbydefinitionnotpermanent.

4.Astringoflieshadlandedherinsuchahopeless__________thatshedidn’tknowhowto__________herself.

(A)status...clear

(B)pinnacle...explain

(C)confusion...help

(D)predicament...extricateThecorrectansweris(D).“Hopelesspredicament”isanidiomaticexpressionmeaning“impossiblesituation.”Thisisareasonablepositionforonetobeinafterastringoflies.Thesecondblankiscorrectlyfilledwithatermthatimpliesthatshecouldn’tgetoutofthemessshehadcreated.

HOWDOESTHEISEEMEASUREQUANTITATIVEABILITY?TheISEEtestsyourunderstandingofquantitativeconceptsandyourabilitytoapplythoseconceptswithtwoquestiontypes:

Standardmultiple-choicequestionsQuantitativecomparisons

Thequantitativereasoningtestconsistsof37questions.Abouthalfofthesearestandardmultiple-choicequestions,andtheremainderarequantitativecomparisons.Fivequestionsonthissectionareunscored.

MultipleChoice

Thedirectionsforthestandardmultiple-choicequestionslooksimilartothis:

Directions:Workeachprobleminyourheadoronthespaceavailableonthepagesofthetestbookletandchoosethecorrectanswer.Allfiguresareaccuratelydrawnunlessotherwisenoted.Alllettersstandforrealnumbers.

HerearetwosampleISEEstandardmultiple-choicequantitativeabilityquestions.Tryeachoftheseonyourownbeforeyoureadtheexplanationthataccompaniesit.

5.IfA2+B2=A2+X2,thenBequals

(A)

(B)X2–2A2

(C)

(D)A2+X2

Thecorrectansweris(A).SubtractA2frombothsidesoftheequation:B2=X2,therefore.

6.Howmuchtimeistherebetween8:30a.m.todayand3:15a.m.tomorrow?

(A)17 hours

(B)18 hours

(C)18 hours

(D)18 hours

Thecorrectansweris(D).12:00=11:60From8:30a.m.untilnoontoday:11:60–8:30=3:30

3hrs.30min.

Fromnoonuntilmidnight:12hours

Frommidnightuntil3:15a.m.:3hrs.15min.

Total:3hrs.30min.+12hrs.+3hrs.15min.=18hrs.45min.=18 hours

QuantitativeComparisons

ISEEquantitativecomparisonsareprobablynotlikeanyothermathquestionyou’veeverseen.Thesequestionspresentyouwithtwoquantities,oneinColumnAandoneinColumnB.Yourjobistodecidewhetheronequantityisgreater,whetherthetwoquantitiesareequal,orwhethernocomparisonispossible.Therearealwaysfouranswerchoicesforthisquestiontype,andtheyarealwaysthesame.Thedirectionsonthispartofthetestaresimilartothis:

Directions:Foreachofthefollowingquestions,twoquantitiesaregiven—oneinColumnA,theotherinColumnB.Comparethetwoquantitiesandchoose:

(A)ifthequantityinColumnAisgreater

(B)ifthequantityinColumnBisgreater

(C)ifthequantitiesareequal

(D)iftherelationshipcannotbedeterminedfromtheinformationgiven

Rememberthefollowinginformationasyoutacklequantitativecomparisonquestions:•Forsomequestions,informationconcerningoneorbothofthequantitiestobecomparediscenteredabovetheentriesinthetwocolumns.

•SymbolsthatappearinbothcolumnsrepresentthesamethinginColumnAasinColumnB.

•Letterssuchasx,n,andkaresymbolsforrealnumbers.•Allfiguresareaccuratelydrawnunlessotherwisenoted.

HerearetwosampleISEEquantitativecomparisonquestions.Tryeachoftheseonyourownbeforeyoureadtheexplanationthataccompaniesit.

7.

NOTE:Figurenotdrawntoscale.

ColumnA ColumnB

180–a d+c–b

Thecorrectansweris(D).

Sincewedonotknowifa>bora<b,therelationshipcannotbedetermined.

ColumnA ColumnB

8.

Thecorrectansweris(B).

ColumnA<ColumnB

NOTEInISEEreadingcomprehensionquestions,theanswerswillalwaysbedirectlystatedorimpliedinthepassage.Youcanthinkofitasanopen-booktest!

HOWDOESTHEISEEMEASUREREADINGCOMPREHENSION?TheISEEmeasuresyourabilitytoreadquicklyandtounderstandwhatyoureadbyaskingyouquestionsaboutpassagesthatyoumustread.The36questionsinthereadingcomprehensionsectionarebasedonsixreadingpassages.Sixofthequestionsareunscored.BecausetheISEEisinterestedinbothyourlevelofreadingandyourabilitytocomprehendmaterialfromthesciencesandfromsocialstudies,someofthecontentofthereadingpassagesisbasedonsocialstudiesandscience.Youmayalsoseehumanitiespassagescoveringtheartsandliterature,aswellaspassagesoncontemporarylife.

ThedirectionsfortheISEEtestofreadingcomprehensionlooksimilartothis:

Directions:Eachpassageisfollowedbyquestionsbasedonitscontent.Answerthequestionsfollowingeachpassageonthebasisofwhatisstatedorimpliedinthepassage.

HerearesamplesoftwoISEEreadingpassages,eachfollowedbytwoquestions.Thefirstpassageisasocialstudiespassageandthesecond,ascience-basedpassage.Readeachpassageandtrytoanswerthequestionsonyourownbeforeyoureadtheexplanations.

Alargeproportionofthepeoplewhoarebehindbarsarenotconvictedcriminals,butarepeoplewhohavebeenarrestedandarebeinghelduntiltheirtrialincourt.Expertshaveoftenpointedoutthatthisdetentionsystemdoesnotoperatefairly.Forinstance,apersonwhocanaffordtopaybailusuallywillnotgetlockedup.Thetheoryofthebailsystemisthatthepersonwillmakesuretoshowupincourtwhenheissupposedtosinceheknowsthatotherwisehewillforfeithisbail—hewilllosethemoneyheputup.Sometimesapersonwhocanshowthatheisastablecitizenwithajobandafamilywillbereleasedon“personalrecognizance”(withoutbail).Theresultisthatthewell-to-do,theemployed,andthosewithfamiliescanoftenavoidthedetentionsystem.Thepeoplewhodowindupindetentiontendtobethepoor,theunemployed,thesingle,andtheyoung.

9.Accordingtotheprecedingpassage,peoplewhoareputbehindbars

(A)arealmostalwaysdangerouscriminals.

(B)includemanyinnocentpeoplewhohavebeenarrestedbymistake.

(C)areoftenpeoplewhohavebeenarrestedbuthavenotyetcometotrial.

(D)areallpoorpeoplewhotendtobeyoungandsingle.Thecorrectansweris(C).Theanswertothisquestionisdirectlystatedinthefirstsentence.Choice(B)mightbepossible,butitisneitherstatednorimpliedbythepassage.Thewordallinchoice(D)makesitanincorrectstatement.

10.Supposethattwomenwerebookedonthesamechargeatthesametimeandthatthesamebailwassetforbothofthem.Onemanwasabletoputupbail,andhewasreleased.Thesecondmanwasnotabletoputupbail,andhewasheldindetention.Thewriterofthepassagewouldmostlikelyfeelthatthisresultis

(A)unfair,becauseitdoesnothaveanyrelationtoguiltorinnocence.

(B)unfair,becausethefirstmandeservesseverepunishment.

(C)fair,becausethefirstmanisobviouslyinnocent.

(D)fair,becausethelawshouldbetougheronpoorpeoplethanontherich.Thecorrectansweris(A).Youshouldhavenodifficultyinferringthisattitudefromthetoneofthepassage.

Fireoftentravelsinsidethepartitionsofaburningbuilding.Manypartitionscontainwoodenstudsthatsupportthepartitions,andthestudsleaveaspaceforthefiretotravelthrough.Flamesmayspreadfromthebottomtotheupperfloorsthroughthe

partitions.Sparksfromafireintheupperpartofapartitionmayfallandstartafireatthebottom.Somesignsthatafireisspreadinginsideapartitionare:(1)blisteringpaint,(2)discoloredpaintorwallpaper,or(3)partitionsthatfeelhottothetouch.Ifanyofthesesignsispresent,thepartitionmustbeopeneduptolookforthefire.Findingcobwebsinsidethepartitionisonesignthatfirehasnotspreadthroughthepartition.

11.Firescanspreadinsidepartitionsbecause

(A)therearespacesbetweenstudsinsideofpartitions.

(B)firescanburnanywhere.

(C)partitionsaremadeoutofmaterialsthatburneasily.

(D)partitionsareusuallypaintedorwallpapered.Thecorrectansweris(A).Thisstatementoffactismadeinthesecondsentence.

12.Ifafirefighterseesthepaintonapartitionbeginningtoblister,heshouldfirst

(A)wetdownthepartition.

(B)checkthepartitionsinotherrooms.

(C)chopaholeinthepartition.

(D)closewindowsanddoorsandleavetheroom.Thecorrectansweris(C).Blisteringpaint(line6)isasignthatfireisspreadinginsideapartition.Ifthissignispresent,thefirefightermustopenthepartitiontolookforthefire(lines7–8).Thewaytoopenthepartitionistochopaholeinit.

NOTELeaveyourcalculatorandscratchpaperathome.Youcan’tusethemonthetest,buttherewillbespaceprovidedinthetestbookletforscratchwork.

HOWDOESTHEISEEMEASUREMATHEMATICSACHIEVEMENT?TheISEEmeasuresyourmathematicsachievementbyaskingyoutoanswermathquestionsthatrelateto:

•Numbersandoperations•Algebraicconcepts•Geometry•Measurement

•Dataanalysisandprobability

Themathematicsachievementtestconsistsof47questions,allofthemofthestandardmultiple-choicetype.Fiveofthesequestionsareunscored.

Thedirectionsforthemathematicsachievementtestlooksimilartothis:

Directions:Readeachquestionandchoosethebestanswerbasedoncalculationsinyourheadorinthemarginsofthetestbooklet.

HerearefoursampleISEEmathematicsachievementquestions.Tryeachoftheseonyourownbeforeyoureadtheexplanationthataccompaniesit.

13.If ofaclassisabsentand ofthosepresentleavetheroom,whatfractionofthe

originalclassremainsintheroom?

(A)

(B)

(C)

(D)

Thecorrectansweris(C).If areabsent, arepresent.If ofthe presentleave,

ofthe remain. remainintheroom.

14.Acogwheelhaving8cogsplaysintoanothercogwheelhaving24cogs.Whenthesmallwheelhasmade42revolutions,howmanyhasthelargerwheelmade?

(A)10

(B)14

(C)16

(D)20Thecorrectansweris(B).Thelargerwheelis3timesthesizeofthesmallerwheel,soitmakes therevolutions:42÷3=14.

15.75%of4isthesameaswhatpercentof9?

(A)25

(B)33

(C)36

(D)40Thecorrectansweris(B).75%of4=3

of9

16.If cupofspinachcontains80caloriesandthesameamountofpeascontains300

calories,howmanycupsofspinachhavethesamecaloriccontentas cupofpeas?

(A)

(B)1

(C)2

(D)2

Thecorrectansweris(D). cupspinach=80calories

cuppeas=300calories

1cuppeas=600calories

cuppeas=400calories

400÷80=5halfcupsofspinach

=2 cupsofspinach

WHATISTHEESSAYQUESTION?AttheendofeachISEEtestingsession,youmustwritea30-minuteessayonanassignedsubject.Thisessayisnotscored.Itisduplicatedandsenttoeachschoolasasampleofyourabilitytoexpressyourselfinwritingunderthesameconditionsasallothercandidatesforadmissiontotheschool.

ThedirectionsfortheISEEessayquestioncanbesummarizedasfollows:

Directions:Readtheessaytopicanddecidewhatyouwanttosay.Organizeyourthoughtscarefully,thenwritealegible,coherent,andgrammaticallycorrectessayonthetopic.

HereisasampleISEEessaytopic,alsocalleda“prompt.”Trytoorganizeandwriteanessayonthistopic.

Topic:AnexchangestudentfromChinahasjustenteredyourschool.Whatwillyoutellthisstudentaboutstudentlifeatyourschool?

SUMMINGITUP

•TheISEEmeasuresverbalabilitywithtwoquestiontypes:synonymsandsentencecompletions.

•TheISEEtestsquantitativeabilitywithtwoquestiontypes:standardmultiple-choiceandquantitativecomparisons.

•TheISEEmeasuresreadingabilitywithreadingcomprehensionquestionsbasedoninformationthatiseitherdirectlystatedorimpliedinthepassages.

•TheISEEmeasuresmathematicsachievementinfiveareas:numbersandoperations,algebraicconcepts,geometry,measurement,anddataanalysisandprobability.

•Calculatorsarenotpermitted.Instead,usethespaceprovidedinthetestbookletforscratchwork.

•AttheendofeachISEEtest,youmustwritea30-minuteessayonanassignedsubject.

PARTIIDIAGNOSINGSTRENGTHS

ANDWEAKNESSES

CHAPTER4:PracticeTest1:Diagnostic

PracticeTest1:DiagnosticTest

PartIIcontainsaDiagnosticTesttohelpyouseewhereyoustand.TaketheDiagnosticTestfirst,tolearnyourstrengthsandweaknesses.Thenuseyourresultstodevelopastudyplan.

Usethetablebelowtodeterminewhichsectionsofthetesttotake.

No. Section SSAT ISEE

1 Synonyms X X

2 VerbalAnalogies X

3 SentenceCompletions X

4 ReadingComprehension X X

5 QuantitativeAbility X X

6 QuantitativeComparisons X

TheDiagnosticTestdoesnotcontainanessayquestion.However,youwillhavechancestopracticeyourwritingskillsinPartVII.

ThesectionsonthisDiagnosticmaydifferfromyourtestincertainways.Forinstance,theDiagnosticsectionsmaybeshorterthanthesectionsonyourtest.ThoughtheDiagnosticwillnotbeidenticaltothetestyouwilltake,itwillstillhelpyouassesyourskilllevels.Onceyouknowwhereyouneedwork,youcanbettertargetyourstudying.

SECTION1:SYNONYMS(SSATANDISEE)20Questions•10Minutes

Directions:ChoosethewordorphraseclosestinmeaningtotheCAPITALIZEDword.

1.INTERMITTENTLY

(A)constantly

(B)annually

(C)usingintermediaries

(D)atirregularintervals

2.DECEPTION

(A)secrets

(B)fraud

(C)mistrust

(D)hatred

3.ACCLAIM

(A)amazement

(B)laughter

(C)booing

(D)applause

4.ERECT

(A)paint

(B)design

(C)destroy

(D)construct

5.RELISH

(A)care

(B)speed

(C)amusement

(D)enjoy

6.FORTNIGHT

(A)twoweeks

(B)oneweek

(C)twomonths

(D)onemonth

7.IMPOSE

(A)disguise

(B)escape

(C)require

(D)tax

8.ALIAS

(A)enemy

(B)sidekick

(C)hero

(D)othername

9.ITINERANT

(A)traveling

(B)shrewd

(C)insurance

(D)aggressive

10.AMPLE

(A)plentiful

(B)enthusiastic

(C)well-shaped

(D)overweight

11.STENCH

(A)puddleofslimywater

(B)pileofdebris

(C)foulodor

(D)deadanimal

12.SULLEN

(A)grayishyellow

(B)soakingwet

(C)verydirty

(D)angrilysilent

13.TERSE

(A)pointed

(B)trivial

(C)nervous

(D)lengthy

14.INCREMENT

(A)animprovisation

(B)anincrease

(C)feces

(D)specification

15.MISCONSTRUED

(A)followeddirections

(B)ledastray

(C)actingtosupervise

(D)interpretederroneously

16.VESTIGE

(A)design

(B)trace

(C)strap

(D)robe

17.CAPITULATE

(A)surrender

(B)execute

(C)finance

(D)retreat

18.EXTENUATING

(A)mitigating

(B)opposing

(C)incriminating

(D)distressing

19.SUBSERVIENT

(A)underestimated

(B)underhanded

(C)subordinate

(D)evasive

20.COLLUSION

(A)decision

(B)insinuation

(C)connivance

(D)conflict

SECTION2:VERBALANALOGIES(SSATONLY)20Questions•10Minutes

Directions:Findtherelationshipsamongthewords.Selecttheanswerchoicethatbestcompletesthemeaningofthesentence.

1.Redistopinkasblackisto

(A)beige

(B)white

(C)dark

(D)gray

(E)brown

2.Youthistoyoungasmaturityisto

(A)people

(B)parents

(C)grandmother

(D)old

(E)child

3.Oneistotwoasthreeisto

(A)two

(B)five

(C)six

(D)thirty

(E)nine

4.Lightistolampasheatisto

(A)furnace

(B)light

(C)sun

(D)room

(E)hot

5.Weekistomonthasseasonisto

(A)holiday

(B)spring

(C)harvest

(D)planting

(E)year

6.Squareistocircleasrectangleisto

(A)round

(B)triangle

(C)oval

(D)cube

(E)angle

7.Choiristodirectorasteamisto

(A)sport

(B)coach

(C)player

(D)athlete

(E)winner

8.Sandistobeachasblackdirtisto

(A)earth

(B)plants

(C)water

(D)farm

(E)soil

9.Tableistolegasautomobileisto

(A)wheel

(B)axle

(C)door

(D)fuel

(E)car

10.Arouseistopacifyasagitateisto

(A)disrupt

(B)ruffle

(C)understand

(D)ignore

(E)smooth

11.Margarineistobutteras

(A)creamistomilk

(B)laceistocotton

(C)nylonistosilk

(D)eggistochicken

(E)mapleistosyrup

12.Woodsmanistoaxeas

(A)carpenteristosaw

(B)mechanicistowrench

(C)soldieristogun

(D)draftsmanistoruler

(E)cowboyistolasso

13.Worriedistohystericalas

(A)hotistocold

(B)sadistonervous

(C)lonelyistocrowded

(D)happyistoserious

(E)happyistoecstatic

14.Controlistoorderas

(A)jokeistoclown

(B)teacheristopupil

(C)disorderistoclimax

(D)anarchyistochaos

(E)classroomistodiscipline

15.Horseistofoalas

(A)donkeyistoass

(B)cowistocalf

(C)bullistosteer

(D)whinnyistomoo

(E)apeistochimpanzee

16.Sleepistofatigueas

(A)wateristothirst

(B)restistoweary

(C)pillowistoblanket

(D)fatigueistorun

(E)hungeristoeat

17.Islandistooceanas

(A)hillistostream

(B)forestistovalley

(C)oasisistodesert

(D)treeistofield

(E)hillistomountain

18.Dramaistodirectoras

(A)classistoprincipal

(B)movieistoscenario

(C)actoristoplaywright

(D)magazineistoeditor

(E)studentistoteacher

19.Requestistodemandas

(A)replyistorespond

(B)inquireistoask

(C)wishistocrave

(D)seekistohide

(E)desireistoneed

20.Woodistocarveas

(A)treeistosway

(B)paperistoburn

(C)lightistoshine

(D)pipeistoblow

(E)clayistomold

SECTION3:SENTENCECOMPLETIONS(ISEEONLY)20Questions•10Minutes

Directions:Eachquestionismadeupofasentencewithoneortwoblanks.Thesentenceswithoneblankindicatethatonewordismissing.Thesentenceswithtwoblanksindicatethattwowordsaremissing.Eachsentenceisfollowedbyfourchoices.Choosetheletterofthechoicethatwillbestcompletethemeaningofthesentenceasawhole.

1.Althoughfortune-tellersclaimto_____futurehappenings,thereisnoscientificevidenceoftheir_____.

(A)cloud...ability

(B)effect...knowledge

(C)foretell...fees

(D)predict...accuracy

2.Greatideashave_____youth:theyare_____.

(A)no...petrified

(B)eternal...immortal

(C)constant...ephemeral

(D)little...frivolous

3.Eachhumanrelationshipisunique,andtheloverswhothinkthereneverwasaloveliketheirsare_____.

(A)foolish

(B)blind

(C)prejudiced

(D)right

4.Ratsgivesome_____asscavengers,butthisisover-balancedbytheir_____activities.

(A)help...useful

(B)service...harmful

(C)problems...nocturnal

(D)trouble...breeding

5.Ancientsocietiesgaveauthoritytothosewhoknewandpreserved_____,fortheideaofwhatwasrightlayinthepast.

(A)order

(B)law

(C)intelligence

(D)tradition

6.Theraredesertrainsoftencomein_____,causinglossoflifeandproperty;thus,peoplelivinginanoasisthinkofrainwith_____.

(A)floods...longing

(B)torrents...terror

(C)sprinkles...fear

(D)winter...snow

7.Hisadmirerswerenot_____,forhisessayswerenotwidelyknown.

(A)respected

(B)numerous

(C)ardent

(D)interested

8.Archaeologistsfoundruinsoftemplesandpalaces,butno_____;itwasasthoughthesepeoplenever_____.

(A)food...lived

(B)tombs...died

(C)plans...built

(D)monasteries...worshipped

9.Safedrivingprevents_____andtheendless_____ofknowingyouhavecausedotherspain.

(A)disease...reminder

(B)tragedy...remorse

(C)accidents...hope

(D)lawsuits...expense

10.Atrueamateurplaysbecausehe_____thegameandwillnotcheatbecausethatwould_____thegame.

(A)studies...lose

(B)understands...improve

(C)knows...forfeit

(D)loves...degrade

11.Companieshavefounditpaystohave_____handywhenameetingislikelytobe_____.

(A)food...prolonged

(B)secretaries...enjoyable

(C)telephones...successful

(D)money...interesting

12.He_____apart,forheprefers_____tothecompanyofothers.

(A)lives...books

(B)stays...throngs

(C)remains...vivacity

(D)dwells...solitude

13.TheConstitutionaldutyto“takecarethatthelawsbefaithfullyexecuted”makesthepresidenttheheadoflaw_____.

(A)development

(B)interpretation

(C)education

(D)enforcement

14.Areductionoftheworkweektofourdayswouldcertainly_____the_____industry.

(A)destroy...automobile

(B)stimulate...steel

(C)improve...electrical

(D)benefit...leisure

15.HistorytellsusittookAthenslessthanagenerationtochangefromachampionof_____intoaruthless_____.

(A)democracy...republic

(B)freedom...tyrant

(C)independence...commonwealth

(D)dictatorship...liberator

16.Thesocietywasnot_____andrequiredmuchoutsideaid.

(A)philanthropic

(B)destitute

(C)democratic

(D)self-sufficient

17.The_____climateofthecountry_____thedelicateelectronicequipment.

(A)intolerable...restored

(B)dry...vaporized

(C)changeable...demoralized

(D)humid...corroded

18.Thevalueof_____sciencetomodernprogressis_____.

(A)research...unimportant

(B)physical...unquestionable

(C)medical...unlikely

(D)statistical...unreliable

19.Thefinalendofanonadaptingsocietyisthesameasforanonadaptinganimal:_____.

(A)admiration

(B)resignation

(C)extinction

(D)immortality

20.Sometemperamentalactressesfailtounderstandthatadirector’scriticismisaimedattheir_____andnotattheir_____.

(A)weaknesses...conduct

(B)stupidity...graciousness

(C)performance...personality

(D)prosperity...inability

SECTION4:READINGCOMPREHENSION(SSATANDISEE)22Questions•15Minutes

Directions:Readeachpassageandanswerthequestionsthatfollowit.

Ifyouareaskedthecoloroftheskyonafairdayinsummer,youranswerwillmostprobablybe“blue.”Thisanswerisonlypartiallycorrect.Blueskynearthehorizonisnotthesamekindofblueasitisstraightoverhead.Lookattheskysomefinedayandyouwillfindthattheblueskynearthehorizonisslightlygreenish.Asyoureyemovesupwardtowardthezenith,youwillfindthatthebluechangesintopureblue,andfinallyshadesintoaviolet-blueoverhead.

Haveyouheardthestoryofafarmerwhoobjectedtothecolorofthedistanthillsintheartist’spicture?Hesaidtotheartist,“Whydoyoumakethosehillsblue?Theyaregreen,I’vebeenoverthereandIknow!”

Theartistaskedhimtodoalittleexperiment.“Bendoverandlookatthehillsbetweenyourlegs.”Asthefarmerdidthis,theartistasked,“Nowwhatcolorarethehills?”

Thefarmerlookedagain,thenhestoodupandlooked.“Bygosh,theyturnedblue!”hesaid.

Itisquitepossiblethatyouhavelookedatmanycolorsthatyoudidnotreallyrecognize.Skyisnotjustblue;itismanykindsofblue.Grassisnotplaingreen;itmaybeoneofseveralvarietiesofgreen.Ared-brickwallfrequentlyisnotpurered.Itmayvaryfromyellow-orangetoviolet-redincolor,buttotheunseeingeyeitisjustredbrick.

1.Whichtitlebestexpressestheideasofthispassage?

(A)“TheSummerSky”

(B)“Artistsvs.Farmers”

(C)“RecognizingColors”

(D)“BlueHills”

2.Atthezenith,theskyisusually

(A)violet-blue.

(B)violet-red.

(C)greenish-blue.

(D)yellow-orange.

3.Theauthorsuggeststhat

(A)farmersarecolor-blind.

(B)perceivedcolorvaries.

(C)brickwallsshouldbepaintedpurered.

(D)someartistsusepoorcolorcombinations.

4.Thewordzenithinthefirstparagraphprobablyrefersto

(A)acolor.

(B)apointdirectlyoverhead.

(C)apointonthehorizon.

(D)thehills.

WhiletheEuropeanswerestillcreepingcautiouslyalongtheircoasts,PolynesiansweremakingtripsbetweenHawaiiandNewZealand,adistanceof3,800miles,infrailcanoes.ThesefearlesssailorsofthePacificexploredeveryislandintheirvastdomainwithouteventhesimplestofnavigationaltools.

Inthedaytime,thePolynesiansguidedtheircraftbythepositionofthesun,thetrendofthewavesandwind,andtheflightofseabirds.

Starswereusedduringlongtripsbetweenislandgroups.Youthsstudyingnavigationweretaughttoviewtheheavensasacylinderonwhichthehighwaysofnavigationweremarked.AninvisiblelinebisectedtheskyfromtheNorthStartotheSouthernCross.

Inadditiontosinglecanoes,thePolynesiansoftenusedtwincanoesfortranspacificvoyages.Thetwoboatswerefastenedtogetherbycanopiedplatformsthatshieldedpassengersfromsunandrain.Suchcraftswereremarkablyseaworthyandcouldaccommodate60to80people,inadditiontowater,food,anddomesticanimals.Someofthesevesselshadasmanyasthreemasts.

ThesePacificmarinersusedpaddlestopropelandsteertheircanoes.Thesteeringpaddlewassoimportantthatitwasalwaysgivenapersonalname.Polynesianlegendsnotonlyrecitethenamesofthecanoeandtheherowhodiscoveredanewislandbutalsothenameofthesteeringpaddleheused.

5.Whichtitleisbestforthisselection?

(A)“EuropeanSailors”

(B)“TheHistoryofthePacificOcean”

(C)“TheStudyofNavigation”

(D)“EarlyPolynesianNavigation”

6.ThePolynesiansmadetripsto

(A)NewZealand.

(B)theAtlantic.

(C)theSouthernCross.

(D)Europe.

7.Thewordmarinermeans

(A)propeller.

(B)seaman.

(C)paddle.

(D)navigation.

8.ThispassagesuggeststhatthePolynesians

(A)trainedseabirdstoguidetheircanoes.

(B)hadseenalineintheskythatwasinvisibletoothers.

(C)usedaprimitivetelescopetoviewtheheavens.

(D)wereastronomersaswellasexplorers.

Theseasonalcomingsandgoingsofbirdshaveexcitedtheattentionandwonderofallsortsofpeopleinallagesandplaces.TheoraclesofGreeceandtheaugursofRomewovethemintoancientmythology.TheyarespokenofintheBooksofJobandJeremiah.

Nevertheless,ithasbeendifficultformanytobelievethatsmallbirds,especially,arecapableofmigratoryjourneys.AristotlewasconvincedthatthebirdsthatwinteredinGreecewerenotnewarrivals,butmerelyGreece’ssummerbirdsinwinterdress.Accordingtoabeliefpersistinginsomepartsoftheworldtothisday,swallowsandswiftsdonotmigrate,butspendthewinterinhibernation.(Swiftsandswallowsdomigrate,justasmostotherNorthernHemispherebirdsdo.)Anotheroldandcharming,butuntrue,legendenliststheaidofthestorkingettingsmallbirdstoandfromwinterquarters:Smallbirdsaresaidtohitchridesonthe

Europeanstork’sback.

ItisclearwhyNorthernHemispherebirdsflysouthinthefall;theygotoassurethemselvesoffoodandamorefavorableclimateforthewintermonths.Itisalsoclearwheremostofthemigrantscomefromandwheretheygo.Yearsofbird-bandinghavedisclosedtheroutesofthemainmigratoryspecies.

Butthereareotheraspectsofmigrationthatremain,forallourpowersofscientificinvestigation,aspuzzlingandmysterioustomodernmanastotheancients.Whydomigrantbirdscomenortheachspring?Whydon’ttheysimplystayinthewarmtropicsthewholetwelvemonthsoftheyear?Whatdeterminesthemomentofdeparturefornorthorsouth?Aboveall,howdobirds—especiallyspeciesliketheremarkablegoldenplover,whichflieshugedistancesdirectlyacrosstracklessoceanwastes—findtheirway?

9.Whichofthefollowingisthebesttitleforthisselection?

(A)“TheSolutionofanAncientProblem”

(B)“MysteriousMigrations”

(C)“TheSecretofthePlover”

(D)“Aristotle’sTheory”

10.Bird-bandinghasrevealed

(A)thekindoffoodbirdseat.

(B)whythebirdspreferthetropicsinthesummer.

(C)whybirdsleaveatacertaintime.

(D)theroutestakenbydifferenttypesofbirds.

11.Swallowsandswifts

(A)remaininGreeceallyear.

(B)changetheirplumageinwinter.

(C)hibernateduringthewinter.

(D)flysouthforthewinter.

12.Thearticleprovesthat

(A)naturestillhassecretsthatmanhasnotfathomed.

(B)thesolutionsofAristotleareacceptedbymodernscience.

(C)weliveinanagethathaslostallinterestinbirdlore.

(D)manhasnomeansofsolvingtheproblemsofbirdmigration.

Usingnewtoolsandtechniques,scientists,almostunnoticed,areremakingtheworldofplants.Theyhavealreadyremodeledsixty-fivesortsofflowers,fruits,vegetables,andtrees,givingusamongotherthingstobaccothatresistsdisease,cantaloupesthatareimmunetotheblight,andlettucewithcrisperleaves.Thechiefnewtooltheyareusingiscolchicine,apoisonousdrug,whichhasastoundingeffectsupongrowthanduponheredity.Itcreatesnewvarietieswithastonishingfrequency,whereassuchmutationsoccurbutrarelyinnature.Colchicinehasthrownnewlightonthefascinatingjobsoftheplanthunters.TheDepartmentofAgriculturesendsagentsallovertheworldtofindplantsnativetootherlandsthatcanbegrownhereandaresuperiortothosealreadyhere.Scientistshavecrossedtheseforeignplantswiththoseathome,therebyaddingtoourfarmcropsmanydesirablecharacteristics.Thecolchicinetechniquehasenormouslyfacilitatedtheirwork,becausehybridssooftencanbemadefertileandbecauseittakessofewgenerationsofplantsnowtobuildanewvarietywiththequalitiesdesired.

13.Whichtitlebestexpressestheideasoftheparagraph?

(A)“PlantGrowthandHeredity”

(B)“NewPlantsforOld”

(C)“RemodelingPlantLife”

(D)“AMoreAbundantWorld”

14.Mutationinplantliferesultsin

(A)diseasedplants.

(B)hybrids.

(C)newvarieties.

(D)fertility.

15.Colchicinespeedstheimprovementofplantspeciesbecauseit

(A)makespossibletheuseofforeignplants.

(B)makesuseofnaturalmutations.

(C)createsnewvarietiesveryquickly.

(D)canbeusedwithsixty-fivedifferentvegetables,fruits,andflowers.

16.Accordingtothepassage,colchicineisa

(A)poisonousdrug.

(B)blight.

(C)kindofplanthunter.

(D)hybridplant.

Abouttheyear1812,twosteamferryboatswerebuiltunderthedirectionofRobertFultonforcrossingtheHudsonRiver,andoneofthesamedescriptionwasbuiltforserviceontheEastRiver.Theseboatswerewhatareknownastwinboats,eachofthemhavingtwocompletehullsunitedbyadeckorbridge.Becausetheseboatswerepointedatbothendsandmovedequallywellwitheitherendforemost,theycrossedandre-crossedtheriverwithoutlosinganytimeinturningabout.Fultonalsocontrived,withgreatingenuity,floatingdocksforthereceptionoftheferryboatsandameansbywhichtheywerebroughttothedockswithoutashock.TheseboatswerethefirstofafleetthathassincecarriedhundredsofmillionsofpassengerstoandfromNewYork.

17.Whichtitlebestexpressesthemainideaofthisselection?

(A)“CrossingtheHudsonRiverbyBoat”

(B)“TransportationofPassengers”

(C)“TheInventionofFloatingDocks”

(D)“TheBeginningofSteamFerryboatService”

18.Thesteamferryboatswereknownastwinboatsbecause

(A)theyhadtwocompletehullsunitedbyabridge.

(B)theycouldmoveaseasilyforwardasbackward.

(C)eachferryboathadtwocaptains.

(D)twoboatswereputintoserviceatthesametime.

19.Whichstatementistrueaccordingtotheselection?

(A)BoatsbuiltunderFulton’sdirectionarestillinuse.

(B)Fultonplannedareceptiontocelebratethefirstferryboat.

(C)FultonpilotedthefirststeamferryboatsacrosstheHudson.

(D)Fultondevelopedasatisfactorywayofdockingtheferryboats.

20.RobertFultonworkedinthe

(A)seventeenthcentury.

(B)eighteenthcentury.

(C)nineteenthcentury.

(D)twentiethcentury.

21.Inline8,thewordshockisusedtomeanan

(A)unpleasantsurprise.

(B)impact.

(C)illnessfollowinganaccident.

(D)electricalimpulse.

22.Thefirststeamferryboatswerebuiltto

(A)crosstheErieCanal.

(B)crosstheHudsonRiver.

(C)carrypassengerstoMassachusetts.

(D)traveldowntheDelawareRiver.

SECTION5:QUANTITATIVEABILITY(SSATANDISEE)30Questions•30Minutes

Directions:Choosethecorrectanswertoeachquestion.

1.Whichsquarehashalfofitsareashaded?

2.If2packagesofcookiesareenoughfor10children,howmanywillbeneededfor15children?

(A)6

(B)5

(C)4

(D)3

3.Whichisequalto9?

(A)4×5

(B)9×0

(C)9×1

(D)3×6

4.Jeffearns12dollarsaweek.Whichofthefollowingstatementstellshowmanydollarshewillearnin5weeks?

(A)12–5

(B)12÷5

(C)12+5

(D)12×5

5.ThedistancefromCityXtoSanFranciscois3timesthedistancefromCityXtoChicago.HowmanymilesawayfromCityXisSanFrancisco?Tosolvethisproblem,whatelsedoyouneedtoknow?

(A)ThedistancefromChicagotoSanFrancisco

(B)ThedistancefromCityXtoChicago

(C)Thecityoforigination

(D)Nothingelse

6.Ifanoddnumberissubtractedfromanoddnumber,whichofthefollowingcouldbetheanswer?

(A)1

(B)2

(C)7

(D)9

7.If7×6=Y,whichistrue?

(A)Y÷7=6

(B)Y×7=6

(C)7÷Y=6

(D)Y+6=7

8.759–215=

Whichisclosestto ?

(A)200

(B)300

(C)400

(D)500

9.Ednabought4packagesofballoonswith6ineachpackage,and2packageswith3largeballoonsineach.HowmanyballoonsdidEdnabuy?

(A)10

(B)15

(C)26

(D)30

10.

Whatnumberisa?

(A)0

(B)1

(C)7

(D)4028

11.7+ =15

Whichnumberisequalto ?

(A)15÷7

(B)15–7

(C)15×7

(D)15+7

12.Colleenis14yearsold.Shebabysitsfor$4.50anhour.Yesterdayshebaby-satfor3

hours.Whichshowshowmuchsheearned?

(A)14×$4.50

(B)2×3

(C)3 ×$4.50

(D)(3 ×2)×$4.50

13.

Whichofthefollowingdescribesthechartabove?

(A)

(B)

(C)

(D)

14.Whatnumbershouldbeinsertedtomaketheabovestatementcorrect?

(A)9

(B)8

(C)7

(D)Itcannotbedeterminedbytheinformationgiven.

15.Afenceisbeinginstalledaroundthe156-meterperimeterofaswimmingpool.Howmanypostswillbeusediftheyarespaced12metersapart?

(A)11

(B)12

(C)13

(D)14

16.Whatis28,973roundedtothenearestthousand?

(A)30,000

(B)29,000

(C)28,900

(D)28,000

17.Thebakeryreceivedashipmentof170cupcakesthatwillbesoldbythebox.Ifeachboxholds12cupcakes,approximatelyhowmanyboxeswillbeneeded?

(A)8

(B)14

(C)20

(D)25

18.

Whattimewillitbein3 hours?

(A)9

(B)9:15

(C)9:30

(D)9:45

19.WhichofthesenumbersentencesisNOTtrue?

(A)

(B)

(C) =1

(D)

20.

Whatistheareaofthisfigure?

(A)1sq.inch

(B)7sq.inches

(C)12sq.inches

(D)14sq.inches

21.Inordertomake cupofsaladdressingwithourrecipe,youadd cupofvinegarto

theoil.Howmuchoilwillyouuse?

(A) cup

(B) cup

(C) cup

(D) cup

22.Whichdigitisinthethousandthsplace?

(A)OnlyA

(B)OnlyB

(C)OnlyC

(D)BandC

23.WhichisNOTtrue?

(A)

(B)

(C)

(D)

24.Whichsequenceoffractionsisarrangedinorderofleasttogreatest?

(A)

(B)

(C)

(D)

25.Inwhichofthefollowingnumbersdoesthedigit6haveavalue10timesgreaterthanthevalueofthe6in603?

(A)60

(B)600

(C)6000

(D)60,000

26.Whichquotientwouldbeapproximately5?

(A)

(B)

(C)

(D)

27.

Ifthetemperaturedecreasesby15degreesbelowthatshownonthethermometer,whatwillthenewtemperaturebe?

(A)20°

(B)10°

(C)–10°

(D)–20°

28.5×(2+x)=15Whatnumberisx?

(A)1

(B)2

(C)4

(D)5

29.WhichofthefollowinghasaquotientthatisNOTsmallerthanthedividend?

(A)0÷8

(B)1÷8

(C)2÷8

(D)8÷8

30.

Whichofthefollowingisshownbythegraph?

(A)Therewasnochangeintemperaturebetween1and2p.m.

(B)Therewasnochangeintemperaturebetween3and4p.m.

(C)Thehighesttemperatureoccurredat12noon.

(D)Thelowesttemperatureoccurredat5p.m.

SECTION6:QUANTITATIVECOMPARISONS(ISEEONLY)20Questions•20Minutes

Directions:Foreachofthefollowingquestions,twoquantitiesaregiven—oneinColumnA,theotherinColumnB.Comparethetwoquantitiesandchoosefromtheanswerchoicesbelow:

(A)ifthequantityinColumnAisgreater

(B)ifthequantityinColumnBisgreater

(C)ifthequantitiesareequal

(D)iftherelationshipcannotbedeterminedfromtheinformationgiven

ColumnA ColumnB

1. 2x+3=5

3y+7=10x y

ColumnA ColumnB

2.

ColumnA ColumnB

3. xisaninteger

x

ColumnA ColumnB

4.

Numberofsecondsinoneday

NumberofminutesinApril

ColumnA ColumnB

5. 9<x<10

9<y<11x y

ColumnA ColumnB

6.

Areaofatrianglewithbase5andheight7

Twicetheareaofarectanglewithbase5andheight7

ColumnA ColumnB

7.

ABCDisaparallelogramAB DC

ColumnA ColumnB

8. 4•a•4•4=3•3•3•3

a 3

ColumnA ColumnB

9.

Asinglediscountof10% Twosuccessivediscountsof5%and5%

ColumnA ColumnB

10.

ColumnA ColumnB

11.TheaverageageofAlan,

Bob,andCarlis17.ThesumofAlan’sageandBob’sage

ThesumofAlan’sageandCarl’sage

ColumnA ColumnB

12.

a b

ColumnA ColumnB

13.

m 1+m 2+m 3 m 2+m 4

ColumnA ColumnB

14.Jack’ssalaryis ofJim’ssalaryand

Joe’ssalaryis ofJack’ssalary.

Jim’ssalary Joe’ssalary

ColumnA ColumnB

15.

AB BC

ColumnA ColumnB

16.

ColumnA ColumnB

17.

Thetotalsurfaceareaofacubewithedge6

6timesthetotalsurfaceareaofacubewithedge2

ColumnA ColumnB

18.

0.5%

ColumnA ColumnB

19.

18•563•10 12•563•16

ColumnA ColumnB

20.

Thenumberofpostsneededforafence100feetlongifthepostsareplaced10feetapart

10posts

ANSWERSANDEXPLANATIONSSection1:Synonyms(SSATandISEE)

1.Thecorrectansweris(D).ThatwhichhappensINTERMITTENTLYstopsandstartsagainatintervalsorpausesfromtimetotime.Apedestriancrossinglightthatisactivatedbyapushbuttonsays“WALK”and“DON’TWALK”intermittently.Youmayseearelationshiptointermission,whichisapausebetweenpartsofaperformance.

2.Thecorrectansweris(B).OnewhopracticesDECEPTIONwillfullytriestomakeanotherbelievethatwhichisnottruetomisleadordefraud.Theverbonwhichthisnounisbasedisdeceive.

3.Thecorrectansweris(D).ToACCLAIMistogreetwithloudapplauseandapproval.ThewinnersoftheWorldSeriesreturnedtothecitytowildacclaim.

4.Thecorrectansweris(D).ToERECTistoraise,construct,setup,orassemble.Theadjectiveerectdescribesthatwhichisverticalandstraightup.Youcanseetherelationshipofverbandadjective.

5.Thecorrectansweris(D).IfyouRELISHsomething,youenjoyit.Youmaybemorefamiliarwiththenoun,withtherelishthatyouputonyourhotdogtomakeitmoreappetizingandtoaddtoyourenjoyment.

6.Thecorrectansweris(A).AFORTNIGHT(fourteennights)isaperiodoftwoweeks.

7.Thecorrectansweris(C).ToIMPOSEataxistoplaceataxuponthetaxpayersandtorequirethatitbepaid.Becarefulhere.Thetax(noun)istheobjectoftoimpose(verb),notthedefinitionorasynonym.

8.Thecorrectansweris(D).AnALIASisanassumedname.Thealiasmaybeapennameorastagename,oritmaybeafalsenametakenonforpurposesofdisguise.

9.Thecorrectansweris(A).AnITINERANTpeddlertravelsfromplacetoplace,sellinghisorherwaresateachstop.Theitinerantmayfollowanitinerary,adetailedoutlineoftheproposedjourney.

10.Thecorrectansweris(A).ThatwhichisAMPLEislarge,spacious,abundant,orplentiful.Whenreferringtoafigure,amplemayimplyoverweight,butthemeaningofthewordampleissimplymorethanenough.

11.Thecorrectansweris(C).StagnantwateroradecayingdeadanimalmayhaveaSTENCHorfoulodor.STENCHmeansstink.

12.Thecorrectansweris(D).ASULLENpersonisresentful,unsociable,andgloomy.

13.Thecorrectansweris(A).TERSEmeansconciseandsuccinct.Itistheoppositeof“lengthy”andhasnothingatalltodowithnervesortension.

14.Thecorrectansweris(B).AnINCREMENTisaspecifiedincrease,usuallyasmallone.Sometimestryingtofigureoutthemeaningsofwordsbylookingattheirrootsjustdoesn’twork.Thiswordhasnothingtodowith“excrement,”eventhoughitlookssimilar.

15.Thecorrectansweris(D).ToMISCONSTRUEistomisinterpret.“Wordarithmetic”workswellwiththissynonymquestion.Theprefix“mis”meanswrong;therootisthesamerootas“construct,”sotomisconstrueistobuildupwronginone’smind.

16.Thecorrectansweris(B).AVESTIGEisatrace.Thisisadifficultword,onethatispickedupinthecourseofwidereading.

17.Thecorrectansweris(A).ToCAPITULATEistogivein,giveup,orsurrender.Capitulationismoreatotalactthanamereretreat.

18.Thecorrectansweris(A).EXTENUATINGismakinglightofsomethingorlesseningtherealorapparentseriousnessofsomething.Insentencingaguiltyparty,thejudgemayconsiderextenuatingcircumstances,detailsthatmaketheguiltthinnerorweaker.Ifthereareextenuatingormitigatingcircumstances,thesentencemaybelighter.Ifyoumissedthisquestion,donotbeupset.Boththewordanditsdefinitionareveryhigh-levelwords.

19.Thecorrectansweris(C).SUBSERVIENT—servingunder—meansinferior,subordinate,orsubmissive.

20.Thecorrectansweris(C).COLLUSIONisaconspiracy,asecretagreement,oraconnivance.Useofthiswordimpliesanillegalgoal.

Section2:VerbalAnalogies(SSATOnly)

1.Thecorrectansweris(D).Therelationshipisintensityordegree.PINKisamutedformofRED;GRAYisamutedformofBLACK.Whiteistheoppositeofblack,whiledarkisoneofblack’scharacteristics.

2.Thecorrectansweris(D).YOUTHisanounform,YOUNGanadjective.Bothrefertotheearlyyears.MATURITYisanounform,OLDanadjective.Maturityandoldrefertothelateryearsofexistence.

3.Thecorrectansweris(C).Therelationshipbetweenthefirsttwotermscannotbedetermineduntilyoulookatthethirdtermandthechoices.Atfirstyoumightthink,“Two

followsone,sofourshouldfollowthree.”Theproblemisthat“four”isnotofferedasananswerchoice.Ofthechoicesgiven,SIXismostrelatedtoTHREE,sinceSIXistwiceTHREE.Lookingbackatthefirstpair,youcanseethatTWOistwiceONE.Sincethe“timestwo”relationshipappliestobothsidesoftheproportion,choice(C)isthecorrectanswer.

4.Thecorrectansweris(A).Therelationshipiseffect/cause.LIGHTisproducedbyaLAMP;HEATisproducedbyaFURNACE.Thesunalsoproducesheat,butthesunisanaturalsource.Lampintheoriginalpairisanartificialsourceoflight;furnace,thebestanswer,isanartificialsourceofheat.

5.Thecorrectansweris(E).Therelationshipisparttowhole.WEEKispartofMONTH;SEASONispartofYEAR.

6.Thecorrectansweris(C).ASQUAREisangular,aCIRCLErounded;aRECTANGLEisangular,anOVALrounded.Youmightsay“angularistoroundedasangularistorounded.”

7.Thecorrectansweris(B).Therelationshipisthatofobjecttoactor.TheDIRECTORleadstheCHOIR;theCOACHleadstheTEAM.

8.Thecorrectansweris(D).Therelationshipisoneofcharacteristic.SANDisthesoilthatischaracteristicoftheBEACH;BLACKDIRTisthesoilthatischaracteristicofaproductiveFARM.

9.Thecorrectansweris(A).Thisisatruefunctionalrelationship.TheTABLEissupportedbyLEGS;theAUTOMOBILEissupportedbyWHEELS.Itisimportanttonarrowtothefunctionalrelationshipinordertoeliminatechoices(B)and(C).Useofthepart/wholerelationshipwouldyieldtoomanycorrectanswers.

10.Thecorrectansweris(E).AROUSEandPACIFYareantonyms;AGITATEandSMOOTHareantonyms.Ifyouknowthemeaningsofthewords,thisisaneasyanalogyquestion.Ruffleisasynonymforagitate.

11.Thecorrectansweris(C).Therelationshipisthatofartificialtorealorsubstitutetotherealthing.MARGARINEisimitationBUTTER;NYLONisimitationSILK.Laceisoftenmadefromcottonbutdoesnotsubstituteforit.

12.Thecorrectansweris(A).Therelationshipisaveryspecificfunctionalone.TheWOODSMANusesanAXEtocut;theCARPENTERusesaSAWtocut.Inalltheotherchoices,weareofferedaworkerandatoolofhistrade,butnoneofthesetoolsisusedspecificallyforcutting.

13.Thecorrectansweris(E).Therelationshipisoneofdegree.OnewhoisWORRIEDmay

becomeHYSTERICAL(overwhelmedbyfear);onewhoisHAPPYmaybecomeECSTATIC(overwhelmedbyjoy).

14.Thecorrectansweris(D).Therelationshipisoneofcauseandeffect.CONTROLofagroupresultsinORDER;ANARCHY(lackofgovernmentorcontrol)resultsinCHAOS(disorder).

15.Thecorrectansweris(B).Therelationshipisoneofdegree(large/small)or,morespecifically,thatofparenttochild.TheHORSEistheparentoftheFOAL;theCOWistheparentoftheCALF.Donkeyandassaresynonyms.Asteerisabullraisedforitsmeat.

16.Thecorrectansweris(A).Thebestwaytoverbalizethisrelationshipistocallitthatofthecuretoitsailment.SLEEPcuresFATIGUE;WATERcuresTHIRST.Bewareofthegrammaticalinconsistencyinchoice(B).Restcures“weariness,”notweary.Therelationshipofchoice(D)iseffecttocause.

17.Thecorrectansweris(C).Thisisavariationofthepart-to-wholerelationship.AnISLANDisasmallbodyoflandwithinandwhollysurroundedbyanOCEAN;anOASISisasmall,green,fertilespotwithinandwhollysurroundedbyalargearidexpanseofDESERT.

18.Thecorrectansweris(D).Therelationshipisthatofactortoobject.TheDIRECTORisresponsiblefortheproductionofaDRAMA;theEDITORisresponsiblefortheproductionofaMAGAZINE.Theprincipalisresponsibleforanentireschool,notjustaclass.Responsibilityforasingleclassrestswithateacher.

19.Thecorrectansweris(C).Therelationshipisoneofdegree.ToDEMANDistoREQUESTverystrongly;toCRAVEistoWISHforverystrongly.

20.Thecorrectansweris(E).Atfirstglance,therelationshipisobjecttoaction.However,thisdefinitionisnotspecificenoughtoallowforasingleanswerchoice.Acarefullookatthepossibleanswershelpsleadtoarefinementoftherelationshipto“creativemanualactionuponanobjectormediumthatischangedbythataction.”Theonlyanswerwiththisrelationshipischoice(E),CLAYistoMOLD.

Section3:SentenceCompletions(ISEEOnly)

1.Thecorrectansweris(D).Thefirstblankmustbefilledwithwhatitisthatfortune-tellersdo.Thisnarrowsyouranswertochoices(C)or(D).Therebeingnoscientificevidenceoftheirfeesmakesnosense,sotheanswerischoice(D).

2.Thecorrectansweris(B).Thesentencecallsforsynonyms,asthesecondclausemerelyexpandsuponthefirst.ETERNALandIMMORTALbothmeaneverlasting.

3.Thecorrectansweris(D).Thesecondclauseofthesentenceismeanttocorroboratethefirst.Sinceeachhumanrelationshipisunique,theloverswhoassumetheuniquenessoftheirownrelationshipareRIGHT.

4.Thecorrectansweris(B).Theword“but”givesthecluethattheblanksmustbefilledwithcontrastingterms.Choice(B)bestmeetsthiscondition.

5.Thecorrectansweris(D).Iftheideaofwhatwasrightlayinthepast,thenauthoritywouldhavetobegiventothosewhoknewandpreservedancientloreandhabits,e.g.,TRADITION.

6.Thecorrectansweris(B).Sincethedesertrainscauselossoflifeandproperty,peopleinanoasismustthinkofrainwithTERROR,choice(B),orfear,choice(C).Thewordthatbestfillsthefirstblankwilldeterminewhetherthecorrectanswerischoice(B)or(C).Sprinkles,choice(C),wouldbeunlikelytocauselossoflifeandproperty;TORRENTS,choice(B),ismoreappropriate.

7.Thecorrectansweris(B).Iftheessayist’sessayswerenotwidelyknown,hewouldhavefew(notNUMEROUS)admirers.

8.Thecorrectansweris(B).Inlightofthefactthatarchaeologistsfoundtemplesandpalaces,choices(A),(C),and(D)cannotbecorrect.Thepeopleclearlylived,thought,andbuilt.Choice(B)presentsamystery,butitdoescreatealogicallycorrectsentence.

9.Thecorrectansweris(B).SafedrivingpreventsTRAGEDY,choice(B),accidents,choice(C),andlawsuits,choice(D).Thesecondpartofthesentence,however,isreasonablycompletedonlywithchoice(B).

10.Thecorrectansweris(D).Thepurposeofamateursportsisrecreation;theamateurparticipatesforfun,forLOVEofthesport.(ThewordamateurisderiveddirectlyfromLatinandmeans“lover.”)CheatingisDEGRADING;itmakesanactivitylessdesirable.

11.Thecorrectansweris(A).Ifthereistobeameeting,itmightbewelltohaveFOOD,choice(A),secretaries,choice(B),andtelephones,choice(C),available.However,noneoftheseisspecificallyneededatanenjoyable,choice(B),meetingoratasuccessful,choice(C),meeting.IfthemeetingislikelytobePROLONGED,choice(A),thenitisworthwhiletoprovideFOOD.

12.Thecorrectansweris(D).Choices(A)and(D)formexcellentcompletionsforthissentence.Choice(D)isthebestanswerbecauseSOLITUDEisindirectcontrasttothecompanyofothers.

13.Thecorrectansweris(D).ExecutingthelawsrequireslawENFORCEMENT.

14.Thecorrectansweris(D).Afour-dayworkweekwouldgiveworkersmuchmoreLEISUREtime,whichwould,inturn,greatlyBENEFITtheLEISUREindustry.

15.Thecorrectansweris(B).Theadjectiveruthless(cruel)aptlydescribesaTYRANT.Sincethesenseofthesentencecallsforcontrast,achangefromchampionofFREEDOMtoruthlessTYRANTfulfillstherequirement.

16.Thecorrectansweris(D).ThereasonthatasocietywouldneedoutsideaidwouldbethatthesocietywasnotSELF-SUFFICIENT.

17.Thecorrectansweris(D).Thesentencerequiresthattherebeacause-and-effectrelationshipbetweenthetwowordsthatfilltheblanks.Onlychoice(D)meetsthisrequirement.

18.Thecorrectansweris(B).Themeaningofanyoneofthechoiceswouldbeequallyappropriateforfillingthefirstblank,althoughchoice(A)wouldbegrammaticallyincorrect.Thewordsvalueandprogressarepositivewords,suggestingtheneedforapositivecompletionforthesecondblank.Theonlypositivesecondtermisinchoice(B).

19.Thecorrectansweris(C).ThefinalendisEXTINCTION.

20.Thecorrectansweris(C).TheroleofadirectoristoperfectthePERFORMANCEthroughnecessarycriticism.AtemperamentalactressmightmisinterpretdirectionasPERSONALcriticism.

Section4:ReadingComprehension(SSATandISEE)

1.Thecorrectansweris(C).Themainidea,thetopicofthispassage,islookingcloselyatcolorsinordertotrulyrecognizethem.Thefarmeristhevehicleformakingthepoint,andtheskyandhillsareusedasillustrations.

2.Thecorrectansweris(A).Thelastsentenceofthefirstparagraphgivesyouthisdetail.

3.Thecorrectansweris(B).Youshouldbeabletoinferthisanswerfromthepassage.Ifnot,youcanchoosethecorrectanswerbyeliminatingalltheotherchoicesasbeingridiculous.

4.Thecorrectansweris(B).Rereadthefirstparagraphcarefullyifyougotthiswrong.Thephrase“movesuptowardthezenith”shouldleadyoutothecorrectanswer.

5.Thecorrectansweris(D).Youshouldhavenotroublefindingthemainideaofthispassage.

6.Thecorrectansweris(A).ThefirstsentencetellsusthatthePolynesiansmadetripstoNewZealand.

7.Thecorrectansweris(B).Thefirstsentenceofthelastparagraphmakesclearthatmarinersarepeople.

8.Thecorrectansweris(D).ThethirdparagraphtellsushowthePolynesiansusedthestarsfornavigation.Peoplewhostudyandunderstandmovementsofthestarsareastronomers.AlthoughthePolynesiansusedmovementsofseabirdsasaguideduringthedaytime,theyhadnottrainedthebirds.Thereisnomentionoftelescopes.

9.Thecorrectansweris(B).Thetopicofthepassageisbirdmigrationandthemysteryithaspresentedthroughouttheages.

10.Thecorrectansweris(D).Thelastsentenceofthethirdparagraphmakesthisstatement.

11.Thecorrectansweris(D).Youwillfindthisinformationwithintheparenthesesinthesecondparagraph.

12.Thecorrectansweris(A).Thisisthepointofthelastparagraph.

13.Thecorrectansweris(C).Choosingthetitleforthisparagraphtakesmorethanonereadingoftheparagraph.Thisisnotaneasyquestion.Afteracoupleofreadings,however,youshouldbeabletoconcludethattheall-inclusivesubjectoftheparagraphistheremodelingofplants.Anequallycorrecttitle,notofferedhere,mightbe“UsesandEffectsofColchicine.”

14.Thecorrectansweris(C).Buriedinthemiddleoftheparagraph(lines6–7)isthesentence:“Itcreatesnewvarietieswithastonishingfrequency,whereassuchmutationsoccurbutrarelyinnature.”

15.Thecorrectansweris(C).Thisquestionbecomeseasytoanswerafteryouhavedealtwiththepreviousquestion.

16.Thecorrectansweris(A).Thethirdsentencestatesthatcolchicineisapoisonousdrug.

17.Thecorrectansweris(D).Theselectionisaboutthebeginningofsteamferryboatservice.

18.Thecorrectansweris(A).Seelines3–4,whereitstatesthattheboatswere“knownastwinboats,eachofthemhavingtwocompletehullsunitedbyadeckorbridge.”

19.Thecorrectansweris(D).Thenext-to-lastsentencediscussesFulton’sinventionoffloatingdocksfortheferryboats.

20.Thecorrectansweris(C).1812wasinthenineteenthcentury.

21.Thecorrectansweris(B).Inthecontextoftheparagraph,shockmustrefertotheimpactoftheboatrunningintothedock.

22.Thecorrectansweris(B).ThefirstlineofthepassagestatesthattwoofthefirststeamferryboatswerebuiltforcrossingtheHudsonRiver.

Section5:QuantitativeAbility(SSATandISEE)

1.Thecorrectansweris(A).Thissquareisdividedintosixsections,andthreeareshaded.Threeishalfofsix.

2.Thecorrectansweris(D).If2packagesofcookieswillserve10children,youmayassumethat1packagewillserve5children.Therefore,3packageswillserve15children.

3.Thecorrectansweris(C).

4.Thecorrectansweris(D).$12.00×5weeks=12×5

5.Thecorrectansweris(B).IfCrepresentsthedistancefromCityXtoChicago,andSrepresentsthedistancefromCityXtoSanFrancisco,then3×C=S.YoumustknowthevalueofCinordertofindS.

6.Thecorrectansweris(B).Anoddnumbersubtractedfromanoddnumberwillalwaysresultinanevennumber.Theonlyevennumbergivenis2.

7.Thecorrectansweris(A).Since7×6=Y,Y=42.So42÷7=6.

8.Thecorrectansweris(D).759–215=544,which,whenroundedtothenearesthundred,is500.

9.Thecorrectansweris(D).4×6=24and2×3=6;24+6=30.

10.Thecorrectansweris(D).Anynumberdividedbyitselfequalsone.

11.Thecorrectansweris(B).15–7=8;7+8=15

12.Thecorrectansweris(C).Threenumbersaregiveninthisproblem,butonlytwoarenecessarytosolvetheproblem:thechargeperhourandthenumberofhoursColleen

babysat.$4.50perhour×3 hours=$4.50×3

13.Thecorrectansweris(A).Thepiehasbeendividedinto8pieces.Onepiece= .Four

pieces= .Fourpiecesareleftinthispicture,andoneisbeingtakenaway;therefore,

describesthechart.

14.Thecorrectansweris(A).Thesymbol<means“islessthan.”Usingchoice(A),thestatementreads:8862islessthan8962.

15.Thecorrectansweris(C).156meters÷12meters=13.

16.Thecorrectansweris(B).Inthenumber28,973,thedigit8isinthethousandsplace.Thehundredsdigitisgreaterthan5,sothenextnearestthousandis9;therefore,29,000istheanswer.

17.Thecorrectansweris(B).170÷12=14witharemainderof2.Theclosestnumbergivenis14.

18.Thecorrectansweris(B).Thetimeisnow5:45.In3morehoursitwillbe8:45.Thirtyminuteslaterthetimewillbe9:15.

19.Thecorrectansweris(D).

20.Thecorrectansweris(C).Theareaofarectangleisfoundbymultiplyingthelengthbythewidth. squareinches

21.Thecorrectansweris(B). ,and .

22.Thecorrectansweris(C).Aisinthethousandsplace,andBisinthehundredthsplace.

23.Thecorrectansweris(B).

24.Thecorrectansweris(B).Thedenominatorofafractionshowshowmanypartsthewholehasbeendividedinto.

Therefore, < < < .

25.Thecorrectansweris(C).The6in603represents600;600×10=6000.

26.Thecorrectansweris(D).1005÷200=5,witharemainderof5.Theanswerisapproximately5.

27.Thecorrectansweris(C).5°–15°=–10°

28.Thecorrectansweris(A).5×(2+1)=5×3=15

29.Thecorrectansweris(A).0÷8=0.Theotherresponsesworkoutthisway:choice(B)

is1÷8= ;choice(C)is2÷8= ;andchoice(D)is8÷8=1.In0÷8,0isthe

dividendand8isthedivisor.Thequotientistheresultobtainedwhenthedividendisdividedbythedivisor.

30.Thecorrectansweris(A).Accordingtothegraph,thetemperatureat1p.m.was20°,andat2p.m.itwasstill20°.

Section6:QuantitativeComparisons(ISEEOnly)

1.Thecorrectansweris(C).Solvingtheequations,x=1,y=1.

2.Thecorrectansweris(C).Solvingforx,x=12,or .

3.Thecorrectansweris(D).Ifx>0,Aisgreater.Ifx<0,Bisgreater.Ifx=0,AandBareequal.

4.Thecorrectansweris(A).60seconds=1minute;60minutes=1hour;24hours=1day;sothenumberofsecondsin1day=60•60•24.Thereare30daysinApril;thenumberofminutesinApril=60•24•30.Withoutanycomputation,Ahasthegreaterfactors.

5.Thecorrectansweris(D).xandycantakeonvaluessothatAisgreater,Bisgreater,ortheyarebothequal.

6.Thecorrectansweris(B).

Areaoftriangle= •5•7

Twiceareaofrectangle=2•5•7

7.Thecorrectansweris(C).Oppositesidesofaparallelogramarecongruent.

8.Thecorrectansweris(B).If64a=81,thena=alittlemorethan1.

9.Thecorrectansweris(A).ColumnArepresentsagreaterdiscountsincethefull10%istakenfromtheoriginalprice.

10.Thecorrectansweris(A).

isalittlemorethan8.

11.Thecorrectansweris(D).Thereisnoinformationabouttheageofanyoftheseindividuals.

12.Thecorrectansweris(D).aandbcantakeonvaluestomakeeitheronegreaterortomakethemequal.

13.Thecorrectansweris(C).Eachoftheseis180°.

14.Thecorrectansweris(B).IfJim’ssalary=8x,thenJack’ssalary=6x,andJoe’ssalary=9x.

15.Thecorrectansweris(C).Thereare120°leftbetweenangleBandangleC.Sincetheymustbecongruent,theyareeach60°andthetriangleisequilateral.

16.Thecorrectansweris(B).Sincethenumeratorsarethesame,thefractionwiththesmallerdenominatorwillbelarger.

17.Thecorrectansweris(A).Surfaceareaofacubeisthesumof6equalsquares.

ThefactorsaregreaterinA.

18.Thecorrectansweris(B).

=0.5

0.5%=0.005

19.Thecorrectansweris(B).18×10=180;12×16=192

20.Thecorrectansweris(A).Sinceapostisneededattheverybeginningaswellasattheend,Arequires11posts.

EVALUATINGYOURSCOREANDPLANNINGYOURSTUDYTIMEBeginbyenteringonthescoresheetthenumberofquestionsyougotcorrectineachsectionandcalculateyourpercentagescore.Yourscoresontheactualexamwillnotbereportedinpercentages.Youwillreceivepercentilescoresthatcompareyourperformancewiththatofotherstudentsyourageandinyourgrade.Yourpercentilestandingisveryusefulinformationfortheschoolthatmustbaseitsadmissiondecisiononyourpreparedness.Foryourpurposes,however,thepercentofquestionsyougotrightcanhelpyoucompareyourownperformanceoneachquestiontype.Yourpercentscorecanpointouttoyouyourownstrengthsandweaknessesandhelpyoutoplanyourstudytime.

ScoreSheet

SectionNo.

correct ÷No.of

questions = __×100 = __%

Synonyms(SSATandISEE) _____ ÷ 20 = __×100 = __%

VerbalAnalogies(SSATOnly) _____ ÷ 20 = __×100 = __%

SentenceCompletions(ISEEOnly) _____ ÷ 20 = __×100 = __%

ReadingComprehension(SSATandISEE) _____ ÷ 22 = __×100 = __%

QuantitativeAbility(SSATandISEE) _____ ÷ 30 = __×100 = __%Quantitative

Comparisons(ISEEOnly) _____ ÷ 20 = __×100 = __%

Asyoulookoveryouranswersheet,youmaynoticethatyoumissedsomeoftheearlier,“easier”questionsandgotsomerightneartheendofasection.Thisshouldnotsurpriseyou.Afterall,youareanindividualwithyourowninterests,talents,andthoughtprocesses,andyourcurrentschoolmayintroducesometopicsinalessusualorder.Ifyouwereabletoanswercorrectlymanyquestionsonthediagnosticexam,youarealreadyaheadofthegame.Ifyoudidn’tgettoofarinthetimelimitorifyougotmanyanswerswrong,donotbediscouraged.Firstconsideryourageandinexperiencewithsomeofthequestionstyles.Thencongratulateyourselfthatyouhadtheforesighttoplanastudyprogram,andgettowork.

Plotyourstandingoneachsectiononthecomparisonchartforyourtest.Putacheckmarkintotheboxesinwhichyourscoresfall.Nowyoucanseeataglanceexactlywhereyouwillhavetoconcentrateyourstudy.Ifyouhavelowerpercentagescoresincertainareas,focusyourpreparationonthoseareas.

ComparisonChart—SSAT

PercentCorrect

0–30 31–45 46–60 61–75 76–100

Synonyms

VerbalAnalogies

ReadingComprehension

QuantitativeAbility

ComparisonChart—ISEE

PercentCorrect

0–30 31–45 46–60 61–75 76–100

Synonyms

SentenceCompletions

ReadingComprehension

QuantitativeAbility

QuantitativeComparisons

PARTIIIVOCABULARYREVIEW

CHAPTER5:WordArithmetic(SSATandISEE)

Chapter5

WordArithmetic(SSATandISEE)

OVERVIEW

•Whydotheytestmyvocabulary?•Howarewordsbuilt?•Howdowordpartswork?•Listofcommonwordparts•Wordlist•Summingitup

WHYDOTHEYTESTMYVOCABULARY?Yourabilitytounderstandandtousewordsisessentialtoallofyourlearning.Ifyourtestperformanceonverbalabilityexamsshowsthatyouhavearichvocabulary,youareagoodprospectforsuccessattheschoolstowhichyouhaveapplied.

HOWAREWORDSBUILT?Thesubjectisnotlettersofthealphabetbutratherthepartsofwordsthemselves.Youcanactuallyincreaseyourvocabulary—andyourtestscore—bylearningaboutthestructureofwords.Thiswillhelpyoufigureoutthemeaningsofunfamiliarwordsyoucomeacrossintheverbalabilitysectionofyourexam.

Knowingwhatthepartsofwordsmeanisthekeytodecipheringwordsyou’veneverseenbefore.Let’stakealookatthewordbiographyanditsparts.Youknowthatabiographyissomethingwrittenaboutaperson’slife.Howdothewordpartstellyouthis?Well,thesecondpartoftheword,graphy,comesfromaGreekwordthatmeans“writing.”Thefirstpartofthe

word,bio,isalsofromGreekandmeans“life.”Putthembothtogetherandyougetbiography,thestoryofaperson’slife.IfyouaddtheLatinwordfor“self”—auto—yougetautobiography,astoryyouwriteaboutyourownlife.Thinkaboutsomeotherwordsthatuseoneormoreoftheseparts,likeautomobile,biochemistry,andautograph.Canyouseehowthemeaningfitsthewordparts?

TIPPracticewordarithmetic.Wordsarethesumoftheirparts.Ifyouencounteranunfamiliarwordonyourexam,decipheritspartsandaddthemuptogetthemeaning.

HOWDOWORDPARTSWORK?Differentkindsofwordpartsworktogethertomakeafullyfunctioningword.Thinkaboutit:Ifyourcarisgoingtodomorethanjustsitthere,itneedsacollectionofpartsputtogetherintherightway.Twosteeringwheelswon’tdoyouanygoodifyoudon’thaveagastank.

Eachkindofwordparthasaspecificpurpose.Therearethreebasictypesofwordparts:Prefixesattachtothebeginningofarootwordtoalteritsmeaningortocreateanewword.Suffixesattachtotheendofarootwordtochangeitsmeaning,helpmakeitgrammaticallycorrectincontext,orformanewword.Suffixesoftenindicatewhetherawordisanoun,verb,adjective,oradverb.Rootsorstemsarethebasicelementsofawordthatdetermineitsmeaning.Rootsorstemsarenotwords.Theymustbecombinedwithprefixes,suffixes,orboth.Groupsofwordsfromthesamerootwordarecalled“wordfamilies.”

Awordcanhavearoot,aprefix,andasuffix;itcanhavearootandtwosuffixesorarootandoneprefix.Thepossibilitiesareendless(almost),butyoumustalwayshavearoot.Wordanalysisisakindofarithmetic.Insteadofaddingnumbers,weaddthemeaningscontainedineachpartofanunfamiliarword.Thesumofthesepartsisthedefinitionofthewholeword.

Usethewordlistthatfollowstoexpandyourwordhorizons.Onceyoubegintolearnthewordpartsonthelist,you’llbeabletotakeapartunfamiliarwordslikeamastermechanic.Asyoumakeyourwaythroughthelist,trytothinkofotherwordswiththesameparts.Ifyouhavetime,checktheirmeaningsinadictionaryandtakealookatthewordoriginsintheentry.

LISTOFCOMMONWORDPARTS

Prefix Meaning Example

a not amoralab awayfrom absentad,ac,ag,at to,against aggressive,attractan without anarchyante before antedateanti,ant against antipathy,antonymbene well benefactorbi two biannualcircum around circumventcom,con,col together commit,collatecontra against contrabandde from,down descenddis,di apart,away distract,divertdom home,rule domicile,dominateex,e out,from exit,emitextra beyond,outside extracurricularin,im,ir,il,un not inept,irregular,illegalin,im in,into interest,imbibeinter between interscholasticintra,intro within intramuralmal bad malcontentmis wrong misspellnon not nonentityob against obstacleomni all omnivorousper through permeateperi around,about periscopepoly many polytheismpost after postmortempre before premonitionpro forward,for proposere again,back review,redeemse apart,away secludesemi half semicirclesub under submarinesuper above superimposesur on,upon surmountsyn,sym together,with sympathytrans across,beyond transposeun not unwelcome

Suffix Meaning Example

able,ible capableof applicable,reversible(adj.)age place,thing,idea storage(n.)al pertainingto instructional(adj.)ance relatingto reliance(n.)ary relatingto dictionary(n.)ate anactionof confiscate(v.)cy thequalityof democracy(n.)ed pastaction subsided(v.)ence relatingto confidence(n.)er,or onewho adviser,actor(n.)

ic pertainingto democratic(adj.)ing presentaction surmising(v.)ion theactorstateof radiation(n.)ious fullof rebellious(adj.)ive havingthequalityof creative(adj.)ize tomake harmonize(v.)ly todowiththequalityof carefully(adv.)ment theresultof amusement(n.)ness thequalityofbeing selfishness(n.)ty conditionofbeing sanity(n.)

Stem Meaning Example

ag,ac do agenda,actionagri farm agricultureaqua water aquaticauto self automaticbiblio book bibliographybio life biographycad,cas fall cadence,casualcap,cep,cept take captive,acceptcapit head capitalced,cede,ceed,cess go intercedeceler speed acceleratechrom color monochromaticchron time chronologicalcide,cis cut incisionclude,clud,clus close,closein include,clustercog,cogn knowledgeof recognizecur,curs run incur,recurded give dedicatedent,dont tooth dentalduce,duct lead induce,deductfact,feet,fict make,do perfect,fictionfer,late carry refer,dilateflect,flex bend,turn reflectfring,fract break infringe,refractgraph,gram picture,writing graphic,telegramgreg group,gather gregariousgress,grad move progress,degradehydr water hydrateject throw injectjud right judicialjunct join conjunctionjuris law,justice juristlect,leg read,choose collectlogue speech,speaking dialoguelogy studyof psychologyloq,loc speak elocutionlude,lus play,perform eludemanu byhand manuscriptmand order remandmar sea maritimemed middle intermediate

ment,mem mind,memory mentionmeter measure thermometermicro small microscopemin lessen miniaturemis,miss,mit send remit,dismissmot,mov move remote,removemute change commutenaut sailor,sail nauticalnounce,nunci declare,state announce,enunciateped,pod foot pedalpel,pulse drive,push dispel,impulsepend,pense hang,way depend,dispenseplac please placateplic fold implicateport carry portablepose,pone put,place depose,componentreg,rect rule regulate,directrupt break disruptionscend,scent move ascentscribe,script write describesec,sect cut bisectsed remain sedentarysert state,place insertserve keep,save preservesist stand,set insistspect look inspectspire,spirat breath,breathe perspirestrict tighten restricttain hold detainterm end terminatetort twist distorttract draw,drag detractvene,vent come intervene,inventvict overcome,conquer evictvolve,volu roll,turn evolve,evolution

WORDLISTThewordsinthislistaregroupedin“families,”bytheirstems.Thestemsarearrangedinalphabeticalorder.Ifyouneedhelpwithwordsotherthanthoselistedhere,checkyourdictionaryorthesaurus.

AQUA,AQUE:water

aquarium—atankforfishandwaterplants

aquatic—havingtodowithwater

aqueous—watery

subaqueous—underwater

AUTO:self

autobiography—thestoryofone’slife,writtenbyoneself

autograph—aperson’sownsignature

automatic—self-operating

automation—asysteminwhichmachinerydoesmostoftheworkitself

BIBLIO:book

bibliography—alistofbooksusedforreference

bibliolatry—worshipofbooks

BIO:life

antibiotics—medicinesthatworkagainstharmfullife-formsinthebody

biography—thestoryofaperson’slife

biology—thestudyofvariouslife-forms

symbiosis—mutualinterdependenceoftwodifferentlivingorganisms

CAP,CEP,CEPT:take

accept—totakein

capture—totakebyforce;totakeprisoner

exception—somethingtakenorleftout

inception—theactoftakingsomethingin;abeginning

ED,CEDE,CEED,CESS:go,move

accede—tomovetoward;togrant

concede—togowith;admit

concession—anadmission

exceed—togooveroroutsideof

excess—goingovercertainlimits

intercede—tomovebetween

precede—togobeforeoraheadof

proceed—tomoveforward

recede—tomoveback

secession—theactofmovingapart;separationfromthewhole

CHRON:time

anachronism—somethingcontrarytoaparticularera

chronometer—atoolthatmeasurestime

CIDE,CIS:cut

decide—toact;tocutofffromfurtherconsideration

excise—tocutoutoraway

incision—acut

CLUDE,CLUS:close,shut

exclude—tocloseorshutout

exclusive—havingthequalityofshuttingout

include—toshutorclosein

recluse—apersonwhoshutshim-orherselfawayfromothers

seclude—toshutapartfrom

COG,COGN:knowledgeof

cognizant—thequalityofbeingknowledgeable

incognito—unknown;disguised

recognition—theactofknowingagain;recalling

CUR,CURS:run

concurrent—runningwith;atthesametime

cursory—thequalityofrunningthroughquickly

precursor—aforerunner

recurrent—runningagain

DUCE,DUCT:lead

abduct—toleadaway;tokidnap

conducive—havingthequalityofleadingtogether;persuasive

deduce—toleadfrom;toconclude

introduce—toleadinto

reduction—theactofleadingbackward;aloss

FACT,FECT,FICT:make,do

affectation—somethingmadeup;apretense

defect—somethingmadeapartfromtheordinary

effect—somethingdoneoutwardly;achange

fictitious—madeup;nottrue

FER:carry,bring

conference—thestateofbringingtogether;ameeting

differ—tocarryapart;todisagree

ferret—toforceoutofhiding;tosearchabout

infer—tobringin;toconclude

offer—tocarryout

preference—theactofbringingfirstorbefore

reference—theactofcarryingback

transfer—tocarryacross

FLECT,FLEX:bend,turn

circumflex—anaccentmarkthatisbentoveraletter

flexible—abletobebentorchanged

genuflect—tobendthekneetothefloor

reflect—tobendorturnsomethingback

reflex—areturnmovement;aresponse

FRING,FRACT:break

fracture—abreakinsomethinghard

infraction—thebreakingofarule

infringe—tobreakinto

refract—tobreakup

GRAPH,GRAM:picture,writing

diagram—informationinpictureform

epigram—abriefpieceofwriting

graph—informationinpictureform

seismograph—apicturerecordofearthmovements

telegram—communicationusingsoundandwriting

telegraph—communicationwithsoundsthataretranslatedintowriting

GREG:group,gather

aggregation—agroup

congregate—togathertogether

gregarious—havingthecharacteristicofgettingalongwellinagroup;social

segregate—togroupapart;tokeepgroupsseparate

GRESS,GRAD:move

aggressive—thecharacteristicofmovingtowardsomething

degrade—tomovedown

graduate—tomovefromonelevelofprestige,proficiency,orexperiencetoahigherone

ingredient—somethingthatismovedintosomethingelse

progress—tomoveforward

regress—tomovebackward

upgrade—tomoveup

HYDR:water

dehydrate—toremovewaterfrom

hydrant—somethingthatgivesoutwater

hydraulic—relatingtowaterpower

hydrogen—watergas

hydrology—thestudyofwater

hydrophobia—fearofwater

hydrotherapy—curebywater

JECT:throw

conjecture—somethingthrowntogether;aguess

dejected—throwndown

eject—tothrowout

inject—tothrowin

projectile—anobjecttothrowforward

subjected—thrownunder

JUNCT:join

conjunction—awordthatjoinspartsofsentencestogether

enjoin—tojoininto;toenforce

injunction—anenjoiningaction

junction—ajoiningoftwopartsofsomething

JURIS:law,justice

jurisdiction—sphereoflegalauthority

jurisprudence—legalscience

jurist—anexpertinlaw

LOGUE:speaking,speech

dialogue—speechbetweentwopeople

epilogue—ashortendingspeech

eulogy—aspeechofpraise

monologue—oneperson’sspeech

prologue—aspeechgivenasaforeword

LOQ,LOC:talk,speak

colloquy—talkingtogether

elocution—clearspeech

interlocutors—speakers

loquacious—thequalityofbeingtalkative

LUDE,LUS:play

allude—torefertocasuallyorindirectly

allusion—areferenceto

delude—todeceive;tomislead

elude—toescapethroughcleverness

prelude—anintroductoryperiodbeforeamainevent

MAND:order,command

countermand—anorderplacedagainstanotherorder

mandate—command

mandatory—orderedorrequired

remand—toorderback

MANU:hand

manicure—careofthehands

manifest—asfromanopenhand;madeobvious

manipulate—tohandle

manual—ahandbook;donebyhandandnotbymachine

manuscript—adocumentwrittenbyhand

MEM,MENT:mind,memory

commemorate—torememberwith

demented—outofone’smind

mention—acalltomind

reminisce—tocallbackmemories

MIS,MISS,MIT:send

commit—tosendtogether

dismiss—tosendaway

emit—tosendout

remission—thestateofbeingsentback

submit—tosendunder

transmit—tosendacross

PED,POD:foot

biped—ananimalthatwalksontwofeet

impede—toputafootagainst;toobstruct

pedestrian—apersonwhoiswalking

podiatrist—afootdoctor

tripod—anobjectwiththreefeet

PEL,PULSE:drive,push

dispel—todriveaway

expel—topushout

impulse—todriveorpushin

propel—topushforward

repel—topushback

PEND,PENSE:hang,weigh

appendix—partthathangs;aportionofabookusuallyfoundintheback

dispense—toapportion

expenditure—asumpaidout

impending—hangingoversomething

suspend—tohangfrom

PLAC:please

implacable—notabletobepleasedorappeased

placate—toplease

placebo—aninactivemedicinedesignedtopleaseorsatisfy

placid—quiet,pleasing

PLI,PLIC:fold

implicate—toinvolve

pliable—easilyfoldedorinfluenced

PONE,POSE,POSIT:placeorput

components—unitstobeputtogether

depose—toputdown

exponent—somethingorsomeonewhoputssomethingforth

oppose—toplaceagainst

proponent—apersonwhoputssomethingforward;anadvocate

PORT:carry,bring

deportation—theactofcarryingoutorawayfrom

export—tocarrytradeoutofacountry

import—tocarrytradeintoacountry

report—tocarryback

transport—tocarryacross

RECT,REG:toruleorlead,straight

erect—straightenedupward

rectify—tostraightenout;tocorrect

rectitude—moraluprightness

regulate—torule

RUPT:break

disrupt—tobreakthroughordown

erupt—tobreakout

rupture—abreak

SCEND,SCENT:climb

ascend—toclimbup

condescend—toclimbdownwith

descent—downwardslope

transcend—toclimbbeyond

SCRIBE,SCRIPT:write

describe—towriteabout

inscription—somethingwritteninsomethingelse

prescription—writtenbeforereceiving

proscribe—towritesomethingout;toban

SEC,SECT:cut

bisect—tocutintwo

dissect—tocutapart

sector—acuttingorpartofawhole

SERT:declare,state

assert—tostatefirmly

insert—toplacewithin

SERVE:keep,save

conserve—tosave(together)

preservation—thestateofmaintainingasbefore

reservation—somethingkeptorsavedaside

SIST:stand,set

desist—tostandaway

inconsistent—notstandingtogether;changing

insistent—havingthequalityofstandingfirmly

SPEC,SPECT:see,look

aspect—awayoflookingatsomething

prospective—forwardlooking;future

spectrum—somethingseenbroadly

speculate—tolookatmentally

TAIN,TEN:hold

contain—toholdtogether

containment—thestateofbeingheldtogether

detain—toholdaside

retain—toholdback

tenacious—holdingpowerfully

TERM:end,limit

exterminate—toeliminate

interminable—notabletobeended;unending

terminate—toend

terminus—endingplace

TORT:twist

contort—totwisttogether

distortion—somethingthattwistsawayfromthetruth

extort—totwistawayoroutof

TRACT:draw,drag

attract—todrawtoward

distraction—somethingthatdrawsattentionawayfromsomethingelse

extract—todrawout

intractable—noteasilydrawnorpersuaded

protracted—drawnout

VENE,VENT:come

advent—acoming

circumvent—toavoidbygoingaround

convene—tocometogether

intervene—tocomebetween

VINCE,VICT:conquer,overcome

convince—toconquer

invincible—notabletobeconquered

victor—conqueror

VOLVE,VOLU:roll,turn

convoluted—rolledortwistedtogether

evolution—theactofrollingforth;agradualdevelopment

involve—todrawin

revolution—arollingback;anoverturning

EXERCISES:WORDARITHMETIC15Questions

Directions:Togetyourselfstarted,takealookattheseexamplesofwordanalysisandarithmetic.

PROCESSION

pro-isaprefixthatmeans“forward.”

cessisastemthatmeans“go”or“move.”

-ionisanounsuffixthatmeans“theactof.”

pro+cess+ion=theactofmovingforward

RECEDING

re-isaprefixthatmeans“back.”

cedeisastemthatmeans“go.”

-ingisanactivewordsuffix.

re+ced+ing=goingback

DISSECTED

dis-isaprefixthatmeans“apart.”

sectisastemmeaning“cut.”

-edisaverbsuffixshowingpastaction.

dis+sect+ed=cutapartortookapart

Directions:Practicewordarithmeticwiththesewords.TheListofCommonWordPartsandtheWordListwillhelpyou.Writeadefinitionforeachwordinthespace.Thecorrectanswersfollow.

1.revision________________

2.audible________________

3.adhere________________

4.retract________________

5.projection________________

6.preclude________________

7.recline________________

8.erupt________________

9.regression________________

10.revolution________________

11.retain________________

12.inscription________________

13.divert________________

14.conduct________________

15.import________________

ANSWEREXPLANATIONS1.RE-back,again;VISlook,see;-IONtheactofRE+VIS+ION=theactoflookingatorseeingagain

2.AUDhear;-IBLEableAUD+IBLE=abletobeheard

3.AD-to;HEREcling,stickAD+HERE=clingto

4.RE-back,again;TRACTdraw,pullRE+TRACT=todrawback

5.PRO-forward;JECTthrow;-IONtheactofPRO+JECT+ION=theactofthrowingforward

6.PRE-before;CLUDEclose,closeinPRE+CLUDE=tocloseinbefore

7.RE-back,again;CLINEleanRE+CLINE=toleanback

8.E-out;RUPTbreakE+RUPT=tobreakout

9.RE-back,again;GRESSmove;-IONtheactofRE+GRESS+ION=theactofmovingbackward

10.RE-back,again;VOLUroll;-TIONtheactofRE+VOLU+TION=theactofrollingagain

11.RE-back,again;TAINtoholdRE+TAIN=toholdback

12.IN-in;SCRIPTwrite;-IONtheactofIN+SCRIPT+ION=theactofwritingin

13.DI-away,aside;VERTturnDI+VERT=toturnaway

14.CON-together;DUCTlead

CON+DUCT=toleadtogether

15.IM-in;PORTtocarryIM+PORT=tocarryin

SUMMINGITUP

•Knowingwhatthepartsofwordsmeanisthekeytodecipheringwordsyou’veneverseenbefore.

•Practicewordarithmetic.Wordsarethesumoftheirparts.•Ifyoucomeupagainstanunfamiliarwordonyourtest,decipheritspartsandaddthemuptogetthemeaning.

PARTIVVERBALABILITYREVIEW

CHAPTER6:Synonyms(SSATandISEE)

CHAPTER7:VerbalAnalogies(SSATOnly)

CHAPTER8:SentenceCompletions(ISEEOnly)

Chapter6

Synonyms(SSATandISEE)

OVERVIEW

•Whatdosynonymquestionslooklike?•Howdoyouanswersynonymquestions?•Testyourselfquizzes•Answerkeys•Summingitup

WHATDOSYNONYMQUESTIONSLOOKLIKE?Synonymsarewordswithsimilarmeanings.BoththeSSATandISEEtestyouwithstraightforwardsynonymquestions.Theypresentasinglewordincapitallettersandaskyoutochoosethewordthatisthebestsynonymforthequestionword.

Directions:ChoosethewordorphraseclosestinmeaningtotheCAPITALIZEDword.

PROFICIENT

(A)resentful

(B)amiable

(C)famous

(D)adept

Thecorrectansweris(D).Someonewhoisproficientisparticularlygoodatdoingacertaintaskoractivity.Adeptisasynonymforproficient.

ALERT!Neverchooseananswersimplybecauseitistheonewordyoudon’trecognize.Thecorrectanswermaybetheeasiestandmostobviousword.

HOWDOYOUANSWERSYNONYMQUESTIONS?Toanswersynonymquestions,followthesefoursteps:Synonyms:GettingItRight

Lookatthewordcarefully.Considereveryanswerchoice.Onechoicemustalwaysbethebestresponse.

Eliminateobviouslywrongresponsesimmediately.

Useword-analysistechniquestohelpyouwithdifficultwords.

Tryusingthewordinasentenceofyourown;thinkaboutthemeaningofthewordasyouhaveusedit.

NOTEWhatifnoanswerseemsexactlyright?Remember:Thedirectionstellyoutochoosethebestanswer.Thecorrectanswerwon’tnecessarilybeaperfectfit,butitwillworkbetterthantheotherchoices.

TESTYOURSELFQUIZZESUsethefollowingquizzestohelpyoudeterminewhatyourweaknessesmightbe.

Markeverywordyoucannotdefine—capitalizedwordoranswerchoice.Lookupthesewordsinadictionaryandcreateyourownpersonalvocabularylistforfurtherstudy.Clickherefortheanswers.

TestYourself1

Directions:SelectthewordthatisclosestinmeaningtotheCAPITALIZEDword.

1.CHASSIS

(A)frame

(B)body

(C)form

(D)lubrication

2.ENLIGHTEN

(A)reduce

(B)bleach

(C)educate

(D)absorb

3.AFFIRM

(A)prove

(B)validate

(C)sign

(D)stick

4.FROCK

(A)dress

(B)coat

(C)hermit

(D)veil

5.KEG

(A)beer

(B)nails

(C)barrel

(D)understanding

6.LOGO

(A)symbol

(B)copyright

(C)gameplan

(D)magnet

7.MOUSY

(A)brown

(B)alcoholic

(C)gnawing

(D)timid

8.PARCH

(A)boil

(B)bum

(C)dry

(D)steam

9.REFUGE

(A)alibi

(B)shelter

(C)exile

(D)church

10.SKEPTIC

(A)doubter

(B)critic

(C)heretic

(D)opponent

11.TENUOUS

(A)boring

(B)impermanent

(C)nervous

(D)flimsy

12.VIE

(A)defeat

(B)hurry

(C)seek

(D)compete

13.ARMADA

(A)fleet

(B)battle

(C)defeat

(D)ship

14.DRIVEL

(A)saliva

(B)foam

(C)tension

(D)nonsense

15.BOOTY

(A)piracy

(B)riot

(C)plunder

(D)leather

16.PORTAL

(A)dockside

(B)carriage

(C)peephole

(D)gate

17.CAGEY

(A)imprisoned

(B)protected

(C)shrewd

(D)wild

18.JUMBO

(A)egg

(B)elephant

(C)huge

(D)mixture

19.MYTHICAL

(A)ancient

(B)religious

(C)explanatory

(D)imaginary

20.NUGGET

(A)gold

(B)candy

(C)collision

(D)lump

TestYourself2

Directions:SelectthewordthatisclosestinmeaningtotheCAPITALIZEDword.

1.ADULTERATE

(A)cheat

(B)age

(C)shorten

(D)idolize

(E)dilute

2.BOARDER

(A)carpenter

(B)lumberman

(C)edge

(D)roomer

(E)traveler

3.LEERY

(A)obscene

(B)uncontrolled

(C)wicked

(D)suspicious

(E)sheltered

4.OVATION

(A)speech

(B)applause

(C)eggdish

(D)misjudgment

(E)exaggeration

5.CORPULENT

(A)ruddy

(B)spiritual

(C)bloody

(D)overweight

(E)gluttonous

6.DIVERT

(A)behave

(B)amuse

(C)annoy

(D)arrange

(E)disclose

7.SUPERFICIAL

(A)fantastic

(B)familyties

(C)realestate

(D)well

(E)withoutdepth

8.COMBATIVE

(A)honesty

(B)posture

(C)constructive

(D)correction

(E)argumentative

9.TRUCE

(A)treaty

(B)peace

(C)pause

(D)amnesty

(E)silence

10.INTRIGUE

(A)plot

(B)jolly

(C)dishonest

(D)greedy

(E)strict

11.SPECTRUM

(A)rainbow

(B)prism

(C)range

(D)magnifier

(E)idea

12.PERILOUS

(A)dangerous

(B)waterproof

(C)frightening

(D)poor

(E)perfect

13.INSENSITIVE

(A)brief

(B)lacy

(C)uncaring

(D)servile

(E)sad

14.BENEFICIAL

(A)badluck

(B)helpful

(C)blessing

(D)thanks

(E)enlightenment

TestYourself3

Directions:SelectthewordthatisclosestinmeaningtotheCAPITALIZEDword.

1.HUMUS

(A)nerve

(B)topsoil

(C)modesty

(D)tonnage

2.INVALUABLE

(A)useless

(B)untrue

(C)uniform

(D)priceless

3.BELLIGERENT

(A)warlike

(B)windy

(C)noisy

(D)overweight

4.ACRID

(A)burnt

(B)smoky

(C)bitter

(D)artificial

5.RIFE

(A)widespread

(B)mature

(C)quarrelsome

(D)broken

6.TACIT

(A)understood

(B)sensitive

(C)sticky

(D)skillful

7.SHROUD

(A)cummerbund

(B)coffin

(C)veil

(D)wake

8.WRIT

(A)law

(B)order

(C)deed

(D)prohibition

9.DOWDY

(A)young

(B)fluffy

(C)widowed

(D)shabby

10.MAUDLIN

(A)spotted

(B)sentimental

(C)silent

(D)juicy

11.ULCER

(A)pain

(B)stomachache

(C)sore

(D)swelling

12.GREGARIOUS

(A)haggling

(B)sociable

(C)quick

(D)worm-like

13.SURLY

(A)positively

(B)confidently

(C)unfriendly

(D)overly

14.TOKEN

(A)symbol

(B)coin

(C)omen

(D)facsimile

15.ASYLUM

(A)madness

(B)illness

(C)prison

(D)sanctuary

16.PRUNE

(A)raisin

(B)fruit

(C)trim

(D)wrinkle

TestYourself4

Directions:SelectthewordthatisclosestinmeaningtotheCAPITALIZEDword.

1.AMORPHOUS

(A)insomniac

(B)drug-free

(C)shapeless

(D)headless

(E)unloved

2.PALTRY

(A)silo

(B)larder

(C)fowl

(D)foul

(E)meager

3.QUANDARY

(A)swamp

(B)prey

(C)argument

(D)nausea

(E)puzzlement

4.STERLING

(A)genuine

(B)plated

(C)excellent

(D)shiny

(E)heavy

5.TIC

(A)click

(B)twitch

(C)checkmark

(D)insect

(E)game

6.VALET

(A)manservant

(B)bootblack

(C)chauffeur

(D)doorman

(E)rascal

7.BRUNT

(A)wideend

(B)heavypart

(C)sorespot

(D)harshsound

(E)weakestmember

8.LIMPID

(A)deep

(B)clear

(C)weak

(D)blue

(E)lame

9.NARRATE

(A)write

(B)dramatize

(C)tell

(D)summarize

(E)explain

10.TRIATHLON

(A)drunkenbrawl

(B)dance

(C)sportingevent

(D)wildparty

(E)longspeech

11.MINCE

(A)chop

(B)meat

(C)fruit

(D)suet

(E)pie

12.CORRUPT

(A)dishonest

(B)belittle

(C)remove

(D)portray

(E)destroy

13.GARBLE

(A)build

(B)overeat

(C)dazzle

(D)drool

(E)confuse

14.EBB

(A)flow

(B)wax

(C)tide

(D)wane

(E)ocean

TestYourself5

Directions:SelectthewordthatisclosestinmeaningtotheCAPITALIZEDword.

1.GOAD

(A)frog

(B)tadpole

(C)aspiration

(D)prod

2.EQUABLE

(A)hot

(B)fair

(C)measured

(D)calm

3.GUILELESS

(A)clumsy

(B)clever

(C)wistful

(D)naïve

4.EXORBITANT

(A)essential

(B)lacking

(C)literal

(D)excessive

5.DESPERADO

(A)cowboy

(B)hunter

(C)settler

(D)criminal

6.FATIGUE

(A)femininity

(B)length

(C)weariness

(D)perseverance

7.DISPENSARY

(A)militarystore

(B)infirmary

(C)confessional

(D)busstation

8.JOGGLE

(A)tinkle

(B)shake

(C)shove

(D)race

9.KEEPER

(A)miser

(B)caretaker

(C)box

(D)jeweler

10.VOUCH

(A)pay

(B)guarantee

(C)agree

(D)comfort

11.ACCRUE

(A)blame

(B)accumulate

(C)authorize

(D)praise

12.ROUSE

(A)complain

(B)awake

(C)dig

(D)annoy

13.PROLETARIAT

(A)workers

(B)voters

(C)royalty

(D)judiciary

14.LIMBER

(A)nowhere

(B)tied

(C)loose

(D)happy

15.HIATUS

(A)mountain

(B)cymbals

(C)rumpus

(D)gap

16.UNBLUSHING

(A)pale

(B)shameless

(C)stoic

(D)bold

TestYourself6

Directions:SelectthewordthatisclosestinmeaningtotheCAPITALIZEDword.

1.DELECTABLE

(A)hindrance

(B)enjoyable

(C)explanation

(D)debate

2.GAINSAY

(A)repeat

(B)somersault

(C)deny

(D)enjoy

3.FOREGONE

(A)previous

(B)later

(C)ended

(D)preordained

4.OTTOMAN

(A)footstool

(B)rug

(C)sofa

(D)blanket

5.GRATUITOUS

(A)thankful

(B)polite

(C)unnecessary

(D)annoying

6.OPULENT

(A)busy

(B)showy

(C)abundant

(D)enchanted

7.INTERMENT

(A)apprenticeship

(B)questioning

(C)referral

(D)burial

8.SUNDER

(A)separate

(B)vary

(C)darken

(D)depress

9.QUADRANT

(A)perpendicular

(B)rightangle

(C)quarter

(D)corner

10.PORTENTOUS

(A)overweight

(B)ominous

(C)overbearing

(D)ostentatious

11.VAULT

(A)boast

(B)display

(C)jump

(D)defy

12.PONDER

(A)brag

(B)consider

(C)beat

(D)mimic

13.ETIQUETTE

(A)manners

(B)dresscode

(C)wedding

(D)aristocracy

14.GOUGE

(A)measure

(B)scoop

(C)stuff

(D)stab

15.CUPIDITY

(A)greed

(B)love

(C)archery

(D)boldness

16.APOLOGIST

(A)defender

(B)petitioner

(C)prisoner

(D)repenter

TestYourself7

Directions:SelectthewordthatisclosestinmeaningtotheCAPITALIZEDword.

1.WHEEDLE

(A)breatheloudly

(B)givebirth

(C)coax

(D)insinuate

(E)squirm

2.UNLETTERED

(A)pantomimed

(B)illustrated

(C)manuscript

(D)numbered

(E)illiterate

3.SOJOURN

(A)trip

(B)convent

(C)pilgrimage

(D)worry

(E)visit

4.PENSIVE

(A)sorrowful

(B)hanging

(C)thoughtful

(D)poor

(E)stingy

5.RADIATE

(A)heat

(B)expand

(C)illumine

(D)shine

(E)energize

6.TRACERY

(A)searching

(B)design

(C)copy

(D)track

(E)recording

7.AGOG

(A)frightened

(B)surprised

(C)open

(D)angry

(E)upset

8.HIGHWAYMAN

(A)truckdriver

(B)statetrooper

(C)mountaindweller

(D)hermit

(E)robber

9.BIBLIOGRAPHY

(A)storyofone’sownlife

(B)storyofanother’slife

(C)listofbooks

(D)footnote

(E)cardcatalog

10.JOLLITY

(A)piracy

(B)peace

(C)dessert

(D)merriment

(E)teasing

11.IGNOBLE

(A)stupid

(B)base

(C)drunken

(D)regal

(E)erect

12.MUNIFICENT

(A)lavish

(B)urban

(C)splendid

(D)enormous

(E)important

13.ODIOUS

(A)impossible

(B)perfumed

(C)badsmelling

(D)unpleasant

(E)strange

14.CRONE

(A)complainer

(B)bee

(C)hag

(D)singer

(E)friend

TestYourself8

Directions:SelectthewordthatisclosestinmeaningtotheCAPITALIZEDword.

1.CITADEL

(A)castle

(B)barricade

(C)mansion

(D)fort

(E)church

2.INCORRIGIBLE

(A)incredible

(B)immaterial

(C)unruly

(D)selective

(E)immovable

3.FURL

(A)wave

(B)flap

(C)rollup

(D)sail

(E)billow

4.MORTIFY

(A)change

(B)embed

(C)fasten

(D)embarrass

(E)piecetogether

5.FACILE

(A)lithe

(B)hairy

(C)copy

(D)partisan

(E)easy

6.NOSEGAY

(A)lunchbag

(B)goldring

(C)bouquet

(D)wreath

(E)ribbon

7.GURNEY

(A)cow

(B)bubble

(C)bodybag

(D)waterspout

(E)stretcher

8.ETCH

(A)print

(B)engrave

(C)paint

(D)capture

(E)frame

9.HAMLET

(A)smallpig

(B)glove

(C)basket

(D)village

(E)cabin

10.INSOLVENT

(A)solid

(B)liquid

(C)bankrupt

(D)suspended

(E)mixture

11.KNACK

(A)junk

(B)ability

(C)knowledge

(D)sausage

(E)noise

12.AVARICE

(A)gluttony

(B)starvation

(C)intelligence

(D)greed

(E)wealth

13.SUCCUMB

(A)yield

(B)irritate

(C)echo

(D)succeed

(E)refuse

14.PEER

(A)equal

(B)juror

(C)legislator

(D)judge

(E)neighbor

TestYourself9

Directions:SelectthewordthatisclosestinmeaningtotheCAPITALIZEDword.

1.CISTERN

(A)watertank

(B)sewagesystem

(C)stewpot

(D)drainagepipe

2.FORSWEAR

(A)curse

(B)repent

(C)empty

(D)giveup

3.DOUSE

(A)soak

(B)divine

(C)conjure

(D)depress

4.LITIGANT

(A)reader

(B)cleric

(C)wrapper

(D)suer

5.ORDINANCE

(A)ammunition

(B)rule

(C)simplification

(D)suffering

6.MARQUEE

(A)nobleman

(B)billboard

(C)canopy

(D)gemstone

7.IMPASSIVE

(A)active

(B)obstructive

(C)unfeeling

(D)fair

8.NEOPHYTE

(A)novice

(B)complainer

(C)baby

(D)woodnymph

9.HENCHMAN

(A)bartender

(B)gardener

(C)criminal

(D)follower

10.JUBILEE

(A)meeting

(B)year

(C)anniversary

(D)service

11.BAILIFF

(A)sailor

(B)moneylender

(C)dog

(D)courtofficer

12.TREMENDOUS

(A)frightening

(B)enormous

(C)shaking

(D)noisy

13.ROOT

(A)hurry

(B)hibernate

(C)dig

(D)perch

14.RAZE

(A)bum

(B)destroy

(C)loot

(D)obscure

15.REVULSION

(A)disgust

(B)insurgency

(C)twisting

(D)correction

16.GYRATE

(A)barbecue

(B)twist

(C)wobble

(D)spin

TestYourself10

Directions:SelectthewordthatisclosestinmeaningtotheCAPITALIZEDword.

1.WREAK

(A)smash

(B)rage

(C)poke

(D)emanate

(E)inflict

2.PHOBIA

(A)attraction

(B)love

(C)fear

(D)hatred

(E)illness

3.STATIC

(A)lively

(B)electrical

(C)unpleasant

(D)shocking

(E)inactive

4.AMULET

(A)omen

(B)necklace

(C)charm

(D)armband

(E)flask

5.HAZY

(A)polluted

(B)indistinct

(C)brown

(D)annoyed

(E)insane

6.KILN

(A)skirt

(B)plaid

(C)oven

(D)relative

(E)weight

7.MIGRATE

(A)gosouth

(B)sleep

(C)headache

(D)travel

(E)return

8.OFFICIOUS

(A)insulting

(B)formal

(C)meddling

(D)distant

(E)ceremonial

9.NIB

(A)irritation

(B)bite

(C)bump

(D)chill

(E)point

10.GAINFUL

(A)profitable

(B)overflowing

(C)enthusiastic

(D)injurious

(E)busy

11.FILAMENT

(A)steak

(B)thread

(C)flavoring

(D)decoration

(E)failure

12.DELL

(A)farm

(B)barnyard

(C)valley

(D)bell

(E)ring

13.COBBLE

(A)limp

(B)weave

(C)eatgreedily

(D)repairshoes

(E)shoehorses

14.UNNUMBERED

(A)unclassified

(B)countless

(C)few

(D)lettered

(E)listed

15.IMPISH

(A)disrespectful

(B)mischievous

(C)rash

(D)unsaid

(E)relentless

16.BICEPS

(A)muscle

(B)tweezers

(C)dinosaur

(D)rowboat

(E)crossroads

17.THERAPEUTIC

(A)manipulative

(B)active

(C)athletic

(D)rigorous

(E)curative

18.STAID

(A)leftover

(B)guest

(C)sedate

(D)dirty

(E)immobile

19.PURIST

(A)perfectionist

(B)sanitarian

(C)laundress

(D)exorcist

(E)steward

20.FETISH

(A)charm

(B)foot

(C)dustball

(D)gremlin

(E)hairdo

ANSWERKEYSTestYourself1

1.A2.C3.B4.A

5.C6.A7.D8.C

9.B10.A11.D12.D

13.A14.D15.C16.D

17.C18.C19.D20.D

TestYourself2

1.E2.D3.D

4.B5.D6.B

7.E8.E9.C

10.A11.C12.A

13.C14.B

TestYourself3

1.B2.D3.A4.C

5.A6.A7.C

8.B9.D10.B

11.C12.B13.C

14.A15.D16.C

TestYourself4

1.C2.E3.E

4.C5.B6.A

7.B8.B9.C

10.C11.A12.A

13.E14.D

TestYourself5

1.D2.B3.D4.D

5.D6.C7.B

8.B9.B10.B

11.B12.B13.A

14.C15.D16.B

TestYourself6

1.B2.C3.D4.A

5.C6.C7.D

8.A9.C10.B

11.C12.B13.A

14.B15.A16.A

TestYourself7

1.C2.E3.E

4.C5.D6.B

7.B8.E9.C

10.D11.B12.A

13.D14.C

TestYourself8

1.D2.C3.C

4.D5.E6.C

7.E8.B9.D

10.C11.B12.D

13.A14.A

TestYourself9

1.A2.D3.A4.D

5.B6.C7.C

8.A9.D10.C

11.D12.B13.C

14.B15.A16.D

TestYourself10

1.E2.C3.E4.C

5.B6.C7.D8.C

9.E10.A11.B12.C

13.D14.B15.B16.A

17.E18.C19.A20.A

EXERCISES:SYNONYMS

Directions:Choosetheletterofyouranswertoeachquestion.

1.IMAGE

(A)newspaper

(B)picture

(C)fantasy

(D)oldest

2.GARRULOUS

(A)complaining

(B)overlyfriendly

(C)careless

(D)overlytalkative

3.STIMULATE

(A)reward

(B)giveanincentiveto

(C)antagonize

(D)lowertheefficiencyof

4.INSTILL

(A)measureexactly

(B)predictaccurately

(C)impartgradually

(D)restraineffectively

5.IRRELEVANT

(A)unproven

(B)hardtounderstand

(C)notpertinent

(D)insincere

6.PRIOR

(A)private

(B)definite

(C)later

(D)previous

7.DEPLETE

(A)exhaust

(B)include

(C)deliver

(D)reject

8.CANDID

(A)biased

(B)written

(C)frank

(D)confidential

9.AILMENT

(A)illness

(B)foodallergy

(C)operation

(D)problem

10.NONCHALANCE

(A)interest

(B)poverty

(C)care

(D)indifference

11.FUNDAMENTAL

(A)adequate

(B)detailed

(C)basic

(D)truthful

12.TERMINATE

(A)end

(B)ignore

(C)postpone

(D)continue

13.RELUCTANCE

(A)eagerness

(B)ability

(C)unreliability

(D)unwillingness

14.DILIGENT

(A)incompetent

(B)careless

(C)cheerful

(D)industrious

15.DIVERSITY

(A)similarity

(B)triviality

(C)value

(D)variety

16.INTACT

(A)undamaged

(B)unattended

(C)atotalloss

(D)repaired

17.RESOLVED

(A)offered

(B)refused

(C)hesitated

(D)determined

18.RIGOROUSLY

(A)usually

(B)never

(C)strictly

(D)leniently

19.AMICABLE

(A)friendly

(B)tender

(C)accessible

(D)inimical

20.CLAMOR

(A)murmur

(B)noise

(C)questions

(D)singing

21.DECLINED

(A)suspected

(B)misunderstood

(C)consented

(D)refused

22.NOXIOUS

(A)concentrated

(B)harmful

(C)gaseous

(D)heavy

23.TRIVIAL

(A)ofapersonalnature

(B)verysignificant

(C)interestingandeducational

(D)oflittleimportance

24.OBSOLETE

(A)complicated

(B)outofdate

(C)highlysuitable

(D)reliable

25.DEXTERITY

(A)skill

(B)punctuality

(C)courtesy

(D)cooperation

26.RECALCITRANT

(A)cooperative

(B)highlyexcited

(C)accustomedtohardwork

(D)stubbornlyresistant

27.PLACATE

(A)escort

(B)appease

(C)interview

(D)detain

28.LATITUDE

(A)adviceandencouragement

(B)assistance

(C)causeforannoyance

(D)freedomfromrestriction

29.EXPEDIENT

(A)inconvenient

(B)expensive

(C)efficient

(D)time-consuming

30.PREROGATIVES

(A)ideals

(B)privileges

(C)demands

(D)weapons

31.ANTICIPATE

(A)foresee

(B)annul

(C)approve

(D)conceal

32.PROLIFIC

(A)talented

(B)popular

(C)forwardlooking

(D)productive

33.FRUGAL

(A)friendly

(B)hostile

(C)thoughtful

(D)economical

34.IMPERATIVE

(A)impending

(B)impossible

(C)compulsory

(D)flawless

35.ACCESS

(A)toomuch

(B)admittance

(C)extra

(D)arrival

36.SUBSEQUENT

(A)preceding

(B)early

(C)following

(D)winning

37.HERITAGE

(A)will

(B)believer

(C)legend

(D)inheritance

38.CULTURED

(A)malformed

(B)decomposed

(C)exiled

(D)cultivated

39.ATONE

(A)repent

(B)rebel

(C)sound

(D)impotent

40.PREDATORY

(A)introductory

(B)intellectual

(C)preaching

(D)carnivorous

41.MAIL

(A)armor

(B)seaside

(C)rapidtravel

(D)wool

42.FLORID

(A)seedy

(B)ruddy

(C)hot

(D)overflowing

43.FEASIBLE

(A)simple

(B)practical

(C)visible

(D)lenient

44.SUPPLANT

(A)approve

(B)displace

(C)widespread

(D)appease

45.PREVALENT

(A)current

(B)permanent

(C)widespread

(D)temporary

46.CONTEND

(A)assert

(B)agree

(C)temper

(D)appease

47.FLAGRANT

(A)glaring

(B)hopeless

(C)engrossing

(D)motioning

48.ENTHRALL

(A)throwin

(B)captivate

(C)support

(D)deceive

49.DESECRATE

(A)improveupon

(B)occupy

(C)profane

(D)hide

50.OSTRACIZE

(A)delight

(B)exclude

(C)include

(D)hide

51.EXORBITANT

(A)priceless

(B)worthless

(C)extensive

(D)excessive

52.OBLITERATE

(A)annihilate

(B)review

(C)demonstrate

(D)detect

53.AUSTERITY

(A)priority

(B)anxiety

(C)self-discipline

(D)solitude

54.CORROBORATION

(A)expenditure

(B)compilation

(C)confirmation

(D)reduction

55.SALUTARY

(A)popular

(B)beneficial

(C)urgent

(D)forceful

56.ACQUIESCE

(A)endeavor

(B)discharge

(C)agree

(D)inquire

57.DIFFIDENCE

(A)shyness

(B)distinction

(C)interval

(D)discordance

58.REPRISAL

(A)retaliation

(B)advantage

(C)warning

(D)denial

59.CAPITULATE

(A)repeat

(B)surrender

(C)finance

(D)retreat

60.REPUTABLE

(A)star

(B)capable

(C)significant

(D)honest

ANSWERSANDEXPLANATIONS

1.Thecorrectansweris(B).Hercollageconsistedofmanyimagesfromherchildhoodandbeyond.

2.Thecorrectansweris(D).MissPaynewasagarrulousoldgossip.

3.Thecorrectansweris(B).Thechangeinprocedurestimulatedthementoworkharder.

4.Thecorrectansweris(C).Courageisdifficulttoinstillinaperson.

5.Thecorrectansweris(C).Hisreportscontainedmanyirrelevantstatements.

6.Thecorrectansweris(D).Hehadapriorappointmentwiththemanager.

7.Thecorrectansweris(A).Thesupplyofpamphletshasbeendepleted.

8.Thecorrectansweris(C).Mr.Dormanaskedforacandidopinion.

9.Thecorrectansweris(A).Thepatientwasbeingtreatedforaseriousailment.

10.Thecorrectansweris(D).Hisnonchalancewasdisturbingtothecourtroomobservers.

11.Thecorrectansweris(C).Ourargumentwasbasedonfundamentaleconomicprinciples.

12.Thecorrectansweris(A).Hewishestoterminatehismembership.

13.Thecorrectansweris(D).MissFultonshowedherreluctancetoserveasareliefoperator.

14.Thecorrectansweris(D).Hisassistantwasadiligentworker.

15.Thecorrectansweris(D).Thereisconsiderablediversityinthesubmittedsuggestions.

16.Thecorrectansweris(A).Thevehiclewasleftintactaftertheaccident.

17.Thecorrectansweris(D).Heresolvedtoactatonce.

18.Thecorrectansweris(C).Thedepartmentalruleswererigorouslyenforced.

19.Thecorrectansweris(A).RelationsbetweenEnglandandtheUnitedStatesareamicable.

20.Thecorrectansweris(B).Icouldplainlyheartheclamorofthecrowd.

21.Thecorrectansweris(D).Hedeclinedourofferstohelphim.

22.Thecorrectansweris(B).Itwasreportedthatnoxiousfumeswereescapingfromthetanks.

23.Thecorrectansweris(D).Theyarediscussingtrivialmatters.

24.Thecorrectansweris(B).Thisequipmentisobsolete.

25.Thecorrectansweris(A).Theoperatorwascommendedforherdexterity.

26.Thecorrectansweris(D).Thewitnesswasrecalcitrant.

27.Thecorrectansweris(B).Hewasaskedtoplacatethevisitor.

28.Thecorrectansweris(D).Hewasgivenconsiderablelatitudeindesigningtheprogram.

29.Thecorrectansweris(C).Thisisthemostexpedientmethodforachievingthedesiredresults.

30.Thecorrectansweris(B).Themenrefusedtogiveuptheirprerogativeswithoutastruggle.

31.Thecorrectansweris(A).Anticipateyouropponent’sargumentsandprepareyourresponses.

32.Thecorrectansweris(D).Aprolifictreebearsmuchfruit.

33.Thecorrectansweris(D).Shehassavedagreatdealofmoneybecauseshelivesfrugally.

34.Thecorrectansweris(C).Itisimperativethatyouseeadoctorbeforetherashspreads.

35.Thecorrectansweris(B).Jimistheonlypersonwhohasaccesstothesafe.

36.Thecorrectansweris(C).Insubsequentmeetingswewillbediscussingtheprogressofthisproject.

37.Thecorrectansweris(D).Americanswereleftawonderfulheritagebytheirancestors.

38.Thecorrectansweris(D).Culturedpearlsarelessexpensivethannaturalones.

39.Thecorrectansweris(A).Theprisonerwantedtoatoneforhispastcrimes.

40.Thecorrectansweris(D).Lionsarepredatoryanimals.

41.Thecorrectansweris(A).Swordscouldnotpierceaknight’ssuitofmail.

42.Thecorrectansweris(B).Thesalesmanhadafloridcomplexion.

43.Thecorrectansweris(B).Theengineersthoughtthebridgewouldbeeconomicallyfeasible.

44.Thecorrectansweris(B).Inindustrytoday,newideasareconstantlybeingsupplantedbyevennewerones.

45.Thecorrectansweris(C).Abeliefintheexistenceofwitcheswasprevalentduringtheseventeenthcentury.

46.Thecorrectansweris(A).Thedefenseattorneycontendsthathisclientwasoutoftownwhenthecrimewascommitted.

47.Thecorrectansweris(A).Theactionshowedherflagrantdisregardforschoolrules.

48.Thecorrectansweris(B).Thestorytellerenthralledhisyoungaudience.

49.Thecorrectansweris(C).Vandalsdesecratedtheflagbyburningit.

50.Thecorrectansweris(B).Childrenoftenostracizeclassmateswhoseemdifferentinanyway.

51.Thecorrectansweris(D).Sometenantsarechargedexorbitantrentsbygreedylandlords.

52.Thecorrectansweris(A).Civilizationcouldbeobliteratedbyanatomicwar.

53.Thecorrectansweris(C).Crudeoilshortagesmakeausterityanecessity.

54.Thecorrectansweris(C).Corroborationofthedefendant’salibiwillbedifficulttofind.

55.Thecorrectansweris(B).Adecreaseincontagiousdiseasesshowsthesalutaryeffectsofpreventivemedicine.

56.Thecorrectansweris(C).Thepolicewereforcedtoacquiescetothekidnapper’sdemands.

57.Thecorrectansweris(A).Janet’sdiffidencekeptherfromparticipatinginclassdiscussions.

58.Thecorrectansweris(A).Reprisalswereorganizedagainsttheterrorists.

59.Thecorrectansweris(B).Mr.Jonescapitulatedtohisstudents’demands.

60.Thecorrectansweris(D).Iknewthehistorybookwasareputablesourcebecauseitsauthorisrenownedforaccuratelyreportingevents.

SUMMINGITUP

•Synonymsarewordswithsimilarmeanings,andsynonymquestionsaskyoutochooseawordwithameaningsimilartothatofagivenword.

•Neverchooseananswersimplybecauseitistheonewordyoudon’trecognize.Thecorrectanswermaybetheeasiestandmostobviousword.

•Tryusingthewordinasentenceofyourown;thinkaboutthemeaningofthewordasyouhaveusedit.

Chapter7

VerbalAnalogies(SSATOnly)

OVERVIEW

•Whatmakesaverbalanalogy?•Whatdoverbalanalogyquestionslooklike?•Howdoyousolveverbalanalogies?•Whatdosmarttest-takersknow?•Testyourselfquizzes•Answerkeys•Summingitup

WHATMAKESAVERBALANALOGY?Verbalanalogiesareallaboutrelationships.Theytestyourabilitytoseearelationshipbetweentwowordsandtorecognizeasimilarrelationshipbetweentwootherwords.Verbalanalogytestsmeasurenotonlyyourunderstandingofthewordsthemselvesbutalsoyourmentalflexibilityandabilitytomanipulaterelationships.Thekeytoanalogysuccessisbeingabletoexpresstherelationshipbetweenthewordsinapair—notwhatthewordsmean,buthowtheyarerelated.

WHATDOVERBALANALOGYQUESTIONSLOOKLIKE?Ananalogycanbewritteninseveraldifferentways.Itmaybewrittenasasentence,usingonlywords,orsymbolsmaybesubstitutedfortheconnectingwords.SSATanalogyquestionsuseonlywords.

Winteristosummerascoldisto

(A)wet

(B)future

(C)hot

(D)freezing

(E)spring

Thecorrectansweris(C).WINTERandSUMMERareopposites,orantonyms.TheantonymforCOLDisHOT.

Someanalogiessupplythreeofthefournecessarywords.Youmustfindtherelationshipbetweenthefirsttwowordsandthenchooseawordthatisrelatedtothethirdwordinthesameway.

Spellingistopunctuationasbiologyisto

(A)science

(B)animals

(C)dissection

(D)chemistry

(E)experiment

Thecorrectansweris(D).SPELLINGandPUNCTUATIONaretwosubjectsstudiedinEnglish.BIOLOGYandCHEMISTRYaretwosubjectsstudiedinthefieldofscience.

Otheranalogiesbeginwithapairofwords.Youmustfirstdecidehowthosewordsarerelated.Then,fromalistoffivepairs,youmustchoosethepairthatillustratesthesamerelationship.

Spellingistopunctuationas

(A)pajamasistofatigue

(B)powderistoshaving

(C)bandageistocut

(D)biologyistochemistry

(E)readingistobook

SSATverbalanalogyquestionsappearinbothoftheseformats.

TIPWhatifnoneoftheanswersareexactlyright?Remember:Thedirectionstellyoutochoosethebestanswer.Thecorrectanswerwon’tnecessarilybeaperfectfit,butitwillworkbetterthantheotherchoices.

HOWDOYOUSOLVEVERBALANALOGIES?Tosolveverbalanalogies,followthesefivesteps:FiveStepsForAnalogies:GettingItRight

Figureouthowthefirsttwowordsarerelated.Makeupasentencethatexpressesthatrelationship.Tryoutyoursentenceoneachanswerchoiceandeliminatetheonesthatdon’twork.Ifyou’releftwithmorethanoneanswer—ornoansweratall—gobackandmakeyoursentencefitbetter.Choosethebestanswer.Ifnoneofthechoicesfitexactly,choosetheonethatworksbest.

WHATDOSMARTTEST-TAKERSKNOW?ASentenceCanMaketheConnection

Scribbleistowriteas

(A)informistosupply

(B)mutteristolisten

(C)nuzzleistofeel

(D)rambleistoplay

(E)staggeristowalk

Thecorrectansweris(E).Summarizeeachanalogyrelationshipwithasentence.Inthiscase,scribblingisabadkindofwriting.Usethesamesentencetotestconnectionsbetweenthewordsintheanswerchoices.Whenyoufindonethatworks,you’vefoundyouranswer.

(A)Informingisabadkindofsupplying.(No.)

(B)Mutteringisabadkindoflistening.(No.)

(C)Nuzzlingisabadkindoffeeling.(No.)

(D)Ramblingisabadkindofplaying.(No.)

(E)Staggeringisabadkindofwalking.(Yes!)

TheMorePreciseYourSentence,theBetter

Youcannotexpecttosolveeveryanalogybysimplyplugginginalistofcommonanalogytypes.Rememberthattheanalogiesgetmoredifficultasyouworkyourwaythrougheachgroup.Usethecommoncategoriesasastartingpoint,butbepreparedtorefinetherelationshipbymakingyoursentencemoreprecise.Considerthisexample:

Grainistosiloas

(A)pilotistoplane

(B)judgeistocourtroom

(C)wateristoreservoir

(D)clockistotime

(E)automobileistohighway

Thecorrectansweris(C).

Ifyouapplythe“placewhere”ideawithoutthinking,hereiswhathappens:

Asiloisaplacewhereyouwouldfindgrain.

(A)Aplaneisaplacewhereyouwouldfindapilot.

(B)Acourtroomisaplacewhereyouwouldfindajudge.

(C)Areservoirisaplacewhereyouwouldfindwater.

(D)Aclockisaplacewhereyouwouldfindtime.

(E)Ahighwayisaplacewhereyouwouldfindautomobiles.

Youcaneliminatechoice(D),butthatstillleavesyouwithfourpossibleanswers.Nowisthetimetogobackandmakeyouroriginalsentencefitbetter.Howcanyouexpresstherelationshipbetweensiloandgrainmoreprecisely?

Asiloisaplacewheregrainisstored.

(A)Aplaneisaplacewhereapilotisstored.

(B)Acourtroomisaplacewhereajudgeisstored.

(C)Areservoirisaplacewherewaterisstored.

(E)Ahighwayisaplacewhereautomobilesarestored.

KnowtheMostCommonVerbalAnalogyCategories

Thesamerelationshipsappearoverandoveragaininverbalanalogyquestions.Knowingwhatthecategoriesareandlookingforthemasyoutackleeachproblemwillmakeyourjobeasier.SomeofthemostcommonlyusedcategoriesinSSATanalogyquestionsare:

SynonymRelationships

Synonymsarewordsthathavesimilarmeanings.

Enormousistohugeasmuddyisto

(A)unclear

(B)clean

(C)rocky

(D)roguish

(E)sharp

Thecorrectansweris(A).SomethingthatisdescribedasMUDDYisclouded,orUNCLEAR.

AntonymRelationships

Antonymsarewordsthathaveoppositemeanings.

Goodistoevilas

(A)suaveistoblunt

(B)northistoclimate

(C)hotistotemperature

(D)sorrowistohappiness

(E)angelistodevil

Thecorrectansweris(E).ANGEListheoppositeofDEVIL.

NOTE:Thewordsthathaveapositiveassociation,GOODandANGEL,arethefirstwordsineachpair.Thewordswithanegativeassociation,EVILandDEVIL,arethesecondwordsineachpair.“Sorrowistohappiness”alsorepresentstheantonymrelationship,butitisanincorrectanswerbecausethetermsareinreversedorder.Aproperlycompletedanalogyconsistsoftermswiththesamerelationshipoccurringinthesameorder.

Part-WholeRelationships

Inthistypeofanalogy,oneofthewordsineachpairrepresentsasinglepartofawholeperson,place,thing,oridea.

Snakeistoreptileas

(A)patchistothread

(B)handistoclock

(C)handistofinger

(D)struggleistofight

(E)frogistosnake

Thecorrectansweris(B).ASNAKEispartoftheREPTILEfamily.AHANDispartofaCLOCK.Choice(C)alsoshowsasimilarrelationship,butthewordsaregiveninthewrongorder.

Noun-VerbRelationships

Inthistypeofanalogy,oneofthewordsinapairnamesaperson,place,thing,oridea.Theotherwordrepresentsanactionthatcanbeassociatedwiththatword.

Steakistobroilas

(A)foodistosell

(B)wineistopour

(C)breadistobake

(D)sugaristospill

(E)wateristodrink

Thecorrectansweris(C).OnewaytocookaSTEAKistoBROILit;similarly,weBAKEBREADinordertocookit.Choices(B),(D),and(E)allshownoun-verbrelationships,withthenounsandverbsinthecorrectsequence,butneitherusesaverbthatrelatestocooking.Therefore,thebestanswerwouldbechoice(C)becausebroilingandbakingarebothformsofcookingfood.

Cause-and-EffectRelationships

Twotypesofcause-and-effectrelationshipsmaybeusedinanalogies.Inthefirsttype,onewordinthepairwillsometimesresultinthesecondword.

Raceistofatigueasfastisto

(A)track

(B)rest

(C)run

(D)obesity

(E)hunger

Thecorrectansweris(E).RunningaRACEmaycausetherunnerFATIGUE.FASTINGmaycauseHUNGER.

ALERT!Don’tconfusetheorderofthewords.Therelationshipofthewordsintheanswermustbeinthesameorderastherelationshipofthewordsinthefirstpair.

Inthesecondtypeofcause-and-effectanalogy,onewordinapairmayproducetheother.

Cowistomilkasbeeisto

(A)honey

(B)drone

(C)nest

(D)wasp

(E)sting

Thecorrectansweris(A).ACOWproducesMILK;aBEEproducesHONEY.

PurposeRelationships

Inthistypeofanalogy,oneofthewordsineachpairisusedinataskinvolvingtheotherwordinthepair.

Gloveistoballas

(A)hookistofish

(B)winteristoweather

(C)gameistopennant

(D)stadiumistoseats

(E)ballistobat

Thecorrectansweris(A).AGLOVEisusedinbaseballtocatchaBALL.Whenfishing,aHOOKisusedtocatchaFISH.

AssociationRelationships

Inthistypeofanalogy,onewordinapairiscommonlythoughtofinconnectionwiththesecondword.

Youngistolambas

(A)ramistoewe

(B)oldistomutton

(C)lambistochop

(D)woolistoshear

(E)cowistocalf

Thecorrectansweris(B).ThemeatofYOUNGsheepiscalledLAMB.ThemeatofOLDsheepiscalledMUTTON.

OtherAnalogyCategories

Otheranalogycategorieswithwhichyoushouldalsobecomefamiliarare:

“Typeof”Analogies

SWORDistoWEAPON Aswordisatypeofweapon.GRIMACEistoEXPRESSION Agrimaceisatypeofexpression.OAKistoTREE Anoakisatypeoftree.WATERCOLORistoPAINTING Awatercolorisatypeofpainting.

“Partofthedefinitionof”Analogies

GENEROSITYistoPHILANTHROPIST Generosityispartofthedefinitionofaphilanthropist.BRAVERYistoHERO Braveryispartofthedefinitionofahero.FALSEistoLIE Itispartofthedefinitionofaliethatitisfalse.INVENTIONistoORIGINAL Itispartofthedefinitionofaninventionthatitisoriginal.FACULTYistoTEACH Itispartofthedefinitionofafacultythatitissupposedtoteach.

“Lackofsomethingispartofthedefinition”Analogies

TRAITORistoLOYALTY Lackofloyaltyispartofthedefinitionofatraitor.NEGLIGENTistoCARE Itispartofthedefinitionofbeingnegligentthatsomeonelackscare.ARIDistoMOISTURE Lackofmoistureispartofthedefinitionofanaridregion.IGNORANCEistoKNOWLEDGE Lackofknowledgeispartofthedefinitionofignorance.POVERTYistoFUNDS Partofthedefinitionofpovertyisalackoffunds.

TIPMakethesentenceconnection.Turntheanalogypairsintosentencestohelpyouseetheconnection.Thenfittheanswerpairsintothesamesentenceuntilyoufindtheonethatworksbest.

“Aplacefor”Analogies

WITNESSistoCOURTROOM Acourtroomistheplaceforawitness.ACTORistoSTAGE Astageistheplaceforanactor.GRAINistoSILO Asiloisaplaceforstoringgrain.PILOTistoAIRPLANE Anairplaneisaplacewhereyouwouldfindapilot.OREistoMINE Amineistheplacewhereyouwouldfindore.

“Degree”Analogies

BREEZEistoGALE Agaleismorepowerfulthanabreeze.TRICKLEistoGUSH Togushismoreforcefulthantotrickle.ANNOYistoENRAGE Toenrageisstrongerthantoannoy.MOUNTAINistoHILL Amountainisaverylargehill.WASHistoSCRUB Toscrubisstrongerthantowash.

ALERT!Don’tbefooledbysimilarwords.Inanalogyquestions,you’relookingforsimilarrelationships,notsimilarwords.

WordswithSimilarMeaningsCanFoolYou

Inanalogyquestions,whatcountsistherelationshipbetweenthefirsttwowords.Thewordsinthecorrectanswerchoicemusthaveasimilarrelationship.Thereisnoneedforoneorbothtoberelatedinmeaningtothewords.Considerthisanalogy:

Tangledistoknotas

(A)snarledistorope

(B)crumpledistowrinkle

(C)mussedistohair

(D)emptyistocup

(E)cannedistopreserves

Thecorrectansweris(B).Thisisa“partofthedefinitionof”analogy.Partofthedefinitionofaknotisthatitissomethingtangled.Likewise,partofthe

definitionofawrinkleisthatitissomethingcrumpled.Don’tbemisledbychoice(A).Althoughsnarledissimilarinmeaningtotangled,aropedoesnothavetobesnarled.

SomeAnalogiesWorkBetterWhenYouTurnThemAround

Sometimesthefirsttwowordsfalleasilyintoasentencethatexpressestheirrelationship—andsometimestheydon’t.Ifyou’rehavingtroublemakingupasentencethatrelatesthetwowords,bepreparedtoshiftgears.Tryreversingtheorderoftheoriginalwordpair.Let’sseehowthistechniqueworksonthefollowinganalogy:

Iceistoglacieras

(A)trainistotrestle

(B)sandistodune

(C)pathistoforest

(D)featheristobird

(E)oceanistoship

Thecorrectansweris(B).Onlychoice(B)exhibitsthesamerelationshipastheoriginalpair.

Ifyoucan’tcomeupwithasentencerelatingICEtoGLACIER,tryrelatingGLACIERtoICE:

Aglacierismadeupofice.

Here’stheonlycatch:Ifyoureversetheorderofthewords,youmustalsoreversetheorderofthewordsineachanswerchoice.Sowhenyouapplyyoursentencetotheanswerchoices,thisishowyou’llhavetodoit:

(A)Atrestleismadeupofatrain.

(B)Aduneismadeupofsand.

(C)Aforestismadeupofapath.

(D)Abirdismadeupofafeather.

(E)Ashipismadeupofanocean.

TestYourselfQuizzes

Usethefollowingquizzestohelpyoudeterminewhatyourweaknessesmightbe.Clickherefortheanswers.

TestYourself1

Directions:Lookatthefirsttwowordsanddecidehowtheyarerelatedtoeachother.Thendecidewhichoftheanswerchoicesrelatetothethirdwordinthesamewaythatthefirsttwoarerelated.

1.Mowistolawnaspruneisto

(A)plum

(B)raisin

(C)tree

(D)hair

(E)meadow

2.Antecedentistoprecedentasconsequentisto

(A)decadent

(B)subsequent

(C)ebullient

(D)transient

(E)penitent

3.Bassinetistocribascaristo

(A)bus

(B)airplane

(C)stroller

(D)bed

(E)taxicab

4.Restaurantistoeatingasbarracksisto

(A)cleaning

(B)military

(C)inspection

(D)nutrition

(E)sleeping

5.Flameistofireassmokeisto

(A)heat

(B)ashes

(C)water

(D)fire

(E)match

6.Bludgeonistospearaslatheisto

(A)vise

(B)carpenter

(C)battle

(D)curve

(E)construction

7.Riskistoescapadeasageisto

(A)hilarity

(B)intemperance

(C)luminosity

(D)mayhem

(E)heirloom

8.Assetistoblackasdebitisto

(A)debt

(B)red

(C)blue

(D)left

(E)ledger

9.Darknessistoeclipseastidalwaveisto

(A)tsunami

(B)eruption

(C)ocean

(D)beach

(E)earthquake

10.Henistobroodasmotheristo

(A)family

(B)chickens

(C)children

(D)shoe

(E)mom

TestYourself2

Directions:Decidehowthewordsinthefirstpairarerelated.Chooseapairofwordsbelowthatshowsthesamerelationship.

1.Floodistodroughtas

(A)richistopoor

(B)camelistodesert

(C)drizzleistodownpour

(D)eveningistonight

(E)goldistosilver

2.Voraciousistogluttonousas

(A)hungryistothirsty

(B)warmistohot

(C)potentistostrong

(D)flightistofight

(E)yardistometer

3.Hurricaneistowindas

(A)windistowater

(B)typhoonistowind

(C)tornadoistotwister

(D)tornadoistotyphoon

(E)hurricaneistotyphoon

4.Dungistoelephantas

(A)horseistomanure

(B)fishistofood

(C)wormistosoil

(D)oxygenistotree

(E)aquariumistoterrarium

5.Centipedeistospideras

(A)pentagonistotriangle

(B)rowboatistosailboat

(C)BCistoAD

(D)percussionistostring

(E)duetistotrio

6.Headistohammeras

(A)toothistosaw

(B)nailistoscrew

(C)awlistopunch

(D)screwistodriver

(E)beginningistoend

7.Anthemistoinspireas

(A)aspirinistopain

(B)lightistosee

(C)organistogrind

(D)apeistocopy

(E)shuttleistotransport

8.Hookistoeyeas

(A)sleeveistocoat

(B)buttonistohole

(C)bootistoshoe

(D)adhesiveistotape

(E)honeyistobear

9.Actoristoscriptas

(A)architectistodesign

(B)painteristomural

(C)musicianistoscore

(D)judgeistobrief

(E)studentistotextbook

10.Goalieistonetas

(A)hockeyistosoccer

(B)playeristogame

(C)sentryistofort

(D)batistoball

(E)iceistoturf

11.Topographyistogeographyas

(A)wateristoland

(B)physicsistomathematics

(C)geographyistohistory

(D)mountainistovalley

(E)biologyistoscience

12.Rocketistotorpedoas

(A)fireistowater

(B)airistowater

(C)explosionistohole

(D)upistodown

(E)waristopeace

TestYourself3

Directions:Lookatthefirsttwowordsanddecidehowtheyarerelatedtoeachother.Thendecidewhichoftheanswerchoicesrelatetothethirdwordinthesamewaythatthefirsttwoarerelated.

1.Verticalistohorizontalaserectisto

(A)honest

(B)construct

(C)prone

(D)lumber

(E)proper

2.Oustistooverthrowasmoltisto

(A)melt

(B)shed

(C)shape

(D)weaken

(E)spoil

3.Inlineskatesistomotorcycleasskisisto

(A)snowmobile

(B)bicycle

(C)snow

(D)iceskates

(E)snowplow

4.Scaleistofishashideisto

(A)seek

(B)tan

(C)hole

(D)ride

(E)horse

5.Windistoseedasbeeisto

(A)pollen

(B)honey

(C)hive

(D)bear

(E)flower

6.Gloveistohandashoseisto

(A)garden

(B)water

(C)foot

(D)nozzle

(E)shoe

7.Pigletistopigasisletisto

(A)pond

(B)lace

(C)lake

(D)rivulet

(E)island

8.Depressedistomopeastiredisto

(A)yawn

(B)cope

(C)car

(D)laugh

(E)suppressed

9.Masticateistochewasgesticulateisto

(A)vomit

(B)digest

(C)urbanize

(D)point

(E)offer

10.Treeistoforestassandisto

(A)dune

(B)details

(C)hours

(D)time

(E)sandwich

11.Seemlyistobehaviorasgawkyisto

(A)length

(B)teenager

(C)volume

(D)barnyard

(E)appearance

12.Flitistodartasfoilisto

(A)fence

(B)thwart

(C)aluminum

(D)change

(E)surprise

TestYourself4

Directions:Decidehowthewordsinthefirstpairarerelated.Chooseapairofwordsbelowthatshowsthesamerelationship.

1.Ravenousistohungryas

(A)hungryistothirsty

(B)stingyistothrifty

(C)boilistobake

(D)fullistoempty

(E)smallistolittle

2.Bonanzaistowindfallas

(A)earnistomerit

(B)westernistoeastern

(C)horseistoapple

(D)sunistorain

(E)giftistoaccident

3.Salvageistowreckas

(A)buryistotreasure

(B)paintistoantique

(C)sinkistovessel

(D)retrieveistoproperty

(E)excavateistoruin

4.Dimpleistopimpleas

(A)faceistoback

(B)downistoup

(C)chinistocheek

(D)babyistoadolescent

(E)loveistoloathing

5.Filchistopilferas

(A)stealistosquander

(B)squealistowriggle

(C)pinchistopuff

(D)fidgetistosquirm

(E)robistofence

6.Hindsightistoforesightas

(A)cureistoprevention

(B)thenistonow

(C)lateristonever

(D)predictionistopredilection

(E)visionistoperception

7.Fodderistocattleas

(A)wateristofish

(B)birdistoworm

(C)restaurantistopeople

(D)siloistocorn

(E)fuelistoengine

8.Affixistostampas

(A)letteristopostmark

(B)hammeristonail

(C)runistohorse

(D)glueistomucilage

(E)mailistodeliver

9.Leewardistowindwardas

(A)portistostarboard

(B)meadowistoocean

(C)motoristosail

(D)pirateistoprivateer

(E)tugboatistoliner

10.Punishistoberateas

(A)leafyistogreen

(B)deepistoocean

(C)jumpistoleap

(D)soakistodampen

(E)hikeistotrek

11.Tobaccoistocigaretteas

(A)liquoristodrink

(B)coughistocold

(C)wheatistobread

(D)cowistomilk

(E)smokingistocancer

12.Shunistoembraceas

(A)shrinkistopounce

(B)slenderistosloping

(C)tousleistomuss

(D)showistotell

(E)kissistohug

TestYourself5

Directions:Lookatthefirsttwowordsanddecidehowtheyarerelatedtoeachother.Thendecidewhichoftheanswerchoicesrelatetothethirdwordinthesamewaythatthefirsttwoarerelated.

1.Comprehensiveistoinclusiveasapprehensiveisto

(A)exclusive

(B)misunderstood

(C)caught

(D)uneasy

(E)understanding

2.Chalkistocrayonasbedisto

(A)chair

(B)sleep

(C)bunk

(D)ladder

(E)seat

3.Ceremoniousistoinformalasclericalisto

(A)typographical

(B)numerical

(C)religious

(D)retail

(E)secular

4.Desististoceaseasresististo

(A)oppose

(B)giveup

(C)resolve

(D)remain

(E)presume

5.Putridistogarbageasaromaticisto

(A)smell

(B)pleasant

(C)spirits

(D)spray

(E)spice

6.Harpoonistowhalingasbuffoonisto

(A)sorcery

(B)clowning

(C)badminton

(D)spelunking

(E)ballooning

7.Hornistoblowasharpisto

(A)democracy

(B)play

(C)denounce

(D)pluck

(E)pants

8.Baldistohirsuteasanemicisto

(A)tiny

(B)fat

(C)robust

(D)loud

(E)redundant

9.Greenistoyouthasgrayisto

(A)age

(B)hair

(C)gloom

(D)mare

(E)elderly

10.Gridironistofootballasgridlockisto

(A)waffles

(B)chess

(C)prison

(D)wrestling

(E)traffic

11.Sowistoreapascrawlisto

(A)sneak

(B)harvest

(C)walk

(D)cultivate

(E)wheat

12.Hordeistothrongashurlisto

(A)thrust

(B)jump

(C)throw

(D)defeat

(E)smash

TestYourself6

Directions:Decidehowthewordsinthefirstpairarerelated.Chooseapairofwordsbelowthatshowsthesamerelationship.

1.Salaryistoincomeas

(A)incomeistotax

(B)moneyistoevil

(C)springistowater

(D)wageistowork

(E)dollarsistocents

2.Treeistoclimbas

(A)scaleistomountain

(B)horseistoride

(C)gardenistoeat

(D)gemistodie

(E)catistomouse

3.Consideristodismissas

(A)reflectistoabsorb

(B)wadeistoswim

(C)currentistorecent

(D)helpistoassist

(E)decideistodetermine

4.Compostistofertilizeras

(A)dentististoteeth

(B)ballistobasket

(C)holeistodrill

(D)valueistoprice

(E)iceistorefrigerant

5.Tranceistohypnosisas

(A)knowledgeistostudy

(B)poisonistoivy

(C)vaccinationistoimmunity

(D)bananaistoplantain

(E)pieistoapple

6.Pianististomusicianas

(A)organististopianist

(B)violinististofiddler

(C)musicianistowriter

(D)mathematicsistomathematician

(E)psychiatrististophysician

7.Poetryistoproseas

(A)seeingistohearing

(B)lightistoheavy

(C)sonataistoétude

(D)operaistobook

(E)melodyistorhythm

8.Hullistostrawberryas

(A)treeistoleaf

(B)hullistoship

(C)puppyistodog

(D)milkistocow

(E)butteristobread

9.Poisonistoskullandcrossbonesas

(A)ropeistogallows

(B)pirateistogangplank

(C)antidoteistopoison

(D)loveistoheart

(E)bowistoarrow

10.Trivialistogrievousas

(A)acidistobitter

(B)lightistoheavy

(C)boringistosad

(D)oilistowater

(E)funnyistohilarious

11.Axistohatchetas

(A)fishistonursery

(B)chickenistoegg

(C)chopistocut

(D)hammeristosickle

(E)celloistoviolin

12.Fortuitousistoluckas

(A)chivalrousistomanners

(B)fossilistofuel

(C)formulaistomilk

(D)musicistodance

(E)poolistoswimming

TestYourself7

Directions:Lookatthefirsttwowordsanddecidehowtheyarerelatedtoeachother.Thendecidewhichoftheanswerchoicesrelatetothethirdwordinthesamewaythatthefirsttwoarerelated.

1.Hassockistofeetaspillowisto

(A)cushion

(B)chair

(C)boots

(D)fight

(E)head

2.Ghostistohauntasguruisto

(A)frighten

(B)teach

(C)amuse

(D)adhere

(E)tempt

3.Nailistotoeaslockisto

(A)key

(B)combination

(C)hair

(D)hammer

(E)barrel

4.Subistounderassuperisto

(A)janitor

(B)chief

(C)terrific

(D)over

(E)better

5.Deciduousistoleavesasevergreenisto

(A)needles

(B)tree

(C)Christmas

(D)conifer

(E)forest

6.Flounderistosharkastickisto

(A)clock

(B)deer

(C)flea

(D)insect

(E)woods

7.Starchistostiffasbleachisto

(A)smooth

(B)soft

(C)colorful

(D)dry

(E)white

8.Shellfishistolobsteraspoultryisto

(A)fowl

(B)spider

(C)chicken

(D)bird

(E)octopus

9.Ovalistooblongascircleisto

(A)round

(B)square

(C)sphere

(D)cube

(E)rectangle

10.Capaciousistocrampedasagapeisto

(A)ajar

(B)painful

(C)surprised

(D)empty

(E)sealed

11.Broodistolitteraspeonisto

(A)farmer

(B)slave

(C)migrant

(D)laborer

(E)scatter

12.Anorexiaistobulimiaasasthmaisto

(A)eating

(B)breathing

(C)allergy

(D)emphysema

(E)headache

TestYourself8

Directions:Decidehowthewordsinthefirstpairarerelated.Chooseapairofwordsbelowthatshowsthesamerelationship.

1.Felonistocrimeas

(A)judgeistojury

(B)courtroomistotrial

(C)physicianistocure

(D)verdictistosentence

(E)pharmacyistopharmacist

2.Odoristostenchas

(A)sadistotragic

(B)richistopoor

(C)greenistobrown

(D)summeristowinter

(E)floweristoanimal

3.Overblownistoexaggeratedas

(A)warrantistojustify

(B)anachronismistotimely

(C)malapropismistoaccurate

(D)requirementistooptional

(E)indefiniteistotomorrow

4.Loomistoappearas

(A)weaveistowool

(B)rootistodig

(C)comeistogo

(D)seemistobe

(E)warpistowoof

5.Escapeistofleeas

(A)runistohide

(B)breakistoenter

(C)hideistoseek

(D)captiveistocaptor

(E)dismountistoalight

6.Assentistodissentas

(A)assertistodesert

(B)compactistoexpansive

(C)assististodesist

(D)obtainistoretain

(E)futureistofutility

7.Natureistonurtureas

(A)fatheristomother

(B)authenticistoartificial

(C)congenitalistoacquired

(D)nativeistoalien

(E)homeistoschool

8.Squatistocrouchas

(A)inciseistoprecise

(B)countenanceistoface

(C)laxistostrict

(D)moltenistosolid

(E)mongrelistopuppy

9.Rudderistosteeringas

(A)razoristoshaving

(B)wheelistoturning

(C)cowistograzing

(D)cloudistoraining

(E)staplingistostapler

10.Standeeistoseatas

(A)kittyistolitter

(B)appleistopie

(C)saltistoocean

(D)acornistooak

(E)nomadistohome

11.Pauperistopooras

(A)shoeistopair

(B)bookistolong

(C)skyscraperistohigh

(D)bedistomake

(E)cleveristoowl

12.Cursoryistosuperficialas

(A)dismalistocheerful

(B)approbationistoconsecration

(C)deathistovictory

(D)desultoryistoaimless

(E)heroismistoreward

TestYourself9

Directions:Lookatthefirsttwowordsanddecidehowtheyarerelatedtoeachother.Thendecidewhichoftheanswerchoicesrelatetothethirdwordinthesamewaythatthefirsttwoarerelated.

1.Prattleistobabyasdiscourseisto

(A)conversation

(B)scholar

(C)speech

(D)subject

(E)lecture

2.Doseistomedicineasportionisto

(A)dessert

(B)serving

(C)potion

(D)food

(E)section

3.Bowlistosoupasplateisto

(A)cup

(B)fork

(C)dinner

(D)china

(E)meat

4.Gramistoounceasmeteristo

(A)mile

(B)pound

(C)yard

(D)weigh

(E)measure

5.Floweristoseedasseedisto

(A)plant

(B)water

(C)grow

(D)food

(E)nut

6.Oilistoearthassaltisto

(A)shaker

(B)pepper

(C)bloodpressure

(D)lick

(E)sea

7.Pestistoannoyingasplateauisto

(A)level

(B)calming

(C)boring

(D)curved

(E)hilly

8.Surviveistosuccumbasswimisto

(A)sail

(B)dive

(C)row

(D)sink

(E)float

9.Tauntistoteaseasvolubleisto

(A)loud

(B)large

(C)talkative

(D)willing

(E)loyal

10.Tiaraistojewelaswreathisto

(A)crown

(B)flower

(C)head

(D)woman

(E)door

11.Torridistoextremeastemperateisto

(A)warm

(B)hot

(C)climate

(D)moderate

(E)heat

12.Axleistowheelsaschainisto

(A)necklace

(B)handcuffs

(C)door

(D)gang

(E)daisies

TestYourself10

Directions:Decidehowthewordsinthefirstpairarerelated.Chooseapairofwordsbelowthatshowsthesamerelationship.

1.Peacockistoplumageas

(A)lionistomane

(B)friendistodiamonds

(C)coatistotails

(D)shineistoshoes

(E)dowageristofurs

2.Quagmireistoquarryas

(A)hunteristoprey

(B)hideistoseek

(C)mudistostone

(D)sinkistoswim

(E)findistokeep

3.Frogistotadpoleas

(A)toadistotree

(B)mushroomistotoadstool

(C)mothistopolliwog

(D)butterflyistocaterpillar

(E)imagoistoearthworm

4.Cottonistosummeras

(A)rainistosnow

(B)woolistowinter

(C)bollistoweevil

(D)furistocoat

(E)leafistotree

5.Motionistoqueasyas

(A)itchyistorash

(B)unfamiliaristostrange

(C)roundistodizzy

(D)feveristohot

(E)oceanistowavy

6.Nayistoyeaas

(A)voicesistohands

(B)nowistolater

(C)horseistosleigh

(D)neitheristonor

(E)negateistoaffirm

7.Knockeristodoorbellas

(A)penistopencil

(B)soundistoalarm

(C)strikeistohours

(D)inistoout

(E)pleasantistounpleasant

8.Bulletistogunas

(A)bladeistohatchet

(B)slingshotistopebble

(C)arrowistobow

(D)swordistospear

(E)victimistotarget

9.Walrusistosealas

(A)cowistocalf

(B)doeistodeer

(C)elephantistolizard

(D)ramistoewe

(E)gooseistoduck

10.Freezeristoicecreamas

(A)safeistomoney

(B)jellyistojam

(C)paperistopencil

(D)rainistosnow

(E)coinsistopiggybank

11.Humistosingas

(A)speakistosee

(B)deafistomute

(C)pantomimeistoact

(D)playistoopera

(E)proseistopoetry

12.Patriarchistomatriarchas

(A)churchistostate

(B)leaderistofollower

(C)nobleistocommon

(D)tailoristoseamstress

(E)ancientistomodern

TestYourself11

Directions:Lookatthefirsttwowordsanddecidehowtheyarerelatedtoeachother.Thendecidewhichoftheanswerchoicesrelatetothethirdwordinthesamewaythatthefirsttwoarerelated.

1.Calligraphyistopreciseasscrawlisto

(A)handwriting

(B)primitive

(C)careless

(D)illegible

(E)scratchy

2.Agitatedistostoicassociableisto

(A)hermit

(B)socialite

(C)cheerleader

(D)friendly

(E)acrobat

3.Sweatistoperspireasswatisto

(A)fly

(B)destroy

(C)exercise

(D)kill

(E)hit

4.Nostalgiaistopastasanticipationisto

(A)excitement

(B)future

(C)present

(D)apprehension

(E)past

5.Gleefulistogloomyasorganizedisto

(A)original

(B)neat

(C)unintellectual

(D)disinterested

(E)messy

6.Venisonistodeerasvealisto

(A)lamb

(B)steer

(C)cow

(D)sheep

(E)calf

7.Obedienceistoobstinacyaspeaceisto

(A)prosperity

(B)patriotism

(C)penury

(D)war

(E)tranquility

8.Traitoristotreasonaspatriotisto

(A)espionage

(B)jingoism

(C)loyalty

(D)heritage

(E)sacrifice

9.Rinkistoskateassinkisto

(A)wash

(B)swim

(C)fish

(D)soap

(E)kitchen

10.Gullyistowaterasfurrowisto

(A)brow

(B)farm

(C)erosion

(D)rabbit

(E)plow

11.Desertistojungleasdustyisto

(A)damp

(B)dirty

(C)dry

(D)hot

(E)dark

12.Steamistoboilingassmokeisto

(A)heat

(B)chimney

(C)combustion

(D)freezing

(E)vapor

TestYourself12

Directions:Decidehowthewordsinthefirstpairarerelated.Chooseapairofwordsbelowthatshowsthesamerelationship.

1.Tillistocultivateas

(A)plowistohorse

(B)tractoristoplow

(C)plantistofurrow

(D)farmistosoil

(E)toilistowork

2.Toothistoboneas

(A)tireistofanbelt

(B)pondistoforest

(C)fishistobird

(D)staristosun

(E)houseistohome

3.Fryistostewas

(A)chickenistobeef

(B)heatistofire

(C)panistopot

(D)dryistowet

(E)meatistopotatoes

4.Rustleistoroaras

(A)wateristowind

(B)walkistorun

(C)rainistosnow

(D)cattleistocoyote

(E)shoutistoyell

5.Fallistoleafas

(A)meltistosnow

(B)buildistohouse

(C)shrubistoflower

(D)sneezeistocough

(E)digistospade

6.Spaistohealthas

(A)museumistopictures

(B)conservationistozoo

(C)sportistostadium

(D)libraryistoinformation

(E)quietistochurch

7.Exactingistodemandingas

(A)burrowingistobuilding

(B)testingistoproving

(C)changingistoappearing

(D)thrillingistoboring

(E)extractingistoremoving

8.Labyrinthistonetworkas

(A)weatheristoclimate

(B)veinistoblood

(C)mazeistopassages

(D)epicistohero

(E)Greeceistomyths

9.Kissistocheekas

(A)cheekistojowl

(B)shakeistohand

(C)hugistobear

(D)noseistorub

(E)milkistocookies

10.Ingestistodigestas

(A)sergeantistocaptain

(B)eatistodrink

(C)drinkistodrive

(D)spideristofly

(E)understandistostudy

11.Impistohornsas

(A)devilistoangel

(B)evilistogood

(C)haloistowings

(D)mischiefistoelf

(E)cherubistowings

12.Kneadistodoughas

(A)rottedistoapple

(B)fallenistoarches

(C)soakistobeans

(D)driftedistosnow

(E)spentistomoney

TestYourself13

Directions:Findarelationshipbetweenthefirsttwowordsineachquestion.Tocompletetheanalogy,choosetheanswerthatshowsasimilarrelationship.

1.Lightistodarkaswetisto

(A)snow

(B)rain

(C)hinge

(D)dry

(E)water

2.Summeristowinteraseveningisto

(A)sunset

(B)coolness

(C)darkness

(D)morning

(E)shadow

3.Rightistoleftaslowisto

(A)high

(B)bottom

(C)sorrow

(D)note

(E)deep

4.Nothingistoeverythingaswhisperisto

(A)mystery

(B)something

(C)shout

(D)ghost

(E)quiet

5.Dayistonightassunisto

(A)solar

(B)heat

(C)universe

(D)moon

(E)star

6.Weakistostrongasincapableisto

(A)clumsy

(B)cowardly

(C)adept

(D)failure

(E)impossible

7.Advanceistohaltas

(A)stopistogo

(B)returnistochange

(C)goistostop

(D)conqueristotake

(E)yieldistochange

8.Skillfulistoclumsyas

(A)embarrassingistoawkward

(B)quickistoswift

(C)smoothistoslick

(D)tiredistoexhausted

(E)alertistosleepy

9.Enemiesistofriendsas

(A)pacifyistoquiet

(B)hateistodislike

(C)defeatistolose

(D)despiseistoesteem

(E)jumpistoleap

10.Blameistopraiseas

(A)ascendistoclimb

(B)defeatistocondemn

(C)succeedistodefeat

(D)emergeistoemanate

(E)failistosucceed

TestYourself14

Directions:Findarelationshipbetweenthefirsttwowordsineachquestion.Tocompletetheanalogy,choosetheanswerthatshowsasimilarrelationship.

1.Windowistopaneasdooristo

(A)panel

(B)knob

(C)hinge

(D)key

(E)lock

2.Paragraphistosentenceassentenceisto

(A)modifier

(B)word

(C)composition

(D)grammar

(E)contraction

3.Nutistoshellaspeaisto

(A)shooter

(B)soup

(C)green

(D)bean

(E)pod

4.Antleristodeeras

(A)tuskistohusk

(B)animalistoelephant

(C)tuskistoelephant

(D)hornistoantler

(E)catistowhisker

5.Jewelistoringas

(A)stoneistoruby

(B)locketistonecklace

(C)preciousistostone

(D)ironistochain

(E)rockistodiamond

6.Pitistopeachas

(A)sunistostar

(B)moonistoEarth

(C)planeistoEarth

(D)moonistoorbit

(E)sunistosolarsystem

7.Pageistobookas

(A)periodistocomma

(B)novelistobook

(C)wordistopage

(D)libraryistostory

(E)paragraphistoword

8.Stateistocountryas

(A)continentistoworld

(B)worldistocontinent

(C)countryistocity

(D)oceanistoshore

(E)cityistotown

9.Handistobodyasstaristo

(A)sky

(B)universe

(C)eye

(D)movie

(E)moon

10.Playistoprologueasconstitutionisto

(A)preamble

(B)BillofRights

(C)preview

(D)SupremeCourt

(E)amendment

TestYourself15

Directions:Findarelationshipbetweenthefirsttwowordsineachquestion.Tocompletetheanalogy,choosetheanswerthatshowsasimilarrelationship.

1.Letteristomailasmoneyisto

(A)bank

(B)savings

(C)invest

(D)account

(E)buy

2.Sealistofloatasbirdisto

(A)flap

(B)wing

(C)soar

(D)nest

(E)feather

3.Eggistoscrambleaspotatoisto

(A)mash

(B)skin

(C)butter

(D)slice

(E)root

4.Artististopaintascontractoristo

(A)agreement

(B)build

(C)masonry

(D)carpentry

(E)contract

5.Scissorsistotrimasscalesisto

(A)fish

(B)weight

(C)measure

(D)weigh

(E)pound

6.Bedistosleepaschairisto

(A)carry

(B)sit

(C)stare

(D)recline

(E)dream

7.Tasteistotongueastouchisto

(A)skin

(B)arms

(C)feelings

(D)ears

(E)smell

8.Boxistocoverasbottleisto

(A)glass

(B)contain

(C)plastic

(D)break

(E)cork

9.Shoeistolaceasdooristo

(A)hinge

(B)enter

(C)swing

(D)window

(E)lock

10.Typewriteristowriteascalculatoristo

(A)add

(B)compute

(C)percentage

(D)predict

(E)math

TestYourself16

Directions:Findarelationshipbetweenthefirsttwowordsineachquestion.Tocompletetheanalogy,choosetheanswerthatshowsasimilar

relationship.

1.Satisfactionistogooddeedasimprovementisto

(A)sin

(B)fault

(C)criticism

(D)kindness

(E)happiness

2.Seedistoplantaseggisto

(A)yolk

(B)crack

(C)bird

(D)shell

(E)nest

3.Wheatistoflourasgrapeisto

(A)vintage

(B)wine

(C)vine

(D)vineyard

(E)bunch

4.Heatistofireascloudisto

(A)sky

(B)snow

(C)sun

(D)moisture

(E)smoke

5.Threatistoinsecurityas

(A)challengeistofight

(B)thunderistolightning

(C)reasonistoanger

(D)speedistobrake

(E)laughistojoke

6.Heatistoradiatoras

(A)seaistowave

(B)treeistobreeze

(C)windistotrees

(D)breezeistofan

(E)iceistofreeze

7.Medicationistocureas

(A)drugistodiagnosis

(B)examinationistotreatment

(C)physicianistorelief

(D)vaccinationistoprevention

(E)patientistodoctor

8.Waristogriefas

(A)joyistopeace

(B)happinessistosadness

(C)peaceistofinish

(D)joyistohappiness

(E)peaceistohappiness

9.Moonistolightas

(A)sunsetistosun

(B)earthistoorbit

(C)eclipseistodark

(D)gravityistoearth

(E)tideistoocean

10.Ignitionistostarteras

(A)radioistoantenna

(B)brakeistostop

(C)airistotire

(D)shutistodoor

(E)engineistogo

TestYourself17

Directions:Findarelationshipbetweenthefirsttwowordsineachquestion.Tocompletetheanalogy,choosetheanswerthatshowsasimilarrelationship.

1.Ringistofingerascuffisto

(A)arm

(B)shoulder

(C)hand

(D)shirt

(E)wrist

2.Gasistovehicleaswoodisto

(A)tree

(B)fire

(C)ax

(D)heat

(E)stove

3.Refrigeratoristomeatasbankisto

(A)cashier

(B)combination

(C)watchman

(D)money

(E)robber

4.Watchmanistoprotectasnavigatoristo

(A)navy

(B)guide

(C)plan

(D)map

(E)drive

5.Gymnasiumistogameasauditoriumisto

(A)production

(B)script

(C)cafeteria

(D)actors

(E)director

TestYourself18

Directions:Findarelationshipbetweenthefirsttwowordsineachquestion.Tocompletetheanalogy,choosetheanswerthatshowsasimilarrelationship.

1.Presentistobirthdayasrewardisto

(A)accomplishment

(B)medal

(C)punishment

(D)money

(E)gold

2.Guestistoacceptanceashostisto

(A)party

(B)invitation

(C)partygiver

(D)refreshments

(E)contract

3.Stickistopuckasbatisto

(A)cricket

(B)ball

(C)touchdown

(D)bowl

(E)base

4.Feveristospringasleavesisto

(A)October

(B)season

(C)autumn

(D)sadness

(E)rake

5.Honorableistobraveryasguiltyisto

(A)judge

(B)peaceful

(C)important

(D)productive

(E)criminal

ANSWERKEYSTestYourself1

1.C2.B3.A

4.E5.D6.A

7.E8.B

9.E10.C

TestYourself2

1.A2.C3.B

4.D5.A6.A

7.E8.B9.C

10.C11.E12.B

TestYourself3

1.C2.B3.A

4.E5.A6.C

7.E8.A9.D

10.A11.E12.B

TestYourself4

1.B2.A3.E

4.B5.D6.A

7.E8.B9.A

10.D11.C12.A

TestYourself5

1.D2.C3.E

4.A5.E6.B

7.D8.C9.A

10.E11.C12.C

TestYourself6

1.C2.B3.A

4.E5.A6.E

7.C8.B9.D

10.B11.E12.A

TestYourself7

1.E2.B3.C

4.D5.A6.C

7.E8.C9.A

10.E11.D12.D

TestYourself8

1.C2.A3.A

4.B5.E6.B

7.C8.B9.B

10.E11.C12.D

TestYourself9

1.B 4.C5.A

7.A 10.B11.D

2.D3.E

6.E 8.D9.C

12.B

TestYourself10

1.A2.C3.D

4.B5.D6.E

7.A8.C9.E

10.A11.C12.D

TestYourself11

1.C2.A3.E

4.B5.E6.E

7.D8.C9.A

10.E11.A12.C

TestYourself12

1.E2.A3.C

4.B5.A6.D

7.E8.C9.B

10.A11.E12.C

TestYourself13

1.D2.D3.A

4.C5.D6.C

7.C8.E

9.D10.E

TestYourself14

1.A2.B3.E

4.C5.B6.E

7.C8.A

9.B10.A

TestYourself15

1.C2.C3.A

4.B5.D6.B

7.A8.E

9.E10.B

TestYourself16

1.C2.C3.B

4.D5.A6.D

7.D8.E

9.C10.C

TestYourself17

1.E2.E

3.D 4.B 5.A

TestYourself18

1.A2.B

3.B 4.C 5.E

EXERCISES:MIXEDRELATIONSHIPSTheanalogiesinthissectionarenotarrangedinanyparticularpattern.Youmustdecidewhattypeofrelationshipisbeingusedineachitem.

Directions:Findarelationshipbetweenthefirsttwowordsineachquestion.Tocompletetheanalogy,choosetheanswerthatshowsasimilarrelationship.

1.ThanksgivingistoNovemberasChristmasisto

(A)SantaClaus

(B)holiday

(C)snow

(D)JingleBells

(E)December

2.Rememberistoforgetasfindisto

(A)locate

(B)keep

(C)lose

(D)return

(E)watch

3.Shipistoanchorasautomobileisto

(A)brake

(B)wheel

(C)stop

(D)accelerator

(E)car

4.Endistoabolishasbeginisto

(A)establish

(B)finish

(C)tyranny

(D)crusade

(E)monarchy

5.Woodistodecayasironisto

(A)dampness

(B)rod

(C)steel

(D)ore

(E)rust

6.Monthistoweekasweekisto

(A)month

(B)hour

(C)year

(D)day

(E)calendar

7.Flouristowheatasgravelisto

(A)brick

(B)road

(C)coal

(D)bread

(E)rock

8.Attackistoprotectasoffenseisto

(A)combat

(B)defense

(C)conceal

(D)reconcile

(E)battle

9.Divideistomultiplyassubtractisto

(A)plus

(B)reduce

(C)multiply

(D)add

(E)takeaway

10.Mineistomyasyoursisto

(A)you

(B)ours

(C)your

(D)you’re

(E)us

11.Glassesistovisionas

(A)glassistomirror

(B)lightistovision

(C)eatingistofork

(D)handistoobject

(E)drinkistowater

12.Flameistoburnas

(A)insultistoanger

(B)glassistocrack

(C)birthistolife

(D)sunistoorbit

(E)plantistogrow

13.Lookistoseeas

(A)illuminateistolight

(B)auditionistospeak

(C)thinkistoguess

(D)followistolead

(E)listenistohear

14.Wolfistopackas

(A)cowistoherd

(B)cattleistofarmer

(C)cowistograze

(D)farmeristofarm

(E)flockistogeese

15.Leaveistostayasdepartisto

(A)home

(B)disembark

(C)run

(D)remain

(E)exit

16.Caristomechanicaspeopleisto

(A)butcher

(B)lawyer

(C)spouse

(D)doctor

(E)teacher

17.Forestistotreeascrowdisto

(A)person

(B)alone

(C)men

(D)many

(E)crowded

18.Fictionistonovelistasfactisto

(A)legend

(B)story

(C)historian

(D)research

(E)essay

19.Strangeristostrangeas

(A)oddestistoodd

(B)artististoartistic

(C)artistoartist

(D)satirististoartist

(E)paintistopainter

20.Librarianistolibraryas

(A)schoolistoeducation

(B)officeistoprincipal

(C)studentistoschool

(D)gymnasiumistoworkout

(E)teacheristoschool

21.Sicknessistohealthasdeathisto

(A)mortician

(B)skull

(C)old

(D)pirate

(E)life

22.Inventoristomachineasauthoristo

(A)book

(B)poet

(C)creator

(D)computer

(E)artist

23.Weightistopoundasdistanceisto

(A)liter

(B)mile

(C)ruler

(D)space

(E)race

24.Concealistorevealasascendisto

(A)embark

(B)descend

(C)mount

(D)leave

(E)climb

25.Capeistocontinentas

(A)oceanistolake

(B)lakeistoreservoir

(C)reservoiristowater

(D)gulfistoocean

(E)riveristopond

26.Visionistolensas

(A)hospitalistodoctor

(B)crutchistolegs

(C)walkingistoparalysis

(D)doctoristoparalysis

(E)mobilityistocrutches

27.Busistoroadas

(A)wheelistostreet

(B)steelistorails

(C)locomotiveistotrack

(D)locomotiveistosteam

(E)trainistocar

28.YearistoJulyas

(A)weekistomonth

(B)dayistoweek

(C)monthistocentury

(D)millimeteristocentimeter

(E)meteristocentimeter

29.Resignationistosighas

(A)hopeistowish

(B)faithistopray

(C)surpriseistogasp

(D)terroristofear

(E)startleistosurprise

30.LincolnistoNebraskaas

(A)WashingtonistoOregon

(B)NewYorkistoKentucky

(C)ChicagoistoNewYork

(D)TrentonistoNewJersey

(E)OregonistoPortland

31.Pencilistosharpenas

(A)knifeistocut

(B)carpenteristobuild

(C)woodistosaw

(D)wellistofill

(E)sawistoax

32.Fingeristohandas

(A)armistosleeve

(B)shoeistofoot

(C)strandistohair

(D)blouseistoskirt

(E)legistofoot

33.Convexistoconcaveas

(A)hillistohole

(B)inistowithin

(C)roundistosquare

(D)noseistomouth

(E)underistobeneath

34.Cowistomilkas

(A)ratistocheese

(B)beeistohoney

(C)birdistowing

(D)catistodog

(E)catistofish

35.Exerciseistoreduceas

(A)grumbleistoresign

(B)snowistofreeze

(C)spendingistosave

(D)luckistowin

(E)runistorace

36.Mountainistopeakas

(A)holeistohill

(B)stormistoocean

(C)tideistoocean

(D)stormistohurricane

(E)waveistocrest

37.Foodistobodyas

(A)fuelistoengine

(B)batistoball

(C)kiteistotail

(D)mechanicistoengine

(E)gasolineistooil

38.Pressureistobarometeras

(A)centimeteristometer

(B)speedometeristodistance

(C)meteristoperimeter

(D)comptometeristocomptroller

(E)temperatureistothermometer

39.Famineistoabundanceas

(A)hungeristostarvation

(B)squaloristostarvation

(C)povertyistowealth

(D)famineistohunger

(E)wealthistoriches

40.Studyistostudiouslyas

(A)workistolearning

(B)playistoplayfully

(C)bookistobookish

(D)habitistohabitual

(E)teachistoteacher

41.Discardistodeleteas

(A)attachistodetach

(B)contagiousistospread

(C)fartheristofar

(D)alteristorevise

(E)tearistomend

42.Rejectistoaversionas

(A)thinkistoconsider

(B)ponderistodesire

(C)imposeistoact

(D)contentistochange

(E)chooseistopreference

43.Measlesistodiseaseas

(A)felonyistocrime

(B)measlesistomumps

(C)felonyistomisdemeanor

(D)crimeistolaw

(E)lawistojudge

44.Occasionalistoconstantas

(A)intermittentistoincessant

(B)intramuralistoinconsistent

(C)frequentistoincessant

(D)inadvertentistoaccidental

(E)infrequentistoirregular

45.Activeistomobileas

(A)mobileistoimmobile

(B)inflammableistoextinguished

(C)sedentaryistoimmobile

(D)sensationalistomovement

(E)immobileistomovement

46.Eliminateistooptionalas

(A)maintainistooption

(B)retainistoessential

(C)optionistoopportunity

(D)sequentialistoorder

(E)deleteistoamend

47.Alludeistoreferas

(A)illusionistoreality

(B)similaristodisparate

(C)implyistostate

(D)concludeistoinfer

(E)guessistoknow

48.Planeistocharteras

(A)workeristouse

(B)assistantistoappoint

(C)personistohire

(D)manageristosalary

(E)lawyeristojudge

49.Superscriptistosubscriptas

(A)exportistoexpert

(B)introvertistoconvert

(C)impedeistointercede

(D)subscriptionistoprescription

(E)introvertistoextrovert

50.Complementistocomplimentas

(A)wonderistowander

(B)surfeitistosurface

(C)interfaceistosurface

(D)styleistostile

(E)effectistoaffect

51.Premonitionistoprophesyas

(A)prefaceistoprologue

(B)predictistorelate

(C)prologueistoepilogue

(D)historyistoarchives

(E)preambleistoconclusion

52.Collaborationistocooperationas

(A)collationistocorrection

(B)concentrationistoquiet

(C)coercionistosubmission

(D)compromiseistopromise

(E)computationistoadd

53.Revertistoreversionas

(A)interestistointercession

(B)invertistoovert

(C)sympathizeistosympathy

(D)sympathyistosympathetic

(E)introvertistoextrovert

54.Salespersonistocommissionas

(A)authoristoroyalty

(B)agentistoactor

(C)tipistowaiter

(D)feeistocharge

(E)royaltyistofee

55.Harassmentistoangeras

(A)disappointmentistosorrow

(B)heightistoweight

(C)laughteristotears

(D)marriageistolove

(E)engagementistomarry

56.Divulgeistodiscloseas

(A)reviseistocreate

(B)adviseistoedit

(C)dispelistocollect

(D)bulgeistoclose

(E)appraiseistoestimate

57.Sodiumistosaltas

(A)torchistoacetylene

(B)ammoniaistopneumonia

(C)oxygenistowater

(D)balloonistohelium

(E)airistocarbon

58.Cautiousistoimpulsiveas

(A)furiousistoanger

(B)causticistobiting

(C)creativeistowork

(D)punishistobehavior

(E)secretiveistocandid

59.Hiddenistoobviousas

(A)reservedistorambunctious

(B)emphaticistovehement

(C)embezzleistosteal

(D)openistodoor

(E)encounteristomeet

60.Swordistoduelingaspenisto

(A)writer

(B)inkwell

(C)ink

(D)writing

(E)manuscript

61.Reprimandistodisapprovalascomplimentisto

(A)flatter

(B)approval

(C)affirmation

(D)improvement

(E)insult

62.Goldistoyellowasbloodisto

(A)black

(B)purple

(C)blue

(D)white

(E)red

63.Recurrenceistoperiodicasdeterminationisto

(A)cowardly

(B)hopeless

(C)literary

(D)persevering

(E)lazy

64.Anarchyistolawasdiscordisto

(A)difference

(B)amendment

(C)adaptation

(D)confusion

(E)agreement

65.Ceilingistochandelieraspuppeteeristo

(A)puppet

(B)puppetshow

(C)stage

(D)ventriloquist

(E)hand

66.Accidentistocarelessnessasresponseisto

(A)answer

(B)correct

(C)stimulus

(D)effect

(E)respond

67.Correctionistoerroneousasclarificationisto

(A)criticism

(B)failure

(C)amend

(D)ambiguous

(E)mistake

68.Automobileistohorseastelephoneisto

(A)wagon

(B)telegraph

(C)communication

(D)transportation

(E)wireless

69.Intimidateistodauntasdismayisto

(A)horrify

(B)destroy

(C)dismantle

(D)forego

(E)delight

70.Spontaneousistocalculatedasimpromptuisto

(A)adlib

(B)scheduled

(C)verbose

(D)prolific

(E)taciturn

71.Criticistoplayasrevieweristo

(A)job

(B)work

(C)newspaper

(D)book

(E)essay

72.Lionistoprideas

(A)birdistovanity

(B)cowistopasture

(C)packistodog

(D)fishistoschool

(E)cowistocalf

73.Refereeistorulesasconscienceisto

(A)citizen

(B)thoughts

(C)regulations

(D)behavior

(E)morality

74.Moneyistostealasideaisto

(A)lose

(B)manuscript

(C)plagiarize

(D)thief

(E)elaborate

75.Bookistopaperasscrollisto

(A)cloth

(B)binding

(C)roll

(D)parchment

(E)cover

ANSWERSANDEXPLANATIONS

1.Thecorrectansweris(E).THANKSGIVINGisinNOVEMBER;CHRISTMASisinDECEMBER.

2.Thecorrectansweris(C).Thisisanantonymrelationship.

3.Thecorrectansweris(A).ANCHORSstopSHIPS;BRAKESstopAUTOMOBILES.Thesecondwordineachpairisthepartusedtostopthefirstnoun.

4.Thecorrectansweris(A).Thewordsineachpairaresynonyms.

5.Thecorrectansweris(E).WOODDECAYS;IRONRUSTS.Thefirstwordineachpairisanoun.Thesecondisaverbthatrelatestothefirst.

6.Thecorrectansweris(D).AWEEKispartofaMONTH;aDAYispartofaWEEK.

7.Thecorrectansweris(E).FLOURisgroundWHEAT;GRAVELisbrokenROCK.

8.Thecorrectansweris(B).Thewordsineachpairareantonyms.

9.Thecorrectansweris(D).Thewordsineachpairareantonyms.

10.Thecorrectansweris(C).Eachpaircontainssimilarpossessivepronouns.

11.Thecorrectansweris(B).GLASSESandLIGHTenhanceaperson’sVISION.

12.Thecorrectansweris(A).AFLAMEmaycauseaBURN;anINSULTmaycauseANGER.

13.Thecorrectansweris(E).SEEINGistheresultofLOOKING;HEARINGistheresultofLISTENING.

14.Thecorrectansweris(A).WOLVESgatherinPACKS;COWSgatherinHERDS.

15.Thecorrectansweris(D).Thewordsineachpairareantonyms.

16.Thecorrectansweris(D).MECHANICSrepairCARS;DOCTORS“repair”PEOPLE.

17.Thecorrectansweris(A).ATREEisonepartofaFOREST;aPERSONisonepartofaCROWD.

18.Thecorrectansweris(C).ANOVELISTwritesFICTION;aHISTORIANwritesabout

FACTS.

19.Thecorrectansweris(B).STRANGERisthenounformoftheadjectiveSTRANGE;ARTISTisthenounformoftheadjectiveARTISTIC.

20.Thecorrectansweris(E).ALIBRARIANworksinaLIBRARY;aTEACHERworksinaSCHOOL.

21.Thecorrectansweris(E).Thewordsineachpairareantonyms.

22.Thecorrectansweris(A).NewMACHINESarecreatedbyINVENTORS;newBOOKSarecreatedbyAUTHORS.

23.Thecorrectansweris(B).APOUNDisaunitofWEIGHT;aMILEisaunitofDISTANCE.

24.Thecorrectansweris(B).Thewordsineachpairareantonyms.

25.Thecorrectansweris(D).ACAPEisageographicalpartofaCONTINENT;aGULFispartofanOCEAN.

26.Thecorrectansweris(E).ALENSisanaidtoVISION;CRUTCHESaidMOBILITY.

27.Thecorrectansweris(C).ABUStravelsalongaROAD;aLOCOMOTIVEmovesonaTRACK.

28.Thecorrectansweris(E).YEARisthewhole;JULYisthepart;METERisthewhole;CENTIMETERisthepart.Allotherpart-wholerelationshipsarereversed.

29.Thecorrectansweris(C).ASIGHisakindofbreaththatexpressesRESIGNATION;aGASPisakindofbreaththatexpressesSURPRISE.

30.Thecorrectansweris(D).LINCOLNisthecapitalofNEBRASKA;TRENTONisthecapitalofNEWJERSEY.

31.Thecorrectansweris(C).SHARPENaPENCIL;SAWapieceofWOOD.

32.Thecorrectansweris(C).AFINGERispartofaHAND;aSTRANDofhairispartofaheadofHAIR.

33.Thecorrectansweris(A).CONVEX,anoutwardcurve,istheantonymofCONCAVE,anindentedcurve.Similarly,HILLandHOLEareantonyms.

34.Thecorrectansweris(B).COWSproduceMILK;BEESproduceHONEY.

35.Thecorrectansweris(D).EXERCISEaidsweightREDUCTION;LUCKisanaidtoWINNING.

36.Thecorrectansweris(E).APEAKisthetoppartofaMOUNTAIN;aCRESTisthetopofaWAVE.

37.Thecorrectansweris(A).FOODenablesaBODYtodowork;FUEL,suchasgasoline,enablesanENGINEtoworkinasimilarway.

38.Thecorrectansweris(E).ABAROMETERmeasuresatmosphericPRESSURE;aTHERMOMETERmeasuresTEMPERATURE.

39.Thecorrectansweris(C).Thewordsineachpairareantonyms.

40.Thecorrectansweris(B).STUDIOUSLYistheadverbformoftheverbSTUDY;PLAYFULLYistheadverbformoftheverbPLAY.

41.Thecorrectansweris(D).Thewordsineachpairaresynonyms.

42.Thecorrectansweris(E).HavinganAVERSIONtosomethingmaycauseonetoREJECTit;aPREFERENCEcausesonetoCHOOSEit.

43.Thecorrectansweris(A).MEASLESisakindofDISEASE;aFELONYisakindofCRIME.

44.Thecorrectansweris(A).Thewordsineachpairareantonyms.

45.Thecorrectansweris(C).ThewordACTIVEisassociatedwithMOBILITY,ormovement;SEDENTARYisassociatedwithIMMOBILITY,orlackofmovement.

46.Thecorrectansweris(B).AnOPTIONALitemmaybeELIMINATED;anESSENTIALonemustbekept,orRETAINED.

47.Thecorrectansweris(C).Thewordsineachpairhavesimilarmeanings.

48.Thecorrectansweris(C).Chartermeanstohire;CHARTERaPLANE,HIREaPERSON.

49.Thecorrectansweris(E).TheprefixesSUPER-andSUB-haveoppositemeanings;similarly,INTRO-andEXTRO-areantonyms.

50.Thecorrectansweris(E).Thewordsofeachpairhaverelatedbutdifferentmeaningsandsimilarpronunciationsbutdifferentspellings.

51.Thecorrectansweris(D).APREMONITIONisafeelingaboutthefuture;aPROPHESYisastatementaboutthefuture.HISTORYiswhathashappenedinthepast;ARCHIVESaretherecordsofthosehappenings.

52.Thecorrectansweris(B).COLLABORATIONrequiresCOOPERATION;similarly,QUIETisnecessaryforCONCENTRATION.

53.Thecorrectansweris(C).REVERSIONisthenounformoftheverbREVERT;SYMPATHYisthenounformoftheverbSYMPATHIZE.

54.Thecorrectansweris(A).ACOMMISSIONisapercentageofsalespaidtotheSALESPERSON;aROYALTYisapercentageofbooksalespaidtotheAUTHOR.

55.Thecorrectansweris(A).HARASSMENTcausesANGER;DISAPPOINTMENTcausesSORROW.

56.Thecorrectansweris(E).Thewordsineachpairaresynonyms.

57.Thecorrectansweris(C).SODIUMisanelementofSALT;OXYGENisanelementofWATER.

58.Thecorrectansweris(E).Thewordsineachpairareantonyms.

59.Thecorrectansweris(A).Thewordsineachpairareantonyms.

60.Thecorrectansweris(D).ASWORDisthetoolusedforDUELING;aPENisatoolusedforWRITING.

61.Thecorrectansweris(B).AREPRIMANDisaverbalshowofDISAPPROVAL;aCOMPLIMENTisaverbalshowofAPPROVAL.

62.Thecorrectansweris(E).GOLDisadeepshadeofYELLOW;BLOODisadeepshadeofRED.

63.Thecorrectansweris(D).APERIODICeventRECURS;aDETERMINEDpersonPERSEVERES.

64.Thecorrectansweris(E).ANARCHYoccursintheabsenceofLAW;DISCORDistheresultofalackofAGREEMENT.

65.Thecorrectansweris(A).ACHANDELIERisattachedtoaCEILING;aPUPPETisheldbyaPUPPETEER.

66.Thecorrectansweris(C).AnACCIDENTmaybetheresultofCARELESSNESS;aRESPONSEmaybetheresultofaSTIMULUS.

67.Thecorrectansweris(D).SomethingERRONEOUSisinerrorandsubjecttoCORRECTION;somethingAMBIGUOUSisconfusingandsubjecttoCLARIFICATION.

68.Thecorrectansweris(B).HORSESwereusedfortransportationbeforeAUTOMOBILES;TELEGRAPHSwereusedforcommunicationbeforeTELEPHONES.

69.Thecorrectansweris(A).Thewordsineachpairaresynonyms.

70.Thecorrectansweris(B).Thewordsineachpairareantonyms.

71.Thecorrectansweris(D).CRITICScriticizePLAYS;REVIEWERScriticizeBOOKS.

72.Thecorrectansweris(D).ALIONisamemberofaPRIDE;aFISHisamemberofaSCHOOL.

73.Thecorrectansweris(E).AREFEREEenforcesRULES;aCONSCIENCEenforcesMORALITY.

74.Thecorrectansweris(C).Thesecondwordineachpairreferstothetheftofsomethingrepresentedbythefirstwordinthepair.

75.Thecorrectansweris(D).BOOKSconsistofPAPERpages;SCROLLSarepiecesofPARCHMENT.

SUMMINGITUP

•Verbalanalogiestestyourabilitytoseearelationshipbetweentwowordsandtorecognizeasimilarrelationshipbetweentwootherwords.

•Thekeytoanalogysuccessisbeingabletoexpresstherelationshipbetweenthewordsinapair—notwhatthewordsmean,buthowthey’rerelated.

•Rememberthattheanalogiesgetmoredifficultasyouworkyourwaythrougheachgroup.•Alsorememberthattherelationshipofthewordsintheanswermustbeinthesameorderastherelationshipofthewordsinthefirstpair.

Chapter8

SentenceCompletions(ISEEOnly)

OVERVIEW

•Whatmakesasentencecompletion?•Howdoyouanswersentencecompletionquestions?•Whatdosmarttest-takersknow?•Testyourselfquizzes•Answerkeys•Summingitup

WHATMAKESASENTENCECOMPLETION?Areyoudrawingablank?Getusedtoit,becauseyou’llseealotoftheminthesentencecompletionquestionsontheISEE.Inthiskindofquestion,youaregivenasentencethathasoneormoreblanks.Anumberofwordsorpairsofwordsaresuggestedtofillintheblankspaces.It’suptoyoutoselectthewordorpairofwordsthatwillbestcompletethemeaningofthesentence.

WhyaretheresentencecompletionquestionsontheISEE?Sentencecompletionquestionstestyourvocabularyaswellasyourabilitytounderstandwhatyouread.Inatypicalsentencecompletionquestion,severalofthechoicescouldbeinsertedintotheblankspaces.However,onlyoneanswerwillmakesenseandcarryoutthefullmeaningofthesentence.

HOWDOYOUANSWERSENTENCECOMPLETION

QUESTIONS?Areyoureadytostartfillinginsomeofthoseblanks?Thefollowingsixstepswillhelpyouanswersentencecompletionquestions:SentenceCompletions:GettingItRight

Readthesentencecarefully.Guessattheanswer.Scantheanswerchoicesforthewordyouguessed.Ifit’sthere,markitandgoon.Ifit’snot,goontoStep4.Examinethesentenceforcluestothemissingword.Eliminateanyanswerchoicesthatareruledoutbytheclues.Trytheonesthatareleftandpickwhicheverisbest.

Now,let’stryoutthesestepsonacoupleofsentencecompletionquestions:

Thosewhofeelthatwarisstupidandunnecessarythinkthattodieonthebattlefieldis____.

(A)courageous

(B)pretentious

(C)useless

(D)illegal

Followingthestepsoutlinedabove:Readthesentence.Thinkofyourownwordtofillintheblank.You’relookingforawordthatcompletesthelogicofthesentence.Youmightcomeupwithsomethinglikedumb.Lookfordumbintheanswerchoices.It’snotthere,butuselessis.That’sprettyclose,somarkitandgoon.Ifyoucouldn’tguesstheword,takeyourcluefromthewordsstupidandunnecessaryinthesentence.Theydefinitelypointtowardsomenegative-soundingword.Thecluesimmediatelyeliminatechoice(A),courageous,whichisapositiveword.Trytheremainingchoicesinthesentence,andyou’llseethatuselessfitsbest.

Unrulypeoplemaywellbecome____iftheyaretreatedwith____bythosearoundthem.

(A)angry...kindness

(B)calm...respect

(C)peaceful...abuse

(D)interested...medicine

Readthesentence.Thistimetherearetwoblanks,andthemissingwordsneedtohavesomelogicalconnection.Thinkofyourownwordstofillintheblanks.Youmightguessthattheunrulypeoplewillbecomewell-behavediftheyaretreatedwithconsideration.Nowlookforyourguessesintheanswerchoices.They’renotthere,buttherearesomepossibilities.Gobacktothesentenceandlookforclues.Becomesignalsthattheunrulypeoplewillchangetheirbehavior.Howthatbehaviorchangeswilldependonhowtheyaretreated.Youcaneliminatechoice(A)becauseanegativebehaviorchange(angry)doesn’tlogicallyfollowapositivetreatment(kindness).Likewise,youcaneliminatechoice(C)becauseapeacefulbehaviorchangeisnotlikelytofollowfromabuse.Finally,youcaneliminatechoice(D)becauseinterestedandmedicinehavenologicalconnection.Theonlyremainingchoiceis(B),whichfitsthesentenceandmustbethecorrectanswer.

WHATDOSMARTTEST-TAKERSKNOW?ThinkingUpYourOwnAnswerIstheWaytoStart

Robertwasextremely____whenhereceivedaBontheexam,forhewasalmostcertainhehadgottenanA.

(A)elated

(B)dissatisfied

(C)fulfilled

(D)harmful

Thecorrectansweris(B).Ifyoureadthissentencecarefully,youarelikelytocomeupwiththerightanswer,dissatisfied,onyourown,Butevenifyoudidn’t,you’dcomeupwithsomethingclose,suchasupsetordisappointed.Thenwhenyoulookattheanswerchoices,youwillimmediatelyseethattheclosestwordisdissatisfied.

NOTESomewordssignalblanksthatgowiththeflow:

•and

•also

•consequently

•asaresult

•thus

•hence

•so

•forexample

IdentifyClueWords

Ifyoucan’tcomeupwiththemissingwordimmediately,lookforcluewordsinthesurroundingsentence.Cluewordscantellyou“wherethesentenceisgoing.”Isitcontinuingalongonelineofthought?Ifitis,you’relookingforawordthatsupportsthatthought.Isitchangingdirectioninmidstream?Thenyou’relookingforawordthatsetsupacontrastbetweenthethoughtsinthesentence.

SomeBlanksGowiththeFlow

Themissingwordmaybeonethatsupportsanotherthoughtinthesentence,soyouneedtolookforananswerthat“goeswiththeflow.”

Theserviceattherestaurantwassoslowthatbythetimethesaladhadarrived

wewere____.

(A)ravenous

(B)excited

(C)incredible

(D)forlorn

Thecorrectansweris(A).Whereisthissentencegoing?Therestaurantserviceisveryslow.Thatmeansyouhavetowaitalongtimeforyourfood,andthelongeryouwait,thehungrieryou’llget.Sothewordintheblankshouldbesomethingthatcompletesthistrainofthought.Answerchoice(A),ravenous,whichmeansveryhungry,isthebestanswer.Itworksbecauseit“goeswiththeflow.”

Asateenager,Johnwaswithdrawn,preferringthecompanyofbookstothatofpeople;consequently,asayoungadultJohnwassocially____.

(A)successful

(B)uninhibited

(C)intoxicating

(D)inept

Thecorrectansweris(D).Thewordconsequentlysignalsthatthesecondideaisanoutcomeofthefirst;soagain,youarelookingforawordthatcompletesthetrainofthought.Whatmighthappenifyouspenttoomuchtimewithyournosestuckinabook(exceptforthisone,ofcourse)?Mostlikelyyouwouldbemorecomfortablewithbooksthanwithpeople.Choice(D),inept,meaningawkward,isagooddescriptionofsomeonewholackssocialgraces,makingthistherightanswer.

Adecisionthatismadebeforealloftherelevantdataarecollectedcanonlybecalled____.

(A)calculated

(B)laudable

(C)unbiased

(D)premature

Thecorrectansweris(D).Thewordcalledtellsyouthattheblankisthewordthattherestofthesentencedescribes.Adecisionthatismadebeforeallthefactsarecollectedcanonlybedescribedaspremature,choice(D).

SomeBlanksShiftGears

Themissingwordmaybeonethatreversesathoughtinthesentence,soyouneedtolookforananswerthat“shiftsgears.”

Theadvanceofsciencehasdemonstratedthatafactthatappearstocontradictacertaintheorymayactuallybe____amoreadvancedformulationofthattheory.

(A)incompatiblewith

(B)inoppositionto

(C)consistentwith

(D)eliminatedby

Thecorrectansweris(C).Lookatthelogicalstructureofthesentence.Thesentencehassetupacontrastbetweenwhatappearstobeandwhatisactuallytrue.Thisindicatesthatthecorrectanswerwill“shiftgears”andbetheoppositeofcontradict.Thechoiceconsistentwithprovidesthismeaning.Theotherchoicesdonot.

Althoughsheknewthattheartist’sworkwasconsideredbysomecriticstobe____,thecuratorofthemuseumwasanxioustoacquireseveraloftheartist’spaintingsforthemuseum’scollection.

(A)insignificant

(B)important

(C)desirable

(D)successful

Thecorrectansweris(A).Theveryfirstwordofthesentence,although,signalsthatthesentenceissettingupacontrastbetweenthecriticsandthecurator.Thecriticshadoneopinion,butthecuratorhadadifferentone.Sincethecuratorlikedtheartworkswellenoughtoacquirethem,youcananticipatethatthecriticsdislikedtheartworks.Sotheblankrequiresawordwithnegativeconnotations,andchoice(A),insignificant,istheonlyonethatworks.

Afterwitnessingseveralviolentinteractionsbetweentheanimals,theanthropologistwasforcedtoreviseherearlieropinionthatthemonkeyswere____.

(A)peaceable

(B)quarrelsome

(C)insensitive

(D)prosperous

Thecorrectansweris(A).Wheredoyoubegin?Thewords“forcedtorevise”clearlysignalashiftintheanthropologist’sideas.Herdiscoverythatthemonkeyswereviolentmadeherabandonanearliercontrastingopinion.Amongtheanswerchoices,theonlycontrasttoviolentischoice(A),peaceable.

NOTESomewordssignalblanksthatshiftgears:

•but

•yet

•although

•ontheotherhand

•incontrast

•however

•nevertheless

TheRightAnswerMustBeBothLogicalandGrammaticallyCorrect

Whenansweringsentencecompletionquestions,youcanalwayssimplytossoutanyanswerchoicesthatdonotmakesenseinthesentenceorthatwouldnotbegrammaticallycorrect.

Anadvocateofconsumerrights,Naderhasspentmuchofhisprofessionalcareerattemptingto____thefraudulentclaimsofAmericanbusiness.

(A)expose

(B)immortalize

(C)reprove

(D)import

Thecorrectansweris(A).Whatwouldyoudowithafraudulentclaim?Immortalizeit?Importit?Notlikely.Thesechoicesarenotlogical.Theonlylogicalansweris(A).Youwouldexposeafraudulentclaim.

Despitetheharshtoneofhercomments,shedidnotmeanto____anycriticismofyoupersonally.

(A)infer

(B)aim

(C)comply

(D)imply

Thecorrectansweris(D).Implymeans“suggestindirectly.”Choice(A),infer,isawordoftenconfusedwithimply.Itmeans“concludefromreasoningorimplication.”Aspeakerimplies;alistenerinfers.Choice(C),comply,meaning“obey,”makesnosenseinthiscontext.Choice(B),aim,ismorelikely,butitdoesn’tworkinthesentenceasgiven.Youmightsay,“shedidnotmeantoaimanycriticismatyou,”butyouwouldnotnormallysay,“shedidnotmeantoaimanycriticismofyou.”

ALERT!Rememberthatmorethanoneanswercanseemtomakesense,butthereisonlyonecorrectanswer.Makeyourbestguessbasedonthefullmeaningofthesentence.

Two-BlankQuestionsGiveYouTwoWaystoGetItRight

Whentherearetwoblanksinasentencecompletionquestion,youhavetwowaystoeliminateanswerchoices.Youcanstartwitheitherblanktoeliminatechoicesthatdon’twork.Sopicktheonethat’seasierforyou.Ifyoucaneliminatejustoneofthewordsinatwo-wordanswerchoice,thewholechoicewon’twork,soyoucantossitoutandgoon.

TESTYOURSELFQUIZZESTakethefollowingquizzestohelpyoudeterminewhatyourweaknessesmightbe.Clickherefortheanswers..

TestYourself1

Directions:Eachofthefollowingquestionsconsistsofanincompletesentencefollowedbyfourwordsorpairsofwords.Choosethewordorpairofwordswhich,whensubstitutedfortheblankspaceorspaces,bestcompletesthemeaningofthesentence.

1.TheSpanishdancerstampedherfeetand____therhythmwiththeclickof____.

(A)ignored...dice

(B)kept...cutlery

(C)accented...castanets

(D)diffused...aguitar

2.Abustlinghospitalfloorisnot____toagoodnight’ssleep.

(A)related

(B)conducive

(C)necessary

(D)productive

3.Thejobapplicantwasso____asheapproachedtheinterviewthathishandswere____andhiskneesshook.

(A)confident...agitated

(B)impressed...clean

(C)unhappy...stiff

(D)nervous...clammy

4.Aserioussideeffectofsomedrugsisrecurrent____.

(A)elucidations

(B)hallucinations

(C)formulations

(D)flagellations

5.Disgruntled____oftenbecomehighlyeffectivespies.

(A)defectors

(B)officers

(C)anarchists

(D)censors

6.IwishIcouldguaranteethatthemachineis____reliable,butintruthitsperformanceissomewhat____.

(A)invariably...sporadic

(B)often...skittish

(C)serially...erratic

(D)consistently...invincible

7.Heextolledthejuicysweetfruitasbeingnothinglessthan____.

(A)surreal

(B)saccharine

(C)cloying

(D)succulent

8.Themilitary____oftheburningbuildingsledtheinvestigatortoconcludethatthefireshadbeensetinanactof____.

(A)location...arson

(B)significance...sabotage

(C)stance...sobriety

(D)discipline...treason

9.Asthenameoftheprizewinnerwas_____,therunner-uplookedtotally______.

(A)extolled...exonerated

(B)awarded...devastated

(C)announced...crestfallen

(D)proclaimed...credulous

10.Theunexplodeddevicefoundrustingintheweedsmusthavebeena____.

(A)bomb

(B)decoy

(C)calamity

(D)dud

TestYourself2

Directions:Eachofthefollowingquestionsconsistsofanincompletesentencefollowedbyfourwordsorpairsofwords.Choosethewordorpairofwordswhich,whensubstitutedfortheblankspaceorspaces,bestcompletesthemeaningofthesentence.

1.Theprofessor____hadhadalonganddistinguishedcareerandwasheldinhigh____byherpeers.

(A)eminent...stratosphere

(B)emeritus...esteem

(C)lecturing...tribute

(D)triumphant...spirits

2.Afamouspersonwillsometimespreparehisown____tobeengravedonhistombstone.

(A)epitaph

(B)elegy

(C)epithet

(D)eulogy

3.Commonexamplesofverbalredundanciesincludeexpressionssuchas____poorand____natives.

(A)indignant...indolent

(B)indulgent...industrious

(C)insolvent...iniquitous

(D)indigent...indigenous

4.Theteacher’s____toneofvoice____thatthethreatwasnottobetakenseriously.

(A)gruff...belied

(B)sprightly...inferred

(C)jocular...implied

(D)languid...asserted

5.Collectingcansandbottlesandredeemingthemforthedepositprovidesameager____.

(A)squalor

(B)subsistence

(C)subsidy

(D)supplement

6.Recenttrialsofserialmurderersreveal____behavior,including____andcannibalism.

(A)lunatic...manipulation

(B)lethargic...massacre

(C)macabre...torture

(D)mercurial...grimaces

7.Asthebusroundedthecurve,it____violentlyandcausedsomestandeestolosetheirbalance.

(A)lunged

(B)overturned

(C)lurched

(D)plummeted

8.Theelderlygentlemanwasnoticeablyshakenwhenhereadhisown____inthelocalnewspaper.

(A)editorial

(B)obituary

(C)necromancy

(D)necrology

9.Wehavenarrowedourvacationoptionstotwodestinations,____,theFloridaKeysoraCaribbeanisland.

(A)infact

(B)both

(C)therefore

(D)namely

10.Thechildappearedtobetalland____,sohisfrequentabsencesforillnessledtheteachertosuspectthathewas____.

(A)fat...faking

(B)ruddy...sick

(C)robust...malingering

(D)lanky...truant

TestYourself3

Directions:Eachofthefollowingquestionsconsistsofanincompletesentencefollowedbyfourwordsorpairsofwords.Choosethewordorpairofwordswhich,whensubstitutedfortheblankspaceorspaces,bestcompletesthemeaningofthesentence.

1.Thespeakerrosefromhisseat,placedhisnotesonthe____,andbeganhisaddress.

(A)platform

(B)dais

(C)rostrum

(D)lectern

2.Thesenator’sattitudeso____thecommunitythatthevariousgroupscameto____oneanother.

(A)energized...contradict

(B)antagonized...respect

(C)polarized...despise

(D)divided...assist

3.Itis____totaperecordthetestimonyofawitnesssoastohavea(n)____record.

(A)advisable...verbatim

(B)illegal...accurate

(C)permissible...facsimile

(D)prudent...visible

4.Theprodigy’sgrandmotherfelt____pleasureathergrandchild’spianosuccess.

(A)violent

(B)vicarious

(C)virtuous

(D)vocal

5.Manypeopleofferedtoadoptthe________whohadbeenabandonedinthemall.

(A)rabid...mongrel

(B)crippled...cart

(C)winsome...waif

(D)fluffy...boa

6.The____costumesgreatlyincreasedthechildren’s____astheywitnessedtheirfirstballetperformance.

(A)elaborate...understanding

(B)grotesque...discomfort

(C)fanciful...delight

(D)comfortable...skill

7.Theconstructioncrewunexpectedlyunearthedtracesofa____unknownburialground.

(A)historically

(B)hitherto

(C)fortunately

(D)persistently

8.I____mygrandfather;tomeheisthe____ofagentleman.

(A)idolize...epitome

(B)despise...prototype

(C)adore...antithesis

(D)excuse...model

9.Theyoungsuspectwasreleasedonhisown____withthe____thatheremainsinschool.

(A)volition...caveat

(B)recognition...promise

(C)signature...prerequisite

(D)recognizance...proviso

10.Ifabrokenlegremainsinacasttoolong,themusclesmay____fromdisuse.

(A)petrify

(B)exacerbate

(C)atrophy

(D)putrefy

TestYourself4

Directions:Eachofthefollowingquestionsconsistsofanincompletesentencefollowedbyfourwordsorpairsofwords.Choosethewordorpairofwordswhich,whensubstitutedfortheblankspaceorspaces,bestcompletesthemeaningofthesentence.

1.Thejudge____thephysiciantorefrainfromusingtoomuchmedical____inhisstatement.

(A)admonished...jargon

(B)ordered...literature

(C)permitted...economics

(D)implored...jingoism

2.Iamnot____towalkingintherain,____ifgiventheoptionIwillride.

(A)accustomed...however

(B)opposed...therefore

(C)averse...nonetheless

(D)agreeable...moreover

3.Thepsychiatristfearedthatthesuspectmightcommitsuicide,sothesuspectwaskeptunder24-hour____.

(A)arrest

(B)constraints

(C)interrogation

(D)surveillance

4.Thestatement“You’reonlyyoungonce”isa(n)____.

(A)spoof

(B)truism

(C)adage

(D)motto

5.Thatricepuddingismost____;itlookslikea(n)____mass.

(A)delicious...gargantuan

(B)unappetizing...glutinous

(C)cloying...quivering

(D)uninspiring...esoteric

6.Withthecurrentriseof____feeling,____arebeingkeptbusydelineatingtheshiftingboundaries.

(A)entrepreneurial...artists

(B)antagonistic...governments

(C)cooperative...economists

(D)nationalistic...cartographers

7.Isatintheparkandlistenedtothe_____fromthenearbychurchtower.

(A)carillon

(B)harangue

(C)calliope

(D)sycophant

8.Inthehotsummermonths,ourcity’sairisoften____withthe____murkofpollution.

(A)endowed...luminous

(B)infused...negligible

(C)suffused...oppressive

(D)alleviated...aromatic

9.Couldyousupplysomeexamplesto____yourargument?

(A)refute

(B)bolster

(C)destroy

(D)exaggerate

10.Ifearthedisciplineinthathouseholdisso____thatthechildrenwillgrowupwithoutcharacter.

(A)rigid

(B)ludicrous

(C)attainable

(D)lax

TestYourself5

Directions:Eachofthefollowingquestionsconsistsofanincompletesentencefollowedbyfourwordsorpairsofwords.Choosethewordorpairofwordswhich,whensubstitutedfortheblankspaceorspaces,bestcompletesthemeaningofthesentence.

1.Thatstrainofcancerisso____thatnoamountoftreatmentcan____it.

(A)abundant...describe

(B)capricious...enhance

(C)virulent...destroy

(D)arcane...cure

2.Heisavery____person,alwayscarryingarabbit’sfootinhispocketasa(n)____.

(A)timorous...omen

(B)superstitious...talisman

(C)gullible...charm

(D)conscientious...keyring

3.Sometimestheeffectofa____mealistomakethedinerverysleepy.

(A)hearty

(B)haughty

(C)healthy

(D)hasty

4.The____gardenis____andfragrant.

(A)formal...overgrown

(B)artificial...infested

(C)public...forbidding

(D)vernal...verdant

5.The____vendingmachinespewedchangealloverthefloor.

(A)recalcitrant

(B)zealous

(C)hulking

(D)rambunctious

6.Thefiretrucksarrivingatthesceneofthe____createdamassivetrafficjam.

(A)convocation

(B)simulation

(C)conflagration

(D)mutiny

7.A____tendstorelyon____evidence.

(A)visionary...prodigious

(B)pragmatist...empirical

(C)jury...spurious

(D)scientist...theoretical

8.IampleasedthatIwillbeabletogoswimmingwearingmy____watch.

(A)aquamarine

(B)digital

(C)liquefied

(D)submersible

9.Caringforyoungersiblingscanbea(n)____responsibilityforthechildofa(n)____.

(A)onerous...alcoholic

(B)preordained...acrobat

(C)negligible...supplicant

(D)appreciable...parent

10.Theclergymanwassohighlyrespectedthatnoonecouldbelievethathewasa(n)____.

(A)transgressor

(B)hemophiliac

(C)executor

(D)altruist

ANSWERKEYSTestYourself1

1.C2.B

3.D4.B

5.A6.A

7.D8.B

9.C10.D

TestYourself2

1.B2.A

3.D4.C

5.B6.C

7.C8.B

9.D10.C

TestYourself3

1.D2.C

3.A4.B

5.C6.C

7.B8.A

9.D10.C

TestYourself4

1.A2.C

3.D4.B

5.B6.D

7.A8.C

9.B10.D

TestYourself5

1.C2.B

3.A4.D

5.D6.C

7.B8.D

9.A10.A

EXERCISES:SENTENCECOMPLETIONS

Directions:Choosethecorrectanswerfromthechoicesgiven.

1.Histheoryisnot____;itonlysoundsplausibletotheuninformedbecausehe____severalfactsandfailstomentionthemountainofevidencethatcontradictshisideas.

(A)tenable...distorts

(B)pliable...pursued

(C)predominant...embellished

(D)sufficient...invokes

2.Thatorganization____itsconcernforendangeredspeciesofwildlifebyencouragingcongresspeopletopasslawsthat____theseanimals.

(A)imposes...promote

(B)manipulates...defend

(C)manifests...protect

(D)supplants...prohibit

3.Hisfilingsystemwasso____thatnooneelseinthedepartmentcouldlocatematerialquickly.

(A)specific

(B)appropriate

(C)intense

(D)peculiar

4.Whenthelastitemonthe____hadbeentakencareof,themeetingwas____.

(A)roster...calledtoorder

(B)itinerary...finalized

(C)table...sequestered

(D)agenda...adjourned

5.Ithasbeenpredictedthatthenew____barringdiscriminationinemploymentonthebasisofsexualorientationwilldramatically____hiringpractices.

(A)morality...effect

(B)permissiveness...reflect

(C)legislation...affect

(D)rulings...reset

6.TheNavyscouredtheareaforoveramonth,butthe____searchturnedupnoclues.

(A)cursory

(B)fruitful

(C)present

(D)painstaking

7.Althoughherpersonalityissometimes____,sheisaconscientiousworkerandis____bettertreatmentthanshehasreceived.

(A)pleasing...consciousof

(B)abrasive...entitledto

(C)gloomy...eligiblefor

(D)cheerful...granted

8.____manipulationofthestockmarketandother____practicesinsecuritysalesresultedinthe1933legislationforthecontrolofsecuritymarkets.

(A)Degenerate...lucrative

(B)Economic...useless

(C)Continual...productive

(D)Unscrupulous...unethical

9.Thehandbook____forbeginnerswaswritteninanelementarystyle.

(A)bound

(B)intended

(C)paged

(D)authored

10.Whenajobbecomestoo____,workersget____,theirattentionwanders,andtheystarttomakecarelesserrors.

(A)diverse...busy

(B)hectic...lazy

(C)tedious...bored

(D)fascinating...interested

11.Becauseofheruncompromisingstandsondivisiveissues,shewasunableto____broadsupportamongthevoters;however,theminoritywhodidsupportherwereexceptionally____.

(A)alienate...many

(B)survey...divided

(C)cutacross...quiet

(D)amass...loyal

12.Hisremarksweretoo____tobetakenseriously.

(A)germane

(B)crucial

(C)pointed

(D)insipid

13.Notrainingcoursecanoperatetofulladvantagewithoutjobdescriptionsthat____thosepartsofthejobthatrequirethemosttrainingbeforethetrainingcourseis____.

(A)list...improved

(B)identify...implemented

(C)teach...predicted

(D)insulate...finished

14.Sincethecoursewasnotonly____butalsohadareputationforbeingextremelydifficult,____studentsregisteredforit.

(A)enjoyable...many

(B)required...some

(C)useful...practical

(D)optional...few

15.Thenewsecretaryhasamorebusinesslikemannerthanher____inthejob.

(A)precedent

(B)ancestor

(C)successor

(D)predecessor

16.Becauseofthe____hazard,regulationsforbidtheuseofhighly____materialsincertainitemssuchaschildren’spajamas.

(A)health...synthetic

(B)fire...flammable

(C)drug...inflammatory

(D)chemical...flame-retardant

17.The____reportwassubmitted,subjecttosuch____aswouldbemadebeforethefinaldraft.

(A)preliminary...revisions

(B)ubiquitous...submissions

(C)ultimate...editions

(D)committee’s...references

18.____actiononthepartofapasserbyrevivedthevictimbeforebraindamagecouldoccur.

(A)Physical

(B)Prompt

(C)Violent

(D)Delayed

19.Astheworkload____,she____responsibilityformanyroutinetaskstoanassistant.

(A)evolved...preserved

(B)changed...handled

(C)increased...delegated

(D)steadied...abased

20.Formanyyears____havebeenrecognizedasbreedingdisease,juveniledelinquency,andcrime,whichnotonlythreatenthehealthandwelfareofpeoplewholivetherebutalso____thestructureofsocietyasawhole.

(A)prisons...rebuild

(B)schools...disengage

(C)colonialization...alienate

(D)slums...weaken

21.Ascitizenswewouldbe____ifwedidnotmakethesefactspublic.

(A)entitled

(B)nominative

(C)elective

(D)derelict

22.A____inthediplomaticservice,shehadnotyet____suchaquestionofprotocol.

(A)success...dispatched

(B)volunteer...avoided

(C)veteran...battered

(D)novice...encountered

23.Excessivefatiguecan____beattributedto____workingconditionssuchaspoorlighting.

(A)inevitably...archaic

(B)occasionally...inadequate

(C)always...obsolete

(D)never...demoralizing

24.Thecompanyreceiveda____fromthegovernmenttohelpdevelopnewsourcesofenergy.

(A)reward

(B)compendium

(C)memorandum

(D)subsidy

25.The____withwhichtheagentcalmedtheanxietiesandsoothedthetempersofthetravelers____bythedelaywasamarkoffrequentexperiencewithsimilarcrises.

(A)evasiveness...angered

(B)reverence...pleased

(C)facility...inconvenienced

(D)mannerism...destroyed

26.Thegreaterthe____ofamineralinanore,thelessitcoststorefineit.

(A)expense

(B)weight

(C)oxidation

(D)concentration

27.Thechurchhadtraditionallyservedasa____fordebtors,andthose____itweresafefromprosecution.

(A)prison...leaving

(B)blessing...obeying

(C)court...denying

(D)sanctuary...entering

28.Today’sstudentsareencouragedtoabsorbfactsratherthantoapply____.Educationisbecoming____.

(A)understanding...regrettable

(B)intelligence...invaluable

(C)knowledge...passive

(D)formulas...extensive

29.Man’ssurvivalisaresultofmutualassistance,sinceheisessentially____ratherthan____.

(A)superior...inferior

(B)cooperative...competitive

(C)individualistic...gregarious

(D)selfish...stingy

30.AncientGreekswerenotonlyconcernedwiththedevelopmentofthe____butalsofelttrainingofthebodywasof____importance.

(A)muscles...equal

(B)psyche...little

(C)mind...prime

(D)physical...vital

31.Althoughforyears____resourceshadbeendevotedtoalleviatingtheproblem,asatisfactorysolutionremained____.

(A)natural...costly

(B)adequate...probable

(C)substantial...elusive

(D)capital...decisive

32.Thepolicedepartmentwillnotacceptfor____areportofapersonmissingfromhisorherresidenceifsuch____islocatedoutsideofthecity.

(A)convenience...location

(B)control...report

(C)filing...department

(D)investigation...residence

33.Thetreatycannotgointoeffectuntilithasbeen____bytheSenate.

(A)considered

(B)debated

(C)ratified

(D)shelved

34.His____ofpracticalexperienceandhispsychologicalacuitymorethan____hislackofformalacademictraining.

(A)claims...comprise

(B)background...educatefor

(C)brief...accountfor

(D)wealth...compensatefor

35.BecauseIwantedtousea(n)____,Ilookedthewordupinthe____.

(A)synonym...thesaurus

(B)homonym...directory

(C)antonym...encyclopedia

(D)pseudonym...dictionary

36.Youwillhavetospeaktotheheadofthedepartment;Iamnot____togiveoutthatinformation.

(A)willing

(B)authorized

(C)programmed

(D)happy

37.Researchinthatfieldhasbecomeso____thatresearchersondifferentaspectsofthesameproblemmaybe____eachother’swork.

(A)secure...bombardedwith

(B)partial...surprisedat

(C)departmental...inimicalto

(D)specialized...unfamiliarwith

38.She____thewaythingsweredone,butmanyofthe____forwhichshebrokegroundwerelefttobefullyrealizedbyothers.

(A)disliked...provocations

(B)eliminated...foundations

(C)implemented...buildings

(D)revolutionized...innovations

39.Achangeinenvironmentisverylikelyto____achangeinone’sworkhabits.

(A)affect

(B)inflict

(C)propose

(D)effect

40.Ashifttogreateruseof____orinexhaustibleresourcesintheproductionofpowerwouldslowthedepletionof____fuelmaterials.

(A)synthetic...regional

(B)natural...chemical

(C)renewable...irreplaceable

(D)unknown...fossil

41.A____islikelytogiveyou____advice.

(A)fool...useful

(B)doctor...lethal

(C)friend...harmful

(D)charlatan...unreliable

42.Anaccidentreportshouldbewrittenassoonaspossibleafterthenecessary____hasbeenobtained.

(A)bystander

(B)formulation

(C)information

(D)permission

43.Toprotecttherespondents’____,namesandsocialsecuritynumbersare____thequestionnairesbeforetheresultsaretabulated.

(A)privilege...referredto

(B)privacy...deletedfrom

(C)information...retainedin

(D)rights...appendedto

44.Whilefewerdocumentsarebeingkept,theusefulnessofthose____isnow____byanimprovedcatalogingsystem.

(A)printed...documented

(B)discarded...concurred

(C)read...emblazoned

(D)retained...insured

45.Theman____thespeakeratthemeetingbyshoutingfalseaccusations.

(A)corrected

(B)arguedwith

(C)disconcerted

(D)interferedwith

46.Forthesakeofpublic____,publicofficialsshouldavoideventhe____ofaconflictofinterest.

(A)confidence...appearance

(B)relations...actuality

(C)appearances...apparition

(D)commotion...hint

47.Aprofessionaljournalistwillattemptto____thefactslearnedinaninterviewbyindependent____.

(A)endorse...questions

(B)query...situation

(C)garnish...sources

(D)verify...investigation

48.The____oftheawardstoppedbythefinancialaidofficetopickuphischeck.

(A)recipient

(B)subject

(C)donor

(D)sponsor

49.Heoften,outofmodesty,____hisowncontribution;withouthisefforts,____,theprogramwouldstillbeintheplanningstage.

(A)affirms...therefore

(B)represses...notwithstanding

(C)belittles...however

(D)rescinds...moreover

50.Thecommitteewasso____aboutthelegitimatesourcesofthestudents’unrestthatitsrecommendationswere____value.

(A)incensed...ofmoderate

(B)uninformed...devoidof

(C)uninterested...depreciatingin

(D)blasé...ofincontestable

ANSWERSANDEXPLANATIONS

1.Thecorrectansweris(A).Thefirstblankmightbefilledequallywellbythefirsttermofchoice(A)orchoice(D);however,thecoordinating“and”impliesthatthesecondblankmustbefilledwithsomethingnegativethathedoestothefacts.DISTORTSisthebestwordhere.

2.Thecorrectansweris(C).Choice(D)makesnosense.“Encouraging”isnotacoercivetypeofactivity,sochoices(A)and(B)wouldbetooforcefulascompletionsinthefirstblank.

3.Thecorrectansweris(D).Ifnooneelsecouldlocatethematerial,youmaybeprettysurethathisfilingsystemwasPECULIAR.

4.Thecorrectansweris(D).ThelistofitemsforconsiderationatameetingistheAGENDA.Whenthebusinessiscompleted,onemightaswellADJOURNthemeeting.

5.Thecorrectansweris(C).Thebarringofdiscriminationisanofficialact,soonlychoice(C)orchoice(D)couldfillthefirstblank.Thesecondblankisbestfilledwiththeidiomatic“AFFECThiringpractices.”

6.Thecorrectansweris(D).AsearchthatlastsmorethanamonthismostcertainlyaPAINSTAKINGone.

7.Thecorrectansweris(B).Thefirstblankcallsforanegativetraittocontrastwithherconscientiousness.Choices(B)and(C)mightbothbecorrect,butENTITLEDTObetterfitstheinformalityofthesentence.“Eligiblefor”impliesalegalrequirement.

8.Thecorrectansweris(D).Thecoordinating“and”inthecompoundsubjectrequiresthatbothwordshavethesameconnotation.Sincetheseactsledtotheimpositionofcontrols,wemustassumethattheywerenegativeacts.

9.Thecorrectansweris(B).ThestyleofamanualmustbeappropriatetotheaudienceforwhichitisINTENDED.

10.Thecorrectansweris(C).Thekeyhereisthattheworkers’attentionwanders.AttentionwanderswhenoneisBORED.OnebecomesBOREDwhentheworkisTEDIOUS.

11.Thecorrectansweris(D).OnlyAMASSreallymakessenseinthefirstblank.

12.Thecorrectansweris(D).AnyremarksotherthanstupidorINSIPIDonesshouldbetakenseriously.

13.Thecorrectansweris(B).Thesecondblankcanbefilledonlybychoice(B)orchoice(D).Choice(D)makesnosenseinthefirstblank.

14.Thecorrectansweris(D).The“notonly...butalso”constructionimpliestwocomplementaryreasonswhyaclassificationofstudentsmightregisterforthecourse.Onlychoice(D)reallyfitsthisrequirement.AcoursethatisOPTIONALandverydifficultwilldrawFEWregistrants.

15.Thecorrectansweris(D).ThepersonwhoheldthisjobbeforethecurrentsecretarywasherPREDECESSOR.

16.Thecorrectansweris(B).Onlychoice(A)orchoice(B)makessenseinthefirstblank.Syntheticsarenotinthemselvesahealthhazard.

17.Thecorrectansweris(A).Thereportwassubmittedbeforethefinaldraft.Onlychoice(A)orchoice(D)candescribethereport.Areportisnotmadesubjecttoreferences;itismadesubjecttoREVISIONSbeforethefinaldraft.

18.Thecorrectansweris(B).Sincetheactionwastakenbeforebraindamagecouldoccur,thebestcompletionimpliesspeed.

19.Thecorrectansweris(C).ThebestthingtodowithexcessresponsibilityistoDELEGATEit.

20.Thecorrectansweris(D).Thesecondblankmustbefilledbyanegativeword.Thislimitstheanswertochoices(B),(C),and(D).Ofthethreefirst-termchoices,onlySLUMSarerecognizedasthebreedinggroundsofdisease,juveniledelinquency,andcrime.

21.Thecorrectansweris(D).Thestructureofthesentenceleavesonlychoice(D)asasensiblecompletion.

22.Thecorrectansweris(D).Thewords“notyet”implythatshewasnewtooraNOVICEinthediplomaticservice.

23.Thecorrectansweris(B).Excessivefatiguecanoftenbeattributedtofactorsotherthaninhospitableworkingconditions,butOCCASIONALLY,INADEQUATEworkingconditionsareitscause.

24.Thecorrectansweris(D).Moneyhelpsinthedevelopmentofnewprocessesandproducts.ASUBSIDYismoneyreceivedinadvanceofthework.Rewardsandhonorariafollowaservice.

25.Thecorrectansweris(C).Theblankscouldbefilledwithchoices(A)or(C).However,frequentexperienceshouldleadtoFACILITYinsoothingINCONVENIENCEDtravelers,sochoice(C)isthebestanswer.

26.Thecorrectansweris(D).ItisreasonabletoassumethatamoreCONCENTRATEDorewouldyieldagreaterquantityofthemineralandwouldthusbemorecostefficient.

27.Thecorrectansweris(D).Theword“and”connectingthetwoclausesimpliesthatsafetyfromprosecutionappliestothechurch.ASANCTUARYisaplaceforrefugeandprotection,andallwhoENTERitaresafe.

28.Thecorrectansweris(C).AbsorptionoffactsisPASSIVE,asopposedtothemoreactivemodeofeducation,theapplicationofKNOWLEDGE.

29.Thecorrectansweris(B).Thesentence,bythewords“ratherthan,”requiresthatthetwowordsfillingtheblanksbeopposites.Onlychoice(D)doesnotmeetthisrequirement.However,mutualassistanceimpliesCOOPERATION;hencechoice(B)isthecorrectanswer.

30.Thecorrectansweris(C).Thesentencerequiresthatthefirstblankbefilledbyawordthatcontrastswithbody.Ofthechoices,MIND,choice(C),bestmeetsthiscriterion.

31.Thecorrectansweris(C).Theconstructionofthesentencedemandsthatthefirstblankbefilledwithapositivewordwhilethesecondisfilledwithalesspositiveword.Choice(C)bestfitstheserequirements.

32.Thecorrectansweris(D).Amissing-personreportdemandsINVESTIGATION.

33.Thecorrectansweris(C).Alltheconsideration,amending,anddebatewillnotputatreatyintoeffectuntilitisRATIFIED.

34.Thecorrectansweris(D).Thewordsintheblanksshouldcontrastwithhislackofformalacademictraining.

35.Thecorrectansweris(A).TheTHESAURUSisabookofSYNONYMS.

36.Thecorrectansweris(B).Allchoicesexcept(C)mightbecorrect,buttheimperativeof“youwillhaveto”impliesthatIamnotAUTHORIZED.

37.Thecorrectansweris(D).Noenmityisimpliedinthissentence,sochoice(D)isabetteranswerthanchoice(C).

38.Thecorrectansweris(D).Onemightbreakgroundforchoices(B),(C),or(D),butonly

INNOVATIONSforwhichonebreaksgroundarefullyrealizedbyothers.

39.Thecorrectansweris(D).ToEFFECTistocause.Inflicthasanegativeconnotationthatisuncalledforinthissentence.

40.Thecorrectansweris(C).Fuelmaterialsthatcouldbedepletedareeitherchoice(C)orchoice(D).Onlychoice(C)fitsintothefirstblank.TheuseofRENEWABLEresourceswouldslowthedepletionofIRREPLACEABLEfuelmaterials.

41.Thecorrectansweris(D).Thetypeofadvicemustbeappropriatetothegiver.TheadviceofaCHARLATANorimposterislikelytobeUNRELIABLE.

42.Thecorrectansweris(C).AnaccidentreportshouldbedependentonlyuponINFORMATIONandnotuponpermissionorformulation.

43.Thecorrectansweris(B).DELETIONofidentificationinsuresPRIVACY.

44.Thecorrectansweris(D).Iffewerdocumentsarebeingkept,weareprobablydiscussingthosethatareRETAINED.TheirusefulnessisINSUREDbyanimprovedcatalogingsystem.

45.Thecorrectansweris(C).Themanwouldhaveinterferedwiththespeakerevenifhehadshoutedwordsofagreement.Sincethepointismadethatthemanshoutedfalseaccusations,thebestansweristhatheDISCONCERTEDthespeaker.

46.Thecorrectansweris(A).Thesecondblankcouldbefilledwithchoice(A)orchoice(D);however,thefirsttermofchoice(D)makesnosenseinthefirstblank.

47.Thecorrectansweris(D).VERIFICATIONisattheheartofprofessionaljournalism.

48.Thecorrectansweris(A).OnlytheRECIPIENTwouldpickupthecheck.

49.Thecorrectansweris(C).ModestywouldleadonetoBELITTLEone’srole.TheconnectiveHOWEVERmakesthebesttransitionbetweenthetwoclauses.

50.Thecorrectansweris(B).Thesecondblankmightbefilledbyallchoicesexcept(C).However,thetwotermsofchoice(D)makenosenseinapposition.Choice(A)isnotthecorrectanswer.Thecommitteemighthavebeenincensedaboutthestudents’unrestbutitwouldbeunlikelytobeincensedaboutthelegitimatecausesofthatunrest.IfitwereUNINFORMEDofthelegitimatecauses,itsrecommendationswouldbeDEVOIDOFvalue.

SUMMINGITUP

•Thesestepswillhelpyouasyouworkthroughthissection:readthesentencecarefully;guessattheanswer;scantheanswerchoicesforthewordyouguessed(ifit’sthere,markitandgoon;ifit’snot,examinethesentenceforcluestothemissingword);eliminateanyanswerchoicesthatareruledoutbytheclues;andtrytheonesthatareleftandpickwhicheverisbest.

•Bealertforcluesinthesentence.Lookfornegativewordsorpositivewords.•Sentencecompletionquestionsgofromeasytohard.

PARTVREADINGREVIEW

CHAPTER9:ReadingComprehension(SSATandISEE)

Chapter9

ReadingComprehension(SSATandISEE)

OVERVIEW

•Whyismyreadingabilitybeingtested?•Whatkindsofquestionswillbeasked?•Howdoyouanswerreadingcomprehensionquestions?•Whatdosmarttest-takersknow?•Testyourselfquizzes•Answerkeys•Summingitup

WHYISMYREADINGABILITYBEINGTESTED?Readingisaprerequisitetolearning,anditisevaluatedbyallaptitude,admissions,andachievementtests.

WHATKINDSOFQUESTIONSWILLBEASKED?Thequestionsthatfolloweachpassageareinthestandardmultiple-choiceformatwitheitherfour(ISEE)orfive(SSAT)answerchoices.Onhighschoolentranceexams,thequestionstendtofallintofourcategories.Thesequestionsaskyoutodooneofthefollowing:

•Identifythemainideaortheauthor’spurpose•Locatedetailsthatsupportthemainidea

•Defineawordbasedonitsmeaninginthepassage•Drawinferencesfromideasinthepassage

Otherreadingquestiontypesappearonbothtests,butthesefourarethemostcommon.

MainIdea

Thistypeofquestionpresentsseveraltitlesorphrasesandasksyoutochoosetheonethatbestexpressesthemainideaofthepassage.Mainideasoftencanbefoundinatopicsentence.Topicsentencesusuallyappearinthefirstparagraph,aspartoftheintroduction,orinthelastparagraph,asasummary.

Thesocialstandingofawifeincolonialdayswasdeterminedbythestandingofherhusbandaswellasbyherownabilityandresourcefulness.Shemarriednotonlyahusbandbutalsoacareer.Herpositioninthecommunitywasestablishedinpartbythequalityofthebreadshebaked,bythefoodshepreservedforthewinter’suse,bythewhitenessofherwashingontheline,bythewayherchildrenwereclothed,andbyherskillinnursing.Doctorswerescarce.Incaseoftheillnessordeathofaneighbor,awomanwouldputasideherownworktohelp,andshewashonoredforwhatshecoulddo.

Whichtitlebestexpressesthemainideaofthisselection?

(A)“CareofChildreninColonialTimes”

(B)“CommunitySpirit”

(C)“MedicalCareinPre-RevolutionaryTimes”

(D)“TheColonialHousewife”

Thecorrectansweris(D).Thisselectiondescribesthevarioushomemakingdutiesacolonialwomanwasexpectedtoperform.

Details

Detailsarethefactsandideasinaselectionthatexplainandsupportthemainidea.

Therearemanysignsbywhichpeoplepredicttheweather.Someofthesehave

atruebasis,butmanyhavenot.Thereis,forexample,noevidencethatitismorelikelytostormduringonephaseofthemoonthanduringanother.IfithappenstorainonEaster,thereisnoreasontothinkthatitwillrainforthenextsevenSundays.ThegroundhogmayormaynotseehisshadowonGroundhogDay,butitprobablywon’taffecttheweatheranyway.WhichofthefollowingisNOTlistedasapredictiveweatherphenomenon?

(A)RainonEaster

(B)Thephasesofthemoon

(C)Paininaperson’sjoints

(D)Thegroundhog’sshadow

Thecorrectansweris(C).Theotherchoiceswerementionedinthepassage.

Vocabulary

Thistypeofquestion,sometimescalled“wordsincontext,”asksyoutochooseasynonymforoneofthewordsinthepassage.

Themaritimeandfishingindustriesfindperhaps250applicationsforropeandcordage.Therearehundredsofdifferentsizes,constructions,tensilestrengths,andweightsinropeandtwine.Ropeissoldbythepoundbutorderedbylengthandismeasuredbycircumferenceratherthanbydiameter.

Inthiscontext,thewordapplicationsmeans

(A)uses.

(B)descriptions.

(C)sizes.

(D)types.

Thecorrectansweris(A).Tryitinthesentenceinplaceofthewordapplication:“Uses”makessenseinthatcontextandkeepsthemeaningofthesentenceintact.

Inference

Aninferenceisaconclusionthatisdrawnfromthedetailsinareadingselection.Theanswertoaninferentialquestionwillnotbefoundinthepassageandisthereforethemostdifficulttypeofcomprehensionquestiontoanswer.Youmustreadcarefullyandthinklogicallyinordertodrawthecorrectconclusionfromtheinformationgiven.

Thefacts,asweseethem,ondruguseandthedangerousbehaviorscausedbydrugsarethatsomepeopledogetintotroublewhileusingdrugs,andsomeofthosedrugusersaredangeroustoothers.Sometimesadrugisanecessaryelementinorderforapersontocommitacrime,althoughitmaynotbethecauseofhisorhercriminality.Ontheotherhand,theuseofadrugsometimesseemstobetheonlyconvenientexplanationbymeansofwhichtheobservercanaccountfortheundesirablebehavior.

Theauthorapparentlyfeelsthat

(A)theuseofdrugsalwaysresultsincrime.

(B)drugsandcrimeareonlysometimesrelated.

(C)therelationshipofdrugusetocrimeispurelycoincidental.

(D)drugsareusuallyanelementinaccidentsandsuicides.

Thecorrectansweris(B).Theauthorstatesthatdrugsaresometimesanecessaryelementinacrime,butatothertimesisjustanexcuseforcriminalbehavior.

TIPDon’tgofortheordinary.Vocabulary-in-contextquestionsseldomtestthemostcommonmeaningofaword.Lookforameaningthatisnottheusualone.

HOWDOYOUANSWERREADINGCOMPREHENSIONQUESTIONS?Successwithreadingcomprehensionquestionsrequiresbothspeedinreadingandefficiencyin

answeringquestions.

Twoimportanttechniquesthatyoumustmasterare:SkimmingScanning

Skimming

Skimmingisaformofspeed-readingthatisusefulforextractingthemainideaandsupportingdetailsfromareadingselection.Asyouskimapassage,payspecialattentiontothefirstandlastsentencesorparagraphs.Thepurposeofskimmingistolocatethetopicsentence,themainidea,andsomeofthemajorsupportingdetails.Thisoverviewofthelocationofinformationwithinthepassagewillhelpyoutoanswerthemoredifficultinferencequestionsquickly.

Scanning

Scanningisamethodoflookingforspecificinformationwithouttrulyreadingbutbylookingforkeywords.Thefollowingsixstepswillhelpyouanswerreadingcomprehensionquestions.

SixStepsforReadingComprehension:GettingitRight

Readthroughthequestionsquickly.Thiswillguideyourreadingbyshowingyouwhatinformationyouwillbeexpectedtofind.Skipovertheanswerchoicesfornow.Readthepassage.Answervocabularyquestionsfirst.Findtheanswersbyscanningthepassage.Answerdetailquestionsnext.Pickakeywordortwofromthequestionitselfandscanthepassageuntilyoufindit.Thesentenceinwhichthewordappearsprobablycontainstheanswertothequestion.Answermain-ideaquestionsbyreadingthefirstandlastsentencesofthepassage.Leaveinferencequestionsandothermoredifficultquestiontypesforlast.Skimthepassage.Eliminatechoicesthatareobviouslywrong.Takeyourbestguess.

WHATDOSMARTTEST-TAKERSKNOW?Readingcomprehensionquestionscaneatupyourtimeveryquickly.Checkoutthesetipsforsmartersolutions.

YouOnlyHaveTimetoReadEachPassageOnce

Becausethere’sonlytimetoreadeachpassageonce,you’llwanttoanswereveryquestionthatyoucanaboutthepassagebeforemovingon.Ifyouskipaquestionandtrytocomebacktoitlater,youmighthavetorereadthewholepassagetofindtheanswerandyou’llbeoutoftime.Guessifyouhaveto,butfinishallthequestionsthatyoucan.

EverythingYouNeedtoKnowIsRightThereinFrontofYou

Theintroductoryparagraphandthepassagehavealltheinformationyou’llneedtoanswerthequestions.EvenifthepassageisaboutthepriceofbeansinBulgariaorthegeneticmakeupofawombat,don’tworry.It’sallrightthereonthepage.

NOTEReadingcomprehensiontakesalotoftime,butthereisawaytogofaster.Readthequestionsfirstsoyouknowwhattolookforasyoureadthepassage.Thenyouwon’thavetoreadthingstwicetofindtheinformationyouneed.

ThePassagesAreSupposedtoBeUnfamiliar

Inordertoputallcandidatesonalevelplayingfield,test-makerschooseobscurereadingpassages.ISEEpassagesfocusonsocialscience,naturalscience,andthehumanities;SSATpassagesfocusonthesesubjectsaswellasfiction.Eitherway,youprobablyhavenotseenthereadingmaterialbefore,anditdoesn’tmatter.Remember,you’renotbeingtestedonyourknowledgeofthetopicbutonhowwellyou:

•Figureoutthemeaningofanunfamiliarwordfromitscontext•Determinewhatanauthormeansbynotingcertainwordsandphrases•Understandtheauthor’sassumptions,pointofview,andmainidea

ALERT!Don’tletunfamiliartopicsthrowyou.There’snoneedtoworrywhetheryouknowanythingaboutatopicinapassage.Theanswersarebasedontheinformationinthepassage,notonyourknowledgeorexperience.

PassagesThatInterestYouAreEasierforYoutoWorkOn

Ifthere’sachoice,it’sbesttostartwiththepassagethat’smoreinterestingtoyou,whetherit’sfiction,asciencearticle,orwhatever.Ifthestyleappealstoyou,youwillprobablygothrough

thepassagemorequicklyandfindthequestionseasiertodealwith.

ItPaystoBeanActiveReader

Sinceyou’vealreadyscannedthequestions,youknowwhattolookforasyouread.Whenyoufindthesepoints,useyourpenciltounderlineorcirclethem.You’llbeabletofindthemeasilywhenyouneedthemtoanswerthequestions.

TIPCompleteonepassageatatime.Answerallthequestionsforthepassageyouhavejustreadbeforemovingtoanotherpassage.Thereisnotimetogobackandreadagain.

DetailsCanBogYouDown

Remember,youdon’thavetounderstandeverybitofinformation.Youjusthavetofindtheinformationyouneedtoanswerthequestions.Don’twasteyourtimeontechnicaldetailsoroninformationthatthequestionsdon’taskfor.

What’sTrueIsNotNecessarilytheAnswer

Whatdoesthatmean?Itmeansthatacertainanswerchoicemaybeperfectlytrue,butitmightnotbethecorrectanswertothequestionthat’sbeingasked.Readcarefully—anddon’tbefooled!

YouCanSolveVocabulary-in-ContextQuestionsbyPlugginginChoices

Forvocabulary-in-contextquestions,plugthechoicesintotheoriginalsentenceanddon’tbefooledbytheobvioussynonym.

TheAnswertoaMainIdeaQuestionIsNeitherTooGeneralNorTooSpecific

Foraquestionaboutthemainideaortheauthor’spurpose,lookforananswerchoicethatstatesit.Don’tbetoogeneralortoospecific.

ALERT!

Truedoesn’tmeancorrect!Besurethattheansweryoumarkistheanswertothequestionthatisasked.

TheAnswertoaMainIdeaQuestionIsOftenintheFirstorLastParagraph

Lookinthefirstorlast(orboth)paragraphofthepassageforanswerstomainidea/author’spurposequestions.

YouHavetoReadBetweentheLines

Whenareadingcomprehensionquestionasksforsomethingtheauthorhassuggested,implied,ornotstateddirectly,youhavetousetheinformationinthepassageanddrawyourownconclusions.Readbetweenthelinestoseeiftheauthorhasgivenanyhintsthatwouldleadyoutothecorrectanswer.

TESTYOURSELFQUIZZESTakethefollowingquizzestohelpyoudeterminewhatweaknessesyoumighthave.AnswerscanbefoundonpagesClickherefortheanswers.

TestYourself1

Directions:Readthefollowingpassageandthendecidewhichoftheresponsesisthebestanswertoeachquestion.

Earlyinthenineteenthcentury,Americanyouthswereplayingagame,somewhatliketheEnglishgameofrounders,whichcontainedalltheelementsofmodernbaseball.Itwasneitherscientificallyplannednorskillfullyplayed,butitfurnishedconsiderableexcitementforplayersandspectatorsalike.Theplayingfieldwasasixty-footsquarewithgoals,orbases,ateachofitsfourcorners.Apitcherstationedhimselfatthecenterofthesquare,andacatcherandanindefinitenumberoffielderssupportedthepitcherandcompletedtheteam.Noneoftheseplayers,usuallybetweeneightandtwentyonaside,coveredthebases.Thebatterwasoutonballscaughtontheflyorthefirstbound,andabaserunnerwasoutifhewashitbyathrownballwhileoffbase.Thebatwasnothingmorethanastoutpaddlewithatwo-inch-thickhandle.Theballwasapttobeanimpromptuaffaircomposedofabullet,cork,ormetalslugtightlywoundwithwoolyarnandstring.Withitssimple

equipmentandonlyafewrules,thisgamesteadilyincreasedinpopularityduringthefirsthalfofthecentury.

1.Whichtitlebestexpressesthemainideaofthisselection?

(A)“BaseballRules”

(B)“AnEnglishGame”

(C)“Baseball’sPredecessor”

(D)“AmericanPastimes”

2.Therulesofthisgamerequired

(A)eightfielders.

(B)apitcher,acatcher,andonefielderforeachbase.

(C)twentyfielders.

(D)nospecificnumberofplayers.

3.Thisselectionsuggeststhat

(A)thegameofbaseballhasgrownmorecomplicatedovertheyears.

(B)thegamedescribedwasverydangerous.

(C)baseballoriginatedintheUnitedStates.

(D)thegamedescribedrequiredskilledplayers.

4.Thewordimpromptumeans

(A)carefullyplanned.

(B)careless.

(C)informal.

(D)skillful.

TestYourself2

Directions:Readthefollowingpassageandthendecidewhichoftheresponsesisthebestanswertoeachquestion.

JohnJ.Audubon,abirdwatcher,oncenoticedthatapairofphoebesnestedinthesameplaceyearafteryear,andhewonderediftheymightbethesamebirds.Heput

tinysilverbandsontheirlegs,andthenextspringthebandedbirdsreturnedtothesamenestingplace.

Thispairofphoebeswerethefirstbirdstobebanded.Sincethattime,naturalists,withtheaidofthefederalgovernment’sFishandWildlifeDepartment,bandbirdsinanefforttostudythem.Thebands,whicharemadeoflightweightaluminumsoasnottoharmthebirds,bearamessagerequestingfinderstonotifythedepartment.Carefulrecordsofthesenotificationsarekeptandanalyzed.Inthisway,naturalistshavegainedagreatdealofknowledgeaboutthenestinghabits,migrationpatterns,andpopulationsofalargevarietyofbirdspecies.Mostimportantly,theyareabletoidentifythosespeciesthatareindangerofextinction.

1.Whichtitlebelowbestexpressesthemainideaofthispassage?

(A)“TheMigrationofBirds”

(B)“OneMethodofStudyingBirds”

(C)“TheHabitsofBirds”

(D)“TheWorkofJohnAudubon”

2.Audubon’spurposeinbandingthephoebeswasto

(A)satisfyhisowncuriosity.

(B)startagovernmentstudyofbirds.

(C)gainfameasthefirstbirdbander.

(D)chartthephoebe’smigrationpatterns.

3.Audubonprovedhistheorythat

(A)silverandaluminumarethebestmetalsforbirdbands.

(B)thegovernmentshouldstudybirds.

(C)phoebesarethemostinterestingbirdstostudy.

(D)birdsreturntothesamenestingplaceeachspring.

4.Thewordhabitsmeans

(A)naturalists.

(B)livingenvironments.

(C)behaviors.

(D)ecosystem.

TestYourself3

Directions:Readthefollowingpassageandthendecidewhichoftheresponsesisthebestanswertoeachquestion.

Anancientdinosaur-likeanimalcalledtheArchaeopteryxhaslongbeenconsideredtobetheworld’sfirstbird.Basedonwhathasbeenpiecedtogetherfromfossilevidence,Archaeopteryxwasmorelikeacrossbetweenabirdandadinosaur,ratherthanfittingclearlyinonegrouportheother.Ithadteeth,atail,andclawsonitswing—allcharacteristicsthatmorecloselyresembledinosaurs.Italsohadfeathersandwings,liketoday’sbirds.Whetherthecreaturewasabirdwithteethorafeather-covereddinosaurisamatterofdebate,butitsexistenceprovedthatalinkexistsbetweenbirdsandreptiles.

ArecentdiscoveryleadsustoquestiontheacceptedwisdomofArchaeopteryxasthefirstknownbird.AccordingtoanarticleinNationalGeographicmagazinebyauthorBrianSwitek,acreatureknownasAurornisxuiwasdescribedbypaleontologistPascalGodefroitinMayof2013.AurorniswasdiscoveredinChina.Itisbelievedtohavelived160millionyearsago,about10millionyearsbeforeArchaeopteryx.

SwiteknotesthatcontroversyexistsoverwhetherArchaeopteryxevenwasabird.Butforthosewhobelieveitwas,itappearsthatitmaynothavebeenthefirst.

ResearcherscannotsaywithcertaintythatAurorniswasabirdeither.Therearetoofewdetailsinthefossilrecordtomakethisdeterminationdefinitively.Butonethingbasedonthesediscoveriesisclear:birdsaredescendantsofdinosaurs.Today’scommonbirdsharesanancestrywithpredatordinosaurssuchasthevelociraptor,lendingevenmoremeaningtotheideathattheearlybirdgetstheworm.

1.Whichtitlebestexpressesthetopicofthisselection?

(A)“StudyingDinosaurs”

(B)“TheEarliestBirds”

(C)“BirdsversusDinosaurs”

(D)“TheHistoryoftheVelociraptor”

2.Accordingtothepassage,Archaeopteryxcouldbestbedescribedas

(A)anancientbirdwithoutfeathers.

(B)morelikeadinosaurthanabird.

(C)partbirdandpartdinosaur.

(D)morelikeamammalthanareptile.

3.Inthecontextofthepassage,thephraseacceptedwisdommostnearlymeans

(A)invalidtheory.

(B)historicalmyth.

(C)helpfuladvice.

(D)widelyheldview.

4.Accordingtothepassage,today’sbirdsaredescendantsof

(A)dinosaursthathuntedotheranimals.

(B)land-baseddinosaurswithouttails.

(C)dinosaursthatatevegetariandiets.

(D)ancientEgyptianandAsianreptiles.

TestYourself4

Directions:Readthefollowingpassageandthendecidewhichoftheresponsesisthebestanswertoeachquestion.

Theproud,nobleAmericaneagleappearsononesideoftheGreatSealoftheUnitedStates,whichisprintedoneverydollarbill.Thesamemajesticbirdcanbeseenonstateseals,halfdollars,andeveninsomecommercialadvertising.Infact,thoughweoftenencounterartisticrepresentationsofournationalsymbol,itisrarelyseenaliveinitsnativehabitat.Itisnowallbutextinct.

Inthedaysofthefoundingfathers,theAmericaneagleresidedinnearlyeverycorneroftheterritorynowknownasthecontinentalUnitedStates.Todaytheeaglesurvivesinwhatornithologistscallsignificantnumbersonlyintworegions.Anestimated350pairsinhabitFlorida,andperhapsanother150liveintheChesapeakeBayareaofDelaware,Maryland,andVirginia.Afewstragglersremaininotherstates,butinmost,eagleshavenotbeensightedforsometime.

Afederallawpassedin1940protectsthesebirdsandtheirnestingareas,butitcametoolatetosavemorethanapitifulremnantofthespecies’originalpopulation.

1.Anornithologistisapersonwhostudies

(A)geographicalregions.

(B)thehistoryofextinctspecies.

(C)thepopulationsofcertainareas.

(D)thehabitsandhabitatsofbirds.

2.Todayeaglesarefoundinthegreatestnumbersin

(A)Florida.

(B)Delaware.

(C)theChesapeakeBayregion.

(D)Virginia.

3.Theselectionimpliesthat

(A)thenumberofeaglesislikelytoincrease.

(B)theeaglepopulationdecreasedbecauseofalackofprotectivegamelaws.

(C)therewereonlytwolocalitieswhereeaglescouldsurvive.

(D)thegovernmentknowsverylittleabouteagles.

TestYourself5

Directions:Readthefollowingpassageandthendecidewhichoftheresponsesisthebestanswertoeachquestion.

TheAlaskaHighway,whichruns1,523milesfromDawsonCreek,BritishColumbia,toFairbanks,Alaska,wasbuiltbyU.S.ArmyEngineerstocounterathreatenedJapaneseinvasionofAlaska.Itwasrushedthroughinanincrediblyshortperiodofninemonthsandwasthereforeneverproperlysurveyed.Someoftheterritoryitpassesthroughhasnotevenbeenexplored.

Althoughthestorythatthebuildersfollowedthetrailofawanderingmooseisprobablynottrue,theeffectismuchthesame.Theleadingbulldozersimplycrashedthroughthebrushwhereverthegoingwaseasiest,avoidingthebigtrees,swampyhollows,androcks.TheprojectwasmademorecomplicatedbythenecessityoffollowingnottheshortestoreasiestroutebutonethatwouldservethestringofUnitedStates-CanadianairfieldsthatstretchfromMontanatoAlaska.Evenonflatland,theroadtwistsintohairpincurves.Inroughterrainitgoesupanddownlikearollercoaster.Inthemountains,sometimesclingingtothesidesofcliffs400feethigh,itturnssharply,withoutwarning,andgivesrearseatpassengersthestomach-

grippingsensationoftakingoffintospace.Thereisnotaguardrailinitsentire1,500-milelength.Dustkicksupingiantplumesbehindeverycarandonwindlessdayshoversintheairlikeathickfog.

BoththeCanadianArmyandtheAlaskanRoadCommission,whichtookoverfromtheArmyEngineersin1946,doacommendablebutnearlyimpossiblejobofmaintainingtheroad.Whereitisbuiltoneternallyfrozenground,itbucklesandheaves,onthejellylikemuskegitiscontinuallysinkingandmustbegraveledafresheverymonth.Bridgesthrownacrossriversaresweptawayinflashfloods.Torrentialthawswashoutmilesofhighwayeveryspring.Onmountain-sides,youcantelltheageoftheroadbycountingtheremainsofearlierroadsthathaveslippeddowntheslope.

1.Whichtitlebestexpressesthemainideaofthisselection?

(A)“TheAlaskanRoadCommission”

(B)“BuildingandMaintainingtheAlaskaHighway”

(C)“ExploringAlaska”

(D)“DrivingConditionsintheFarNorth”

2.TheAlaskaHighwaywasbuiltto

(A)maketheroutebetweenAlaskaandtheStatesshorter.

(B)promotetradewithCanada.

(C)meetawartimeemergency.

(D)aidexplorationandsurveyingefforts.

3.Thejobofmaintainingtheroadiscomplicatedbythe

(A)threatofinvasion.

(B)forcesofnature.

(C)lackofsurveying.

(D)ageoftheroad.

4.Thewordterrainrefersto

(A)geographicalfeaturesoftheland.

(B)geographicalmountainsandvalleys.

(C)aspecificlandarea.

(D)swamps.

TestYourself6

Directions:Readthefollowingpassageandthendecidewhichoftheresponsesisthebestanswertoeachquestion.

WhenthefirstwhitemencametoNorthAmerica,theyfoundanabundanceofvaluablenaturalresources.Forestscoveredenormousareas;thesoilwasextremelyfertile;andtheforests,prairies,streams,andriversaboundedwithwildlife.Later,hugequantitiesofgas,oil,andmineralswerediscovered.

Theseresourcesweresovastthatitseemedtheycouldneverbeexhausted.Theforestswereclearedforfarmland.Grasslandsandprairieswereplowedandplantedwithcrops.Mammalsandbirdswerehuntedforfoodandsport,andeventuallyfactories,mills,andpowercompanieswerebuiltonnearlyeveryriver.Mineralsandoilwereusedtosupplyandpowerayoungindustrialnation.

Theeffectsoftheseactionsbecameapparentwithinarelativelyshortperiodoftime.Timbershortageswerepredicted.Thefertilesoilwaswashedawaybyrainandblownaboutingreatduststormsbythewind.Severalspeciesofbirdsbegantodisappear,andsomeofthegreatmammalsbecameextinct.Manyriversweremadeunfitforfishbythepollutionoffactories.Theseeminglyinexhaustiblestoresofoilandmineralsbegantoshowsignsofdepletion.

Sincethattime,Americanshavesponsoredthecreationofconservationprogramsinthehopethatfuturegenerationsmaycontinuetoshareandenjoythenaturalresourcesthatarepartofourheritage.

1.Whichtitlebestexpressesthemainideaofthisselection?

(A)“TheFirstWhiteMeninAmerica”

(B)“TheLossofAmerica’sNaturalResources”

(C)“OurAmericanHeritage”

(D)“TheCauseofOurTimberShortages”

2.Theworddepletionmeans

(A)extinction.

(B)runningout.

(C)havingthequalityofbeinginexhaustible.

(D)destruction.

3.Itseemedtotheearlysettlersthat

(A)therewasashortageofminerals.

(B)therehadbeenagreatdealofsoilerosion.

(C)thenaturalresourceswereinexhaustible.

(D)resourcesshouldbecarefullyused.

TestYourself7

Directions:Readthefollowingpassageandthendecidewhichoftheresponsesisthebestanswertoeachquestion.

ThepeoplingoftheNorthwestTerritorybycompaniesfromtheeasternstates,suchastheOhioCompanyundertheleadershipofReverendManassehCutlerofIpswich,Massachusetts,furnishesuswithmanyinterestinghistoricaltales.

ThefirsttownstobeestablishedwereMarietta,Zanesville,Chillicothe,andCincinnati.AftertheOhioCompanycametheConnecticutCompany,whichsecuredalltheterritoryborderingLakeErie,saveasmallportionknownasfirelandsandanotherportionknownasCongresslands.ThelandtakenupbytheConnecticutpeoplewascalledtheWesternReserveandwassettledalmostentirelybyNewEnglandpeople.TheremainderofthestateofOhiowassettledbyVirginiansandPennsylvanians.BecausetheBritishcontrolledLakesOntarioandErie,theMassachusettsandConnecticutpeoplemadetheirjourneyintotheWesternReservethroughthesouthernpartofthestate.GeneralMosesCleaveland,theagentfortheConnecticutLandCompany,ledabodyofsurveyorstothetract,proceedingbywayofLakeOntario.HequietedtheIndianclaimstotheeasternportionofthereservebygivingthemfivehundredpounds,twoheadsofcattle,andonehundredgallonsofwhiskey.LandingatthemouthoftheConneautRiver,GeneralMosesCleavelandandhispartyoffifty,includingtwowomen,celebratedIndependenceDay,1796,withafeastofporkandbeanswithbread.Alittlelater,avillagewasestablishedatthemouthoftheCuyahogaRiverandwasgiventhenameofCleaveland,inhonoroftheagentofthecompany.ItisrelatedthatthenamewasafterwardshortenedtoClevelandbyoneoftheearlyeditorsbecausehecouldnotgetsomanylettersintotheheadingofhisnewspaper.

1.ReverendManassehCutler

(A)ledtheOhioCompany.

(B)ownedtheWesternReserve.

(C)ledtheConnecticutLandCompany.

(D)settledtheCongresslands.

2.Whichtitlebestexpressesthemainideaofthisselection?

(A)“TheSettlingoftheNorthwestTerritory”

(B)“ControloftheGreatLakeRegion”

(C)“TheAccomplishmentsofReverendManassehCutler”

(D)“TheNamingofCleveland,Ohio”

3.Inthelastsentenceoftheselection,thewordrelatedisusedtomean

(A)associatedwith.

(B)rumored.

(C)reported.

(D)thought.

4.TheselectionsuggeststhatGeneralCleavelandatfirstfoundtheIndianstobe

(A)extremelynoisypeople.

(B)hostiletohispartyofstrangers.

(C)starving.

(D)eagertoworkwithhim.

TestYourself8

Directions:Readthefollowingpassageandthendecidewhichoftheresponsesisthebestanswertoeachquestion.

AlongtheshoresoftheIndianOcean,fromAfricaaroundtothelargeislandssoutheastofAsia,isfoundaprettylittleshellfishthatisnotedforfurnishingwhatmayhavebeenthefirstmoneyeverused.Itsshell,calledacowrie,iswhiteorlightyellow,andisaboutoneinchlong.Millionsofpeoplearoundtheoceanwereusingthesecowries,separatelyoronstrings,formoneylongbeforefursorcattleorotherkindsofmoneywereusedanywhere,asfarasisknown.CowrieshavebeenfoundinAssyria,manymilesinland,andinChinatheywereusedwithseveralotherkindsofshells.Tortoiseshellshadthehighestvaluethere,soitmightbesaidthatthetortoiseshellswerethedollarbillswhilethecowrieswerethecoins.Now,afterthousandsofyears,therearestillsometribesinAfrica,India,andtheSouthSeas

thatusecowries.

1.Theauthorbelievesthattheearliestmoneymayhavebeenintheformof

(A)cattle.

(B)furs.

(C)shells.

(D)string.

2.ItissurprisingtolearnthatcowrieswereusedinAssyriabecause

(A)cowriesareonlyoneinchlong.

(B)cattlewereplentifulinAssyria.

(C)Assyriaisawayfromtheseacoast.

(D)tortoiseshellstooktheplaceofdollars.

3.TheChineseused_________formoney.

(A)cattle.

(B)tortoiseshells.

(C)shellfish.

(D)whale’steeth.

TestYourself9

Directions:Readthefollowingpassageandthendecidewhichoftheresponsesisthebestanswertoeachquestion.

FromGettysburgtotheBattleoftheBulge,carrierpigeonshavewingedtheirwaythroughskiesfairandfoultodeliverthevitalmessagesofbattle.Today,inspiteofelectronicsandatomicweapons,thesefeatheredheroesarestillanimportantcommunicationlinkinanyarmy.

NoonecouldbesurerofthisthanthemenatFortMonmouth,NewJersey,thesoleArmypigeonbreedingandtrainingcenterinthiscountry.OntheroostsatFortMonmouthperchmanygenuinebattleheroes,amongthemveteranG.I.Joe.

In1943,onethousandBritishtroopsmovedspeedilyaheadoftheAlliedadvanceinItalytotakethesmalltownofColviVecchia.Sincecommunicationscouldnotbe

establishedintimetorelaythevictorytoheadquarters,thetroopsweredueforapreviouslyplannedAlliedbombingraid.Then,oneofthemenreleasedcarrierpigeonG.I.Joe.Withawarningmessageonhisback,heflew20milesin20minutes,arrivingjustasthebomberswerewarminguptheirmotors.ForsavingthedayfortheBritish,theLordMayorofLondonlaterawardedG.I.JoetheDickinMedal,England’shighestawardtoananimal.

Evenwhenregularmessagechannelsaresetup,equipmentcanbreakorbeoverloadedorradiosilencemustbeobserved.Then,thecarrierpigeoncomesintohisown.Ninety-ninetimesoutofahundred,hecompleteshismission.InKorea,Homerthehomingpigeonwasflyingfromthefronttoarearcommandpostwhenhedevelopedwingtrouble.Undaunted,Homermadeaforcedlanding,hoppedthelasttwomilesanddeliveredhismessage.Forinitiativeandloyalty,HomerwaspromotedtoPfc.—PigeonFirstClass!

1.Thewriterofthispassageevidentlybelievesthatcarrierpigeons

(A)havenousefulnessinmodernwarfare.

(B)shouldbeforcedtoflyonlyinemergencies.

(C)areremarkablyreliableasmessagecarriers.

(D)shouldreceiveregularpromotions.

2.G.I.Joewasrewardedfor

(A)preventingunnecessarylossoflife.

(B)guidingabomber’sflight.

(C)returninginspiteofaninjuredwing.

(D)bringingthenewsofanallieddefeat.

3.G.I.Joe’srewardwasa

(A)promotion.

(B)receptiongivenbytheLordMayor.

(C)chancetoretiretoFortMonmouth.

(D)medal.

4.Thewordvitalmeans

(A)extremelyimportant.

(B)frequent.

(C)recent.

(D)written.

TestYourself10

Directions:Readthefollowingpassageandthendecidewhichoftheresponsesisthebestanswertoeachquestion.

“Sophisticationbythereel”isthemottoofPeretzJohannes,whoselectsjuvenilefilmsforSaturdayviewingattheMuseumoftheCityofNewYork.SamplingtheintellectualclimateoftheyoungfansinthiscityforthepasttwoyearshasconvincedhimthatmanypeopleunderestimatethetastelevelofyoungNewYorkers.Consequently,ayearagohebegantoshowfilmsordinarilyrestrictedtoartmoviedistribution.Theseriesprovedenormouslysuccessful,andinSeptember,whentheprogramcommencedforthisseason,youngstersfromthefiveboroughsfilledthetheater.

Asastudentofhistory,Mr.Johanneshasnotconfinedhimselftoproductionsgivenawardsinrecentyears,buthasspentmanyhoursamongdustyreelsferretingoutsuchpre-warfavoritesasthesilhouettefilmsofLotteReinigerthatweremadeinGermany.Oneprogramincludedtwofilmsbasedonchildren’sstories,“TheLittleRedLighthouse”and“MikeMulliganandHisSteamShovel.”Themoviesareshownat11a.m.and3p.m.,withashortprogramofstoriesandademonstrationoftoyspresentedduringtheintermission.

1.Mr.Johannesfoundthatthechildren’stasteinmotionpictures

(A)wasmorevariedthanhadbeenthought.

(B)ruledoutpicturesmadebeforetheirownday.

(C)waslimitedtocartoons.

(D)wasevenpoorerthanadultshadsuspected.

2.Admissiontotheprogramdescribedis

(A)limitedtochildrenintheneighborhoodofthemuseum.

(B)forManhattanresidentsonly.

(C)availableforallthecity.

(D)forteenagersonly.

3.Mr.Johannes

(A)followedanestablishedpolicyinplanninghisprograms.

(B)hasfailedsofartosecureagoodaudience.

(C)limitshisprogramstothenewestaward-winningpictures.

(D)evidentlyisagoodjudgeofchildren’stastes.

4.Ferretingoutapictureis

(A)givingitatrialrun.

(B)searchingdiligentlyforit.

(C)revivingit.

(D)banningit.

TestYourself11

Directions:Readthefollowingpassageandthendecidewhichoftheresponsesisthebestanswertoeachquestion.

About86percentofthetotalweightofaglassofmilkiswater.Theremaining14percentisacombinationofnutritioussolidssuspendedinthewater.Thesolidsconsistofmilksugar,fat,protein,minerals,andvitamins.

Milkisauniquefoodbecauseitmeetsmostofthebody’srequirementsforgrowthandhealth.ItisespeciallyrichinVitaminsAandBandthemineralscalciumandphosphorus,noneofwhichcanbeeasilyobtainedfromotherfoods.Thesesubstancesareessentialfornormaldevelopmentandmaintenanceofhealthybonesandteeth.

Inspiteofthis,itisfortunateforusthatwedonothavetoliveonmilkalone,asitdoesnotsupplyuswiththeironweneedtopreventanemia.Initsnaturalstate,milkalsolacksVitaminD,whoseproductionwithinthebodycanbestimulatedbysunshine,andthecommercialpreparationofmilkeliminatesitsVitaminC.Itisthereforenecessarytogettheseessentialvitaminsandmineralsfromotherfoodsources.

1.Thelargestpartofmilkis

(A)water.

(B)sugar.

(C)vitamins.

(D)minerals.

2.Milkisanespeciallyimportantfoodbecause

(A)itischeap.

(B)itiseasilyavailable.

(C)itcontainssomuchprotein.

(D)anumberofitsnutrientsarenoteasilyobtainedfromotherfoodsources.

3.Milkdoesnotcontain

(A)phosphorus.

(B)iron.

(C)fat.

(D)VitaminA.

4.Accordingtothearticle,sunshineisimportantintheproductionof

(A)VitaminA.

(B)VitaminC.

(C)VitaminD.

(D)calcium.

TestYourself12

Directions:Readthefollowingpassageandthendecidewhichoftheresponsesisthebestanswertoeachquestion.

InAugustof1814,whennewscamethattheBritishwereadvancingonWashington,threeStateDepartmentclerksstuffedallrecordsandvaluablepapers—includingtheArticlesofConfederation,theDeclarationofIndependence,andtheConstitution—intocoarselinensacksandsmuggledthemincartstoanunoccupiedgristmillontheVirginiasideofthePotomac.Later,fearingthatacannonfactorynearbymightattractaraidingpartyoftheenemy,theclerksprocuredwagonsfromneighboringfarmers,tookthepapers35milesawaytoLeesburg,andlockedtheminanemptyhouse.ItwasnotuntiltheBritishfleethadleftthewatersoftheChesapeakethatitwasconsideredsafetoreturnthepaperstoWashington.

OnDecember26,1941,thefivepagesoftheConstitutiontogetherwiththesingleleafoftheDeclarationofIndependenceweretakenfromtheLibraryofCongress,

wheretheyhadbeenkeptformanyyearsandwerestoredinthevaultsoftheUnitedStatesBullionDepositoryatFortKnox,Kentucky.Herethey“rodeoutthewar”safelyduringWorldWarII.

Since1952,visitorstoWashingtonmayviewthesehistoricdocumentsattheExhibitionHalloftheNationalArchives.Sealedinbronzeandglasscasesfilledwithhelium,thedocumentsareprotectedfromtouch,light,heat,dust,andmoisture.Atamoment’snotice,theycanbeloweredintoalargesafethatisbombproof,shockproof,andfireproof.

1.BeforetheWarof1812,theConstitutionandtheDeclarationofIndependenceweremostlikelykeptin

(A)IndependenceHallinPhiladelphia.

(B)FortKnox,Kentucky.

(C)anofficeoftheStateDepartment.

(D)agristmillinVirginia.

2.Nowadays,thesedocumentsareonviewinthe

(A)NationalArchives’ExhibitionHall.

(B)LibraryofCongress.

(C)UnitedStatesBullionDepository.

(D)UnitedStatesTreasuryBuilding.

3.Animportantreasonfortheinstallationofadevicetofacilitatethequickremovalofthedocumentsismostlikelythe

(A)possibilityofasuddendisaster.

(B)increasingnumberoftourists.

(C)needformorestoragespace.

(D)lackofrespectforthedocuments.

4.Accordingtothepassage,thedocumentshavebeenremovedfromWashingtonatleasttwiceinordertopreservethemfrom

(A)dust,heat,andmoisture.

(B)carelesshandling.

(C)possiblewardamage.

(D)saletoforeigngovernments.

TestYourself13

Directions:Readthefollowingpassageandthendecidewhichoftheresponsesisthebestanswertoeachquestion.

OnJuly1,1882,abriefnoticeappearedinthePortsmouth(England)EveningNews.Itreadsimply,“Dr.Doylebegstonotifythathehasremovedto1,BushVillas,ElmGrove,nexttotheBushHotel.”Sowasannouncedthenewlyformedmedicalpracticeofa23-year-oldgraduateofEdinburghUniversity—ArthurConanDoyle.ButthetownofSouthsea,thePortsmouthsuburbinwhichDoylehadopenedhisoffice,alreadyhadseveralwell-establishedphysicians,andwhilehewaitedforpatientstheyoungDr.Doylefoundhimselfwithagreatdealoftimeonhishands.Tofillit,hebeganwriting—shortstories,historicalnovels,whateverwouldkeephimbusyand,hopefully,bringadditionalfundsintohissparselyfilledcoffers.

Bythebeginningof1886,hispracticehadgrowntothepointofprovidinghimwitharespectableifnotmunificentincome,andhehadmanagedtohaveafewpiecespublished.Althoughliterarysuccessstilleludedhim,hehaddevelopedanideaforanewbook,adetectivestory,andinMarchhebeganwritingthetalethatwouldgivebirthtooneofliterature’smostenduringfigures.AlthoughhewasfamiliarwithandimpressedbythefictionaldetectivescreatedbyEdgarAllanPoe,EmileGaboriau,andWilkieCollins,Doylebelievedhecouldcreateadifferentkindofdetective,oneforwhomdetectionwasascienceratherthananart.Asamodel,heusedoneofhismedicalschoolprofessors,Dr.JosephBell.AsBell’sassistant,Doylehadseenhow,byexercisinghispowersofobservationanddeductionandaskingafewquestions,Bellhadbeenablenotonlytodiagnosehispatients’complaintsbutalsotoaccuratelydeterminetheirprofessionsandbackgrounds.Adetectivewhoappliedsimilarintellectualpowerstothesolvingofcriminalmysteriescouldbeacompellingfigure,Doylefelt.AtfirsttitledATangledSkein,thestorywastobetoldbyhisdetective’scompanion,aDr.OrmandSacker,andthedetectivehimselfwastobenamedSherrinfordHolmes.ButbyApril,1886,whenDoylefinishedthemanuscript,thetitlehadbecomeAStudyinScarlet,thenarratorDr.JohnH.Watson,andthedetectiveMr.SherlockHolmes.

1.Asusedinthepassage,theworddeductionmostnearlymeans

(A)decreasing.

(B)discounting.

(C)reducing.

(D)reasoning.

(E)subtracting.

2.ItcanbeinferredfromthepassagethatSherlockHolmesdifferedfrompreviousfictionaldetectivesinthat

(A)hefocusedhisdetectiveskillsonthesolvingofcrimes.

(B)heconductedhisinvestigationsonascientificbasis.

(C)heusedhisownbackgroundinmedicineasasourceofdetectivemethods.

(D)hiscaseswerechronicledbyacompanionratherthanbythedetectivehimself.

(E)hisexploitswerebasedontheexperiencesofarealindividual.

3.Thewordcompellingmostnearlymeans

(A)forceful.

(B)inescapable.

(C)believable.

(D)fascinating.

(E)insistent.

4.Whichofthefollowingtitlesbestsummarizesthecontentofthepassage?

(A)“ArthurConanDoyleandtheCreationoftheModernDetectiveStory”

(B)“ADetective’sReluctantChronicler:TheBirthofSherlockHolmes”

(C)“PhysicianandAuthor:HowArthurConanDoyleBalancedTwoCallings”

(D)“TheManyStrandsintheCharacterofSherlockHolmes”

(E)“JosephBell:TheReal-LifeInspirationforSherlockHolmes”

TestYourself14

Directions:Readthefollowingpassageandthendecidewhichoftheresponsesisthebestanswertoeachquestion.

LeavingtheelevatedrailroadwhereitdivesundertheBrooklynBridgeatFranklinSquare,scarceadozenstepswilltakeuswherewewish,yetinourears,wehaveturnedthecornerfromprosperitytopoverty.Westanduponthedomainofthetenement.Intheshadowofthegreatstoneabutments,theoldknickerbockerhouseslingerlikeghostsofadepartedday.DownthewindingslopeofCherryStreet—

proudandfashionableCherryHillthatwas—theirbroadsteps,slopingroofs,anddormerwindowsareeasilymadeout;allthemoreeasilyforthecontrastwiththeuglybarracksthatelbowthemrightandleft.

Theseneverhadotherdesignthantoshelter,ataslittleoutlayaspossible,thegreatestcrowdsoutofwhichrentcouldbewrung.Theywerethebadafterthoughtofaheedlessday.Theyearshavebroughttotheoldhousesunhonoredage,aqueruloussecondchildhoodthatisoutoftunewiththetime,theirtenants,theneighbors,andcriesoutagainstthemandagainstyouinfretfulprotestineverystepontheirrottenfloorsorsqueakystairs.Goodcausehavetheyfortheirfretting.Thisone,withitsshabbyfrontandpoorlypatchedroof—whatflowingfiresides,whathappychildrenmayitoncehaveowned?Heavyfeet,toooftenwithunsteadystep,forthesaloonisnextdoor—whereisitnotnextdoorintheseslums?—havewornawaythebrownstonestepssince;thebrokencolumnsatthedoorhaverottedawayatthebase.Ofthehandsomecornicebarelyatraceisleft.Dirtanddesolationreigninthewidehallway,anddangerlurksonthestairs.Roughpineboardsfenceofftheroomyfireplaces;wherecoalisboughtbythepailattherateoftwelvedollarsaton,thesehavenoplace.

Thearchedgatewayleadsnolongertoashadyboweronthebanksoftherushingstream,invitingday-dreamswithitsgentlerepose,buttoadarkandnamelessalley,shutinbyhighbrickwalls,cheerlessasthelivesofthosetheyshelter.Thewolfknocksloudlyatthegateinthetroubleddreamsthatcometothisalley,echoesoftheday’scares.Ahordeofdirtychildrenplayaboutthedrippinghydrant,theonlythinginthealleythatthinksenoughofitschancetomakethemostofit:itisthebestitcando.Thesearethechildrenofthetenements,thegrowinggenerationoftheslums;thistheirhome.Fromthegreathighwayoverhead,alongwhichthrobsthelife-tideoftwogreatcities,onemightdropapebbleintohalfadozensuchalleys.

1.Thispassageservesprimarilyto

(A)argueforthedemolitionoftenementbuildingsandrestorationoftheoldhouses.

(B)decrythelifestyleandhabitsoftheCherryStreettenementdwellers.

(C)describehowpreviousgenerationsenjoyedtheirprosperouslifeonCherryStreet.

(D)contrastpresentandpastconditionsoflifeonCherryStreet.

(E)giveadetailedaccountingofthestructuraldemiseoftheoldknickerbockerhouses.

2.Thewordoutlaymostnearlymeans

(A)expense.

(B)inconvenience.

(C)need.

(D)danger.

(E)distance.

3.Theauthorascribeshumanfeelingstotheoldhouses(“Theyearshavebrought...whathappychildrenmayitoncehaveowned?”)primarilyinorderto

(A)contrastthegracefulhouseswiththepoortenementdwellers.

(B)emphasizehowtimeandpovertyhaveravagedthehouses.

(C)suggestthatinanimateobjectsarecapableoffeelingsandsensations.

(D)elicitsympathyfromreaderswhomaycaremoreforhousesthanpeople.

(E)implyavaluejudgmentaboutthecurrentresidentsofthehouses.

4.TheauthorimpliesthatthepresentresidentsoftheCherryStreethouses

(A)aretoounrefinedtoappreciatethearchitecturalbeautyoftheirhouses.

(B)aretoopoortoproperlymaintaintheoldhouses.

(C)wouldbebetteroffinthemorerecentlyerectedbarracksnearby.

(D)areresponsibleformostofthephysicaldamagetothehouses.

(E)couldeasilyescapethepovertyoftheirsurroundingsiftheysowished.

5.Theauthor’sdescriptionofthechildrenatplaysuggeststhatheviewsthemwith

(A)disdain.

(B)revulsion.

(C)pity.

(D)admiration.

(E)fear.

ANSWERKEYSTestYourself1

1.C 2.D 3.A 4.C

TestYourself2

1.B 2.A 3.D 4.C

TestYourself3

1.B 2.C 3.D 4.A

TestYourself4

1.D 2.A 3.B

TestYourself5

1.B 2.C 3.B 4.A

TestYourself6

1.B 2.B 3.C

TestYourself7

1.A 2.A 3.C 4.B

TestYourself8

1.C 2.C 3.B

TestYourself9

1.C 2.A 3.D 4.A

TestYourself10

1.A 2.C 3.D 4.B

TestYourself11

1.A 2.D 3.B 4.C

TestYourself12

1.C 2.A 3.A 4.C

TestYourself13

1.D 2.B 3.D 4.B

TestYourself14

1.D 3.B 5.C

2.A 4.B

EXERCISES:MAINIDEA

Directions:Readthefollowingpassagesandthendecidewhichoftheresponsesisthebestanswertoeachquestion.

Atadistanceofapproximately250,000milesfromEarth,themoonisournearestcelestialneighbor.Aruggedterrainofmountains,cliffs,plains,andcraterscoversthisglobeof2,000milesindiameter,butthislandscapecontainsnowater.Thereisnoprecipitationofanykindonthemoonbecauseitlacksanatmosphere.Forthesamereason,aconstantbarrageofmeteoritesandotherspacedebrisreachesitssurfacewithouthindrance.Thebeautiful,silverymoonis,inactuality,abarrendesert,sufferingfromgreatextremesoftemperatureanddevoidofanylifeasweknowit.

1.Whichtitlebestexpressesthemainideaofthisselection?

(A)“LandscapesinSpace”

(B)“LifeontheMoon”

(C)“TheMoon’sAtmosphere”

(D)“ConditionsontheMoon”

Themorecomplicatedourthoughtsandemotions,thelesseffectiveislanguageasatoolofexpression.Thisisnotasimplematterofstyleoreloquence,foreventhefinestspeakersandwriters,usingthemostsensitivelanguage,wouldbeincapableofputtingcertainthoughtsintowords.Forthisreason,manypeopleusepoetryandmusicinsteadofprose.Thesetwoformsofcommunicationconveysubtleyetpowerfulmeaningsthatcannotbeexpressedwithordinarywords.

2.Whichtitlebestexpressesthethemeofthisselection?

(A)“Words,Poetry,andMusic”

(B)“TheHiddenMeaningsofWords”

(C)“TheEloquenceofFineSpeakers”

(D)“LimitationsofLanguage”

TheCaribbeanSeaistoNorthandSouthAmericawhattheMediterraneanistotheEuropeancontinent—acentralsea.TheAmericanbodyofwaterisnotlandlocked.Doublestringsofislands—theCubagroupandtheBahamas—formanarcattheAtlanticentrance,andthisarcisnowfirmlyfortified.SincetheMediterraneanof

theWestisthepassagebetweentheAmericas,itmustbecontrolledbythesecountriesinordertocarryontrade.

ThisseaisasnecessarytotheCaribbeancountriesastheMediterraneanistoItaly.Thecountriesofthisareaproducelargequantitiesofoil,tropicalfruits,andvegetables.Theyarealsorichinminerals.ThisregioniscapableofsupplyingtheUnitedStateswithmanygoodsformerlyimportedfromAfricaandAsia.Inexchange,thecountriesofthisregionneedthemanufacturedgoodsthatcanbeprovidedonlybyanindustrialnation.

3.TheCaribbeanSeaandtheMediterraneanarealikewithrespecttotheir

(A)varietyofexports.

(B)epidemicsofseriousdiseases.

(C)geographicalimportance.

(D)livingstandards.

Thedangerstowhichmigratorybirdsaresubjectedduringtheirjourneysarebutlittlelessthanthosethatwouldbefallthemiftheyremainedinunsuitablezones.Duringlongoverseapassages,fatigueandhungerweedouttheweaklings.Suddenstormsandadversewindsstrikemigratingbirdswherenolandisnear,andtheyareoftencarriedfarfromtheirintendeddestinations.Predatorybirdsaccompanythem,takingatollenroute,andpredatorymanwaitsforthetiredwandererswithgunandnet.

4.Whichtitlebestexpressesthemainideaofthispassage?

(A)“DangersofStorms”

(B)“PerilsofMigration”

(C)“UnsuitableEnvironment”

(D)“HowBirdsReachGoals”

InhislibraryatMonticello,Jeffersonmadehundredsofarchitecturaldrawings,allofwhichhavebeenpreserved.Hemusthavehadagreatgiftofconcentrationandarealloveforhissubjecttobeabletoworkinaroomwithsuchanoutlook.Andwhatenergyhehadtofindtimeandwillforthispreciseandexquisiteworkwasalsodevotedtoridingoverhisestate,workinginhisgarden,andcarryingoutcorrespondencewitheveryonefromtheMarquisdeLafayettetohisyoungestgrandchild.“Somethingpursuedwithardor”wasJefferson’sprescriptionforlife,andhegotthelastounceofexcitementandinterestoutofeverythingthatcametohisnotice.

5.Themainideaofthisselectionisexpressedinwhichtitle?

(A)“TheLibraryatMonticello”

(B)“TheCareoftheEstate”

(C)“AFullLife”

(D)“Jefferson,theArchitect”

Specifictypesoflightingarerequiredatfirst-classairportsbytheDepartmentofCommerce.Toidentifyanairport,theremustbeabeaconlightofnotlessthan100,000candlepower,withabeamthatproperlydistributeslightupintheairsothatitcanbeseenallaroundthehorizonfromanaltitudeof500to2000feet.AllflashingbeaconsmusthaveadefiniteMorsecodecharacteristictoaidinidentification.Coloredlightsarerequiredtoindicatewherethesafeareaforlandingends,redlightsbeingusedwherelandingisparticularlydangerous.

6.Whichisthebesttitleforthisselection?

(A)“LandingAreas”

(B)“ColoredLightsatAirports”

(C)“IdentificationofAirports”

(D)“AirportLightingRequirements”

ANSWERSANDEXPLANATIONS

1.Thecorrectansweris(D).Thewordconditionscoversallaspectsofthemoondiscussedinthisselection:themoon’ssize,itsterrain,itsatmosphere,temperature,etc.Choice(A)isincorrectbecauseitsuggestsdiscussionofmorethanonecelestialbody,butthisselectiontalksonlyaboutthemoon.Choice(B)isincorrectbecausethereisnolifeonthemoon,andchoice(C)isincorrectbecausethemoon’satmosphereisonlyoneofthesubtopics—notthemainidea—oftheselection.

2.Thecorrectansweris(D).Theselectiondiscussessomeofthereasonsandremediesforthelimitationsoflanguage.Choices(A),(B),and(C)areincorrectbecause,thoughthetopicsarementioned,theyarenotthethemeoftheselection.

3.Thecorrectansweris(C).Bothbodiesofwaterservetheborderinglandmassesinsimilarways.

4.Thecorrectansweris(B).Eachsentenceintheparagraphexplainswhymigrationisperilous.Choices(A),(C),and(D)aretopicsthataren’tdiscussed.

5.Thecorrectansweris(C).TheselectiondescribesindetailJefferson’s“fulllife”:hisarchitecturaldrawings,hisloveofhisestateandgardens,hiscorrespondences,andhis“prescriptionforlife.”Choices(A),(B),and(D)areeithernotdiscussedorareonlyoneaspectofthetopic.

6.Thecorrectansweris(D).Theselectiondescribesseverallightingrequirementsforairports.Choices(A),(B),and(C)areincorrectbecausetheyrefertoonlyonespecifictypeoflightingorlightingrequirement.

EXERCISES:DETAILS

Directions:Readthefollowingpassagesandthendecidewhichoftheresponsesisthebestanswertoeachquestion.

Antsareveryinterestinginsects.Thereareabout8,000differentkindswithvariouswaysoffindingfood.Therearehunterantsthatcaptureotherinsects,shepherdantsthatcareforaphidsfromwhichtheygetsweethoneydew,thiefantsthatlivebystealing,slave-makingantsthatkidnapthechildrenofotherantnations,andmightymilitaryantsthatlivebyplunderinganddestroying,drivingevenmenandelephantsbeforethem.

Acityofantsincludesthequeen,theworkers,thebabyants,andtheirnurses.Antbabieschangetheirformthreetimes.First,theyaresmall,whiteeggs.Whentheyhatch,theyarelittle,fat,whitewormscalledlarvae.Thelarvaechangeintopupae,andthepupaechangeintoadults.Thequeenisthemotherofalltheantsinthecommunity.Theworkersbringfoodtoherandprotectherfrominvaders.

1.Hunterants

(A)careforaphids.

(B)kidnapyoungantsfromothercolonies.

(C)captureotherinsects.

(D)plunderanddestroy.

2.Acolonyofants

(A)includesaqueen,workers,babies,andtheirnurses.

(B)mayhaveasmanyas8,000members.

(C)isbuiltinahill.

(D)protectsitsmembers.

3.Immediatelypriortoenteringtheadultstage,ants

(A)hatchfromeggs.

(B)comefromlarvae.

(C)areallworkers.

(D)comefrompupae.

Commercialinterestswerequicktorecognizethegreatpossibilitiesofpresentingbymeansofradiowhatisineffectaperson-to-personappeal.Atfirstthenoveltymadepeoplelistentoalmostanything,butastheaudiencesbecamemoreaccustomedtobroadcasts,variedmethodsofcapturingandholdingtheattentionhavedeveloped.Thesevaryfromthefrankinterjectionofadvertisingmatterinaprogramofentertainmenttothemeresponsoringoftheprogram.Entertainmentatfirstappearedtohavethegreatestappeal,andlowcomedyandjazzmusicfilledtheair.Therehascome,however,therealizationthattheradioaudienceisnowascomplexasthepublicandthatprogramsmustbesetuptoattracttheattentionofasmanydifferenttypesofhearersaspossible.

4.Whenradiowasnew,

(A)peoplewouldlistentoalmostanything.

(B)advertisingwaspoor.

(C)advertisingwasinterjectedintotheprogramming.

(D)entertainmentwaslimited.

Thepartoftheearweseeisonlyacartilageandskintrumpetthatcatchessoundwaves.Buriedinboneatthebaseoftheskullisthedelicateapparatusthatmakeshearingpossible.

Apassageleadsfromtheoutereartoamembranecalledtheeardrum.Soundwavesstrikingtheeardrummakeitvibrate.Ontheothersideoftheeardrumliesaspacecalledthemiddleear.Acrossthisachainofthreetinybonescarriessoundvibrationstoanotherspacecalledtheinnerear.Soundmessagesareconductedalongtheauditorynerve,locatedintheinnerear,tothebrainforinterpretation.ThemiddleearisconnectedtothethroatbytheEustachiantube.Thistubeendsnearthethroatopeningofthenose,closetothetonsils.Themiddleearalsocommunicateswiththemastoid,oraircellsinthebonebehindtheear.

5.Theouterearismadeof

(A)adelicateapparatus.

(B)amembrane.

(C)cartilageandskin.

(D)threetinybones.

6.Theeardrumisa(n)

(A)membrane.

(B)pieceofthincartilage.

(C)aircell.

(D)shorttube.

7.Soundvibrationsarecarried

(A)alongtheauditorynerve.

(B)throughtheeardrum.

(C)totheinnerearacrossachainofthreetinybones.

(D)tothebaseoftheskull.

Track-and-fieldeventsaretheonlymodernsportsthatwouldberecognizableintheiroriginalform.Theycanbetracedbackmorethan2,500yearstotheancientcivilizationofGreece.TheGreeksheldtheirathletesinhighesteem,andchampionswerelookeduponasnationalheroes.

TheGreeksbegantheoriginalOlympicgamesforthepurposeofassemblingthegreatestathletesoftheircountry.Thegameswerereligiouspageantsaswellaspeerlessathleticeventsandwereheldeveryfouryearsformorethaneightcenturies.

8.InancientGreece,athleteswere

(A)trainedasprofessionals.

(B)forcedtoparticipateinthegames.

(C)usuallydefeatedbytheRomans.

(D)regardedveryhighlybythepublic.

9.Thepresent-dayOlympics

(A)havea2,500-year-oldhistory.

(B)arereligiouspageants.

(C)havebeenheldeveryfouryearsforeightcenturies.

(D)arecompletelydifferentfromtheGreekgames.

Observethepeoplewhomakeanabidingimpressionofstrengthandgoodnessandyouwillseethattheirpersonalattractivenessandforcearerootedinfundamentalsofcharacter.Theyhavethephysicalvitality,endurance,andcouragethatcomefromgoodliving.Theyhavethementalstaminaandpenetrationthatcomefromfacingup

toone’sproblems,howeverdifficult,andfromkeepingone’smindonthingsthatreallymatter.Theyhavethemoralpowerthatcomesfromanactivesenseofwhatisright,fromdoingtheirparttomaketruth,justice,andbeautyprevailintheworld.Theyhavetheinnerpeaceandgracethatarethebasicsofatrulycharismaticpersonality.Peopletrustthem,liketobewiththem,anddependontheminemergencies.Theyarethesaltoftheearth.

10.AqualityNOTmentionedbytheauthoris

(A)courage.

(B)dependability.

(C)tolerance.

(D)innergrace.

Althoughyoumaystillenjoyfairytales,theyprobablydonotengrossyoutothedegreethattheymighthaveafewyearsago.Fairytalesbelongprimarilytoastageinourliveswhenwearemostinterestedbytheworldoffantasy.Goblins,wizards,anddwarfsappealtotheyoungchild’swanderingimaginationandcontributegreatlytothedevelopmentofcreativity,butitisatemporaryinfatuation.

Aswegrowolder,realchallengesbegintointerestusmore.Theimaginaryvictoriesbroughtaboutbyfairygodmotherslosetheirpowerofenchantment,andwebecomeabsorbedinthestoriesofrealpeople,realsuccess,andrealaccomplishment.Thefascinationof“JacktheGiantKiller”giveswaytoakeeninterestinCommanderByrd’sAntarcticexploration,HelenKeller’sbiography,ortheharrowingadventuresofspelunkers,deep-seadivers,andmountainclimbers.Thisstepmarksoneofthefirstgreatadvancesintheprocessofintellectualmaturation.

11.Youngchildrenareprimarilyinterestedin

(A)fantasystories.

(B)horrorstories.

(C)goblinsandwitches.

(D)adventurestories.

12.Peoplebecomeinterestedinreal-lifestorieswhenthey

(A)areyoung.

(B)areadults.

(C)begintomature.

(D)arebored.

ANSWERSANDEXPLANATIONS

1.Thecorrectansweris(C).Seeparagraphone,sentencethree.

2.Thecorrectansweris(A).Seeparagraphtwo,sentenceone.

3.Thecorrectansweris(D).Seeparagraphtwo,sentencefive.

4.Thecorrectansweris(A).Seethesecondsentence.

5.Thecorrectansweris(C).Seeparagraphone,sentenceone.

6.Thecorrectansweris(A).Seeparagraphtwo,sentenceone.

7.Thecorrectansweris(C).Seeparagraphtwo,sentencefour.Soundmessagesarecarriedalongtheauditorynerve.

8.Thecorrectansweris(D).Seeparagraphone,sentencethree.

9.Thecorrectansweris(A).Seeparagraphone,sentencetwo.

10.Thecorrectansweris(C).Theotherchoicesareallmentionedinthepassage.

11.Thecorrectansweris(A).Seethefirstparagraph.

12.Thecorrectansweris(C).Seeparagraphtwo,sentencesoneandtwo.

EXERCISES:VOCABULARY

Directions:Readthefollowingpassagesandthendecidewhichoftheresponsesisthebestanswertoeachquestion.

InMayofeachyear,theghostofMarkTwainmusthoveroverAngel’sCamp,California,whilealleyesinthiscolorfuloldminingtownturntothetailless,leapingamphibiansofthegenusRana.ItwasjustthissortofeventthatTwainmadefamousinhisearlyhumorousstory,“TheCelebratedJumpingFrogofCalaverasCounty.”

Thousandsofspectatorsgathereachyeartowatchthecounty’schampionshoptheirwaytofameandcompetefora$500firstprize.Eachfrogmustundergoarigidinspectiontoinsureagainstfoulplay,suchastheloadingofthecompetitorwithbuckshot,ashappenedinTwain’stalltale.

Backin1944,AlfredJermywastheproudownerofFlash,afrogthatheldtheworld’schampionshipwithafifteen-foot,ten-inchleap.In1950,aseven-year-oldboy’spet,X-100,stoletophonorswiththreejumpsaveragingfourteenfeet,nineinches.Asamazingasthesemightseemtothenovice,thesearemerepuddlejumps.

HalfthefuninvisitingthisCalaverasCountycontestistobefoundinlisteningtothetalesof600-footleapsinafavorablewind—well,whynot?

1.Theamphibiansmentionedinthefirstparagrapharethe

(A)storytellers.

(B)frogs.

(C)citizensofCalaverasCounty.

(D)humancontestants.

2.Thewordnoviceinthethirdparagraphmeans

(A)thejudges.

(B)thespectators.

(C)theownersofthefrogs.

(D)inexperiencedreaders.

Intheyear1799,anofficeroftheFrenchArmywasstationedinasmallfortressontheRosettaRiver,amouthoftheNile,nearAlexandria,Egypt.HewasinterestedintheruinsoftheancientEgyptiancivilizationandhadseentheSphinxandthepyramids,thosemysteriousstructuresthatwereerectedbymenofanotherera.

Oneday,asatrenchwasbeingdug,hefoundapieceofblackslateonwhichlettershadbeencarved.HehadstudiedGreekinschoolandknewthiswasaninscriptionwritteninthatlanguage.Thereweretwomorelinescarvedintothestone:oneintheEgyptiancharactershehadseenonotherruins,theotherincompletelyunfamiliarcharacters.

TheofficerrealizedtheimportanceofsuchafindandrelinquishedittoscholarswhohadbeenpuzzlingoverEgyptianinscriptions.

In1802,aFrenchprofessorbythenameofChampollionbeganstudyingthestoneinanattempttodecipherthetwounknownsetsofcharactersusingtheGreeklettersasakey.Heworkedwiththestoneforover20yearsand,in1823,announcedthathehaddiscoveredthemeaningofthefourteensignsandindoingsohadunlockedthesecretofancientEgyptianwriting.Some5,000yearsafteranunknownpersonhadmadethosethreeinscriptions,theRosettaStonebecameakey,unlockingthewrittenrecordsofEgyptandsharingthehistoryofthatcivilizationwiththerestoftheworld.

3.Theworddecipherissynonymouswith

(A)translate.

(B)encode.

(C)transcribe.

(D)transmit.

4.Thewordinscriptionmeans

(A)apicturecarvedinstone.

(B)areliefsculpture.

(C)letterscarvedintoahardsubstance.

(D)awrittenmessage.

5.Thewordrelinquishmeansto

(A)giveuppossessionofsomething.

(B)lendtosomeone.

(C)sellanobject.

(D)studyanobject.

Theimpressionsthatanindividualgetsfromhisenvironmentaregreatlyinfluencedbyhisemotionalstate.Whenheishappy,objectsandpeoplepresentthemselvestohiminafavorableaspect;whenheisdepressed,heviewsthesamethingsinanentirelydifferentlight.Ithasbeensaidthataperson’smoodsarethelensesthatcolorlifewithmanydifferenthues.Notonlydoesmoodaffectimpression;impressionalsoaffectsmood.Thebeautyofaspringmorningmaydissipatethegloomofagreatsorrow;thegood-naturedchuckleofachildmayturnangerintoasmile;oratelegrammaytransformahouseofmirthintoahouseofmourning.

6.Theworddissipatemeans

(A)condense.

(B)drawout.

(C)meltaway.

(D)inflate.

7.Thewordtransformissynonymouswith

(A)convert.

(B)conclude.

(C)interpret.

(D)convey.

ANSWERSANDEXPLANATIONS

1.Thecorrectansweris(B).Frogsareatypeofamphibian.

2.Thecorrectansweris(D).Thewordchoicenovicemeans“inexperienced.”

3.Thecorrectansweris(A).Deciphermeanstobreakacodeandtranslatethemessage.

4.Thecorrectansweris(C).Aninscriptionislettersorwordsthatarecarvedintoahardsubstance.Otherexamplesofinscriptionsarenamescarvedintogravestonesandinitialsinscribedintojewelry.Toinscribeistowriteinto.

5.Thecorrectansweris(A).Thewordrelinquishmeans“letgo”or“giveup.”

6.Thecorrectansweris(C).Othersynonymsfortheworddissipateare“scatter,”“dissolve,”and“evaporate.”

7.Thecorrectansweris(A).Whensomethingischangedtosomethingelse,itistransformed,orconverted.

EXERCISES:INFERENCE

Directions:Readthefollowingpassagesandthendecidewhichoftheresponsesisthebestanswertoeachquestion.

Intuitionisnotaqualityeveryonecanunderstand.Astheunimaginativearemiserableaboutaworkoffictionuntiltheydiscoverwhatflesh-and-bloodindividualservedasamodelfortheheroorheroine,so,too,manyscientistsscoffattheunscientificnotionthatintuitionasaforceexists.Theycannotbelievethatablindmancanseesomethingtheycannotsee.Theyrelyutterlyonthecelebratedinductivemethodofreasoning:exposethefactsandconcludefromthemonlywhatcanbeproven.Generallyspeaking,thisisaverysoundrule,butcanwebecertainthatthereallygreataccomplishmentsareinitiatedinthisploddingfashion?Dreamsaremadeofquitedifferentstuff,andifanyareleftintheworldwhodonotknowthatdreamshaveremadetheworld,thenperhapsthereislittlewecanteachthem.

1.Theauthorimpliesthatintuition

(A)istheproductofimagination.

(B)reliesonfactualinformation.

(C)isaninductivereasoningprocess.

(D)isvalueless.

Itisexceedinglydifficulttodrawonacanvasthemanwhosenatureislargeandcentral,withoutcranksoroddities.Theverysimplicityofsuchsoulsdefiesaneasysummary,fortheyareasspaciousintheireffectasdaylightorsummer.Oftenwerememberfriendsbyagestureoratrickofexpression,orbyafavoritephrase.ButwithNelsonIdonotfindmyselfthinkingofsuchidiosyncrasies.Hispresencewarmedandlitupsobigaregionoflifethatinthinkingofhimoneisoverwhelmedbythemultitudeofthingsthathemadebetterbysimplyexistingamongthem.Ifyouremoveafirefromthehearth,youwillrememberthelook,notsomuchoftheblazeitself,asofthewholeroominitspleasantglow.

2.Thephrase“todrawonacanvas”isusedinthiscontexttomeanto

(A)paintaportrait.

(B)summarize.

(C)makeacollage.

(D)describe.

3.ThelastsentenceisametaphorcomparingNelsonto

(A)theblazeinafireplace.

(B)ahearth.

(C)fire.

(D)apleasantglow.

4.Fromthetoneofthisselection,youmightdrawtheconclusionthattheauthor

(A)thinksofNelsonasastrangeman.

(B)isdescribingamanwhohasdied.

(C)isoverwhelmedbyNelson.

(D)remembersNelsononlybyhisgestures.

AglasscaseintheBritishMuseumhousesthemummifiedremainsoftwoEgyptiankingswholivedbesidetheNile.Theexhibitincludesabrokenplow,arustedsickle,andtwostickstiedtogetherwithaleatherstrap.Thesewerethe“breadtools”ofEgyptianswholived4,000yearsagoduringthereignsofthetwokings.Theyarenotunlikethetoolsusedbyeighteenth-centuryAmericanfarmers,and,infact,similarsicklesmaybeviewedatMountVernon,GeorgeWashington’sVirginiahome.

5.WemayconcludefromthisselectionthattheancientEgyptians

(A)hadonlytwoimportantkings.

(B)taughtfarmingtechniquestoeighteenth-centuryAmericans.

(C)wererelativelyadvancedintheuseofagriculturaltools.

(D)neglectedtheirequipment.

Thehornofanautomobileisavaluableaidtogooddrivingifproperlyused.Whenabouttopassanothercar,itisadvisabletonotifythedriverofthecarahead.Childrenoranimalsonthestreetshouldbegivenawarningnote.Ofcourse,acourteousdriverwouldnotblowhishornunnecessarilyinthevicinityofahospitaloraplaceofworship.Heshouldalsobeconsiderateofschools,wherequietisimportant.Thewayinwhichadriveruseshishornisafairlyaccurateindextohischaracter,forthroughthesoundheexpresseshisimpatienceandhisgoodmanners,

orthelackofthem.

6.Theplacethatagooddriverwouldbeleastlikelytousehishornis

(A)St.JamesTheater.

(B)RiverdaleApartments.

(C)MemorialConvalescentHome.

(D)YankeeStadium.

7.Thecharacterofadriverwhofailstosoundhishornwhenadogiscrossingthestreetis

(A)noble.

(B)impatient.

(C)uncaring

(D)bold.

AccordingtoearlyEnglishhistory,asmallgroupofpeoplefromnortheasternEurope,calledEasterlings,camebyinvitationtoEnglandtodeviseanddevelopanewsystemofcoinage.Thesepeoplelivedintownsthatwerefamousfortheaccuracyoftheircoins.ThecoinsthattheyworkedoutforEnglandweremadeofsilverandcametobeknownastheEasterlingcoins.LaterthewordEasterlingwasshortenedtosterling.Thewordsterlinggraduallycametobeappliedtoallsilverarticlesofveryfinequality.

8.ThepassageimpliesthattheEasterlings

(A)hadanexcellentreputation.

(B)usedsilverexclusively.

(C)weresilversmiths.

(D)coinedthewordsterling.

9.Thewordsterlingbegantobeusedforhigh-qualitysilverbecause

(A)itwasusedtomakeEnglishcoins.

(B)theEasterlingswereknownforthequalityoftheirwork.

(C)silverisveryexpensive.

(D)theEasterlingsweretheonlypeoplewhocouldmakesilvercoins.

ANSWERSANDEXPLANATIONS

1.Thecorrectansweris(A).Theauthorlikensintuitionto“dreams”(line18)andscoffsatthebeliefthatinductivereasoninginitiatesallgreataccomplishments(lines16–19).

2.Thecorrectansweris(D).Lines4and5furtherexplaintheauthor’sassertionthatitisnoteasytodescribeNelsonandhisaffectonher.

3.Thecorrectansweris(A).Theauthordescribesthe“pleasantglow”thatcomesfromNelsonasifhewerefire.

4.Thecorrectansweris(B).TheentirepassageisaboutrememberingNelson,andlines17–20makereference(inmetaphor)toNelson’sleaving.

5.Thecorrectansweris(C).ThetoolsusedbyEgyptians4,000yearsagoaresimilartothosetoolsusedbyAmericanfarmersintheeighteenthcentury.

6.Thecorrectansweris(C).Agood(andcourteous)driverwouldnot“blowhishornunnecessarilyinthevicinityofahospital”(lines8–10).

7.Thecorrectansweris(C).Theparagraphstatesthat“animalsonthestreetshouldbegivenawarningnote”(lines6–7).Anuncaringorinattentivedriverwouldnotblowhisorherhornasawarning.

8.Thecorrectansweris(A).ThepassagestatesthattheEasterlingswere“invited”byEnglandtodevelopanewcoinagesystem.

9.Thecorrectansweris(B).Accordingtothepassage,theEasterlingsdevelopedcoinageofthehighestquality.

SUMMINGITUP

Reviewthispagethenightbeforeyoutakeyourhighschoolentranceexam.Itwillhelpyougettheanswerstoreadingcomprehensionquestions.

•Whenyougettothereadingcomprehensionsection,takeadeepbreathandfollowthesesteps:readthroughthequestionsquickly,notingwhatinformationyouwillhavetofind(skipovertheanswerchoicesfornow);readthepassage;scanthepassageandanswerthevocabularyquestionsfirst;tackledetailquestionssecond(pickakeywordortwofromthequestionandscanthepassageforthelocationoftheanswer);readthefirstandlastsentencesofthepassagetoanswermain-ideaquestions;skimforanswerstoinferencequestions;andeliminatewronganswersandmakeyourbestguess.

•Alltheinformationyouneedisrightinthepassage.•Readingcomprehensionquestionsarenotarrangedinorderofdifficulty.•Don’tgetboggeddownindetails.Lookforimportantideasandmarkthemasyoufindthem.

•Answereveryquestionforapassagebeforestartingthenextpassage.

PARTVIMATHEMATICSREVIEW

CHAPTER10:Mathematics(SSATandISEE)

CHAPTER11:QuantitativeAbility(SSATandISEE)

CHAPTER12:QuantitativeComparisons(ISEEOnly)

Chapter10

Mathematics(SSATandISEE)

OVERVIEW

•WhatmathematicsmustIknow?•HowdoIestimatetheanswer?•WhenmustIcalculate?•Testyourselfquizzes•Answerkeys•Summingitup

WHATMATHEMATICSMUSTIKNOW?Theanswertothisquestionisaquestion.Whatgradeareyouin?Ifyouareineighthgrade,youmustknowbasicarithmetic,fundamentaloperationsusingfractionsanddecimals,percents,andverybasicalgebraandgeometry.Ifyouareintwelfthgradeseekinganextrayearbeforecollege,youshouldbethoroughlyfamiliarwithcomplexalgebraandgeometryandwithrootsandexponents.

Rememberthatscoringofyourhighschoolentranceexamisbaseduponyourgrade.Youdonotneedtoknowwhatyouhavenotyetbeentaught.Butyoumusthavemasteredallthemathematicsappropriatetoyourgradelevel.Useyourmathtextbooktohelpyoulimittheextentofyourstudyinthischapter.Don’ttrytolearnahead.Concentrateondoingwellonthemaththatapersonyouragemustknow.

TheFundamentalOperations

Addition,subtraction,multiplication,anddivisionarethebasicoperationsuponwhichthestructureofmathematicsisbased.Thereisnosubstituteforhavinggoodskillsincomputationto

achievesuccessononeofthehighschoolentranceexaminations.Proceedthroughthissectioncarefully,beinghonestwithyourselfabouttheaccuracyandspeedwithwhichyousolvetheseproblems.Noteproblemsthataredifficultforyouaswellasthosethatareeasy.Adjustyourstudyplansaccordingly.

TheNumberLine

Anumberlineisaconvenientconcepttouseasamentalpicture.Thenumberlineaboveshowswholenumbersandfractionsgreaterthanzeroandlessthanzero.Numbersincreaseinsizeasyoumovetotherightanddecreaseinsizeasyoumovetotheleft.Thenumberlineabovehasanarrowateachend,meaningthatthenumberlinegoesoninfinitelyinbothpositiveandnegativedirections.

Numberlinescanbedrawntoaidinbasicmathematicalcalculations.Eitherfractions,wholenumbers,ordecimalscanbeusedtonametheintervalsontheline.Wesuggestthatyouusenumberlineswhendealingwithsigned(+,–)numbersandinequalities.

Addition

Intheprocessofaddition,weaddtogethernumbers,whichwecalladdends,toresultinasum.Addendsmaybeaddedinanyorder(commutativeproperty).

Example1:

Example2:

Example3:

Example4:

Simpleadditionproblemsmayconsistofonlywholenumberaddends(asinExample1),ofonlydecimalorfractionaladdends(asinExamples2and3),orofamixtureofallthree(asinExample4).

TIPRemember:Linethenumbersupunderneatheachothercarefully.

WholeNumberAddition

ToaddwholenumbersasinExample1,lineuptheaddendsinacolumn.Addeachcolumnofnumberscarefully,makingsuretocarrytenstothenextcolumn:

203155+80438

HOWDOIESTIMATETHEANSWER?Inmultiple-choicemathquestions,theanswerisinfrontofyou.Youdon’tneedtocalculateit;youonlyneedtorecognizeit.Theproblemmaybemuchsimplerthanitlooksorreads.Infact,ifitlooksasifthecalculationwillbeverylongorcomplicated,youareprobablyheadingdownthewrongtrack.Highschoolentranceexamsarenotmeanttobetestsofyourcomputationalskills;theyaretestingyourunderstandingofconceptsandabilitytoapplythatunderstanding.Leaveyourpencilonthedeskforamomentandstartwithcommonsense:

3.01+10.73+2.01+0.781=

(A)13.522

(B)16.531

(C)20.860

(D)36.036

Notethatinthiscaseitisbestnottoproceedbywritingdownthenumberstobeadded,addingthem,andthencheckingyouransweragainstthosesupplied.Calculatingtheanswerinthiswaywouldwastevaluabletimethatcouldbeusedlateron.

Thecorrectansweris(B).

FourStepsforEstimatingtheAnswer:GettingItRight

Readthequestion,andnotethatitisadecimaladditionproblem.DON’TCALCULATEYET.Readthepossibleanswers;noticetherangetheycover.Estimatethesumofthefournumbersas3+11+2+1=17.Notethatchoice(B)istheonlyansweranywherenearasumof17.Chooseitasthecorrectanswer.

Or,evensimpler:Readthequestion,andnotethatitisadecimaladditionproblem.DON’TCALCULATEYET.Lookatthefouraddends.Notethatthreehavedigitstwoplacestotherightofthedecimalpointandthatonlyonehasadigitinthethousandthsplace.Thatdigitisa“1.”Lookatthefouranswerchoices.Concentrateonthedigitinthethousandthsplace.Choose(B)astheonlypossiblecorrectanswer,andmoveonquicklytothenextquestion.

WHENMUSTICALCULATE?Therewillbesomeproblemsthatwillrequireyoutouseapencilandpaper(nocalculators,alas!).Youmaybeabletoestimateananswertothosequestions,butthechoicesgivenwillmakeestimatingtoorisky.

Wesuggestyoucalculatetheanswerwhen:•Theanswerchoicesdifferonlyveryslightly.•Theproblemrequiresthreeormoresteps,makingitdifficulttorememberaccuratelyyourintermediatesteps.

•Youhavetochangelargerunitsintosmallerones,orviceversa,forpurposesofcalculation.ItiseasytolosetrackofunitswithintheEnglishsystemofmeasurementswhenworkingwithtime,unitsofmeasurement,andsoon.

TESTYOURSELFQUIZZESReadthroughthefollowingsections,andtakethequizzestohelpyoudetermineyourweaknesses.Clickherefortheanswers.

TestYourself1

Directions:Trytheseproblems.Theanswersarehere.Aimfor100percentaccuracyandnoteyourerrors.

1.463+729+36

2.257+32

3.174+20,962

4.1732+32,629

5.33+472+8

6.138+76+82+1224

7.59+732+111

8.137,921+29+71

9.393+462+1701+733

10.145+66+78

DecimalAddition

Decimalsareawayofwritingfractionsusingtenths,hundredths,thousandths,andsoforth.Ifyoucancountmoney,makechange,orunderstandabattingaverage,decimalsshouldpresentnoproblem.

Whenwritingdecimals,themostimportantstepisplacingthedecimalpoint.Thewholesystemisbaseduponitslocation.Rememberthedecimalplaces?

TestYourself2

Directions:Ifyouneedpracticereadingdecimals,trytheexercisesbelow.

1.0.0076

2.11.3

3.1402.639

4.$7222.93

5.0.50

6.0.05

7.16.2163

8.0.00029

9.3.0006

10.62.391

TestYourself3

Directions:Trytodeterminewhichofthesepairsofnumbersislarger.

1.0.5or0.05

2.5.12or5.012

3.0.007or0.07

4.16.20or16.2

5.10.7or1.70

6.0.762or7.62

7.3.009or3.0009

8.0.143or0.1430

Addingdecimalsisnoharderthanaddingwholenumbers,aslongasyoupayattentiontothedecimalpoint.Toaddagroupofdecimals,placetheminacolumn,beingcertaintolineupthedecimalpoints.

Example:17.4+6.2+2.2=25.8

Solution:17.46.2+2.225.8

Noticethatthedecimalpointisbroughtstraightdown!Now,trythenextexampleforpractice.Wheredecimalplacesmaybe“missing,”fillinwithzerosifyouneedto.

Example:Add22.0061+7.003+2.1+0.001+100.01.

Solution:22.006107.003002.100000.0010+100.0100131.1201

Theunderlinednumbersshowwhereyoumayfillinwithzerosifyouwant.

TestYourself4

Directions:Trytheseproblems.

1.7.223+60.1

2.0.0792+5.06

3.100.23+9.7962

4.82.48+21.2417

5.0.0323+0.06

6.9623.2+43.788

7.14.1414+0.044

8.0.02+3.63+92.003

9.720.72+69.58

10.4.7+3.2+0.9+1.2

Fractions

Fractionsareusedwhenwewanttoindicatepartsofthings.Afractionconsistsofanumeratorandadenominator.

Thedenominatortellsyouhowmanyequalpartstheobjectornumberhasbeendividedinto,andthenumeratortellshowmanyofthosepartsweareconcernedwith.

Examples:Divideabaseballgame,afootballgame,andahockeygameintoconvenientnumbersofparts.Writeafractiontoanswereachquestion.1.Ifapitcherplayedtwoinnings,howmuchofthewholebaseballgamedid(s)heplay?2.Ifaquarterbackplayedthreequartersofafootballgame,howmuchofthewholegamedid(s)heplay?

3.Ifagoalieplayedtwoperiodsofahockeygame,howmuchofthewholegamedid(s)heplay?

Solutions:1.Abaseballgameisconvenientlydividedintonineparts(eachaninning).Thepitcher

pitchedtwoinnings.Therefore,(s)heplayed ofthegame.Thedenominator

representstheninepartsthegameisdividedinto;thenumeratorrepresentsthetwopartsweareconcernedwith.

2.Similarly,therearefourquartersinafootballgame,andaquarterbackplayingthree

ofthosequartersplaysin ofthegame.

3.Therearethreeperiodsinhockey,andthegoalieplayedintwoofthem.Therefore,

(s)heplayedin ofthegame.

SimplestFormandEquivalence

Fractionshavingdifferentdenominatorsandnumeratorsmayactuallyrepresentthesameamount.Suchfractionsareequivalentfractions.

Forexample,thecirclebelowisdividedintotwoequalparts.Writeafractiontoindicatethathalfofthecircleisshaded.

= ofcircleisshaded.

Thecirclebelowisdividedintofourequalparts.Writeafractiontoindicatehowmuchofthecircleisshaded.

= ofcircleisshaded.

Thiscircleisdividedintoeightequalparts.Writeafractiontoindicatehowmuchofthecircleisshaded.

= ofthecircleisshaded.

Ineachcirclethesameamountwasshaded.Thisshowsyouthatthereismorethanonewaytoindicateonehalfofsomething.

Thefractions , ,and thatyouwroteareequivalentfractions,becausetheyallrepresent

thesameamount.Noticethatthedenominatoristwiceaslargeasthenumeratorineverycase.Anyfractionyouwritethathasadenominatorthatisexactlytwiceaslargeasthenumeratorwill

beequivalentto .

Example:Writeotherfractionsequivalentto .

Solution:Anyfractionthathasadenominatorthatistwiceaslargeasthenumerator: , ,

, ,etc.

Example:Writeotherfractionsequivalentto .

Solution:Anyfractionthathasadenominatorthatisfourtimesaslargeasthenumerator: ,

, , ,etc.

Example:Writeotherfractionsequivalentto .

Solution:Anyfractionthathasadenominatorthatisone-and-one-halftimesaslargeasthenumerator:

, , , ,etc.

Whenthenumeratoranddenominatorofafractioncannotbedividedevenlybythesamewhole

number(otherthan1),thefractionissaidtobeinsimplestform.Intheexamplesabove, , ,

and areinsimplestform.

TestYourself5

Directions:Indicatewhichofthefollowingfractionsareinsimplestform.Forthosefractionsthatarenot,trytosimplifythem.

1.

2.

3.

4.

5.

6.

7.

8.

9.

10.

Towriteequivalentfractionswherethenumeratorisnot1requiresonemorestep:

Example:Whatistheequivalentfractionfor using10asadenominator?

Solution:Each isequivalentto ;therefore, isequivalentto .

Thequickestwaytofindanequivalentfractionistodividethedenominatorofthefractionyouknowintothedenominatoryouwant.Taketheresultandmultiplyitbythenumeratorofthefractionyouknow.Thisbecomesthenumeratoroftheequivalentfraction.

Example:Change toanequivalentfractionhaving16asadenominator.

Solution:16÷8=2;2×3=6;Answer:

Example:Change intoequivalentfractionshaving8,12,24,and32asdenominators.

Solution: = (8÷4=2;2×3=6)

= (12÷4=3;3×3=9)

= (24÷4=6;6×3=18)

= (32÷4=8;8×3=24)

TestYourself6

Directions:Trytheproblemsbelow.

1.

2.

3.

4.

5.

6.

7.

8.

9.

10.

Afractionthathasanumeratorlargerthanthedenominatoriscalledanimproperfraction.Anumberexpressedasanintegertogetherwithaproperfractioniscalledamixednumber.

Examplesofimproperfractionsinclude , ,and .Notethateachisinsimplestform

becausethenumeratoranddenominatorcannotbedividedevenlybyanumberotherthan1.

Examplesofmixednumbersinclude1 ,1 ,and1 .Thesearecalledmixednumbersbecause

theyhaveawholenumberpartandafractionalpart.Thesemixednumbersareequivalenttotheimproperfractionsgivenabove.

Torenameamixednumberasanimproperfractioniseasy:

Example:Rename2 asanimproperfraction.

Solution:Thewholenumber2contains8fourths.Addtoitthe tocreatetheequivalent

fraction .

Analternativewayoffiguringthisistomultiplythedenominatorofthefractionbythewholenumberandaddthenumerator:

Example:Rename2 asanimproperfraction.

Solution:4×2=8; + =

Torenameanimproperfractionasamixednumber,justproceedbackward.

Example:Rename asamixednumber.

Solution:Dividethedenominatorintothenumeratorandusetheremainderasthefraction:

9÷4=2;R=1; =2

TestYourself7

Directions:Trythese,renamingeachasitsequivalentform.

1.3

2.2

3.11

4.7

5.1

6.

7.

8.

9.

10.

AdditionofFractions

Toaddfractionsyoumustfirstbesurethattheaddendshavethesamedenominators.

Example:Add: .

Solution:Thedenominatorsarethesame,sojustaddthenumeratorstoarriveattheanswer,

,or1 .

Inmostcases,denominatorswillbedifferent,soyouwillhavetofindacommondenominator.

Example:Add: .

Solution: isequivalentto ,so + = .

Example:Add: .

Solution:Thisproblemistrickier.Itrequiresthewritingofequivalentfractionsinacommondenominatortowhich4and3caneasilybeconnected.

isequivalentto .

isequivalentto .

Wecannowaddthefractionsbecausewehavewrittenequivalentfractionsinacommondenominator:

Therefore, .

Seventwelfthsisinsimplestform,because7and12donothaveawholenumber(otherthanl)thatdividesintobothevenly.

HowtoFindaCommonDenominator

Youcanalwaysfindacommondenominatorbymultiplyingthedenominatorstogether.

Example:Findacommondenominatorfor and .

Solution:Multiply4×8;32isacommondenominator.However,16and24arealsocommondenominatorsfor4and8.

Don’tworryaboutfindingthelowestcommondenominatorwhenyouareaddingfractions.Whenyouseethesum,youwillprobablynoticethatthefractioncanbesimplifiedtosimplestform.Whenyouhavesimplifiedthefractionasfarasyoucan,youhaveprobablyfoundthelowestcommondenominator.

TestYourself8

Directions:Trythefollowingproblems.Findacommondenominator,thensimplifytheanswertoitssimplestform.

1.

2.

3.5 +

4.

5.

6.

7.

8.

9.2 +

10.

Subtraction

Tosubtractonenumberfromanothermeanstofindthedifferencebetweenthemonthenumberline.Thenumberbeingsubtractediscalledthesubtrahend;thenumberbeingsubtractedfromistheminuend.

Anumberline,suchastheoneabove,letsyouseethedifferencebetweennumbersbeforesubtracting.Forexample,thedifferencebetweensixandtwoisfourunits;therefore,6–2=4.Or,thedifferencebetweensixandzeroissixunits;therefore,6–0=6.Ifyourememberthatwhensubtractingnumbersyouareinterestedinthedifferencebetweenthemonthenumberline,youwillunderstandsubtractioneasily.

Subtractioncannotoccurinanyorder,ascanaddition.Forexample,6–3isnotthesameas3–6,noris100–1thesameas1–100.

SubtractingWholeNumbers

Tofindthedifferencebetweenapairofwholenumbers,writethesmallerbeneaththelarger.Borrowagroupfromthenextlargercolumnwhenyouaresubtractingalargernumeralfromasmallerone.

Example:Findthedifferencebetween6937and4178.

Solution:6937–4178

2759

Step1:Beginbyborrowingagroupfromthe3(leaving2)tomake17.Eightfrom17is9.

Step2:Next,borrowagroupfromthe9(leaving8)tomake12.Sevenfrom12is5.

Step3:Subtract1from8,giving7.

Step4:Subtract4from6,giving2.

Thecorrectdifference,2759,canbecheckedbyaddingittothenumberyoufirstsubtracted,4178.Youshouldendwith6937again.

TestYourself9

Directions:Practicesubtractionwiththeproblemsbelow.

1.703–98

2.1762–983

3.429–108

4.63,921–4930

5.278–88

6.9000–699

7.13,706–4838

8.863–92

9.7333–6444

10.290,723–176,731

SubtractingDecimals

Ifnecessary,reviewthebasicsofdecimalnotationthatyoustudiedearlierunderdecimaladdition.Mostofthesameguidelinesapplytothesubtractionofdecimals.

Whenfindingthedifferencebetweentwonumberswrittenindecimalnotation,besuretoarrangethesmallerbeneaththelarger,keepingthedecimalpointsinline.Thenproceedjustasifyouweresubtractingwholenumbers.

Aswithaddition,youmaywanttowriteinzerosto“fillin”thosedecimalplaceshavingnonumeralsinthem.

Example:Subtract22.02from23.001.

Solution:23.001–22.02000.981

Rememberthatthedecimalpointintheanswerisplaceddirectlybelowthedecimalpointsofthesubtrahendandminuend.

TestYourself10

Directions:Trythefollowingproblemsaspractice.

1.16.17–0.9902

2.0.83–0.0624

3.0.918–0.759

4.0.360–0.204

5.6.57–2.43

6.28.47–3.622

7.809.03–24.9

8.37.94–0.4223

9.38.83–9.003

10.67.2115–3.79

SubtractingFractions

Tofindthedifferencebetweentwofractionsthathavethesamedenominators,simplysubtractthenumerators,leavingthedenominatorsalone.

Example:Findthedifferencebetween and .

Solution: .Simplifiedtosimplestform: .

Whensubtractingmixednumbersthathavethesamedenominators,onemorestepisrequired:

Example:Subtract2 from9 .

Solution:Step1:Renamethemixednumbersasimproperform:

2 = (4×2+3=11)

9 = (4×9+1=37)

Step2:Subtractthenumerators,leavingthedenominatorsalone:

Simplifiedtosimplestform=

Renamedasamixednumber=6

Beforetryingtosubtractmixednumbershavingdifferentdenominators,reviewtheprocedureforwritingequivalentfractionsasitwasexplainedearlierinthebook.Findingthedifferencebetweentwomixednumbersiseasyafteryoufindcommondenominatorsandequivalentfractions:

Example:Subtract3 from6 .

Solution:Step1:Renameeachmixednumberasitsimproperform:

3 = (4×3+3=15)

6 = (8×6+3=51)

Step2:Findacommondenominator.Forfourthsandeighths,eighthsareagoodchoice.

Step3:Writetheequivalentfractions:

(8÷4=2;2×15=30)

Step4: ,or2

TestYourself11

Directions:Practicethefollowingproblems.

1.

2.

3.1 –

4.9 –

5.3 –1

6.

7.7 –

8.

9.2 –

10.4 –

Multiplication

Multiplicationisashortcutforaddition.Forexample,ratherthanaddanumber12times,wesimplymultiplyitby12.Theresultofmultiplyingtwonumbersiscalledtheproduct.Thenumbersthataremultipliedarecalledfactors.

Mosterrorsinmultiplicationresultfromnothavingmemorizedthemultiplicationtables.Oneofthebestwaystoimproveyourmathematicsabilityistopracticerecitingthemultiplicationtablesuntilyouknowthemthoroughly.Youcanfindthesetablesinanyarithmetictextbook.

MultiplicationofWholeNumbers

Thefollowingexamplesshowtheproceduresusedinmultiplyingwholenumbers.Notethecarewithwhichthepartialproductsarewrittenincolumnsbeforebeingadded.Youshouldaimtobejustascareful.

Example:Multiply435by253.

Solution:435×2531305

Eachlineisapartialproduct.

2175;870

product 110,055

TestYourself12

Directions:Practiceyourmultiplicationusingthefollowingexamples,andtimeyourself.Thenmakeuptensimilarproblemsandtryagain,emphasizingspeedandaccuracy.

1.726×29

2.33×l4

3.1064×397

4.512×136

5.3112×223

6.11,550×32

7.4619×550

8.217×118

9.1214×104

10.64,397×1472

MultiplicationofDecimals

Multiplicationofdecimalsisnoharderthanmultiplicationofwholenumbers.However,youmustrememberonemorestep:tocountoffthecorrectnumberofdecimalplacesintheproduct.

Theruleforcountingoffthecorrectnumberofdecimalplacesis:Thenumberofdecimalplacesintheproductisequaltothetotalnumberofdecimalplacesinbothfactors.

Thefollowingthreeproblemsshouldmakethisruleclear:

Example:3.11 2decimalplaces×2 0decimalplaces6.22 2decimalplaces

Example:3.11 2decimalplaces×0.2 1decimalplace0.622 3decimalplaces

Example:0.311 3decimalplaces×0.2 1decimalplace0.0622 4decimalplaces(Azerowasaddedtomakeafourthplace.)

Whenmultiplyinglargerdecimals,lineupthepartialproductscarefully,countupthetotalnumberofdecimalplacesinthefactors,andplacethedecimalpointintheproductthesamenumberofplacesfromthelastnumeral.

Example:2.301 3decimalplaces×18.73 2decimalplaces6903

1610718408230143.09773 5decimalplacesfromright

TestYourself13

Directions:Practicethefollowingproblems.

1.7209×0.3741

2.103.2×97.1

3.638.63×83.6

4.29.10×0.04

5.7.720×0.34

6.8143.6×20.13

7.0.0034×0.276

8.93.2×1.26

9.103.621×0.43

10.72.7×63.8

MultiplicationofFractions

Themultiplicationoffractionsissimpleandstraightforward.Itconsistsoftwosteps:MultiplyingthenumeratorbythenumeratorandthedenominatorbythedenominatorSimplifyingtheproduct(answer)tothesimplestform

Example:Multiply by .

Solution:Wemultiplystraightacross.

Step1:

Step2:Simplify tosimplestform.

Thereisnoneedtofindcommondenominatorswhenmultiplying(ordividing)fractions.Simplyremembertomultiplystraightacrossandsimplifytheproductifnecessary.

Insomeproblems,onefactormaybeamixednumberandtheotheraproperfraction.Inthatcase,proceedasfollows:

Example:Multiply by2 .

Solution:Writethefractionsasbefore,renaming2 asanimproperfraction.Multiply

straightacross:

Step1:

Step2:Becausetheproductisinsimplestform,leaveitthatwayorrewriteitasthemixednumber2 .

TestYourself14

Directions:Practicethefollowingproblems.

1.

2.

3.

4.

5.

6.100

7.

8.

9.

10.

SquaresandSquareRoots

Squares

Theproductofanumbertimesitselfiscalledthesquareofthatnumber.Forexample,9isthesquareof3;16isthesquareof4;and25isthesquareof5.Anynumberweworkwithhasasquare;wesimplymultiplythenumberbyitselftofindit.

Examples:Findthesquaresofthefollowingnumbers:

(A)15

(B)3.22

(C)

(D)0.01

(E)125

(F)

Solutions:(A)15×15=225

(B)3.22×3.22=10.3684

(C)

(D)0.01×0.01=0.0001

(E)125×125=15,625

(F)

Thewordsquareisalsousedasaverbtodescribetheprocessusedtofindtheproductofanumbertimesitself.Tosquareanumbermeanstomultiplyitbyitself.Specialnotationisusedwhenworkingwithsquares.Ratherthanwrite15×15=225,weuseanexponent,152=225.

Thereisfurtherdiscussionandpracticeinusingexponentsinthesection“Exponents”laterinthischapter.

Youwillfindthattest-makersandmathematicstextbookwritersrelyoneasilyrecognizedsquaresinmanyproblems.Forthisreason,wethinkitisveryimportantthatyoulearntorecognizecertaincommonnumbersassquaresofothernumbers.

Thetablelaterinthischapterlistsnumbersandtheirsquares.Notethatonceyoulearnthelinkbetweenanumberanditssquare,youcanapplythatknowledgeregardlessofwherethedecimalpointislocated.Forexample,ifyouknowthat152=225,youalsoknowthat1.52=2.25,and0.152=0.0225,and1502=22,500.

Studythetablecarefully.Itisagoodideatomemorizethesquaresofthenumbers1through25,30,40,50,60,70,80,90,and100,althoughyoushouldnotspendagreatdealoftimeonit.Theimportantthingistounderstandtherelationshipsinthetableandtolinkinyourmindawholenumberwithitssquare.

SquareRoots

Everynumberthatisasquarehasasquareroot.Inthetablelaterinthischapter,forexample,youcanseethatthenumbermultipliedbyitselftofindthesquareisthesquarerootofthesquare.

Forexample,15isthesquarerootof225,1.5isthesquarerootof2.25,and0.15isthesquarerootof0.0225.

TestYourself15

Directions:Trythese,usingthetableofsquares.Findthesquarerootofthefollowing.

1.32,400

2.1.96

3.225

4.441

5.5.29

6.625

7.6.25

8.900

Aspecialnotationcalledaradical( )isusedwhenworkingwithsquareroots.Forexample,=3isread:“Thesquarerootofnineequalsthree.”Theradicaloverthenumbernineisread

“squarerootof.”

Ifyouareaskedtofindthesquarerootofafraction,simplyconsiderthenumeratoranddenominatorasseparatenumbers,andfindthesquarerootofeach.

Example:Find: .

Solution: .Thesquarerootof is .

Check:

Nodoubtyouhavelearnedwaysoffindingsquarerootsofnumbersthatdon’tfitsoneatlyintoatablesuchastheonebelow.However,becausesuchproblemsoccursorarelyonhighschoolentranceexaminations,wewillnotreviewthosemethodsinthisbook.Inmostcases,youwillbeabletoestimatethesquarerootofanumberaccuratelyenoughtoselectthecorrectanswer.Thatiswhywesuggestyoustudythetablecloselyandlearntorecognizethosenumbersandtheirsquaresandsquareroots.

TestYourself16

Directions:Trythefollowingproblemswithoutusingthetable.

1.142=

2. =

3. =

4. =

5.1002=

6.252=

7.202=

8. =

9. =

10.2.52=

Youwillfindfurtherworkwithexponentsandotherrootsinthesectionentitled“Exponents”.

RelatedNumbersandTheirSquares

12=1 0.12=0.01 102=10000

22=4 0.22=0.04 202=40000

32=9 0.32=0.09 302=90000

42=16 0.42=0.16 402=1600

52=25 0.52=0.25 502=2500

62=36 0.62=0.36 602=3600

72=49 0.72=0.49 702=4900

82=64 0.82=0.64 802=6400

92=81 0.92=0.81 902=8100

102=100 1.02=1 1002=10,000

112=121 1.12=1.21 1102=12,100

122=144 1.22=1.44 1202=14,400

132=169 1.32=1.69 1302=16,900

142=196 1.42=1.96 1402=19,600

152=225 1.52=2.25 1502=22,500

162=256 1.62=2.56 1602=25,600

172=289 1.72=2.89 1702=28,900

182=324 1.82=3.24 1802=32,400

192=361 1.92=3.61 1902=36,100

202=400 2.02=4000 2002=40,000

212=441 2.12=4.41 2102=44,100

222=484 2.22=4.84 2202=48,400

232=529 2.32=5.29 2302=52,900

242=576 2.42=5.76 2402=57,600

252=625 2.52=6.25 2502=62,500

Youcanrefertothetabletofindthesquarerootsofsomecommonlyusednumbers.

Division

Theprocessofdivisionisusedtodeterminethenumberofpartsintowhichanothernumbercanbedivided.Adivisionproblemismadeupofthenumberthatisbeingdivided,calledthedividend;thenumberthatisdoingthedividing,calledthedivisor;andtheanswer,calledthequotient.

DivisionofWholeNumbers

Thefollowingexamplesshowhowlongdivisionisusedwhenonewholenumberisdividedintoanother.Notethatintwocasestheremainderisexpressedasthenumeratorofafractionhavingthedivisorasthedenominator.Analternativewaytoexpresstheremainderisbycontinuingthelongdivisionandcreatingaquotienthavingadecimalremainderinstead.

Studytheseexamplescarefully.Notethatthedecimalpointinthequotientislocateddirectlyabovethedecimalpointinthedividend.Notealsothatthefirstnumeralofthequotientisplacedverycarefullyinthecorrectdecimalplace.

Example:

or

Example:

or

Example:

or

Whenusingdecimalquotientsandremainders,itisusuallyallowabletostopafterthreeorfourdecimalplaceshavebeencalculated.Notethatyoucancontinuetodivide—insomecases,forever.Yourgoalshouldbetodivideonlyasfarasisnecessaryforyoutocomeupwithananswerthatcorrespondstotheanswerchoicesgiveninaparticularquestion.

TestYourself17

Directions:Trythefollowingproblemsforpractice.

1.3867÷47

2.935÷22

3.103÷272

4.5760÷139

5.5015÷462

6.4211÷104

7.76÷93

8.2200÷1113

9.678÷803

10.1930÷48

DivisionofDecimals

Whenthedivisorisnotawholenumberbuthastenths,hundredths,thousandths,andsoforthaspartofit,oneadditionalstepisrequiredtosolvetheproblem.Forexample:

Here,wemovethedecimalpointinthedivisorasmanyplacestotherightasnecessarytomakethedivisorawholenumber.Thenweaddthatsamenumberofplacestothedividend.Intheexampleabove:

Now,theproblembecomesoneofsimplewholenumberlongdivision:

Thus,250÷12.5=20.

Example:Divide0.666into0.333.

Solution: Movethedecimalpoint,andaddzeros.

Inthosecasesinwhichthedivisorisnotawholenumber,besuretomovethedecimalpointthecorrectnumberofplacesinthedivisoranddividend.Then,placethedecimalpointinthequotientdirectlyaboveitsnewplace.

TestYourself18

Directions:Trytheproblemsbelowforpractice.

1.0.396÷1.3

2.493.2÷85.63

3.1034.62÷7.88

4.972.1÷0.0543

5.42.678÷501.3

6.45.776÷62.11

7.9.1494÷933.06

8.203.4÷38.32

9.280.420÷1.980

10.0.092÷47.4284

DivisionofFractions

Ifyoumultiplyfractionsaccurately,youcandividethemjustaseasily.Forexample,todivide

by ,multiplybythereciprocalofthedivisor.Theproblem ÷ thusbecomes .

Simplymultiplystraightacross,andsimplifytheresultingfraction:

or1

Thisprocedureworkswhetherbothnumbersarefractionsornot.Ifawholenumberistobe

inverted,youshouldchangeittoafractionfirst.Forexample ÷2becomes ÷ .To

divide,multiplybythereciprocalofthedivisor:

Anotherexampleis3 ÷ ,whichbecomes ÷ .

Todivide,multiplybythereciprocalofthedivisor:

TestYourself19

Directions:Whendividingfractions,estimatingthequotientcanbeanimportanthelp.Trythefollowingproblemstopracticedividingfractions.

1. ÷ =

2.4 ÷2 =

3. ÷ =

4. ÷9 =

5. ÷6 =

6.20 ÷ =

7. ÷6=

8. ÷ =

9. ÷2=

10. ÷ =

OperationsUsingFractionsandDecimals

Itisnotuncommontohavefractionsanddecimalsappearinthesameproblem.However,becausetheyaredifferentformsofnotation,onemustberenamedastheotherbeforeyoucanperformanyofthebasicoperationswiththem.Youmaynotadd,subtract,multiply,ordivideusingbothkindsofnotationatonce.

Forexample,toadd3 and1.35,youmustrenameeither3 todecimalnotationor1.35to

fractionalnotation.Theexamplesbelowwillshowyoubothways.

Torenameafractionasitsequivalentdecimal,simplydividethedenominatorintothenumerator.

Example:Findthedecimalequivalentof .

Solution:

=0.375

Thisprocesswillalwayswork,regardlessofthefraction.Itwillbeusefultomemorizethedecimalfractionalequivalentsonpage267.Remember,though,thatyoucanalwaysfindadecimalequivalentofafractionbydividingitsdenominatorintoitsnumerator.

Torenameadecimalasafractionrequiresasimilarmethod.Simplywritethedecimalasafraction,andsimplifyitasnecessary.

Example:Rename0.125asafraction.

Solution:0.125isread“onehundredtwenty-fivethousandths.”

Writeitas ,andsimplifyitbydividingthenumeratoranddenominatorby

125:

TestYourself20

Directions:Practicetheproblemsbelow.Renamefractionsasdecimalsanddecimalsasfractions.

1.

2.0.75

3.

4.

5.0.435

6.0.18

7.

8.0.45

9.0.125

10.

CombiningOperations

Frequently,morethanoneoperationmustbeusedtoarriveatananswertoaproblem.Thatis,aseriesofcalculationsmayhavetobestrungtogethertoproducethecorrectanswer.Problemsofthistypearenomorecomplicatedthantheonesyouhavedonealready,butrequireoneadditionalpieceofknowledge.

Forexample,howwouldyouapproachthisproblem?

2 +10÷1 –3.125×0.80

Whichoperationswouldyoudofirst?Fortunately,theanswerisclear,ifyouobservethefollowingtworules:

Domultiplicationanddivisionfirst,inorder,fromlefttoright.Doadditionandsubtractionsecond,inorder,fromlefttoright.

Therefore,thefirststepistoputparenthesesaroundthemultiplicationanddivisionoperations.

2 + –(3.125×0.80)

Then,dotheoperationsinsidetheparentheses,andsimplify.

2 +8–2.5=8

Anotherexampleisaproblemthatlookssimplebutmaybeconfusing,unlessyouunderstandtheorderofoperationsdescribedabove.

2÷3+1÷2+2×3+1

Tosolvethisproblem,putparenthesesaroundthemultiplicationanddivisionoperations.Then,performtheoperationsinsidetheparentheses,andsimplify.

(2÷3)+(1÷2)+(2×3)+1=

+ +6+1=8

Solvethefollowingproblems,interchangingfractionalanddecimalnotationwhennecessary.Itisuptoyoutodecide,baseduponwhicheveriseasier,whethertorenamethedecimalsas

fractionsortorenamethefractionsasdecimals.Forexample,whenadding5 and0.6157,itis

easiertorename5 as5.750andadditto0.6157ratherthantheotherwayaround.Youmust

makethesamechoicewhenmultiplyingordividingfractionswithdecimals.

TestYourself21

Directions:Now,goontotheproblemsthatfollow.

1.7 ×0.467

2.3.27–2

3.13 –5.2

4.6 ÷0.375

5.9 ÷2.76

6.2 ×10.65

7.1 +0.05

8.1 –0.789

9.4 ×0.9

10.6 +15.65

Percentage

Itisagoodideatohavestudiedalloftheprevioussectionsaboutdecimalsinthisbookbeforestartingthissection.Ifyoucanworkeasilywithdecimals,percentagesshouldpresentnodifficultyforyou.

Onepercentisonehundredthofsomething.Thelastsyllableofthewordpercent,-cent,isthenamewegivetoonehundredthofadollar.

Onepercentof$1,then,isonecent.Usingdecimalnotation,wecanwriteonecentas$0.01,fivecentsas$0.05,twenty-fivecentsas$0.25,andsoforth.

Twenty-fivecentsrepresentstwenty-fivehundredthsofadollar.Ratherthansaythatsomethingissomanyhundredthsofsomethingelse,weusethewordpercent.Twenty-fivecents,then,istwenty-fivepercentofadollar.Weusethesymbol%tostandforpercent.

Percentage(“hundredthsof”)isaconvenientandwidelyusedwayofmeasuringallsortsofthings.Bymeasuringinhundredths,wecanbeverypreciseandnoticeverysmallchanges.

TestYourself22

Directions:SupposeJanedroveacar100milesonMondayand101milesonTuesday.NoticethatshedroveonepercentfartheronTuesdaythanonMonday.Forthedayslistedbelow,byhowmuchpercentmore,orless,didshedrivecomparedtoMonday,ifshedrovethefollowingdistances?

1.Wednesday,110miles

2.Thursday,140miles

3.Friday,100miles

4.Saturday,99miles

5.Sunday,90miles

TestYourself23

Directions:Bywhatpercentisthesecondnumberofeachpairlistedbelowlargerorsmallerthanthefirstnumber?

1.100,150

2.100,73

3.100,80

4.100,1

5.100,0.5

6.100,200

7.100,450

8.100,0.01

Percentageisnotlimitedtocomparingothernumbersto100.Youcandivideanynumberintohundredthsandtalkaboutpercentage.

Example:Find1%of200.

Solution:1%of200isonehundredthof200.

200÷100=2

Usingdecimalnotationwecancalculateonepercentof200by:

200×0.01=2

Similarly,wecanfindapercentageofanynumberwechoosebymultiplyingitbythecorrectdecimalnotation.Forexample:

5percentoffifty: 0.05×50=2.53percentofonehundredfifty: 0.03×150=4.5

10percentofsixty: 0.10×60=6

TestYourself24

Directions:Dothefollowingproblemsforpractice.

1.10percentofeight

2.25percentofsixty

3.11percentoften

4.1percentofthreehundredfifty

5.99percentofeighty

6.33percentofthirty-three

7.75percentofonehundredtwelve

8.50percentoftwohundred

Allpercentagemeasurementsarenotbetweenonepercentandonehundredpercent.Wemaywanttoconsiderlessthanonepercentofsomething,especiallyifitisverylarge.

Forexample,ifyouwerehandedabookonethousandpageslongandweretoldtoreadonepercentofitinfiveminutes,howmuchwouldyouhavetoread?

1000×0.01=10pages

Quiteanassignment!Youmightbargaintoreadonehalfofonepercent,oronetenthofonepercent,inthefiveminutesallottedtoyou.

Usingdecimalnotation,wewriteonetenthofonepercentas0.001,thedecimalnumberforonethousandth.Ifyourememberthatapercentisonehundredthofsomething,youcanseethatonetenthofthatpercentisequivalenttoonethousandthofthewhole.

Inpercentnotation,onetenthofonepercentiswrittenas0.1%.Onhighschoolentranceexams,studentsoftenmistakenlythinkthat0.1%isequalto0.1.Asyounowknow,0.1%isreallyequalto0.001.

TestYourself25

Directions:Forpractice,renamethefollowingpercentsasdecimalnotationandviceversa.

1.1%

2.1.2%

3.0.5%

4.0.05

5.0.001

6.0.003

7.1.5%

8.0.015

9.15%

10.0.0001

Sometimesweareconcernedwithmorethan100%ofsomething.But,youmayask,if100%constitutesallofsomething,howcanwespeakofmorethanallofit?

Wherethingsaregrowing,orincreasinginsizeoramount,wemaywanttocomparetheirnewsizetothesizetheyoncewere.Forexample,supposewemeasuredtheheightsofthreeplantstobe6inches,9inches,and12inchesoneweek,anddiscoveraweeklaterthatthefirstplantisstill6inchestallbutthesecondandthirdonesarenow18inchestall.

•The6-inchplantgrew0%,becauseitdidn’tgrowatall.•Thesecondplantadded100%toitssize.Itdoubledinheight.•Thethirdplantadded50%toitsheight.

Wecanalsosay:•Thefirstplantis100%ofitsoriginalheight.

•Thesecondplantgrewto200%ofitsoriginalheight.

•Thethirdplantgrewto150%ofitsoriginalheight.

TestYourself26

Directions:Practicethefollowingproblems.Foreachpairofnumbers,tell(1)whatpercentageofthefirstnumberwouldhavetobeaddedtogetthesecondnumber,and(2)whatpercentageofthefirstnumberisthesecondnumber.

Example:100,150

Solution:Step1:Fiftypercentof100wouldhavetobeaddedto100toget150.

Step2:150represents150%of100.

1.50,75

2.50,100

3.10,15

4.100,132

5.20,24

6.1,1.5

7.0.5,0.75

8.33,44

9.55,55

10.100,1000

Youmaywanttoknowacertainpercentageofafraction.

Example:Whatis50%of ?orWhatis20%of1 ?

Solution:Rename50%asitsequivalentfractionandmultiply ,or

rename andchange50%todecimalnotationandmultiply:

0.50×0.666=0.333

Similarly,thesecondexamplecanbecalculatedas:

or0.20×1.5=0.30

Herearesomecommonpercentageandfractionalequivalentsyoushouldremember:

•Tenpercent(10%)=onetenth(0.10)or

•Twelveandone-halfpercent(12.5%)=oneeighth(0.125)or

•Sixteenandtwo-thirdspercent =onesixth(0.16666 )or

•Twentypercent(20%)=onefifth(0.20)or

•Twenty-fivepercent(25%)=onequarteroronefourth(0.25)or

•Thirty-threeandone-thirdpercent =onethird(0.33 )or

•Thirty-sevenandone-halfpercent(37.5%)=threeeighths(0.375)or

•Fiftypercent(50%)=onehalf(0.50)or

•Sixty-twoandone-halfpercent(62.5%)=fiveeighths(0.625)or

•Sixty-sixandtwo-thirdspercent =twothirds(0.66 )or

•Seventy-fivepercent(75%)=threequartersorthreefourths(0.75)or

•Eighty-threeandone-thirdpercent =fivesixths(0.83 )or

•Eighty-sevenandone-halfpercent(87.5%)=seveneighths(0.875)or

TestYourself27

Directions:Dothefollowingforpractice.

1.Whatis75%of ?

2.Whatis10%of3 ?

3.Whatis40%of ?

4.Whatis27%of4 ?

5.Whatis33 %of6 ?

6.Whatis16%of7 ?

TypesofPercentageProblems

Therearethreetypesofpercentageproblems:Theproblemasksyoutofindapercentageofacertainnumber.

Example:Find22%of50.

Solution:0.22×50=11Theproblemgivesyouanumberandthenasksyoutofindanothernumber,ofwhichthefirstisacertainpercentage.

Example:30is20%ofwhatnumber?

Solution:If30is20%ofanumber,itisonefifthofit.Thirtyisonefifthof150.Or,30=0.20ofthenumber.Longdivisionleadstotheanswer:150.

Problemslikethisareeasilysolvedusingashortalgebraicsentence.Weknowthat30equals20%ofanunknownnumber.Thus,30=0.20n.Dividingbothsidesby0.20,wegettheanswer,n=150.

Theproblemasksyoutofindwhatpercentonenumberisofanother.

Example:15iswhatpercentof60?

Solution:Thisisasimpledivisionprobleminlongdivision.

Analternativemethodistosimplifythefraction toitssimplestform, .The

percentageequivalentof is25%.

FinalWordsonPercentage

Whensolvingproblemsinvolvingpercentages,becarefulofcommonerrors:•Readthenotationcarefully.0.50%isnotfiftypercent,butonehalfofonepercent.•Readtheproblemcarefully.It’simportantnottomisreadtheproblemwhensolvingforpercentageincreasesordecreasesinsize.

•Usecommonsense.Ifyouwanttofindlessthan100%ofanumber,yourresultwillbesmallerthanthenumberyoustartedwith.Forexample,43%of50islessthan50.

•Usingcommonsenseworksintheotherdirectionaswell.Forexample,70is40%ofwhatnumber?Thenumberyouarelookingformustbelargerthan70,because70isonly

ofit.Moreover,youcanestimatethatthenumberyouarelookingforwillbealittle

morethantwiceaslargeas70,because70islessthanhalf(50%)ofthatnumber.

TestYourself28

Directions:Solvetheseproblems.

1.28%of45is____.

2. %of75is____.

3.2 %of____is75.

4.35is____%of70.

5.50is____%of12.5.

6.1.5is____%of9.

7.3%of1.75is____.

8.12is5%of____.

9.130%of60is____.

10.180is200%of____.

11.2.5%of50is____.

12.60is____%of90.

13.30is20%of____.

14.66 %of____is104.

Exponents

Itisfrequentlybeneficialtouseshorthandmethodsofwritingnumbersinmathematics.Oneofthemostcommonistheuseofexponents.

Anexponentisanumberthattellsyouhowmanytimesthenumberitrefersto(calledthebase)isusedasafactorinagivencalculation.

Forexample:

isashorthandwayofwriting10•10•10,or1000.Notethattheexponentiswrittentotherightandabovethebase,and,toavoidconfusion,thenumeralissmallerinsize.

TestYourself29

Directions:Takeaminutetowriteoutthefollowingnumbersaswasdoneearlier.Thereisnoneedtocalculatetheactualproduct.Forexample:54=5•5•5•5.

1.107

2.13

3.32

4.

5.(0.45)5

6.112

7.02

8.x3

9.b8

10.(ab)5

Exponentsaremostusefulincertainscientificrealmsinwhichverylargeorverysmallnumbersareinvolved.Forexample,itismucheasiertowrite109ratherthan1,000,000,000.

Geometryisanothersubjectthathasfrequentuseforexponents.Areaandsurfaceareaaremeasuredinsquareunitssuchassquarefeet,squareinches,andsoforth.Volumeismeasuredincubicfeet,cubicinches,orinothercubicunits.

Forexample,theareaofafloormightbe200squarefeet.Usinganexponent,wecanwrite200ft.2.Thevolumeofacubemightbe8cubicfeet.Wecanwritethisas8ft.3.

Theexponent“2”isread“square”or“squared.”Theexponent“3”isread“cube”or“cubed.”

Forexponentsotherthan2and3,weusethephrase“tothe____power.”Forexample,56wouldberead“fivetothesixthpower.”

Whentheexponentisnotwritten,aswhenwewritemostnumbers,theexponentisunderstoodtobeequalto1.Anynumbertothefirstpowerisequivalenttoitself.

Forexample,10=101.Wedonotwrite1asanexponent.

Therearetwomajorrulestohelpyoucalculatenumberswritteninexponentialform.Bothrequirethatthebasesofthenumbersbethesame.

Thefirstruleinvolvesmultiplyingnumbersinexponentialformhavingthesamebase.Insuchinstances,theproductmaybefoundbyaddingtheexponentsasshownonthenextpage.

Example:Multiply103by105.

Solution:103•105=103+5=108

Example:Multiply23by24.

Solution:23•24=23+4=27

Example:Multiplyx2byx3.

Solution:x2•x3=x2+3=x5

Example:Multiply33by3.

Solution:33•3=33+1=34

Itisveryimportanttonotethatthebaseswereequalineachoftheprecedingproblems.Theexponentsmaybedifferent.

Thesecondruleinvolvesdivisionofnumbersinexponentialformhavingthesamebase.Infindingtheproductofnumbersinexponentialform,weaddedtheexponents.Tofindtheirquotient,wesubtracttheexponentofthedivisorfromthatofthedividendasshownbelow:

Example:Divide103by102.

Solution:103÷102=103–2=101=10

Example:Divide56by53.

Solution:56÷53=56–3=53

Example:Dividex4byx2.

Solution:x4÷x2=x4–2=x2

Example:Dividea3bya.

Solution:

Foreachoftheprecedingexamples,youmaywanttocalculatetheprobleminstandardfashiontoprovetoyourselfthatitworks.

TestYourself30

Directions:Practicetheproblemsbelow.

1.13÷12

2.610÷68

3.155•153

4.M3•M3

5.1010÷108

6.36•32

7.a2•a3

8.10010÷1009

9. •

10.85÷84

11.a5÷a3

12.

13.x2•x2

ZeroandNegativeExponents

Youmayhavenoticedwhilepracticingthatyoucandividenumberswritteninexponentialnotationandendupwithanexponentthatisnegativeorequaltozero.Bothresultsareperfectlyacceptable;theywillbementionedonlybrieflybecauseyouwillprobablynotencounterthemonthehighschoolentranceexaminationyoutake.

Example:Divide53by53.

Solution:53÷53=53–3=50

Youmayhaverealizedthatwhenwedivideanumberbyitself,theresultis1.Therefore,anynumber(orvariablerepresentinganumber)havingzeroasanexponentisequalto1.Forexample:

106÷106=106–6=100=1

x3÷x3=x3–3=x0=1

Whathappensifwedivide53by54?

53÷54-53–4=5–1

Noticethattheexponentbecomesnegative.Anegativeexponentisthesymbolforareciprocal.Forexample:

Youmaywanttomultiplytheseexamplesouttoconvinceyourselfoftheirtruth.

Algebra

Ifyouarefinishingtheeighthgradethisyear,youmaynotyethavehadaformalalgebraclass.

Nevertheless,youhaveprobablyusedalgebraictermsandexpressions,andyouhaveprobablysolvedsimpleequations.Thissectionreviewstheskillsyouhaveacquiredsofarandshowsyouthekindsofquestionsyoucanexpecttofindonahighschoolentranceexamination.

Thissectioncontainsareviewof:•Signednumbers•Variablesandcoefficients•Operationswithalgebraicexpressions•Evaluatingalgebraicexpressions•Solvingequations

SignedNumbers

Thenumberlineexiststobothsidesofzero.Eachpositivenumbertotherightofzerohasanegativecounterparttotheleftofzero.Thenumberlinebelowshowsthelocationofsomepairsofnumbers(+4,–4;+2,–2;+1,–1).

Becauseeachnumberofapairislocatedthesamedistancefromzero(althoughindifferentdirections),eachhasthesameabsolutevalue.Twoverticalbarssymbolizeabsolutevalue:

|+4|=|–4|=4

Theabsolutevalueof+4equalstheabsolutevalueof–4.Bothareequivalentto4.Ifyouthinkofabsolutevalueasdistancefromzero,regardlessofdirection,youwillunderstanditmoreeasily.Theabsolutevalueofanynumber,positiveornegative,isalwaysexpressedasapositivenumber.

AdditionofSignedNumbers

Whentwooppositelysignednumbershavingthesameabsolutevalueareadded,thesumiszero:

Example:+10+(–10)=0

Example:–1.5+(+1.5)=0

Example:–0.010+(+0.010)=0

Example:

Ifoneofthetwooppositelysignednumbersislargerinabsolutevalue,thesumisequaltotheamountofthatexcessandcarriesthesamesignasthenumberhavingthelargerabsolutevalue:

Example:+2+(–1)=+1

Example:–2.5+(+2.0)=–0.5

Example:+8+(–9)=–1

Example:

TestYourself31

Directions:Addthefollowinggroupsofnumbers.

1.–2,+4,–10,and–3

2.–1.2,2.6,and–0.0005

3.9.001,–9.002,and1.0

4.125,–130,–27,and63

5. ,–2,and–4

6. ,–1.25,and–6

7.

8.–100.1,–62.35,and42.2

9.0.0002,–3.6,and1.85

10.68.25,–74.35,and6.10

SubtractionofSignedNumbers

Subtractionistheoperationthatfindsthedifferencebetweentwonumbers,includingthedifferencebetweensignednumbers.

Whensubtractingsignednumbers,itishelpfultorefertothenumberline:

Forexample,ifwewanttosubtract+2from+5,wecanusethenumberlinetoseethatthedifferenceis+3.Wegivethesigntothedifferencethatrepresentsthedirectionwearemovingalongthenumberline,fromthenumberbeingsubtractedtothenumberfromwhichwearesubtracting.Inthiscase,becausewearesubtracting+2from+5,wecountthreeunitsinapositivedirectionfrom+2to+5onthenumberline.

Whensubtractingsignednumbers:•Thedistancebetweenthetwonumbersgivesyoutheabsolutevalueofthedifference.•Thedirectionyouhavetomovefromthenumberbeingsubtractedtogettothenumberfromwhichyouaresubtractinggivesyouthesignofthedifference.

Example:Subtract–3from+5.

Solution:Distanceonnumberlinebetween–3and+5is8units.Directionisfromnegativetopositive—apositivedirection.Answeris+8.

Example:Subtract–6from–8.

Solution:Distanceonnumberlinebetween–6and–8is2units.Directionisfrom–6to–8—anegativedirection.Answeris–2.

Example:Subtract+1.30from–2.70.

Solution:Distancebetweenthemonthenumberlineis4.0.Directionisfrom+1.30to–2.70—anegativedirection.Answeris–4.0.

TestYourself32

Directions:Trythesesubtractionproblems.Thinkbeforeyouanswer!

1.–2from–3

2. from

3.6.8from2.2

4.–3.6from5.5

5.–7.65from0.002

6.–1from1

7.0.0019from–0.0010

8.102from96.5

9. from

10. from

Aquickwaytosubtractsignednumbersaccuratelyinvolvesplacingthenumbersincolumns,reversingthesignofthenumberbeingsubtracted,andthenaddingthetwo.

Example:Subtract+26from+15.

Solution: =

Example:Subtract–35from+10.

Solution: =

Noticethatineachoftheexamples,thecorrectanswerwasfoundbyreversingthesignofthenumberbeingsubtractedandthenadding.

MultiplicationofSignedNumbers

Signednumbersaremultipliedasanyothernumberswouldbe,withthefollowingexceptions:•Theproductoftwonegativenumbersispositive.•Theproductoftwopositivenumbersispositive.•Theproductofanegativenumberandapositivenumberisnegative.

Example:–3×–6=+18

Example:–3.05×+6=–18.30

Example:

Example:+1×–1×+1=–1

TestYourself33

Directions:Practicewiththefollowingexamples.Remember,theonlywaytogetanegativeproductiswithapairofoppositelysignedfactors.

1.–5×–6

2.–2.5×–1.3

3.

4.

5.–1.2×–0.75×–0.1

6.

7.–10.6×3.3×–1.01

8.

9.0.001×–3.25×10

10.

DivisionofSignedNumbers

Aswithmultiplication,thedivisionofsignednumbersrequiresyoutoobservethreesimplerules:

•Whendividingapositivenumberbyanegativenumber,theresultisnegative.

•Whendividinganegativenumberbyapositivenumber,theresultisnegative.

•Whendividinganegativenumberbyanegativenumber,orapositivenumberbyapositivenumber,theresultispositive.

Example:+6÷–3=–2

Example:–6÷+3=–2

Example:–6÷–3=+2

Example:+6÷+3=+2

TestYourself34

Directions:Trythefollowingproblems.

1.120÷–8

2.

3.0.43÷–0.2

4.–0.063÷9

5.122÷–10

6.

7.

8.–2.0002÷–0.01

9.

10.–100÷–0.25

VariablesandCoefficients

Algebrausesletterstostandfornumbers.Lettersofthiskindhavingseveralpossiblevaluesarecalledvariables.Themostcommonlyusedvariablesarethelettersxandy,althoughallother

lettersofthealphabetarealsoused.

Thevariablexlooksverysimilartothemultiplicationsignusedinarithmetic.Forthisreason,itisagoodideatouseadotorparenthesestoindicatemultiplication,ratherthanthe×symbol.

Forexample,ifyouwanttowrite“sixtimesfive,”writeitlikethis6•5or6(5),ratherthan6×5.

Numbersusedinfrontofvariablestoindicatehowmanyofeachvariableyouareworkingwitharecalledcoefficients.Coefficientsmaybewholenumbers,decimals,fractions,orevenGreekletters.

Example1indicatesanumberthreetimesthesizeofx;Example2,anumberonehalfthesizeofy;andExample3,anumberonefourththesizeofa.

Whereyouseenocoefficientwritten,thecoefficientisassumedtobe1.

Coefficientsincludenegativenumbersaswell.Forexample,theexamplesbelowhavecoefficientsof1or–1.

AddingandSubtractingAlgebraicExpressions

Additionofalgebraicexpressionsortermssuchastheonesdescribedaboveisquiteeasy.Youcanaddexpressionsonlyifthevariablesareidentical,andyoudothatbyaddingthecoefficientstogether.

Example:Add2xand3x.

Solution:5x

Example:Add and2y.

Solution:

Example:Add0.50aand0.75a.

Solution:1.25a

Example:Addbandb.

Solution:2b

TestYourself35

Directions:Intheexercisesbelow,addtheexpressionstogethertofindananswer.Somecannotbecombinedbecausethevariablesarenotidentical.Othershavenegativecoefficients,sobecareful.

1.a,3a,5a

2.10x,4x,20x

3.3m,4r,3s

4.–2b,–3b,6b

5.–1.5r,2r,3s,–2s

6.

7.0.001S,0.210S,–1.25S

8.

9.–2x,–3y,4x,–4z

10.1.02p,–0.62p,–40r

Subtractingalgebraicexpressionsisaccomplishedbysimplysubtractingthecoefficients.Youmustbecarefulifthecoefficientsarenegativenumbers,however.Usewhatyoulearnedabout

signednumbersearlierinthisbook.

Example:Subtractxfrom4x.

Solution:4x–x=3x

Example:Subtract–2xfrom4x.

Solution:4x–(–2x)=6x(Rememberthenumberline!)

TestYourself36

Directions:Trythefollowingsubtractionproblems.Somehavefractionalcoefficients,somehavedecimalcoefficients,andotherscannotbesubtractedbecausethevariablesarenotidentical.

1.3dfrom4d

2.

3.6yfrom–2b

4.–afrom–a

5.1.25mfrom–2.25m

6.–0.001xfrom–0.002x

7.2.483ffrom–5.0f

8.

9.3pfrom–2r

10.–16xfrom–14x

MultiplyingAlgebraicExpressions

First,reviewthesectioninthisbookaboutexponents.Payparticularattentiontooperationsinvolvingmultiplicationanddivisionofnumbersexpressedwithexponents.

Whenmultiplyingalgebraicexpressions,multiplythecoefficientsasyouwouldanynumbersandthenaddtheexponentsoftheidenticalvariablestofindtheproduct.

Example:Multiply2x•3x.

Solution:Multiplythecoefficientsandaddtheexponentsoftheidenticalvariables:2x•3x=6x2.

Rememberthatwhereanexponentisnotwritten,itisequivalentto1.

Example:

Example:–5y•2y=–10y2

Example:–2a2•2a=–4a3

Example:0.25m3•–0.25m3=–0.0625m6

TestYourself37

Directions:Trythefollowingproblems.

1.3x•2x

2.

3.–1.5a•3.2a

4.–2z2•3z

5.1.021r•1.010r2

6.–3.65f•1.60f3

7.

8.

9.2.02x2•–1.1x2

10.–7.1b•10.1b3

Anytwoexpressionscanbemultipliedtogetherandrewrittenasoneexpression.Thesame

multiplicationrulesapply.

Example:Multiply2x•3y.

Solution:Multiplythecoefficientsandaddtheexponentsofidenticalvariables.Becausethevariablesarenotidenticalinthisproblem,wesimplymultiplythemtogether.Theproductisthus6xy.

Example:–2a•3b=–6ab

Example:0.35m•2f=0.70fm

Example:–2x2•–4y=8x2y

Example:4a2b•–3ab2=–12a3b3(Theexponentsoftheidenticalvariableswereaddedtogether.)

TestYourself38

Directions:Trythefollowingproblems.

1.7x•2y

2.

3.1.2d•1.3f

4.–45x2•0.50y

5.–6.9a2•3.2b

6.100abc•–a

7.

8.–x2•–y2•–z2

9.–2x2•4y2•–3z

10.–3a•5b•2c

DividingAlgebraicExpressions

Divisionisaprocessthatreversesmultiplication.Whendividingalgebraicexpressions,dividethecoefficientsandsubtracttheexponentsoftheidenticalvariables.Youmustalsoobeytherulesgoverningdivisionofsignednumbersifthecoefficientsaresigned.

Reviewthesectiononexponentsifyoucannotfollowtheseexamples:

Example:Divide6xby2.

Solution:6x÷2=3x

Example:Divide4a2by2a.

Solution:4a2÷2a=2a

Example:Divide–3b2by0.5b.

Solution:–3b2÷0.5b=–6b

Example:Divide2x2y2by0.4xy.

Solution:2x2y2÷0.4xy=5xy

Example:Divide–6x3y4by–x2y.

Solution:–6x3y4÷–x2y=6xy3

TestYourself39

Directions:Trythefollowingproblemsforpractice.

1.100c2÷10c

2.–3.5x÷0.7x

3.2.20y2÷–1.1y

4.–4a2b÷2a

5.22ab2÷–11b

6.–5.1abc÷1.7bc

7.4 ax2÷–1 a

8.–0.001y3÷0.01y2

9.–2.8r2s÷–0.7rs

10.70xy2÷–35xy2

RemovingGroupingSymbols

Mathematicsusesparenthesesandbracketstogroupnumbersforvariousreasons.Whencalculationshavetobemade,itisnecessarytoremovethegroupingsymbolsandcombineasmanyofthenumbersasyoucan.

Example1:6(3+5)=6•8=48

Example2:–6(1+2)=–6•3=–18

Example3:–2(a+b)=–2a+–2b=–2a–2b

InExamples1and2,theoperationinsidetheparentheseswascarriedoutfirst,withtheresultthenmultipliedwiththenumberoutsidetheparentheses.InExample3,thelettersinsidetheparenthesescouldnotbeadded,so–2wasmultipliedbybothaandb.Insomecases,additionalstepsarerequired.

Example:Simplify:2–[4–(3–1)+6].

Solution:Beginbyworkingwiththeinnermostparentheses,removingonesetofgroupingsymbolsateachstep.

2–[4–(3–1)+6]

=2–[4–2+6]

=2–[8]

=2–8

=–6

Thissameprocedurecanbeusedwithvariables.

Example:Simplify:a–[b–(a–2b)+3a].

Solution:Beginwiththeinnermostgroupandworkoutward.

=a–[b–(a–2b)+3a]

=a–(b–a+2b+3a)

=a–(3b+2a)

=a–3b–2a

=–a–3b

Intheexampleabove,itisimportanttonotethatthenegativesigninfrontofthegroupingsymbolreversesthesignofthenumberswithin.Apositivesignchangesnothing.

TestYourself40

Directions:Trythefollowingproblems.Remembertoremovethegroupingsymbolsinorder,beginningwiththeinnermost.

1.2+(3–2)–2

2.–2–[1+(6–2)]

3.[a+(1–2)+b–3]

4.–1+2–(3–4)

5.–5–[(6–3)–(1–2)]

6.a–[3–(b–2)–(a–1)]

EvaluatingAlgebraicExpressions

Itisoftennecessarytodeterminethevalueofanalgebraicexpressionifitsvariablesaregivenprecisenumericalvalues.Youhavedonethiswhenfindingtheareas,perimeters,andvolumesofgeometricfigures.Whengivenaformulafortheareaofatriangle,forexample,youcanfindtheareaifyouknowthebaseandtheheight.Thesameprocedureisusedinevaluatingalgebraicexpressions.

Example:Findthevalueof10xifx=2.

Solution:Substitute2forx,andmultiply10•2=20.

Example:Evaluate3a2ifa=5.

Solution:Substitute5fora,andmultiply3•52=3•25=75.

Example:Evaluate–2x2y2ifx=2andy=3.

Solution:–2•22•32=–2•4•9=–72

Example:Evaluate ifx= .

Solution: =16

TestYourself41

Directions:Trythefollowingproblems.

1.3(x+y)ifx=2,y=–1

2.a2–b2ifa=3,b=4

3.4m2n2ifm= ,n=1

4. ax2ifa=32,x=4

5.(b+c)(d+2)ifb= ,c=2,d=0

6.

7.

8. (a+b)hifa=4,b=3,h=1.5

9. ifm1=32,m2=320,r=10

10. ifb=4.8,c=1.2,y=1

SolvingSimpleEquations

Muchoftheworkinalgebraconsistsoffindingprecisevaluesforvariables.Tofindtheseprecisevalues,variablesaresetequaltoknownquantitiesinequations.Forexample,thesimplestequationpossibleisx=2.Thisequationsays,“Thevariablexisequaltotwo.”

Onestepfurtheris:2y=2.Thissays,“Twotimesyequalstwo.”

Intheequation2y=2,let’sfindwhatyequals.If2timesyequals2,weknowyequals1.

Example:a+1=3

Solution:Becausethevariableaplus1equals3,3is1largerthana.a=2.

Example:Z–3=6

Solution:ThevariableZminus3equals6.Zis3largerthan6,soZ=9.

TestYourself42

Directions:Trythefollowingexamples.Solvetheequationforthevariable.

1.a+3=6

2.2x–1=7

3.–2x=6

4.r–3=–1

5.3b–6=6

6.100y–1=99

7.10+x=5

SolvingMoreDifficultEquations

Equationsmayhavevariableswithfractionalanddecimalcoefficients.Variablesmayalsoappearonbothsidesoftheequalsign.Thefollowingexamplesshouldhelptoreviewhowtosolvethesekindsofequations:

Example1:2x–3=x+2

Example2: =15

Example3:0.25a=a–1.5

Example4:6(x–2)=12

Whenworkingwithequationssuchasthoseabove,itisnecessarytoworkontheleftsideoftheequalsignaswellastherightside.

Tomakeequationseasiertoworkwith,elementsoftheequationmustbemovedacrosstheequalsignfromonesidetotheother.Thegoalistoplaceallofthetermshavingavariableononesideoftheequationandallofthetermsnothavingavariableontheother.Indoingthis,obeyonesimplerule:Whenmovingatermtotheothersideoftheequation,reverseitssign.

Example1wouldbesolvedinthreestepslikethis:

Solution1:Solveforx:2x–3=x+2.Putalltermscontainingxontheleftsideoftheequalsign.Theothersgoontheright.Reversethesignsofthosetermsmovedfromonesidetotheother.2x–x=2+3Simplify:x=5.Theequationissolved.

Insomecases,multiplicationanddivisionmaybeinvolved,asinExample2.Example2wouldbesolvedinthreestepslikethis:

Solution2:Solvefory: =15.

Thetermhavingthevariableisalreadyontheleft,andtheotherisalreadyontheright.

Dividebothsidesoftheequationbythecoefficientofthevariable: .y=20

Youmayhavebeenabletosolvethisproblemwithoutcalculations.Bythinkingaboutwhatnumber15wasthreequartersof,youmayhavefiguredoutthatywasequalto20.

Example3wouldbesolvedinfourstepslikethis:

Solution3:Solvefora:0.25a=a–1.5.Putthetermswiththevariableontheleft;thosewithout,ontheright.Reversethesignsofthosethatcrosstheequalsign.0.25a–a=–l.5Combine:–0.75a=–1.5.Dividebothsidesbythecoefficientofthevariable:

Example4wouldbesolvedinfourstepslikethis:

Solution4:Solveforx:6(x–2)=12.Removethegroupingsymbolbymultiplyingby6:6x–12=12.Move–12totherightandchangeitssign:6x=12+12.Combine:6x=24.Dividebothsidesbythecoefficientofthevariable:

TestYourself43

Directions:Trythefollowingproblems.Youmaybeabletosolvesomeoftheminyourheadwithoutcalculatingonpaper.

1.

2.

3.2x–2=4x–10

4.

5.3.2x=64

6.5(x–3)=x+9

7.

8.

9.3b=b

10.6m–1=m+4

Geometry

Geometryisthatpartofmathematicsthatstudieslines,curves,andanglesandthevariousshapestheycreatewhenplacedtogetherindifferentways.Usually,geometryisdividedintotwosubgroups:planegeometryandsolidgeometry.

Planegeometrystudiesanyshapesandanglesthatcanbedrawninoneplane.Thismeansthatshapesthatcanbemeasuredinonlyoneortwodimensions,ordirections,arestudied.Forexample:

•Alinehasonlyonedimension,itslength.•Atriangle,asquare,oracircledrawnonapieceofpapercanbemeasuredinonlytwodirections,ordimensions:lengthandwidth.

Solidgeometrystudiesshapesthathavethreedimensions:length,width,andthickness.Forexample:

•Anobjectsuchasabrickorashoeboxisarectangularsolidthatcanbemeasuredinthreedirectionsordimensions:length,width,andheight.

•Cubes,cones,spheres,cylinders,pyramids,ortetrahedronsareexamplesofshapesthatarethree-dimensionaland,assuch,requiretheuseoftheprinciplesofsolidgeometry.

Firstwewillreviewsomebasicsofplanegeometry.

Points,Lines,andAngles

Apointisanexactlocationandhasnodimensions.

Byplacinglotsofpointsinarow,webuildaline.Alinehasinfinitelengthinbothdirectionsandhasasymbollikethis:

Thearrowheadateachendindicatesthatthelineisinfinite.Usually,weselecttwopointsontheline,givethemnames,andnamethelinethesameway:

+ read:“lineAB”

Notethatweneedaminimumoftwopointstomakealine.Thereisnomaximumnumberofpointsonaline,however.

Arayisalinethathasoneendpoint,andgoesinfinitelyinonedirection.Werefertoaraybyitsendpointandoneotherpointalongit.

= read:“rayAB”

Alinesegmentisapieceofalinehavingtwoendpoints.Wenameitbynamingtheendpoints:

+ read:“linesegmentAB”

Becausetheirlengthisinfinite,linesandrayscannotbemeasured.Linesegments,ontheotherhand,arefiniteandcanbemeasured.

Angles

Wherelines,linesegments,orraysmeetorcrosseachother,anglesareformed.Thesimplestangleisthatformedbytworayshavingthesameendpointbutgoingindifferentdirections.Theendpointthatthesetworaysshareiscalledthevertexoftheangle.

Anglesaremeasuredinunitscalleddegrees.Adegreeis ofacompleterevolutionaround

thepointcalledthevertex.

Forexample,thedrawingsbelowshowonerayofananglegoingthroughonecompleterevolutionaroundthevertex.Ineachcase,themeasureoftheangleisshown.

a.

b.

c.

d.

e.

f.

g.

Thesequenceshouldremindyouofanumberofrules:•Eachcompleterevolutionaroundthevertexcreatesanangleof360°.•Anglesaremeasuredcounterclockwise.•Themeasureoftheangleisthesame,nomatterhowlongtheraysare.Thetipsofeachrayhavethesameanglebetweenthemasdotwopointsclosertothevertex.

KindsofAngles

A90°angleisalsocalledarightangle.Squares,rectangles,andsometriangleshaverightangles.

Anglessmallerthan90°arecalledacuteangles.Themeasureofanacuteangleisgreaterthanzero,butlessthan90°.

Ananglethatmeasuresgreaterthan90°butlessthan180°iscalledanobtuseangle.

Anangleequivalentto180°isastraightangle.Linesmaybethoughtofasstraightangles.

Angleswhosesumis180°aresupplementaryangles.Forexample,thesumof60°and120°is180°.Eachangleisasupplementoftheother.

Alineintersectingastraightanglecutsitintosupplementaryangles.

Angle1andangle2aresupplementaryangles.

Angleswhosesumis90°arecomplementaryangles.Forexample,thesumof60°and30°is90°.Eachangleisacomplementoftheother.

TestYourself44

Directions:Forthefollowingproblems,statewhethertheangleisacuteorobtuse.Ifitisobtuse,nameitssupplement.Ifitisacute,nameitssupplementandcomplement.

1.170°

2.30°

3.142°

4.60°

5.90°

6.27°

7.135°

8.95°

9.57°

10.45°

Perimeter

Theperimeterofanobjectisthedistancearoundit.Forexample,ifyouwalkedallthewayaroundthe“outside”ofafootballfield,atrack,orabuilding,youwouldhavewalkedalongitsperimeter.

Perimetersareusuallyeasytocompute.Wesimplyaddupthelengthsofthesides.

Forexample,whatistheperimeteroftheobjectbelow?

Justaddthelengthofeachside:

Frequently,objectsaremuchmoreirregularthantheoneshownonthepreviouspage.Foranobjectsuchastheonebelow,forexample,youmustfindthelengthofeachsmallsegmentoftheperimeter.Thenaddthelengthstogethertofindtheperimeter.

Theperimeterofacircleiscalleditscircumference,whichiscomputedinaspecialway.

AncientGreekmathematiciansdiscoveredanimportantfactaboutcircles.Nomatterhowlargethecirclewas,theyfoundthatitscircumference(perimeter)wasalmostexactly3.14timesitsdiameter.Theynamedthenumberbywhichtheymultipliedacircle’sdiametertogetits

circumferencepi.Weusethevalue3.14,orthefraction ,torepresentpi,althoughitsvalueis

slightlylarger.Thesymbolπstandsforpi.

Tofindtheperimeter(circumference)ofthiscircle:

Wecanalsoworkbackwardtofindthediameterofacirclefromitscircumference.Forexample,ifthecircumferenceofacircleis21.98inches,whatisitsdiameter?

Becausethecircumferenceistheproductofπandthediameter,dividethecircumferencebyπtofindthediameter.

21.98"÷3.14=7"(diameter)

TestYourself45

Directions:Foreachcircle,usethegiveninformation,diameter(d)orcircumference(c),tofindtheotherquantity.

1.d=3.5"

2.d=5.0"

3.c=22"

4.d=

5.c=31.4"

6.c=15.7"

AreaofPlaneFigures

Whenaplanefiguresuchasarectangle,triangle,orcircleliesflat,itcoversacertainamountofarea.Whenitisnecessarytobuycarpeting,grassseed,paint,andmanyotherthings,theareaoftheplacetobecoveredmustbecalculated.

Areaisalwaysmeasuredinsquareunits,suchassquareinches,squarefeet,squareyards,orsquaremiles.Metricsystemunitsforareaincludesquarecentimeters,squaremeters,andsquarekilometers.Generally,theunitofmeasurementtobeusedisbasedupontheareaoftheobjectbeingmeasured.

Forexample:•Theareaofacitywouldbemeasuredinsquaremilesorsquarekilometers.•Theareaofafootballfieldwouldbemeasuredinsquareyardsorsquaremeters.•Theareaofthispagewouldbemeasuredinsquareinchesorsquarecentimeters.

Whencalculatingarea,itismostimportanttorememberthatyouaredealingwithsquareunits.Areaisalwaysgiveninsquareinches,squaremeters,andsoon.

AreaofSquaresandRectangles

Theareaofsquaresandrectanglesisfoundbymultiplyingthelengthofanyonesidebythelengthofthesideadjoiningit.

Forexample:

Theareaofthisrectangleistheproductof8cm×3cm=24sq.cm

Theareaofanyrectanglecanbefoundbymultiplyingthelengthofitslongestsidebythelengthofitsshortestside.

Areaofarectangle=lengthtimeswidth

A=l×w

Theareaofsquaresiscalculatedthesameway.Youjusthavetorememberthatthesidesofasquareareallthesamelength.Ifyouknowthelengthofoneside,youknowthelengthsofallthesides.

Forexample:

Theareaofthesquareis6in.×6in.=36sq.in.

TestYourself46

Directions:Findtheareaoftherectanglesorsquareshavingthefollowingdimensions.

1.3"long,2"wide

2.3feet6incheslong,2feetwide

3.10cmlong,10cmwide

4. mileslong, mileswide

5.12"long,12"wide*

6.3'long,3'wide**

7. "wide,10"long

8. "wide, "long

*144sq.in.equals1sq.ft.

**9sq.ft.equals1sq.yd.

AreaofTriangles

Trianglesmaybe:•Acute,ifeachoftheiranglesislessthan90°:

Ifeachangleis60°,thetriangleiscalledequilateral,becauseallanglesareequalinmeasure.•Right,ifoneangleis90°:

•Obtuse,ifoneangleislargerthan90°:

Familiarizeyourselfwiththesekindsoftriangles,becausefindingtheirarearequiresyoutobeverycarefulaboutonething:measuringthealtitudecorrectly.

Wefindtheareaoftrianglesbyusingthefollowingformula:

Area=onehalfoftheproductofthebaseandthealtitude

Thealtitudeofatriangleisthedistancefromavertextothelinecontainingtheoppositeside(base).Thealtitudeisalwaysperpendiculartothebase.

Thesedrawingsshowhowtomeasurealtitudecorrectly.

Inarighttriangle:

Inanobtusetriangle:

Inanacutetriangle:

Noticethatineachcase,thealtitudeorheightofatrianglemustbemeasuredalongalinethatmakesarightanglewiththebase.

Example:Findtheareaofthetriangleshownbelow.

Solution:

TestYourself47

Directions:Findtheareasofthetriangleswhosedimensionsaregivenbelow.

1.b=16",a=8"

2.b=4 feet,a=2feet

3.b=11.5cm,a=4.5cm

4.b=2.4feet,a=6.3feet

5.b=8inches,a=2.5feet

6.b=1 yds.,a=3 yds.

AreaofParallelogramsandTrapezoids

Aparallelogramisafour-sidedfigurewithoppositesidesparalleltoeachother.Theareaofaparallelogramcanbefoundbymultiplyingthelengthofthebasebythealtitude.

Area=base×altitude

A=b×aorA=ba

Hereagain,youmustbecarefultomeasurethealtitudeperpendiculartothebase,justasyoudidwhenfindingtheareasoftriangles.

Atrapezoidisafour-sidedfigurewithonepairofsidesparallel,andonepairnonparallel.Theparallelsidesarecalledthebases,andwefindtheareaasfollows:

Area= ×altitude×(lengthofbase1+lengthofbase2)

A= ×a×(b1+b2)orA= a(b1+b2)

Besuretomeasurethealtitudeofthetrapezoidalongalinethatmakesarightanglewiththebase.

TestYourself48

Directions:Findtheareasofthefollowingparallelogramsandtrapezoids.“P”meansparallelogram;“T”meanstrapezoid.

1.P:b=6",a=4"

2.P:b=5",a=2"

3.T:b1=10",b2=15",a=3"

4.T:b1=20cm,b2=40cm,a=6cm

5.P:b=4.125ft.,a=3.34ft.

6.T:b1=5",b2=7",a=3"

AreaofaCircle

Theareaofacircleiseasilycalculated,ifyouremembertwothings:

Usethenumberpi(π=3.14,or ).

Usetheradiusofthecircleinthecalculationinsteadofthediameter.Theradiusisone

halfofthediameter:r= .

Tofindtheareaofacircle,usethefollowingformula:

Area=π×lengthofradius×lengthofradius

A=πr2

Forexample,thiscirclehasaradiusof3".

Thus,itsareais:

A=πr2

=3.14"×3"×3"

=28.26sq.in.

Sometimes,toavoidtheextracalculation,areasofcirclesarewritteninpi.Intheexampleabove,wewouldwrite:

A=π×3"×3"

=9πsq.in.

TestYourself49

Directions:Practicefindingtheareasofthecirclesbelow.Remembertousethelengthoftheradiusinthecalculation.Writeyouranswersintermsofpi,aswellasmultipliedout.

1.r=2.17inches

2.r=3.5cm

3.d=l0feet

4.d=20yds.

5.r=22inches

6.d=11inches

7.r= cm

8.d=100cm

9.c=31.4inches

10.c=125.6miles

VolumeofSolidFigures

Three-dimensionalfiguressuchascubes,cones,spheres,andrectangularsolidstakeupspace.Theamountofspaceanobjectorsubstancetakesupiscalleditsvolume.

Becausewepurchaseitemsandplanthesizesofbuildings,homes,ships,andsoforthaccordingtoourneedsforacertainvolumeofsomething,volumeisoneofthemostimportantmeasurementsthatwemake.Inthissection,youwillreviewhowtocalculatethevolumeofcertaineasy-to-measureshapes.

VolumeofRectangularSolids

Afamiliarrectangularsolidisashoebox.Itsvolumeiscalculatedbymultiplyingitslengthtimesitswidthtimesitsthicknessordepth.Theformulais:

Volume=l×w×h,orV=lwh

Forexample,theboxshownabovehasthevolume:

V=3"×4"×8"

=96cu.in.

Noticethatwemultipliedthemeasurementofeachdimensionoftheboxonlyonce.Theunitsthatweuseinmeasuringvolumearecubicunits.

Aspecialrectangularsolidisacube.Acubehasallofitsdimensionsthesamelength,soyouneedonlythelengthofoneedgetofinditsvolume.

Forexample,thedrawingbelowshowsacubewithanedge4"long.

Itsvolumeiscalculated:

V=4"×4"×4"

=64cu.in.

Whenyouarecalculatingthevolumeofrectangularsolids,shoeboxes,rooms,andsoforth,becertainthatyouhavemeasurementsforeachdimensionbeforeyoucalculate.

VolumeofaCylinder

Cylindersareobjectswedealwithallthetime.Softdrinkcansandmanyothercontainers,aswellaspipesandsmokestacks,arecylinders.

Inmanyways,acylinderresemblesastackofcoinsorastackofround,thinobjects,suchascocktailcoastersorslicesofbologna.Bythinkingofacylinderasastackofslicesorcoins,itiseasytorememberhowtocalculateitsvolume.

Thevolumeofacylindercanbecalculatedbyfirstfindingtheareaofthecircularbase,andmultiplyingthatvaluebytheheight.

Byfindingtheareaofthebase,youarefindingtheareaofone“slice”ofthecylinder.Whenyoumultiplythatbytheheightorlength,youarecalculatingvolumebycountingallofthe“slices”thatyoucouldmake.

Forexample,findthevolumeofthecylinderbelow:

Aswitharea,youmayfindthattheanswercanbeleftintermsofpi.Thatis,itmaynotbenecessarytomultiplyby3.14.Inthepreviousexample,theanswerV=250πcubicinchesisacceptable.

TestYourself50

Directions:Findthevolumesofthecylinders.Volume=πr2h.Stateyouranswersintermsofpi,aswellasmultipliedout.

1.r=1",h=1"

2.r=2",h=2"

3.r=3.5",h=6.20"

4.r=1ft.,h=2ft.

5.r=1.1cm,h=3.2cm

6.r=10",h=1"

7.r=0.25",h=1"

8.r=20cm,h=70cm

9.r=0.5",h=1.0"

10.r=x",h=y"

WordProblems

Twoverycommonkindsofwordproblemsthatyouwillencounteronhighschoolentranceexaminationsarerate,time,anddistanceproblemsandworkproblems.

Rate,Time,andDistanceProblems

Thebasicformulausedinsolvingproblemsfordistanceis:

d=rt(distance=rate×time)

Usethisformwhenyouknowrate(speed)andtime.

Tofindrate,use:

r= (rate=distance÷time)

Tofindtime,use:

t= (time=distance÷rate)

Studythefollowingproblems:

Example:Twohikersstartwalkingfromthecitylineatdifferenttimesbutinthesamedirection.Thesecondhiker,whosespeedis4milesperhour,starts2hoursafterthefirsthiker,whosespeedis3milesperhour.Determinetheamountoftimeanddistancethatwillbeconsumedbeforethesecondhikercatchesupwiththefirst.

Solution:Becausethefirsthikerhasa2-hourheadstartandiswalkingattherateof3miles

perhour,thathikeris6milesfromthecitylinewhenthesecondhikerstarts.

Rate×Time=Distance

Subtracting3milesperhourfrom4milesperhourgivesus1mileperhour,orthedifferenceintheratesofspeedofthetwohikers.Inotherwords,thesecondhikergains1mileonthefirsthikerineveryhour.

Sincethereisa6-miledifferencetocoveranditisdecreased1mileeveryhour,itisclearthatthesecondhikerwillneed6hourstoovertakethefirst.

Inthistime,thesecondhikerwillhavetraveled4×6=24,or24miles.Thefirsthikerwillhavebeenwalking8hours,becauseofthe2-hourheadstart,8×3=24,or24miles.

Example:Thesametwohikersstartwalkingtowardeachotheralongaroadconnectingtwocitiesthatare60milesapart.Theirspeedsarethesameasintheprecedingproblems,3and4milesperhour,respectively.Howmuchtimewillelapsebeforetheymeet?

Solution:Ineachhouroftraveltowardeachother,thehikerswillcoveradistanceequaltothesumoftheirspeeds,3+4=7milesperhour.Tomeettheymustcover60miles,andat7milesperhourthiswouldbe:

Theproblemmightalsohaveasked:“Howmuchdistancemusttheslowerhikercoverbeforethetwohikersmeet?”Insuchacaseweshouldhavegonethroughthesamestepsplusoneadditionalstep:

Thetimeconsumedbeforemeetingwas8 hours.Tofindthedistancecoveredbytheslowerhiker,wemerelymultiplyhisratebythetimeelapsed:

TestYourself51

Directions:Solvetheproblemsbelow,usingtheformulafortherate,time,anddistanceproblems.

l.Mattwalked3milesin ofanhour.Atwhatrateofspeeddidhewalkinmilesperhour?

2.AcommutertraintravelsthedistancefromActontoBostonin1hourand10minutes.Ifthetrainistravelingatanaveragespeedof48milesperhour,whatisthedistancebetweenthetwocities?

3.Paddling7milesupstream,Sarahandherbrotheraveraged2milesperhour.Ontheway

back,theirrateofspeedwas3 milesperhour.Howlongdidittakethemtomaketheroundtrip?

4.Twocarsbegindrivingtowardeachotherfromtownsthatare150milesapart.Thefirstcaristraveling45milesperhour,andthesecondistraveling55milesperhour.Ifeachleavesat2:15p.m.,whenwillthetwocarspassontheroad?

WorkProblems

Workproblemsgenerallyinvolvetwoormoreworkersdoingajobatdifferentrates.Theaimofworkproblemsistopredicthowlongitwilltaketocompleteajobifthenumberofworkersisincreasedordecreased.Workproblemsmayalsoinvolvedetermininghowfastpipescanfilloremptytanks.Studytheexamplesinthissectioncarefully.

Example:IfAdoesajobinsixdays,andBdoesthesamejobinthreedays,howlongwillittakethetwoofthem,workingtogether,todothejob?

Solution:Problemsofthissortcanbedoneusingfractions.

Step1:Writetheamountofthejobeachworkerdoeseachdayasafraction.

Adoes ofthejobinoneday.

Bdoes ofthejobinoneday.

Step2:Writetheamountofthejobcompletedbybothworkersinoneday.

A= ofjob

B= ofjob

Onehalfofthejobiscompletedinonedaybybothworkersworking

together.

Step3:ComparetheresultfromStep2todeterminehowmuchworkhastobedone.Becauseonehalfofthejobisfinishedinoneday,itwilltaketwodaysforbothworkerstofinishthejobworkingtogether.

Ingeneral,ifyouaregiventheamountoftimethatajobtakes,youmustfindthereciprocalofthattimetofindouthowmuchofthejobiscompletedinoneday,1hour,andsoon.

Forexample,ifajobtakes ,or days,youcoulddo ofthejobinoneday.

Ifajobtakes4 hours,youcoulddo ofthejobin1hour.

Ifyouaregiventhefractionofthejobcompletedinoneday,findthereciprocalofthefractiontodeterminehowlongthewholejobwilltake.

Forexample,if ofalawncanbemowedinanhour,thewholelawnwilltake ,or ,

hourstomow.

If ofajobcanbedoneinoneday,thewholejobcanbedonein ,or ,days.

Example:AandB,workingtogether,doajobin days.B,workingalone,isabletodothe

jobintendays.HowlongwouldittakeAtodothejobworkingalone?

Solution:Step1:Thewholejobtakes ,or ,days.B,workingalone,cando ofthe

jobinoneday.

Step2:TofindtheworkdonebyAinoneday,subtractB’sworkfromtheamountofworkdonebythetwoworkerstogetherinoneday:

representstheportionofthetotaljobdonebyAinoneday.

Step3:Takingthereciprocal,findhowlongitwouldtakeAtodotheentirejob.

Example:Acandoajobin6daysthatBcandoin daysandCcandoin days.How

longwillthejobtakeifA,B,andCwereworkingtogether?

Solution:Adoesthejobin6days: ofthejobinoneday.

Bdoesthejobin days: ofthejobinoneday.

Cdoesthejobin days: ofthejobinoneday.

AddtheworkdonebyA,B,andCinonedaytofindtheworkdonebyallthreeinoneday:

Findthereciprocalof inordertofindhowlongthetotaljobwouldtake:

days

Example:Onepipecanfillapoolin20minutes,asecondcanfillthepoolin30minutes,andathirdcanfillitin10minutes.Howlongwouldittakethethreetogethertofillthepool?

Solution:Firstpipefillsthepoolin20minutes: ofpoolin1minute.

Secondpipefillsthepoolin30minutes: ofpoolin1minute.

Thirdpipefillsthepoolin10minutes: ofpoolin1minute.

Addthethreefractionstogethertodeterminewhatpartofthepoolwillbefilledinoneminutewhenthethreepipesareworkingtogether.

If ofthepoolisfilledinoneminute,thereciprocalofthefractionwilltellus

howmanyminuteswillberequiredtofillthewholepool.

min.

TestYourself52

Directions:Solvetheproblemsbelowusingthesolutionstepsshownforworkproblems.

1.MichelleandBarbcancompleteajobin2hourswhenworkingtogether.IfMichellerequires6hourstodothejobalone,howmanyhoursdoesBarbneedtodothejobalone?

2.IfJohncando ofajobin ofaday,howmanydayswillittakehimtodotheentire

job?

3.Ittakes hourstofillanewundergroundgasolinestoragetank.Whatpartofthetank

wouldbefullifthegasolinehadbeenshutoffafter1hour?

4.Marycancleanthehousein6hours.HeryoungerbrotherJimcandothesamejobin9hours.Inhowmanyhourscantheydothejobiftheyworktogether?

ANSWERKEYSTestYourself1

1.1228

2.289

3.21,136

4.34,361

5.513

6.1520

7.902

8.138,021

9.3289

10.289

TestYourself2

1.Seventy-sixten-thousandths

2.Elevenandthreetenths

3.Onethousandfourhundredtwoandsixhundredthirty-ninethousandths

4.Seventhousandtwohundredtwenty-twodollarsandninety-threecents

5.Fiftyhundredths(fivetenths)

6.Fivehundredths

7.Sixteenandtwothousandonehundredsixty-threeten-thousandths

8.Twenty-ninehundred-thousandths

9.Threeandsixten-thousandths

10.Sixty-twoandthreehundredninety-onethousandths

TestYourself3

1.0.5

2.5.12

3.0.07

4.equal

5.10.7

6.7.62

7.3.009

8.equal

TestYourself4

1.67.323

2.5.1392

3.110.0262

4.103.7217

5.0.0923

6.9666.988

7.14.1854

8.95.653

9.790.3

10.10.0

TestYourself5

1.Simplestform

2.

3.

4.Simplestform

5.

6.

7.

8.Simplestform

9.

10.

TestYourself6

1.

2.

3.

4.

5.

6.

7.

8.

9.

10.

TestYourself7

1.

2.

3.

4.

5.

6.

7.

8.

9.

10.

TestYourself8

1.

2.

3.

4.

5.

6.

7.

8.

9.

10.

TestYourself9

1.605

2.779

3.321

4.58,991

5.190

6.8301

7.8868

8.771

9.889

10.113,992

TestYourself10

1.15.1798

2.0.7676

3.0.159

4.0.156

5.4.14

6.24.848

7.784.13

8.37.5177

9.29.827

10.63.4215

TestYourself11

1.

2.

3.

4.

5.

6.

7.

8.

9.

10.

TestYourself12

1.21,054

2.462

3.422,408

4.69,632

5.693,976

6.369,600

7.2,540,450

8.25,606

9.126,256

10.94,792,384

TestYourself13

1.2696.8869

2.10,020.72

3.53,389.468

4.1.164

5.2.6248

6.163,930.668

7.0.0009384

8.117.432

9.44.55703

10.4638.26

TestYourself14

1.

2.

3.

4.

5.

6.

7.

8.

9.

10.

TestYourself15

1.180

2.1.4

3.15

4.21

5.2.3

6.25

7.2.5

8.30

TestYourself16

1.196

2.13

3.

4.2.4

5.10,000

6.625

7.400

8.

9.12

10.6.25

TestYourself17

1.82.277,or

2.42.50,or

3.0.379,or

4.41.439,or

5.10.855,or

6.40.49,or

7.0.817,or

8.1.977,or

9.0.844,or

10.40.208,or

TestYourself18

1.0.305

2.5.760

3.131.297

4.17,902.394

5.0.085

6.0.737

7.0.0098

8.5.308

9.141.626

10.0.0019

TestYourself19

1.

2.

3.

4.

5.

6.

7.

8.

9.

10.

TestYourself20

1.0.80

2.

3.0.28

4.0.333

5.

6.

7.0.30

8.

9.

10.0.26

TestYourself21

1.3.27

2.1.02

3.8.2

4.

5.3.31

6.25.294

7.1.25

8.0.711

9.

10.21.95

TestYourself22

1.Wednesday,10milesfarther,10%farther

2.Thursday,40milesfarther,40%farther

3.Friday,0milesfarther,0%farther

4.Saturday,1mileless,1%less

5.Sunday,10milesless,10%less

TestYourself23

1.50%larger

2.27%smaller

3.20%smaller

4.99%smaller

5.99.5%smaller

6.100%larger

7.350%larger

8.99.99%smaller

TestYourself24

1.0.10×8=0.8

2.0.25×60=15

3.0.11×10=1.1

4.0.01×350=3.50

5.0.99×80=79.2

6.0.33×33=10.89

7.0.75×112=84

8.0.50×200=100

TestYourself25

1.0.01

2.0.012

3.0.005

4.5%

5.0.1%,or ofonepercent

6.0.3%,or ofonepercent

7.0.015

8.1.5%

9.0.15

10.0.01%,or ofonepercent

TestYourself26

1.50%,150%

2.100%,200%

3.50%,150%

4.32%,132%

5.20%,120%

6.50%,150%

7.50%,150%

8.

9.0%,100%

10.900%,1000%

TestYourself27

1. ,or0.656

2. ,or0.375

3. ,or0.267

4. ,or1.215

5. ,or2.22

6. ,or1.22

TestYourself28

1.12.6

2.0.1875

3.3000

4.50%

5.400%

6.

7.0.0525

8.240

9.78

10.90

11.1.25

12.

13.150

14.156

TestYourself29

1.10•10•10•10•10•10•10

2.1•1•1

3.3•3

4.

5.(0.45)(0.45)(0.45)(0.45)(0.45)

6.11•11

7.0•0

8.x•x•x

9.b•b•b•b•b•b•b•b

10.(ab)(ab)(ab)(ab)(ab)

TestYourself30

1.13–2=1

2.610–8=62

3.155+3=158

4.M3+3=M6

5.1010–8=102

6.36+2=38

7.a2+3=a5

8.10010–9=1001=100

9.

10.85–4=81=8

11.a5–3=a2

12.

13.x2+2=x4

TestYourself31

1.–11

2.1.3995

3.0.999

4.31

5.

6.–10.75

7.

8.–120.25

9.–1.7498

10.0

TestYourself32

1.–1

2.2

3.–4.6

4.9.1

5.7.652

6.2

7.–0.0029

8.–5.5

9.10

10.0

TestYourself33

1.30

2.3.25

3.

4.

5.–0.09

6.

7.35.3298

8.

9.–0.0325

10.

TestYourself34

1.–15

2.–2

3.–2.15

4.–0.007

5.–12.2

6. ,or0.5

7.–7

8.200.02

9.–1

10.400

TestYourself35

1.9a

2.34x

3.3m+4r+3s

4.b

5.0.5r+s

6.

7.–1.039S

8.–15.75k,or–15 k

9.2x–3y–4z

10.0.4p–40r

TestYourself36

1.d

2.3 x

3.–2b–6y

4.0

5.–3.50m

6.–0.001x

7.–7.483f

8.

9.–2r–3p

10.2x

TestYourself37

1.6x2

2.

3.–4.8a2

4.–6z3

5.1.03121r3

6.–5.84f4

7.

8.

9.–2.222x4

10.–71.71b4

TestYourself38

1.14xy

2.

3.1.56df

4.–22.5x2y

5.–22.08a2b

6.–100a2bc

7.

8.–x2y2z2

9.24x2y2z

10.–30abc

TestYourself39

1.10c

2.–5

3.–2y

4.–2ab

5.–2ab

6.–3a

7.–3x2

8.–0.1y

9.4r

10.–2

TestYourself40

1.1

2.–7

3.a+b–4

4.2

5.–9

6.2a+b–6

TestYourself41

1.3

2.–7

3.1

4.256

5.

6.a2

7.

8.5.25

9.102.4

10.4

TestYourself42

1.a=3

2.x=4

3.x=–3

4.r=2

5.b=4

6.y=1

7.x=–5

TestYourself43

1.x=12

2.x=3

3.x=4

4.x=

5.x=20

6.x=6

7.

8.

9.b=0

10.m=1

TestYourself44

1.Obtuse;10º

2.Acute;150º,60º

3.Obtuse;38º

4.Acute;120º,30º

5.Right;90º,nocomplement

6.Acute;153º,63º

7.Obtuse;45º

8.Obtuse;85º

9.Acute;123º,33º

10.Acute;135º,45º

TestYourself45

1.10.99"

2.15.7"

3.7"

4.

5.10"

6.5"

TestYourself46

1.6sq.in.

2.7sq.ft.

3.100sq.cm

4. sq.mi.

5.144sq.in.,or1sq.ft.

6.9sq.ft.,or1sq.yd.

7.5sq.in.

8. sq.in.

TestYourself47

1.64sq.in.

2. sq.ft.

3.25.875sq.cm

4.7.56sq.ft.

5. sq.ft.,or120sq.in.

6.2 sq.yd.

TestYourself48

1.24sq.in.

2.10sq.in.

3.37 sq.in.

4.180sq.cm

5.13.7775sq.ft.

6.18sq.in.

TestYourself49

1.4.71πsq.in.,or14.79sq.in.

2.12.25πsq.cm,or38.47sq.cm

3.25πsq.ft.,or78.5sq.ft.

4.100πsq.yds.,or314sq.yds.

5.484πsq.in.,or1519.76sq.in.

6.30.25πsq.in.,or94.985sq.in.

7. πsq.cm,or0.785sq.cm

8.2500πsq.cm,or7850sq.cm

9.r=5;A=25πsq.in.,or78.5sq.in.

10.r=20;A=400πsq.mi.,1256sq.mi.

TestYourself50

1.πcu.in.;3.14cu.in.

2.8πcu.in.;25.12cu.in.

3.75.95πcu.in.;238.48cu.in.

4.2πcu.ft.;6.28cu.ft.

5.3.872πcu.cm;12.158cu.cm

6.100πcu.in.;314cu.in.

7.0.0625πcu.in.;0.19625cu.in.

8.28,000πcu.cm;87,920cu.cm

9.0.25πcu.in.;0.785cu.in.

10.πx2ycu.in;3.14x2ycu.in.

TestYourself51

1.4milesperhour

2.56miles

3.5 hours

4.3:45p.m.

TestYourself52

1.3hours

2.3days

3.

4.3 hours

EXERCISES:MATHEMATICS

Directions:Solvethefollowingquestionsandchoosethecorrectanswer.

1.Wherex= ,thereciprocalofxequals

(A)

(B)

(C)

(D)

2.Theproductof andanumberxis1.Thenumberis

(A)

(B)

(C)

(D)1

3. isequivalentto

(A)1

(B)

(C)

(D)1+x

4. equals

(A)

(B)

(C)

(D)

5.IntheformulaL= bxh,ifb=2,x=7,andh= ,Lequals

(A)

(B)

(C)

(D)

6.Twoanglesofatriangleare45ºand75º.Whatisthemeasureofthethirdangle?

(A)60º

(B)35º

(C)180º

(D)45º

7.

Isosceles ABCisinscribedincircleDthathasadiameterof10centimeters.Theareaofthetriangleis

(A)78.5sq.cm

(B)12.5sq.cm

(C)25sq.cm

(D)50sq.cm

8.Thevolumeofasmallwarehousemeasuring75feetlong,50feetwide,and30feethighis

(A)1,112,500cubicfeet.

(B)112,500squarefeet.

(C)112,500feet.

(D)112,500cubicfeet.

9.Adepartmentstoremarksupitsclothing80%overcost.Ifitsellsbluejeansfor$14,howmuchdidthestorepayforthem?

(A)$7.78

(B)$17.50

(C)$11.20

(D)$1.12

10.Thesamestoreputsthesame$14jeansonsaleata25%discount.Whatisthenewsellingprice?

(A)$13.75

(B)$10.50

(C)$3.50

(D)$13.65

11.Ify=0.25,thevalueofy2+ is

(A)1.125

(B)0.1125

(C)0.6750

(D)0.5625

12.Themonthlyfinancechargeonachargeaccountis %ontheunpaidamountupto$500,

and1%ontheunpaidamountover$500.Whatisthefinancechargeonanunpaidamountof$750?

(A)$22.50

(B)$1

(C)$10

(D)$100

13.Simplify:1–[5+(3–2)].

(A)–3

(B)–5

(C)6

(D)0

14.Simplify:–3–[–2+(5–6)–3].

(A)+3

(B)–1

(C)+1

(D)–3

15.

Thegraphaboveshows

(A)moreitemscostmoremoneytomake.

(B)bymakingmoreitems,theproductioncostperitemislower.

(C)thereisalimittothenumberofitemsthatcanbemade.

(D)noneoftheabove.

16.Baseduponthegraphabove,whatisthecostperitemif300itemsaremanufactured?

(A)$40

(B)$28

(C)$20

(D)>$20

17.Ifthecompanyproducedonly100items,approximatelyhowmuchwouldeachitemcost?

(A)$100

(B)$75

(C)$10

(D)$60

18.Howmanyitemswouldhavetobemanufacturedtobringthecosttobetween$5and$10peritem?

(A)Fewerthan100

(B)1000

(C)2000

(D)Itcannotbedeterminedwiththeinformationgiven.

19.0

(A)2.31

(B)2.52

(C)2.85

(D)

20. =

(A)

(B)

(C)

(D)Noneoftheabove

21.6÷0.0006=

(A)0.0036

(B)10,000

(C)60,000

(D)100,000

22.

Thesquareabovehasaside4" long.Theareaoftheshadedportionis

(A) sq.in.

(B)16sq.in.

(C) sq.in.

(D) sq.in.

23.Evaluate: .

(A)102

(B)1000

(C)1

(D)1012

24.Whatisthevalueofx5,ifx=3?

(A)81

(B)243

(C)15

(D)35

25.Theratioofteacherstostudentsinacertainschoolis1:14.Iftherearefourteenteachersintheschool,howmanystudentsarethere?

(A)14

(B)196

(C)206

(D)176

26.Of27peopleinacertaingroup,15aremenand12arewomen.Whatistheratioofmentowomeninsimplestform?

(A)15:12

(B)12:15

(C)5:4

(D)27:12

27.

TheperimeteroffigureAis

(A)19in.

(B)30in.

(C)23sq.in.

(D)19sq.in.

28.TheareaoffigureAis

(A)26sq.in.

(B)19sq.in.

(C)44sq.in.

(D)30sq.in.

29.Thesurfaceareaofabrickwiththedimensions6"×3"×2"is

(A)36sq.in.

(B)72sq.in.

(C)128sq.in.

(D)72in.

30.Theratioofsurfaceareatovolumeofacubehavinganedgeof2inchesis

(A)2:3

(B)1:3

(C)6:1

(D)3:1

31.

Thelengthof inthetriangleaboveis

(A)4.5

(B)3.5

(C)5

(D)4

32.

TwodriversbeginatpointCsimultaneously.OnedrivesfromCtoBtoA.TheotherdrivesdirectlytoAat50mph.HowfastmustthefirstpersondrivetogettoAfirst?

(A)Lessthan50mph

(B)Lessthan60mph

(C)Lessthan70mph

(D)Morethan70mph

33.Ifx=0.25, =

(A)

(B)4

(C)

(D)1

34.

ThemeasureofangleAis

(A)15º

(B)20º

(C)25º

(D)35º

35.AboyMyearsoldhasabrothersixyearsolderandasisterfouryearsyounger.Whatisthecombinedageofthethree?

(A)M+10

(B)3M+2

(C)3M–2

(D)2M–6

36.IfAnumberofpeopleeachmakeLthings,thetotalnumberofthingsmadeis

(A)A÷L

(B)A+L

(C)A–L

(D)AL

37.IfamanrunsMmilesinThours,hisspeedis

(A)M÷T

(B)M+T

(C)M–T

(D)MT

38.HowmanysquareinchesarethereinRrooms,eachhavingSsquarefeet?

(A)RS

(B)144RS

(C)9÷RS

(D)S+R

39.Thedrawingofawheelinabookisdoneat scale.Ifthedrawingis1.8inchesin

diameter,whatisthetruediameterofthewheel?

(A)32"

(B)28.8"

(C)24"

(D)0.1125"

40.Oneofthescalesusedindrawingtopographicmapsis1:24,000.Onascaleofthissort,1inchonthemapwouldequalhowmuchdistanceontheground?

(A)1inch

(B)2000feet

(C)24,000feet

(D)1mile

41.Onablueprint,2inchesrepresent24feet.Howlongmustalinebetorepresent72feet?

(A)36inches

(B)12inches

(C)6inches

(D)4inches

42. =

(A)25

(B)

(C)3000

(D)Noneoftheabove

43.X3•X2=

(A)X6

(B)X5

(C)2X5

(D)2X6

44.EventAoccursevery14minutesandeventBevery12minutes.Iftheybothoccurat1p.m.,whenwillbethenexttimethatbothoccurtogether?

(A)2:12p.m.

(B)1:48p.m.

(C)2:24p.m.

(D)3:48p.m.

45.EventAoccursevery4years,eventBevery11years,andeventCevery33years.Iftheylastoccurredtogetherin1950,whenwilltheynextoccursimultaneously?

(A)3402

(B)1983

(C)2082

(D)6804

46.Ifxisanoddwholenumber,whichofthefollowingalsorepresentsanoddnumber?

(A)2x+1

(B)x–2

(C)4x–3

(D)Alloftheabove

47.Thesumof4hours17minutes,3hours58minutes,45minutes,and7hours12minutesis

(A)15hr.32min.

(B)17hr.32min.

(C)16hr.12min.

(D)14hr.50min.

48.If8lb.12oz.offruitweretobedividedamong8people,howmuchwouldeachreceive?

(A)1lb.1.5oz.

(B)10.5oz.

(C)2.0lbs.

(D)13.5oz.

49.Howmuchfasterdoesarunnerwhofinishesamarathonin2hours12minutes38secondscompletetheracethanarunnerwhofinishesin3hours2minutes24seconds?

(A)48min.56sec.

(B)49min.46sec.

(C)1hr.51min.22sec.

(D)1hr.26min.12sec.

50.Inthenumber6,000,600,000,thereare

(A)6billionsand6hundredthousands.

(B)6millionsand6thousands.

(C)6billionsand6millions.

(D)6millionsand60thousands.

51.

Thegraphaboveshows

(A)xincreasingfasterthany.

(B)yincreasingfasterthanx.

(C)xincreasingasfastasy.

(D)norelationshipbetweenxandy.

52.Whatisthedifferencebetween1,001,000and999,999?

(A)101,001

(B)1999

(C)10,001

(D)1001

53.If3x–6=2,findx.

(A)

(B)8

(C)–

(D)

54.If–3(y+2)=9,findy.

(A)–3

(B)15

(C)–5

(D)3

55.Evaluate–4xy2z3ifx= ,y=–2,andz=1.

(A)–

(B)–16

(C)

(D)

56.WhatisthevalueoftheexpressionA= bh,whenh=2andb= ?

(A)

(B)

(C)

(D)

57.Simplify:–6–[2–(3a–b)+b]+a.

(A)4–3a+2b

(B)–6+3a+b

(C)–8+4a–2b

(D)–8+3a–b

58.Simplify:–2[–4(2–1)+(3+2)].

(A)18

(B)2

(C)–18

(D)–2

59.5:6as15:

(A)25

(B)16

(C)18

(D)12

60.Theratioofthe6inchesto6feetis

(A)1:6

(B)12:1

(C)1:12

(D)24:1

ANSWERSANDEXPLANATIONS

1.Thecorrectansweris(C).Thereciprocalofafractionisthefraction“turnedupside

down.” isequivalentto .Thereciprocalof is .Choice(D)isadistractor.

Becausexhasaprecisevalueintheproblem,wemustchooseananswerhavingaprecisevalue.

2.Thecorrectansweris(B).Theproductofanynumberanditsreciprocalis1.Therefore,

,andchoice(B)isthecorrectchoice.Evenifyoudidn’tknowthisrule,you

couldhaveexaminedtheanswersandeliminatedbothchoice(A),becausetheproductwasgreaterthan1,andchoice(D),becausetheproductwaslessthan1.

3.Thecorrectansweris(A).Thisproblemlooksmuchharderthanitreallyis.Thenumeratorofthiscomplexfractionisthesameasthedenominator.Whennumeratoranddenominatorareequivalent,thefractionisequalto1.Choice(A)isthecorrectanswer.

4.Thecorrectansweris(D).Thisisacomplexfractionrequiringallofyourskillsinworkingwithfractions.Toestimatethecorrectanswer,notethatthenumeratorisslightly

largerthan ,andthedenominatorisequivalentto ,or .

Therefore,anumberslightlylargerthan1dividedby isslightlylargerthan16.The

closestanswerischoice(D), ,whichisequivalentto .Tosolvetheproblemby

calculation,simplifythenumeratoranddenominator,andthendivide.

5.Thecorrectansweris(B).Thisisaprobleminwhichyoumustsubstitutethevaluesgivenintotheformula.Onceyoudothat,itisasimpleproblem.

Therefore,(B)isthecorrectanswer.Theotheranswerswouldhaveresultedifyouhadforgottentomultiplyoneofthenumbersinthenumerator.Choice(D)mighthavebeenchosenbysomeonewhodidn’tknowwhattodobutthoughtthemostdifficult-lookinganswerwouldbethebest.

6.Thecorrectansweris(A).Thesumoftheanglesofatriangleisalways180º.Thecorrectanswer,therefore,is(A),because45º+75º+60º=180º.Choices(B),(C),and(D)givesumslargerorsmallerthan180ºwhenaddedto45ºand75º.

7.Thecorrectansweris(C.)Notethatthebaseofthetriangleisthesameasthediameterofthecircle.Because ABCisisosceles,itsaltitudeisthesamelengthastheradiusofthecircle.Usetheformulafortheareaofatriangle,andsubstitutethecorrectvalues:

=25cm2

Thecorrectanswer,then,ischoice(C).Choice(A)istheareaofthecircle.

8.Thecorrectansweris(D).Thesemeasurementsdescribealargerectangularroom30feethigh.UsetheformulaV=l•w•htofindthevolume:

V=75feet•50feet•30feet

=112,500cubicfeet

Choices(B)and(C)usethewrongunits.Volumeisalwaysmeasuredincubicunits.

9.Thecorrectansweris(A).Astoremarkupof100%wouldexactlydoubletheprice.An80%markupalmostdoublestheprice.The$14jeansarepricedatalmostdoubletheircosttothestore.Byestimation,thebestanswerischoice(A).Tofigureprecisely,rememberthatan80%markupistheequivalentofmultiplyingthecostby180%,or1.80.

cost•1.80=14

cost=14÷1.80

cost=$7.78

10.Thecorrectansweris(B).Reducethe$14priceby25%.

25%of$14=$14×0.25=$3.50

$14–$3.50=$10.50(newprice)

Therefore,choice(B)isthecorrectanswer.Choice(A)indicatesareductionofonly25cents.Choice(C)representsareductionto25%oftheoriginalprice,ora75%decreaseinprice.

11.Thecorrectansweris(D).Substitute0.25intotheproblem.

=0.0625+0.5

=0.5625

Thismightbeagoodtimetoreviewthetableofsquaresandsquarerootshere.Choice(A)wouldhaveresultedifyouhadsquared0.25,gotten0.625,andaddeditto0.5.(0.25)2=0.0625.Choice(B)wouldhaveresultedifyouhadsquared0.25correctlybutthoughtthesquarerootof0.25was0.05.Choice(C)wouldhaveresultedifyouhadmadeerrorsinbothsteps.

12.Thecorrectansweris(C).Thefinancechargewillbethesumof %of$500,plus1%

of$250.Youcanwritethisasfollows:

(0.015•500)+(0.01•250)=$7.50+$2.50=$10

Youcanestimatetheanswerifyourememberthatpercentmeans“hundredthsof.”Onehundredthof$500is$5.00;onehundredthof$250is$2.50.Theonlyanswernearthissumischoice(C).Choices(B)and(D)wouldhaveresultedifyouhadmisplacedadecimalpoint.

13.Thecorrectansweris(B).Beginremovingtheinnermostgroupingsymbols,rewritingeachtimeasetofsymbolsisremoved:

Step1:1–[5+(3–2)]

Step2:1–[5+1]

Step3:1–[6]

Step4:1–6=–5

Aminussigninfrontofabracketorparenthesisreversesthesignofthenumberinside.Apositivesigndoesnot.

14.Thecorrectansweris(A).

Step1:–3–[–2+(5–6)–3]

Step2:–3–[–2+(–1)–3]

Step3:–3–[–2–1–3]

Step4:–3–[–6]

Step5:–3+6=+3

Aminussigninfrontofabracketorparenthesisreversesthesignofthenumberinside.A

positivesigndoesnot.

15.Thecorrectansweris(B).Eachpointonthelinehasacoordinateonthehorizontalaxisandoneontheverticalaxis.Thecoordinatesofpointslocatedatthelowerrightpartofthegrapharelowcostperitemandlargenumbersofitemsmanufactured.Therefore,interpretthisgraphtomeanthatwherelargernumbersofitemsaremanufactured,thecostperitemislower.

16.Thecorrectansweris(B).Find300onthehorizontalaxis.Drawaverticallineupwarduntilyoutouchtheline.Movehorizontallyfromthispointonthelinetotheverticalaxis.Notethatyoutouchtheverticalaxisatapointroughlyequivalentto$28.Wesuggestyouuseastraightedgetosketchyourline.

17.Thecorrectansweris(B).Find100onthehorizontalaxisandfollowthesameprocedureasabove.Thecoordinateontheverticalaxisisapproximately$75.

18.Thecorrectansweris(D).Findtheapproximatelocationof$5–$10ontheverticalaxis.Trytofollowahorizontalpathacrossuntilyouhitthelineofthegraph.Notethatyourpathdoesnotintersecttheline.Therefore,thegraphdoesnotpredicttheinformationaskedfor.

19.Thecorrectansweris(C).Byfar,theeasiestwaytosolvethisproblemistochangethefractionstodecimals:0.6+1.5+0.75=2.85.Ifyouweretoconverttofractions,the

correctanswerwouldbe .

20.Thecorrectansweris(A).Becarefultousethecorrectprocess.Firstusemultiplication,division,addition,andfinallysubtraction.

21.Thecorrectansweris(B).Becarefulincountingplacesandinpositioningthedecimalpoint.

22.Thecorrectansweris(C).Theareaoftheshadedportionisequaltotheareaofthesquare,lesstheareaofthecircle.Thelengthofthesideofthesquareisequaltothe

diameterofthecircle.Therefore,using forpi:(4"×4")–(π22)=16sq.in.– sq.

in.= sq.in.Ifyouansweredchoice(D), ,checkyourskillsinsubtractingfractions

fromwholenumbers.

23.Thecorrectansweris(C).Thelongwaytosolvethisproblemistomultiplyboththenumeratoranddenominatorout,andthendivide.Ifyounoticethat1004canalsobewritten

as108,theanswerisobviouslychoice(C).

1004=(102)4=102•102•102•102=108

24.Thecorrectansweris(B).Substitute3forx.Theproblem,then,istocompute35.

35=3•3•3•3•3

=243

Choice(C)wouldhaveresultedifyouhadmultiplied5×3,insteadof3timesitself5times.

25.Thecorrectansweris(B).Foreachteacher,thereare14students.Becausethereare14teachers,theremustbe14×14,or196,students.

26.Thecorrectansweris(C).Theratioofmentowomenis15:12,butthisratiomustbeexpressedinsimplestform.Because15and12have3asacommonfactor,theratioexpressedcorrectlyis5:4.Choice(C)isthecorrectchoice.

27.Thecorrectansweris(B).Tofindtheperimeter,weaddupthedimensionsofallofthesides.Notethattherearesomepartsthathavenotbeenassignedmeasurements,sowehavetoinferthattheyarethesameasthosecorrespondingpartswhosemeasurementshavebeendesignated.Beginningatthebottomandmovingclockwise,thedimensionsare:

5"+7"+1"+3"+3"+3"+1"+7"

Theseequal30inches.Ifyouchosechoices(A),(D),or(C),youfailedtoaddupallofthesegments.

28.Thecorrectansweris(A).Theareaismosteasilyfoundbymultiplyingthelengthofthefigurebyitswidth,andthensubtractingtheareaofthesmall3"×3"square.

(7"×5")–(3"×3")=Area

35sq.in.–9sq.in.=26sq.in.

Shapessuchasthisareoftenusedforirregularpiecesofcarpetingorcovering.

29.Thecorrectansweris(B).Thesurfaceofarectangularsolidsuchasabrickisfoundbycalculatingtheareaofeachfaceofthebrickandfindingthesumoftheareasofthefaces.Thebrickhassixfaces:

Twofaces6"×3";Total36sq.in.

Twofaces6"2";Total24sq.in.

Twofaces3"×2";Total12sq.in.

Total72sq.in.

Choice(D)iswrongbecauseitisnotwritteninsquareunits.

30.Thecorrectansweris(D).Calculatethesurfaceareaofthecube.Ithassixfaces,each2"×2".Itssurfacearea,then,is6×4sq.in.,or24sq.in.Itsvolumeisfoundbymultiplyingitslength×width×height,or2"×2"×2"=8cu.in.Theratioofsurfaceareatovolumeis24:8,or3:1.

31.Thecorrectansweris(C).ThePythagoreanTheoremisusedtofindthelengthofthesidesofrighttriangles.Thesquareofthelengthofthelongestside(thehypotenuse)isequaltothesumofthesquaresoftheothertwosides.Onceweknowthesquareofthelengthofthelongestside,itiseasytofindthelength.

(AC)2=(AB)2+(BC)2

(AC)2=32+42

(AC)2=25

(AC)= =5

32.Thecorrectansweris(D).Thisisatwo-stepproblem.First,findthelengthofthehypotenuse,soyouknowhowfartheotherpersonisdriving.

(AC)2=(AB)2+(BC)2

=(40)2+(30)2

=1600+900

(AC)= =50miles

ThepersondrivingfromCtoAmustdrive50milesat50mph.Heorshewillgettherein1hour.Theothermustdrive70miles.Togettherefirst,heorshemustdrivefasterthan70milesperhour.

33.Thecorrectansweris(B).Thisisasimpledivisionproblem.Divide1.0by0.25.Hereis

anotherwaytosolvethisproblem.Since0.25= ,thereciprocalof is4.

34.Thecorrectansweris(C).Astraightlinerepresentsa“straightangle”of180º.Anangleof60ºisgiven,so Cmustbe120ºtocompletetheline.Alltheanglesinatriangleaddedtogetherequal180º,therefore:

35.Thecorrectansweris(B).Theboy’sageisMyears.HisolderbrotherisM+6yearsold,andhisyoungersisterisM–4yearsold.Addingthethreeagestogether,

M+(M+6)+(M–4)=3M+2

36.Thecorrectansweris(D).Thisisaliteralproblemrequiringyouto“thinkwithoutnumbers.”Creatingmentalpicturesmayhelpyousolvethistypeofproblem.IfeachpersoninagroupmakesLnumberofthings,thegroup’soutputwillbetheproductofthenumberofpeopleinthegroupandthenumberofthingseachmakes.Choice(D)representstheproductandisthecorrectchoice.

37.Thecorrectansweris(A).Thisproblemasksyoutofindspeedorrate.Speedorrateisfoundbydividingthedistancetraveledbythetimerequired.Thechoiceinwhichdistanceisdividedbytimeischoice(A).

38.Thecorrectansweris(B).RroomseachwithSsquarefeetcontainatotalofRSsquarefeet.Becausethereare144squareinchesineachsquarefoot,theroomscontain144RSsquareinches.

39.Thecorrectansweris(B).Ifthedrawingisat scale,itmeansthatthedrawingis

thesizeoftheactualwheel.Therefore,multiplythesizeofthedrawingby16.

1.8×16=28.8inches.

40.Thecorrectansweris(B).Ascaleof1:24,000meansthat1inchonthemapequals24,000inchesontheground.24,000inchesequal2000feet.

41.Thecorrectansweris(C).If2inchesequal24feet,1inchequals12feet.Alinerepresenting72feet,therefore,mustbe6incheslong(72÷12=6).

42.Thecorrectansweris(A).Inthisproblem,thebasesarethesame,soyoumustsubtracttheexponentofthedivisorfromthatofthedividendtofindtheanswer:

55÷53=55–3=52=25

Therefore,choice(A)isthecorrectanswer.Notethatthebasesmustbeidenticalandtheexponentsmustbesubtracted.Youcancheckthisanswerbymultiplyingeachnumberout

anddividing.

43.Thecorrectansweris(B).Whenmultiplying,ifthebasesareidentical,addtheexponents:

X3•X2=X5

Notethatbecausewearemultiplying,thecoefficientremains1.

44.Thecorrectansweris(C).Thisproblemrequirestwosteps.First,findthesmallestnumberdivisibleby14and12(theleastcommonmultiple,orLCM).Secondly,addthatnumberto1andconverttotimeofday.TheLCMof14and12is84.Botheventswilloccursimultaneously84minutespast1:00or2:24p.m.

45.Thecorrectansweris(C).Here,threeeventsoccurperiodically,sowemustfindtheLCMof4,11,and33,andaddthatnumberto1950.Thatyearwillbethenextcommonoccurrence.TheLCMof4,11,and33is132.1950+132=2082.Therefore,choice(C)iscorrect.

46.Thecorrectansweris(D).Inthewholenumbersystem,everyothernumberisodd,andeveryotheriseven.Ifxisodd,x+1iseven,x+2isodd,x+3iseven,andsoforth.Also,ifxisodd,x–1iseven,x–2isodd,andx–3iseven.Ifanevenoroddnumberisdoubled,theoutcomeiseven.Therefore,ifxisodd,2x+1isodd,x–2isodd,and4x–3isodd.

47.Thecorrectansweris(C).Arrangetheperiodsoftimeincolumnsandaddasyouwouldaddwholenumbers:

4hr.17min.

3hr.58min.

45min.

+7hr.12min.

14hr.132min.

Weknowthereare60minutesineachhour.Therefore,132minutesequals2hours12minutes.Thecorrectanswerforthisadditionis16hours12minutes,orchoice(C).Whenworkingwithunitsthatmeasuretime,volume,andlength,itisusuallybesttorepresenttheanswerusingasmanylargerunitsaspossible.That’swhy16hours12minutesispreferableto14hours132minutesasananswer.

48.Thecorrectansweris(A).Youdonothavetocalculatethisanswer.If8peopleare

sharingequallyof8poundsandsomeouncesoffruit,eachpersonwouldreceive1poundandafewounces.

49.Thecorrectansweris(B).Thisisasubtractionproblem.Youmustfindthedifferencebetweenthelengthsoftimerequiredtofinishtherace.Aswithotherproblemsinvolvingunitsofmeasurement,youmustworkcarefully.

3hr.2min.24sec.

–2hr.12min.38sec.

Because38secondsislargerthan24secondsand12minutesislargerthan2minutes,borrowfromtheminutecolumnandthehourcolumnandrewritetheproblemasfollows:

2hr.61min.84sec.

–2hr.12min.38sec.

0hr.49min.46sec.

50.Thecorrectansweris(A).Thefirst6isinthebillionsplace;thesecond,inthehundredthousandsplace.Ifyouhadtroublewiththisproblem,reviewthesectionsonhowtoreadnumbersanddetermineplacevaluesinyourmathtextbook.

51.Thecorrectansweris(C).Thisgraphcontainsalinethathaspointswithcoordinates(1,1),(2,2),(3,3),and(4,4).Fromonepointtoanotherthevalueofthex-coordinatechangesjustasmuchasthevalueofthey-coordinate.Therefore,choice(C)isthecorrectanswer.Thislineisata45ºanglefromthex-axisandwillbecreatedwheneverthex-andy-coordinatesareequal.

52.Thecorrectansweris(D).Thisisasimplesubtractionproblemdesignedtotesthowcarefullyyoucansubtract.Itispossibletocalculatethecorrectanswerwithoutpencilandpaper.999,999isonly1lessthanamillion,and1,001,000is1000greaterthanamillion.Thedifference,then,is1000+1,or1001.Or,youmayfiguretheprobleminthefollowingway:

1001000

–999999

1001

53.Thecorrectansweris(A).Thisequationcanbesolvedintwosteps:

Step1:Move–6totherightsideandchangethesign.

3x=2+6

3x=8

Step2:Dividebythecoefficientofthevariable.

54.Thecorrectansweris(C).

Step1:First,removethegroupingparentheses:

–3(y+2)=9

–3y–6=9

Step2:Thenmove–6totherightside,changeitssign,andcombine:

–3y=9+6

–3y=15

Step3:Dividethroughbythecoefficientofthevariable:

y=–5

55.Thecorrectansweris(A).Tosolvethisproblem,simplypluginthevaluesgivenandmultiply:

Step1:–4xy2z3

Step2:

Step3:

Step4:

56.Thecorrectansweris(B).Hereagain,simplypluginthevaluesandmultiplythefractions:

Step1:

Step2:

Step3:

Ifyouhadtroublewiththesefractions,reviewmultiplicationoffractions.

57.Thecorrectansweris(C).Whensimplifying,beginwiththeinnermostgroupingsymbolsfirst,andworkyourwayoutward:

Step1:–6–[2–(3a–b)+b]+a

Step2:–6–[2–3a+b+b]+a

Step3:–6–[2–3a+2b]+a

Step4:–6–2+3a–2b+a

Step5:–8+4a–2b

58.Thecorrectansweris(D.)Beginwiththeinnermostparenthesesandworkyourwayoutward.Notethataminussigninfrontofagroupingsymbolreversesthesignsofallnumberswithin:

Step1:–2[–4(2–1)+(3+2)]

Step2:–2[–4(1)+(5)]

Step3:–2[–4+5]

Step4:–2[1]=–2

59.Thecorrectansweris(C).Thisproportionasksyoutofindthemissingelement.Aproportionisastatementofequalitybetweentworatios,soweknowthat5bearsthesamerelationshipto15as6doestotheunknownnumber.Because3×5equals15,weknow3×6equalstheunknownnumber.Thenumber,thus,is18.Thecompletedproportionshouldread:5:6as15:18.Proportionsmayalsobewrittenwithasetoftwocolonsreplacingtheword“as.”Inthiscase,theproportionwouldread:5:6::15:18.

60.Thecorrectansweris(C).

Step1:Tofindthecorrectratio,writeitas:

Step2:Rewriteeachquantityininches:

Step3:Simplifytheratio:

SUMMINGITUP

•Ifyou’reintheeighthgrade,youmustknowbasicarithmetic,fundamentaloperationsusingfractionsanddecimals,percents,andverybasicalgebraandgeometry.

•Ifyou’reinthetwelfthgradeseekinganextrayearbeforecollege,youshouldbethoroughlyfamiliarwithcomplexalgebraandgeometry.

•Rememberthatscoringofyourhighschoolentranceexamisbasedonyourgrade.Youdonotneedtoknowwhatyouhavenotyetbeentaught,butyoumusthavemasteredallthemathappropriatetoyourgradelevel.

•Addition,subtraction,multiplication,anddivisionarethebasicoperationsuponwhichthestructureofmathematicsisbased.

•Makesureyounotetheproblemsthatweredifficultforyou,aswellasthosethatwereeasy,andadjustyourstudyplans.

Chapter11

QuantitativeAbility(SSATandISEE)

OVERVIEW

•Whatmakesmultiple-choicematheasier?•Howdoyousolvemultiple-choiceQuantitativeAbilityquestions?•Whatdosmarttest-takersknow?•Testyourselfquizzes•Answerkeys•Summingitup

WHATMAKESMULTIPLE-CHOICEMATHEASIER?Howcanonekindofquantitativeabilityquestionpossiblybeeasierthananother?Well,multiple-choicemathiseasierthanthemathtestsyoutakeinschool.Why?Simple.Becauseit’smultiplechoice,thecorrectanswerisalwaysonthepageinfrontofyou.Soevenifyouareestimating,you’llbeabletonarrowdownthechoicesandimproveyourguessingodds.

Somemultiple-choicequestionsrequirenocalculationatall;thecorrectanswerisbaseduponyourgraspoftheconceptsintroducedbythequestion.Somequestionsarestraightcalculations;othersarepresentedintheformofwordproblems.Someincludegraphs,charts,ortablesthatyouwillbeaskedtointerpret.Allthequestionshaveeitherfour(ISEE)orfive(SSAT)answerchoices.Thesechoicesarearrangedinorderbysizefromsmallesttolargestorfromlargesttosmallest.

HOWDOYOUSOLVEMULTIPLE-CHOICEQUANTITATIVEABILITYQUESTIONS?Althoughitsoundsquiteofficial,QuantitativeAbilityisreallyjustafancywaytosay“math.”

Keepingthatinmind,walkthroughthefollowingstepsandfirmupyourattackplanforhandlingthemathproblemsyou’llfindonyourtest.

FiveStepsforMultiple-ChoiceMath:GettingItRight

Readthequestioncarefullyanddeterminewhat’sbeingasked.Decidewhichmathprinciplesapplyandusethemtosolvetheproblem.Lookforyouransweramongthechoices.Ifit’sthere,markitandgoon.Iftheansweryoufoundisnotthere,recheckthequestionandyourcalculations.Ifyoustillcan’tsolvetheproblem,eliminateobviouslywronganswersandtakeyourbestguess.

Nowlet’stryoutthesethreestepsonacoupleofmultiple-choicemathquestions.

Inthefigureabove,x=

(A)15°

(B)30°

(C)40°

(D)60°

(E)75°TheproblemasksyoutofindthemeasureofoneangleofrighttrianglePQR.Twomathprinciplesapply:(1)thesumofthemeasures,indegrees,oftheanglesofatriangleis180°,and(2)45°-45°-90°righttriangleshavecertainspecialproperties.BecausePQ=PS,trianglePQSisa45°-45°-90°righttriangle.Therefore,anglePQS=45°andanglePQR=45°+15°=60°.Therefore,anglex=180°–90°–60°=30°.Thecorrectansweris(B).Ifxandyarenegativenumbers,whichofthefollowingisnegative?

(A)xy

(B)(xy)2

(C)(x–y)2

(D)x+y

(E)

Theproblemasksyoutopickananswerchoicethatisanegativenumber.Theprinciplesthatapplyarethosegoverningoperationswithsignednumbers.Becausexandyarenegative,bothchoices(A)and(E)mustbepositive.Asforchoices(B)and(C),aslongasneitherxnoryiszero,thoseexpressionsmustbepositive.(Anynumberotherthanzerosquaredgivesapositiveresult.)Choice(D),however,isnegativebecauseitrepresentsthesumoftwonegativenumbers.Byapplyingtherulesgoverningsignednumberstoeachanswerchoice,youcandeterminethatchoices(A),(B),(C),and(E)canonlybepositivenumbers.

Therefore,thecorrectanswermustbechoice(D).Ifyouhavetroubleworkingwithletters,trysubstitutingeasynumbersforxandyineachchoice.

TIPLookforshortcuts.Mathproblemstestyourmathreasoning,notyourabilitytomakeendlesscalculations.Ifyoufindyourselfcalculatingtoomuch,you’veprobablymissedashortcutthatwouldhavemadeyourworkeasier.

WHATDOSMARTTEST-TAKERSKNOW?Someoftheseyou’veheardbefore,somewillbenewtoyou.Whateverthecase,readthem,learnthem,lovethem.Theywillhelpyou.

TheQuestionNumberTellsYouHowHardtheQuestionWillBe

Justasinmostoftheothertestsections,thequestionsgofromeasytohardasyouworktowardtheend.Thefirstthirdofthequestionsareeasy,themiddlethirdareaveragebutharder,andthefinalthirdgetmoreandmoredifficult.Takealookatthesethreeexamples.Don’tsolvethemyet

(you’llbedoingthatinacoupleofminutes),justgetanideaofhowthelevelofdifficultychangesfromQuestion1toQuestion12toQuestion25.

1.Ifx–2=5,thenx=

(A)–10

(B)–3

(C)

(D)3

(E)7

12.Forhowmanyintegersxis–7<2x<–5?

(A)None

(B)One

(C)Two

(D)Three

(E)Indefinitenumber

25.Inasetof5books,notwoofwhichhavethesamenumberofpages,thelongestbookhas150pagesandtheshortestbookhas130pages.Ifxpagesistheaverage(arithmeticmean)ofthenumberofpagesinthe5-bookset,whichofthefollowingbestindicatesallpossiblevaluesofx?

(A)130<x<150

(B)131<x<149

(C)133<x<145

(D)134<x<145

(E)135<x<145

Canyouseethedifference?YoucanprobablydoQuestion1withyoureyesclosed.ForQuestion12,youprobablyhavetoopenyoureyesanddosomecalculationsonscratchpaper.Question25maycauseyoutowincealittleandthengetstartedonsomeheavy-dutythinking.

EasyQuestionsHaveEasyAnswers;DifficultQuestionsDon’t

Duh,butwithanexplanation.Theeasyquestionsarestraightforwardanddon’thaveanyhiddentricks.Theobviousanswerisalmostalwaysthecorrectanswer.SoforQuestion1theanswerisindeedchoice(E).

Whenyouhitthedifficultstuff,youhavetothinkharder.Theinformationisnotstraightforwardandtheanswersaren’tobvious.Youcanbetthatyourfirst-choice,easyanswerwillbewrong.Ifyoudon’tbelieveit,let’stakeacloserlookatthedifficultsolutionforQuestion25.

25.Inasetof5books,notwoofwhichhavethesamenumberofpages,thelongestbookhas150pagesandtheshortestbookhas130pages.Ifxpagesistheaverage(arithmeticmean)ofthenumberofpagesinthe5-bookset,whichofthefollowingbestindicatesallpossiblevaluesofxandonlypossiblevaluesofx?

(A)130<x<150

(B)131<x<149

(C)133<x<145

(D)134<x<145

(E)135<x<145Yes,it’sdifficultmostlybecausetheprocessyouhavetousetofindthesolutionisdifficult.Let’sstartbyeliminatinganswerchoices.Choice(A)isabadguess.Youseethesameinfoasyouseeinthewordproblemsoyoufigureit’sgottoberight.Wrong.Allitdoesissaythattheshortestbookis130pages,thelongestbookis150pages,andtheaverageisbetween130and150.Simpleandwrong.

Choice(B)illustratesthereasoningthat“notwobookshavethesamenumberofpages,sotheaveragemustbeonepagemorethantheshortestbookandonepagelessthanthelongest.”Remember,it’sadifficultquestion,it’salsonotthateasyananswer.

OKthen,let’sskiptothecorrectanswer,whichischoice(E),andfindouthowwegotthere.First,youwanttofindtheminimumvalueforx,soyouassumethattheotherthreebookscontain131,132,and133pages.Sotheaveragewouldbe:

Soxmustbemorethan135.Nowassumethattheotherthreebookscontain149,148,and147pages.Thentheaveragelengthofallfivebookswouldbe:

Thenxwouldbegreaterthan135butlessthan145.

WhenGuessingatHardQuestions,YouCanTossOutEasyAnswers

Nowthatyouknowthedifficultquestionswon’thaveeasyorobviousanswers,useaguessing

strategy.(Useallthehelpyoucanget!)Whenyouhavelessthanaclueaboutadifficultquestion,scantheanswerchoicesandeliminatetheonesthatseemeasyorobvious,suchasanythatjustrestatetheinformationinthequestion.Ditchthoseandthentakeyourbestguess.

QuestionsofAverageDifficultyWon’tHaveTrickAnswers

Let’slookagainatQuestion12:

12.Forhowmanyintegersxis–7<2x<–5?

(A)None

(B)One

(C)Two

(D)Three

(E)IndefinitenumberThisisabitmoredifficultthanQuestion1,butit’sstillprettystraightforward.Thereisonlyoneintegerbetween–7and–5,andthat’s–6.There’salsoonlyonevalueforintegerxsothat2xequals–6,andthatis–3.Getit?2(–3)=–6.So,choice(B)isthecorrectanswer.Trustyourjudgmentandyourreasoning;notrickshere.

It’sSmarttoTestAnswerChoices

Everystandardmultiple-choicemathproblemincludesfour(ISEE)orfive(SSAT)answerchoices.Oneofthemhastobecorrect;theothersarewrong.Thismeansthatit’salwayspossibletosolveaproblembytestingeachoftheanswerchoices.Justplugeachchoiceintotheproblemandsoonerorlateryou’llfindtheonethatworks!Testinganswerchoicescanoftenbeamucheasierandsurerwayofsolvingaproblemthanattemptingalengthycalculation.

WhenTestingChoices,It’sSmarttoStartNeartheMiddle

Remember,theanswerissomewhererightinfrontofyou.Ifyoutestalltheanswerchoices,you’llfindtherightone.However,thesmartplacetostartisalwaysasclosetothemiddleaspossible.Why?Becausethequantitiesinthechoicesarealwaysarrangedinorder,eitherfromsmallesttolargestortheotherwayaround.Ifyoustartatthemiddleandit’stoolarge,you’lljusthavetoconcentrateonthesmallerchoices.There,you’veknockedoffsomechoicesinaheartbeat.Let’sgiveita“test”run,sotospeak....

Ifarectanglehassidesof2xand3xandanareaof24,whatisthevalueofx?

(A)2

(B)3

(C)4

(D)5

(E)6Youknowthatoneoftheseisright.Getstartedbytestingchoice(C),andassumethatx=4.Thenthesideswouldhavelengths2(4)=8and3(4)=12andtherectanglewouldhaveanareaof8×12=96.Because96islargerthan24(theareainthequestion),startworkingwiththesmalleranswerchoices.[Whichmeans,ofcourse,thatyoucanimmediatelyforgetaboutchoices(D)and(E).Great!]Whenyouplug3intothefiguring,youget2(3)=6and3(3)=9and6×9=54;stilltoolarge.Theonlychoiceleftischoice(A),anditworks.

Nowtrythistestingbusinesswithamoredifficultquestion:

Afarmerraiseschickensandcows.Ifheranimalshaveatotalof120headsandatotalof300feet,howmanychickensdoesthefarmerhave?

(A)50chickens

(B)60chickens

(C)70chickens

(D)80chickens

(E)90chickensHeregoes—startingwithchoice(C).Ifthefarmerhas70chickens,shehas50cows.(Youknowthefarmerhas120animals,becausetheyeachhaveonlyonehead,right?)Sonowyou’retalkingabout70×2=140chickenfeetand50×4=200cowfeet,foragrandtotalof340animalfeet.Well,that’smorethanthe300animalfeetinthequestion.Howwillyoulosesomeofthosefeet?First,assumethatthefarmerhasmorechickensandfewercows(cowshavemorefeetthanchickensdo).Givechoice(D)—80—atry.Test80×2=160and40×4=160;yourtotalis320feet,whichiscloserbutnotquiteright.Theonlyanswerleftischoice(E),andthat’sthecorrectone.Checkitout:

90×2=180and30×4=120andthetotalis...300!

It’sEasiertoWorkwithNumbersThanwithLetters

Becausenumbersaremoremeaningfulthanletters,trypluggingthemintoequationsandformulasinplaceofvariables.Thistechniquecanmakeproblemsmucheasiertosolve.Herearesomeexamples:

Ifx–4is2greaterthany,thenx+5ishowmuchgreaterthany?

(A)l

(B)3

(C)7

(D)9

(E)11Chooseanyvalueforx.Let’ssayyoudecidetomakex=4.Allright,4–4=0,and0is2greaterthany.Soy=–2.Ifx=4,thenx+5=4+5=9,andsox+5is11morethany.Therefore,thecorrectanswerischoice(E).

Theunitcostofpensisthesameregardlessofhowmanypensarepurchased.Ifthecostofppensisddollars,whatisthecost,indollars,ofxpens?

(A)xd

(B)xpd

(C)

(D)

(E)

Timetopluginsomerealnumbersbecauseyouneedrealmoneytobuyanything,includingpens.Saythatfourpens(p)cost$2(d),soeachpenwouldcost50cents.Andsaythatyoureallyonlyneedonepen(x),soyou’respendingonly$0.50.Thenp=4,d=2,andx=1,andtherightanswerwouldbe0.5.Now,startusingthesenumberswiththeanswerchoices:

(A)xd=(1)(2)=2(Nope.)

(B)xpd=(1)(4)(2)=8(Nope,again.)

(C) = =0.5(Yes,thereitis.)

(D) = =2(Nope.)

(E) = =8(Nope.)

Ifaquestionasksforanoddintegeroraneveninteger,goaheadandpickanyoddorevenintegeryoulike.

It’sOkaytoWriteinYourTestBooklet,SoUseItforScratchWork

Thetestbookletisyours,sofeelfreetouseitforyourscratchwork.Also,goaheadandmarkupanydiagramswithlengthorangleinformation;ithelps.Butdon’twastetimetryingtoredraw

diagrams;it’sjustnotworthit.

ARealityCheckCanHelpYouEliminateAnswersThatCan’tPossiblyBeRight

Knowingwhetheryourcalculationsshouldproduceanumberthat’slargerorsmallerthanthequantityyoustartedwithcanpointyoutowardtherightanswer.It’salsoaneffectivewayofeliminatingwronganswers.Here’sanexample:

Usinghisbike,Darylcancompleteapaperroutein20minutes.Jennifer,whowalkstheroute,cancompleteitin30minutes.Howlongwillittakethetwokidstocompletetherouteiftheyworktogether,onestartingateachendoftheroute?

(A)8minutes

(B)12minutes

(C)20minutes

(D)30minutes

(E)45minutesImmediatelyyoucanseethatchoices(C),(D),and(E)areimpossiblebecausethetwokidsworkingtogetherwillhavetocompletethejobinlesstimethaneitheroneofthemworkingalone.Infact,thecorrectanswerischoice(B),12minutes.

Multiplyby60toclearfractions:

3x+2x=60

5x=60

x=12

TIPCirclewhat’sasked.Formultiple-choicemathquestions,

circlewhat’sbeingaskedsothatyoudon’tpickawronganswerbymistake.Thatway,forexample,youwon’tpickananswerthatgivesaperimeterwhenthequestionasksforanarea.

YourEyeIsaGoodEstimator

Figuresinthestandardmultiple-choicemathsectionarealwaysdrawntoscaleunlessyouseethewarning“Note:Figurenotdrawntoscale.”Thatmeansyoucansometimessolveaproblemjustbylookingatthepictureandestimatingtheanswer.Here’showthisworks:

IntherectanglePQRSshown,TUandWVareparalleltoSR.IfPS=6,UV=1,andPR(notshown)=10,whatistheareaofrectangleTUVW?

(A)8

(B)12

(C)16

(D)24

(E)32Tosolvetheproblem,youwillneedtofindthelengthofTU.YoucandothisbyusingthePythagoreanTheorem.ThetrianglePSRhassidesof6and10,soSR=8.BecauseTU=SR,TU=8,sotheareaofthesmallrectangleisequalto1×8=8.Asanalternative,youcouldsimplyestimatethelengthofTU.TUappearstobelongerthanPS(6),andTUmustbeshorterthanPR(10).Therefore,TUappearstobeapproximately8.Andtheareamustbeapproximately1×8=8.Isthatsufficientlyaccuratetogettherightanswer?Lookatthechoices.Choice(A)is8,andit’stheonly

choicethatisevencloseto8.

IfSomeQuestionsAlwaysGiveYouTrouble,SaveThemforLast

Youknowwhichlittledemonshauntyourmathskills.Ifyoufindquestionsthatyouknowwillgiveyounightmares,savethemforlast.Theywilltakeupalotofyourtime,especiallyifyou’repanicking,andyoucanusethattimetodomoreoftheeasierquestions.

TESTYOURSELFQUIZZESTakethefollowingquizzestohelpyoudeterminewhatyourweaknessesmightbe.Answerscanbefoundhere.

TestYourself1

Directions:Eachofthefollowingquestionshasfoursuggestedanswers.Decidewhichoneisbest.

1.Inthesimplestform,–11–(–2)is

(A)7

(B)9

(C)–9

(D)–11

2.Findtheaverageof6.47,5.89,3.42,0.65,and7.09.

(A)3.920

(B)4.704

(C)4.705

(D)5.812

3.Change0.03125toacommonfraction.

(A)

(B)

(C)

(D)

4.Arollofcarpetingwillcover224squarefeetoffloorspace.Howmanyrollswillbeneededtocarpetaroom36'×8'andanother24'×9'?

(A)2.25

(B)2.50

(C)4.25

(D)4.50

5.Afterdeductingadiscountof30%,thepriceofacoatwas$35.Whatwastheregularpriceofthecoat?

(A)$24.50

(B)$42

(C)$50

(D)$116.67

6.Findthesumof–16,14,–38,26,and20.

(A)6

(B)4

(C)0

(D)–6

7.Thenumberofcubicfeetofsoilneededforaflowerbox3feetlong,8incheswide,and1footdeepis

(A)24

(B)12

(C)

(D)2

8.Usingexponents,write359inexpandedform.

(A)(3×102)+(5×10)+9

(B)(32×10)+(52×10)+9

(C)(3×102)+(5×10×32)

(D)(3×103)+(5×10+32)

9.Thescaleofacertainmapis4inches=32miles.Thenumberofinchesthatwouldrepresent80milesis

(A)16

(B)12

(C)10

(D)8

10.15is20%of

(A)3

(B)18

(C)35

(D)75

11.Thebargraphbelowshowsthepopulationofatownduringa30-yearperiod.Bywhatnumberofpeopledidthepopulationdecreasebetween1960and1970?

(A)1000

(B)1500

(C)2500

(D)15,000

12.Whichofthesefractionsisgreaterthan ?

(A)

(B)

(C)

(D)

13.Ify=x–4,theny–2=

(A)x–6

(B)x+2

(C)x–2

(D)x+6

14. =

(A)

(B)

(C)

(D)

15.Ifa–b=7andaandbarebothpositiveintegers,whatistheminimumpossiblevalueofa+b?

(A)11

(B)9

(C)8

(D)0

TestYourself2

Directions:Eachofthefollowingquestionshasfoursuggestedanswers.Decidewhichoneisbest.

1.Whatisthesumof and ?

(A)

(B)

(C)

(D)

2.Astoresoldsuitsfor$65each.Thesuitscostthestore$50each.Whatisthepercentageofincreaseofsellingpriceovercost?

(A)40%

(B) %

(C) %

(D)30%

3.72dividedby0.0009=

(A)0.125

(B)80

(C)800

(D)80,000

4.Whatisthesimpleintereston$460fortwoyearsat %?

(A)$20

(B)$23

(C)$25

(D)$28

5.Ahouseplanusesthescale inch=1foot,and,inthedrawing,thelivingroomis7

incheslong.Ifthescaleischangedto1inch=1foot,whatwillthelengthofthelivingroombeinthenewdrawing?

(A)18inches

(B)28inches

(C)30inches

(D)36inches

6.Duringhissummervacation,aboyearned$14.50perdayandsaved60%ofhisearnings.Ifheworked45days,howmuchdidhesave?

(A)$287.93

(B)$391.50

(C)$402.75

(D)$543.50

7. expressedasapercentis

(A)40.625%

(B)42.188%

(C)43.750%

(D)45.313%

8.Iftheformulafortheareaofacircleisπr2,findtheareaofacirclethathasadiameter8in.long.

(A)50.24sq.in.

(B)100.48sq.in.

(C)102.34sq.in.

(D)200.96sq.in.

9.If8>x>5and3>y>–2,then

(A)x<y

(B)x≤y

(C)x≥y

(D)x>y

10.53×34=

(A)5×3×3×4

(B)5×5×5×3×3×3

(C)5×5×5×3×3×3×3

(D)5×5×5×5×3×3×3

11.Ifa+b=200°,andc+d+e+f=140°,whatisthenumberofdegreesinangleg?

(A)10°

(B)20°

(C)30°

(D)45°

12.Ifasteelbaris0.39feetlong,itslengthininchesis

(A)lessthan4.

(B)between4and4 .

(C)between4 and5.

(D)between5and6.

13.Thegraphbelowshowsthenumberofhoursineach8-hourworkingdaythatMr.Smithspentonthetelephonelastweek.WhatfractionrepresentstheaverageofthepartofthedayMr.Smithspendsonthetelephone?

(A)

(B)

(C)

(D)

14.Multiply:(10a3)(5a).

(A)14a3

(B)50a4

(C)40a3

(D)15a4

15.TheformulafortheareaofatriangleisA= bh.FindAifb=12andh=10.

(A)22

(B)32

(C)50

(D)60

TestYourself3

Directions:Eachofthefollowingquestionshasfoursuggestedanswers.Decidewhichoneisbest.

1.Findthehypotenuseofarighttriangleifthelegsare6and8.

(A)8

(B)9

(C)10

(D)11

2.Onemanearns$24,000peryear.Anotherearns$1875permonth.Howmuchmoredoesthefirstmanmakeinayearthanthesecondman?

(A)$2500

(B)$2000

(C)$1500

(D)$1000

3.Thesquarerootof53isclosestto

(A)8

(B)7

(C)6.5

(D)6

4.Solveforx:4x–8=16.

(A)4

(B)6

(C)8

(D)10

5.Alibrarycontains60booksonartsandcrafts.Ifthisis0.05%ofthetotalnumberofbooksontheshelves,howmanybooksdoesthelibraryown?

(A)1200

(B)12,000

(C)120,000

(D)1,200,000

6.ThegraphbelowrepresentsMs.Lawson’smonthlybudget.Whatpercentageofhersalarydoesshespendonthingsotherthanhousingcosts?

(A)35%

(B)40%

(C)52%

(D)65%

7.FindthelengthininchesofdiagonalDBintherectanglebelow:

(A)5

(B)7

(C)9

(D)11

8.Awomanborrowed$5000andagreedtopay %simpleinterest.Ifsherepaidthe

loanin6months,howmuchinterestwouldshepay?

(A)$287.50

(B)$575

(C)$2875

(D)$5750

9.Solveforx:2x2–6=44.

(A)±4

(B)±5

(C)±12.5

(D)±25

10.Atanannualrateof$0.40per$100,whatistheannualfireinsurancepremiumforahousethatisinsuredfor$80,000?

(A)$3.20

(B)$32

(C)$320

(D)$3200

11.Simplify: .

(A)

(B)

(C)

(D)

12.Whichofthefollowingistrue?

(A)

(B)

(C)

(D)

13.Findthemissingtermintheproportionx:5=24:30.

(A)3

(B)4

(C)6

(D)8

14.Adistanceof25milesisrepresentedonamapby inches.Onthemap,howmany

milesarerepresentedby1inch?

(A)6

(B)8

(C)10

(D)12

15.Iftheschooltaxrateinacommunityis$33.50per$1000ofassessedvaluation,findtheamountoftaxonpropertyassessedat$50,000.

(A)$167.50

(B)$420.02

(C)$1675.00

(D)$4200.02

TestYourself4

Directions:Eachofthefollowingquestionshasfoursuggestedanswers.Decidewhichoneisbest.

1.Ajuniorhighschoolestablishedaschoolsavingsaccount,andtheeighthgradesaved$990.Ifthiswas45%ofthetotalamountsavedbytheschool,findthetotalamount.

(A)$220

(B)$1210

(C)$2100

(D)$2200

2.Ateamwon25gamesandlost8.Whatistheratioofthenumberofgameswontothenumberofgamesplayed?

(A)33:25

(B)25:33

(C)58:25

(D)25:58

3.Acircularflowergardenhasadiameterof21feet.Howmanyfeetoffencingwillberequiredtoenclosethisgarden?

(A)72

(B)66

(C)60

(D)

4.Solveforx:7x–4=115.

(A)17

(B)19

(C)21

(D)22

5.Thebestapproximateanswerfor1.2507623×72.964896is

(A)100

(B)90

(C)70

(D)9

6.

(A)

(B)

(C)

(D)

7.Theareaofarectangularroomis1000squarefeet.Ifthewidthoftheroomis25feet,whatisthelengthoftheroom?

(A)30feet

(B)35feet

(C)40feet

(D)45feet

8.Whatwillanorderfor345machineboltsat$4.15perhundredcost?

(A)$0.1432

(B)$1.14

(C)$14.32

(D)$143.20

9.Onthefirstdayofitsdrive,ajuniorhighschoolraised$40,whichwas %ofits

quota.Howmuchwasthequota?

(A)$120

(B)$100

(C)$80

(D)$68

10.Accordingtothe1950census,14,830,192peoplewerethenlivinginthestateofNewYork.Roundthisnumberofftothenearestthousand.

(A)14,000

(B)148,300

(C)1,483,000

(D)14,830,000

11.Simplify:x–[8–(x–2)].

(A)–10

(B)2x–10

(C)2x+10

(D)2x–6

12.Sharonmadeacirclegraphanddrewanangleof45ºtoshowthenumberofpupilsinhergradewhohadearnedanAinmathematics.WhatpercentofhergradeearnedanA?

(A)10%

(B)12 %

(C)16 %

(D)20%

QUESTIONS13–15REFERTOTHEGRAPHBELOW.

13.Thenumberofpupilshavingthehighestintelligencequotient(I.Q.)isabout

(A)5

(B)20

(C)145

(D)160

14.ThenumberofpupilshavinganI.Q.of80isidenticalwiththenumberofpupilshavinganI.Q.of

(A)68

(B)100

(C)110

(D)128

15.Theintelligencequotientthathasthegreatestfrequencyis

(A)95

(B)100

(C)105

(D)160

TestYourself5

Directions:Eachofthefollowingquestionshasfoursuggestedanswers.Decidewhichoneisbest.

1.Yourollafair,six-sideddietwice.Whatistheprobabilitythatthediewilllandwiththesamesidefacingupbothtimes?

(A)

(B)

(C)

(D)

2.Theareaoftheshadedportionoftherectanglebelowis

(A)90sq.in.

(B)54sq.in.

(C)45sq.in.

(D)36sq.in.

3.Theratioof16:36is

(A)3:5

(B)3:21

(C)4:8

(D)4:9

4.595writteninexpandedformwithexponentsis

(A)(5×10)+(9×10)+(5×1)

(B)(5×102)+(9×10)+5

(C)(5×102)+(9×10)+(5×5)

(D)(5×102)+(3×10)+5

5.Janesaved$5bybuyingajacketatasalewherea25%discountwasgiven.Whatwastheoriginalpriceofthejacket?

(A)$14

(B)$16

(C)$18

(D)$20

6.Howmanysquareyardsarethereintheareaofarugthatis15feetlongand12feetwide?

(A)18

(B)20

(C)22

(D)24

7.Solveforx:3x2+15=90.

(A)±75

(B)±60

(C)±25

(D)±5

8.Ascaledrawingofabirdis theactualsize.Ifthedrawingis incheshigh,findin

inchestheheightoftherealbird.

(A)2

(B)4

(C)6

(D)8

9.Findthesumof–8,17,29,–12,–3,and5.

(A)–18

(B)28

(C)29

(D)33

10.Ifamanwalks milein5minutes,whatishisaveragerateofwalkinginmilesperhour?

(A)4

(B)

(C)

(D)

11.Solveforx: .

(A)18

(B)20

(C)22

(D)24

12.Onamap,1inchrepresents500miles.Howmanymilesapartaretwoplacesthatare

inchesapartonthemap?

(A)750

(B)1000

(C)1250

(D)1500

13.(6×103)+(8×102)+(7×10)+3isequalto

(A)6873

(B)60,873

(C)68,730

(D)600,873

14.Asalesmanreceivedasalaryof$150aweekplus5%commissiononhistotalweeklysales.Duringoneweekhistotalsalesamountedto$1800.Howmuchdidheearnthatweek?

(A)$90

(B)$240

(C)$330

(D)$340

15.Ifacaraverages18milestoagallonofgasoline,howmanygallonsofgasolinewillbeusedonatripof369miles?

(A)18

(B)

(C)

(D)22

TestYourself6

Directions:Eachofthefollowingquestionshasfivesuggestedanswers.Decidewhichoneisbest.

1.Aboy’squarterlytestmarkswere67,74,86,and89.Whatwastheaverageofhistestmarks?

(A)75

(B)79

(C)81

(D)84

(E)85

2.Whatfractionisequalto2.5%?

(A)

(B)

(C)

(D)

(E)

3.Tothenearesttenth,findthesquarerootof48.

(A)5.9

(B)6.5

(C)6.9

(D)7.2

(E)7.3

4.Solveforx: .

(A)

(B)2

(C)

(D)3

(E)

5.Simplify:

(A)

(B)

(C)

(D)

(E)

6.Usingascaleof inch=1foot,whatlengthlinewouldbeneededtorepresent23feet?

(A)

(B)

(C)3

(D)

(E)

7.Whichofthesefractionsisequalto ?

(A)

(B)

(C)

(D)

(E)

8.Theformulaforfindingtheareaofatriangleis .Findtheareaofthetriangle

below:

(A)3sq.in.

(B) sq.in.

(C) sq.in.

(D)1sq.in.

(E) sq.in.

9.Solveforx: =27.

(A)272

(B)243

(C)181

(D)81

(E)3

10.Asalespersongetsacommissionof4%onhersales.Ifshewantshercommissiontoamountto$40,shewillhavetosellmerchandisetotaling

(A)$10

(B)$100

(C)$160

(D)$1000

(E)$10,000

11.UsingtheformulaA=πr2,findtheareaofacirclewhosediameteris14feet.

(A)154sq.ft.

(B)256sq.ft.

(C)286sq.ft.

(D)544sq.ft.

(E)615.4sq.ft.

12.UsingtheformulaV=lwh,findthevolumeofarectangularsolidwhosedimensionsarel=46ft.,w=38ft.,h=40ft.

(A)89,176cu.ft.

(B)79,507cu.ft.

(C)75,380cu.ft.

(D)69,920cu.ft.

(E)7538cu.ft.

13.Whatisthestandingofaseventh-gradebaseballteamthatwontengamesandlostfivegames?

(A)0.667

(B)0.500

(C)0.333

(D)0.250

(E)0.200

14.Ifx=10,whichofthefollowingstatementsistrue?

(A)x2<2x

(B)x2=15x–50

(C)3x>x3

(D)x3=x2+2x

(E)x3<x2+x2

15.921writteninexpandedexponentialformis

(A)(9×102)+(2×10)+21

(B)(9×100)+(2×10)+1

(C)(9×l02)+(2×10)+1

(D)(9×103)+(2×102)+1

(E)(9×103)+(2×10)+1

TestYourself7

Directions:Eachofthefollowingquestionshasfivesuggestedanswers.Decidewhichoneisbest.

1.HowmanydegreesareinangleAofthetrianglebelow?

(A)60º

(B)70º

(C)80º

(D)90º

(E)100º

2.Whatisthecostofnineouncesofcheeseat$0.80perpound?

(A)$0.36

(B)$0.45

(C)$0.48

(D)$0.52

(E)$0.55

3.Amanborrowed$3600foronemonthatanannualrateof5%.Howmuchinterestdidheowe?

(A)$5

(B)$7.50

(C)$12.50

(D)$15

(E)$30

4.Onepieceofwireis25ft.8in.longandanotheris18ft.10in.long.Whatisthedifferenceinlength?

(A)6ft.l0in.

(B)6ft.11in.

(C)7ft.2in.

(D)7ft.4in.

(E)7ft.5in.

5.Theratioof24to64is

(A)8:3

(B)24:100

(C)3:8

(D)64:100

(E)8:12

6.Expressalgebraicallytheperimeterofthefigurebelow:

(A)3x

(B)5x

(C)7x

(D)9x

(E)24x

7.Ifthreetimesacertainnumber,increasedby4,isequalto19,whatisthenumber?

(A)10

(B)8

(C)7

(D)6

(E)5

Questions8–10refertothefollowinggraph.

8.Whatwastheapproximatetemperatureat3:30p.m.?

(A)26°

(B)27°

(C)28°

(D)29°

(E)30°

9.Whatwasthepercentofdecreaseintemperaturebetween2and5p.m.?

(A)50%

(B)45%

(C)38%

(D)25%

(E)20%

10.Theratiooftherateoftemperaturedecreasebetween1and3p.m.and4and7p.m.is

(A)2:3

(B)3:4

(C)1:4

(D)1:2

(E)3:5

11.Ifaboxof24candybarsisboughtfor$0.80,andthebarsaresoldfor$0.05each,whatis

thepercentofprofitonthecost?

(A)25%

(B)30%

(C)45%

(D)50%

(E)75%

12.Findthelengthofthediagonalintherectanglebelow:

(A)12"

(B)13"

(C)16"

(D)17"

(E)20"

13.Multiply:(–8)(+6).

(A)–2

(B)–14

(C)–48

(D)+24

(E)+48

14.Ifx>9,then

(A)x2>80

(B)x2–2=47

(C)x2<65

(D)x2–2>90

(E)x2+x<90

15.Divide by .

(A)

(B)1

(C)

(D)

(E)0

TestYourself8

Directions:Eachofthefollowingquestionshasfivesuggestedanswers.Decidewhichoneisbest.

1.Ifthescaleonamapindicatesthat inchesequal500miles,5inchesonthatmap

representapproximately

(A)1800miles.

(B)1700miles.

(C)1300miles.

(D)700miles.

(E)350miles.

2.Change0.03125toacommonfraction.

(A)

(B)

(C)

(D)

(E)

3.Solveforx:2x2–5=93.

(A)±7

(B)±9

(C)±12

(D)±36

(E)±49

4.Findtheareaoftheshadedportionofthefigurebelow:

(A)18.50sq.in.

(B)23.98sq.in.

(C)25.72sq.in.

(D)28.86sq.in.

(E)32.24sq.in.

5.Whatisthesimpleinterestonaloanof$20,000takenforfiveyearsat12%interest?

(A)$240

(B)$1200

(C)$2400

(D)$6000

(E)$12,000

6.Whatisthesumof(103+2)+(3×102)?

(A)13,002

(B)3102

(C)1302

(D)312

(E)132

7.Awomanwhosesalaryis$225perweekhas12%deductedfortaxesand5%deductedforSocialSecurity.Whatishertake-homesalary?

(A)$213.75

(B)$198

(C)$186.75

(D)$112.80

(E)$39.25

8.Findthevolumeofacubewhosesidemeasures5yds.

(A)25cu.yd.

(B)100cu.yd.

(C)125cu.yd.

(D)225cu.yd.

(E)625cu.yd.

9.42÷0.06=

(A)0.07

(B)0.7

(C)7

(D)70

(E)700

10.Solveforx:x–62=–18.

(A)124

(B)80

(C)44

(D)–44

(E)–80

11.Onamathematicstest, %ofaclassreceivedmarksofD, %receivedCs,25%

receivedmarksofB,andtheremainderreceivedAs.WhatpercentoftheclassreceivedamarkofA?

(A)20%

(B)25%

(C)28%

(D)30%

(E) %

12.Thescaleonablueprintis =1'.Aroomwhoseactualdimensionsare28'by14'

wouldbewhatsizeinthedrawing?

(A)14"×7"

(B)

(C)

(D)

(E)

13.(2×104+1)–(103+9)=

(A)18,992

(B)16,540

(C)1892

(D)1654

(E)992

14.Findtheapproximateareaofacirclewhoseradiusis21in.

(A)138sq.in.

(B)795sq.in.

(C)989sq.in.

(D)1385sq.in.

(E)3725sq.in.

15.Ifx=3.5,then

(A)x2<10

(B)4x+6=18

(C)x3–5<4

(D)x2+x2=15.75

(E)x3+x<50

TestYourself9

Directions:Eachofthefollowingquestionshasfivesuggestedanswers.Decidewhichoneisbest.

1.Onemanearns$21,000peryear.Anothermanearns$1675permonth.Howmuchmoredoesthefirstmanearninayearthanthesecondman?

(A)$19

(B)$90

(C)$190

(D)$900

(E)$1900

2.Solveforx:2x2–2=30.

(A)±2

(B)±4

(C)±5

(D)±16

(E)±32

3.Bypurchasinghercoatonsale,Jansaved$25,asavingsof %.Whatwasthe

originalpriceofthecoat?

(A)$120

(B)$100

(C)$90

(D)$75

(E)$60

4.Findthevolumeofacubewhoseedgeis4incheslong.

(A)32cu.in.

(B)48cu.in.

(C)64cu.in.

(D)84cu.in.

(E)96cu.in.

5.Whichofthefollowinghasthesamevalueas0.5%?

(A) %

(B) %

(C) %

(D) %

(E) %

6.Onamap, inchesrepresent25miles.Howmanymilesapartaretwoplacesthatare

inchesapartonthemap?

(A)50miles

(B)55.5miles

(C)60miles

(D)62.5miles

(E)67.5miles

7.Findthesumof–11,–22,60,2,and–36.

(A)–131

(B)–7

(C)0

(D)7

(E)131

8.Acertaindepartmentstorearrivesatitsretailpricesbyaddinga150%mark-uptowholesaleprices.Ifadresscostthestore$30wholesale,whatwillitsretailpricebe?

(A)$75

(B)$70

(C)$65

(D)$60

(E)$45

9.Findthedifferencebetween(34+4)and(43+1).

(A)21

(B)20

(C)18

(D)14

(E)3

10.Solveforx: =7.5.

(A)78

(B)80

(C)84

(D)90

(E)102

11.Awomanboughtgroceriestotaling$22.73andgavetheclerkthreeten-dollarbillsandthreepennies.Howmuchchangedidshereceive?

(A)$8.30

(B)$8.20

(C)$7.30

(D)$7.27

(E)$7.20

12.Findthetotalsurfaceareaforarectangularsolidwiththesedimensions:l=8",w=4",h=8".

(A)64sq.in.

(B)128sq.in.

(C)256sq.in.

(D)320sq.in.

(E)512sq.in.

13.Awomanboughtalampfor$16.75.Shereturneditthenextdayandchosealampthatcost$18.95.Shegavetheclerkafive-dollarbilltopaythedifferenceinprice.Howmuchchangedidsheget?

(A)$2.10

(B)$2.30

(C)$2.40

(D)$2.60

(E)$2.80

14.Aboysold$60worthofmagazinesubscriptions,forwhichhewaspaid$18.Whatrateofcommissionwashepaid?

(A)20%

(B)30%

(C)40%

(D)50%

(E)60%

15.Whichconsecutiveintegersforxandywouldmake true?

(A)100and101

(B)40and50

(C)27and33

(D)14and15

(E)10and11

TestYourself10

Directions:Eachofthefollowingquestionshasfivesuggestedanswers.Decidewhichoneisbest.

1.Lastyeartheenrollmentinakindergartenwas150pupils.Thisyeartheenrollmentis180pupils.Whatwasthepercentoftheincreaseinenrollment?

(A)10%

(B)15%

(C)20%

(D)25%

(E)30%

2.Thedimensionsofaroomare:l=15feet,w=12feet,andh=8feet.Howmanysquarefeetofwallpaperwillberequiredtocoverthewallsofthisroom?

(A)192sq.ft.

(B)216sq.ft.

(C)240sq.ft.

(D)432sq.ft.

(E)648sq.ft.

3.Ifa=8andb=3,findthevalueof4a+3b2.

(A)39

(B)41

(C)59

(D)66

(E)70

4.Modeltrainsarebuiltonascaleof1inch=1foot.Ifaparticularmodeltrainis3feet7incheslong,howmanyyardslongwouldtheactualtrainbe?

(A)14.3

(B)28

(C)36

(D)43.5

(E)57.75

5.Arecipecallsfor1 cupsofsugar.Itisnecessarytomakeeighttimestherecipefora

churchsupper.If2cupsofsugarequal1pound,howmanypoundsofsugarwillbeneededtomaketherecipeforthesupper?

(A)4

(B)6

(C)8

(D)10

(E)12

6.Thetaxrate,indecimalform,foracertaincommunitywas0.029oftheassessedvaluation.Expressthistaxrateindollarsper$1000oftheassessedvaluation.

(A)$25per$1000

(B)$29per$1000

(C)$32per$1000

(D)$34per$1000

(E)$290per$1000

7.Whichofthefollowingisamemberofthesolutionsetofx–y>10?

(A)(8,–3)

(B)(6,–3)

(C)(3,–7)

(D)(–3,7)

(E)(–6,3)

8.Findtheareaofthetrianglepicturedbelow:

(A)72sq.in.

(B)40sq.in.

(C)36sq.in.

(D)32sq.in.

(E)18sq.in.

9.Solveforx: =18.

(A)±72

(B)±36

(C)±9

(D)±8

(E)±6

10.Thedailyalmanacreportforonedayduringthesummerstatedthatthesunroseat6:14a.m.andsetat6:06p.m.Findthenumberofhoursandminutesinthetimebetweentherisingandsettingofthesunonthatday.

(A)11hr.2min.

(B)11hr.48min.

(C)11hr.52min.

(D)12hr.8min.

(E)12hr.48min.

11.Express894inexpandedform,usingexponents.

(A)(8×102)+(9×10)+4

(B)(8×10)+(9×10)+4

(C)(92+8)+4

(D)(8×103)+(9×102)+4

(E)(82+33)+22

12.Thepriceofeggsincreasedfrom50centsto60centsadozen.Whatwasthepercentofincreaseinprice?

(A)10%

(B)15%

(C)20%

(D)25%

(E)30%

13.Tomspent2hoursand30minutesstudyingforthreeclasses.Whatwastheaveragetimeinminutesthathespentinstudyingforeachclass?

(A)30

(B)40

(C)45

(D)50

(E)55

14.Findtheperimeterofaparallelogramwhosedimensionsarel=16inches,w=12inches.

(A)24in.

(B)28in.

(C)38in.

(D)56in.

(E)63in.

15.Approximatethesquarerootof91tothenearesttenth.

(A)9.9

(B)9.5

(C)9.1

(D)8.9

(E)8.5

ANSWERKEYSTestYourself1

1.C2.B3.D

4.A5.C6.A

7.D8.A9.C

10.D11.B12.C

13.A14.C15.B

TestYourself2

1.C2.D3.D

4.B5.B6.B

7.B8.A9.D

10.C11.B12.C

13.B14.B15.D

TestYourself3

1.C2.C3.B

4.B5.C6.D

7.A8.A9.B

10.C11.D12.C

13.B14.C15.C

TestYourself4

1.D2.B3.B

4.A5.B6.D

7.C8.C9.A

10.D11.B12.B

13.A14.C15.B

TestYourself5

1.A2.B3.D

4.B5.D6.B

7.D8.C9.B

10.C11.D12.A

13.A14.B15.B

TestYourself6

1.B2.B3.C

4.C5.D6.B

7.D8.E9.B

10.D11.A12.D

13.A14.B15.C

TestYourself7

1.C2.B3.D

4.A5.C6.D

7.E8.C9.E

10.D11.D12.B

13.C14.A15.B

TestYourself8

1.B2.D3.A

4.C5.E6.C

7.C8.C9.E

10.C11.B12.D

13.A14.D15.E

TestYourself9

1.D2.B3.D

4.C5.A6.D

7.B8.A9.B

10.D11.C12.C

13.E14.B15.D

TestYourself10

1.C2.D3.C

4.A5.B6.B

7.A8.E9.E

10.C11.A12.C

13.D14.D15.B

EXERCISES:QUANTITATIVEABILITY

Directions:Followingeachprobleminthissection,therearefivesuggestedanswers.Workeachprobleminyourheadorinthespaceprovided(therewillbespaceforscratchworkinyourtestbooklet).Thenlookatthefivesuggestedanswersanddecidewhichisbest.

1.Agastankis empty.Whenfull,thetankholds18gallons.Howmanygallonsareinthe

tanknow?

(A)3

(B)6

(C)8

(D)12

(E)18

2.Whichofthefollowingistheleast?

(A)

(B)

(C)

(D)

(E)

3.Ifthesumofxandx+3isgreaterthan20,whichisapossiblevalueforx?

(A)–10

(B)–8

(C)–2

(D)8

(E)10

4.Ifasquarehasaperimeterof88,whatisthelengthofeachside?

(A)4

(B)11

(C)22

(D)44

(E)110

5.IfasetRcontainsfourpositiveintegerswhoseaverageis9,whatisthegreatestnumbersetRcouldcontain?

(A)4

(B)9

(C)24

(D)33

(E)36

6.WhichofthefollowingisNOTamultipleof4?

(A)20

(B)30

(C)36

(D)44

(E)96

Questions7and8refertothefollowingdefinition:Forallrealnumbersm,*m=10m–10.

7.*7=

(A)70

(B)60

(C)17

(D)7

(E)0

8.If*m=120,thenm=

(A)11

(B)12

(C)13

(D)120

(E)130

9.AttheShopHere,anitemthatusuallysellsfor$9isonsalefor$6.Whatapproximatediscountdoesthatrepresent?

(A)10%

(B)25%

(C)33%

(D)50%

(E)66%

10.InLinda’sgolfclub,8ofthe12membersareright-handed.Whatistheratioofleft-handedmemberstoright-handedmembers?

(A)1:2

(B)2:1

(C)2:3

(D)3:4

(E)4:3

11.Thesumoffiveconsecutivepositiveintegersis35.Whatisthesquareofthegreatestoftheseintegers?

(A)5

(B)9

(C)25

(D)81

(E)100

12.22×23×23=

(A)24

(B)64

(C)28

(D)210

(E)218

13.Iftheareaofasquareis100s2,whatisthelengthofonesideofthesquare?

(A)100s2

(B)10s2

(C)100s

(D)10s

(E)10

14.If10bookscostddollars,howmanybookscanbepurchasedfor4dollars?

(A)

(B)40d

(C)

(D)

(E)

15.Ifgisaneveninteger,hisanoddinteger,andjistheproductofgandh,whichofthefollowingmustbetrue?

(A)jisafraction.

(B)jisanoddinteger.

(C)jisdivisibleby2.

(D)jisbetweengandh.

(E)jisgreaterthan0.

16.Ifaclassof6studentshasanaveragegradeof78beforeaseventhstudentjoins,whatmusttheseventhstudentgetasagradeinordertoraisetheclassaverageto80?

(A)80

(B)84

(C)88

(D)92

(E)96

17.If6isafactorofacertainnumber,whatmustbefactorsofthatnumber?

(A)1,2,3,and6

(B)2and3only

(C)6only

(D)2and6only

(E)1,2,and3

18.

x=

(A)8

(B)30

(C)50

(D)65

(E)70

19.Forwhatpriceditemdoes40%offequala$2.00discount?

(A)$5.00

(B)$4.00

(C)$10.00

(D)$80.00

(E)$40.00

20.OnMonday,Gerriate ofanapplepie.OnTuesday,sheate ofwhatwasleftofthe

pie.WhatfractionoftheentirepiedidGerrieatonbothdays?

(A)

(B)

(C)

(D)

(E)

21.Iftheareaofasquareisequaltoitsperimeter,whatisthelengthofonesideofthatsquare?

(A)1

(B)2

(C)4

(D)8

(E)10

ANSWERSANDEXPLANATIONS

1.Thecorrectansweris(D).Ifthetankis empty,itmustbe full. ofthetotalcapacity

of18gallonsis12.

2.Thecorrectansweris(E).Thevalueofchoice(A)is ;thevalueofchoice(B)is ;

thevalueofchoice(C)is or ;thevalueofchoice(D)is or ;andthevalueof

choice(E)is or .Therefore,choice(E)hastheleastvalue.

3.Thecorrectansweris(E).Ifx+(x+3)>20,then2x>17.Sox>8.5.Theonlyanswerthatisappropriateis10.

4.Thecorrectansweris(C).Theperimeterofasquareisfoundbysummingthelengthsofeachside.Becausethelengthsareequalonasquare,youcanmultiplyonesideby4togettheperimeter.Therefore,4s=88,sos=22.

5.Thecorrectansweris(D).Tofindthegreatestvalueofthefour,assumetheremainingthreevaluesaretheleastpossiblepositiveinteger,1.Theaveragethenis

.Solveforx.3+x=36,sox=33.

6.Thecorrectansweris(B).Multiplesof4include:4,8,12,16,20,24,28,32,36,40,44,etc.Comparingthesewiththeanswersprovided,noticethatthenumber30isnotamultipleof4.

7.Thecorrectansweris(B).Substitute7form.*7=10(7)–10=70–10=60.

8.Thecorrectansweris(C).If*m=10m–10and*m=120,then10m–10=120.Solveform:10m=130,m=13.

9.Thecorrectansweris(C).Thetotaldiscountedamountis$3or($9–$6).Theoriginalamount×thediscountedpercent=thetotaldiscountedamount.

$9×discountedpercent=$3.

Thediscountedpercent=

10.Thecorrectansweris(A).Thenumberofleft-handedmembersisequalto12–8,or4.Theratioofleft-handerstoright-handersis4:8,whichsimplifiesto1:2.

11.Thecorrectansweris(D).Letthefiveconsecutiveintegersbe:x,x+1,x+2,x+3,andx+4.

Thenx+x+1+x+2+x+3+x+4=35;5x+10=35;5x=25;x=5.

Sincetheleastofthefiveintegersis5,thegreatestis5+4,or9.92=81.

12.Thecorrectansweris(C).Whenmultiplyinglikevaluesraisedtoapower,addtheexponents.

22×23×23=22+3+3=28

13.Thecorrectansweris(D).Theareaofasquareisequaltothe(lengthoftheside)2,orL2.

14.Thecorrectansweris(D).Setuparatioforthisproblemandsolve:

Letxrepresentthenumberofbookspurchasedwith4dollars.

10×4=d×x(usingcross-multiplication)

15.Thecorrectansweris(C).Sinceintegerscanbebothpositiveandnegative,andtheproductofapositiveandnegativeintegerisalwaysnegative,choice(E)mustbefalse.Lookingfurtherattheanswers,noticethatchoices(B)and(C)areoppositesofoneanother.Therefore,oneofthosemustbetrueandtheotherfalse.Substitutetwonumbersforgandhandseewhichofthetwoistrue.Ifg=–4andh=5,g×h=–4×5=–20.Since–20iseven,choice(C)iscorrect.

16.Thecorrectansweris(D).Thesumofthefirstsixgradesis78×6=468.(Tofindtheaveragegradeof78,dividethesumofthesixgradesby6.)

Theaveragewithsevenstudentsis468+x=80×7.

468+x=560;x=92

17.Thecorrectansweris(A).

Allfactorsof6arefactorsofthenumber.Thefactorsof6are:

1×6

2×3

18.Thecorrectansweris(C).Sincethisisanisoscelestriangle,theanglesoppositethecongruentsidesarealsocongruent.Thesumoftheanglesinatriangleequal180°.So65°+65°+x°=180°andx=50°.

19.Thecorrectansweris(A).

Letpequalthepriceoftheitem.

Price×DiscountRate=DiscountAmount

20.Thecorrectansweris(C).OnMonday, ofthepiewaseaten.OnTuesday,therewas

ofthepieleft.

21.Thecorrectansweris(C).Theperimeterofasquareequals4s.Theareaofasquareequalss2.Settingthemequalwilldeterminethelengthofoneside,s.

s2=4s

s=4or0

Sinceitwouldmakenosenseforthelengthtobe0,thecorrectansweris4.

SUMMINGITUP

•Withmultiple-choicequestions,thecorrectanswerisalwaysonthepageinfrontofyou.Soevenifyou’reestimating,you’llbeabletonarrowdownthechoices.

•Somemultiple-choicequestionsrequirenocalculations—thecorrectanswerisbasedonhowwellyouknowtheconceptsinthequestion.

•Somemultiple-choicequestionsincludegraphs,charts,ortablesforyoutointerpret.•Theanswerchoicesarearrangedinorderbysizefromsmallesttolargestorfromlargesttosmallest.

•Thequestionnumberusuallytellsyouhowhardthequestionwillbesincethequestionsgofromeasytohardasyouworktowardtheend.

•Formultiple-choicemathquestions,circlewhat’sbeingaskedsoyoudon’tpickawronganswerbymistake.Forexample,youwon’tpickananswerthatgivesaperimeterwhenthequestionasksforanarea.

•QuantitativeAbilityisreallyafancywaytosay“math.”

Chapter12

QuantitativeComparisons(ISEEOnly)

OVERVIEW

•Whatarethesestrange-lookingquestions?•Howdoyousolvequantitativecomparisons?•Whatdosmarttest-takersknow?•Testyourselfquizzes•Answerkeys•Summingitup

WHATARETHESESTRANGE-LOOKINGQUESTIONS?Youpickedupthisbook.Youflippedthroughsomepages.Yougottothischapterandthought,“WhattheheckamIsupposedtodowiththeseweirdquestions?Whatdotheywantfrommenow?”Well,quantitativecomparisonsarenotquiteaswackyastheylook.Youcanrecognizequantitativecomparisonquestionseasilybecausetheylookverydifferentfromothermathquestions.Eachonehastwoside-by-sideboxescontainingquantitiesthatyoumustcompare.Thenyouchoosethecorrectanswersfromchoices(A)through(D):

(A)ifthequantityinColumnAisgreater

(B)ifthequantityinColumnBisgreater

(C)ifthequantitiesareequal

(D)iftherelationshipcannotbedeterminedfromtheinformationgiven

Therearesomegoodthingstorememberaboutthese.First,thechoicesarealwaysthesame

(checkoutthesampledirectionshere).Second,youdon’tactuallyhavetosolveaproblem.Thequestionsarereallytestingknowledgeofmathematicalprinciplesratherthanyourcalculatingskills.Third,sometimesthetestingfolksgiveyoualittlehelpandprovideadiagramorotherinformationcenteredabovetheboxeswiththeinformationyouarecomparing.

HOWDOYOUSOLVEQUANTITATIVECOMPARISONS?Yourestimatingandcomparisonskills,aswellasthefollowingfoursteps,willhelpyoucopewiththesequestions.Onceyougetusedtothechoices,thingscanmoveprettyquickly.

FourStepsforQuantitativeComparisons:GettingItRight

Memorizetheanswerchoices.Foreachquestion,comparetheboxedquantities.Considerallpossibilitiesforanyvariables.Chooseyouranswer.

Nowlet’slookatthesefourstepsinmoredetail:Don’tjustlearnthedirections;trytomemorizetheanswerchoices.(Remembertheyarealwaysthesame.)Thenyoucansavetimebecauseyouwon’tneedtorefertothemforeveryquestion.Eventhoughtherearetwoquantitiesineachquestion,dealwithoneatatime.Ifthereisextrainformationabovetheboxes,seehoweachquantityrelatestoit.Thendoanyfiguringyouneedtodo.(Therewon’tbemuch.)Considerallpossibilitiesforanyunknowns.Thinkwhatwouldhappenifspecialnumberssuchas0,negativenumbers,orfractionswereputintoplay.Chooseyouranswer.Youshouldn’thavetodoinvolvedcalculationstogettotheanswer.Ifyou’recalculatingendlessly,you’veprobablymissedthemathematicalprinciplethequestionisaskingabout.

ColumnA ColumnB

Thepriceofapoundofcheeseincreasedfrom$2to$2.50.

Thepercentincreaseinthepriceofcheese 25%

Memorizetheanswerchoices.Thecenteredinformationtellsyouthatcheeseincreasedinpricefrom$2to$2.50per

pound.ColumnAasksforthepercentincrease,whichis .ColumnBrequiresnocalculation,andit’sequaltoColumnA.Therearenovariables,sogoontoStep4.

Sincethetwocolumnsareequal,theanswerischoice(C).Markchoice(C)ontheanswersheet.

ColumnA ColumnB

x2+y2 (x+y)2

Memorizetheanswerchoices.TheexpressioninColumnBis(x+y)2=x2+2xy+y2.ThisisthesameastheexpressioninColumnAwiththeadditionofthemiddleterm2xy.Thetermsxandyarevariablesthatcanbepositiveornegativeorzero.Forexample,ifxwere1andywere2,ColumnAwouldbe12+22=5andColumnBwouldbe(1+2)2=9.Thecorrectanswerwouldthenbechoice(B).Butifxwere–1andywere2,ColumnAwouldbe(–1)2+22=5andColumnBwouldbe(–1+2)2=12=1.Thistimethecorrectanswerwouldbechoice(A).

TIPVariablesstayconstant.Avariablethatappearsinbothcolumnshasthesamemeaningineachcolumn.

Anytimemorethanoneanswercanbetrueforacomparison—asisthecasehere—thentheanswertothatquestionmustbechoice(D),“therelationshipcannotbedeterminedfromtheinformationgiven.”Markchoice(D)ontheanswersheet.

ALERT!Figurescanbedeceiving.Ifafigurecarriesawarningthatitisnotdrawntoscale,don’tdependonestimatingormeasuringtohelpyousolvetheproblem.

Rememberthatmanyquantitativecomparisonscanbesolvedwithoutdoinganycalculatingatall.Inmanycases,youshouldbeabletoarriveatthecorrectanswersimplybyapplyingyourknowledgeofbasicmathrulesandprinciples.Lookattheseexamples:

ColumnA ColumnB

A C

A B

a+b c

1.Iftwosidesofatriangleareunequal,theanglesoppositethemareunequal,andthelargerangleisoppositethelongerside.So,theanswerforthisquestionischoice(B).

2.Becausewecan’treallytellwhetheraorbisgreater,wecan’ttellwhichangleisgreater,sochoice(D)isthecorrectanswer.

3.Thesumofanytwosidesofatrianglemustalwaysbegreaterthanthelengthofthethirdside.Theonlyanswerforthisquestionischoice(A).

WHATDOSMARTTEST-TAKERSKNOW?Quantitativecomparisonsmaylookcomplex,butifyoucomeatthemfromtherightangle(pardonthepun),youcanstreamlinetheansweringprocess.

AComparisonIsForever

Inquantitativecomparisons,choice(A)iscorrectonlyifthequantityisalwaysgreaterthanthatinColumnB.Thereverseistrueofchoice(B);itmustalwaysbegreaterthantheinformationinColumnA.Ifyouchoose(C),itmeansthatthetwoquantitiesarealwaysequal.Theconditionmustholdtrueregardlessofwhatnumberyoupluginforavariable.

TheHighertheNumber,theToughertheChoice

Justliketheothersetsofquestions,quantitativecomparisonsgofromeasytodifficultasyouprogressthroughthesection.

QuantitativeComparisonsAreNotAboutCalculating

Ifyoufindyourselfcalculatingupastormonaquantitativecomparison,you’veprobablymissedtheboat.There’ssuretobeasimpler,shorterwaytosolvetheproblem.Findawaytoreducetheamountofactualmathyouneedtodo.Takealookattheseexamples:

ColumnA ColumnB

31×32×33×34×35 32×33×34×35×36

Youdon’thavetodoanycalculationstogettheanswer.Youwouldbecomparingtheproductoffiveconsecutiveintegers,butnoticethattheintegersinColumnBarelarger.Therefore,theproductofthosenumberswouldbegreaterthantheproductofthoseinColumnA.So,thecorrectansweris(B),andyoudidn’thavetomultiplyathing.

ColumnA ColumnB

TheformulaforthevolumeofarightcircularcylinderisV=πr2h.

Thevolumeofarightcircularcylinderwithr=3andh=6

Thevolumeofarightcircularcylinderwithr=6andh=3

Youmightthinkthatforthisquestionyouabsolutelyhavetodothecompletecalculationstofindthevolumeofeachcylinder.Butyoudon’t!Takealookathowsimplythisproblemcanbesolved.

VolumeA=π(32)(6)=(3.14)(3)(3)(6)

VolumeB=π(62)(3)=(3.14)(6)(6)(3)

Sinceyou’redoingthesameoperationforbothformulas—multiplyingby3.14—thatcancelsout.Sotheproblemthenshiftstotheotherfactors:Whichislarger,(32)(6)or(62)(3)?Atthispointyoushouldbeabletoseethatthesecondoneislarger.Ifyoustillneedtotakeitanotherstep,multiply(32)(6)=(9)(6)=54andthen(62)(3)=(36)(3).Youdon’thavetofinishbecauseyoucanseethat(36)(3)islargerthan54.

IftheMathIsNotDifficult,YouShouldDoIt

ColumnA ColumnB

(0.6)(0.6)

Thecorrectansweris(C).Dothissimplemathandyou’vegotaguaranteedcorrectanswer.Theybothequal0.36,soyourchoiceis(C).

WhentheCenteredInformationHasUnknowns,YouShouldSolvefortheUnknowns

ColumnA ColumnB

3x=124y=20

x y

Thecorrectansweris(B).Youneedtoknowwhateachoftheunknownsis,soyouhavetosolveforboth.

NOTEFormostofthequestionsyoucanestimate,oruseyourknowledgeofbasicmathematicalprinciplestomakeyourcomparisons.

ItPaystoSimplify

ColumnA ColumnB

Thecorrectansweris(B).Thereareacoupleofthingstonoticeherethatwillhelpyousimplifytheproblem.First,bothdenominatorsarereallythesame:462is231×2,sothedenominatorsbecome(231)(2)(8).Next,the(17)’sinbothnumeratorscanceleachotherout.Now,allyouhavetodoisfigureouttheresultsof(8)(45)and(9)(42),compare,andmarkthecorrectanswer.(Itischoice(B),sincethequantityinColumnBisgreater.)

YouCanSimplifybyAddingorSubtractingtheSameValueinEachColumn

ColumnA ColumnB

4x+5 3x+6

Thecorrectansweris(D).Youmightnotseethatrightaway,sowe’llshowyouwhyit’strue.Thefirstthingyoudoissubtract5frombothsides.Theresultis4xand3x+1.Nowsubtract3xfrombothsides;youendupwithxand1.Sinceyoudon’tknowwhatxis,youcan’tknowifitislargerorsmallerthan1.That’swhychoice(D)isthecorrectanswer:therelationshipcannotbedeterminedfromtheinformationgiven.

YouCanSimplifybyMultiplyingorDividingEachSidebytheSamePositiveNumber

ColumnA ColumnB

999–998 998

Youbegintosimplifybydividingbothsidesby998.

91–90 90

1

9–1

8 1

Thecorrectansweris(A).ThisprovesthatthequantityinColumnAislarger,eventhoughyouhaven’tsolvedfortheexactquantity.

YouHavetoConsiderAllthePossibilities

Whenthereareunknownsinthequantitiesbeingcompared,youhavetoremembertoconsiderallpossibilitiesforwhatthoseunknownsmightbe.Forexample,anunknownmightbe1,0,afraction,oranegativenumber.Ineachofthesecases,thenumberhasspecialpropertiesthatwillaffectyourcalculations.Or,unlessotherwisestated,twounknownscouldevenbeequal.

AnUnknownMightBeaZero

Zerohasspecialpropertiesthatcomeintoplaywhenyouplugitinforanunknown.

ColumnA ColumnB

x>0,y>0,z=0

3z(2x+5y) 3x(2z+5y)

Thecorrectansweris(B).Ifz=0,then3z=0andtheproductofColumnAis0.InColumnB,though,2z=0,soitcomesoutoftheexpression.Theproductwillbe(3x)(5y),whichwillbeapositivenumber.

ColumnA ColumnB

x<0,y>0,z=0

3z(2x+5y) 3x(z+5y)

Thecorrectansweris(A).Again,theproductofColumnAis0,because3zstillequals0.ThechangecomesinColumnB.Becausexislessthan0,3xwillbenegativeand5ywillbepositive,sotheproductwillbeanegativenumber.

AnUnknownMightBeaNegativeNumber

ColumnA ColumnB

3x=4y

x y

Thecorrectansweris(D).Don’tthinkthatchoice(A)isthecorrectanswer,eventhoughifxandyarepositive,xisgreaterthany.Whatifxandyarenegative,asin3(–4)=4(–3);thenyisgreaterthanx.Andifxandyarebothzero,bothcolumnsareequal.Sinceyouhavenowayofknowingwhatthevaluesare,thecorrectansweris(D).

AnUnknownMightBeaFraction

ColumnA ColumnB

x>0andx≠1

x2 x

Thecorrectansweris(D).Ifxislargerthan1,thenx2islargerthanx.Butifxisbetween0and1—afraction—thenx2issmallerthanx.

TIPNeedaneasywaytorememberthedirectionsforquantitativecomparisons?Justkeepthiskeyinmind:

(A)=Aisbigger

(B)=Bisbigger

(C)=Columnsareequal

(D)=Determinationimpossible

FractionsCanPlayTricks

Rememberthataproperfractionraisedtoapositivepowerissmallerthanthefractionitself.

ColumnA ColumnB

Thecorrectansweris(A).Ifyoukeepthemathprincipleinmind,youdon’tevenhavetothinkaboutdoingthesecalculations.Sinceeachsuccessivemultiplicationwouldresultinasmallerfraction,ColumnAwillalwaysbelargerthanColumnB,soyouranswerischoice(A).

InQuantitativeComparisons,FiguresAreNotNecessarilyDrawntoScale

ColumnA ColumnB

MinorarcsPQandRShaveequallengthandeachcirclehascenterO.

NOTE:Figurenotdrawntoscale.

DegreemeasureofanglePOQ

DegreemeasureofangleROS

Inthefigure,anglesPOQandROSseemtobeequal,butrememberthewarning.You’retoldthatthefigureisnotdrawntoscale,sodon’tbefooled.

Thecorrectansweris(A).Youcanprovethisbythefigurebelowthatisdrawntoscale.

PlugginginNumbersCanHelp

Ifyou’restuckonacomparisonwithunknowns,trysubstitutingnumbers.Choosethenumbersatrandomandplugthemintotheequations.Dothiswiththreedifferentsubstitutionsandseeifthereisanyconsistentresult.It’snotaguarantee,butit’sdefinitelyworthashot.

StrategicGuessingCanRaiseYourScore

Whenallelsefails,callupyourguessingskills.Here’showyoucantipthescalesinyourfavor,evenifit’sonlyalittlebit:

•Ifacomparisoninvolvesonlynumberswithoutanyunknowns,chancesarethatyou’llbeabletofigureoutthequantitiesandmakeacomparison.Sointhissituation,don’tguesschoice(D).

•Ifthecomparisondoescontainanunknownorafigure,asalastresortguesschoice(D).

TESTYOURSELFQUIZZESTakethefollowingquizzestohelpyoudeterminewhatyourweaknessesmightbe.Clickherefortheanswers.

TestYourself1

Directions:Foreachofthefollowingquestions,twoquantitiesaregiven—oneinColumnA,theotherinColumnB.Comparethetwoquantitiesandchoosethebestanswerfromthechoicesbelow:

(A)ifthequantityinColumnAisgreater

(B)ifthequantityinColumnBisgreater

(C)ifthequantitiesareequal

(D)iftherelationshipcannotbedeterminedfromtheinformationgiven

ColumnA ColumnB

1. Theaverageof18,20,22,24,26 Theaverageof19,21,23,25

2. 8+14(8–6) 14+8(8–6)

3. 6%of30 Thenumber30is6%of

4.

5. 22

6. (8–6)(2+7)

ColumnA ColumnB

Theratioofgirlstoboysinamathclassis3:1.

7. Ratioofboystotheentireclass

Asportjacketpriced$48aftera20%discount.

8. Originalpriceofthesportjacket $60

9. 117 171

10.Priceofpackageofmeatweighing1.8lbs.(unitprice

92.6¢perlb.)

Priceofpackageofmeatweighing2.3lbs.(unitprice

67.5¢perlb.)

x–y=–6x+y=–2

11. x y

(x–6)(x+4)=0

12. Thesmallestrootoftheequation

Thenegativeofthegreatestrootoftheequation

ColumnA ColumnB

13. x y

x=–1

14. 3x2–2x+4 2x3+x2+4

6>y>–2

15.

TestYourself2

Directions:Foreachofthefollowingquestions,twoquantitiesaregiven—oneinColumnA,theotherinColumnB.Comparethetwoquantitiesandchoosethebestanswerfromthechoicesbelow:

(A)ifthequantityinColumnAisgreater

(B)ifthequantityinColumnBisgreater

(C)ifthequantitiesareequal

(D)iftherelationshipcannotbedeterminedfromtheinformationgiven

ColumnA ColumnB

x<0y<0

1. x+y x–y

t<0

2. t3 t2

ColumnA ColumnB

3. a+b c+d

AB=ACA< B

4. BC AB

5. Sumofthemissingnumbersonthenumberline

Sumofthemissingnumbersonthenumberline

6. $0.41 Sumofonequarter,twonickels,andthreepennies

7. Thewholenumberinthisgroupoffractions

Thewholenumberinthisgroupoffractions

ColumnA ColumnB

8. Thelargestnumberthatcanbewrittenbyrearrangingthedigitsin263

Thelargestnumberthatcanbewrittenbyrearrangingthedigitsin192

9. Circumferenceofthecircle Perimeterofthesquare

10. 2morethan of63 2lessthan of66

11. Twoeightsplusthree Eighttwosplusthree

12. 4×4×6 6×6×4

13. Theaverageofthenumbers2,7,9 Theaverageofthenumbers3,5,9

ColumnA ColumnB

14. AC BD

15. x y

ANSWERKEYSTestYourself1

1.C2.A3.B

4.B5.C6.A

7.B8.C9.B

10.A11.B12.A

13.D14.A15.D

TestYourself2

1.B2.B3.D

4.B5.B6.A

7.C8.B9.A

10.A11.C12.B

13.A14.B15.A

EXERCISES:QUANTITATIVECOMPARISONS

Directions:Foreachofthefollowingquestions,twoquantitiesaregiven—oneinColumnA,theotherinColumnB.Comparethetwoquantitiesandchoosethebestanswerfromthechoicesbelow:

(A)ifthequantityinColumnAisgreater

(B)ifthequantityinColumnBisgreater

(C)ifthequantitiesareequal

(D)iftherelationshipcannotbedeterminedfromtheinformationgiven

ColumnA ColumnB

a>0x>0

1. a–x a+x

2. Theaverageof18,20,22,24 Theaverageof17,19,21,23

3. 5%of34 Thenumber34is5%of

s=1t=3a=–2

4. [5a(4t)]3 [4a(5s)]2

ColumnA ColumnB

a<b

5. KR KT

4>x>–3

6.

7.

a>bx<a+b

8. a+b y

ColumnA ColumnB

y=anoddinteger

9. Thenumericalvalueofy2 Thenumericalvalueofy3

10. (8+6)÷[3–7(2)] (6+8)÷[2–7(3)]

11. threefourthsof

NC=NYN> C

12. NC CY

13. Agivenchordinagivencircle

Theradiusofthesamecircle

14.

15.

ANSWERSANDEXPLANATIONS

1.Thecorrectansweris(B).Thegiveninformation,a>0andx>0,informsusthatbothaandxarepositivenumbers.Thesumoftwopositivenumbersisalwaysgreaterthantheirdifference.

2.Thecorrectansweris(A).ThenumbersinColumnAarerespectivelylargerthanthenumbersinColumnB;therefore,theiraveragemustbegreater.

3.Thecorrectansweris(B).

4.Thecorrectansweris(B).

Apositiveproductisgreaterthananegativeone.

5.Thecorrectansweris(A).

b>a (given)KR>KT (inatriangle,thegreatersideliesoppositethegreaterangle)

6.Thecorrectansweris(D).Becausexcouldbeanyintegerfrom–3to4,thevaluesofthefractionsareimpossibletodetermine.

7.Thecorrectansweris(A).

8.Thecorrectansweris(C).

y=a+b(anexteriorangleofatriangleisequaltothesumofthetwointeriorremoteangles)

9.Thecorrectansweris(D).Thereisnotenoughinformation,asycouldequal1,whichwouldmakebothquantitiesequal;orycouldbegreaterthan1,whichwouldmakey3greaterthany2.Ifywereanegativeinteger,theny2wouldbegreaterthaty3.

10.Thecorrectansweris(B).

(8+6)÷[3–7(2)]

=14÷–11=

(6+8)÷[2–7(3)]

=14÷–19=

11.Thecorrectansweris(C).

12.Thecorrectansweris(B).

NC=NY (given)C= Y (anglesoppositeequalsidesareequal)N> C (given)N> Y (substitution)CY>NC (thegreatersideliesoppositethegreaterangle)

13.Thecorrectansweris(D).Theradiuscouldbelessthan,equalto,orgreaterthanthechord.

14.Thecorrectansweris(C).

15.Thecorrectansweris(C).

SUMMINGITUP

•Rememberthesestepsasyouworkthroughthequestionsinthissection:memorizetheanswerchoices;foreachquestion,comparetheboxedquantities;considerallpossibilitiesforanyvariables;andchooseyouranswer.

•Thequantitativecomparisonquestionsusuallyrequirelessreadingandcomputationthanthestandardmultiple-choicequestions.

•Youmaynotactuallyhavetosolvetheproblem;youjustneedtodeterminewhichexpression,ifany,isgreater—orifthereisnotenoughinformationtodeterminetherelationship.

•Quantitativecomparisonquestionsgofromeasytohard.

PARTVIIWRITINGSAMPLEREVIEW

CHAPTER13:WritingMechanics(SSATandISEE)

CHAPTER14:TheEssay(SSATandISEE)

Chapter13

WritingMechanics(SSATandISEE)

OVERVIEW

•Whatarewritingmechanics?•Whataretherulesofspelling?•Spellingdemons•Whataretherulesofpunctuation?•Whataretherulesforcapitalization?•Whataretherulesofgrammar?•WhatiscorrectEnglishusage?•>HowcanIimprovemywriting?•Summingitup

WHATAREWRITINGMECHANICS?Themechanicsofwritingarespelling,capitalization,punctuation,grammar,andusage.NeithertheSSATnortheISEEincludestestquestionsthatdirectlymeasureanyofthesetopics,althoughsomeoftheCatholichighschoolentranceexamsandotherlesscommonlyusedprivatesecondaryschooladmissionstestsdo.

Youwon’thavetotakeatestofyourknowledgeofwritingmechanics,butyouwillhavetosubmitawritingsampleintheformofanessayonanassignedtopic.Theessaytestsfarmorethanmechanics.Itshowsyourabilitytoorganizeandconveyyourthoughts.Youressaywillnotbescored.Itwillbephotocopiedandsenttotheschoolstowhichyouapply.Theessaywillgivetheschoolanoverallimpressionofyourmaturityandpowerofself-expression.

Clearthinkingandagoodvocabularyareimportantaspectsofawell-writtenessay.Correctspelling,capitalization,punctuation,grammar,andEnglishusagealsodomuchforthequalityofanessay.Thischaptergivesyousomeofthemostimportantrulestohelpyouthroughthemechanicsofyouressaywritingandsomepracticalexercisestohelpyouputtousetheinformationyouarelearning.

WHATARETHERULESOFSPELLING?Rule1

Ifaone-syllablewordendswithashortvowelandoneconsonant,•DOUBLETHEFINALCONSONANTbeforeaddingasuffixthatbeginswithavowel.•DONOTDOUBLETHEFINALCONSONANTbeforeaddingasuffixthatbeginswithaconsonantorifthewordhastwovowelsbeforetheconsonantorendsintwoconsonants.

DoubletheFinalConsonant

-er -er,-est -y -en -ing -ed

blotter biggest baggy bidden budding noddedchopper dimmer blurry bitten clipping plottedclipper fattest funny fatten dropping rubbedfitter flatter furry flatten fanning scarredhopper gladdest muddy gladden fretting skippedplotter grimmer sloppy hidden grinning stabbedquitter hottest starry madden gripping steppedshipper madder stubby sadden hopping stoppedshopper reddest sunny trodden quitting tanned

DoNOTDoubletheFinalConsonant

-ing,-ed,-er -ly -ness -ful -y

acting badly baseness baleful dirtyburned dimly bigness boastful duskyclimber gladly coldness doleful fishycoasted madly dimness fitful frostycooked manly fatness fretful leafyfarmer nearly grimness masterful mistyfeared sadly redness sinful rainyfeasting thinly sadness soapyrailed trimly wetness weedy

Rule2

Ifawordofmorethanonesyllableendswithashortvowelandoneconsonant,•DOUBLETHEFINALCONSONANTbeforeaddingasuffixthatbeginswithavowelif

theaccentisonthelastsyllable.•DONOTDOUBLETHEFINALCONSONANTiftheaccentisnotonthelastsyllableorifthesuffixbeginswithaconsonant.

DoubletheFinalConsonant

-ing,-ed -ence,-ent -ance -al

befitting abhorrence acquittance acquittalbefogged concurrent admittance transmittalcommitting excellence remittance noncommittalcompelled intermittent transmittancecontrolling occurrenceimpelling recurrentincurredomitting -er -en -ablepermitted beginner forbidden controllablepropelling propeller forgotten forgettableregretted transmitter regrettablesubmitting

DoNOTDoubletheFinalConsonant

EndingINTWOConso-nants

TwovowelsBeforetheConsonant

Accentnotonthefinalsyllable

SuffixBeginswithaconsonant

-ing,-ed -ing,-ed -ing,-ed -ment

consenting concealing benefiting allotmentconverted contained blossomed annulmentdemanding detaining differed commitmentdiverted disdained gathered defermentrequesting refraining limiting equipmentsubsisted remounted profited intermentsupplanting restraining quarreling prefermentsupported retained solicitingtranscending revealing summoned

Rule3

Ifawordendswithasilente,•DROPTHEEbeforeaddingasuffixthatbeginswithavowel.•DONOTDROPTHEEbeforeasuffixthatbeginswithaconsonant.

DroptheSilentE

-ing,-ed -able -ation -ive

achieving believable admiration abusivebalanced debatable continuation appreciativebelieving desirable declaration creative

capsized endurable derivation decorativerelieved excitable exhalation demonstrativerevolving imaginable inclination expensivetelephoned measurable inhalation illustrativetrembled observable quotation intensivetrembling pleasurable repulsiveexchanging

DoNOTDroptheSilentE

-ful -ment -ly -ness

careful achievement accurately completenessdisgraceful amusement affectionately cutenessdistasteful announcement bravely finenessfateful engagement extremely genuinenesshopeful enlargement genuinely lamenessprideful enslavement immediately latenesstasteful entanglement intensely likenessvengeful management intimately ripenesswasteful replacement sincerely wideness

EXCEPTIONSacknowledgment changeable judgment peaceableacreage chargeable manageable pronounceableadvantageous duly noticeable replaceableargument dyeing(color) outrageous serviceableawful

Rule4

Tomakeawordplural,•ADD-EStowordsendingins,x,z,ch,orsh.•ADD-Stoallotherwords.

ADD-S ADD-ES

advantages croutons annexes fizzesangles distances birches hoaxesbeacons effects brushes marshesbriquets rings caresses witnessescandles coaches

Rule5

Ifawordendswithaythathasavowelsound,•CHANGETHEYTOIbeforeaddinganysuffixEXCEPTonethatbeginswiththeletteri.

•DONOTCHANGETHEYifitisprecededbyanothervowel,orifthesuffixbeginswithi.

ChangetheYtoI

-er,-est,-ly,-ness -ous -ance,-ant -able,-ful

craftier ceremonious alliance beautifuldaintiest harmonious appliance fancifulhealthier industrious compliant justifiableheavily injurious defiant mercifulmoldiness luxurious pliant pitiablemoodiest melodious reliance pliablemurkiness mysterioussleepiness studioussteadily victorious

DoNOTChangetheY EXCEPTIONS-ing -ly,-ness -ous

allying dryly beauteousapplying dryness bounteouscomplying shyly duteousdefying shyness miscellaneousfortifying slyly piteousjustifying slyness plenteousmultiplying sprylypitying wrylysupplying

Rule6

Putibeforee,

Exceptafterc,

Orwhensoundedlikea,

Asinneighbororweigh.

IBEFOREEEXCEPTAFTERC

ORSOUNDSLIKEA EXCEPTIONS

achieve ceiling deign ancientbelieve conceit eight consciencefiend conceive freight deficientfierce deceit inveigh efficientgrief deceive neighbor foreignrelieve perceive reign glacierreprieve receipt skein heiferretrieve receive vein leisure

sieve weigh proficientweird

Rule7

Thesuffix-fulneverhastwol’s.When-fulisaddedtoaword,thespellingofthebaseworddoesnotchange.

Examples

careful disdainful distastefulforceful grateful hopefulmasterful powerful sorrowful

Rule8

Whenthesuffix-lyisaddedtoaword,thespellingofthebaseworddoesnotchange.

Examples

coyly quickly frankly swiftly forcefully

EXCEPTIONS

When-lyisaddedtoawordendingwith-le,theeisreplacedbyay.forcibly despicably illegiblyindelibly probably suitablyWhenthebasewordendswithayfollowingaconsonant,theyischangedtoibefore-ly.busily daintily heavilyluckily merrily sleepily

Rule9

Whenasyllableendsinalongvowelsound,thatsoundismadebythevowelalone:OPENSYLLABLE.

Alongvowelsoundoccurringinaone-syllableword,orinasyllablethatendswithaconsonant,isusuallyspelledbyavowelteam:CLOSEDSYLLABLE.

OPENSYLLABLE CLOSEDSYLLABLE

recent sublimepremium infantilesequence crayonstationary attainmentfatality cavalcade

OPENSYLLABLE CLOSEDSYLLABLE

abrasion genteelmotivate intercedecustodian sincerecomponent ridicule

proprietor vestibulemicrobe clapboardcyclone disclosecucumber telescopehumane growth

SPELLINGDEMONSaberration essential patientabscess exaggerate peculiarabundance exceed picnicaccumulation exhortation pneumoniaacquaint existence possessionadjunct fascinated preciousaggravate feudal presumptuousalleged financier publicityamendment harassment punctiliousancient hearth regrettableanecdote heritage rehearsalannoyance hindrance relevantapparatus imminent repetitiousarraignment impartiality resilienceascertain incongruous rhetoricalassessment indict rhythmbeleaguered inimitable sacrilegiousbureau irreparably scissorscharacter jeopardy separatecolumn journal sophomorecommittal judgment sourcecommittee laboratory sovereigncommunity lacquer specializedconfectionery liquidate specificallycorrelation maneuver staunchcrystallized masquerade subversivecurrency matinee surgeondeferred mechanical symmetricalderogatory medieval temperamentaldesecrated memoir thoroughdilapidated mischievous tomorrowdisappearance negligible transientdissatisfied nickel vacillatedistinguished occasionally vacuumecstasy occur vengeanceembarrass official whethereminent pamphlet whollyemphasis panicky wieldemphatically parliamentary yacht

WHATARETHERULESOFPUNCTUATION?

Apostrophe(’)

Theapostropheisused:•Toindicatepossession:

Bob’shat;Burns’poems

NOTE:Useapostropheonly(withoutthes)forcertainwordsthatendins:a.Whensorzsoundcomesbeforethefinals:

Moses’journey

Cassius’planb.Afterapluralnoun:

girls’shoes

horses’reins

WheretoPlacetheApostrophe

Example:

These(ladie’s,ladies’)blousesareonsale.

Theapostrophemeansbelongingtoeverythingtotheleftoftheapostrophe.

ladie’smeansbelongingtoladie(nosuchword)

ladies’meansbelongingtoladies(correct)

Example:

These(childrens’,children’s)coatsaresize8.

Onecannotsaybelongingtochildrens(childrens’);therefore,children’s(belongingtochildren)iscorrect.

NOTE:a.Whentwoormorenamescompriseonefirm,possessionisindicatedinthelastname:

Lansdale,Jackson,andRoosevelt’slawfirm

SacksandCompany’ssaleb.Inacompoundnoun,separatedbyhyphens,theapostrophebelongsinthelastsyllable—father-in-law’s.

Notethatthepluralsofcompoundnounsareformedbyaddingthes(noapostrophe,ofcourse)tothefirstsyllable:Ihavethreebrothers-in-law.

Theapostrophehastwootherusesbesidesindicatingpossession:

•Forpluralsoflettersandfigures:threed’s;five6’s•Toshowthataletterhasbeenleftout:let’s(forletus)

NOTEA:ours,yours,his,hers,its,theirs,andwhose—allarepossessivebuthavenoapostrophe.

NOTEB:Theapostropheisomittedoccasionallyintitles:TeachersCollege,ActorsEquityAssociation.

Colon(:)

Thecolonisused:•Aftersuchexpressionsas“thefollowing,”“asfollows,”andtheirequivalents:

Thesciencesstudiedinhighschoolsareasfollows:biology,chemistry,andphysics.

Thegraduatesareallattendingcollegesanduniversities,includingthefollowing:Amherst,Harvard,Wesleyan,andYale.

•Afterthesalutationinabusinessletter:Gentlemen:

DearMr.Jones:

NOTE:Acomma(seebelow)isusedafterthesalutationinafriendlyletter:

DearTed,

DearAuntSharon,

Comma(,)

Ingeneral,thecommaisusedinwritingjustasyouuseapauseinspeaking.Herearethespecificsituationsinwhichcommasareused:

•Directaddress:Mr.Adams,hasthereportcomeinyet?

•Apposition:Sam,ourbuyer,gaveussomegoodadvice.

•Parentheticalexpression:Wecouldnot,however,gethimtoagree.

•Complimentaryclosingofaletter:Sincerely,

Trulyyours,•Date,address:

November11,2008

Cleveland,Ohio•Series:

Wehadsoup,salad,icecream,andmilkforlunch.•Phraseorclauseatthebeginningofasentence(iflongerthanfourwords):

AsIlefttheroomtogotoschool,mymothercalledme.•Separatingtwoindependentclausesjoinedbyaconjunction:

WeaskedforMr.Smith,buthehadalreadyleftforhome.•Clarity:

Afterplanting,thefarmerhadhissupper.•Directquotation:

Mr.Arnoldblurtedout,“Thisisafinemess!”•Modifierexpressionsthatdonotrestrictthemeaningofthethoughtthatismodified:

Airtravel,whichmayormaynotbesafe,isanessentialpartofourwayoflife.NOTE:Travelthatisonthegroundissaferthanairtravel.(NOCOMMAS)

EMDash(—)

Thedashisabouttwiceaslongasthehyphen.Thedashisused:•Tobreakupathought:

Therearefive—rememberIsaidfive—goodreasonstorefusetheirdemands.•Insteadofparentheses:

Abeautifulhorse—BlackBeautyisitsname—istheheroofthebook.

ExclamationMark(!)

Theexclamationmarkisusedafteranexpressionofstrongfeeling:

Ouch!Ihurtmythumb.

Hyphen(-)

Thehyphendividesaword:

mother-in-law

NOTE:Whenwrittenout,numbersfromtwenty-onethroughninety-ninearehyphenated.

Parentheses()

Parenthesessetoffthatpartofthesentencethatisnotabsolutelynecessarytothecompletenessofthesentence:

Iwasabouttoremark(thismayberepetition)thatwemustarrivethereearly.

Parenthesesarealsousedtoencloseortosetofffigures,letters,signs,anddates:

Shakespeare(1564–1616)wasagreatdramatist.

Thefourformsofdiscourseare(a)narration,(b)description,(c)exposition,(d)argument.

Period(.)

Theperiodisused:•Afteracompletethoughtunit:

Thesectionmanagerwillreturnshortly.•Afteranabbreviation:

LosAngeles,Calif.;Mr.;Mrs.;Dr.

QuestionMark(?)

Thequestionmarkisusedafterarequestforinformation:

Whendoyouleaveforlunch?

QuotationMarks(“”)

Quotationmarksareused:•Toenclosewhatapersonsaysdirectly:

“Noonecouldtell,”shesaid,“thatitwouldoccur.”

Heexclaimed,“Thisistheend!”

“Don’tleaveyet,”thebosstoldher.•Toencloseatitleofshortwork:

Ihavejustfinishedreadingthepoem“TheRoadNotTaken.”

Semicolon(;)

Thesemicolonisnotusedmuch.Thefollowing,however,arethecommonusesofthesemicolon:

•Toavoidconfusionwithnumbers:Addthefollowing:$1.25;$7.50;and$12.89.

•Beforeexplanatorywordsorabbreviations—namely,e.g.,etc.:

Weareabletosupplyyouwithtwodifferentgaugesofnylonstockings;namely,45and51.

NOTE:Thesemicolongoesbeforetheexpression“namely.”Acommafollowstheexpression.

•Toseparateshortstatementsofcontrast:Warisdestructive;peaceisconstructive.

WHATARETHERULESFORCAPITALIZATION?Capitalize:

•Thefirstwordofasentence:Withcooperation,adepressioncanbeavoided.

•Allpropernames:America,SanteFeChief,GeneralMotors,AbrahamLincoln

•Daysoftheweekandmonths:ThecheckwasmailedonThursday.

NOTE:Theseasonsarenotcapitalized.Example:InFlorida,winterismild.•Theworddearwhenitisthefirstwordinthesalutationofaletter:DearMr.Jones:

but—MydearMr.Jones:•Thefirstwordofthecomplimentarycloseofaletter:Trulyyours,

Verytrulyyours,•Thefirstandallotherimportantwordsinatitle:TheArtofSalesmanship

•Awordusedaspartofapropername:WilliamStreet(but—Thatstreetisnarrow.)

MorningsideTerrace(but—Wehaveaterraceapartment.)•Titles,whentheyrefertoaparticularofficialorfamilymember:

ThereportwasreadbySecretaryMarshall.

(but—MissShaw,oursecretary,isill.)

Let’svisitUncleHarry.

(but—Ihavethreeuncles.)•Pointsofacompass,whentheyrefertoparticularregionsofacountry:

We’regoingtotheSouthnextweek.(but—NewYorkCityissouthofAlbany.)

NOTE:Write:theFarWest,thePacificCoast,theMiddleEast,etc.•Thefirstwordofadirectquotation:

ItwasAlexanderPopewhowrote,“Alittlelearningisadangerousthing.”

NOTE:Whenadirectquotationsentenceisbroken,thefirstwordofthesecondhalfofthesentenceisnotcapitalized.

“Don’tphone,”Lillytoldme,“becausethey’renotinyet.”

WHATARETHERULESOFGRAMMAR?Therulesofgrammargovernthewaysinwhichpartsofspeechareorganizedinasentence.Therearerulesconcerningwordendings,wordorder,andwhichwordsmaybeusedtogether.Youmustknowthepartsofspeechinordertofollowtherulesofgrammar.

PartsofSpeech

ANOUNisthenameofaperson,place,thing,oridea:

teacher city desk democracy

PRONOUNSsubstitutefornouns:

he they ours those

AnADJECTIVEdescribesanoun:

warm quick tall blue

AVERBexpressesactionorastateofbeing:

yell interpret feel are

AnADVERBmodifiesaverb,anadjective,oranotheradverb:

fast slowly friendly well

CONJUNCTIONSjoinwords,sentences,andphrases:

and but or nor

APREPOSITIONshowspositionintimeorspace:

in during after behind

Nouns

Therearedifferentkindsofnouns.

Commonnounsaregeneral:

house girl street city

Propernounsarespecific:

WhiteHouse Jane MainStreet NewYork

Collectivenounsnamegroups:

team crowd organization Congress

Nounshavecases:

Nominative—thesubject,nounofaddress,orpredicatenoun

Objective—thedirectobject,indirectobject,orobjectofthepreposition

Possessive—theformthatshowspossession

Pronouns

Theantecedentofthepronounisthenountowhichapronounrefers.Apronounmustagreewithitsantecedentingender,person,andnumber.

Thereareseveralkindsofpronouns.(Pronounsalsohavecases.)

Demonstrativepronoun:this,that,these,those

Indefinitepronoun:all,any,nobody

Interrogativepronoun:who,which,what

Personalpronoun:

NOMINATIVE OBJECTIVE POSSESSIVE

SINGULAR

1stperson I me my,mine

2ndperson You you your,yours

3rdperson he,she,it him,her,it his,hers

PLURAL

1stperson We us our,ours

2ndperson You you your,yours

3rdperson They them their,theirs

Adjectives

Adjectivesanswerthequestions:

“Whichone?”

“Whatkind?”

“Howmany?”

Therearethreeusesofadjectives:

Anounmodifierisusuallyplaceddirectlybeforethenounitdescribes:

Heisatallman.

Apredicateadjectivefollowsaninactiveverbandmodifiesthesubject:

Sheishappy.Ifeelterrible.

Articleornounmarkerareothernamesfortheseadjectives:

the,a,an.

Adverbs

Adverbsanswerthequestions:

“Why?”

“How?”

“Where?”

“When?”

“Towhatdegree?”

Adverbsshouldnotbeusedtomodifynouns.

TwentyPrinciplesofGrammar

Thesubjectofaverbisinthenominativecaseeveniftheverbisunderstoodandnotexpressed.Thewordwhoisinthenominativecase.Whomisintheobjectivecase.Thewordwhoeverisinthenominativecase.Whomeverisintheobjectivecase.Nounsorpronounsconnectedbyaformoftheverbtobeshouldalwaysbeinthenominativecase.Theobjectofaprepositionorofatransitiveverbshoulduseapronounintheobjectivecase.Itisunacceptabletousethepossessivecaseinrelationtoinanimateobjects.Apronounagreeswithitsantecedentinperson,number,gender,andcase.Anounorpronounlinkedwithagerundshouldbeinthepossessivecase.Each,every,everyone,everybody,anybody,either,neither,noone,nobody,andsimilarwordsaresingularandrequiretheuseofsingularverbsandpronouns.Whenmodifyingthewordskindandsort,thewordsthisandthatalwaysremaininthesingular.Theworddon’tisnotusedwiththird-personsingularpronounsornouns.Averbagreesinnumberwithitssubject.Averbshouldnotbemadetoagreewithanounthatispartofaphrasefollowingthesubject.Thenumberoftheverbisnotaffectedbytheadditiontothesubjectofwordsintroducedbywith,togetherwith,nolessthan,aswellas,andsoon.Singularsubjectsjoinedbythewordsnorandortakeasingularverb.Asubjectconsistingoftwoormorenounsjoinedbythewordandtakesapluralverb.Averbshouldagreeinnumberwiththesubject,notwiththepredicatenoun.Inthereisandthereare,theverbshouldagreeinnumberwiththenoun(s)thatfollow(s)it.Anadjectiveshouldnotbeusedtomodifyaverb.Statementsequallytrueinthepastandinthepresentareusuallyexpressedinthepresenttense.Thewordwereisusedtoexpressaconditioncontrarytofactorawish.

WHATISCORRECTENGLISHUSAGE?CorrectEnglishusagereferstowordchoice.CorrectEnglishusagemeansusingtherightword

withthespecificmeaningintended.ManyEnglishwordsareeasilyconfusedandmisused.Hereisalistofcommonlymisusedwordsandexamplesofhowtousethemcorrectly.

accede—meanstoagreewith.

concede—meanstoyield,butnotnecessarilyinagreement.

exceed—meanstobemorethan.

Weshallaccedetoyourrequestformoreevidence.

Toavoiddelay,weshallconcedethatmoreevidenceisnecessary.

Federalexpendituresnowexceedfederalincome.

access—meansavailability.

excess—meanstoomuch.

Thelawyerwasgivenaccesstothegrandjuryrecords.

Theexpendituresthismontharefarinexcessofincome.

accept—meanstotakewhenoffered.

except—meansexcluding.(preposition)

except—meanstoleaveout.(verb)

Thedraftboardwillacceptallseniorsasvolunteersbeforegraduation.

Alleighteen-year-oldsexceptseniorswillbecalled.

Thedraftboardwillexceptallseniorsuntilaftergraduation.

adapt—meanstoadjustorchange.

adopt—meanstotakeasone’sown.

adept—meansskillful.

Childrencanadapttochangingconditionsveryeasily.

Thewarorphanwasadoptedbythegeneral’sfamily.

Properinstructionmakeschildrenadeptinvariousgames.

NOTE:adaptto,adoptby,adeptinorat.

adaptedto—impliesoriginalornaturalsuitability.

Thegillsofthefishareadaptedtounderwaterbreathing.

adaptedfor—impliescreatedsuitability.

Atomicenergyisconstantlybeingadaptedfornewuses.

adaptedfrom—implieschangedtobemadesuitable.

ManyofRichardWagner’soperalibrettoswereadaptedfromoldNorsesagas.

addition—meanstheactorprocessofadding.

edition—meansaprintingofapublication.

Inadditiontoadictionary,shealwaysusedathesaurus.

ThefirsteditionofShakespeare’splaysappearedin1623.

advantage—meansasuperiorposition.

benefit—meansafavorconferredorearned(asaprofit).

Hehadanadvantageinexperienceoverhisopponent.

Theruleswerechangedforhisbenefit.

NOTE:totakeadvantageof,tohaveanadvantageover.

adverse—(pronouncedAD-verse)meansunfavorable.

averse—(pronounceda-VERSE)meansdisliking.

Hetooktheadversedecisioninpoortaste.

Manystudentsareaversetocriticismbytheirclassmates.

advise—meanstogiveadvice.Adviseislosingfavorasasynonymfornotify.

Acceptable:Theteacherwilladvisethestudentinhabitsofstudy.

Unacceptable:Weareadvisingyouofadeliveryunderseparatecover.(SAY:notifying)

affect—meanstoinfluence.(verb)

effect—meansaninfluence.(noun)

effect—meanstobringabout.(verb)

Youreducationmustaffectyourfuture.

Theeffectofthelastwarisstillbeingfelt.

Adiplomaeffectedatremendouschangeinherattitude.

NOTE:Affectalsohasameaningofpretend.

Shehadanaffectedmanner.

after—isunnecessarywiththepastparticiple.

SAY:Aftercheckingthetimetable,Ileftforthestation.

DON’TSAY:Afterhavingchecked(omitafter)thetimetable,Ileftforthestation.

ain’t—isanunacceptablecontractionforamnot,arenot,orisnot.

aisle—isapassagewaybetweenseats.

isle—isasmallisland.(Bothwordsrhymewithpile.)

allready—meanseverybodyoreverythingready.

already—meanspreviously.

Theywereallreadytowritewhentheteacherarrived.

Theyhadalreadybegunwritingwhentheteacherarrived.

alright—isunacceptable.

allright—isacceptable.

all-round—meansversatileorgeneral.

allaround—meansalloveragivenarea.

RaferJohnson,decathlonchampion,isanall-roundathlete.

Thepolicewerelinedupformilesallaround.

alltogether—meanseverybodyoreverythingtogether.

altogether—meanscompletely.

Theboysandgirlssangalltogether.

Thiswasaltogetherstrangeforapersonofthattype.

allways—meansineverypossibleway.

always—meansatalltimes.

Shewasinallwaysacceptabletothevoters.

Hisreputationhadalwaysbeenspotless.

allude—meanstomakeareferenceto.

elude—meanstoescapefrom.

OnlyincidentallydoesColeridgealludetoShakespeare’spuns.

Itisalmostimpossibleforonetoeludetaxcollectors.

allusion—meansareference.

illusion—meansadeceptionoftheeyeormind.

Thestudentmadeallusionstohisteacher’shabits.

Illusionsofthemind,unlikethoseoftheeye,cannotbecorrectedwithglasses.

alongsideof—meanssidebysidewith.

alongside—meansparalleltotheside.

BillstoodalongsideofBarb.

Parkthecaralongsidethecurb.

alot—isunacceptable.Itshouldalwaysbewrittenastwowords:alot.

among—isusedwithmorethantwopersonsorthings.

NOTE:Amongstshouldbeavoided.

between—isusedwithtwopersonsorthings.

Theinheritancewasequallydividedamongthefourchildren.

Thebusiness,however,wasdividedbetweenthetwodaughters.

amount—appliestoquantitiesthatcannotbecountedonebyone.

number—appliestoquantitiesthatcanbecountedonebyone.

Alargeamountofgrainwasdeliveredtothestorehouse.

Alargenumberofbagsofgrainweredelivered.

annual—meansyearly.

biannual—meanstwiceayear.(Semi-annualmeansthesame.)

biennial—meansonceintwoyearsoreverytwoyears.

anywheres—isunacceptable.

anywhere—isacceptable.

SAY:Wecan’tfinditanywhere.

ALSOSAYnowhere(NOTnowheres),somewhere(NOTsomewheres).

aren’tI—iscolloquial.Itsuseistobediscouraged.

SAY:AmInotentitledtoanexpla-nation?(preferredtoAren’tI…)

as—(usedasaconjunction)isfollowedbyaverb.

like—(usedasapreposition)isNOTfollowedbyaverb.

DoasIdo,notasIsay.

Trynottobehavelikeachild.

Unacceptable:Hewaslikescreamingquiteloudly.

asfaras—expressesdistance.

sofaras—indicatesalimitation.

Wehikedasfarasthenextguesthouse.

Sofarasweknow,thebarnwasadequateforanight’sstay.

asgoodas—shouldbeusedforcomparisonsonly.

Thismotelisasgoodasthenextone.

NOTE:AsgoodasdoesNOTmeanpractically.

Unacceptable:Theyasgoodaspromisedusaplaceinthehall.

Acceptable:Theypracticallypromisedusaplaceinthehall.

asif—iscorrectlyusedintheexpression,“Hetalkedasifhisjawhurthim.”

Unacceptable:“Hetalkedlikehisjawhurthim.”

ascared—nosuchword.Itisunaccept-ableforscared.

Thechildwasscaredofghosts.(NOTascared).

ascent—istheactofrising.

assent—meansapproval.

Theascenttothetopofthemountainwasperilous.

CongressgaveitsassenttothePresident’semergencydirective.

assay—meanstoanalyzeorexamine.

essay—meansashortliterarycomposition.

Thechemistassayedthecontentoftheore.

Thecandidateexpressedherviewsinanessay.

attendto—meanstotakecareof.

tendto—meanstobeinclinedto.

Oneoftheclerkswillattendtothemailinmyabsence.

Inactivepeopletendtogainweight.

back—shouldNOTbeusedwithsuchwordsasreferandreturnsincetheprefixremeansback.

Unacceptable:Referbacktothetext,ifyouhavedifficultyrecallingthefacts.

Bothareacceptableandmaybeusedinterchangeablyasanadverb.

Wetriedtorunbackward.(orbackwards)

Backwardasanadjectivethatmeansslowinlearning.(DON’Tsaybackwardsinthiscase.)

Abackwardpupilshouldbegiveneveryencouragement.

berth—isarestingplace.

birth—meansthebeginningoflife.

Thenewlinerwasgivenawideberthintheharbor.

Hewasafortunatemanfrombirth.

beside—meanscloseto.

besides—referstosomethingthathasbeenadded.

Helivedbesidethestream.

Hefoundwildflowersandweedsbesides.

better—meansrecovering.

well—meanscompletelyrecovered.

Sheisbetternowthanshewasaweekago.Inafewmoreweeks,shewillbewell.

both—meanstwoconsideredtogether.

each—meansoneoftwoormore.

Bothoftheapplicantsqualifiedfortheposition.

Eachapplicantwasgivenagener-ousreference.

NOTE:Avoidusingsuchexpressionsasthefollowing:

Bothgirlshadanewcomputer.(Useeachgirlinstead.)

Bothgirlstriedtooutdotheother.(Useeachgirlinstead.)

Theyarebothalike(OMITboth).

breath—meansanintakeofair.

breathe—meanstodrawairinandgiveitout.

breadth—meanswidth.

Beforeyoudivein,takeaverydeepbreath.Itisdifficulttobreatheunderwater.

Inasquare,thebreadthshouldbeequaltothelength.

bring—meanstocarrytowardthepersonwhoisspeaking.

take—meanstocarryawayfromthespeaker.

Bringthebookshere.

Takeyourraincoatwithyouwhenyougoout.

broke—isthepasttenseofbreak.

broke—isunacceptablefor“withoutmoney.”

Hebrokehisarm.

“Goforbroke”isaslangexpressionwidelyusedingamblingcircles.

bunch—referstothings.

group—referstopeopleorthings.

Thislookslikeadeliciousbunchofbananas.

Whatawell-behavedgroupofchildren!

NOTE:Thecolloquialuseofbunchappliedtopeopleistobediscouraged.

Abunchoftheboyswerewhoopingitup.(Numberispreferable.)

certainly—(andsurely)isanadverb.

sure—isanadjective.

Hewascertainlylearningfast.

Unacceptable:Hesurewaslearningfast.

cite—meanstoquote.

sight—referstovisionorappearance.

site—meansaplaceforabuilding.

HewasfondofcitingfromtheScriptures.

Thesightofthewreckwasappalling.

TheBoardofEducationisseekingasiteforthenewschool.

coarse—meansvulgarorharsh.

course—meansapathorastudy.

Wewereshunnedbecauseofhiscoarsebehavior.

Theshiptookitsusualcourse.

WhichcourseinEnglishareyoutaking?

cometobe—shouldNOTbereplacedwiththeexpressionbecometobe,sincebecomemeanscometobe.

Truefreedomwillcometobewhenalltyrantshavebeenoverthrown.

comic—meansintentionallyfunny.

comical—meansunintentionallyfunny.

Aclownisacomicfigure.

Thepeculiarhatsheworegaveheracomicalappearance.

conscience—meanssenseofright.

conscientious—meansfaithful.

conscious—meansaware.

Hisconsciencepreventedhimfrombecomingcompletelyselfish.

Wealldependonhimbecauseheisconscientious.

Theinjuredwomanwascompletelyconscious.

considerable—isproperlyusedonlyasanadjective,NOTasanoun.

cease—meanstoend.

seize—meanstotakeholdof.

Willyoupleaseceasemakingthosesounds?

Seizehimbythecollarashecomesaroundthecorner.

cent—meansacoin.

scent—meansanodor.

sent—isthepasttenseofsend.

Theone-centpostalcardisathingofthepast.

Thescentofrosesispleasing.

Weweresenttotherearofthebalcony.

calendar—isasystemoftime.

calender—isasmoothingandglazingmachine.

colander—isakindofsieve.

Inthispartoftheworld,mostpeoplepreferthetwelve-monthcalendar.

Inceramicwork,thepottingwheelandthecalenderareindispensable.

Garden-pickedvegetablesshouldbewashedinacolanderbeforecooking.

can—meansphysicallyable.

may—impliespermission.

Icanliftthischairovermyhead.

Youmayleaveafteryoufinishyourwork.

cannothelp—mustbefollowedbyan-ingform.

Wecannothelpfeeling(NOTfeel)distressedaboutthis.

NOTE:cannothelpbutisunacceptable.

can’thardly—isadoublenegative.Itisunacceptable.

SAY:Thechildcanhardlywalkinthoseshoes.

capital—isthecity.

capitol—isthebuilding.

ParisisthecapitalofFrance.

TheCapitolinWashingtonisoccupiedbytheCongress.

(TheWashingtonCapitoliscapitalized.)

NOTE:Capitalalsomeanswealth.

compareto—meanstolikentosome-thingthathasadifferentform.

comparewith—meanstocomparepersonsorthingswitheachotherwhentheyareofthesamekind.

contrastwith—meanstoshowthedifferencebetweentwothings.

Aministerissometimescomparedtoashepherd.

Shakespeare’splaysareoftencomparedwiththoseofMarlowe.

Thewritercontrastedthesensitivityofthedancerwiththegrossnessofthepugilist.

complement—meansacompletingpart.

compliment—isanexpressionofadmiration.

Herwitwasacomplementtoherbeauty.

Hecomplimentedhersenseofhumor.

consul—meansagovernmentrepresentative.

council—meansanassemblythatmeetsfordeliberation.

counsel—meansadvice.

Americansabroadshouldkeepintouchwiththeirconsuls.

TheCityCouncilenactslocallawsandregulations.

Thedefendantheededthecounselofhisfriends.

convenientto—shouldbefollowedbyaperson.

convenientfor—shouldbefollowedbyapurpose.

Willtheseplansbeconvenienttoyou?

Youmustagreethattheyareconvenientfortheoccasion.

copy—isanimitationofanoriginalwork.(NOTnecessarilyanexactimitation)

facsimile—isanexactimitationofanoriginalwork.

Thecounterfeitersmadeacrudecopyofthehundred-dollarbill.

Theofficialgovernmentengraver,however,preparedafacsimileofthebill.

couldof—isunacceptable.(Shouldofisalsounacceptable.)

couldhave—isacceptable.(Shouldhaveisacceptable.)

Acceptable:Youcouldhavedonebetterwithmorecare.

Unacceptable:Icouldofwon.

ALSOAVOID:mustof,wouldof.

decent—meanssuitable.

descent—meansgoingdown.

dissent—meansdisagreement.

Thedecentthingtodoistoadmityourfault.

Thedescentintothecavewastreacherous.

Twooftheninejusticesfiledadissentingopinion.

deduction—meansreasoningfromthegeneral(lawsorprinciples)totheparticular(facts).

induction—meansreasoningfromtheparticular(facts)tothegeneral(lawsorprinciples).

Allhumansaremortal.SinceJohnishuman,heismortal.(deduction)

Thereare10,000orangesinthistruckload.Ihaveexamined100fromvariouspartsoftheloadandfindthemallofthesamequality.Iconcludethatthe10,000orangesareofthisquality.(induction)

delusion—meansawrongideathatwillprobablyinfluenceaction.

illusion—meansawrongideathatwillprobablynotinfluenceaction.

Peoplewereunderthedelusionthattheearthwasflat.

Itisjustanillusionthattheearthisflat.

desert—(pronouncedDEZZ-ert)meansanaridarea.

desert—(pronounceddi-ZERT)meanstoabandon;alsoarewardorpunishment.

dessert—(pronounceddi-ZERT)meansthefinalcourseofameal.

TheSaharaistheworld’smostfamousdesert.

Ahusbandmustnotdeserthiswife.

Executionwasajustdesertforhiscrime.

Wehadplumpuddingfordessert.

differentfrom—isacceptable.

differentthan—isunacceptable.

Acceptable:Jackisdifferentfromhisbrother.

Unacceptable:Florida’sclimateisdifferentthanNewYork’sclimate.

doubtthat—isacceptable.

doubtwhether—isunacceptable.

Acceptable:Idoubtthatyouwillpassthisterm.

Unacceptable:Wedoubtwhetheryouwillsucceed.

dual—meansrelatingtotwo.

duel—meansacontestbetweentwopeople.

Dr.Jekylhadadualpersonality.

AlexanderHamiltonwasfatallyinjuredinaduelwithAaronBurr.

dueto—isunacceptableatthebegin-ningofasentence.Usebecauseof,onaccountof,orsome

similarexpressioninstead.

Unacceptable:Duetotherain,thegamewaspostponed.

Acceptable:Becauseoftherain,thegamewaspostponed.

Acceptable:Thepostponementwasduetotherain.

eachother—referstotwopeople.

oneanother—referstomorethantwopeople.

Thetwogirlshaveknowneachotherformanyyears.

Severalofthegirlshaveknownoneanotherformanyyears.

either...or—isusedwhenreferringtochoices.

neither...nor—isthenegativeform.

EitheryouorIwillwintheelection.

NeitherBillnorBarbisexpectedtohaveachance.

eliminate—meanstogetridof.

illuminate—meanstosupplywithlight.

Letustrytoeliminatetheunneces-sarysteps.

Severallampswereneededtoilluminatethecorridor.

emerge—meanstoriseoutof.

immerge—meanstosinkinto.(ALSOimmerse)

Theswimmeremergedfromthepool.

Thelaundererimmergedthedressinthetubofwater.

emigrate—meanstoleaveone’scoun-tryforanother.

immigrate—meanstoenteranothercountry.

TheNorwegiansemigratedfromNorwayinthemid-1860s.

ManyoftheNorwegianimmigrantssettledintheMiddleWest.

everyone—iswrittenasonewordwhenitisapronoun.

everyone—(twowords)isusedwheneachindividualisstressed.

Everyonepresentvotedfortheproposal.

Everyoneofthevotersacceptedtheproposal.

NOTE:Everybodyiswrittenasoneword.

everywheres—isunacceptable.

everywhere—isacceptable.

Wesearchedeverywhereforthemissingbook.

NOTE:Everyplace(oneword)islikewiseunacceptable.

feelbad—meanstofeelill.

feelbadly—meanstohaveapoorsenseoftouch.

IfeelbadabouttheaccidentIsaw.

Thenumbnessinhislimbscausedhimtofeelbadly.

feelgood—meanstobehappy.

feelwell—meanstobeingoodhealth.

Ifeelverygoodaboutmyrecentpromotion.

Springweatheralwaysmademefeelwell.

flaunt—meanstomakeadisplayof.

flout—meanstoinsult.

HesterPrynneflauntedherscarlet“A.”

Maryfloutedtheauthorityoftheprincipal.

formally—meansinaformalway.

formerly—meansatanearliertime.

Theletterofreferencewasformallywritten.

Maxwasformerlyadelegatetotheconvention.

former—meansthefirstoftwo.

latter—meansthesecondoftwo.

Theformerhalfofthebookwasinprose.

Thelatterhalfofthebookwasinpoetry.

forth—meansforward.

fourth—comesafterthird.

Theywentforthlikewarriorsofold.

TheFourthofJulyisourIndepen-denceDay.

NOTE:spellingofforty(40)andfourteen(14).

get—isaverbthatstrictlymeanstoobtain.

Pleasegetmybag.

TherearemanyslangformsofGETthatshouldbeavoided:

AVOID:Doyougetme?(SAY:Doyouunderstandme?)

AVOID:Youcan’tgetawaywithit.(SAY:Youwon’tavoidpunishmentifyoudoit.)

AVOID:Getwisetoyourself.(SAY:Usecommonsense.)

AVOID:Wedidn’tgettogo.(SAY:Wedidn’tmanagetogo.)

got—meansobtained.

Hegottheticketsyesterday.

AVOID:You’vegottodoit.(SAY:Youhavetodoit.)

AVOID:Wehavegotnosympathyforthem.(SAY:Wehavenosympathyforthem.)

AVOID:Theyhavegotagreatdealofproperty.(SAY:Theyhaveagreatdealofproperty.)

hanged—isusedinreferencetoaperson.

hung—isusedinreferencetoathing.

Theprisonerwashangedatdawn.

Thepicturewashungabovethefireplace.

however—meansnevertheless.

however—meansinwhatpossibleway.

Wearecertain,however,thatyouwilllikethisclass.

Wearecertainthat,howeveryoudecidetostudy,youwillsucceed.

if—introducesacondition.

whether—introducesachoice.

IshallgotoEuropeifIwintheprize.

HeaskedmewhetherIintendedtogotoEurope.(notif)

ifitwas—impliesthatsomethingmighthavebeentrueinthepast.

ifitwere—impliesdoubt,orindicatessomethingthatiscontrarytofact.

Ifyourbookwastherelastnight,itistherenow.

Ifitweresummernow,wewouldallgoswimming.

in—usuallyreferstoastateofbeing.(nomotion)

into—isusedformotionfromoneplacetoanother.

Therecordsareinthatdrawer.

Iputtherecordsintothatdrawer.

NOTE:“Wewerewalkingintheroom”iscorrecteventhoughthereismotion.Themotionisnotfromoneplacetoanother.

irregardless—isunacceptable.

regardless—isacceptable.

Unacceptable:Irregardlessoftheweather,Iamgoingtothegame.

Acceptable:Regardlessofhisability,heisnotlikelytowin.

its—meansbelongingtoit.

it’s—meansitis.

Thehouselostitsroof.

It’sanexposedhouse,now.

areunacceptableforrather.

SAY:Weareratherdisappointedinyou.

last—referstothefinalmemberinaseries.

latest—referstothemostrecentintime.

latter—referstothesecondoftwo.

Thisisthelastbulletin.Therewon’tbeanyotherbulletins.

Thisisthelatestbulletin.Therewillbeotherbulletins.

Ofthetwomostrecentbulletins,thelatterismoreencouraging.

lay—meanstoplace.

lie—meanstorecline.

Notetheformsofeachverb:

Tense Lay(Place)

Present Thechickenislayinganegg.

Past Thechickenlaidanegg.

Pres.Perf. Thechickenhaslaidanegg.

Tense Lie(Recline)

Present Thechildislyingdown.

Past Thechildlaydown.

Pres.Perf. Thechildhaslaindown.

lightening—isthepresentparticipleoftolighten.

lightning—meanstheflashesoflightaccompaniedbythunder.

Leavingtheextrafoodbehindresultedinlighteningthepack.

Summerthunderstormsproducestartlinglightningbolts.

many—referstoanumber.

much—referstoaquantityinbulk.

Howmanyinchesofrainfelllastnight?

Idon’tknow,butIwouldsaymuchrainfelllastnight.

may—isusedinthepresenttense.

might—isusedinthepasttense.

Wearehopingthattheymaycometoday.

Hemighthavedoneitifyouhadencouragedhim.

it’sI—isalwaysacceptable.

it’sme—isacceptableonlyincolloquialspeechorwriting.

alwaysunacceptable:

alwaysacceptable:

noplace—asasolidword,isunaccept-ablefornoplaceornowhere.

Acceptable:Younowhavenowheretogo.

number—issingularwhenthetotalisintended.

Thenumber(ofpagesinthebook)is500.

number—ispluralwhentheindividualunitsarereferredto.

Anumberofpages(inthebook)wereprintedinitalictype.

ofany—(andofanyone)isunacceptableforofall.

SAY:Herswasthehighestmarkofall.

(NOTofanyorofanyone)

offof—isunacceptable.

SAY:Hetookthebookoffthetable.

outloud—isunacceptableforaloud.

SAY:Janereadaloudtoherfamilyeveryevening.

outdoor—(andout-of-door)isanadjective.

outdoors—isanadverb.

Wespentmostofthesummeratanoutdoormusiccamp.

Mostofthetimeweplayedstringquartetsoutdoors.

NOTE:Out-of-doorsisacceptableineithercase.

people—compriseaunitedorcollectivegroupofindividuals.

persons—areindividualsthatareseparateandunrelated.

ThepeopleofNewYorkCityhaveenthusiasticallyaccepted“Shakespeare-in-the-Park”productions.

Onlyfivepersonsremainedinthetheaterafterthefirstact.

persecute—meanstomakelifemiser-ableforsomeone.(Persecutionisillegal.)

prosecute—meanstoconductacrimi-nalinvestigation.(Prosecutionislegal.)

Somepeopleinsistuponpersecutingotherethnicgroups.

TheDistrictAttorneyisprosecutingtheracketeers.

precede—meanstocomebefore.

proceed—meanstogoahead.(Procedureisthenoun.)

supersede—meanstoreplace.

Whatwerethecircumstancesthatprecededtheattack?

Wecanthenproceedwithourplanforresistingasecondattack.

ItisthenpossiblethatPlanBwillsupersedePlanA.

principal—meanschieformain(asanadjective);aleader(asanoun).

principle—meansafundamentaltruthorbelief.

Hisprincipalsupporterscamefromamongthepeasants.

Theprincipaloftheschoolaskedforcooperationfromthestaff.

HumilitywastheguidingprincipleofBuddha’slife.

NOTE:Principalmayalsomeanasumplacedatinterest.

Partofhermonthlypaymentwasappliedasinterestontheprincipal.

sit—meanstakeaseat.(intransitiveverb)

set—meansplace.(transitiveverb)

Notetheformsofeachverb:

Tense Sit(TakeaSeat)

Present Hesitsonachair.

Past Hesatonthechair.

Pres.Perf. Hehassatonthechair.

Tense Set(Place)

Present Hesetsthelamponthetable.

Past Hesetthelamponthetable.

Pres.Perf. Hehassetthelamponthetable.

sometime—meansaportionoftime.

sometime—meansatanindefinitetimeinthefuture.

sometimes—meansoccasionally.

I’llneedsometimetomakeadecision.

Letusmeetsometimeafternoon.

Sometimesitisbettertohesitatebeforesigningacontract.

somewheres—isunacceptable.

somewhere—isacceptable.

stationary—meansstandingstill.

stationery—meanswritingmaterials.

Inancienttimespeoplethoughttheearthwasstationary.

Weboughtwritingpaperatthestationerystore.

stayed—meansremained.

stood—meansremaineduprightorerect.

Thearmystayedinthetrenchesforfivedays.

Thesoldiersstoodatattentionforonehour.

sure—forsurelyisunacceptable.

SAY:Yousurely(NOTsure)arenotgoingtowritethat!

takein—isunacceptableinthesenseofdeceiveorattend.

SAY:Weweredeceived(NOTtakenin)byhisoilymanner.

Weshouldliketoattend(NOTtakein)afewplaysduringourvacation.

their—meansbelongingtothem.

there—meansinthatplace.

they’re—meanstheyare.

Wetooktheirbookshomewithus.

Youwillfindyourbooksoverthereonthedesk.

They’regoingtotheballparkwithus.

theirselves—isunacceptableforthemselves.

SAY:Mostchildrenofschoolageareabletocareforthemselvesinmanyways.

thesekind—isunacceptable.

thiskind—isacceptable.

Iamfondofthiskindofapple.

NOTE:Thesekindswouldalsobeacceptable.

through—meaningfinishedorcompletedisunacceptable.

SAY:We’llfinish(NOTbethroughwith)theworkbyfiveo’clock.

tryto—isacceptable.

tryand—isunacceptable.

Trytocome(NOTtryandcome).

NOTE:planongoingisunaccept-able;plantogoisacceptable.

two—isthenumeral2.

to—meansinthedirectionof.

too—meansmorethanoralso.

Therearetwosidestoeverystory.

Threetwos(or2’s)equalsix.

Weshallgotoschool.

Weshallgo,too.

Theweatheristoohotforschool.

Ifsomethingiscontrarytofact(notafact),usewereineveryinstance.

IwishIwereinBermuda.

Unacceptable:Ifhewassensible,hewouldn’tactlikethat.

(SAY:Ifhewere...)

ways—isunacceptableforway.

SAY:Weclimbedalittleway(NOTways)upthehill.

wentandtook—(wentandstole,andsoon)isunacceptable.

SAY:Theystole(NOTwentandstole)ourtools.

when(andwhere)—shouldNOTbeusedtointroduceadefinitionofanoun.

SAY:Atornadoisatwisting,highwindonland(NOTiswhenatwisting,highwindisonland).

Apoolisaplaceforswimming.(NOTiswherepeopleswim)

whereabouts—isunacceptableforwhere.

SAY:Where(NOTwhereabouts)doyoulive?

NOTE:Whereaboutsasanounmeaningaplaceisacceptable.

Doyouknowhiswhereabouts?

whether—shouldNOTbeprecededbyoforasto.

SAY:ThePresidentwillconsiderthequestionwhether(NOTofwhether)itisbetterto

askforordemandhighertaxesnow.

Heinquiredwhether(NOTastowhether)weweregoingornot.

which—isusedincorrectlyinthefollowingexpressions:

Heaskedmetostay,whichIdid.(CORRECT:HeaskedmetostayandIdid.)

Ithasbeenaseverewinter,whichisunfortunate.(CORRECT:Unfortunately,ithasbeenaseverewinter.)

Youdidnotwrite,besideswhichyouhavenottelephoned.(CORRECT:Omitwhich)

while—isunacceptableforandorthough.

SAY:Thelibraryissituatedonthesouthside;(OMITwhile)thelaboratoryisonthenorthside.

Though(NOTwhile)Idisagreewithyou,Ishallnotinterferewithyourrighttoexpressyouropinion.

Though(NOTwhile)Iaminmyofficeeveryday,youdonotattempttoseeme.

Thefollowingisamethod(withoutgoingintogrammarrules)fordeterminingwhento

useWHOorWHOM:

“Tellme(Who,Whom)youthinkshouldrepresentourcompany?”

Step1:Changethe“who,whom”partofthesentencetoitsnaturalorder.

“Youthink(who,whom)shouldrepresentourcompany?”

Step2:SubstituteHEforWHO,HIMforWHOM.

“Youthink(he,him)shouldrepresentourcompany?”

YouwouldsayHEinthiscase.

THEREFORE:“TellmeWHOyouthinkshouldrepresentthecompany?”iscorrect.

Notetheseconstructions:

ItisIwhoamthemostexperienced.

Itishewhois...

ItisheorIwhoam...

ItisIorhewhois...

ItisheandIwhoare...

whose—meansofwhom.

who’s—meanswhois.

Whoseisthenotebook?

Who’sinthenextoffice?

wouldhave—isunacceptableforhad.

SAY:Iwishyouhad(NOTwouldhave)calledearlier.

youall—isunacceptableforyou(plural).

SAY:Wewelcomeyou,thedelegatesfromEthiopia.

Youareallwelcome,delegatesofEthiopia.

HOWCANIIMPROVEMYWRITING?Writtencommunicationstartswiththesentence.Agroupofrelatedsentencesformsaparagraph.Aseriesofconnectedparagraphsbecomesacomposition.

Thefirststepinimprovingyourwritingistoknowwhatmakesagoodsentence.

WhatIsaSentence?

Asentencemusthaveasubjectandanactionwordorverb.Inaddition,asentencemustexpressacompletethought.

ExamplesofSentences: Bobwalks.

Ericswims.

Otherwordscanbeaddedtomakethesesentencesmoredescriptive.

ImprovedSentences: Bobwalksbriskly.

Ericswimsrapidly.

Addingphrasesthattellmoreaboutthesubjectortheverbcanmakethesesametwosentencesevenmoreinteresting.

GoodSentences: Bobwalksbrisklydowntheroad.

Ericswimsrapidlyacrossthepool.

Theadditionofanotherphraseateitherthebeginningortheendofthesesentencesprovidesan

evenclearerpictureofBobandEric.

BetterSentences: Inahurrytogettoschoolontime,Bobwalksbrisklydowntheroad.

Ericswimsrapidlyacrossthepool,attemptingtoovertakehisopponent.

Themoreyoucanpracticewritingclear,descriptivesentences,thebetteryouwillbecomeatwritingthem.

Thesecondstepinimprovingyourwritingistolearnwhatmakesagoodparagraph.

WhatIsaParagraph?

Aparagraphisagroupofsentencesthatdevelopsonemainidea.Usuallythismainideaortopicisslatedinthefirstsentenceoftheparagraph.Therestoftheparagraphcanprovidedetailsaboutthetopicoritcanclarifythetopicbyprovidingspecificexamples.

Therearenorulesfordeterminingthelengthofaparagraph.However,itisagoodideatomakemostparagraphsinacompositionorreportatleastthreesentenceslong.

ExampleofaParagraphDevelopedbyDetails:

Themanopenedthedoorcautiouslyandslippedquietlyintothecrowdedwaitingroom.Hewasdressedinacleanbutwell-wornovercoatandsneakersthathadseenbetterdays.Onhisheadwasablackknittedcap,pulleddowntocoverhisforeheadandears.

Everysentenceinthisparagraphprovidesadditionaldetailsaboutthetopic—theman.

ExampleofaParagraphDevelopedbyExamples:

Intramuralsportsareavaluablepartofthehighschoolcurriculum.Asportsprogramprovidesaconstructiveoutletfortheenergythathasbeenstoredupduringtheschoolday.Practicesessionsorgamestakeupthetimethatmightotherwisebespenthangingoutonstreetcornerslookingfortrouble.Tossingabasketballaroundthegymprovidesanacceptablealternativetotossingrocksatstreetlightsorstorewindows.

Eachsentenceinthisparagraphprovidesaspecificexampleofthevalueofintramuralsports.

Anewparagraphindicatesachange.Startanewparagraphtoshowachangein:•Thetime,theplace,ortheactioninastory•Themoodorpointofviewinadescription•Ideasorstepsinanexplanation•Speakersinaconversation

ConnectingParagraphs

Justasyoumustprovideforanorderlyflowofsentenceswithinaparagraph,youmustalsoprovideforalogicaltransitionfromparagraphtoparagraphinanycompositionorreport.

Thethreemostcommonmeansofconnectingparagraphsare:Repetitionofakeywordorphraseintroducedinoneparagraphandexpandeduponinthenextparagraph.Useofpronounsthatrefertoapersonoranideamentionedinthepreviousparagraph.

Examplesofpronouns:he,she,they,this,that,these,those,such,both,allUseoftransitionalwordsandphrasestoillustratetherelationshipofonetopictoanother.

Examplesoftransitionalwordsandphrases:although,asaresult,consequently,forexample,incomparison,incontrast,infact,nevertheless,therefore,thus.

ExampleofParagraphsConnectedbyRepetitionofaKeyWord:

LastsummerourwholefamilypiledintothecaranddrovetoDisneyWorldinFlorida.Althoughwehadheardabouttheamusementparkfromfriendswhohadalreadybeenthere,thiswouldbeourfirstexperienceataDisneypark.Wewerealleagertogetthere,butwereallydidnotknowwhattoexpect.

OurfirstdayatDisneyWorldwentbeyondanyexpectationswemighthavehad…

Theseparagraphsareconnectedbytheuseofformsofthesameword.Expectinparagraph1isrepeatedasexpectationsinparagraph2,allowingonethoughttoflowfromthefirstparagraphtothesecond.ThesecondparagraphwillcontinuewithspecificthingsthefamilydidatDisneyWorld.

ExampleofParagraphsConnectedbyTransitionalWords:

Teenagealcoholismisaseriousproblemtoday.Itisaproblemthataffectsyoungpeopleofalltypes,regardlessofethnicbackgroundorsocio-economiclevel.Alcoholismshowsnodiscriminationinchoosingitsvictims.

Althoughtheproblemisfarfrombeingsolved,stepsarebeingtakenbybothfamiliesandschoolstodealwithalcoholismamongteens…

Theseparagraphsareconnectedbytheuseofthetransitionalwordalthoughandtherepetitionofthekeywordproblem.Thesecondparagraphwillcontinuebydetailingsomeofthestepsthatarebeingtakentocombatalcoholism.

EXERCISES:SPELLING

Directions:Eachofthefollowingexercisescontainsagroupoffourwords.Onlyoneofthesewordsisspelledcorrectly.Choosetheletterofthecorrectlyspelledword.Youwillfindthecorrectanswerandareferencetooneofthespellingrulesyouhavejustlearnedintheanswerexplanations.

1.(A)transient

(B)transeint

(C)transent

(D)transint

2.(A)heratage

(B)heritage

(C)heiritage

(D)heretage

3.(A)retreivable

(B)retrievable

(C)retrievible

(D)retreivible

4.(A)foriegn

(B)foreign

(C)foureign

(D)fouriegn

5.(A)witneses

(B)wittnesses

(C)witnesses

(D)wittneses

6.(A)priceing

(B)pricing

(C)priseing

(D)prising

7.(A)intermittent

(B)intermitant

(C)intermittant

(D)intermitent

8.(A)disgracefully

(B)disgracefull

(C)disgracful

(D)disgracefuly

9.(A)complyeing

(B)complieing

(C)complying

(D)compling

10.(A)acheivment

(B)achievment

(C)acheivement

(D)achievement

Directions:Ineachofthefollowingexercisesyoufindagroupofthreewords,pluschoice(D),NOERROR.Youhavetodecidewhetheroneofthethreewordsismisspelled.Theincorrectlyspelledwordisspelledcorrectlyintheanswerexplanation.Youarereferredbacktothespellingrules,ifnecessary.

11.(A)nuisanse

(B)obedience

(C)nonsense

(D)NOERROR

12.(A)confidential

(B)initial

(C)marsial

(D)NOERROR

13.(A)Saturday

(B)Thursday

(C)Wendsday

(D)NOERROR

14.(A)confessing

(B)aroussing

(C)caressing

(D)NOERROR

15.(A)medicine

(B)feminine

(C)paraffin

(D)NOERROR

16.(A)pleasure

(B)measure

(C)liesure

(D)NOERROR

17.(A)libary

(B)contemporary

(C)canary

(D)NOERROR

18.(A)prosperity

(B)university

(C)susceptibility

(D)NOERROR

19.(A)immaterial

(B)immeasurable

(C)implicit

(D)NOERROR

20.(A)ocassionaly

(B)necessarily

(C)recommended

(D)NOERROR

21.(A)feudal

(B)fugitive

(C)muetiny

(D)NOERROR

22.(A)donkies

(B)territories

(C)secretaries

(D)NOERROR

23.(A)squashes

(B)shelfs

(C)lenses

(D)NOERROR

24.(A)blamless

(B)nervous

(C)immensity

(D)NOERROR

25.(A)concurence

(B)remittance

(C)appearance

(D)NOERROR

26.(A)gracefully

(B)intimately

(C)steadyly

(D)NOERROR

27.(A)deficient

(B)wierd

(C)financier

(D)NOERROR

28.(A)forcible

(B)irascible

(C)tyrannical

(D)NOERROR

29.(A)driest

(B)dryly

(C)driness

(D)NOERROR

30.(A)embargos

(B)topazes

(C)sheaves

(D)NOERROR

Directions:Ineachofthefollowingexercisesyoufindagroupoffourwords.Oneofthesefourisspelledincorrectly.Youaretofindtheincorrectlyspelledword.Itisspelledcorrectlyintheanswerexplanation.Youarereferredbacktothespellingrules,ifnecessary.

31.(A)heinous

(B)arrainment

(C)bureau

(D)repetitious

32.(A)corrugated

(B)regrettable

(C)deliberasion

(D)yacht

33.(A)posession

(B)blamable

(C)bookkeeping

(D)whether

34.(A)mediocrity

(B)dilapidated

(C)derogatory

(D)irelevant

35.(A)soverein

(B)mischievous

(C)harassment

(D)masquerade

36.(A)anemia

(B)equilibrium

(C)presumptious

(D)baccalaureate

37.(A)vengance

(B)punctilious

(C)vacillation

(D)resilience

38.(A)beatitude

(B)aggravation

(C)description

(D)beleagered

39.(A)inimitable

(B)iminent

(C)eminent

(D)impartial

40.(A)recognizeable

(B)incongruity

(C)temperamentally

(D)complacency

ANSWERSANDEXPLANATIONS1.Thecorrectansweris(A).SeeRule6andtheDemonslist.

2.Thecorrectansweris(B).SeetheDemonslist.

3.Thecorrectansweris(B).SeeRule6.

4.Thecorrectansweris(B).SeeRule6,Exceptions.

5.Thecorrectansweris(C).SeeRule4.

6.Thecorrectansweris(B).SeeRule3.

7.Thecorrectansweris(A).SeeRule1.

8.Thecorrectansweris(A).SeeRule7.

9.Thecorrectansweris(C).SeeRule5.

10.Thecorrectansweris(D).SeeRules3and6.

11.Thecorrectansweris(A).nuisance

12.Thecorrectansweris(C).martial

13.Thecorrectansweris(C).Wednesday

14.Thecorrectansweris(B).arousing.SeeRule3.

15.Thecorrectansweris(D).Noerror.

16.Thecorrectansweris(C).leisure.SeeRule6.

17.Thecorrectansweris(A).library

18.Thecorrectansweris(D).Noerror.

19.Thecorrectansweris(D).Noerror.

20.Thecorrectansweris(A).occasionally.SeetheDemonsList.

21.Thecorrectansweris(C).mutiny.SeeRule9.

22.Thecorrectansweris(A).donkeys.SeeRule5.

23.Thecorrectansweris(B).Shelvesisthepluralformofthewordshelf.

24.Thecorrectansweris(A).blameless.SeeRule3.

25.Thecorrectansweris(A).concurrence.SeeRule2.

26.Thecorrectansweris(C).steadily.SeeRule8,Exceptions.

27.Thecorrectansweris(B).weird.SeeRule6,Exceptions.

28.Thecorrectansweris(D).Noerror.

29.Thecorrectansweris(C).dryness.SeeRule5.

30.Thecorrectansweris(A).embargoes

31.Thecorrectansweris(B).arraignment.SeetheDemonsList.

32.Thecorrectansweris(C).deliberation

33.Thecorrectansweris(A).possession.SeetheDemonsList.

34.Thecorrectansweris(D).irrelevant

35.Thecorrectansweris(A).sovereign.SeetheDemonsList.

36.Thecorrectansweris(C).presumptuous.SeetheDemonsList.

37.Thecorrectansweris(A).vengeance.SeetheDemonsList.

38.Thecorrectansweris(D).beleaguered.SeetheDemonsList.

39.Thecorrectansweris(B).imminent.SeetheDemonsList.

40.Thecorrectansweris(A).recognizable.SeeRule3.

EXERCISES:PRINCIPLESOFGRAMMAR

Directions:Eachofthesentencesbelowisgrammaticallyincorrect.Rewriteeachsentencecorrectly.TheanswerexplanationsrefertothePrincipleofGrammarthatgovernseachsentence.

1.Theyareasoldasus.

2.Whomdoyousupposepaidusavisit?

3.Punishwhomeverisguilty.

4.Itisme.

5.Canitbethem?

6.ItwouldbeimpossibleforyouandI.

7.Hehaddifficultywiththestore’smanagement.

8.I,who’solder,knowbetterthanyou.

9.IsthereanycriticismofArthurgoing?

10.Everybodytriedtheirhardest.

11.Idonotlikethesesortofcakes.

12.Shedon’tliketoengageinsuchactivity.

13.Theuseofliquorsaredangerous.

14.Thedistrictattorney,aswellasmanyofhisaides,havebeeninvolvedintheinvestigation.

15.Eitherthefifthortheseventhofthecoursestheyhadlaidopenaretobeaccepted.

16.Thefightingandwrestlingofthetwomenisexcellent.

17.Theworstfeatureoftheplayweretheabominableactors.

18.Thereispresentachildandtwodogs.

19.Hespokeslowandcareful.

20.HesaidthatVenuswasaplanet.

21.IwishIwasaclown.

ANSWEREXPLANATIONS1.Theyareasoldaswe(are).SeePrinciple1.

2.Whodoyousupposepaidusavisit?SeePrinciple2.

3.Punishwhoeverisguilty.SeePrinciple3.

4.ItisI.SeePrinciple4.

5.Canitbethey?SeePrinciple4.

6.Itwouldbeimpossibleforyouandme.SeePrinciple5.

7.Hehaddifficultywiththemanagementofthestore.SeePrinciple6.

8.I,whoamolder,knowbetterthanyou.SeePrinciple7.

9.IsthereanycriticismofArthur’sgoing?SeePrinciple8.

10.Everybodytriedhis(orher)hardest.SeePrinciple9.

11.Idonotlikethissortofcake.SeePrinciple10.

12.Shedoesn’tliketoengageinsuchactivity.SeePrinciple11.

13.Theuseofliquorsisdangerous.SeePrinciple12.

14.Thedistrictattorney,aswellasmanyofhisaides,hasbeeninvolvedintheinvestigation.SeePrinciple13.

15.Eitherthefifthortheseventhofthecoursestheyhavelaidopenistobeaccepted.SeePrinciple14.

16.Thefightingandwrestlingofthetwomenareexcellent.SeePrinciple15.

17.Theworstfeatureoftheplaywastheabominableactors.SeePrinciple16.

18.Therearepresentachildandtwodogs.SeePrinciple17.

19.Hespokeslowlyandcarefully.SeePrinciple18.

20.HesaidthatVenusisaplanet.SeePrinciple19.

21.IwishIwereaclown.SeePrinciple20.

SUMMINGITUP

•Themechanicsofwritingarespelling,capitalization,punctuation,grammar,andusage.•TheSSATandtheISEEdonotincludetestquestionsthatdirectlymeasurethemechanicsofwriting,butyouwillhavetosubmitawritingsample.

•Thewritingsampleshowsyourabilitytoorganizeandconveyyourthoughts.•Correctspelling,capitalization,punctuation,grammar,andEnglishusagecontributetothequalityofanessay.

Chapter14

TheEssay(SSATandISEE)

OVERVIEW

•Whatisthepurposeoftheessay?•Howdoyouwriteanessayundertimepressure?•Whatdosmarttest-takersknow?•Summingitup

WHATISTHEPURPOSEOFTHEESSAY?Theessayonyourhighschoolentranceexamservesasawritingsample.Itspurposeistoshowtheschooladmissionscommitteehowwellyouexpressyourselfinwriting.Theschoolisinterestedinhowyouorganizeyourthoughtsandhowyouconveythosethoughtstoareader.Theessayisnotgradedanddoesnotcounttowardyourtestscore.Eachschoolthatreceivesyourtestscorealsoreceivesacopyofyouressay.

TheSSATEssay

TheSSATessayisthefirstpartoftheSSATexam.Youwillbegivenachoiceoftwotopics,onecreativewritingtopic,andonetopicthatasksforyouropinionaboutanissue.Youhave25minutestoreadthetopics,choosethetopicyouwishtowriteabout,organizeyouressay,andwrite.

TheISEEEssay

TheISEEessayisthelastpartoftheISEEexam.Thereisonlyonetopicprovided.Youwillbegiven30minutestoreadandconsiderthetopic,decidewhattosay,organizeyourthoughts,andwritetheessay.

NOTETheessaydoesnotcounttowardyourscoreatall.Infact,theessayisnotevengraded.Theessaydoescounttowardtheimpressionyoumakeontheschool.Admissionsofficersreadyouressaytolearnhowwellyouexpressyourselfinwriting.

HOWDOYOUWRITEANESSAYUNDERTIMEPRESSURE?Towriteacoherent,correctessayin25or30minutes,followthesesixsteps:SixStepsForEssayWriting:GettingItRight

Readthequestiontofindoutexactlywhatitasksyoutodo.Chooseapointofviewordecidehowtoanswerthequestion.Outlineyouressay.Forcreativewritingtopics,usethetopicgivenasthefirstsentenceofyouressay.Thenwriteadescriptivestorywithaclearbeginning,middle,andend.Forallotheressays,youwillprobablywantfourparagraphs:anintroduction,twoparagraphsfortwosupportingideasorillustrations,andaconclusion.Writetheessay.Proofread.Correcterrorsinpunctuation,spelling,grammar,andwordchoice.Ifneeded,makephrasingchangesasneatlyaspossible.

Nowlet’strythesestepsonacoupleofsampletopics:

Topic:Everystudentshouldberequiredtocomplete60hoursofcommunityserviceduringhisorherhighschoolyears.Doyouagreeordisagree?

Thisquestionisaskingyoutochoosesides,thensupportyourposition.Youmustnowdecidewhetheryouwanttowriteinfavororinopposition.Choosethesidethatyouwillfindeasiertodefendwithstrongexamples;whichsideyouchoosedoesnotmatter.Thequestionisnotreallyseekingyouropinion.Forthisexercise,let’sdisagree.Introduction:Communityserviceshouldnotbecompulsory.Point1:Involuntaryactivitiesareneverperformedwell.

A.Beneficiariessufferfromhalf-heartedservice.

B.Studentisresentfulandgainsnosatisfaction.Point2:Teenagersmustlearntoarrangeprioritiesandmanagetheirtimefortheirownbenefit.

A.Somepoorstudentscannotaffordtogiveupsomuchstudytime.

B.Manybeneficialextracurricularactivitiescompeteforprecioustime.

C.Somestudentsmustholdpart-timejobstohelptheirfamilies.Conclusion:Communityserviceshouldbeencouraged,notbecompulsory.Writetheessay.Proofread.Askyourselfthesequestions:•Doeseachparagraphhaveatopicsentence?Isthetopicsentencewelldevelopedwithintheparagraph?

•Ismylanguagecolorfulanddescriptive?HaveIvariedmysentencestructure?

•DoImakeaconvincingargumentformyposition?•Howismyspelling?Ismypunctuationcorrect?Whataboutmygrammar?Refinetheessayifnecessary.Remember:Neatnesscounts.

TIPAnswerthequestionyouarebeingaskedtoaddress.Yougetnocreditforanessaythatdoesnotdowhatisaskedfor.

Topic:Ifyouwereinchargeofplanningyourfamily’snexttwo-weekvacation,wherewouldyougoandwhatwouldyoudo?Why?

Thisquestionisaskingyoutotellaboutaplaceyouwanttoseeoranactivityyouespeciallyenjoyandtoexplainwhythisappealstoyou.Youmustnameaplacetowhichyouwouldliketogoor,ifyourswillbeastay-close-to-homevacation,anactivity,orseriesofactivities.Ifyouhaveanidealvacationinmind,describeit.Youmightalsodescribeavacationyouhavealreadytaken.Thereadersarenotinterestedinthevacationyouchoose;theywanttoknowhowyouwriteaboutit.Forthisexercise,let’schooseasummervacationineasternCanada.Introduction:Canadaisanearbyneighborwithmuchtoofferintermsofcultureandvacationactivities.

Paragraph:Weshouldknowmoreaboutourclosestneighborandtradingpartner.

A.PeoplespeakEnglish;easytolearnaboutlifestylesandideas.

B.Easytogetto,candrive.Paragraph:Canadaoffersscenicbeauty,recreationalactivities,andforeignculture.

A.NovaScotia,PrinceEdwardIsland,andLaurentiansallscenic.

B.Hikingandwatersportsavailable.

C.MontrealandQuebecofferfood,architecture,generalfeelofFrenchcities.Conclusion:SummervacationinCanadawillbeinteresting,fun,andnottooexpensive.

TIPOrganizeyourthoughtsbeforeyoubegin.Yourfirstdraftisyouronlydraft.Thereisnotimetorewrite.

Writetheessay.Proofread.Askyourselfthesequestions:•Doeseachparagraphhaveatopicsentence?Isthetopicsentencewelldevelopedwithintheparagraph?

•Ismylanguagecolorfulanddescriptive?HaveIvariedmysentencestructure?•DoImakeaconvincingargument?Doesmyvacationsoundappealing?•Howismyspelling?Ismypunctuationcorrect?Whataboutmygrammar?

Refinetheessayifnecessary.Remember:Neatnesscounts.Becertainyourwritingislegible.

WHATDOSMARTTEST-TAKERSKNOW?PlanningComesFirst

Allowyourself2to3minutesforplanning.

Donoteventhinkaboutbeginningtowriteuntilyouhavecarefullyreadtheessaytopicandhaveansweredthesequestionsforyourself:

•WhatmustIprove?

•HowmanythingsamIbeingaskedtodo?•HowmanyparagraphswillIneedforthis?

Ifyouareaskedtodiscussadvantagesanddisadvantages,youmustrepresentbothsides.

Ifyouareaskedforyouropinion,youmuststateitclearlyandsupportyourpositionwithgoodreasons.

Ifyouareaskedtosupportyourstatementswithaspecificnumberofexamplesfromyourownexperience,fromhistory,orfromliterature,youmustprovidetherequestednumberofexamplesfromtheappropriatesources.

Ifyouchooseacreativewritingtopic,youmustdevelopastorylinebasedaroundtheideagiveninthetopic.Thetopicmustbethefirstsentenceofyourcreativewritingresponse.

Jotdownyourideasonthetopic.Thinkofwhatyouwanttosay,andsketchoutyourpointsandsupportingstatements.Asgooddescriptivewordsorphrasespopintoyourhead,writethemdownonthesamescratchpaper.Don’tletanythoughtsgetaway.You’llwanttorefertoyourlistofideasasyouwritesothatyoudon’thavetosqueezeideasbetweenthelinesafteryouhavewrittenyouressay.

OutlinesAreImportant

Spend3to5minutesdraftingyouroutline.

Nomatterhowlittletimeyouhave,anoutlinewillsaveyoutimeintheend.Theoutlineisaframeworkforyouressaytohangon.Youroutlineconsistsofspecificdetailsintheorderinwhichyouwouldlikethemtoappearintheessay.Youroutlinewillnotbesenttotheschools,soyoudonotneedtobeconcernedwithcompletesentences,spellingandpunctuation,andlegibilityforanyonebutyourself.

Thefollowingisatypicaloutlineplanforanopinion-typeessay,butyoumayuseanyformatthatworkswellforyou.Includeasmanyparagraphoutlinesasyouneedtocoverthepointsyoumustmakeinyouressay.

I.Introductoryparagraph

A.Topicsentence(rephraseorstateyourpositiononthequestion)

B.Sentencethatintroducessecondparagraph

C.Sentencethatintroducesthirdparagraph

D.Optionalsentenceleadingintosecondparagraph

II.Thefirstpointyouhavetomake

A.Topicsentence

B.Firstideathatsupportsthispoint

1.Detailorillustration—experience,citingofexample

2.Detailorillustration

C.Secondideasupportingfirstpoint

1.Detailorillustration

2.Detailorillustration

III.Thesecondpointyouhavetomake—repeatthesamestructureasinPartII.

IV.Conclusion

Finishyouressaywithasummarystatement.

TopicSentencesAreaMust

Thoughtfultopicsentenceswillkeepyourwritingontargetandhelpyouproveyourpoints.

AWell-TargetedTopicSentenceIsCrucial

Remember:Atopicsentencehasanideathatcanbefullyproveninoneparagraph.Forexample,thesentence“Youcanlearnalotabouthumannaturejustbyobservingpeople”issobroadthatitcannotbeproveninonesingleparagraph.Butifwewrite:

“Youcanlearnalotabouthumannaturebywatchingpeopleatabusstation”

or

“Youcanlearnalotabouthumannaturebywatchingpeopleatthebeach,”

wehaveatopicthatwecanproveinoneparagraph.

Anotherwaytolookattopicsentencesisthroughthecontrollingidea.Thisisakeywordorgroupofwordsthatexpressesthebasicideaofthesentence.Whenthecontrollingideaisclear,theentiresentencewillbespecificandclear.

Example:Anencyclopediaisahandybookforstudents.“Handy”isthecontrollingidea.Intheparagraphthatfollows,youwillexplainhowtheencyclopediaishandy.Example:Travelingbytrainhasseveraladvantagesovertravelingbycar.“Severaladvantages”isthecontrollingidea.Theparagraphwilldetailtheseadvantages.Example:GoodEnglishisclear,appropriate,andvivid.

“Clear,appropriate,andvivid”isthecontrollingidea.Theparagraphwillofferillustrationsofclear,appropriate,andvividEnglish.

DescriptiveWordsMakeWritingInteresting

Toproveyourpointandmakeyourwritinginteresting,youhavetousespecificwordsandphrases.Thisisespeciallyhelpfulforcreativewritingessays.Hereisanexample:

Hisfacewas_____withfright.

colorless scarlet chalky pale

“Chalky”isthebestword,becauseinadditiontocolor—apale,drywhite—itimpliesatexture—dryandlifeless.“Scarlet”isincorrectbecauseyourfacedoesnotbecomescarlet(red)whenyouareafraid.“Colorless”and“pale”aretoovague.“Chalky”isthemostdescriptivewordandtheonethatmakesthissentencemosteffective.

Thesunishighandhot,theairissultry;itis_____time.

siesta sleep nap rest

“Siesta”describesanapthatistakenwhenitisverywarmduringthemiddleofthedayandisthusthemostpreciseword.Thenwouldcome:

nap(ashortsleep)

sleep(atypeofrest)

rest(anysittingdownandrelaxing)

TIPKeepyourhandwritinglegible.Iftheycan’treadit,theywon’tbeabletoassessyourwritingskills.

ProofreadingWillImproveYourEssay

Writingyouressayshouldtakeyouabout15minutes.Allowatleast3minutestoproofreadandmakecorrections.Checkforthefollowing:

•DidIanswerthequestion?•DidIprovidegood,specificdetailstosupportmyideas?•DidIorganizemyanswerinthebestpossiblewaytomakemypointclearly?•DidImakeanyerrorsinspelling,grammar,punctuation,orworduse?

Proofreadingtimeistimewellspent.Checkoverwhatyousaid,beingsuretoreadwhatisreallythereandnotwhatyouthinkisthere.Donotreadtooquicklyoryoumaymissobviouserrors.Ifyoufindthatasentencemightbeimprovedbydifferentphrasingorthatalineisillegible,rewritemoreclearlyatthebottomofthepage,crossouttheoffendingportionneatly,andindicatebyarrowswherethesubstitutionshouldbeinserted.

EXERCISES:ESSAYWRITING1

Directions:Choosetheessaytopicappropriatetoyourexamandwriteanessay.Sampleresponsesbeginonthenextpage.

SSAT-StyleTopic

25Minutes

Topic:Someeducatorssuggestthatallelementary,middleschool,andhighschoolstudentsshouldberequiredtowearschooluniforms.Whatdoyouthink?

ISEE-StyleTopic

30Minutes

Topic:Tellaboutatimewhenyoufeltaverystrongemotion.Whatwasit?Whydidyoufeelit?

EXERCISES:ESSAYWRITING2

Directions:Youarealreadyfamiliarwiththefollowingessaytopics,whattheyrequireofyou,andhowtheymightbeorganized.Choosethetopicappropriatetoyourexamandwriteyourownessay.

SSAT-StyleTopic

25Minutes

Topic:Everystudentshouldberequiredtocomplete60hoursofcommunityserviceduringhisorherhighschoolyears.Doyouagreeordisagree?

ISEE-StyleTopic

30Minutes

Topic:Ifyouwereinchargeofplanningyourfamily’snexttwo-weekvacation,wherewouldyougoandwhatwouldyoudo?Why?

SAMPLERESPONSES1SSAT-StyleTopic

Manypeoplehavesuggestedthatallstudentsberequiredtowearuniformstoschool.Somestudentsobjecttothisideabecauseittakesawaytheirindividuality,butIthinkthereareanumberofreasonswhyschooluniformsmightbeagoodidea.

Overthepastfewyears,therehavebeennodressguidelinesandnodresscodesinourschools.Somestudentsjustnaturallydressneatlyandappropriately,butothersaretrulysloppy.Whenpeopledressinsloppyclothing,theytendtobetoorelaxed.Thisleadstosloppythinking.Prettysoontheyloserespectforschoolandteachersandthewholelearningprocess.Schooluniformswouldremindthesestudentsthattheyareinschoolforapurpose.Ithinkthatifeveryoneweredressedinthesameuniformtherewouldbemoreschoolspirittoo.Studentswouldallfeelasiftheywerepartofsomethingimportant.

Somestudents,especiallygirls,worrytoomuchabouttheirclothesandhowtheylook.Theybothertheirparentstospendtoomuchonclothes,oftenmorethantheirfamiliescanafford,andarealwaystryingtocompete.Ihaveheardaboutboysfightingover“status”clothes.Therehavebeencasesofstealingfancyjacketsandsneakersandevensomeknifings.Schooluniformsmightcostmore,buteachstudentneedsonlytwoorthreeofthem.Thiswouldtakethepressureoffpoorerfamilies.Ifeveryonedressedalike,therewouldbenocompetition.Aspecialbenefitthateducatorsprobablyhaven’tthoughtofisthatofextrasleep.Withnochoiceofwhattowear,itwillbemuchquickertodressandgetoutinthemorning.

Ithinkweshouldtryoutschooluniforms.Iexpectthatdiscipline,payingattention,andschoolspiritwillgoupwhilesquabblesaboutappearanceandfightsoverclothingwillgodown.AndIthinkthatmanystudentswillbehappierwithoutthecompetition,andparentswillbehappiertoo.

ISEE-StyleTopic

OnOctober15thoflastyearmygrandmotherdied.Grandmotherhadbeensickandinpainforsomemonths,soherdeathwasnotasurpriseandinsomewaysitwasareliefandablessing.ButIwasunbelievablysaddenedwhenshedied.Ihadneverfeltthistypeordegreeofsadnessbefore.

Mygrandmotherwasanextraordinarywoman.Herownparentsdiedwhenshewasateenager,andsheraisedtwoyoungersistersalone.Thenherhusbanddiedwhileshewasstillinherthirties,leavingherwithmymother,thenonlysevenyearsold,andmyuncle,thenonlynine.Still,shemaintainedasenseofhumorandgreatdignity.AsIgrewup,mygrandmotherwasalwaysthereforme.Shelistenedtomyjoysandproblemsandalwaysgavemegoodadvice.Shewasneverjudgmentalbut

alwaysgavemeunconditionalloveandmuchwarmth.Iwillmissheralot.

Partofmysadness,Ithink,wassadnessformymotherwhoisnowanorphanherself.Sheandmygrandmotherwerealwaysveryclose.Grandmotherwasthelastfamilymemberofhergeneration.Thereisnowavoidinthefamily,andcelebrationswillalwayshaveanemptyplace.

Ilovedmygrandmotherverymuch,andnowsheisgone.Iwasvery,verysadwhenshedied.Ihaveovercomethatsadnessbynow,butmanylittlethingsremindmeofher.EveryonceinawhilesomethinghappensandIfirstthinkthatImusttellGrandmother—thatshewillbeinterestedoramused.ThenIrememberthatGrandmotherisgone,andIfeelatwingeofsadnessagain.

SAMPLERESPONSES2SSAT-StyleTopic

Theschoolboardhasbeendebatingaboutaproposednewgraduationrequirement.Theyaresuggestingthateverystudentberequiredtocomplete60hoursofcommunityserviceduringthehighschoolyears.Ithinkthatcommunityserviceisanobleconceptbutthatitshouldnotbecompulsory.

Astudentwhoisforcedtoperformanactivityagainsthisorherwillisunlikelytoperformitwell.Theveryfactofthecoercionalmostguaranteesahalf-heartedapproach.Communityservicenotdonewellishardlyserviceatall.Yet,someonethinksthatserviceisbeingperformed.Thentheintendedbeneficiariessufferfromthelackofservice.Communityserviceissupposedtobeennobling.Yetthestudentwhoisforcedtoperformthisserviceisresentfulandgainsnosatisfactionfromit.

Therearemoredownsidestocompulsorycommunityservice.Studentsmustlearntoarrangetheirownprioritiesandtomanagetheirowntime.Somepoorstudentscannotaffordtogiveupsomuchstudytime,yettheyarenotpermittedtodevotethestudytimetheywouldlikebecauseoftherequiredcommunityservice.Otherstudentsmustholdpart-timepaidjobstohelpouttheirfamiliesortohaveanyspendingmoneyforthemselves.Communityservicetotheextentproposedwouldbeahardshipforthem.Stillotherstudentsaresodeeplyintoextracurriculars—sports,music,drama,orreligiousstudies—thatcompulsorycommunityservicewouldcutintostudytimeorsleeptime.

Communityserviceiscertainlyaworthwhilegoal,andallstudentsshouldbeencouragedtoengageinsomeservicethatsuitstheirinterests,abilities,andtimeschedule.However,Idon’tthinkthatcompulsorycommunityserviceisagoodidea.Schooladministratorsandteachersshouldinsteadhelpstudentsdevisecreativeformsofcommunityservicethatthestudentswillwanttofitintotheirbusylives.

ISEE-StyleTopic

Canadaisaforeigncountry,yetitisonlyanautomobiledriveawayfrommanystatesinthenorthernpartoftheUnitedStates.InCanada,therearerivers,lakes,mountains,andseacoastalongwithinterestingcities.ItshouldbeeasytoplanaCanadianvacation.

TheUnitedStatessharesitsborderswithonlytwocountries—MexicotothesouthandCanadatothenorth.Wereallyshouldknowmoreabouttheseneighbors.SinceIliveinNewEngland,Canadaiseasierformyfamilytovisit.WecandrivetoCanada’snortheastinoneday.Bestofall,nearlyallCanadiansspeakEnglishsoitshouldbeeasytogetaroundandeasytolearnaboutlifestylesandideas.

BecauseCanadaissobig,itofferseverykindofvacation.NovaScotiaand

PrinceEdwardIslandarefilledwithquaintfishingvillagesandspectacularseacoastscenery.Theyareprobablycoldforoceanswimming,butgreatforhiking.TheLaurentianMountainsareknownforskiing,buttheirlakesofferallsortsofwatersportsandwemightcampinthemountains.Andtovarythevacation,weshouldvisitthecitiesofMontrealandQuebec,whichretainmuchFrenchinfluence.IamlookingforwardtorealFrenchfood.

Forasummervacationthatwillbefun,nottooexpensive,andeducationalbesides,Canadacan’tbebeat.Itshouldnotbetoohardtoconvincemyfamily.

SUMMINGITUP

Reviewthispagethenightbeforeyoutakeyourexam.Itwillhelpyouwriteanimpressiveessay.

•Theessaydoesnotcounttowardyourscore,butitdoesinfluencetheadmissionscommittee.

•Followthesesteps:readthequestiontofindoutexactlywhatitasksyoutodo;chooseapointofviewordecidehowtorespondtothetopic;outlineyouressay(sketchinandorganizetheideasthatyouwanttoincludeineachparagraph);writetheessay,payingattentiontomechanics;proofread;correcterrorsinpunctuation,spelling,grammar,andwordchoice;andmakechangesandrefinementsasneatlyaspossible.

•Remember:Youressaymustbelegible.

PARTVIIIFOURPRACTICETESTS

PRACTICETEST2:SSAT(UpperLevel)

PRACTICETEST3:SSAT(UpperLevel)

PRACTICETEST4:ISEE(UpperLevel)

PRACTICETEST5:ISEE(UpperLevel)

PracticeTest2:SSAT(UpperLevel)

PARTI:WRITINGSAMPLE25Minutes

Directions:Readthetopics,choosetheonethatinterestsyouthemost,andplanyouressayorstorybeforewriting.Writealegibleessay.

TopicA:Toreducetheaccidentrate,thestatelegislatureshouldpassaproposaltoraisetheminimumdrivingagefrom16to18.

Doyouagreeordisagreewiththisstatement?Supportyourpositionwithexamplesfromyourownexperience,theexperienceofothers,currentevents,oryourreading.

TopicB:Shecouldn'tbelievewhatshewashearing.

Writeastoryusingthisstatementasthefirstsentence.Besureyourstoryhasaclearbeginning,middle,andend.

PARTII:MULTIPLECHOICESection1:Quantitative(Math)

25Questions•30Minutes

Directions:Calculatetheanswertoeachofthefollowingquestions.Selecttheanswerchoicethatisbest.

1.1 +0.750+0.1010=

(A)1.001

(B)2.051

(C)2.055

(D)2.351

(E)2.551

2.Evaluate: .

(A)220

(B)16

(C)8

(D)2

(E)120

3.503.384÷62.3=

(A)7.08

(B)7.68

(C)8.08

(D)9.08

(E)10.08

4.Evaluate: .

(A)1

(B)2

(C)12

(D)13

(E)14

5.2.01÷1.02=

(A)0.507

(B)1.83

(C)1.97

(D)2.0001

(E)3.03

6.–3–[(2–1)–(3+4)]=

(A)12

(B)6

(C)3

(D)–6

(E)–9

7.3003–699=

(A)2294

(B)2304

(C)2314

(D)2404

(E)2414

8.Ifa=5andb= ,whatisthevalueofawhenexpressedintermsofb?

(A)25b

(B)20b

(C)5 b

(D)5b

(E) b

9.140%of70is

(A)0.98

(B)9.8

(C)98

(D)150

(E)9800

10.

(A)2gal.2qt.1pt.

(B)2gal.6qt.2pt.

(C)3gal.3qt.1pt.

(D)4gal.3qt.1pt.

(E)4gal.9qt.1pt.

11.Inthefraction ,ifthevalueofzisdoubledandthevalueofxishalved,thevalueof

thefractionis

(A)multipliedbyfour.

(B)decreasedby .

(C)increasedby .

(D)doubled.

(E)dividedbyfour.

12.20is8%of

(A)1.60

(B)160

(C)200

(D)250

(E)400

13.Howmuchlargerthan80is100?

(A)18%

(B)20%

(C)25%

(D)35%

(E)40%

14.If inchesonascaledrawingisequivalenttoonefootatfullscale,whatdistanceonthe

drawingwillstandfor40inches?

(A) inches

(B) inches

(C)1 inches

(D)2 inches

(E)8 inches

15.6÷ + ×9=

(A)

(B)11

(C)24

(D)54

(E)168

16.Ifx–3<12,xmaybe

(A)lessthan15.

(B)greaterthan16.

(C)equalto15.

(D)lessthan18.

(E)equalto18.

17.Ifa=9,b=2,andc=1,whatisthevalueof ?

(A)16

(B)7

(C)6

(D)4

(E)2

18.Theaverageof–10,6,0,–3,and22is

(A)4

(B)3

(C)2

(D)–3

(E)–6

19.Inthefraction , canbereplacedbyallofthefollowingEXCEPT

(A)+3

(B)+2

(C)0

(D)–1

(E)–2

20.0.10101÷10isequivalentto

(A)0.0010101

(B)0.0100

(C)0.010101

(D)0.1001

(E)1.0101

21.Davidwalkedfromhishometotown,adistanceof5miles,in1hour.Thereturntriptook2hoursbecausehemadeseveralstopsalongtheway.Whatwashisaveragerateofspeed(inmilesperhour)fortheentirewalk?

(A) mph

(B)1 mph

(C)1 mph

(D)3 mph

(E)4mph

22.7isto21as isto

(A)3

(B)2

(C)

(D)1

(E)

23.Ifn= ,then

(A)9>n>8

(B)n=9.5

(C)10>n>9

(D)8<n<9

(E)n2>100

24.

Thesumofwhichpointsonthenumberlineabovewouldbeequaltozero?

(A)B,D,E,I

(B)C,D,G,H

(C)A,C,F,I

(D)D,E,F,G

(E)B,C,H,I

25.Howmanyfourthsaretherein ?

(A)

(B)

(C)1

(D)2

(E)3

Section2:ReadingComprehension

40Questions•40Minutes

Directions:Readeachpassagecarefully.Thendecidewhichofthepossibleresponsesisthebestanswertoeachquestion.

Asrecentlyasthe1860s,mostpeoplebelievedthattheearth,andhumanitywithit,wascreatedamere6,000to7,000yearsago.Forcenturies,beautifullyworkedflintswereregardedastheworkofelves,anotiononcefarmoreplausiblethantheideathathumansroamedtheworld’swildernessesinsmallbandslongbeforethedaysoftheGreekandRomanEmpires.Evenwhenthesestoneswereacceptedasman-madetools,theywereattributedtotheRomansorEarlyBritons.

Today,wethinkinwiderterms,buttheolderideasabouthumanity’sbeginningsfadedslowly.Duringthelateeighteenthandearlynineteenthcenturies,excavators,mainlyenthusiasticamateurs,begantoassociatefossilremainsofmenandextinctanimalswiththestonetools.Still,mostgeologistscontinuedtothinkinBiblicalterms,maintainingthattheseassociationsweremerelycoincidental.TheybelievedtheFloodhadmixedthebonesofancientanimalswiththetoolsandremainsofrecenthumans.Thesetheoriesfinallycrumbledasarchaeologistsbegantofindbonesandtoolstogetherinunflooded,undisturbeddeposits,includinganumberofimportantsitesonthebanksoftheSommesRiver.BritishinvestigatorscametochecktheFrenchdeposits,wereconvincedthatthebonesandtoolshadnotcollectedasaresultofflooding,andannouncedtheirconclusionsin1859.ThiswasthesameyearthatDarwinpublishedOntheOriginofSpecies,thedatethatmarksthebeginningofmodernresearchintohumanevolution.

1.AllofthefollowingtypesofarchaeologicalevidencewerementionedEXCEPT

(A)carbondating.

(B)fossils.

(C)extinctanimalremains.

(D)man-madeobjects.

(E)flint.

2.Theturningpointinscientifictheoriesabouttheageofhumanity’sexistenceonearthwas

(A)thediscoveryinFranceoftheremainsofextinctanimalsandhumanstogetherinanunfloodedarea.

(B)thepublicationofDarwin’sOntheOriginofSpecies.

(C)newtheologicalresearchoftheBible.

(D)newtheoriesabouttheFloodanditseffectsonhumanity.

(E)evidenceleftbytheGreeks,Romans,andearlyBritons.

3.Intheearlynineteenthcentury

(A)elvesmadeflintsincaves.

(B)smallbandsofRomansroamedtheearth.

(C)geologistsdatedhumanity’searlyexistenceto1859.

(D)stoneswereacceptedasancienttoolsandartifactsof20,000-year-oldman.

(E)mostpeoplebelievedthathumanity’sexistencewas6,000to7,000yearsold.

Nextmorning,Isawforthefirsttimeananimalthatisrarelyencounteredfacetoface.Itwasawolverine.Thoughrelativelysmall,rarelyweighingmorethan40pounds,heis,aboveallanimals,theonemosthatedbytheIndiansandtrappers.Heisafinetreeclimberandarelentlessdestroyer.Deer,reindeer,andevenmoosesuccumbtohisattacks.Wesatonarockandwatchedhimcome,abobbingrascalinblackish-brown.Becausethemalewolverineoccupiesaverylargehuntingareaandfightstothedeathanymalethatintrudesonhisdomain,wolverinesarealwaysscarce,andinordertoavoidextinctionneedalltheprotectionthathumanscangive.Asatrapper,Henrywantedmetoshoothim,butIrefused,forthisisthemostfascinatingandlittleknownofallourwonderfulpredators.Hishunchbackgaitwasawkwardandungainly,lopsidedyettireless.

4.Wolverinesareveryscarcebecause

(A)theysufferinthesurvivalofthefittest.

(B)theyareafraidofallhumankind.

(C)theyareseldomprotectedbyman.

(D)trapperstaketheirtollofthem.

(E)theirfoodsupplyislimited.

5.Theauthorofthisselectionismostprobablya(n)

(A)consciousnaturalist.

(B)experiencedhunter.

(C)inexperiencedtrapper.

(D)youngIndian.

(E)farmer.

6.Thewordsuccumbmeans

(A)outmaneuver.

(B)surrender.

(C)overcome.

(D)invite.

(E)repel.

WhenJason,thesonofthedethronedkingofSolcus,wasalittleboy,hewassentawayfromhisparentsandplacedunderthequeerestschoolmasterthateveryouheardof.Thislearnedpersonwasoneofthepeople,orquadrupeds,calledCentaurs.Helivedinacavernandhadthebodyandlegsofawhitehorse,withtheheadandshouldersofaman.HisnamewasChiron;and,inspiteofhisoddappearance,hewasaveryexcellentteacherandhadseveralscholarswhoafterwarddidhimcreditbymakinggreatfiguresintheworld.ThefamousHerculeswasone,andsowasAchilles,andPhiloctetes,likewise,andAesculapius,whoacquiredimmensereputeasadoctor.ThegoodChirontaughthispupilshowtoplayupontheharpandhowtocurediseasesandhowtousetheswordandshield,togetherwithvariousotherbranchesofeducationinwhichtheladsofthosedaysusedtobeinstructed,insteadofwritingandarithmetic.

—fromTheGoldenFleece,byNathanielHawthorne

7.Themainpurposeofthispassageisto

(A)describeJason.

(B)describeChiron.

(C)describeJason’seducation.

(D)explainJason’sfamilyrelationships.

(E)namethescholarstaughtbyChiron.

8.Thewordquadrupedprobablymeansa(n)

(A)creaturewithfourfeet.

(B)creaturewithtwofeet.

(C)strangeschoolmaster.

(D)educatedperson.

(E)scholar.

9.Chiron

(A)taughtwritingandarithmetictohispupils.

(B)acquiredareputationasadoctor.

(C)instructedtheCentaurs.

(D)wasthesonofSolcus.

(E)hadthebodyandlegsofahorseandtheheadandshouldersofaman.

ThekangarooisfoundnowhereintheworldbutinAustralasia.Agesago,whenthatpartofourearthwascutofffromtheAsianmainland,thisfantasticanimalfromnature’slong-agowasalsoisolated.ThereareabouttwodozenspeciesdistributedthroughAustralia,southwardtoTasmaniaandnorthwardtoNewGuineaandneighboringislands.Somearenobiggerthanrabbits;somecanclimbtrees.Theyareknownbyavarietyofpicturesquenames:wallabies,wallaroos,potoroos,boongaries,andpaddymelons.Butthekangaroo—theonethatisAustralia’snationalsymbol—isthegreatgraykangaroooftheplains,admiringlyknownthroughouttheislandcontinentastheOldMan,andalsoasBoomer,Forester,andManoftheWoods.Hissmallermate,inAustraliantalk,iscalledaflyer.TheirbabyisknownasJoey.

Afull-grownkangaroostandstallerthanaman,andcommonlyweighs200pounds.Evenwhenhesitsinhisfavoriteposition,reposingonhishaunchesandtiltingbackontheproppingsupportofhis“thirdleg”—histail—hisheadisfivefeetormoreabovetheground.Hishugehindlegs,withsteel-springpower,cansendhimsailingoveraten-footfencewithease,orinafightcanbeatoffadozendogs.Atwitchofhistailcanbreaksomeone’sleglikeamatchstick.

Kangaroosprovideanendlesssupplyoftalltalestowhichwide-eyedvisitorsaretreatedinthelandDownUnder.Thebeautyofthetalltalesaboutthekangarooisthattheycanbealmostastallasyoupleaseandstillbeclosetofact.

10.Kangaroosarefoundonly

(A)inAustralia.

(B)inAustralasia.

(C)ontheAsianmainland.

(D)inTasmania.

(E)onNewGuinea.

11.Afemalekangarooiscalled

(A)awallaby.

(B)apotoroo.

(C)aJoey.

(D)aflyer.

(E)theOldMan.

12.Theamazingjumpingpowerofthekangarooischieflyduetothe

(A)powerofthehindlegs.

(B)supportofthetail.

(C)kangaroo’ssize.

(D)kangaroo’sweight.

(E)kangaroo’stiltedsittingposition.

13.Whichstatementistrueaccordingtothepassage?

(A)Thename“OldMan”showsthepeople’sdislikeofkangaroos.

(B)VisitorstoAustraliahearverylittleaboutkangaroos.

(C)Akangaroo’stailisapowerfulweapon.

(D)Themostwidelyknownspeciesofkangarooisnolargerthanarabbit.

(E)Kangarooshavethreelegs.

14.Theauthorbelievesthatthestoriestoldaboutkangaroosaregenerally

(A)harmful.

(B)true.

(C)suspicious.

(D)beautiful.

(E)ancient.

Whatisacordofwood?Somepeoplesaythecordisthemostelasticunitofmeasureeverdevisedbythemindofhumans.A“standard”cordisapileofstackedwood4×4×8feet;that’s128cubicfeet.Howmuchofthisiswood?Thatdependsonwhatkindofwood,thesizeandstraightnessofthesticks,andwhodoesthepiling.Smallcrookedsticks,cutfromhardwoodlimbsandpiledbyoneofthosecordwoodartistswhoknowhowtomakeairspaces,maycontainlessthan30cubicfeetofsolidwoodpercord.Smooth,roundwoodsuchasbirchorspruce,insizeseightinchesandbetter,willaverage100cubicfeetormorepercord.That’swiththebarkon.Peeledwoodwillmake10to12percentmorecubicvolumeinthe

samesizedstack.

Theheatingvalueofwoodvariesenormouslywiththekindoftree.Blacklocust,whiteoak,hickory,blackbirch,andironwoodarethebest.Acordofanyofthesewoods,whenseasoned,isworthapproximatelyatonofcoal.Beech,yellowbirch,sugarmaple,ash,andredoakarenext.Whitebirch,cherry,softmaple,sycamore,andelmarecomparativelypoorfuelwoods,withbasswood,butternut,poplar,andthesoftwoodsatthebottomofthescale.

15.Whichtitlebestexpressesthemainideaofthisselection?

(A)“Fuels”

(B)“TheValueofaCordofWood”

(C)“KindsofTrees”

(D)“StandardMeasures”

(E)“ModernHeating”

16.Astandardcordofwood

(A)alwayscontains128cubicfeetofwood.

(B)willaverage100cubicfeetofwood.

(C)containslessthan30cubicfeetofsolidwood.

(D)isstackedwoodinapile4×4×8feet.

(E)ismeasuredbyweightofthewoodperfoot.

17.Removalofthebarkbeforestacking

(A)increasesthecubicvolumeofwoodinacord.

(B)makesthestackingeasier.

(C)allowsmoreairspacesinacordofwood.

(D)preventsseasoningofwood.

(E)decreasesthemeasurementsofthewood.

18.Theamountofheatsuppliedbywooddependsuponthe

(A)personwhohaspiledthewood.

(B)typeoftreefromwhichthewoodcame.

(C)waythewoodwascut.

(D)straightnessofthesticks.

(E)amountofbarkleftonthewood.

19.Themostvaluablefuelwoodscomefrom

(A)allkindsofbirchesandoaks.

(B)anykindofwoodthatiswell-seasoned.

(C)home-grownbeech,maple,cherry,andelmtrees.

(D)hickory,ironwood,blackbirch,blacklocust,andwhiteoak.

(E)sycamore,ash,butternut,andpoplarthathavebeensprayed.

Eightofthecity’stwelveworkersinVenetianglassrecentlyfinishedoneofthemostunusualmuralsevermadeforaNewYorkskyscraper.Itisanabstract,thecreationofHansHofmann,a77-year-oldGerman-bornpainter.

Themuralcovers1,200squarefeetoftheouterwalloftheelevatorshaftintheWilliamKaufmanBuildingat711ThirdAvenue.Morethanahalf-milliontilesincloseto500shadesofcolorhavegoneintoit.Blue,red,andyellowarethechiefcolors.EachtilewasmadeinVeniceandissomewhatlessthanpostage-stampsize.Eachisbeatenintoaspecialeverlastingconcretewithakindofflatwoodenhandtoolusedfornothingelse.

Mr.Hofmanndidtheoriginalcolorsketchaboutone-sixthofthefinalsize.Thiswasphotographed,andfromthenegativeanenlargementwashand-coloredbytheartist,cutintosections,andsentinthatformtotheVincentFoscatoplantinLongIslandCity,whichspecializesinVenetianglasstile,ormosaic.TheretheVenetianspecialists,whosetradehasbeenhandeddownthroughfamiliesthroughthecenturies,seteachmosaicintoplaceonthecartoonsection,withpainstakingfidelitytoMr.Hofmann’scolorrendering.AlthoughMr.Foscato’splantkeeps1,400shadesoftheglassmosaic,ithadtohavetwelveadditionalshadesspeciallymadeinVenicetomatchthesketchcoloringforperfectblending.Whenallthesectionshadbeenfilledandapproved,theywerecarriedbytrucktothebuildinglobby,thewallswerecoveredwithaspecialcement,andtheworkerscarefullybeateachbitintoplace.

20.Whichtitlewouldbebestforthisselection?

(A)“PicturebyGermanArtisttoHanginNewYork”

(B)“NewMosaicDesignedbyVincentFoscato”

(C)“UnusualPhotographDecoratesNewYorkBuilding”

(D)“Venetian-GlassMuralInstalledinSkyscraper”

(E)“TheWilliamKaufmanBuilding”

21.Theoriginaldesignwas

(A)paintedonthewalloftheKaufmanbuilding.

(B)afractionofthesizeofthefinishedmural.

(C)importedfromVenice.

(D)largerthanthefinishedmural.

(E)projectedonalargesheetofpaper.

22.Mr.Hofmann

(A)learnedfromhisfatherhowtodomosaicwork.

(B)isanativeofNewYork.

(C)isapainter.

(D)livesinLongIslandCity.

(E)isaVenetian-glassspecialist.

23.Inmakingthemural

(A)theshadesoftilethattheFoscatoplanthadinstockwerenotadequate.

(B)1,412shadeswereneeded.

(C)halfamillioncolorswereused.

(D)over500shadesofcolorwereused.

(E)1,400specialistswereconsulted.

24.Mr.Hofmann

(A)tookacolorphotographofhispainting.

(B)usedonlythemostunusualshadesofred,blue,andgreen.

(C)hadnofurtherconnectionwiththeworkaftermakingtheoriginalsketch.

(D)diedshortlybeforethemuralwascompleted.

(E)coloredtheenlargedreproductionoftheoriginal.

25.Ofthetilesused

(A)someweremadeofspecialcolorsbyMr.Foscato.

(B)allweremadebytheworkerswhoputthemuralinplace.

(C)allweremadeinItaly.

(D)allweremadeinNewYork.

(E)manyweremadebyawoodenhandtool.

26.Themosaicwasassembledby

(A)HansHofmann.

(B)anartistspecializinginVenetianglass.

(C)VincentFoscatoofLongIsland.

(D)workersintheFoscatoplant.

(E)Venetianworkers.

Thehistoryofmodernpollutionproblemsshowsthatmosthaveresultedfromnegligenceandignorance.Wehaveanappallingtendencytointerferewithnaturebeforeallofthepossibleconsequencesofouractionshavebeenstudiedin-depth.Weproduceanddistributeradioactivesubstances,syntheticchemicals,andmanyotherpotentcompoundsbeforefullycomprehendingtheireffectsonlivingorganisms.Oureducationisdangerouslyincomplete.

Itisoftenarguedthatthepurposeofscienceistomoveintounknownterritory,toexplore,andtodiscover.Itcanbesaidthatsimilarriskshavebeentakenbefore,andthattheserisksarenecessarytotechnologicalprogress.

Theseargumentsoverlookanimportantelement.Inthepast,riskstakeninthenameofscientificprogresswererestrictedtoasmallplaceandabriefperiodoftime.Theeffectsoftheprocesseswenowstrivetomasterareneitherlocalizednorbrief.Airpollutioncoversvasturbanareas.Oceanpollutantshavebeendiscoveredinnearlyeverypartoftheworld.Syntheticchemicalsspreadoverhugestretchesofforestandfarmlandmayremaininthesoilfordecades.Radioactivepollutantswillbefoundinthebiosphereforgenerations.Thesizeandpersistenceoftheseproblemshavegrownwiththeexpandingpowerofmodernscience.

Onemightalsoarguethatthehazardsofmodernpollutantsaresmallcomparedtothedangersassociatedwithotherhumanactivity.Noestimateoftheactualharmdonebysmog,fallout,orchemicalresiduescanobscuretherealitythattherisksarebeingtakenbeforebeingfullyunderstood.

Theimportanceoftheseissuesliesinthefailureofsciencetopredictandcontrolhumaninterventionintonaturalprocesses.Thetruemeasureofthedangerisrepresentedbythehazardswewillencounterifweenterthenewageoftechnologywithoutfirstevaluatingourresponsibilitytotheenvironment.

27.Accordingtotheauthor,themajorcauseofpollutionistheresultof

(A)designingsyntheticchemicalstokilllivingorganisms.

(B)alackofunderstandingofthehistoryoftechnology.

(C)scientistswhoaretoowillingtomoveintounknownterritory.

(D)changingourenvironmentbeforeunderstandingtheeffectsofthesechanges.

(E)notpassingenoughlaws.

28.Theauthorbelievesthattheriskstakenbymodernsciencearegreaterthanthosetakenbyearlierscientificeffortsbecause

(A)theeffectsmaybefeltbymorepeopleforalongerperiodoftime.

(B)scienceisprogressingfasterthaneverbefore.

(C)technologyhasproducedmoredangerouschemicals.

(D)thematerialsusedaremoredangeroustoscientists.

(E)theproblemsaregreater.

29.Theauthorapparentlybelievesthattheproblemoffindingsolutionstopollutiondependson

(A)theremovalofpresenthazardstotheenvironment.

(B)theremovalofallpotentialpollutantsfromtheirpresentuses.

(C)overcomingtechnicaldifficulties.

(D)thewillingnessofscientiststounderstandpossibledangersbeforeusingnewproductsintheenvironment.

(E)anewageofsciencethatwillrepairthefaultsofourpresenttechnology.

30.Theauthorseemstofeelthattheattitudeofscientiststowardpollutionhasbeen

(A)naïve.

(B)concerned.

(C)confused.

(D)ignorant.

(E)nonchalant.

31.Thewordsyntheticmeans

(A)new.

(B)unsafe.

(C)polluting.

(D)man-made.

(E)progressive.

Athirdofourlivesisspentinthemysteriousstateofsleep.Throughoutourhistory,wehaveattemptedtounderstandthisremarkableexperience.Manycenturiesago,forexample,sleepwasregardedasatypeofanemiaofthebrain.Alemaeon,aGreekscientist,believedthatbloodretreatedintotheveins,andthepartiallystarvedbrainwenttosleep.Platosupportedtheideathatthesoulleftthebodyduringsleep,wanderedtheworld,andwokeupthebodywhenitreturned.

Recently,morescientificexplanationsofsleephavebeenproposed.Accordingtoonetheory,thebrainisputtosleepbyachemicalagentthataccumulatesinthebodywhenitisawake.Anothertheoryisthatwearybranchesofcertainnervecellsbreakconnectionswithneighboringcells.Theflowofimpulsesrequiredforstayingawakeisthendisrupted.Thesemorerecenttheorieshavehadtobesubjectedtolaboratoryresearch.

Whydowesleep?Whydowedream?Modernsleepresearchissaidtohavebeguninthe1950s,whenEugeneAserinsky,agraduatestudentattheUniversityofChicago,andNathanielKleitman,hisprofessor,observedperiodsofrapideyemovements(REMs)insleepingsubjects.WhenawakenedduringtheseREMperiods,subjectsalmostalwaysremembereddreaming.Ontheotherhand,whenawakenedduringnon-REMphasesofsleep,thesubjectsrarelycouldrecalltheirdreams.

GuidedbyREMs,itbecamepossibleforinvestigatorsto“spot”dreamingfromoutsideandthenawakenthesleeperstocollectdreamstories.Theycouldalsoalterthedreamers’experienceswithnoises,drugs,orotherstimulibeforeorduringsleep.

Sincethemid-1950s,researchershavebeendrawnintosleeplaboratories.There,bedroomsadjoinotherroomsthatcontainrecordersknownaselectroencephalograph(EEG)machines.

TheEEGamplifiessignalsfromsensorsontheface,head,andotherpartsofthebody,whichtogetheryieldtracingsofrespiration,pulse,muscletension,andchangesofelectricalpotentialinthebrainthataresometimescalledbrainwaves.Theserecordingssupplycluestothechangesofthesleepingperson’sactivities.

32.Themainpurposeofthispassageisto

(A)describeearlybeliefsaboutsleep.

(B)comparemodernscientifictheoriestoearlyideasaboutsleep.

(C)pointouttheimportanceofREMsinhumansleep.

(D)describemodernresearchtechniques.

(E)giveashorthistoryofhuman’sinterestinsleep.

33.ThispassageimpliesthattheimportanceoftheresearchofAserinskyandKleitmanwas

mainlyinthe

(A)reportstheypublished.

(B)problemstheyattacked.

(C)informationtheyobservedandrecorded.

(D)understandingstheyuncovered.

(E)conclusionstheydrewfortreatmentofsleepdisorders.

34.AllofthefollowingwerementionedaspossiblecausesofsleepEXCEPT

(A)exhaustednerveendings.

(B)abuild-upofcertainbodychemicals.

(C)recurrentperiodsofrapideyemovement.

(D)theabsenceoftheconsciousspirit.

(E)thedepartureofthesoulfromthebody.

35.Inparagraph4,thewordstimulimeans

(A)substancesthatmakeapersonmorealert.

(B)drugs.

(C)sleepinducing.

(D)comatose.

(E)thingsthatcausethebodytoreactinacertainway.

Ashethrewhisheadbackinthechair,hisglancehappenedtorestuponabell,adisusedbell,thathungintheroomandcommunicated,forsomepurposenowforgotten,withachamberinthehigheststoryofthebuilding.Itwaswithgreatastonishment,andwithastrangeinexplicabledread,that,ashelooked,hesawthisbellbegintoswing.Soonitrangoutloudly,andsodideverybellinthehouse.

Thiswassucceededbyaclankingnoise,deepdownbelowasifsomepersonweredraggingaheavychainoverthecasksinthewinemerchant’scellar.Thenheheardthenoisemuchlouderonthefloorsbelow;thencomingupthestairs;thencomingstraighttowardhisdoor.

Itcameinthroughtheheavydoor,andaspecterpassedintotheroombeforehiseyes.Anduponitscomingin,thedyingflameleapedup,asthoughitcried,“Iknowhim!Marley’sghost!”

—fromAChristmasCarol,

byCharlesDickens

36.Thewordinexplicablemeans

(A)explaininginsimpleterms.

(B)notabletobetakenoutof.

(C)withoutanexpressedreason.

(D)eerie.

(E)incapable.

37.Thebellthatbeganringing

(A)waslargeandheavy.

(B)didsobyitself.

(C)couldberungfromanotherroom.

(D)wasattachedtoeverybellinthehouse.

(E)restedfirstonhisglance.

38.Themanwhowaslisteningtothebell

(A)draggedachainacrossthewinecasks.

(B)satperfectlystill.

(C)wasapparentlyveryfrightened.

(D)wasMarley’sghost.

(E)wasquitecurious.

39.Thewordspecterprobablymeans

(A)along-handledsword.

(B)abrightlight.

(C)ahazy,recognizablevision.

(D)strangenoises.

(E)clankingchains.

40.Themaninthestory

(A)firstheardnoisesinhisroom.

(B)isprobablyawinemerchant.

(C)hadbeenasleep.

(D)recognizedMarley’sghost.

(E)settheroomonfire.

Section3:Verbal

60Questions•30Minutes

TheVerbalsectionconsistsoftwodifferenttypesofquestions.Therearedirectionsforeachtypeofquestion.

Directions:Eachquestionshowsawordincapitallettersfollowedbyfivewordsorphrases.Choosethewordorphrasewhosemeaningismostsimilartothewordincapitalletters.

1.DETER

(A)halt

(B)steer

(C)sting

(D)turn

(E)hinder

2.HOSTILE

(A)friendly

(B)unfriendly

(C)suspicious

(D)indifferent

(E)doubtful

3.UTILIZE

(A)makeuseof

(B)utilities

(C)modernize

(D)sing

(E)undo

4.ABDICATE

(A)resign

(B)explain

(C)remorse

(D)disprove

(E)control

5.PROMINENT

(A)disturbing

(B)secret

(C)outstanding

(D)extravagant

(E)surreptitious

6.BOUNDARY

(A)hovel

(B)limit

(C)ceiling

(D)map

(E)seam

7.ILLITERATE

(A)unabletovote

(B)unmanageable

(C)sickly

(D)unabletoread

(E)unclean

8.ORATOR

(A)professor

(B)poet

(C)speaker

(D)ear

(E)student

9.CORROBORATE

(A)confirm

(B)understand

(C)cooperate

(D)agree

(E)disagree

10.RATIFY

(A)delete

(B)consider

(C)approve

(D)examine

(E)assess

11.PERILOUS

(A)careless

(B)conniving

(C)irregular

(D)estranged

(E)hazardous

12.STATIONARY

(A)paper

(B)moving

(C)immobile

(D)position

(E)mobile

13.TRANSCRIBE

(A)copy

(B)illustrate

(C)circulate

(D)request

(E)author

14.PROFICIENT

(A)well-known

(B)professional

(C)adept

(D)practice

(E)prolific

15.DECEIVE

(A)rearrange

(B)mislead

(C)pretend

(D)stun

(E)examine

16.AGILE

(A)strong

(B)similar

(C)anxious

(D)rested

(E)nimble

17.DURATION

(A)area

(B)temptation

(C)term

(D)wait

(E)former

18.AMBIGUOUS

(A)unclear

(B)adhere

(C)aspire

(D)afflict

(E)certain

19.PREROGATIVE

(A)command

(B)choice

(C)prerequisite

(D)conviction

(E)haggard

20.INTRIGUING

(A)business

(B)furtive

(C)mystery

(D)fascinating

(E)boorish

21.CLANDESTINE

(A)overt

(B)dated

(C)exclusive

(D)fortunate

(E)secret

22.BOUNTEOUS

(A)elastic

(B)industrious

(C)abundant

(D)mutinous

(E)energetic

23.DIVERGE

(A)annoy

(B)changecourse

(C)stay

(D)analyze

(E)distract

24.BENIGN

(A)gentle

(B)blessed

(C)initial

(D)virulent

(E)malignant

25.CAUCUS

(A)dispersal

(B)corpse

(C)meeting

(D)partnership

(E)cosmetic

26.DISSEMINATE

(A)collate

(B)strip

(C)collect

(D)disagree

(E)spread

27.CHAGRIN

(A)delight

(B)alter

(C)embarrass

(D)wreck

(E)anger

28.VALOR

(A)courage

(B)disclosure

(C)treason

(D)hate

(E)foreboding

29.NONCHALANT

(A)interested

(B)caring

(C)impoverished

(D)indifferent

(E)persecuted

30.LIAISON

(A)permission

(B)laziness

(C)scarf

(D)remedy

(E)association

Directions:Thefollowingquestionsaskyoutofindrelationshipsbetweenwords.Readeachquestion,andthenchoosetheanswerthatbestcompletesthemeaningofthesentence.

31.Begistoborrowasofferisto

(A)lender

(B)bank

(C)lend

(D)repay

(E)security

32.Lazyistoinertasresististo

(A)refuse

(B)reply

(C)respond

(D)active

(E)insist

33.Cylinderistocircleaspyramidisto

(A)sphere

(B)point

(C)triangle

(D)angle

(E)height

34.Crocodileistoreptileaskangarooisto

(A)amphibian

(B)marsupial

(C)opossum

(D)canine

(E)tail

35.Milliliteristoquartas

(A)poundistogram

(B)millimeteristoyard

(C)inchistoyard

(D)pintistoquart

(E)footistoyard

36.Destroyistodemolishas

(A)winistolose

(B)candidistosecret

(C)amendistochange

(D)establishistoabolish

(E)attemptistosucceed

37.Plaintiffistodefendantas

(A)plainistoordinary

(B)lawyeristocourtroom

(C)professoristocollege

(D)complainistocomplainant

(E)prosecuteistodefend

38.Fundamentalistofrivolousas

(A)fantasyistofiction

(B)nonfictionistofact

(C)regulationistorule

(D)truthistononsense

(E)strangeistocommon

39.Wildistowolfasdomesticisto

(A)dog

(B)coyote

(C)pet

(D)cat

(E)animal

40.Hammeristocarpenteras

(A)awlistocobbler

(B)computeristoprinter

(C)sawistotimber

(D)authoristotypewriter

(E)scaleistomusician

41.Subjectistopredicateassenatoristo

(A)congress

(B)president

(C)capitol

(D)representative

(E)senate

42.Pungentistoodoras

(A)intenseistoemotion

(B)pervadeistoatmosphere

(C)infectistospread

(D)proverbistoparagraph

(E)resentistodenial

43.Exploitistoadventureas

(A)ruleistogovernor

(B)safariistoexpedition

(C)schoolistofieldtrip

(D)attackistohunt

(E)chaseistoescape

44.Spreadistoscatterasseparateisto

(A)integrate

(B)distribute

(C)reap

(D)group

(E)displace

45.Exuberantistomoodasadroitisto

(A)proficient

(B)adept

(C)hand

(D)dexterous

(E)movement

46.Defianceistooppositionasexertionisto

(A)expert

(B)vigor

(C)endeavor

(D)restraint

(E)challenge

47.Foodistonutritionaslightisto

(A)watt

(B)bulb

(C)electricity

(D)reading

(E)vision

48.Perpetuityistoimpermanenceasinterminableisto

(A)impertinent

(B)brief

(C)incessant

(D)eternal

(E)occasional

49.Erraticistopredictableasexorbitantisto

(A)reasonable

(B)productive

(C)absorbent

(D)small

(E)implicit

50.Commentistospeechas

(A)questionistoanswer

(B)exclamationistostatement

(C)writtenistospoken

(D)proseistoessay

(E)noteistoletter

51.Flammableistoinflammableas

(A)persistentistoimportant

(B)opportuneistoinopportune

(C)relevantistoincoherent

(D)truculentistointrusion

(E)impartialistodisinterested

52.Tailoristopatternasbuilderisto

(A)architect

(B)contractor

(C)foundation

(D)construct

(E)blueprint

53.Impeachistodismissas

(A)arraignistoindict

(B)accuseistocharge

(C)imprisonistojail

(D)plantistosow

(E)absentistopresent

54.Speedyistogreyhoundas

(A)woolistolamb

(B)sharkistovoracious

(C)cleveristofox

(D)mammalistowhale

(E)finistofish

55.Exhaleistolungas

(A)exhumeistocorpse

(B)pumpistoheart

(C)thinkistobrain

(D)perspireistoskin

(E)tasteistotongue

56.Celebrateistobirthas

(A)grieveistodeath

(B)announceistobirthday

(C)crimeistopenalty

(D)joyistolament

(E)partyistograduation

57.Recommendistourgeas

(A)requestistoplead

(B)refuseistodeny

(C)harassistobother

(D)cajoleistoinsult

(E)applyistoreceive

58.Weepingistotearsasbreathingisto

(A)air

(B)lungs

(C)nose

(D)mouth

(E)carbondioxide

59.Planeistoairpocketas

(A)vehicleistorut

(B)hangaristoairport

(C)groundistosky

(D)safetyistodanger

(E)horseistoreins

60.Arbitrateistodisputeas

(A)solveistomystery

(B)regardistoproblem

(C)exacerbateistoproblem

(D)organizeistolabor

(E)managementistounion

Section4:Quantitative(Math)

25Questions•30Minutes

Directions:Eachquestionbelowisfollowedbyfivepossibleanswers.Selecttheonethatisbest.

1.In2hours,theminutehandofaclockrotatesthroughanangleof

(A)60°

(B)90°

(C)180°

(D)360°

(E)720°

2.Whichofthefollowingfractionsislessthanonethird?

(A)

(B)

(C)

(D)

(E)

3.

Thelengthofeachsideofthesquareaboveis +1.Theperimeterofthesquareis

(A) +4

(B)

(C) +4

(D) +16

(E) +2

4.

Thediagramshowsacube.ThedistancefromAtoXis

(A)2inches

(B) inches

(C) inches

(D)1inch

(E) inches

5.Amotoristtravels120milestohisdestinationatanaveragespeedof60milesperhourandreturnstothestartingpointatanaveragespeedof40milesperhour.Whatistheaveragespeedforhisentiretrip?

(A)53milesperhour

(B)52milesperhour

(C)50milesperhour

(D)48milesperhour

(E)45milesperhour

6.Asnapshotmeasures2 inchesby1 inches.Itistobeenlargedsothatthelonger

dimensionwillbe4inches.Thelengthoftheenlargedshorterdimensionwillbe

(A)2 inches.

(B)2 inches.

(C)3inches.

(D)3 inches.

(E)3 inches.

7.Fromapieceoftinintheshapeofasquare6inchesonaside,thelargestpossiblecircleiscutout.Ofthefollowing,theratiooftheareaofthecircletotheareaoftheoriginalsquareisclosestinvalueto

(A)

(B)

(C)

(D)

(E)

8.Iftheouterdiameterofametalpipeis2.84in.andtheinnerdiameteris1.94in.,thethicknessofthemetalis

(A)0.45in.

(B)0.90in.

(C)1.42in.

(D)1.94in.

(E)2.39in.

9.Asportswriterclaimsthatherfootballpredictionsareaccurate60%ofthetime.Duringfootballseason,afankeptrecordsandfoundthatthewriterwasinaccurateforatotalof16games,althoughshedidmaintainher60%accuracy.Forhowmanygameswasthesportswriteraccurate?

(A)5

(B)15

(C)24

(D)40

(E)60

10.Inacertainboys’camp,30%oftheboysarefromNewYorkStateand20%ofthesearefromNewYorkCity.WhatpercentoftheboysinthecamparefromNewYorkCity?

(A)60%

(B)50%

(C)33%

(D)10%

(E)6%

11.

Aunitblockforconstructionis1×2×3inches.Whatisthenumberofwholeblocks

requiredtocoveranarea1footlongby1 feetwidewithonelayerofblocks?

(A)30blocks

(B)60blocks

(C)72blocks

(D)90blocks

(E)180blocks

12.Ifthenumberofsquareinchesintheareaofacircleisequaltothenumberofinchesinitscircumference,thediameterofthecircleis

(A)4inches.

(B)2inches.

(C)1inch.

(D)πinches.

(E)2πinches.

13.Theleastcommonmultipleof20,24,and32is

(A)240

(B)480

(C)960

(D)1920

(E)15,360

14.If9x+5=23,thenumericalvalueof18x+5is

(A)46

(B)41

(C)38

(D)36

(E)32

15.Whenthefractions , , ,and arearrangedinascendingorderofsize,theresultis

(A) , , ,

(B) , , ,

(C) , , ,

(D) , , ,

(E) , , ,

16.Ifacubicinchofametalweighs2pounds,acubicfootofthesamemetalweighs

(A)8pounds.

(B)24pounds.

(C)96pounds.

(D)288pounds.

(E)3456pounds.

17.Amicromillimeterisdefinedasonemillionthofamillimeter.Alengthof17micromillimetersmayberepresentedas

(A)0.00017mm.

(B)0.000017mm.

(C)0.0000017mm.

(D)0.00000017mm.

(E)0.000000017mm.

18.Tofindtheradiusofacirclewhosecircumferenceis60inches,

(A)multiply60byπ.

(B)divide60by2π.

(C)divide30by2π.

(D)divide60byπandextractthesquarerootoftheresult.

(E)multiply60by .

19.Acarpenterneedsfourboards,each2feet9incheslong.Ifwoodissoldonlybythefoot,howmanyfeetmusthebuy?

(A)9

(B)10

(C)11

(D)12

(E)13

20.Theapproximatedistance,S,infeetthatanobjectfallsintsecondswhendroppedfromaheightcanbefoundbyusingtheformulaS=16t2.In8secondstheobjectwillfall

(A)256feet.

(B)1024feet.

(C)1084feet.

(D)2048feet.

(E)15,384feet.

Questions21and22refertothefollowinggraph.

21.Duringwhichyearsdidthepopulationincreaseatthefastestrate?

(A)Years5–7

(B)Years1–3

(C)Years4–5

(D)Years7–9

(E)Years9–10

22.Duringwhichyeardidthesizeofthepopulationdecreasethemost?

(A)Years4–5

(B)Years3–4

(C)Years9–10

(D)Years1–2

(E)Years4–6

23.ThenumberoftelephonesinAdelaide,Australia,is48,000.Ifthisrepresents12.8telephonesper100people,thepopulationofAdelaidetothenearestthousandis

(A)128,000

(B)375,000

(C)378,000

(D)556,000

(E)575,000

24.Onepersoncanloadatruckin25minutes,asecondcanloaditin50minutes,andathirdcanloaditin10minutes.Howlongwouldittakethethreetogethertoloadthetruck?

(A)5 minutes

(B)6 minutes

(C)8 minutes

(D)10minutes

(E)28 minutes

25.EventAoccursevery4minutes,eventBevery6minutes,andeventCevery15minutes.Iftheyoccursimultaneouslyatnoon,whenisthenexttimeallthreeeventswilloccurtogetheragain?

(A)1p.m.

(B)1:30p.m.

(C)3p.m.

(D)6p.m.

(E)12a.m.

ANSWERSANDEXPLANATIONSPartI:WritingSample

Exampleofawell-writtenessay.

Theproposaltoraisetheminimumlicensingagefrom16to18shouldberejectedforanumberofreasons.Therearenosolidstatisticsprovingthatyouthscausetheaccidentsthattheyareinvolvedin,sothe16-and17-year-oldagegroupshouldnotbepenalizedforthoseaccidents.Also,formanyyoungpeople,useofacarisanabsolutenecessity.

Legislatorsshouldaskthemselveswhy16-to18-year-olddriverstendtobeinvolvedinaccidents.Ithinkthatthemaincauseoftheseaccidentsislackofexperience.Ifastudyweremade,Isuspectthatitwouldshowthatnewdriversofanyagetendtohaveaccidents.Raisingthelicensingagewouldonlyraisetheageofdriversinvolvedinaccidents.Abettercuremightbedrivingeducationprogramsthatstressjudgmentontheroadandarequirementforalongerperiodofdrivingundersupervisionbeforelicensing.

Raisingthedrivingagewouldcreatearealfinancialhardshipforsometeenagersandtheirfamilies.Manyworkingparentscountontheirhighschoolerstotransportyoungerchildrenintheafternoonandtorunerrands.Otherteenshavepart-timejobsinlocationsthatcanbereachedonlybycar.Attemptingtosolvetheaccidentproblembycreatingfinancialproblemsdoesnotseemlogical.

Raisingthelicensingagecouldactuallyleadtoahigheraccidentrate.Teenagerswhohavetodrivewoulddriveanyway,butwithoutbenefitofdrivereducationorthetestingthatisrequiredforgettingthatlicense.Untrained,untestedteenagedriverswouldbeamenaceontheroads.The16-yearminimumshouldberetainedforthesafetyofall.

PartII:MultipleChoice

Section1:Quantitative(Math)

1.Thecorrectansweris(D).Rename1 asthedecimal1.5andadd.

2.Thecorrectansweris(B).Whendividingnumbershavingthesamebase,simplysubtracttheexponents.

=212–8=24=16

3.Thecorrectansweris(C).

4.Thecorrectansweris(D).Simplifythenumeratorofthefraction,andthendivide.

5.Thecorrectansweris(C).

6.Thecorrectansweris(C).Beginworkingwiththeinnermostparenthesesandworkyourwayout.

7.Thecorrectansweris(B).Thisisagoodproblemtodoinyourhead.Mentallysubtract700from3003andget2303.Thenlookattheanswerscarefullyandnotethatonlychoice(B)isclosetoyourestimate.

8.Thecorrectansweris(A).Theproblemstatesthata=5andb= ; is of5.

Therefore,thevalueofaexpressedintermsofbis25× =5,or25b.

9.Thecorrectansweris(C).Thisisagoodproblemtodoinyourhead.Notethat10%of70is7.140%,then,is14×7,or98.

10.Thecorrectansweris(C).Borrowagallonandadditto2quarts.Rewritetheproblem.Rememberthatyouborrowed.

11.Thecorrectansweris(E).Bydoublingthedenominatorofafraction,weactuallydivideitby2.Byhalvingoneofthefactorsinthenumerator,wealsohalvethevalueofthefraction.Bydoingboth,wehaveactuallydividedtheoriginalvalueby4.Pluginsomevaluesforx,y,andz,andtrythis.

12.Thecorrectansweris(D).Thisisagoodproblemtoestimate.Since8%isslightlyless

than ,youcanmultiply20by12toapproximatetheanswer.Notethat250isclose

enoughtoyour240estimate.Tobeprecise:

20÷0.08=

13.Thecorrectansweris(C).100is20largerthan80.20isonefourth,or25%,of80.

14.Thecorrectansweris(C).40inchesequals3 feet.Since in.onthedrawingequals1

footatfullscale,

15.Thecorrectansweris(C).Bracketthemultiplicationanddivisionfirst,andsolvetheproblem.

=18+6

=24

16.Thecorrectansweris(A).Sincex–3<12,xcanbeanynumberlessthan15.

17.Thecorrectansweris(D).Substitutethevaluesintotheexpression.

18.Thecorrectansweris(B).Tofindtheaverage,findthesumoftheaddendsanddividethatsumbythenumberofaddends.

–10+6+0+–3+22=15

15÷5=3

19.Thecorrectansweris(B).Bysubstituting+2forthetriangle,thedenominatorofthefractionbecomeszero.Adenominatorofzeroisundefinedinmathematics.

20.Thecorrectansweris(C).Simplymovethedecimalpointoneplacetotheleftandinsertazerointhenewlycreateddecimalplace.

0.10101÷10=0.010101

21.Thecorrectansweris(D).Theformulaforrateisrate=distance÷time.Inthis

problem,rate=10miles÷3hours,or3 milesperhour.

22.Thecorrectansweris(B).7isonethirdof21,and isonethirdof2.Asaproportion:

=

23.Thecorrectansweris(C).Thesquarerootof85isbetween9,whosesquareis81,and10,whosesquareis100.

24.Thecorrectansweris(A).–6+–2+0+8=0

25.Thecorrectansweris(E).Simplydivide by .

Section2:ReadingComprehension

1.Thecorrectansweris(A).Thisisadetailquestionthatyouanswerthroughaprocessofelimination.Choices(D)and(E)arementionedinthefirstparagraph,whereas(B)and(C)maybefoundinthesecondsentenceofthesecondparagraph.

2.Thecorrectansweris(A).Carefulreadingwillfindthisdetailinlines13–17.

3.Thecorrectansweris(E).Seethefirstsentence.

4.Thecorrectansweris(A).Youmustinferthisanswerfromtheinformationgiven.Because“themalewolverine...fightstothedeathanymalethatintrudesonhisdomain...”onlythefittest,thebestfighters,survive,andnumbersarehelddown.

5.Thecorrectansweris(A).Thewriter’stotalfascinationwithandexpertiseonthesubjectofthewolverineleadsustoinferthatheisanaturalist.Thefactthatheiscarryingaguninwildcountrydoesnotnecessarilyimplythatheisahunter.Prudentnaturalistsmaycarrygunsforself-defenseinthewilderness.

6.Thecorrectansweris(B).Thewolverineisadestroyer,andevenlargeanimalssuccumbtohisattacks.Thelargeanimalsareovercomebythewolverine,butthewordsuccumbisanactiveverb;therefore,whattheanimalswhoareovercomedoissurrender.

7.Thecorrectansweris(B).Thisisamain-ideaquestion.TheparagraphdescribesChiron.

8.Thecorrectansweris(A).TheparagraphstatesthatChironhadthelegsofahorse;ahorsehasfourlegs.Frombasicetymology,youknowthat“quad”means“four”andthat“ped”referstofeet.

9.Thecorrectansweris(E).Readcarefully.Thephysicaldescriptionistheonlychoicesupportedbytheparagraph.ThelastsentencespecificallystatesthatChirondidnotteachwritingandarithmetic.

10.Thecorrectansweris(B).Seethefirstsentence.

11.Thecorrectansweris(D).Seethenext-to-lastsentenceofthefirstparagraph.

12.Thecorrectansweris(A).Lines15–16providethisinformation.

13.Thecorrectansweris(C).Youcertainlycaninferthisfromthelastsentenceofthesecondparagraph.

14.Thecorrectansweris(B).Thisistheauthor’smeaninginthelastsentence,inwhichtheauthorstatesthatthetalltalesareclosetofact.

15.Thecorrectansweris(B).Thisisamain-ideaquestion,andyoumustchoosethemostinclusivetitlefortheentireselection.Theselectiondiscussesthevalueofacordofwood—thevolumevalueinthefirstparagraphandtheheatingvalueinthesecondparagraph.

16.Thecorrectansweris(D).Thisdetailquestionisansweredinthethirdsentence.

17.Thecorrectansweris(A).Seethelastsentenceofthefirstparagraph.

18.Thecorrectansweris(B).Seethefirstsentenceofthesecondparagraph.

19.Thecorrectansweris(D).Seethesecondsentenceofthesecondparagraph.

20.Thecorrectansweris(D).Theselectionisallaboutthemural.

21.Thecorrectansweris(B).Thefirstsentenceofthethirdparagraphgivesthisinformation.

22.Thecorrectansweris(C).ThesecondsentencetellsusthatMr.Hofmannisapainter.Alltheotherchoicesconfuseotherinformation.

23.Thecorrectansweris(A).Thisisadifficultquestionrequiringconcentrationonthemeaningsofstatements.TheselectiontellsusthattheFoscatoplanthad1,400shadesinstockyethadtosendtoItalyfor12specialshades,butitdoesnotsaythatall1,400shadesinstockwereused.

24.Thecorrectansweris(E).Findtheanswertothisquestionbymeansofeliminationandinference.ThepassagetellsusthatMr.Hofmanncoloredtheenlargedreproductionoftheoriginal.Noneoftheotherchoicescanbesupportedbythetext.

25.Thecorrectansweris(C).Thenext-to-lastsentenceofthesecondparagraphstatesthateachtilewasmadeinVenice.VeniceisinItaly.

26.Thecorrectansweris(D).Inthemiddleofthethirdparagraphwelearnthatthemosaicwasassembled—eachmosaicwassetintoplace—byspecialistsintheFoscatoplant.

27.Thecorrectansweris(D).Readcarefullythesecondsentenceofthefirstparagraph.

28.Thecorrectansweris(A).Youcaninferthisanswerfromthethirdparagraphinwhichtheauthorstatesthat“Theeffectsoftheprocesses...areneitherlocalizednorbrief.”

29.Thecorrectansweris(D).Thisanswercanbeinferredfromthewholetoneoftheselection.Noneoftheotherchoicescanbesupportedbytheselection.

30.Thecorrectansweris(E).Nonchalantmeanscasuallyindifferent.Negligenceandignoranceonthepartofscientistscertainlyimpliesnonchalance.

31.Thecorrectansweris(D).Syntheticmeansnotnaturalorman-made.

32.Thecorrectansweris(E).Theapproachoftheentireselectionishistorical.Eachoftheotherchoicesfocusesononeparagraph,onephaseofthishistory.

33.Thecorrectansweris(C).AserinskyandKleitmanobserved,recorded,andreported;theydidnotexplain.Theirobservationsopenedlinesforfutureresearch.

34.Thecorrectansweris(C).Rapideyemovementismentionedasapartofsleep,notasacause.

35.Thecorrectansweris(E).Astimulusisanythingthatproducessomekindofreaction,physicalormental.StimuliisthepluralofthisLatinword.

36.Thecorrectansweris(C).Usewordarithmetic.In=not;able=abletobe;explic=explained;inotherwords,withoutanexpressedreason.

37.Thecorrectansweris(B).Ifadisusedbellsuddenlybegantoswing,itmusthavedonesoallbyitself.

38.Thecorrectansweris(C).Greatastonishmentandinexplicabledreadcertainlyimplyfright.

39.Thecorrectansweris(C).Youcangetthisdefinitionfromthelastparagraph.Aspecterisaghost.

40.Thecorrectansweris(D).Thisistheonlychoicesupportedbytheselection.Theanswerisinthelastsentence.Choice(A)isincorrectbecausethemanfirstsawthebellbegintomove,andthenheheardnoises.

Section3:Verbal

1.Thecorrectansweris(E).ToDETERistodiscourageapersonorgroupfromdoingsomething.Fearofretaliationmaydeterourenemiesfromattacking.

2.Thecorrectansweris(B).HOSTILEmeansantagonisticorunfriendly.Itisthenatureofcatstobehostiletodogs.

3.Thecorrectansweris(A).ToUTILIZEistomakepracticaluseof.Youcanutilizethemicrowaveforquickdefrosting.

4.Thecorrectansweris(A).ToABDICATEistogiveupformallyortoresign.King

EdwardabdicatedfromtheEnglishthronetomarryadivorcedwoman.

5.Thecorrectansweris(C).PROMINENTmeansstickingout,noticeable,oroutstanding.ComedianJimmyDurantehadaprominentnose.

6.Thecorrectansweris(B).ABOUNDARYisaborder.MexicosharesaboundarywiththestateofTexas.

7.Thecorrectansweris(D).ILLITERATEmeansunabletoread.Onewhoisliterateiseducatedandisabletoread.Theprefixil-createsthenegative.Manyofthehomelessareunemployablebecausetheyareilliterate.

8.Thecorrectansweris(C).AnORATORisonewhospeaks.(Canyouseetherootoral?)Thekeynotespeakerattheconventionwasasuperborator.

9.Thecorrectansweris(A).ToCORROBORATEistostrengthen,tosupport,ortoconfirm.Thebloodyknifeservedtocorroboratetheeyewitness’stestimony.

10.Thecorrectansweris(C).ToRATIFYistogiveofficialsanctiontoortoapprove.Ifthreemorestatesratifytheproposedamendment,itwillbecomeapartofourConstitution.

11.Thecorrectansweris(E).PERILOUSmeansriskyordangerous.Constructionofskyscrapersisperilouswork.

12.Thecorrectansweris(C).STATIONARYmeansnotmovingornotmovable.Itistheadjectiveformofthenounstation.Choice(A)referstothehomophonestationery.UnlikeEaster,Christmasisastationaryholiday;italwaysoccursonthesamedate.

13.Thecorrectansweris(A).ToTRANSCRIBEistowriteoutinfullortomakearecording.Thecourtstenographertranscribedthefullproceedingsofthetrial.

14.Thecorrectansweris(C).PROFICIENTmeanshighlyskilled,competent,oradept.Theconcertpianistisproficientattheart.

15.Thecorrectansweris(B).ToDECEIVEistomakeapersonbelievewhatisnottrue.Thepurposeofalieistodeceive.

16.Thecorrectansweris(E).AGILEmeansdeft,active,andlively.Prize-winninggymnastsarealwaysagile.

17.Thecorrectansweris(C).TheDURATIONisthetimethatathingcontinuesorlasts.Thedurationofaschoolsemesterisafour-monthterm.

18.Thecorrectansweris(A).AMBIGUOUSmeanshavingtwomeaningsorbeingvagueanduncertain.Theprefixambi-meansbothandimpliesthatbothpossibleinterpretationsmightbecorrectandthereforeneitherisclear.Herambiguousanswerleftusuncertainastowhethershemeant“Yes”or“No.”

19.Thecorrectansweris(B).APREROGATIVEisaright,privilege,orspecialadvantage.Theagedandthedisabledhavetheprerogativeofsittingatthefrontofthebus.

20.Thecorrectansweris(D).ThatwhichisINTRIGUINGexcitesinterestandcuriosityandisfascinating.Hisinterpretationoftheeventpresentsanintriguingnewtheory.

21.Thecorrectansweris(E).CLANDESTINEmeanssurreptitiousorsecret,usuallyforsomeillicitreasonorpurpose.Themarriedmanhadclandestinemeetingswithhismistress.

22.Thecorrectansweris(C).BOUNTEOUSmeansplentiful,generous,andabundant.Attheendofafavorablegrowingseason,wehadabounteousharvest.

23.Thecorrectansweris(B).ToDIVERGEistomoveoffindifferentdirectionsortobecomedifferent.Parallellinesdonotdiverge.

24.Thecorrectansweris(A).BENIGNmeansgood-natured,kindly,orharmless.Whenappliedtoatumor,benignmeansharmlessasopposedtomalignant,whichimplieslife-threatening.ThePopefacedhisaudiencewithabenignsmile.

25.Thecorrectansweris(C).ACAUCUSisameetingofpeoplewithsimilargoals,usuallyagroupofpeoplewithinalargergroup.TheCongressionalBlackCaucusmeetsperiodicallytodiscussminorityissues.

26.Thecorrectansweris(E).ToDISSEMINATEistoscatterwidely.ItisimportantthatwedisseminateinformationaboutthetransmissionofHIV/AIDS.

27.Thecorrectansweris(C).CHAGRINisembarrassmentorhumiliation.Thebettorwaschagrinedthatthehorsehehadpraisedsoloudlycameinlast.

28.Thecorrectansweris(A).VALORishighvalue,courage,orbravery.ThesoldiersdefendedtheAlamowithvalor.

29.Thecorrectansweris(D).NONCHALANTmeanswithoutenthusiasmorindifferent.Thestudentwassononchalantaboutherawardthatshedidnoteventellherparents.

30.Thecorrectansweris(E).ALIAISONisalinkinguporaconnection.Theliaisonofalliesfromanumberofcountriesledtothedefeatofenemyforces.

31.Thecorrectansweris(C).Therelationshipisnotofprecisesynonyms,butitisclose.Bothbegandborrowhavetodowithaskforandtake.Bothofferandlendhavetodowithgive.Repayalsohastodowithgive,butitimpliesapreviousactivitynotimpliedintherelationshipofbegandborrow.

32.Thecorrectansweris(A).Onewhoislazyisinert.Onewhoresists,refuses.Therelationshipisoneofcharacteristicsorevensynonyms.

33.Thecorrectansweris(C).Acircleisthebaseofacylinder;atriangleisthebaseofapyramid.Wehaveexplainedthisasapart-to-wholerelationship.Theactualstatementoftheanalogyiswhole-to-part.

34.Thecorrectansweris(B).Thisisatruepart-to-wholeanalogy.Acrocodileispartofalargergroup,reptiles.Akangarooispartofalargergroup,marsupials.

35.Thecorrectansweris(B).Thisisanotherpart-to-wholerelationship.Aquartisroughly

equivalenttoaliter,andamilliliteris ofaliter.Ayardisroughlyequivalenttoa

meter,andamillimeteris ofameter.Choice(A)reversestherelationship.The

otherchoicesdonotmovefrommetrictoAmericanmeasures.

36.Thecorrectansweris(C).Thesearetruesynonyms.

37.Thecorrectansweris(E).Thesearetrueantonyms.

38.Thecorrectansweris(D).Inneithersetarethetermstrueantonyms,buttheyclearlyhaveoppositeconnotations.Choice(E)alsooffersoppositeconnotations,buttheorderofthetermsisreversed.

39.Thecorrectansweris(A).Wildisacharacteristicofwolfasdomesticisacharacteristicofbothdogandcat.Youmustnarrowfurthertochoosethebestanswer.Dogisthedomesticcounterpartofwolf,sodogcreatesthebestanalogy.

40.Thecorrectansweris(A).Thisisapurposerelationship.Ahammerisatoolusedbyacarpenter;anawlisatoolusedbyacobbler.Choice(D)reversestheorderoftoolanditsuser.

41.Thecorrectansweris(D).Thisisapart-to-partrelationship.Bothsubjectandpredicatearepartsofasentence;bothsenatorandrepresentativearepartsofthecongress.(A)isanincorrectanswerbecauseasenator’srelationshiptocongressisthatofpart-to-whole.

42.Thecorrectansweris(A).Thisisanassociationrelationship.Pungentisanadjective

usedtodescribeadegreeofodor.Intenseisanadjectiveusedtodescribeadegreeofemotion.

43.Thecorrectansweris(B).Theanalogyisbasedonsynonyms.

44.Thecorrectansweris(B).Allfourtermsaresynonyms.

45.Thecorrectansweris(E).Therelationshipisoneofassociationorcharacteristic.Exuberantisanadjectiveusedtodescribemood;adroitisanadjectiveusedtodescribemovement.

46.Thecorrectansweris(E).Oppositionleadstodefiance;challengeleadstoexertion.Theactualstatementoftheanalogyiseffectanditscause.

47.Thecorrectansweris(E).Thisisatruecause-and-effectrelationship.Foodpromotesnutrition;lightpromotesvision.Lightdoesnotpromoteanyoftheotherchoices.

48.Thecorrectansweris(B).Therelationshipisthatoftrueantonyms.Thefalsechoicesaresynonymsorpartialantonyms,makingthisaverydifficultanalogyquestion.

49.Thecorrectansweris(A).Thisantonymrelationshipiseasiertoseeataglance.

50.Thecorrectansweris(E).Thismaybeeitherapart-to-wholerelationshiporananalogyofdegree.Eitheracommentispartofaspeechandanoteispartofaletteroracommentismuchshorterthanaspeechandanoteismuchshorterthanaletter.

51.Thecorrectansweris(E).Becareful.Flammableandinflammablearesynonyms;bothmeaneasilyinflamed.Disinterestedmeansimpartial.

52.Thecorrectansweris(E).Thisisapurposerelationship.Atailorfollowsapatterntoconstructapieceofclothing;abuilderfollowsablueprinttoconstructabuilding.

53.Thecorrectansweris(A).Therelationshipissequential.Impeachment(accusation)comesbeforedismissal.Arraignment(accusation)comesbeforeindictment(placementofcharges).

54.Thecorrectansweris(C).Therelationshipisofcharacteristictoanimal.Choice(B)reversestherelationship.

55.Thecorrectansweris(D).Allchoicesexcept(A)involvetheactivityofabodilyorgan,soyoumustthinkfurther.Bothexhalationandperspirationinvolvegivingoffsomethingfromwithinthebody.

56.Thecorrectansweris(A).Youcelebrateabirth;yougrieveoveradeath.Theanalogystatestheeffectanditscause.

57.Thecorrectansweris(A).Thisisananalogyofdegree.Tourgeistorecommendstrongly;topleadistorequeststrongly.Choice(B)offerssynonymsofequaldegree;choice(C)reversestheorder.

58.Thecorrectansweris(E).Thisisacause-and-effectrelationship.Whenoneweeps,onegivesofftears;whenonebreathes,onegivesoffcarbondioxide.

59.Thecorrectansweris(A).Therelationshipishardtocategorizebuteasytospot.Anairpocketmakesaplanebounce;aruthasthesameeffectonavehicle.

60.Thecorrectansweris(A).Thisisaverb-to-nounrelationship.Arbitrateiswhatonedoestoadispute;solveiswhatmustbedonetoamystery.

Section4:Quantitative(Math)

1.Thecorrectansweris(E).Inonehour,theminutehandofaclockgoesaroundinacompletecircle.Intwohours,itrevolvesthroughtwocircles.Becauseeachcircleconsistsof360°,tworevolutionsequal720°.

2.Thecorrectansweris(C).Afractionislessthan ifthreetimesthenumeratorisless

thanthedenominator.Ofthefractionslisted,only hasanumeratorthatislessthan

ofthedenominator.

3.Thecorrectansweris(A).Thefigureisasquare,soallfoursidesareequalinlength.Theperimeteristhesumofthelengthsofthefoursides.

Eachsideis +1.

Thesum,then,is + + + = +4.

Youcouldalsomultiply +1by4forthesameresult.

4.Thecorrectansweris(C).Thefaceofthecubeisasquare,1"by1".UsethePythagoreanTheoremtofindthelengthofthediagonalofthesquare.

c2=a2+b2

c2=12+12

c2=2

c=

5.Thecorrectansweris(D).Theaveragespeedfortheentiretripisthetotaldistance(240miles)dividedbythetotaltime(5hours),whichyields48mph.

6.Thecorrectansweris(C).Thisisaproportionproblem.Setuptheproportionasfollows:

Dividebothsidesbythecoefficientofxandcalculate:

7.Thecorrectansweris(D).Tofindtheratiooftheareaofthecircletotheareaofthe

square,firstfindtheareaofeach.Notethatthediameterofthecircleequalsthewidthofthesquare.

Areaofthesquare=6"×6"=36sq.in.

Areaofcircle=π32=9π=9• = =28sq.in.,approximately.

Ratiooftheareaofthecircletotheareaofthesquare: =

8.Thecorrectansweris(A).Thedifferenceis0.90inches,buttheoutsidediameterconsistsoftwothicknessesofmetal(oneoneachside).Therefore,thethicknessofthemetalis0.90÷2=0.45inches.

9.Thecorrectansweris(C).If60%ofthegameswerepredictedaccurately,40%ofthegameswerepredictedinaccurately.

Letx=gamesplayed0.40x=16x=40gamesplayed40–16=24gameswon

Therefore,thesportswriterwasaccuratefor24games.

10.Thecorrectansweris(E).ThirtypercentoftheboysarefromNewYorkState,and20%ofthem(0.20ofthem)arefromNewYorkCity.Therefore,6%(0.20×0.30)oftheboysinthecamparefromNewYorkCity.

11.Thecorrectansweris(A).Anarea1footlongby1 feetwideis12"×15"or180

squareinchesinarea.Eachblockis6squareinchesinarea.Therefore,thenumberofblocksneededis =30blocks.Theheightofeachblockisirrelevanttothesolution

oftheproblem.

12.Thecorrectansweris(A).Theareaofacircleisequaltoπr2.Thecircumferenceofacircleisequaltoπd.Ifthenumberofinchesineachareequal,thenπd=πr2,orthediameterequalsthesquareoftheradius.Theonlyvalueforwhichthediametercanequalthesquareoftheradiusisadiameterof4inches.

13.Thecorrectansweris(B).TheLCMisfoundbyrewritingeachnumberinprimefactorizationandfindingtheproductofeachuniqueprimefactor.22and23arenotselectedbecauseeachisafactorof25.

20=22•5

24=23•332=25

LCM=5•3•25=480Trialanderrorcanalsogiveyouthisanswer.

14.Thecorrectansweris(B).If9x+5=23,then9x=18,andx=2.Therefore,18x+5equals18(2)+5=41.

15.Thecorrectansweris(D).Fractionsaremosteasilycomparedbycomparingcross-

products.Startbycomparing with .Theproductof3and5is15.Theproductof7and

2is14.Therefore, islargerthan .

Continuethisprocesswiththeotherfractionstobecompared.

,note > andalso > ,note >

,note > andalso <

Therefore, < < < .

16.Thecorrectansweris(E).Acubicfootcontains12"×12"×12",or1728,cubicinches.Ifeachcubicinchweighstwopounds,thesubstanceweighs2•1728,or3456pounds.

17.Thecorrectansweris(B).17millionthsindecimalsis0.000017.Thenumberofplacestotherightofthedecimalpointisequaltothenumberofzerosinthewholenumber.17,000,000hassixzeros.

18.Thecorrectansweris(B).Becausethecircumferenceofacircleisequivalenttoπtimesthediameter,thecircumferenceisalsoequaltoπtimestwicetheradius.Dividethecircumferenceby2π.

19.Thecorrectansweris(C).Fourboards,each2'9"long,total11feet.Thecarpentermustbuy11feetofwood.

20.Thecorrectansweris(B).Findtheanswertothisproblembysubstitutingthevaluesgivenintotheformula.

S=16t2

S=16(8)2=16(64)=1024feet

21.Thecorrectansweris(D).Thegraphissteepestbetweenyears7and9.Thepopulationwasapproximately1000inyear7andincreasedtoover2500byyear9.

22.Thecorrectansweris(A).Thesizeofthepopulationwasquiteconstantfromyear3toyear4anddecreasedfromyear4toyear5fromalmost1000to500.Noticethatthepopulationwasthesameinyear3asinyear7.

23.Thecorrectansweris(B).ByknowinghowmanytelephonesareinAdelaide(48,000),andhowmanyserveeachgroupof100inthepopulation(12.8),wecanfindhowmanygroupsof100areinthepopulation.

48,000telephones÷12.8telephonesper100ofpopulation=3750groupsof100inthepopulation.

3750×100=375,000people

24.Thecorrectansweris(B).Thefirstpersondoes ofthejobin1minute.Thesecond

persondoes ofthejobin1minute.Thethirdpersondoes ofthejobin1minute.

Together: + + = ,or ,ofthejobin1minute.Thisis minutesforthe

entirejob,or minutes.

25.Thecorrectansweris(A).Tofindthenumberofminutesthatmustpassbeforetheeventsnextoccursimultaneously,calculatetheleastcommonmultipleof4,6,and15.TheLCMis60minutes.Iftheeventslastoccurredtogetheratnoon,thenextoccurrencewillthusbe60minuteslater,orat1p.m.

SCOREYOURSELFCheckyouranswersagainsttheanswerkeys.Countupthenumberofanswersyougotrightandthenumberyougotwrong.

SECTION NO.RIGHT NO.WRONG

Quantitative(Math)

ReadingComprehension

Verbal

Nowcalculateyourrawscores:

Quantitative(Math): (________) – (_________) = (_________)

No.Right No.Wrong RawScore

ReadingComprehension: (________) – (_________) = (_________)

No.Right No.Wrong RawScore

Verbal: (________) – (_________) = (_________)

No.Right No.Wrong RawScore

NowcheckyourRawScoreagainsttheconversionchartstogetanideaoftherangeinwhichyourtestscoresfell:

RawScore Quantitative(Math)Reading

Comprehension Verbal

60 350

55 350

50 350 343

45 344 334

40 335 348 325

35 325 324 316

30 316 314 307

25 306 304 298

20 297 294 289

15 288 284 280

10 278 274 271

5 269 264 261

0 260 254 252

–5orlower 250 250 250

Remember:•Thesameexamisgiventostudentsingrades8through11.Youarenotexpectedtoknowwhatyouhavenotbeentaught.

•Youwillbecomparedonlytostudentsinyourowngrade.Useyourscorestoplanfurtherstudyifyouhavetime.

PracticeTest3:SSAT(UpperLevel)

PARTI:WRITINGSAMPLE25Minutes

Directions:Readthetopics,choosetheonethatinterestsyouthemost,andplanyouressayorstorybeforewriting.Writealegibleessay.

TopicA:Highschoolsshouldrequirestudentstomaintainacertaingradepointlevelinordertoplayoncompetitivesportsteams.

Doyouagreeordisagreewiththisstatement?Supportyourpositionwithexamplesfromyourownexperience,theexperienceofothers,currentevents,oryourreading.

TopicB:Itwasicecoldintheroom.

Writeastoryusingthisstatementasthefirstsentence.Besureyourstoryhasaclearbeginning,middle,andend.

PARTII:MULTIPLECHOICESection1:Quantitative(Math)

25Questions•30Minutes

Directions:Calculatetheanswertoeachofthefollowingquestions.Selecttheanswerchoicethatisbest.

1.

(A)1.750

(B)1.854

(C)1.9

(D)2.25

(E)2.35

2.Evaluate: .

(A)1billion

(B)1million

(C)1000

(D)100

(E)13

3.71.4×98.2=

(A)4011.38

(B)5321.48

(C)6921.38

(D)7011.48

(E)8231.48

4. =

(A)9

(B)

(C)

(D)

(E)23

5.(0.25)2=

(A)0.00625

(B)0.0625

(C)0.625

(D)1.625

(E)16.25

6.(3+1)+[(2–3)–(4–1)]=

(A)6

(B)2

(C)0

(D)–2

(E)–4

7.10,001–8093=

(A)1908

(B)1918

(C)2007

(D)18,094

(E)20,007

8.Theratioof3quartsto3gallonsis

(A)3:1

(B)1:4

(C)6:3

(D)4:1

(E)1:3

9.10%of of$50is

(A)$100

(B)$5

(C)$1

(D)103

(E)

10.4hours12minutes10sec.–2hours48minutes35sec.

(A)2hr.23min.25sec.

(B)2hr.12min.40sec.

(C)1hr.23min.35sec.

(D)1hr.23min.25sec.

(E)1hr.12min.35sec.

11.Ifwedoublethevalueofaandcinthefraction ,thevalueofthefractionis

(A)doubled.

(B)tripled.

(C)multipliedby4.

(D)halved.

(E)unchanged.

12.Whatpercentageof220is24.2?

(A)909%

(B)99%

(C)40%

(D)27%

(E)11%

13.98reducedby isequivalentto

(A)28

(B)33

(C)66

(D)70

(E)85

14.Howlongshouldanobject feetlongbedrawn,ifaccordingtothescale, inchinthe

drawingequals1foot?

(A) inches

(B) inches

(C) inches

(D) inches

(E) inches

15.

(A)1

(B)

(C)20

(D)28

(E)

16.Ify+2>10,thenymaybe

(A)largerthan8.

(B)largerthan6.

(C)largerthan0.

(D)equalto0.

(E)unknown.

17.Theshadowofaman6feettallis12feetlong.Howtallisatreethatcastsa50-footshadow?

(A)100'

(B)50'

(C)25'

(D)15'

(E)10'

18.Inthefraction , couldbereplacedbyallofthefollowingEXCEPT

(A)0

(B)1

(C)4.2

(D)9

(E)10

19.0.0515×100isequivalentto

(A)5150÷100

(B)5.15×10

(C)0.00515×1000

(D)510,000÷10

(E)5150÷10,000

20.

m 2=60°

NOTE:Figurenotdrawntoscale.

Whichofthefollowingistrue?

(A)m 1+m 3>180°

(B)m 1>m 3

(C)m 1=m 3

(D)m 1–m 3>m 2

(E)m 1+m 3=120°

21.45isto_____as90isto0.45.

(A)0.225

(B)0.900

(C)4.50

(D)9.00

(E)22.5

22.Ifn= ,then

(A)

(B)3>n>2

(C)n=4.5

(D)4<n<5

(E)n>5

23.Howwouldyoumovealongthenumberlineabovetofindthedifferencebetween4and–6?

(A)FromEtoB

(B)FromAtoD

(C)FromBtoD

(D)FromDtoA

(E)FromBtoE

24.Howmanysixthsaretherein ?

(A)

(B)3

(C)

(D)

(E)6

25.Fourgamesdrewanaverageof36,500peoplepergame.Iftheattendanceatthefirstthreegameswas32,000,35,500,and38,000,howmanypeopleattendedthefourthgame?

(A)36,500

(B)37,000

(C)39,000

(D)40,500

(E)43,000

Section2:ReadingComprehension

40Questions•40Minutes

Directions:Readeachpassagecarefully.Thendecidewhichofthepossibleresponsesisthebestanswertoeachquestion.

Backintheseventeenthcentury,whenAbrahamRyckenownedit,RikersIslandwasatinyspitoflandintheEastRiver.ItbecamepartofNewYorkCityinthe1890sandwasusedasaconvenientplacetodeposittherockandsoildebrisofsubwayconstruction.Later,theislandbecametheendofthelineforthediscardsofcityhouseholds,inalandfilloperationthatwentonuntilRikersIslandreacheditspresentsizeof400acres.

RobertMoses,thenNewYork’sParkCommissioner,waslookingforwaystosupplycityparkswithshadetreesandeliminatetheexpenseofbuyingthemfromcommercialnurseries.Henotedthatweedsgrewprodigiouslyinthelandfill,thoughtthattreesandplantsmightdothesame,andarrangedtoclearafewacresforatrialplanting.In1944,thefirst287shrubsandtreesweretransplantedfromthefledglingnurserytothecity’sparks.Thenurserynowcoverssome115acresoftheisland,andseveralhundredthousandofitsshrubsandtreeshavebeenplantedalongcitystreets,inparks,aroundhousingprojects,andaroundthemallsandpathsoftheUnitedNations.

1.ToobtainplantingsforNewYorkCity,authorities

(A)buythemfromtheUnitedNations.

(B)purchasethemfromcommercialnurseries.

(C)transplantthemfromcity-ownedproperty.

(D)buythemfromRobertMoses.

(E)growtheminCentralPark.

2.RikersIslandiscurrently

(A)115acresinarea.

(B)alandfilloperation.

(C)ownedbyAbrahamRycken.

(D)400acresinarea.

(E)adumpinggroundforsubwaydebris.

3.Thesoiloftheisland

(A)isvolcanic.

(B)wasenrichedbydiscardedrubbish.

(C)wasbroughtinfromcommercialnurseries.

(D)isacombinationofmudandrock.

(E)wasbroughtinonsubways.

4.ThefirstplantingsweretakenfromRikersIsland

(A)adecadeago.

(B)about1890.

(C)intheseventeenthcentury.

(D)quiterecently.

(E)in1944.

Hatmakingwasoneofthefirstindustriestodevelopinthecolonies.Asearlyas1640,Americanhatswereoneofthedomesticproductsusedforbarterandexchange.Hatswerewornbybothmenandwomenforstyle,cleanliness,andprotectionagainsttheweather.Theywerealsoanimportantsymbolofsocialstatus,andthedemandforthemincreasedovertimeasthecoloniesgrew.

Woolwastheprincipalrawmaterialusedbyhatters,butlargenumbersofhatswerealsomadeoffurfeltthatcamefromnativebeaverpelts.Theaveragepriceofawoolhatwasbetween40and80cents,andbeaverhatsrangedfrom$2.50to$3.50.Beaverfurwaslong-lastingandwaterproof,makingitahighlydesiredmaterialforhatconstruction.Highdemandforbeaverfur,however,hadcausedasharpdeclineintheEuropeanbeaverpopulation.BeaversinNorthAmericabecameanimportantsourceoffurfeltforthesepopularhats.

Bythebeginningoftheeighteenthcentury,hatmakinghadbecomeoneofNewEngland’smostimportantindustries.ThecityofDanbury,Connecticutwouldeventuallybecomethelargestcenterofhatmakinginthecolonies,producingasmanyas5millionhatsinasingleyear.

Inthe1730s,hatswerebeingexportedfromthecoloniesinsufficientnumberstoarouseuneasinessamonghattersinEngland.Pressurewasexerted,andParliamentpassedalawprohibitingtheexportofhatsfromonecolonytoanotherandfromanycolonytoGreatBritain.TheHatActof1732placedlimitsonthenumberof

workersAmericanhatmakerscouldhireaswell,whichreducedthecapacityofmanufacturingfirmstomeetAmericanconsumerdemands.

5.Whichtitlebestexpressesthemainideaofthisselection?

(A)“ColonialExports”

(B)“KindsofAmericanHats”

(C)“AnEarlyAmericanIndustry”

(D)“HowColonialHatsWereMade”

(E)“HatmakingAroundtheWorld”

6.AlawrestrictinghatexportswasenactedbyParliamentinresponsetocomplaintsby

(A)colonists.

(B)Englishtaxcollectors.

(C)Englishnoblemen.

(D)citizensofforeigncountries.

(E)Englishhatmakers.

7.TheHatActof1732madeitillegalfor

(A)GreatBritaintoexporthats.

(B)thecoloniestoimporthats.

(C)thehatterstousebeaverfur.

(D)thecoloniestoexporthats.

(E)thecoloniststowearhats.

8.Americanhats

(A)weremadeprincipallyofwool.

(B)didnotsuitcustomersinGreatBritain.

(C)wereanunimportantpartofNewEngland’sindustry.

(D)werenotmadefromfuruntil1730.

(E)weremorestylishthanBritishhats.

9.Accordingtothepassage,beaverfelthatswere

(A)unpopular.

(B)cheaperthanwoolhats.

(C)moreexpensivethanwoolhats.

(D)notexported.

(E)warmerthanwoolhats.

Youknow,ofcourse,thatinChinatheEmperorisaChinaman,andallthepeoplearoundhimareChinamentoo.Ithappenedagoodmanyyearsago,butthat’sjustwhyit’sworthwhiletohearthestory,beforeitisforgotten.TheEmperor’spalacewasthemostsplendidintheworld;entirelyandaltogethermadeofporcelain,socostly,butsobrittle,sodifficulttohandlethatonehadtobeterriblycareful.Inthegardenweretobeseenthestrangestflowers,andtothemostsplendidofthemsilverbellsweretied,whichtinkledsothatnobodyshouldpassbywithoutnoticingtheflowers.Oh,theEmperor’sgardenhadbeenlaidoutverysmartly,anditextendedsofarthatthegardenerhimselfdidn’tknowwheretheendwas.Ifyouwentonandon,youcameintotheloveliestforestwithhightreesanddeeplakes.Theforestwentrightdowntothesea,whichwasblueanddeep;tallshipscouldsailrightinunderthebranchesofthetrees;andinthetreeslivedanightingale,whichsangsosweetlythateventhepoorfisherman,whohadmanyotherthingstodo,stoppedstillandlistenedwhenhehadgoneoutatnighttotakeuphisnetsandthenheardthenightingale.

—fromTheNightingale,byHansChristianAndersen

10.Theauthorwantstotellthisstory

(A)becausehecan’tforgetthenightingale.

(B)beforeitisforgotten.

(C)toteachusaboutChina.

(D)becauseheisawriterandstoryteller.

(E)inordertodescribethegarden.

11.TheEmperor’spalacewasmadeof

(A)brick.

(B)silverbells.

(C)hightrees.

(D)largestonesandboulders.

(E)porcelain.

12.Silverbellsweretiedtoflowersinthegardento

(A)drawattentiontotheirbeauty.

(B)frightenbirdsandmiceaway.

(C)playsoftmelodies.

(D)remindthegardenernottopickthem.

(E)sparkleinthesun.

13.TheEmperor’sgarden

(A)wasverystrange.

(B)wastoolargetocarefor.

(C)ledintoalovelyforest.

(D)housedararenightingale.

(E)wasasourceofpleasureforallinthekingdom.

14.Theforest

(A)wasdarkandthreatening.

(B)containedmanyrareanimals.

(C)wasaneasyplaceinwhichtogetlost.

(D)housedthenightingale.

(E)wasafisherman’shidingplace.

TheuseofwoodasamaterialfromwhichtomakepaperwasfirstsuggestedbyRenédeRéaumer,acelebratedFrenchnaturalist,in1719.Réaumerhadobservedwaspsastheybuilttheirnests,andheconcludedthatthewoodfilamentsusedbytheseinsectstoconstructtheirpaper-likenestscouldalsobeusedinthemanufactureofpaper.

Waspslookfordrywood,whichtheysaworraspintheirjaws.Thismaterialisthenmixedwithaglueysubstancesecretedbythewasp’sbodytomakeapastethat,whenspread,becomesthepapersubstanceofthenest.Ineffect,waspsareabletocreatepaperpulpintheirmouths.Nestsaretypicallymadefromanumberofdifferenttypesofwood,dependingonthespeciesofthewaspsandtheirlocation.

Waspnestsareexceptionallylightweight,darkincolor,andboundrepeatedlybybandsofpapertotheplacewheretheyaresuspended.Thenestsarenearlywaterproofbecauseoftheirroundedtopsandthefactthatthepaperstripsoverlapliketheshinglesontheroofofahouse.Thenestsprovideshelterfornewlyhatchedwaspsandensurethattheoffspringhaveadultwaspsnearbytoprotectthem.

Waspnestsaredifferentinsizeandstructure,dependingonwhethertheyarebuiltbysolitarywaspsorsocialwasps.Solitarywaspstendtohavesmallnestsdesignprimarilyforcaringforyoung.Socialwaspsbuildlargernestsandwilldefendtheminaswarmifthreatened.Thenestsofsocialwaspscanrangeinsizefromthesizeofahumanhandtomuchlarger.UpuntilApril2013,thelargestwaspnestonrecordwasaNewZealandnestmeasuring12feetlong.Thisrecordwasbrokenin2013bythediscoveryofa22-footwaspnestinsideanabandonedhouseinSpain.

15.Thewordfilamentsprobablymeans

(A)largechunks.

(B)waves.

(C)files.

(D)dust.

(E)threadlikepieces.

16.Themanwhofirstsuggestedmakingpaperfromwoodwasa

(A)farmer.

(B)industrialist.

(C)lumberjack.

(D)naturalist.

(E)painter.

17.Accordingtothepassage,awaspnestisusually

(A)attachedtoahouse.

(B)exceptionallylightweight.

(C)waterproofedwithaglueysubstance.

(D)constructedwithchunksofwood.

(E)brightincolor.

18.Theprimarymaterialusedbywaspsinnestbuildingis

(A)dirt.

(B)water.

(C)paste.

(D)wood.

(E)leaves.

19.Waspnestsarenearlywaterproofbecause

(A)theyareconstructedofheavymaterials.

(B)theyhangfromtheboughsoftrees.

(C)theirstripsofpaperoverlaplikeshingles.

(D)bandsmadeofrubberareplacedaroundthem.

(E)theyhavelong,straighttopsthatstoptherain.

ThepolicedepartmentofNewYorkCityhasonebranchthatmanydonotknowabout,althoughitwasestablishedalmostacenturyago.Thisistheharborprecinct’s14-boatfleetofpolicelaunches,whichpatrols578milesofwatersaroundthecity,payingparticularattentiontotheareascontaining500piersandsome90boatclubs.

Theboatsareequippedforvariousjobs.Oneboatisanice-breaker;anotherisequippedtorenderaidintheeventofanairplanecrashatLaGuardiaAirport.Alloftheboatsareequippedwithlifelineguns,heavygrapplingironstoraisesunkenautomobiles,andlasso-stickstorescueanimalsinthewater.Theyhavepowerpumpstobailoutsinkingcraft,first-aidkits,extralifepreservers,signalflags,andsearchlights.

Theforceof183officershaveallhadpreviousexperiencewithboats.SomeoftheofficersareNavyandCoastGuardveterans.Manyoftheharborpoliceofficershaveocean-goingMaster’sorHarborCaptain’slicenses.Allarehighlytrainedinthecareandhandlingofenginesandinnavigation.Allareskilledingivingfirstaid,andeachofficerisaqualifiedradiooperatorandatrainedmarksmanwitharevolver.

Theworkofthepoliceincludesmanytasks.Onedutyofthisforceistochecktheoperationofthefleetof43junkboatsthatplytheirtradeintheharbor,buyingscrap,rope,andotheritemsforresaleashore.Theseboatscouldjustaseasilybeusedtosmugglenarcotics,gems,aliens,orspiesintothecountry,sotheyarewatchedcloselybythecity’sharborpoliceforce.Duringthelastsummer,thepolicelaunchestowed450disabledboatsandgavesomekindofhelptothousandsofothers.Theofficersalsoarrestthosewhobreaknavigationlawsorwhoendangerthesafetyofbathersbyapproachingtooneartheshoreinspeedboats.

20.Theharborpolicewere

(A)introducedbyorderofthemayor.

(B)firstusedinthetwentiethcentury.

(C)inusebeforetheCivilWar.

(D)introducedbyveteransofWorldWarII.

(E)infullforcealmost100yearsago.

21.Theboatsused

(A)areuniformindesign.

(B)canallserveasice-breakers.

(C)areallequippedwithdeckguns.

(D)workatKennedyAirport.

(E)varyinfunction.

22.Theharborpolice

(A)arrestanymanfoundonajunkboat.

(B)preventtheresaleofscrapmaterial.

(C)regulatetheadmissionofspies.

(D)ensurelegaltrafficinjunk.

(E)regulatedisabledboats.

23.Theirservicesinclude

(A)towing,life-saving,andsalvage.

(B)customscollection,towing,andthesendingofradiomessages.

(C)firstaid,therescueofanimals,andfirepatrol.

(D)ice-breaking,thecollectionofjunk,andthetransportationofaliens.

(E)smuggling,firstaid,andrescue.

24.Thepoliceboats

(A)havenoresponsibilityforbathers.

(B)unloadshipsatthepiers.

(C)assistboatsofallkinds.

(D)warnoffendersbutdonotmakearrests.

(E)cannotdetainotherboats.

“TherearemanythingsfromwhichImighthavederivedgood,bywhichIhavenotprofited,Idaresay,Christmasamongtherest.ButIamsureIhavealwaysthoughtofChristmas time,whenithascomeround—apartfromthevenerationduetoitssacredorigin,ifanythingbelongingtoitcanbeapartfromthat—asagoodtime;a

kind,forgiving,charitable,pleasanttime;theonlytimeIknowof,inthelongcalendaroftheyear,whenmenandwomenseembyoneconsenttoopentheirshut-upheartsfreelyandtothinkofpeoplebelowthemasiftheyreallywerefellowtravelerstothegrave,andnotanotherraceofcreaturesboundonotherjourneys.Andtherefore,Uncle,thoughithasneverputascrapofgoldorsilverinmypocket,Ibelievethatithasdonemegood,andwilldomegood;andIsay,Godblessit!”

Theclerkinthetankinvoluntarilyapplauded.

“Letmehearanothersoundfromyou,”saidScrooge,“andyou’llkeepyourChristmasbylosingyoursituation!You’requiteapowerfulspeaker,sir,”headded,turningtohisnephew.“Iwonderyoudon’tgointoParliament.”

—fromAChristmasCarol,byCharlesDickens

25.Thewordvenerationprobablymeans

(A)worship.

(B)disapproval.

(C)agreement.

(D)love.

(E)participation.

26.Thefirstspeaker

(A)isaveryreligiousperson.

(B)enjoysandcelebratesChristmas.

(C)isdefendingChristmas.

(D)hasbeenfiredbyScrooge.

(E)isobviouslyfrightenedofScrooge.

27.ThefirstspeakerbelievesthatChristmas

(A)isapleasantnuisance.

(B)isanexcuseforpeopletothrowwildparties.

(C)hasbeenseparatedfromitsreligiousorigin.

(D)couldbeaprofitabletimeofyear.

(E)bringsoutthebestinpeople.

28.Thephrase“byoneconsent”issynonymouswith

(A)affirmation.

(B)reaffirmation.

(C)partially.

(D)unanimously.

(E)contractual.

29.Scroogeprobablyisangrywith

(A)thespeakerandtheclerk.

(B)onlythespeaker.

(C)onlytheclerk.

(D)peoplewhocelebrateChristmas.

(E)noone.

Onedayrecently,amaninaten-gallonhatappearedatthegateofNewYork’sfamousBronxZoo.“Juststoppedbyonmywaythroughtown,”hetoldzooofficials.“I’vegotananimaloutsideIthinkyoumightliketosee.”

Theofficialsraisedtheireyebrowsandlookedateachothermeaningfully,butthemaninthehatdidn’tseemtonotice.HewentontointroducehimselfasGeneHolter.“IcallitaZonkey,”hesaidcalmly,“becauseit’sacrossbetweenadonkeyandazebra.I’vegothisparentsoutthere,too.”

Thezooofficialsdidn’twaittohearabouttheparents.Theylefttheirdesksandstartedforthegate.Outside,Mr.Holteropenedthesidedoorofahugetruckandreachedinside.Calmly,hepulledoutagibbon,andhungit,byitstail,fromatree.ThenhewalkedpastfiveostrichesandcarriedoutthebabyZonkey.

Justthreeweeksold,theonlyZonkeyintheworldhadlongears,afaceandlegscoveredwithcandystripes,andabodycoveredwithbrownbabyfuzz.Theparentswereonhand,too.Thefatherwasnoordinaryzebra.Hewasbrokentoride,andoneofthezooofficialsrealizedalifelongdreamwhenhejumpedonthezebra’sbackandcanteredaround.

Whenlastseen,Mr.HolterandhiscaravanwereontheirwaytoDaytonandthentoAnaheim,California,wheretheyliveyear-round.

30.Mr.Holter’smannerwas

(A)boastful.

(B)excitable.

(C)demanding.

(D)matter-of-fact.

(E)personable.

31.WhenMr.Holterfirstapproachedthezooofficials,they

(A)wereexcitedabouthisannouncement.

(B)thoughthewastellingatalltale.

(C)thoughthewasaninterestingperson.

(D)couldn’twaittorealizealifelongdream.

(E)laughedbehindhisback.

32.Mr.Holterprobablymadealiving

(A)asaveterinarian.

(B)travelingandshowinghisanimals.

(C)breedinganimalsforscientificexperiments.

(D)workingasazooofficial.

(E)filminganimals.

Thereisevidencethattheusualvarietyofhighbloodpressureis,inpart,afamilialdisease.Sincefamilieshavesimilargenesaswellassimilarenvironment,familialdiseasescouldbeduetosharedgeneticinfluences,tosharedenvironmentalfactors,orboth.Forsomeyears,theroleofoneenvironmentalfactorcommonlysharedbyfamilies,namelydietarysalt,hasbeenstudiedatBrookhavenNationalLaboratory.Thestudiessuggestthatexcessiveingestionofsaltcanleadtohighbloodpressureinmanandanimals.Someindividualsandsomerats,however,consumelargeamountsofsaltwithoutdevelopinghighbloodpressure.Nomatterhowstrictlyallenvironmentalfactorswerecontrolledintheseexperiments,somesalt-fedanimalsneverdevelopedhypertension,whereasafewrapidlydevelopedveryseverehypertensionfollowedbyearlydeath.Thesemarkedvariationswereinterpretedtoresultfromdifferencesingeneticmakeup.

33.Themainideaofthisarticleisthat

(A)researchisdesperatelyneededinthefieldofmedicine.

(B)acureforhighbloodpressureisnear.

(C)researchshowssalttobeamajorcauseofhighbloodpressure.

(D)atendencytowardhighbloodpressuremaybeinherited.

(E)someanimalsneverdevelophighbloodpressure.

34.Accordingtothearticle,highbloodpressureis

(A)strictlyageneticdisease.

(B)strictlyanenvironmentaldisease.

(C)duetobothgeneticandenvironmentalfactors.

(D)causedonlybydietarysalt.

(E)amoresevereformofhypertension.

ThedarkandtheseaarefullofdangerstothefishermenofNorway.Awhalemaycomeanddestroythefloatingchainofcorksthatedgethenets,breakit,andcarryitoff.Orastormmaycomesuddenly,unexpectedly,outofthenight.Theseaseemstoturnsomersaults.Itopensandclosesimmensecavernswithterribleclashes,chasingboatsandmenwhomustfleefromtheirnetsandtheexpectedcatch.Thenthemenmaylifttheirnetsasemptyastheysetthem.Atothertimestheherringmaycomeinsuchmassesthatthelinesbreakfromtheweightwhenlifted,andthemenmustreturnhomeempty-handed,withoutlines,nets,ortheherring.

Butoftenthenetsarefullofherringthatshineandglistenlikesilver.Onceinawhile,acoupleofmenwillventureintheirboatsalongthenetlinestoseewhethertheherringarecoming,andwhenthecorksbegintobobandjerk,asifsomethingwerehittingthenetstowhichtheyareattached,thentheyknowthattheherringarethere.Thenetsarebeingfilled,andallthemensitinquietexcitement.Theydareonlytowhispertoeachother,afraidtodisturb,andquiteovercomebytheoverwhelminggenerosityofthesea.Eyesshineinhappyanticipation;handsarefoldedinthanks.Thenmusclesstrainwithpower.Itisasthoughthestrengthofthebodydoubled.Theycanworkdayandnightwithoutathoughtofweariness.Theyneedneitherfoodnorrest;thethoughtofsuccesskeepstheirvigorupalmostendlessly.Theywilltakefoodandrestwhenitisallover.

35.Whatisthebesttitleforthispassage?

(A)“WhalinginNorway”

(B)“ThePerilsandRewardsofFishing”

(C)“HardWorkinNorway”

(D)“RiskyBusiness”

(E)“TheGenerosityoftheSea”

36.Thefishermen’sdifficultiesinclude

(A)theeatingoftheherringbywhales.

(B)becalming.

(C)anattackonthemenbytheherring.

(D)thejerkingofthecorks.

(E)interferencebyroughseas.

37.Atthefirstindicationthatherringareenteringthenets,themen

(A)trynottofrightenthefishaway.

(B)straineverymuscletohaulinthecatch.

(C)glistenlikesilver.

(D)collectthenetsquickly.

(E)rowquicklyalongtheedgeofthenets.

38.Whichqualityoftheseaisnotmentioned?

(A)Itssuddenchanges

(B)Itsgenerosity

(C)Itsbeauty

(D)Itspower

(E)Itsdestroyingstrength

39.Thefishermenaredescribedas

(A)patient,brave,andcautious.

(B)angry,weary,andsickly.

(C)strong,angry,andreckless.

(D)skillful,impatient,andweary.

(E)hardworking,surly,andexcitable.

40.Whichisnotmentionedasaproblemtofishermen?

(A)Destructionofthenets

(B)Toolargeacatch

(C)Roughseas

(D)Unexpectedstorms

(E)Theftofthenetsbyother

fishermen

Section3:Verbal

60Questions•30Minutes

TheVerbalsectionconsistsoftwodifferenttypesofquestions.Therearedirectionsforeachtypeofquestion.

Directions:Eachquestionshowsawordincapitallettersfollowedbyfivewordsorphrases.Choosethewordorphrasewhosemeaningismostsimilartothewordincapitalletters.

1.AGENDA

(A)receipt

(B)agent

(C)combination

(D)correspondence

(E)schedule

2.CREDIBLE

(A)believable

(B)untrue

(C)correct

(D)suitable

(E)fortunate

3.PLACID

(A)explosive

(B)quiet

(C)public

(D)lenient

(E)crystalline

4.INTERVENE

(A)induce

(B)invert

(C)interfere

(D)solve

(E)intermediary

5.MUNDANE

(A)stupid

(B)extraordinary

(C)weekly

(D)immense

(E)common

6.DEHYDRATED

(A)airless

(B)deflated

(C)pointless

(D)worthless

(E)waterless

7.PREVALENT

(A)predating

(B)predominant

(C)preeminent

(D)prior

(E)predictive

8.SUCCINCT

(A)concise

(B)superfluous

(C)alert

(D)despicable

(E)fearful

9.NOCTURNAL

(A)bynight

(B)byday

(C)revolving

(D)alternating

(E)frequent

10.EQUITABLE

(A)preferential

(B)fair

(C)unreasonable

(D)biased

(E)prejudiced

11.EXPEDITE

(A)hinder

(B)harm

(C)send

(D)hasten

(E)block

12.TURBULENT

(A)authentic

(B)tranquil

(C)tamed

(D)fatal

(E)violent

13.TENACIOUS

(A)timid

(B)thin

(C)unyielding

(D)divisive

(E)stranded

14.PERTINENT

(A)applicable

(B)prudent

(C)irreverent

(D)irrelevant

(E)truthful

15.DOGMATIC

(A)bovine

(B)canine

(C)opinionated

(D)individualistic

(E)traditional

16.UNSCRUPULOUS

(A)filthy

(B)honest

(C)austere

(D)unprincipled

(E)unresolved

17.WILY

(A)crooked

(B)narrow

(C)cunning

(D)blunt

(E)broken

18.BLATANT

(A)insipid

(B)obvious

(C)shining

(D)closed

(E)secret

19.PRETEXT

(A)excuse

(B)reason

(C)preface

(D)fit

(E)doubt

20.ACUMEN

(A)beauty

(B)poise

(C)keenness

(D)illness

(E)courtesy

21.EVASION

(A)attack

(B)displeasure

(C)enjoyment

(D)avoidance

(E)fatigue

22.INDISPENSABLE

(A)incontrovertible

(B)essential

(C)impetuous

(D)ungovernable

(E)confused

23.OBLITERATE

(A)obligate

(B)subjugate

(C)exhibit

(D)maintain

(E)erase

24.AMIABLE

(A)allied

(B)disjointed

(C)indignant

(D)friendly

(E)introverted

25.WRITHE

(A)strangle

(B)topple

(C)trouble

(D)slide

(E)twist

26.ABATE

(A)letup

(B)continue

(C)forego

(D)placate

(E)intimidate

27.ENDORSEMENT

(A)inscription

(B)approval

(C)standard

(D)editorial

(E)article

28.CONVERT

(A)reform

(B)predict

(C)weave

(D)transform

(E)translate

29.ERUDITE

(A)knowledgeable

(B)meddlesome

(C)eroded

(D)careless

(E)intrusion

30.ENDEAVOR

(A)expectation

(B)attempt

(C)tack

(D)necessity

(E)ability

Directions:Thefollowingquestionsaskyoutofindrelationshipsbetweenwords.Readeachquestion,andthenchoosetheanswerthatbestcompletesthemeaningofthesentence.

31.Noneistolittleasneveristo

(A)nothing

(B)infrequently

(C)negative

(D)much

(E)often

32.Receiveistoadmitassettleisto

(A)resist

(B)anger

(C)remain

(D)adjust

(E)mediate

33.Dishonestyistodistrustas

(A)violinistobow

(B)handistopaper

(C)moneyistothief

(D)strangeistoodd

(E)carelessnessistoaccident

34.Sociologististogroupas

(A)psychologististoindividual

(B)doctoristonurse

(C)childrenistopediatrician

(D)biologististofrog

(E)mathematicianistoalgebra

35.Generousistofrugalas

(A)wastefulistosquander

(B)philanthropististomiser

(C)tastefulistogarish

(D)gratifyistodesire

(E)importantistononessential

36.Transparentistotranslucentas

(A)wateristomilk

(B)glassistocrystal

(C)translucentistoopaque

(D)muddyistoclear

(E)suspensionistomixture

37.Discontentistorebellionas

(A)frictionistospark

(B)complacentistorevolt

(C)successistostudy

(D)employmentistoretirement

(E)surgeonistooperation

38.Beakeristochemistashammeristo

(A)nails

(B)geologist

(C)construction

(D)architect

(E)noise

39.Followistoleadasdependentisto

(A)subservient

(B)supportive

(C)child

(D)autonomous

(E)anonymous

40.Stateistocountryascountryisto

(A)island

(B)capitol

(C)continent

(D)planet

(E)ocean

41.Acceleratoristomotionas

(A)catalystistochange

(B)inertiaistoimmobile

(C)ignitionistospeed

(D)automobileistovehicle

(E)experimentistohypothesis

42.Probableistocertainas

(A)approachistoreproach

(B)steadyistorocky

(C)correctistoaccurate

(D)saveistorecord

(E)plausibleistodefinite

43.Obstructistoimpedeasimpenetrableisto

(A)impervious

(B)hidden

(C)merciful

(D)porous

(E)transparent

44.Includeistoomitasacknowledgeisto

(A)notice

(B)ignore

(C)recognize

(D)greet

(E)know

45.Nucleusistoelectronas

(A)Earthistosatellite

(B)EarthistoSun

(C)constellationistoSun

(D)neutronistoproton

(E)atomistoneutron

46.Sculptoristostatueas

(A)actoristoplay

(B)paintistoartist

(C)composeristomusic

(D)orchestraistoconductor

(E)programmeristocomputer

47.Drearyistohappyas

(A)lightistograceful

(B)closeistonarrow

(C)dearthistosurplus

(D)curtainistoplay

(E)interdictistoexpect

48.Allowistorestrictas

(A)gainistosuccess

(B)seeingistobelieving

(C)heartistosoul

(D)encourageistoprevent

(E)terribleistoworse

49.Interruptistospeakas

(A)telephoneistotelegraph

(B)interfereistoassist

(C)shoutistoyell

(D)intercedeistointerfere

(E)intrudeistoenter

50.Modestyistoarroganceas

(A)debilityistostrength

(B)causeistopurpose

(C)hateistoemotion

(D)financeistopoverty

(E)agilityistostamina

51.Adversityistohappinessas

(A)fearistomisfortune

(B)solaceistosorrow

(C)graduationistosuperfluous

(D)vehemenceistoserenity

(E)troublesomeistopetulant

52.Extortionististoblackmailas

(A)kleptomaniacistosteal

(B)criminalistoarrest

(C)kidnapperistocrime

(D)businessmanistoprofit

(E)clerkistostock

53.Monsoonistorainas

(A)hurricaneistodestruction

(B)tornadoistowind

(C)sunistospring

(D)famineistodrought

(E)morningistodew

54.Introspectiveistowithdrawnas

(A)hesitantistohasty

(B)quickistofeelings

(C)introvertistoextrovert

(D)importistoexport

(E)gregariousistosocial

55.Equatoristoworldas

(A)boundaryistocountry

(B)capitalistostate

(C)furistoanimal

(D)waististoman

(E)latitudeistolongitude

56.Superficialistosurfaceas

(A)probingistodeep

(B)subwayistosubterranean

(C)crustistoEarth

(D)tepidistowarm

(E)internalistoexternal

57.Stagnantistopondas

(A)sandyistoriver

(B)noisyistosheep

(C)flowingistostream

(D)oceanicistotide

(E)tidalistowave

58.Sanctuaryistofortressas

(A)sanctumistoinner

(B)shelteristohouse

(C)violentistopeaceful

(D)gunsistofort

(E)sanctionistoassassinate

59.Mentoristoprofessoras

(A)advisoristocounselor

(B)childistoparent

(C)learningistoteacher

(D)mentalistophysical

(E)toothistodentist

60.Lucidistoclearas

(A)sullenistogloomy

(B)furtiveistoclever

(C)potentistoweak

(D)drollistoserious

(E)pensiveistohanging

Section4:Quantitative(Math)

25Questions•30Minutes

Directions:Eachquestionbelowisfollowedbyfivepossibleanswers.Selecttheonethatisbest.

1.Intwodaysapointontheearth’ssurfacerotatesthroughanangleofapproximately

(A)90°

(B)180°

(C)360°

(D)480°

(E)720°

2.Whichofthefollowinggroupsisarrangedinorderfromsmallesttolargest?

(A)

(B)

(C)

(D)

(E)

3.Therectanglebelowhasalengthtwiceaslongasitswidth.Ifitswidthisx,itsperimeteris

(A)6

(B)2x2

(C)4x

(D)6x

(E)8x

4.Thissquarehasasideof1".Thediagonaldistancefromonecornertoanotheris

(A)1inch.

(B) inches.

(C) inches.

(D)2inches.

(E)3inches.

5.Aplumberneedseightsectionsofpipe,each3'2"long.Ifpipeissoldonlybythe10'section,howmanysectionsmusthebuy?

(A)1

(B)2

(C)3

(D)4

(E)5

6.Theratiooftheareaoftheshadedparttotheunshadedpartis

(A)

(B)2:1

(C)1:3

(D)1:2

(E)3:1

7.Anairplaneonatransatlanticflighttook4hours20minutestogetfromNewYorktoitsdestination,adistanceof3000miles.Toavoidastorm,however,thepilotwentoffhiscourse,addingadistanceof200milestotheflight.Approximatelyhowfastdidtheplanetravel?

(A)640mph

(B)710mph

(C)738mph

(D)750mph

(E)772mph

8.Aphotographmeasuring5"wide×7"longmustbereducedinsizetofitaspace4incheslonginanadvertisingbrochure.Howwidemustthespacebesothatthepictureremainsinproportion?

(A)

(B)

(C)

(D)

(E)

9.Thetotalareaoftheshadedpartofthefigureis

(A)

(B)

(C)

(D)

(E)

10.AcertainpopulationofmicrobesgrowsaccordingtotheformulaP=A×2n,wherePisthefinalsizeofthepopulation,Aistheinitialsizeofthepopulation,andnisthenumberoftimesthepopulationreproducesitself.Ifeachmicrobereproducesitselfevery20minutes,howlargewouldapopulationofonlyonemicrobebecomeafter4hours?

(A)16

(B)64

(C)128

(D)1028

(E)4096

11.Ifxisapositivenumberand ,asxincreasesinvalue,whathappenstoy?

(A)yincreases

(B)ydecreases

(C)yisunchanged

(D)yincreasesthendecreases

(E)ydecreasesthenincreases

12.Aboxwasmadeintheformofacube.Ifasecondcubicalboxhasinsidedimensionsthreetimesthoseofthefirstbox,howmanytimesasmuchdoesitcontain?

(A)3

(B)9

(C)12

(D)27

(E)33

13.Mr.Adamshasacircularflowerbedwithadiameterof4feet.Hewishestoincreasethesizeofthisbedsothatitwillhavefourtimesasmuchplantingarea.Whatmustbethediameterofthenewbed?

(A)6feet

(B)8feet

(C)12feet

(D)16feet

(E)20feet

14.AtrainleftAlbanyforBuffalo,adistanceof290miles,at10:10a.m.ThetrainwasscheduledtoreachBuffaloat3:45p.m.Iftheaveragerateofthetrainonthistripwas50mph,itarrivedinBuffalo

(A)about5minutesearly.

(B)ontime.

(C)about5minuteslate.

(D)about13minuteslate.

(E)morethan15minuteslate.

15.If3x–2=13,whatisthevalueof12x+20?

(A)5

(B)20

(C)30

(D)37

(E)80

16.Abakeryshopsoldthreekindsofcake.Thepricesofthesewere25¢,30¢,and35¢perpound.Theincomefromthesesaleswas$36.Ifthenumberofpoundsofeachkindofcakesoldwasthesame,howmanypoundsweresold?

(A)120pounds

(B)90pounds

(C)60pounds

(D)45pounds

(E)36pounds

17.Howmanymore9"×9"linoleumtilesthan1'×1'tileswillittaketocovera12'×12'floor?

(A)63

(B)98

(C)112

(D)120

(E)144

18.Ifppencilscostccents,npencilsatthesameratewillcost

(A) cents.

(B) cents.

(C)npccents.

(D) cents.

(E)n+p+ccents.

19.Which,ifany,ofthefollowingstatementsisalwaystrue?

(A)Ifthenumeratoranddenominatorofafractionareincreasedordecreasedbythesameamount,thevalueofthefractionisunchanged.

(B)Ifthenumeratoranddenominatorofafractionaresquared,thevalueofthefractionisunchanged.

(C)Thesquareofanynumberisgreaterthanthatnumber.

(D)Ifunequalquantitiesareaddedtounequalquantities,thesumsareunequal.

(E)Noneoftheabove

20.Ifthelengthandwidthofarectangleareeachdoubled,bywhatpercentistheareaincreased?

(A)50%

(B)75%

(C)100%

(D)300%

(E)400%

21.Ifonepipecanfillatankin hours,andanothercanfillthesametankin45minutes,

howlongwillittakeforthetwopipestofillthetanktogether?

(A) hour

(B) hour

(C) hour

(D)1hour

(E) hours

22.Abaseballteamhaswon50gamesoutof75played.Ithas45gamesstilltoplay.Howmanyofthesemusttheteamwintomakeitsrecordfortheseason60%?

(A)20

(B)22

(C)25

(D)30

(E)35

23.If9millionbarrelsofoilareconsumeddailyintheUnitedStates,howmanybarrelsarerequiredtomeetcommercialandindustrialneeds?

(A)2,840,000

(B)3,420,000

(C)4,750,000

(D)5,670,000

(E)7,400,000

24.Arealestateinvestorbuysahouseandlotfor$44,000.Hepays$1250tohaveitpainted,$1750tofixtheplumbing,and$1000forgradingadriveway.Atwhatpricemusthesellthepropertyinordertomakea12%profit?

(A)$53,760

(B)$52,800

(C)$52,000

(D)$49,760

(E)$44,480

25.Ifa=1,b=2,c=3,andd=5,thevalueof is

(A)2

(B)3.5

(C)4

(D)

(E)50

ANSWERSANDEXPLANATIONSPartI:WritingSample

Exampleofawell-writtenessay.

Icanunderstandwhysomeschoolsrequirestudentstomaintaintheirgradesiftheywanttobeinsports.Sportsaretimeconsumingandcutintostudytime.But,Ithinkthatlesscompetentstudentsshouldnotbedeprivedofthebenefitsofsportsparticipation.

Theargumentthatstudentsshouldkeepuptheirgradesiftheywanttobeinsportsisworthlisteningto.Afterall,thepurposeofgoingtoschoolistogetaneducation.Andsportspracticeandgamesdotakealotoftime.Thegradesofafewstudentsmightinfactsufferfromsportsparticipation,butIthinkthatmorestudentswillworkharderandwilllearntomanagetimebetteriftheyareallowedtoplayontheteam.Learningtoorganizetimeisalsoanimportantlessontobegainedfrom

school.Happypeopletendtoreachtomeetexpectations,andlesscapablestudentsmayevendobetterinschooltoprovethatbeinginsportsdidnotdothemanyharm.

Anequallygoodargumentisthateveryonemustsucceedatsomething.Ifapoorstudentcanexcelatsports,thatstudentwilldevelopself-esteem.Oncethatstudentfeelsgoodabouthimselforherself,thestudentmaytransferthatconfidencetoschoolworkandactuallygetbettergrades.Theoldadagethatsuccessbreedssuccessapplieshere.

Whiletheattitudethatschoolworkcomesfirstdoesmakeagoodpoint,Ithinkthatpermittingastudenttoparticipateinsportsandtodevelopagoodself-imageismoreimportant.Theschoolshouldgiveextrahelptothelesscompetentstudent,especiallyhelpinlearningtimemanagement.Thenitshouldletthatstudentcontributetoschoolspiritontheplayingfieldsaswellasintheclassroom.

PartII:MultipleChoice

Section1:Quantitative(Math)

1.Thecorrectansweris(B).Rename asadecimal: =0.6.

0.6+1.25+0.004=1.854

2.Thecorrectansweris(C).

=106–3=1000

or106=1,000,000and103=1000Therefore,1,000,000÷1000=1000

3.Thecorrectansweris(D).

4.Thecorrectansweris(D).Simplifythenumerator.

Proceedasyouwouldtodivideanyfraction:

5.Thecorrectansweris(B).(0.25)2=0.25×0.25=0.0625

6.Thecorrectansweris(C).Beginwiththeinnermostgroupandworkoutward:(3+1)+[(2–3)–(4–1)]

=(3+1)+[(–1)–(3)]

=(3+1)+[–1–3]

=(3+1)+[–4]

=4+[–4]=0

7.Thecorrectansweris(A).Trytoestimatetheanswerratherthancalculate:

8.Thecorrectansweris(B).3gallonscontain12quarts.Theratiois3quarts:12quarts,or,insimplestform,1:4.

9.Thecorrectansweris(C).Onefifthof$50is$10.Tenpercent,or ,of$10is$1.

10.Thecorrectansweris(C).Borrow1minutefromtheminutescolumn,and1hourfromthehourscolumn.Thensubtract:

11.Thecorrectansweris(E).Bydoublingthesizeofoneofthefactorsofthenumeratorandthesizeofthedenominator,wedonotchangethevalueofthefraction.Weareactuallywritinganequivalentfraction.Trythiswithfractionshavingnumericalvaluesforthenumeratoranddenominator.

12.Thecorrectansweris(E).Thisisagoodproblemforestimation.Notethat10%of220

=22.Onepercentof220=2.2and24.2=22(10percent)+2.2(1percent).Or, =

0.11.

13.Thecorrectansweris(A).Becareful.Thisproblemasksyouto

reduce98by .Inotherwords,find of98.

14.Thecorrectansweris(B).Since1footcorrespondsto inchinthedrawing,the

drawingshouldbe6 incheslong.

15.Thecorrectansweris(D).Bracketthemultiplicationanddivisionoperationsfromlefttoright.Thencalculate.

=[25]+[6]–3

=28

16.Thecorrectansweris(A).Sincey+2>10,y>10–2,ory>8.

17.Thecorrectansweris(C).Thisisasimpleproportion.Amancastsashadowtwiceaslongashisheight.Therefore,sodoesthetree.Therefore,atreethatcastsashadow50'longis25'high.

18.Thecorrectansweris(A).Thedenominatorofafractioncanneverbeequivalenttozero.Divisionbyzeroisundefinedinmathematics.

19.Thecorrectansweris(C).0.0515×100=5.15,andsodoes0.00515×1000.Youshouldbeabletodothisproblembymovingdecimalpointsandnotbymultiplyingout.Todivideby10,movethedecimalpointoneplacetotheleft.Moveittwoplacestothelefttodivideby100,threeplacestodivideby1000,andsoforth.Tomultiplyby10,100,1000,andsoforth,movethedecimalpointthecorrespondingnumberofplacestotheright.Thisisanimportantskilltoreview.

20.Thecorrectansweris(E).Thisisatrickyproblem.Choices(B),(C),and(D)mightbetrueinsomecases,dependingupontheexactmeasurementsof 1and 3Theonlyanswerthatistruenomatterwhatthemeasuresof 1and 3istheoneinwhichtheirsumisequalto120°.

21.Thecorrectansweris(A).Thiscanbesetupasaproportionwherexistheunknownnumber:

Thisisagoodproblemforestimation.Studythenumeratorsofthefractionsandnotethat45isonehalfof90.Therefore,thedenominatorsofthefractionsmusthavethesamerelationship.Onehalfof0.45is0.225.

22.Thecorrectansweris(D).Thesquarerootof20islessthanthesquarerootof25,whichis5,andgreaterthanthesquarerootof16,whichis4.Therefore,nisbetween4and5.

23.Thecorrectansweris(B).Tofindthedifference,wesubtract–6from4andmovefrom–6to4,adistanceof+10units.

24.Thecorrectansweris(C).Simplydivide by tofindtheanswer.

25.Thecorrectansweris(D).Fourgamesaveraging36,500peoplepergametotal146,000attendance.Thetotalforthefirstthreegameswas105,500.Thefourthgameattracted40,500people.

Section2:ReadingComprehension

1.Thecorrectansweris(C).Youwillfindtheanswertothisdetailquestioninthelasttwosentencesoftheselection.

2.Thecorrectansweris(D).Thisdetailisgiveninthelastsentenceofthefirstparagraph.AlltheotheranswerchoicesweretrueofthehistoryofRiker’sIslandbutarenottrueatthepresenttime.

3.Thecorrectansweris(B).Youcaninferthatthediscardsofcityhouseholdsincludedgarbage.Decayedgarbageisanexcellentfertilizer.

4.Thecorrectansweris(E).Seethenext-to-lastsentenceoftheselection.

5.Thecorrectansweris(C).TheselectiondiscusseshatmakingasanearlyAmericanindustry,touchingonmaterials,costs,andmarkets.

6.Thecorrectansweris(E).Seelines17–20,whereitstatesthatthehighnumberofhatsbeingexportedfromthecoloniescaused“uneasinessamongthehattersinEngland,”whoexertedpressureonParliament.

7.Thecorrectansweris(D).ProhibitionofexportofhatsfromonecolonytoanotherandfromanycolonytoGreatBritainwasaneffectivebanonexportofhatsaltogether.

8.Thecorrectansweris(A).Seethestartofthesecondparagraph,whichbegins,“Woolwastheprincipalrawmaterialusedbyhatters”.

9.Thecorrectansweris(C).Pricecomparisonsaremadeinthesecondparagraph:“Theaveragepriceofawoolhatwasbetween40and80cents,andbeaverhatsrangedfrom$2.50to$3.50.”

10.Thecorrectansweris(B).Theauthortellsyouhisreasoninthesecondsentence.

11.Thecorrectansweris(E).Seethethirdsentence.

12.Thecorrectansweris(A).Thefourthsentencegivesthisdetail.

13.Thecorrectansweris(C).Thesixthsentencetellsthatthegardenledtoaforest.Theselectionsaysthatthegardenextendedsofarthatthegardenerdidnotknowwhereitended,butitdoesnotsaythathewasunabletocareforitbecauseofitssize.

14.Thecorrectansweris(D).Inthelastsentencewelearnthattheforestwentdowntothesea,andinthetreesoftheforestatseasidelivedanightingale.

15.Thecorrectansweris(E).Thedictionarydefinitionoffilamentis“averyslenderthreadorfiber.”Youdonotneedtoknowthedictionarydefinitiontoanswerthisquestion.Waspscouldnotpossiblybehandlinglargechunksorfiles.Thewordfilamentsintheselectionappearsintheplural.Ifthemeaningwere“dust,”thewordwouldbeinthesingular.

16.Thecorrectansweris(D).ThefirstsentencedescribesRenédeRéaumerasaFrenchnaturalist.

17.Thecorrectansweris(B).Seethebeginningofthethirdparagraph:“Waspnestsareexceptionallylightweight,darkincolor,andboundrepeatedlybybandsofpapertotheplacewheretheyaresuspended.”

18.Thecorrectansweris(D).Thesecondparagraphoftheselectiondescribesthemannerinwhichwaspsusewoodtomakethepaperfromwhichtheyconstructnests.

19.Thecorrectansweris(C).Seethesecondsentenceofthethirdparagraph:“Thenestsarenearlywaterproofbecauseoftheirroundedtopsandthefactthatthepaperstripsoverlapliketheshinglesontheroofofahouse.”

20.Thecorrectansweris(E).Seethefirstsentence.Acenturyagowas100yearsago.

21.Thecorrectansweris(E).Thefirstsentenceofthesecondparagraphsaysthattheboatsareequippedforvariousjobs,whichmeansthattheyvaryinfunction.

22.Thecorrectansweris(D).Bycheckingontheoperationofthejunkboats,theharborpoliceensurethattheiractivitiesarelegal.

23.Thecorrectansweris(A).Theotherchoicesallincludesomeactivitythatisnotmentionedasanactivityoftheharborpolice.

24.Thecorrectansweris(C).The450disabledboatsthatweretowedandthethousandsthatneededsomesortofhelp(next-to-lastsentence)couldnotpossiblyhaveallbeenofthesamekind.

25.Thecorrectansweris(A).Contextshouldhelpyouhere.“...venerationduetoitssacredorigin...”impliessomethingreligiousandrelatedtoworship.

26.Thecorrectansweris(C).Thisisaninferentialquestion.ThespeakerprobablyenjoysandcelebratesChristmas,choice(B),aswell,buttheprimaryreasonforthisspeechisdefendingtheholidaytohisUncleScroogebylistingitsadvantagestomankind.

27.Thecorrectansweris(E).Thisisthewholepointofthefirstparagraph.

28.Thecorrectansweris(D).Again,useofthewordincontextshouldleadyoutoitsmeaning.Theparagraphspeaksofgoodwillamongallmenandwomen.Thisoneconsentthereforeisunanimousgoodfeeling.

29.Thecorrectansweris(A).Readthelastparagraphcarefully.ScroogeisfirstreactingtotheclerkwhohasjustapplaudedthespeechindefenseofChristmas.Scroogethreatenstheclerkwithfiring.Hethenturnsandmakesasarcasticremarktohisnephew.Itcanbeassumedthatheisangrywithbothcharacters.

30.Thecorrectansweris(D).“Juststoppedby...”isquiteamatter-of-factwayofspeaking.

31.Thecorrectansweris(B).Theraisedeyebrowsofthefirstsentenceofthesecondparagraphimplydisbelief.

32.Thecorrectansweris(B).Mr.Holterhadacaravanofanimals;wasinNewYorkonhiswaytoDayton,Ohio;andactuallylivedinAnaheim,California.Youcaninferthathemadehislivingtravelingandshowinghisanimals.

33.Thecorrectansweris(D).Thearticlediscusseshighbloodpressureasafamilialdisease,adiseasethatrunsinfamilies.Itgoesontodiscusstheroleofgeneticmakeupindeterminingreactiontodietaryfactors.Geneticmakeupreferstohereditaryfactors.

34.Thecorrectansweris(C).Thisisamain-ideaquestion.Themainpointoftheselectionisthatthereisaninterplayofgeneticandenvironmentalfactorsinfluencingthedevelopmentofhighbloodpressure.

35.Thecorrectansweris(B).Thefirstparagraphspeaksoftheperilsoffishing,thesecondaboutitsrewards.

36.Thecorrectansweris(E).Themiddleofthefirstparagraphdiscussestheproblemscreatedbyroughseas.Noneoftheotherchoicesisamentioneddifficulty.

37.Thecorrectansweris(A).Inthemiddleofthesecondparagraphwelearnthatwhenthefishermennotethatherringareenteringthenetstheysitinquietexcitementsoasnottofrightenthefishaway.Theyrowalongthenet,choice(E),inordertofindoutifthenetisfilling,andhaulinthenets,choices(B)and(D),whenthenetsarefull.Itisthefishthatglisten,notthefishermen.

38.Thecorrectansweris(C).Everythingismentionedexceptthebeautyofthesea.

39.Thecorrectansweris(A).Allotherchoicescontainatleastonetraitthatisnotascribedtothefishermen.

40.Thecorrectansweris(E).Onemightaddhonestytothetraitsofthefishermen.Theftisnotmentionedasaproblem.

Section3:Verbal

1.Thecorrectansweris(E).TheAGENDAistheprogramofthingstobedoneortheschedule.Preparationofnextyear’sbudgetwasthetopitemontheagendaforthemeeting.

2.Thecorrectansweris(A).CREDIBLEmeansplausible,reliable,orbelievable.Thepresenceofmanysquirrelsinmyyardisacredibleexplanationforthemanyholes.

3.Thecorrectansweris(B).PLACIDmeanstranquil,calm,orpeaceful.LakePlacidinNewYorkissoplacidthatitswatersareseldomstormy.

4.Thecorrectansweris(C).ToINTERVENEistocomebetweentwopeopleorthingseithertointerfereortoinfluencepositively.Interveneisaverb.Anintermediary(noun)mayinterveneinadispute.

5.Thecorrectansweris(E).MUNDANEmeanscommonplace,earthly,orordinary.EverymorningIperformthemundanetasksofbrushingmyteethandmakingmybed.

6.Thecorrectansweris(E).Todehydrateistoremovewater,thereforeDEHYDRATEDmeanswaterless.Theroothydr-referstowater,andtheprefixde-isanegativeprefix.Dehydratedfoodsarelightweightandareeasytostoreforlongperiodsoftime.

7.Thecorrectansweris(B).PREVALENTmeanswidelyexisting,prevailing,orgenerallyaccepted.Preeminentmeansexcelling.TheprevalentmoodamongtheBoyScoutswasoneofeageranticipation.

8.Thecorrectansweris(A).SUCCINCTmeansbriefandtothepoint.Thelegislatorgaveasuccinctbackgroundofthereasonsfortheproposedlaw.

9.Thecorrectansweris(A).ThatwhichisNOCTURNALhappensatnight.Batsdonotflyaboutinthedaytimebecausetheyarenocturnalcreatures.

10.Thecorrectansweris(B).EQUITABLEmeansfairandjust.Youshouldseetherootequalinthisword.Thewillprovidedforanequitabledistributionoftheproperty.

11.Thecorrectansweris(D).ToEXPEDITEistospeeduptheactionortosendquickly.TheLatinderivationofthiswordis“tofreeonecaughtbythefeet.”YoucanexpeditethedeliveryofmailbyusingZIPCodeplusfour.

12.Thecorrectansweris(E).TURBULENTmeansunrulyoragitated.Astheairplanepassedthroughturbulentair,weallfeltratherqueasy.

13.Thecorrectansweris(C).TENACIOUSmeansholdingontightlyorpersistent.Thetenacioussalesmancallstwiceaweekbetween5and7p.m.

14.Thecorrectansweris(A).PERTINENTmeansrelevant.Testimonyisadmittedincourtonlyifitispertinenttothechargesinthecase.

15.Thecorrectansweris(C).DOGMATICmeansdictatorialoropinionated.Thewordhastodowithdoctrineordogma,notwithdogs.Myuncleissodogmaticthatherefusestoevenlistentomypointofview.

16.Thecorrectansweris(D).OnewhoisUNSCRUPULOUSisnotrestrainedbyideasofrightandwrong.Theunscrupulousstockbrokerusedinsideinformationtosellbeforethestockpriceplummeted.

17.Thecorrectansweris(C).WILYmeanscraftyorsly.ThewilywolfoutwittedRedRidingHood.

18.Thecorrectansweris(B).BLATANTmeansloudandobtrusive.Themisspellingintheaddresswasablatanterrorinanotherwiseexcellentletter.

19.Thecorrectansweris(A).APRETEXTisafalsereasonoranexcuse.Illnesswashispretextforabsencefromschool;actuallyhewenttothebeach.

20.Thecorrectansweris(C).ACUMENiskeennessandquicknessinunderstandinganddealingwithasituation.Acumenwithrespecttoforeignculturesisagreatassetinthediplomaticcorps.

21.Thecorrectansweris(D).EVASIONissubterfugeoravoidance.Hismannerofevasionofembarrassingquestionswastomakealongspeechonanothertopic.

22.Thecorrectansweris(B).ThatwhichisINDISPENSABLEcannotbedispensedwith,thatis,itisabsolutelyessential.Thepresidentofthecompanyrefusedtotakeavacationbecausehehadthemistakennotionthathispresencewasindispensable.

23.Thecorrectansweris(E).ToOBLITERATEistodestroywithoutleavingatrace.Thewashingwavesobliteratedourfootstepsinthesand.

24.Thecorrectansweris(D).AMIABLEmeanspleasant,friendly,andgood-natured.Theamiableshopkeeperallowedustocontinuetryingonshoeseventhoughitwasalreadypastclosingtime.

25.Thecorrectansweris(E).ToWRITHEistotwist,squirm,orcontort,usuallyindiscomfort.Theskierwrithedinpainwhenshebrokeherankle.

26.Thecorrectansweris(A).ToABATEistodiminish.Wewillstaytiedupinportuntilthewindsabate.

27.Thecorrectansweris(B).AnENDORSEMENTisastatementofapproval.Thegovernorgavehisendorsementtothecandidateformayorofthecity.

28.Thecorrectansweris(D).ToCONVERTistochangefromoneformtoanother.UseatransformertoconvertDCcurrenttoACcurrent.

29.Thecorrectansweris(A).ERUDITEmeanslearnedorscholarly.Hehaslittle

information,buthisbeautifulcommandoftheEnglishlanguagemakeshimappeartobeerudite.

30.Thecorrectansweris(B).ToENDEAVORistoattemptortotry.Theexpeditionendeavoredtoreachthemountaintopbeforethethunderstorm.

31.Thecorrectansweris(B).Therelationshipofthetermsisoneofdegree.Noneistheultimate,theemptyset,oflittle;neverbearsthesamerelationshiptoinfrequently.

32.Thecorrectansweris(C).Ifyouthinkintermsofahouse,youcanseethatthetermsoneachsideoftherelationshiparesynonymous.Youcanreceiveapersonintoyourhomeoradmittheperson.Oncethepersondecidestoremain,thatpersonsettlesin.

33.Thecorrectansweris(E).Herethecause-and-effectrelationshipisclear.Recognizeddishonestyleadstodistrust;carelessnessleadstoaccidents.

34.Thecorrectansweris(A).Therelationshipisthatofactortoobject.Asociologiststudiesgroups;apsychologiststudiesindividuals.Therelationshipofthechildrentothepediatricianisinreverseorder.

35.Thecorrectansweris(B).Thetermsareantonyms.Generousistheoppositeoffrugal;aphilanthropististrulytheoppositeofamiser.Thetermsinchoices(C)and(E)arealsoantonyms.Whenfacedwithquestionsinwhichthesamerelationshipismaintainedbyanumberofthechoices,youmustlookforarelationshipamongallfourterms.Inthiscase,thethemetobecarriedthroughamongthechoicesofthecorrectanalogyis“money.”

36.Thecorrectansweris(C).Therelationshipisoneofdegree.Translucentisdenserthantransparent,thatis,onecanactuallyseethroughsomethingthatistransparentwhereasonlylightpassesthroughatranslucentmedium.Carryingontothenextdegree,opaqueisdenserthantranslucent.Notevenlightcanpassthroughsomethingthatisopaque.Choice(A)isincorrectbecauseitskipsadegreeandjumpsfromtransparenttoopaque.Choice(D)reversestheorder.Glassandcrystal,choice(B),maybothbetransparent.

37.Thecorrectansweris(A).Thisisaclassiccause-and-effectrelationship.Discontentleadstorebellion;frictioncreatesaspark.

38.Thecorrectansweris(B).Therelationshipisthatofworkertotool.Achemistusesabeakerinthelaboratory;ageologistusesahammertochipatrocksinthefieldorlaboratory.Avoidthe“trap”ofchoice(C).Ahammeriscertainlyusedinconstruction,buttherelationshipofthefirsttwotermsrequiresthatapersonbeinvolvedtocompletetheanalogy.

39.Thecorrectansweris(D).Thebasisoftheanalogyisantonyms.

40.Thecorrectansweris(C).Thisisapart-to-wholeanalogy.Astateispartofacountry;acountryispartofacontinent.

41.Thecorrectansweris(A).Causeandeffect.Anacceleratorcausesthemotionofthecar;acatalystcausesthechemicalchange.

42.Thecorrectansweris(E).Therelationshipisoneofdegree.Probableislikely,butlesslikelythancertain;plausibleispossible,butlesslikelythandefinite.

43.Thecorrectansweris(A).Therelationshipisoneoftruesynonyms.

44.Thecorrectansweris(B).Thisanalogyinvolvestrueantonyms.

45.Thecorrectansweris(A).Therelationshipisthatofobjecttoactor.Thenucleusistheobjectthatisorbitedbyanelectron;Earthistheobjectthatisorbitedbyasatellite.Choice(B)reversestheorderoftherelationship.

46.Thecorrectansweris(C).Heretherelationshipisthatofactortoobject.Asculptorcreatesastatue;acomposercreatesmusic.Anactorperformsinaplaybutdoesnotcreateit.Aprogrammercreatesaprogramwhileworkingatacomputer.

47.Thecorrectansweris(C).Theanalogyisbasedonanantonymrelationship.

48.Thecorrectansweris(D).Thisanalogyisalsobasedonantonyms.

49.Thecorrectansweris(E).Itishardtocategorizethisrelationship.Oneinterruptsbyspeakingoutofturn;oneintrudesbyenteringoutofturn.Therelationshipinchoice(B)mightbethatofopposites.

50.Thecorrectansweris(A).Thefirsttwotermsaretrueopposites.Onlychoice(A)offerstrueopposites.Financialstabilityistheoppositeofpoverty,butfinancebearsnorelationshiptopovertyatall.

51.Thecorrectansweris(D).Thisanalogyisbestunderstoodasanegativecauseandeffect.Adversityleadstoalackofhappiness;vehemenceleadstoalackofserenity.

52.Thecorrectansweris(A).Therelationshipisthatofactortoaction.Anextortionistblackmails;akleptomaniacsteals.

53.Thecorrectansweris(B).Thisisawhole-to-partrelationship.Amonsoonisamajorstormofwhichrainisacrucialcomponent;atornadoisamajorstormofwhichwindisacrucialcomponent.

54.Thecorrectansweris(E).Therelationshipbetweenthetwosetsofwordsisthatthewordsineachhalfoftheanalogyaresynonyms.Don’tworrythatthewordsinthefirsthalfareantonymsofthesecond.Youaren’tlookingathowallfourwordsrelatetooneanotherinthisanalogy,justathowthewordsineachhalfrelatetooneanother.

55.Thecorrectansweris(D).Youneedn’tcategorizeananalogy;youonlyneedtounderstandit.Theequatoristhemidlinethatcirclestheworld;thewaististhemidlinethatcirclestheman.

56.Thecorrectansweris(A).Oneachsideoftheanalogy,thefirsttermisacharacteristicofthesecond.

57.Thecorrectansweris(C).Thisanalogyisbasedoncharacteristicsofbodiesofwater.Apondmaybestagnant;astreamislikelytoflow.Sheepmaybenoisybutsincetherearetwochoicesthatinvolvecharacteristics,youmustchoosetheonethatisclosestinotheraspectstothefirstsetofterms,thatis,theoneinvolvingwater.

58.Thecorrectansweris(B).Thisisapurposefulorfunctionalrelationship.Afortressgivessanctuary;ahousegivesshelter.

59.Thecorrectansweris(A).Thetermsaresynonyms.

60.Thecorrectansweris(A).Thisanalogyisalsobasedonsynonyms.Choice(E)isincorrectbecausepensivemeansthoughtful.Ifyoumadethischoice,youweremistakingpensiveforpendant,whichdoesmeanhanging.

Section4:Quantitative(Math)

1.Thecorrectansweris(E).Anypointonthesurfacerotatesonceeachdayrelativetoapointinspace.Eachrevolutionisanangleof360°.Intwodays,tworevolutionstakeplace,360°×2=720°.

2.Thecorrectansweris(B). , ,and arealllessthan ; islargerthan .

Comparethesizeoffractionsthisway.

Becausetheproductof7and15islargerthantheproductof32and3, willbefound

tobelarger.Usingthesamemethod, < .

3.Thecorrectansweris(D).Ifthewidthisx,thelength,whichistwiceaslong,is2x.Theperimeterisequaltothesumofthefoursides:2x+2x+x+x=6x.

4.Thecorrectansweris(B).UsethePythagoreanTheoremc2=a2+b2tofindthelengthofthediagonal:

c2=12+12

c2=2

c=

5.Thecorrectansweris(C).Eightsections,each3'2"long,isequivalentto8×38"=304".

304"=25 feet;thereforethree10-footsectionsareneeded.

6.Thecorrectansweris(D).Thewidthoftheshadedareais ofthewidthofthesquare.

Therefore,theareaoftheshadedpartis theareaofthewholesquare.Theunshaded

partistwiceaslargeastheshadedpart.Theratiooftheshadedparttotheunshaded,therefore,is1:2.

7.Thecorrectansweris(C).Sincedistance=rate×time,rate=distance÷time.Totaldistancetraveledis3200miles.Totaltimeis4hours20minutes.

Rate=3200miles÷4hours20minutes

=3200miles÷4 hours

=738mph,approximately

8.Thecorrectansweris(B).Thisisasimpleproportion: .xistheunknownwidth.

Cross-multiply:

7x=20

x= ,or2

9.Thecorrectansweris(C).Subtracttheareaofthecirclefromtheareaofthesquaretofindtheareaofjusttheshadedpart.

Notethatthediameterofthecircleequalsthewidthofthesquare.

Areaofsquare=s2=4sq.in.

Areaofcircle=πr2=π(1)2=πsq.in.Areaofsquare–Areaofcircle

=4sq.in.– sq.in.

= sq.in.,or in.2

10.Thecorrectansweris(E).Thepopulationwouldreproduce12timesin4hours.ThesizethenisP=1×212

=2•2•2•2•2•2•2•2•2•2•2•2=4096

11.Thecorrectansweris(B).Thelargerthenumberofthedenominatorofafraction,the

smallerthequantityrepresented.Forexample, representsalesserquantitythan .

Therefore,asxbecomesgreater,ybecomessmaller.

12.Thecorrectansweris(D).Ifthesecondboxhaseachdimensionthreetimesthatofthefirstbox,thenitsvolumeis3×3×3=27timesasgreat.

13.Thecorrectansweris(B).Theareaoftheflowerbedis4πsq.ft.(A=πr2).Theareaofthenewbedistobefourtimesasgreat,or16πsq.ft.Abedwithanareaof16πsq.ft.musthaveadiameterof8',andaradiusof4',sinceA=πr2.

14.Thecorrectansweris(D).UsetheformulaD=R×TtofindthetimeitactuallytooktogettoBuffalo:time=distance÷rate.Traveltimeoftripwasequalto290miles÷50mph.

Traveltime=5 hours,or5hours48minutes.Scheduledtraveltimewasbetween10:10

a.mand3:45p.m,anintervalof5hours35minutes.Therefore,thetraintookabout13minuteslongerthanscheduled.

15.Thecorrectansweris(E).Solvetheequationforx:3x–2=133x=15x=5Ifx=5,then12x+20=12(5)+20=80.

16.Thecorrectansweris(A).Sincethenumberofpoundsofeachkindofcakesoldwasthesame,wecansaythatapoundofcakesoldforanaveragepriceof30¢perpound.

25¢+30¢+35¢=90¢÷3=30¢perlb.

Dividethetotalsalesincomeof$36by30¢tofindhowmanypoundsweresold.

$36÷0.30=120

17.Thecorrectansweris(C).Afloor12'×12'is144sq.ft.inarea,andwouldrequire144tilesthatareeach1footby1foot.Twelvetileswouldbeplacedalongthewidthandlengthoftheroom.If9"tilesareused,itrequires16ofthemplacedendtoendtocoverthelengthoftheroom.Therefore,itrequires16×16tilestocoverthefloor,or256tiles.Itrequires112more9"tilesthan12"tilestocoverthefloor.

18.Thecorrectansweris(B).Ifppencilscostccents,thecostofeachpencilis cents.

Tofindthecostofnpencils,wemultiplythecostofeachtimesn:

19.Thecorrectansweris(E).Ifnecessary,tryeachoftheanswersforyourself,toseethateachisfalse.Choice(C)isuntrueforthenumber1.

20.Thecorrectansweris(D).Thinkofarectanglewiththedimensions by Itsareais2squareinches.Ifwedoubleeachdimension,to by theareabecomes8squareinches,whichisfourtimestheareaofthefirstrectangle.Thisisequaltoanincreaseof300%.

21.Thecorrectansweris(B).Thefirstpipecanfillthetankin1 ,or ,hours;thatis,it

cando ofthejobin1hour.Thesecondpipecanfillthetankin45minutes,or ofan

hour,oritcando ofthejobin1hour.Togetherthepipescancomplete of

thejobin1hour. =2,ortwicethejobin1hour.Therefore,togetherthe2pipescould

fillthetankin hour.

22.Thecorrectansweris(B).Thewholeseasonconsistsof120games.Foraseasonrecordof60%,theteammustwin72games.Sinceithasalreadywon50,itmustwin22moregamesoutofthoseleft.

23.Thecorrectansweris(D).Commercialandindustrialneedstotal63%ofdailyoilconsumption.Sinceconsumptionis9millionbarrels,63%of9millionis5,670,000barrels.

24.Thecorrectansweris(A).Addthecostofthehouse,driveway,painting,andplumbing:$44,000+$1250+$1750+1000=$48,000

Ifhewantstomakea12%profitwhenresellingthehouse,heshouldincreasethetotalcostby12%tofindthenewsellingprice:

12%of$48,000=$5760

$48,000+$5760=$53,760

25.Thecorrectansweris(A).Thisisaproblemthatmustbedonecarefully.a=1,b=2,c=3,d=5

SCOREYOURSELFCheckyouranswersagainsttheanswerkeys.Countupthenumberofanswersyougotrightandthenumberyougotwrong.

SECTION NO.RIGHT NO.WRONG

Quantitative(Math)

ReadingComprehension

Verbal

Nowcalculateyourrawscores:

Quantitative(Math): (________) – (_________) = (_________)

No.Right No.Wrong RawScore

ReadingComprehension: (________) – (_________) = (_________)

No.Right No.Wrong RawScore

Verbal: (________) – (_________) = (_________)

No.Right No.Wrong RawScore

NowcheckyourRawScoreagainsttheconversionchartstogetanideaoftherangeinwhichyourtestscoresfell:

RawScore Quantitative(Math)Reading

Comprehension Verbal

60 350

55 350

50 350 343

45 344 334

40 335 348 325

35 325 324 316

30 316 314 307

25 306 304 298

20 297 294 289

15 288 284 280

10 278 274 271

5 269 264 261

0 260 254 252

–5orlower 250 250 250

Remember:•Thesameexamisgiventostudentsingrades8through11.Youarenotexpectedtoknowwhatyouhavenotbeentaught.

•Youwillbecomparedonlytostudentsinyourowngrade.Useyourscorestoplanfurtherstudyifyouhavetime.

PracticeTest4:ISEE(UpperLevel)

SECTION1:VERBALREASONING40Questions•20Minutes

Directions:EachquestionismadeupofawordinCAPITALlettersfollowedbyfourchoices.ChoosetheonewordthatismostnearlythesameinmeaningasthewordinCAPITALletters.

1.FEINT

(A)fool

(B)proclaim

(C)penalize

(D)scavenge

2.PEER

(A)officer

(B)beginner

(C)equal

(D)patient

3.TRITE

(A)unskilled

(B)common

(C)unlikely

(D)ignorant

4.AMIABLE

(A)forgetful

(B)friendly

(C)strange

(D)great

5.GRIMACE

(A)sneer

(B)grindstone

(C)journal

(D)treasure

6.COMPELLED

(A)calculated

(B)combined

(C)collected

(D)forced

7.ALLY

(A)opponent

(B)passage

(C)friend

(D)preference

8.SOLICIT

(A)consent

(B)comfort

(C)request

(D)help

9.REFUTE

(A)demolish

(B)postpone

(C)disprove

(D)assist

10.EXPLICIT

(A)ambiguous

(B)clearlystated

(C)giveinformationabout

(D)toblowout

11.RETAIN

(A)payout

(B)play

(C)keep

(D)inquire

12.CORRESPONDENCE

(A)letters

(B)files

(C)testimony

(D)response

13.LEGITIMATE

(A)democratic

(B)legal

(C)genealogical

(D)underworld

14.DEDUCT

(A)conceal

(B)understand

(C)subtract

(D)terminate

15.EGRESS

(A)extreme

(B)extrasupply

(C)exit

(D)highprice

16.HORIZONTAL

(A)marginal

(B)inacircle

(C)leftandright

(D)upanddown

17.CONTROVERSY

(A)publicity

(B)debate

(C)revolution

(D)revocation

18.PREEMPT

(A)steal

(B)empty

(C)preview

(D)appropriate

19.PERCAPITA

(A)foranentirepopulation

(B)byincome

(C)foreachperson

(D)foreveryadult

20.OPTIONAL

(A)notrequired

(B)infrequent

(C)choosy

(D)forsale

Directions:Eachofthefollowingquestionsismadeupofasentencecontainingoneortwoblanks.Thesentenceswithoneblankindicatethatonewordismissing.Sentenceswithtwoblankshavetwomissingwords.Eachsentenceisfollowedbyfourchoices.Choosetheonewordorpairofwordsthatwillbestcompletethemeaningofthesentenceasawhole.

21.Customhasso_____ourlanguagethatwecan_____onlywhathasbeensaidbefore.

(A)improved...repeat

(B)changed...understand

(C)enslaved...say

(D)dominated...hear

22.Afewofthecritics_____theplay,butingeneraltheyeitherdisregardedorridiculedit.

(A)discredited

(B)criticized

(C)denounced

(D)appreciated

23.Politiciansarenottheonlyoneswhohavemade_____;beinghuman,wehaveallblunderedatsometimeinourlives.

(A)explanations

(B)arguments

(C)errors

(D)excuses

24.Becauseofhis_____nature,heoftenactspurelyonimpulse.

(A)stoic

(B)reflective

(C)passionate

(D)wistful

25.Asystemofeducationshouldbe_____bythe_____ofstudentsitturnsout,forqualityispreferredtoquantity.

(A)controlled...intelligence

(B)justified...number

(C)examined...wealth

(D)judged...caliber

26.Weseldomfeel_____whenweareallowedtospeakfreely,butany_____ofourfreespeechbringsanger.

(A)angry...defense

(B)blessed...restriction

(C)scholarly...understanding

(D)enslaved...misuse

27.Theworstteamlostbecauseithadmanyplayerswhothoughnotcompletely_____werealsonotreally_____.

(A)qualified...agile

(B)clumsy...incompetent

(C)inept...proficient

(D)ungraceful...amateurish

28.Althoughthe_____ofthelegislaturebecomelaw,theexact_____ofthelawistheresultofjudicialinterpretation.

(A)ideas...enforcement

(B)bills...wording

(C)works...punishment

(D)words...meaning

29.Sincemovieshavebecomemore_____,manypeoplebelievetelevisiontobe_____.

(A)helpful...utilitarian

(B)expensive...necessary

(C)common...inadequate

(D)costly...useless

30.Sporesareaformoflifethatremain_____untilenvironmentalconditionsexistinwhichtheycanbecome_____.

(A)inactive...vibrant

(B)hidden...dangerous

(C)suppressed...visible

(D)controlled...rampant

31.Thespiritofscienceisalwaystryingtoleadpeopletothestudyof_____andawayfromthespinningoffancifultheoriesoutoftheirownminds.

(A)tradition

(B)order

(C)legalities

(D)literature

32.The_______childthoughtoldpeopleshouldbepolitetohim!

(A)submissive

(B)impertinent

(C)alternate

(D)classless

33.Thefameoftheauthordoesnot_____thequalityofhisorherworks.Wemustavoidequatingsuccesswithinfallibility.

(A)prejudice

(B)assure

(C)dignify

(D)extol

34.Themechanismsthatdevelophatredinmanaremostpotent,sincethereismore_____than_____intheworld.

(A)tolerance...prejudice

(B)joy...rapture

(C)love...hatred

(D)strife...tranquility

35.Miningisoftencalledthe_____industry,sinceitneithercreatesnorreplenisheswhatittakes.

(A)robber

(B)ecology

(C)natural

(D)evil

36.Theracialproblemisofsuch_____thatitmakesgoingtothemoonseem_____.

(A)complexity...helpful

(B)certainty...problematic

(C)magnitude...child’splay

(D)docility...effortless

37.Tobe_____atheatricalsettingmustresemble_____.

(A)believable...home

(B)effective...reality

(C)reasonable...beauty

(D)respectable...ideas

38.The_____mobroamedthroughthestreetsofthecity,shoutingtheir_____oflawandorder.

(A)influential...fear

(B)indifferent...horror

(C)disciplined...disrespect

(D)hysterical...hatred

39.Errorsinexistingtheoriesarediscovered,andthetheoriesareeither_____or_____.

(A)improved...obeyed

(B)removed...followed

(C)altered...discarded

(D)explained...excused

40.Inobservingthe_____societyoftheant,thescientistcanlearnmuchaboutthemore_____societyofman.

(A)hostile...evil

(B)elementary...complicated

(C)plain...homogeneous

(D)unadorned...unsophisticated

SECTION2:QUANTITATIVEREASONING37Questions•35Minutes

Note:YoumayassumethatallfiguresaccompanyingQuantitativeReasoningquestionshavebeendrawnasaccuratelyaspossibleEXCEPTwhenitisspecificallystatedthataparticularfigureisnotdrawntoscale.Letterssuchasx,y,andnstandforrealnumbers.TheQuantitativeReasoningTestincludestwotypesofquestions.Thereareseparatedirectionsforeachtypeofquestion.

Directions:Forquestions1-19workeachprobleminyourheadoronscratchpaper.Choosetheletterofyouranswerchoice.

1.Whichpairofvaluesforxand willmakethefollowingstatementtrue?2x 8

(A)(6,<)

(B)(4,>)

(C)(0,<)

(D)(–3,>)

2.Completethefollowingstatement:7(3×_____)+4=2104

(A)10

(B)10+2

(C)102

(D)103

3.0.5%isequalto

(A)0.005

(B)0.05

(C)

(D)0.5

4.Ascalenetrianglehas

(A)twoequalsides.

(B)twoequalsidesandonerightangle.

(C)noequalsides.

(D)threeequalsides.

5.Ifa–2b=–7,thenwhichexpressionisequaltoa?

(A)2b–7

(B)2b+7

(C)–2b+7

(D)–2b–7

6.Amillimeteriswhatpartofameter?

(A)

(B)

(C)

(D)

7.Whatistheleastcommondenominatorfor

(A)36

(B)32

(C)24

(D)18

8.Findtheareaofatrianglewhosedimensionsare:b=14inches,h=20inches.

(A)140squareinches

(B)208squareinches

(C)280squareinches

(D)288squareinches

9.Whatisthedifferencebetween(4×103)+6and(2×103)+(3×10)+8?

(A)168

(B)1968

(C)3765

(D)55,968

10.Thesetofcommonfactorsfor30and24is

(A){1,2,3,6}

(B){1,2,3,4,6}

(C){1,2,4,6}

(D){1,2,4,6,12}

11.

isequalto

(A)

(B)

(C)

(D)

12.Whatisthevalueoftheexpression

(A)

(B)

(C)

(D)

13.Theboardshownbelowis6feetlong,4incheswide,and2inchesthick.One-thirdofitwillbedrivenintotheground.Howmuchsurfacearearemainsaboveground?

(A)About4sq.ft.

(B)Slightlylessthan5sq.ft.

(C)Slightlymorethan5sq.ft.

(D)About8sq.ft.

14.OnerunnercanrunMmilesinHhours.AnotherfasterrunnercanrunNmilesinLhours.Thedifferenceintheirratescanbeexpressedas

(A)

(B)MH–HL

(C)

(D)

15.IfMaryisxyearsoldnowandhersisteris3yearsyounger,then5yearsfromnowhersisterwillbewhatage?

(A)x+5years

(B)x+3years

(C)x+2years

(D)8years

16.Inthefigurebelow,thelargestpossiblecircleiscutoutofasquarepieceoftin.Theareaoftheremainingpieceoftinisapproximately(insquareinches)

(A)0.14

(B)0.75

(C)0.86

(D)3.14

17.Asquarehasanareaof49sq.in.Thenumberofinchesinitsperimeteris

(A)7.

(B)14.

(C)28.

(D)98.

18.IfanenginepumpsGgallonsofwaterperminute,thenthenumberofgallonspumpedinhalfanhourmaybefoundby

(A)takingonehalfofG.

(B)dividing60byG.

(C)multiplyingGby30.

(D)dividing30byG.

19.Twocarsstartfromthesamepointatthesametime.Onedrivesnorthat20milesperhourandtheotherdrivessouthonthesamestraightroadat36milesperhour.Howmanymilesapartaretheyafter30minutes?

(A)Lessthan10

(B)Between10and20

(C)Between20and30

(D)Between30and40

Directions:Forquestions20–37,twoquantitiesaregiven—oneinColumn

AandtheotherinColumnB.Insomequestions,additionalinformationconcerningthequantitiestobecomparediscenteredabovetheentriesinthetwocolumns.Comparethequantitiesinthetwocolumns,andchooseyouranswerasfollows:

(A)ifthequantityinColumnAisgreater

(B)ifthequantityinColumnBisgreater

(C)ifthequantitiesareequal

(D)iftherelationshipcannotbedeterminedfromtheinformationgiven

ColumnA ColumnB

20.

s=1t=3a=–2

[5a(4t)]3 [4a(5s)]2

21. 4>x>–3

22.

a<bKR KT

23.

ColumnA ColumnB

24.

a>bx<a+b

a+b y

25. y=anoddinteger

Thenumericalvalueofy2 Thenumericalvalueofy3

26. (8+6)÷[3–7(2)] (6+8)÷[2–7(3)]

27. Threefourthsof

28.

NC=NYN> C

NC CY

29.

ColumnA ColumnB

30.

31.

ParallelogramABCDEisapointonAB

Areaof DEC Areaof AED+Areaof EBC

32. x=–1

x3+x2–x+1 x3–x2+x–1

33. Theedgeofacubewhosevolumeis27 Theedgeofacubewhosetotalsurfaceareais54

34.

35. Areaofacirclewhoseradiusisx3 Areaofacirclewhoseradiusis3x

ColumnA ColumnB

36.

Radiusoflargercircle=10Radiusofsmallercircle=7

Areaofshadedportion Areaofsmallercircle

37. a<0<b

a2

SECTION3:READINGCOMPREHENSION36Questions•35Minutes

Directions:Eachreadingpassageisfollowedbyquestionsbasedonitscontent.Answerthequestionsonthebasisofwhatisstatedorimpliedinthepassage.

Whenwesayasnake“glides,”wehavealreadypersuadedourselvestoshiveralittle.Ifwesaythatit“slithers,”weareasgoodasundone.Toavoidunsettlingourselves,weshouldstatethesimplefact—asnakewalks.

Asnakedoesn’thaveanybreastbone.Thetipsofitsribsarefreemovingandamount,sotospeak,toitsfeet.Asnakewalksalongonitsribtips,pushingforwarditsventralscutesateach“step,”anditspeedsupthismodeofprogressbyundulatingfromsidetosideandbytakingadvantageofeveryrough“toehold”itcanfindintheterrain.

Let’slookatitthisway:Ahumanorotheranimalgoingforwardonallfoursisusingasortoflocomotionthat’sfamiliarenoughtoallofusandisn’tatalldismaying.Now:Supposethiswalkerisenclosedinsidesomesortofpliableencasementlikeasacking.Thefront“feet”willstillstepforward,the“hindlegs”stillhitchalongafterward.Itwillstillbeastandardenoughsortofanimalwalking,onlyallwe’llseenowisasortofwigglingofthesackingwithoutvisiblefeet.That’sthesnakeway.Asnakehasitscoveringoutsideitsfeet,asaninsecthasitsskeletononitsoutsidewithnobonesintheinterior.There’snothingmore“horrid”abouttheonearrangementthanabouttheother.

Essentially,whensnakesmoveonland,theyusetheirmusclestopushoffofsomethingontheground.Theymightpushagainstarockorpieceofwood,orevenagainstaroughterrain.Thisformofmovementiscalledserpentinemovement,anditiscommonforsnakesmovingonlandandeveninthewater.Inthewater,snakesactuallyusethewateritselfasapointofresistance,pushingoffthewatertohelpthemmove.

1.Thetitlebelowthatexpressesthemainideaofthisselectionis

(A)“Snake’sLegs.”

(B)“ComparingSnakestoPeople.”

(C)“TheMovementofaSnake.”

(D)“ASlimyAnimal.”

2.Asnake’s“feet”areits

(A)toes.

(B)ribs.

(C)side.

(D)breastbone.

3.Thewordterrainmeans

(A)terraced.

(B)rockyledge.

(C)verticalhole.

(D)groundareas.

4.Wemayconcludethattheauthor

(A)raisesreptiles.

(B)dislikessnakes.

(C)iswellinformedaboutsnakes.

(D)thinkssnakesmovebetterthanhumans.

5.Thewordlocomotionmostnearlymeans

(A)train.

(B)limit.

(C)fuel.

(D)movement.

6.Theauthormostlikelybelievesthatsnakemovementis

(A)naturallyupsetting.

(B)agreatdeallikeflying.

(C)similaronlandandwater.

(D)morelikeglidingthanwalking.

Whenaluxurylineroracargoshipnudgesintoherslipafteranoceancrossing,her

firstphysicalcontactwithlandisaheavingline.Thesestreamerswithaweightattheendcalleda“monkeyfist”archgracefullyfromdecktopier.Onboardtheshiptheheavinglinesaretiedtoheavy,goldenyellowmanilamooringlines.Longshoremenquicklypullintheheavinglinesuntiltheycanfastenthemooringlinestoironbollards(posts).Soontheshipisstrungtoherpierbyfour,eight,orasmanyastwenty-onenine-inchorten-inchmanilalineswithperhapsafewwireropestostaymotionforeandaft.Theshipissecureagainsteventhewrathofthestormorhurricane.Ashipcoulddockwithouttheaidoftugboats—andmayhaveinNewYorkinmaritimestrikes—butnotwithoutthelinestomoorhertoherberth.

Themaritimeandtherelatedfishingindustryfindperhaps250applicationsforropeandcordage.Therearehundredsofdifferentsizes,constructions,tensilestrengths,andweightsinropeandtwine.Ropeissoldbythepoundbutorderedbylength,anditismeasuredbycircumferenceratherthanbydiameter.Themaritimevarietyismadechieflyfromfiberoftheabaca,ormanilaplant,whichisimportedfromthePhilippinesandCentralAmerica.HenequenfromMexicoandCuba,andsisalfromAfrica,theNetherlands,EastIndies,andotherareasarealsoused,butchieflyfortwine.

Nyloniscomingintoincreasinguse,particularlybytowingcompanies.Itismuchstronger,lighterinweight,andlonger-wearingthanmanila.Itislesssusceptibletomildewthanropesmadefromnaturalfibers,anditisalsomoreelasticandparticularlyadaptableforoceantowing.Itselasticityhelpsittocushionwellagainstshock,butadisadvantageisthatitcanbecometoostretchedoutforuseincertainapplications.

7.Indockingaship,ropeis

(A)onlyalittlelessimportantthanatugboat.

(B)essential.

(C)helpfulbutnotnecessary.

(D)seldomused.

8.Amonkeyfistisa

(A)deviceforweavingrope.

(B)slangtermforalongshoreman.

(C)ropestreamer.

(D)weightattheendofarope.

9.Mooringropesare

(A)teninchesindiameter.

(B)twenty-oneinchesincircumference.

(C)sixtimesthickerthanheavingropes.

(D)nineinchesincircumference.

10.WhichofthefollowingareNOTcorrectlypaired?

(A)SisalfromthePhilippines

(B)HenequenfromCuba

(C)AbacafromCentralAmerica

(D)SisalfromtheNetherlandsEastIndies

11.Thewordchieflymostnearlymeans

(A)mainly.

(B)initially.

(C)only.

(D)wisely.

12.Accordingtotheselection,whichofthefollowingisadisadvantageofnylon?

(A)Itcushionspoorlyagainstshock.

(B)Itcanbecomeoverstretched.

(C)Itisnotresistanttomildew.

(D)Itlacksstrengthcomparedtomanila.

Onapopulationmapoftheworld,desertsareshownasgreatblankspaces.Weoftenthinkofdesertsasvastexpansesofnothingness—stretchesofhot,drysandasfarastheeyecansee.Andthoughdesertsmayseembarrenatfirstglance,inrealitytheyareanythingbutemptyandlifeless.Infact,theseareascontributemanythingstoourlives.

Whenyougotothemarkettobuyaboxofdates,youarebuyingabitofsunshineanddryairfromtheoasesoftheSaharaDesertortheCoachellaValley.FreshpeasoralettucesaladforyourwinterdinnermightbetheproductofanirrigationfarmerintheSaltRiverValleyortheImperialValley.Thatfinebroadclothshirtyoureceivedforyourbirthdaywasmadefromsilky,long-fiberedcottongrowninEgypt.Ahalf-wool,half-cottonsweatermightcontainAustralianwoolandPeruviancotton,whicharesteppeanddesertproducts.

Theseproductsareonlyafewofthecontributionsthesedesertareasmaketothequalityofourlives.Theyhavealsomadeimportantculturalcontributions.

OurnumbersystemisderivedfromthesystemusedbytheancientcivilizationsofArabia.Theuseofirrigationtomakefarmingofdryareaspossiblewasdevelopedbytheinhabitantsofdesertregions.ThenecessityofmeasuringwaterlevelsandnotinglandboundariesfollowingfloodingbytheNileRiverledtothedevelopmentofmathematicsandthepracticesofsurveyingandengineering.Thedesertpeoplewerealsoourearlyastronomers.Theystudiedthelocationsofthestarsinordertofindtheirwayacrossthelimitlessexpanseofthedesertatnight.

Thenexttimeyougazeatadesertspaceonamap,then,youmightreflectonthenumerouscontributionsthatdesertareashavemadetoourheritage.Thecommonnotionthatdesertsholdlittleofinterestismerelyamyth.

13.Thepopulationoftheworld’sdesertsis

(A)scattered.

(B)starving.

(C)large.

(D)small.

14.TheImperialValleyproduces

(A)vegetables.

(B)winterdinners.

(C)shirts.

(D)irrigation.

15.Accordingtothispassage,broadclothismadeof

(A)wool.

(B)cotton.

(C)silk.

(D)halfwool,halfcotton.

16.Culturally,desertcivilizationshave

(A)madenocontributions.

(B)madeimportantcontributions.

(C)notinfluencedwesterncivilizations.

(D)beenblankspaces.

17.Surveyingwasdevelopedbecausepeopleneededto

(A)studyastronomy.

(B)findtheirwayacrossthedeserts.

(C)determinelandboundariesafterfloods.

(D)irrigatetheircrops.

18.Accordingtothepassage,thedevelopmentofmathematicswasinfluencedbytheareaofthe

(A)CoachellaValley.

(B)ImperialValley.

(C)NileRiver.

(D)SaltRiverValley.

ResidentsofMontanalaughinglyrefertothesmall,windblownsettlementofEkalakaintheEasternbadlandsas“SkeletonFlats,”butascuriousasitmaysound,thenameisappropriate.

Somanyfossilshavebeendugupinthisotherwiseunremarkabletownthatithasbecomeaparadiseforpaleontologists,scientistswhousefossilstostudyprehistoriclifeforms.Infact,dinosaurbonesaresoplentifulinthisareathatranchershavebeenknowntousethemasdoorstops!

Ekalaka’sfamebegantogrowmorethan50yearsagowhenWalterH.Peck,whosehobbywasgeology,foundthebonesofaStegosaurus,ahuge,plant-eatingdinosaur.TheentirecommunitysoonbecameinfectedwithPeck’senthusiasmforhisfind,andeveryonebegandiggingfordinosaurbones.Ledbythelocalscienceteacher,groupsofpeoplewouldgooutlookingfornewfindseachweekend,andtheyrarelyreturnedempty-handed.ItwouldseemthereisnoendtothefossilrichestobefoundinEkalaka.

AmongthemostprizedfindsweretheremainsofaBrontosaurus,an80-foot-longmonsterthatprobablyweighed40tons.TheskeletonofaTriceratopswasalsofound.Theheadofthisprehistoricgiantaloneweighedmorethan1,000pounds.Carefulsearchingalsoyieldedsmallfossilizedfish,completewithstonyscales,andtheremainsofahugeseareptile.

TheprizefindwasaPachycephalosaurus,adinosaurwhosepeculiarskullwasseveralinchesthick.Whendescriptionsofitreachedscientificcirclesintheeast,therewasgreatexcitementbecausethisparticularprehistoricanimalwasthencompletelyunknowntoscientists.

ResearchershaveusedthefossilremainsinEkalakaandthesurroundingareatohelppiecetogethermoreofwhatweknowaboutthebehaviorsofdinosaurs.Onewell-studiedfindingisthefossilofaTyrannosaurusrexknownasJane.Markings

discoveredontheleftsideofJane’sfaceindicatethatshewasmostlikelybittenbyanotherT.rexabouthersize.AsJanewasonly11or12whenshedied—ateenagerindinosauryears—theinjuriessuggestthatevenrelativelyyoungdinosaursmighthavebecomeinvolvedinferociousfighting,abehavioraldynamicthattheMontanafossilfindingshavehelpedtoconvey.

19.Inthefirstsentence,theauthorplaces“SkeletonFlats”inquotationmarkstoshowthatthisphraseis

(A)anicknamegiventothetownbyMontanaresidents,nottheactualnameofthetown.

(B)spelledincorrectly.

(C)beingspokenbysomeoneotherthantheauthor.

(D)ascientificterm.

20.Thisarticleisprimarilyabout

(A)paleontology.

(B)productsofthestateofMontana.

(C)fossilfindsinEkalaka.

(D)thePachycephalosaurus.

21.Accordingtothispassage,apaleontologistis

(A)someonewhosehobbyisgeology.

(B)aparadise.

(C)aplant-eatingdinosaur.

(D)someonewhostudiesfossils.

22.Inthethirdparagraph,theauthorisdescribingthe

(A)bonesofaStegosaurus.

(B)discoveryofthefirstfossilfindsinEkalaka.

(C)townofEkalaka.

(D)peopleofEkalaka.

23.DiscoveryofthePachycephalosauruscausedexcitementbecause

(A)itsskullwasseveralinchesthick.

(B)itwasthefirstevidenceofthiscreatureeverfoundandreportedtoscientists.

(C)newsofitreachedeasternscientificcircles.

(D)itreceivedaprize.

24.ThediscoveryofthedinosaurknownasJanerevealedthat

(A)ancientfishmostlikelywerecoveredinscales.

(B)theTriceratopshadanextremelylargehead.

(C)dinosaurswerelikelytobebittenbycrocodiles.

(D)teenagedinosaursengagedinintensecombat.

Powderedzirconiumismorefieryandviolentthanthemagnesiumpowderthatwentintowartimeincendiarybombs.Undersomeconditions,itcanbeignitedwithakitchenmatch,anditcannotbeextinguishedwithwater.Munitionsmakersoncetriedtoincorporateitintoexplosives,butturneditdownastoodangerousforeventhemtohandle.

Butwhenthisstrangemetalistransformedintoasolidbarorsheetortube,aslustrousasburnishedsilver,itstemperchanges.Itissodocilethatitcanbeusedbysurgeonsasasafecoveringplateforsensitivebraintissues.Itisalmostasstrongassteel,anditcanbeexposedtohydrochloricacidornitricacidwithoutcorroding.Itisevenusedinnuclearreactorsbecauseofitsabilitytowithstandcorrosion.

Zirconiumisalsosafeandstablewhenitisboundupwithotherelementstoformmineralcompounds,whichoccurinabundantdepositsinNorthandSouthAmerica,India,andAustralia.Althoughitisclassifiedasararemetal,itismoreabundantintheearth’scrustthannickel,copper,tungsten,tin,orlead.Untilnotlongago,scarcelyadozenpeoplehadeverseenzirconiuminpureform,buttodayitisthewondermetalofafantasticnewindustry,anexcitingstructuralmaterialforchemicalequipmentandforsuperrocketsandjetengines,andavitalcomponentoftelevision,radar,andradiosets.Itmaybemostwell-knownforitsroleinjewelrymaking,ascubiczirconiumgemsresemblediamondsinappearancebutsellforafractionoftheprice.

Despiteitswidespreaduseinmanydifferentapplications,somecontemporaryusesforzirconiumhavebeentriedanddiscarded.Itsirritatingpropertiesmakeitapoorchoiceforinclusioninskinproducts,asmanufacturersdiscoveredwhenitwastriedasacomponentintopicalskintreatmentsanddeodorants.

25.Whichtitlebestexpressesthemainideaofthisselection?

(A)“AVitalSubstance”

(B)“ASafe,StableSubstance”

(C)“Zirconium’sUsesinSurgery”

(D)“CharacteristicsofZirconium”

26.Theworddocilemeans

(A)calm.

(B)pliable.

(C)strong.

(D)profuse.

27.Theselectionemphasizesthat

(A)zirconiumrustseasily.

(B)chemistshavefoundmanyusesforzirconium.

(C)keysareoftenmadeofzirconiumnowadays.

(D)zirconiumislessabundantintheearth’scrustthanlead.

28.Zirconiumisnotsafetohandlewhenitis

(A)lustrous.

(B)powdered.

(C)intubes.

(D)inbarform.

29.Theselectiontellsusthatzirconium

(A)isametal.

(B)isfireproof.

(C)dissolvesinwater.

(D)isstrongerthansteel.

30.ZirconiumislikelytobeusefulinallofthesefieldsEXCEPT

(A)surgery.

(B)television.

(C)atomicresearch.

(D)themanufactureoffireworks.

Between1780and1790,inpiecemealfashion,atrailwasestablishedbetweenCatskillontheHudsonandthefrontieroutpost,Ithaca,intheFingerLakescountry.Thispath,bygraceoffollowingthevalleys,managedtothreaditswaythroughthemountainsbywhatareonthewholesurprisinglyeasygrades.Ultimately,thisroute

becametheSusquehannaTurnpike,butinpopularspeechitwasjusttheIthacaRoad.Itwas,alongwiththeMohawkTurnpikeandtheGreatWesternTurnpike,oneofthethreegreateast-westhighwaysofthestate.EventuallyitwastheroutetakenbythousandsofYankeefarmers,moreespeciallyConnecticutYankees,seekingnewfortunesinsouthwesternNewYork.Alongit,thetideofpioneerimmigrationflowedatfloodcrestforafullgeneration.

AstheroadleftCatskill,therewasnostreamthatmightnotbeeitherfordedorcrossedonacrudebridgeuntilthetravelerreachedtheSusquehanna,whichwasaconsiderableriverandarealobstacletohisprogress.TheroadcamedownoutoftheCatskillsviathevalleyoftheOuleoutCreekandstrucktheSusquehannajustabovethepresentvillageofUnadilla.Hitherabouttheyear1784cameaConnecticutman,NathanielWattles,whosettledthereanddedicatedhimselftohelpingtravelerscrosstheformidableriver.Wattlesprovidedbothaskiffandalargeflat-bottomedscowsothatthehomeseeker,hisfamily,team,andhouseholdbaggage—andoftentimesalittlecaravanoflivestock—mightbesetacrosstheriverdry-shodandinsafety.Wattleshereestablishedaninnwhereonemightfindlodgingandentertainment,andageneralstorewheremightbepurchasedsuchstaplesaswereessentialforthejourney.Healsoopenedroadsthatenabledsettlerstotravelfromtheareainanydirection.SoitwasthatWattles’FerrybecamethebestknownlandmarkontheIthacaRoad.

31.TheauthorindicatesthattheSusquehannaTurnpike

(A)beganasanarrowtrail.

(B)wasthemostimportantnorth-southhighwayinthestate.

(C)furnishedtravelerswithsurprisingobstacles.

(D)wentoutofuseafterageneration.

32.ThewesternendoftheSusquehannaTurnpikewaslocatedat

(A)theHudsonRiver.

(B)theConnecticutborder.

(C)Ithaca.

(D)Catskill.

33.TheSusquehannaTurnpikewasalsoknownas

(A)theIthacaRoad.

(B)Wattles’Ferry.

(C)theCatskillTrail.

(D)theMohawkTurnpike.

34.Accordingtothisselection,NathanielWattleswaspreparedtooffertravelersallofthefollowingEXCEPT

(A)guides.

(B)aplacetosleep.

(C)entertainment.

(D)groceries.

35.Thewordstaplesmostnearlymeans

(A)fasteners.

(B)supplies.

(C)transportation.

(D)experiences.

36.ThepurposeofWattles’Ferrywasto

(A)providetravelerswithovernightlodging.

(B)helpresidentsestablishhomesinUnadilla.

(C)helptravelerscrosstheSusquehannaRiver.

(D)teachfamiliesthetechniquesofriverrafting.

SECTION4:MATHEMATICSACHIEVEMENT47Questions•40Minutes

Directions:Eachquestionisfollowedbyfouranswerchoices.Choosethecorrectanswertoeachquestion.

1.Asquaremeasures8inchesononeside.Byhowmuchwilltheareabeincreasedifitslengthisincreasedby4inchesanditswidthdecreasedby2inches?

(A)14sq.in.

(B)12sq.in.

(C)10sq.in.

(D)8sq.in.

2.r=35–(3+6)(–n)n=2r=

(A)53

(B)17

(C)–17

(D)–53

3.(3+4)3=

(A)21

(B)91

(C)343

(D)490

4.WhichvalueisNOTequalto ?

(A)

(B)0.4444444

(C)

(D)

5.Aluminumbronzeconsistsofcopperandaluminum,usuallyintheratioof10:1byweight.Ifanobjectmadeofthisalloyweighs77pounds,howmanypoundsofaluminumdoesitcontain?

(A)7

(B)7.7

(C)10

(D)70

6.Howmanyboxes2inches×3inches×4inchescanfitintoacarton2feet×3feet×4feet?

(A)100

(B)144

(C)1000

(D)1728

7.Aclerkcanadd40columnsoffiguresanhourbyusinganaddingmachineand20columnsoffiguresanhourwithoutusinganaddingmachine.Whatisthetotalnumberofhoursit

willtaketheclerktoadd200columnsoffiguresif oftheworkisdonebymachineand

therestwithoutthemachine?

(A)6hours

(B)7hours

(C)8hours

(D)9hours

8.Mr.Lawsonmakesaweeklysalaryof$150plus7%commissiononhissales.Whatwillhisincomebeforaweekinwhichhemakessalestotaling$945?

(A)$196.15

(B)$206.15

(C)$216.15

(D)$226.15

9.Solveforx:x2+5=41.

(A)±6

(B)±7

(C)±8

(D)±9

10.Tworectangularboardseachmeasuring5feetby3feetareplacedtogethertomakeonelargeboard.Howmuchshorterwilltheperimeterbeifthetwolongsidesareplacedtogetherthanifthetwoshortsidesareplacedtogether?

(A)2feet

(B)4feet

(C)6feet

(D)8feet

11.Ifaplanetravels1000milesin5hours30minutes,whatisitsaveragespeedinmilesperhour?

(A)

(B)

(C)200

(D)215

12.Twoyearsagoacompanypurchased500dozenpencilsat40centsperdozen.Thisyearonly75percentasmanypencilswerepurchasedaswerepurchasedtwoyearsago,butthepricewas20percenthigherthantheoldprice.Whatwasthetotalcostofpencilspurchasedbythecompanythisyear?

(A)$180

(B)$187.50

(C)$240

(D)$257.40

13.Anadult’sskiliftticketcoststwiceasmuchasachild’s.Ifafamilyofthreechildrenandtwoadultscanskifor$49,whatisthecostofanadultticket?

(A)$7

(B)$10

(C)$12

(D)$14

14.Solveforx: +36=37.25.

(A)2.5

(B)3.5

(C)12.5

(D)18.5

15.Agroupof6peopleraised$690forcharity.Oneofthepeopleraised35%ofthetotal.Whatwastheamountraisedbytheother5people?

(A)$448.50

(B)$241.50

(C)$89.70

(D)$74.75

16.Whichexpressionisequivalenttotheexpression(y+5)(y–1)?

(A)y2–5

(B)y2+4

(C)y2–4y–5

(D) y2+4y–5

17.Ifthescaleonablueprintis inch=1foot,givetheblueprintdimensionsofaroomthat

isactually29feetlongand23feetwide.

(A)

(B)

(C)

(D)

18.Findtheareaofarectanglewithalengthof176feetandawidthof79feet.

(A)13,904sq.ft.

(B)13,854sq.ft.

(C)13,804sq.ft.

(D)13,304sq.ft.

19.63÷ =

(A)7

(B)56

(C)67

(D)567

20.Withan18%discount,Johnwasabletosave$13.23onacoat.Whatwastheoriginalpriceofthecoat?

(A)$69.75

(B)$71.50

(C)$73.50

(D)$74.75

21.Ifittakesthreemen56minutestofillatrench4'×6'×5',andtwoofthemenworktwiceasrapidlyasthethird,howmanyminuteswillittakethetwofastermenalonetofillthistrench?

(A)70minutes

(B)60minutes

(C)50minutes

(D)40minutes

22.Populationfiguresforacertainareashowthereare timesasmanysinglemenas

singlewomeninthearea.Thetotalpopulationis18,000.Thereare1122marriedcouples,with756children.Howmanysinglemenarethereinthearea?

(A)3000

(B)6000

(C)9000

(D)Itcannotbedeterminedfromtheinformationgiven.

23.Ifavehicleistocompletea20-miletripatanaveragerateof30milesperhour,itmustcompletethetripin

(A)20minutes.

(B)30minutes.

(C)40minutes.

(D)50minutes.

24.Solveforx:2x2+3=21.

(A)±3

(B)±5

(C)±9

(D)±10

25.Findtheareaofacirclewhosediameteris6".

(A)29.26

(B)28.26

(C)27.96

(D)27.26

26.Thescaleonamapis =25miles.Iftwocitiesare apartonthemap,whatisthe

actualdistancebetweenthem?

(A)31miles

(B)56miles

(C)675miles

(D)775miles

27.Ahousewasvaluedat$83,000andinsuredfor80%ofthatamount.Findtheyearlypremiumifitisfiguredat$0.45perhundreddollarsofvalue.

(A)$83.80

(B)$252.63

(C)$298.80

(D)$664

28.Ifacertainjobcanbeperformedby18clerksin26days,thenumberofclerksneededtoperformthejobin12daysis

(A)24clerks.

(B)30clerks.

(C)39clerks.

(D)52clerks.

29.72.61÷0.05=

(A)1.45220

(B)14.522

(C)145.220

(D)1452.20

30.Acardealersoldthreedifferentmakesofcars.Thepriceofthefirstmakewas$4200,thesecond$4800,andthethird$5400.Thetotalsaleswere$360,000.Ifthreetimesasmanyofthethirdcarweresoldasthefirst,andtwiceasmanyofthesecondmakeweresoldthanthefirst,howmanycarsofthethirdmakeweresold?

(A)15

(B)24

(C)36

(D)Itcannotbedeterminedbytheinformationgiven.

31.Onethirdofthenumberofpeopleattendingafootballgamewereadmittedatthenormalpriceofadmission.Howmanypeoplepaidfullprice,ifthegatereceiptswere$42,000?

(A)2800people

(B)3500people

(C)5000people

(D)Itcannotbedeterminedbytheinformationgiven.

32.7days3hours20minutes–4days9hours31minutes=

(A)2days17hours49minutes

(B)2days17hours69minutes

(C)3days10hours49minutes

(D)3days10hours69minutes

33.Findtheareaofatrianglewhosedimensionsareb=12',h=14'.

(A)168sq.ft.

(B)84sq.ft.

(C)42sq.ft.

(D)24sq.ft.

34.Increasedby150%,thenumber72becomes

(A)108

(B)170

(C)180

(D)188

35.Whichequationrepresentsthestatementfourtimesacertainnumberdividedbythree,minussix,equalstwo?

(A) –6=2

(B)4n2–6=2

(C)4n2÷3–6=2

(D) –6=2

36.If14x–2y=32andx+2y=13,thenx=

(A)8

(B)5

(C)4

(D)3

37.Anordinarydieisthrown.Whataretheoddsthatitwillcomeup1?

(A)

(B)

(C)

(D)

38.Whichisthelongesttime?

(A)25hours

(B)1440minutes

(C)1day

(D)3600seconds

39.Twocarsare550milesapart,bothtravelingonthesamestraightroadtowardeachother.Ifonetravelsat50milesperhour,theotherat60milesperhour,andtheybothleaveat1p.m.,whattimewilltheymeet?

(A)4p.m.

(B)4:30p.m.

(C)5:45p.m.

(D)6p.m.

40.Write493inexpandedform,usingexponents.

(A)(4×103)+(9×102)+(3×10)

(B)(4×102)+(9×10)+3

(C)(4×102)+(9×10)–7

(D)(4×101)+(9×10)+3

41.If10workersearn$5400in12days,howmuchwill6workersearnin15days?

(A)$10,500

(B)$5400

(C)$4050

(D)$2025

42.Thescaleofaparticularmapis =5miles.IfthedistancebetweenpointsAandBis

onthemap,whatisthedistanceinactuality?

(A)12miles

(B)36miles

(C)48miles

(D)60miles

43.Findthediameterofacirclewhoseareais78.5sq.in.

(A)25feet

(B)10feet

(C)25inches

(D)10inches

44.Ifab+4=52,anda=6,b=

(A)4

(B)8

(C)21

(D)42

45.If ofajarisfilledwithwaterin1minute,howmanyminuteslongerwillittaketofill

theremainderofthejar?

(A)

(B)

(C)

(D)

46.Agroupleftonatripat8:50a.m.andreacheditsdestinationat3:30p.m.Howlong,inhoursandminutes,didthetriptake?

(A)3hours10minutes

(B)4hours40minutes

(C)5hours10minutes

(D)6hours40minutes

47.Asquareischangedintoarectanglebyincreasingitslength10%anddecreasingitswidth10%.Itsarea

(A)remainsthesame.

(B)decreasesby10%.

(C)increasesby1%.

(D)decreasesby1%.

SECTION5:ESSAY30Minutes

Directions:Writealegible,coherent,andcorrectessayonthefollowingtopic.

Topic:Ifyoucouldspendanafternoonwithanyauthor,livingordead,withwhomwouldyouspendit?Whatwouldyoutalkabout?

ANSWERSANDEXPLANATIONSSection1:VerbalReasoning

1.Thecorrectansweris(A).ToFEINTistodeceiveormakeapretenseof.Oneoftheskillsoursoccercoachtaughtuswashowtoeludeopponentsbyfeintingintheoppositedirectionofwhereweplannedtopasstheball.

2.Thecorrectansweris(C).APEERissomeoneorsomethingthatisofequalstandingtoanother.Inademocraticsociety,eachcitizenisthepeerofalltheothercitizens.

3.Thecorrectansweris(B).TRITEmeansboringfromtoomuchuse.Heressaywasriddledwithtriteexpressions,suchas“apennysavedisapennyearned”and“allgoodthingscometothosewhowait.”

4.Thecorrectansweris(B).AMIABLEmeansgood-naturedandloveable.Theamiabledogwaggeditstailatthelettercarrier.

5.Thecorrectansweris(A).AGRIMACEisafacialexpressionofdisgustordispleasure.Thefirsttimethebabyatespinach,shescreweduphermouthinagrimaceandspitoutthefood.

6.Thecorrectansweris(D).ToCOMPEListoforce.OurgovernmentcompelsustopayincometaxeseachyearonApril15th.

7.Thecorrectansweris(C).AnALLYisanotherwithacommonpurpose,anassociate,orahelper.Thewordmeaning“passage”is“alley.”GreatBritainwasourallyduringWorldWarII.

8.Thecorrectansweris(C).ToSOLICITmeanstoapproachwitharequestorplea.Our

teacherwantsustosolicithelpfromherwheneverwedon’tunderstandthemathconceptsshepresentstotheclass.

9.Thecorrectansweris(C).ToREFUTEistoshowsomethingtobefalse.Thelawyerrefutedthewitness’stestimonybypresentingcontradictoryevidence.

10.Thecorrectansweris(B).EXPLICITmeansdistinct,observable,orclearlystated.Theexplicitinstructionsonthepackageleftnoopportunityformisunderstandingorerror.

11.Thecorrectansweris(C).ToRETAINistoholdontoortokeep.Retainthereceiptasproofofpayment.

12.Thecorrectansweris(A).CORRESPONDENCEisanexchangeoflettersorthelettersthemselves.IsaveallmycorrespondencewiththeschoolstowhichIhaveappliedinanenvelopemarked“HighSchools.”

NOTE:“Correspondence”canalsomean“agreement”or“conformity.”Yourexamwillneverofferyouachoiceoftwocorrectmeaningsforthesameword.

13.Thecorrectansweris(B).LEGITIMATEmeansconformingtothelaworabidingbytherules.Sincehisnameisonthedeed,hehasalegitimateclaimtoownershipoftheproperty.

14.Thecorrectansweris(C).ToDEDUCTistosubtract.Thewordwithameaningcloseto“understand”is“deduce.”Eachweek,myemployerdeductssocialsecuritytaxesfrommypaycheck.

15.Thecorrectansweris(C).TheEGRESSisthewayout.Theegressismarkedwithared“EXIT”sign.

16.Thecorrectansweris(C).HORIZONTAL,paralleltothehorizon,meansleft-to-rightasopposedto“vertical.”Placethelargebookshorizontallyontheshelfsothattheydonottoppleover.

17.Thecorrectansweris(B).CONTROVERSYistheexchangeofopposingopinionsorargument.Thechoiceofnewwallpaperisasubjectofcontroversy.

18.Thecorrectansweris(D).ToPREEMPTistoseizebeforeanyoneelsecanortoappropriate.Nodishonestyisimplied,justspeedorprivilege.Thepresident’sspeechwillpreemptthetimeslotusuallytakenbymyfavoritegameshow.

19.Thecorrectansweris(C).PERCAPITAliterallymeansforeachhead,thereforeforeachperson,one-by-one,withageirrelevant.Thepercapitaconsumptionofredmeathasdroppedto2poundsperweek.

20.Thecorrectansweris(A).ThatwhichisOPTIONALislefttoone’schoiceandisthereforenotrequired.YoumuststudyEnglishandhistory,butstudyofamusicalinstrumentisoptional.

21.Thecorrectansweris(C).Thesenseofthesentencecallsforawordwithanegativeconnotationinthefirstblank;therefore,weneedconsideronlychoices(C)and(D).Ofthesechoices,ENSLAVED...SAY,choice(C),isclearlythebettercompletion.

22.Thecorrectansweris(D).Sinceitisstatedthatmostcriticsdisregardedorridiculedtheplay,thefewcriticsremainingmusthavedonetheopposite,orAPPRECIATEDthework.

23.Thecorrectansweris(C).Thewordthatisneededmustbeasynonymforblunder(astupidorgrossmistake).ThatwordisERROR.

24.Thecorrectansweris(C).OnewhoactspurelyonimpulseismostlikelytohaveaPASSIONATE(emotionalorintense)nature.

25.Thecorrectansweris(D).If“qualityispreferredtoquantity”inaneducationalsystem,thenthemeasurebywhichthatsystemshouldbeJUDGEDistheCALIBER(degreeofabilityormerit)ofthestudentsitproduces.

26.Thecorrectansweris(B).Freedomofspeechissomethingwetakeforgranted,sowedonotfeelBLESSEDwhenallowedtoexercisethisfreedom;however,wedobecomeangrywhenanyRESTRICTION(limit)isimposedonourrighttospeakfreely.

27.Thecorrectansweris(C).Thequalitiesattributedtotheplayersontheworstteammustbeoppositesforcomparisonandadjectivesforparallelismwithinthesentence.INEPT,whichmeansawkward,andPROFICIENT,whichmeansskilled,makeuptheonlychoicethatmeetsbothrequirements.

28.Thecorrectansweris(D).Itisthefunctionofthelegislaturetowritelaws(theirWORDSbecomelaw).Itisthefunctionofthejudiciarytointerpretthewordsofthelaw(todeterminetheirMEANING).

29.Thecorrectansweris(B).Moviesandtelevisionarebothmediaofentertainment.Thesentencecomparesthetwomediaintermsoftheircost,statingthatmanypeoplebelievetelevision(whichisfreeaftertheinitialinvestmentintheset)isNECESSARYbecausemovieshavebecomesoEXPENSIVE(andthereforeoutofreachformanypeople).

30.Thecorrectansweris(A).Thesenseofthesentencecallsfortwowordsthatareoppositesandthatcanbothbeappliedtolifeforms.Sporesarethetinyparticlesincertainplantsthatactasseedsintheproductionofnewplants.ThesesporesremaindormantorINACTIVEuntiltheproperconditionsexisttorenderthemvigorousorVIBRANT,thuscreatinganewgenerationofplants.

31.Thecorrectansweris(B).Thecompletionneededisawordthatisoppositeinmeaningto“thespinningoffancifultheories.”Ofthechoicesgiven,thestudyofORDERbestfulfillsthisrequirement.

32.Thecorrectansweris(B).Thissentencedescribesarebelliousattitude.Impertinencemeans“insolence.”

33.Thecorrectansweris(B).Thesecondsentenceprovidesthecluetothemeaningofthefirst.Ifsuccessdoesnotmeaninfallibility(certainty),thenthefameofanauthordoesnotASSUREthequalityofhisorherwork.

34.Thecorrectansweris(D).Thecompletionheredemandswordsthatareopposites.Inaddition,thefirstblankrequiresawordthatwouldpromotehatred.OnlySTRIFE,meaningconflict,andTRANQUILITY,meaningpeace,fulfilltheserequirementsandcompletethemeaningofthesentence.

35.Thecorrectansweris(A).Sinceminingtakesawaywithoutreplacingwhatittakes,itmaybecalledaROBBERindustry.Withthesecharacteristics,miningmightalsobeconsideredtobeevil,butROBBERisthemostspecificcompletion.Itistheadjectivethatbestdescribesanindustrythatdoesnotreplenishwhatittakes.

36.Thecorrectansweris(C).Thissentencepresentstwoproblemsthatarebeingcomparedintermsoftheeaseoftheirsolution.TheonlychoicesthatfulfilltherequirementsofsuchacomparisonareMAGNITUDEandCHILD’SPLAY.

37.Thecorrectansweris(B).Atheatricalsettingservestocreateamoodorafeelingofbeinginanothertimeorplace.IfthesettingistobeEFFECTIVE(tomakethedesiredimpressionontheaudience),itmusthavesomesemblanceofREALITY.

38.Thecorrectansweris(D).Theword“mob”hasanegativeconnotationandrequiresanadjectivethatisalsonegative.HYSTERICAL(emotionalandunmanageable)bestmeetsthisrequirement.TheemotionthatashoutingmobismostlikelytoshowisHATREDoflawandorder.

39.Thecorrectansweris(C).Whenerrorsarediscoveredinexistingtheories,thosetheoriesmusteitherbeALTERED(changed)inthelightofthenewinformationortheymustbeDISCARDEDaltogether,ifthenewinformationrenderstheoldtheoriesfalse.

40.Thecorrectansweris(B).Thesentencecomparestwodifferentsocietiesandthereforerequirescompletionsthatarebothparallelandopposite.ELEMENTARY(simple)andCOMPLICATED(intricate)bestmeettheserequirements.

Section2:QuantitativeReasoning

1.Thecorrectansweris(C).Ifx=0,then2x<8because2(0)<8.Noneoftheotherpairsresultinatruestatement.

2.Thecorrectansweris(C).Substitutenfortheblankspace.7(3×n)+4=21047(3n)+4=210421n+4=210421n=2100

n=100,or102

3.Thecorrectansweris(A).Since1%=0.01,onehalfofonepercentiswritten0.005.Refertothepercentagereviewsectionforhelpifnecessary.

4.Thecorrectansweris(C).Ascalenetrianglehasnoequalsides.

5.Thecorrectansweris(A).Tosolvefora,add2btobothsidesoftheequation.Thisgivesusa=–7+2b.Wecanreordertheexpressiontoread2b–7.

6.Thecorrectansweris(C).Thereare1000millimetersinameter.

7.Thecorrectansweris(D).Choice(A)isalsoacommondenominator,butitisnottheleastcommondenominator.

8.Thecorrectansweris(A).Theareaofatriangleisfoundbyusing

A= bh

A= •14•20

=140sq.in.

9.Thecorrectansweris(B).

(4×103)+6=4006

(2×103)+(3×10)+8=2038

Thedifferenceis1968.

10.Thecorrectansweris(A).Thesetoffactorsfor24is:{1,2,3,4,6,8,12,24}

Thesetoffactorsfor30is:

{1,2,3,5,6,10,15,30}

Thesetofcommonfactorsis:

{1,2,3,6}

11.Thecorrectansweris(A).Theintersectionofthetwolinesegmentsistheplacetheyoverlap.Notethattheyoverlapintheintervalmarked .

12.Thecorrectansweris(B).Performthecalculations,usingtheorderofoperations:PEMDAS.First,performthecalculationsinparentheses:

Next,performcalculationsinvolvingexponents:

Now,performmultiplicationanddivision:

13.Thecorrectansweris(A).Onethirdoftheboardwillbedrivenintotheground,leaving4feetexposed.Theexposedpartoftheboardhas5faces:twofaces4feetlongby4incheswide;twofaces4feetlongby2incheswide;andoneface(theend)2inchesby4inches.Becausetheanswerchoicesareinunitsofsquarefeet,wewillcalculateinsquarefeet:

Thesumis sq.ft.ofboardremainingaboveground.

14.Thecorrectansweris(D).Therateofthefirstrunneris milesperhour.Therateof

thesecondis milesperhour.Thesecondrunnerisfaster,sothedifferenceintheirrates

iswritten .

15.Thecorrectansweris(C).Mary’sagenow=x

Hersister’sagenow=x–3In5yearshersister’sagewillbex–3+5=x+2

16.Thecorrectansweris(C).Theareaofasquare=s2

Theareaofthissquare=22=4

Theareaofacircle=π•r2

Theareaofthiscircle

Thedifferencebetweentheareaofthissquareandtheareaofthiscircleis4–3.14=0.86

17.Thecorrectansweris(C).Areaofasquare=s2

49=72

oneside=7inches

P=4sP=4×7"=28inches

18.Thecorrectansweris(C).Onehalfhour=30minutes,soyouwouldmultiplyGby30.Amount=rate(G)×time(30minutes)

19.Thecorrectansweris(C).Onecarwent20mphfor hour=10miles.

Theothercarwent36mphfor hour=18miles.Sincetheywentinoppositedirections,

addthetwodistancestofindthetotalnumberofmilesapart.10+18=28

20.Thecorrectansweris(B).

21.Thecorrectansweris(D).Sincexcouldbeanynon-zerovaluefrom4to–3,thevaluesofthefractionsareimpossibletodetermine.

22.Thecorrectansweris(A).a<b b>a(given).KR>KT(inatrianglethegreatersideliesoppositethegreaterangle)

23.Thecorrectansweris(A).

24.Thecorrectansweris(C).

y=a+b(anexteriorangleofatriangleisequaltothesumofthetwointeriorremoteangles)

25.Thecorrectansweris(D).Thereisnotenoughinformation,asycouldequal1,whichwouldmakebothquantitiesequal;orycouldbegreaterthan1,whichwouldmakey3greaterthany2.Ifywereanegativeinteger,theny2wouldbegreaterthany3.

26.Thecorrectansweris(B).(8+6)÷[3–7(2)]

(6+8)÷[2–7(3)]

27.Thecorrectansweris(C).

28.Thecorrectansweris(B).

NC=NY(given)

C= Y(anglesoppositeequalsidesareequal)

N> C(given)

N> Y(substitution)CY>NC(thegreatersideliesoppositethegreaterangle)

29.Thecorrectansweris(C).

30.Thecorrectansweris(C).

31.Thecorrectansweris(C).Atriangleinscribedinaparallelogramisequalinareatoonehalftheparallelogram.Therefore,theareaof DECequalsthecombinedareasofADEand EBC.

32.Thecorrectansweris(A).

x3+x2–x+1=

(–1)3+(–1)2–(–1)+1

=–1+1+1+1

=2

x3–x2+x–1=

(–1)3–(–1)2+(–1)–1

=–1–1–1–1

=–4

ColumnA>ColumnB

33.Thecorrectansweris(C).

ColumnA=ColumnB

34.Thecorrectansweris(A).

(multiplyingnumeratoranddenominatorby )

=

(multiplyingnumeratoranddenominatorby )

ColumnA>ColumnB

35.Thecorrectansweris(D).Wecannotdeterminetheareasofthecirclesunlessthevalueofxisknown.

36.Thecorrectansweris(A).

=π(102)–π(72)

=100π–49π

=51π

=π(72)

=49π

51π>49π

ColumnA>ColumnB

37.Thecorrectansweris(D).Anumbersmallerthan0isanegativenumber,soaisanegativenumber.Anegativenumbersquaredbecomesapositivenumber.Withoutknowingabsolutevaluesofaandb,thereisinsufficientinformationtodeterminetheanswertothisquestion.

Section3:ReadingComprehension

1.Thecorrectansweris(C).Theselectiongraphicallydetailsthemovementofasnake.Whilemuchofthedescriptionisintermsoflegsandfeet,thepointoftheselectionistofullydescribethemeansoflocomotion.

2.Thecorrectansweris(B).Thesecondsentenceofthesecondparagraphmakesthisstatement.Theremainderoftheparagraphexpandsonthetheme.

3.Thecorrectansweris(D).Thiswordappearsinline8.Readcarefullyandyoucanfigureoutthemeaningfromthecontext.Terrainmeansearth,withreferencetoitstopographicalfeatures.

4.Thecorrectansweris(C).Thedetailinthisselectionindicatesthattheauthorknowsagooddealaboutsnakes.

5.Thecorrectansweris(D).Thesentencestatesthatlocomotionisusedbyahumanorotheranimalgoingforward.Inthecontextofthesentence,locomotionmostnearlymeansmovement.Wecouldsubstitutethewordmovementforlocomotioninthissentence,andthesentencewouldstillmakesense.

6.Thecorrectansweris(C).Serpentinemovementisdescribedinparagraphfourasatypeofmovementthatsnakesuseonlandandeveninthewater.Asimilarmovementisusedbothonlandandwater,sochoice(C)iscorrect.

7.Thecorrectansweris(B).Thelastsentenceofthefirstparagraphtellsusthatropeisabsolutelyvitalfordocking.

8.Thecorrectansweris(D).Thesecondsentenceservesbywayofdefinition.

9.Thecorrectansweris(D).Inlines6–8welearnthattheshipissecuredbynine-inchorten-inchmooringlines.Sinceropeismeasuredbycircumferenceratherthanbydiameter(lines13–14),choice(D)isthecorrectanswer.

10.Thecorrectansweris(A).Checkbackandeliminate.SisaldoesnotcomefromthePhilippines.

11.Thecorrectansweris(A).Thesentencestatesthatthefibershenequenandsisalarealsousedinropemaking,butchieflyfortwine.Thesentenceexplainsthatmostoftheuseofthesefibersoccursinthemakingoftwine.Thewordchieflyhereisclosestinmeaningtotheworldmainly,sochoice(A)iscorrect.

12.Thecorrectansweris(B).Thelastsentenceofthelastparagraphstatesthatnyloncanbecometoostretchedoutforuseincertainapplications,sochoice(B)iscorrect.

13.Thecorrectansweris(D).Greatblankspacesonapopulationmapindicateverysmallpopulation.

14.Thecorrectansweris(A).(Seeline9.)Youmayeatthevegetablesatawinterdinner,butthefarmproducesonlythevegetables;itdoesnotcookthedinner.

15.Thecorrectansweris(B).Inlines9–11welearnthatbroadclothismadefromsilkycottongrowninEgypt.

16.Thecorrectansweris(B).Thethirdparagraphmakesthestatementthatdesertcivilizationshavemadeimportantculturalcontributions.Thelastparagraphtellswhatthesecontributionsare.

17.Thecorrectansweris(C).Seelines17–19.

18.Thecorrectansweris(C).ThepassagestatesinparagraphfourthatmathematicswasdevelopedasaresultoftheneedtomeasurewaterlevelsandnotelandboundariesfollowingfloodingbytheNileRiver.

19.Thecorrectansweris(A).ThenameofthetownisEkalaka,buttheycallit“SkeletonFlats.”

20.Thecorrectansweris(C).Theanswertothismain-ideaquestionshouldbeclear.Thearticleisaboutthevariousfossilfinds.

21.Thecorrectansweris(D).Lines5–6givethedefinition:“...paleontologists,scientistswhousefossilstostudyprehistoriclifeforms.”WalterPeck’shobbywasgeology,andinthecourseofpursuinghishobbyhemadethefirstfind,buthewasnotapaleontologist.

22.Thecorrectansweris(D).ThethirdparagraphdiscussesthepeopleofEkalakaintermsoftheirenthusiasmfordiggingandfossildiscovery.

23.Thecorrectansweris(B).Seethelastsentenceofthefifthparagraph.

24.Thecorrectansweris(D).ThesignificanceofJane’sdiscoveryisdiscussedinthelastparagraphofthepassage.Here,thepassagestatesthatJanehadmarkingsonherfacethatweremostlikelyinflictedbyanotherdinosaurabouthersize.Theseinjuriessuggestthatteenagedinosaurs,suchasJane,mighthavebecomeinvolvedinferociousfighting,sochoice(D)iscorrect.

25.Thecorrectansweris(D).Theselectiondescribesthepropertiesofzirconiuminitsvariousforms.

26.Thecorrectansweris(A).Considertheuseoftheworddocileasappliedtosolidzirconium,incontrasttotheuseofthewordviolentasappliedtopowderedzirconium.

27.Thecorrectansweris(B).Anemphasisoftheselectionisthatincreasingusesarebeingfoundforzirconium.

28.Thecorrectansweris(B).Thefirstparagraphmakesthispoint.

29.Thecorrectansweris(A).Inboththesecondandthirdparagraphs,zirconiumisdescribedasametal.

30.Thecorrectansweris(D).Ifzirconiumistoodangeroustobeusedinammunition,itismostcertainlytoodangeroustobeusedinfireworks.

31.Thecorrectansweris(A).Thefirstparagraphdescribestheoriginaltrailasapath.Theroadisalsodescribedasaneast-westroute.Itpresentedtravelerswithsurprisinglyfewobstacles.

32.Thecorrectansweris(C).Thefrontieroutpost,Ithaca,wasatthewesternendofthehighway.

33.Thecorrectansweris(A).Seeline6.

34.Thecorrectansweris(A).Guidesarenotmentionedintheselection.

35.Thecorrectansweris(B).ThesentencestatesWattlesestablishedageneralstorewheretravelerscouldpurchasesuchstaplesaswereessentialforthejourney.Essentialitemsforthejourneycouldalsobedescribedastravelsupplies,sochoice(B)iscorrect.Thewordfastenersissimilartoonemeaningofthewordstaples,butthisdefinitiondoesnotfitthecontexthere.

36.Thecorrectansweris(C).ThethirdsentenceofthelastparagraphstatesthatNathanielWattlesdedicatedhimselftohelpingtravelerscrosstheformidableriver.HisferryenabledtravelerstogetacrosstheSusquehannaRiverdrylyandsafely,sochoice(C)iscorrect.

Section4:MathematicsAchievement

1.Thecorrectansweris(D).Area=length×widthAreaofsquare=8×8=64sq.in.

Areaofrectangle=(8+4)(8–2)=12×6=72sq.in.

72–64=8sq.in.

2.Thecorrectansweris(A).r=35–(9)(–n)r=35–(9)(–2)r=35–(–18)

r=35+18=53Tosubtractsignednumbers,changethesignofthesubtrahendandproceedasinalgebraicaddition.

3.Thecorrectansweris(C).Firstperformtheoperationwithintheparentheses.Tocubeanumber,multiplyitbyitself,twotimes.

(3+4)3=(7)3=7×7×7=343

4.Thecorrectansweris(C).Thefraction isequivalenttothedecimalnumber

0.4444444.Thenumber4goesoninfinitely,soitcanbewrittenas .Thefraction

reducesto ,soitislargerthan .

5.Thecorrectansweris(A).Copperandaluminumintheratioof10:1means10partscopperto1partaluminum.

Letx=weightofaluminumThen10x=weightofcopper10x+x=7711x=77x=7

6.Thecorrectansweris(D).Volume=L×W×H

Volumeofcarton =2'×3'×4'=24cubicfeet

Volumeofonebox =2"×3"×4"=24cubicinches

1cubicfoot =12"×12"×12"=1728cubicinches

=1728boxeswillfitinthecarton

7.Thecorrectansweris(B). of200=120columnsbymachine@40columnsperhour

=3hours

200–120=80columnswithoutmachine@20columnsperhour=4hours

3hours+4hours=7hourstocompletethejob.

8.Thecorrectansweris(C).Histotalincomeisequalto7%ofhissalesplus$150;7%ofhissalesis$945×0.07=$66.15.

$66.15+$150=$216.15

9.Thecorrectansweris(A).

Ifx2+5=41

x2=41–5

x2=36x=±6

10.Thecorrectansweris(B).Perimeter=2l+2wIfthetwolongsidesaretogether,theperimeterwillbe

5+3+3+5+3+3=22

Ifthetwoshortsidesaretogether,theperimeterwillbe

3+5+5+3+5+5=26

26–22=4feetshorter

11.Thecorrectansweris(A).5hours30minutes= hours

1000miles÷5 hours=

12.Thecorrectansweris(A).500dozen@$0.40perdozen=purchaseoftwoyearsago

75%of500dozen=375dozenpencilspurchasedthisyear

20%of$0.40=$0.08increaseincostperdozen

375×$0.48=$180spentonpencilsthisyear

13.Thecorrectansweris(D).Achild’sticketcostsxdollars.Eachadultticketcoststwiceasmuch,or2xdollars.2(2x)=2adulttickets;3x=3children’stickets.Writeasimpleequation,andsolveforx.

$7isthecostofachild’sticket;$14isthecostofanadult’sticket.

14.Thecorrectansweris(A).

15.Thecorrectansweris(A).Onepersonraised35%of$690.

$690×0.35=$241.50.

Theremainderraisedbytheotherswas$690–241.50=$448.50

16.Thecorrectansweris(D).Todeterminethevalueofthisexpression,multiplythebinomialsusingFOIL.Multiplythefirst,outer,inner,andlastterms:

17.Thecorrectansweris(C).Forthelength,29feetwouldberepresentedby29unitsof

,resultingin ,or inches.Forthewidth,23feetwouldberepresentedby23

unitsof ,resultingin ,or inches.

18.Thecorrectansweris(A).Area=length×width

=176ft.×79ft.

=13,904sq.ft.

19.Thecorrectansweris(D).

Thisisagoodanswertoestimate.Bydividinganumberby ,youare,ineffect,

multiplyingitby9.Onlyoneofthesuggestedanswersisclose.

20.Thecorrectansweris(C).Theproblemasks,“Whatnumberis$13.2318%of?”$13.23÷0.18=$73.50.

21.Thecorrectansweris(A).Eachfastworkerisequivalenttotwoslowworkers;therefore,thethreemenaretheequivalentoffiveslowworkers.Thewholejob,then,requires5×56=280minutesforoneslowworker.Italsorequireshalfthattime,or140minutes,foronefastworker,andhalfasmuchagain,or70minutes,fortwofastworkers.

22.Thecorrectansweris(C).Subtractfromthetotalpopulationof18,000the756childrenandthe2244marriedpeople.18,000–756–2244=15,000singlemenandwomen.

Becausethereare1 timesasmanymenaswomen,weknowthat60%ofthe15,000

singlepeoplearemen,and40%arewomen.60%of15,000=9000.

23.Thecorrectansweris(C).Nocalculationsareneededhere.Notethata20-miletripat60mph(whichis1mileperminute)wouldtake20minutes.Sincethevehicleistravelinghalfasfast(30mph),the20-miletripshouldtaketwiceaslong,or40minutes.

24.Thecorrectansweris(A).

2x2+3=21

2x2=21–3

2x2=18

x2=9x=±3Youshouldhavebeenabletopredictthatxwouldbeasmallnumber,since,accordingtotheequation,twiceitssquareisnolargerthan21.

25.Thecorrectansweris(B).TheareaofacircleisA=πr2;theradiusequals the

diameter.r=3,and ,or3.14.

A=πr2

A=π(3)2

A=9π

A=9(3.14)=28.26sq.in.

26.Thecorrectansweris(D).Thescaleis =25miles.In thereare31 units.

Thedistanceis31•25=775miles.

27.Thecorrectansweris(C).Theamountthehousewasinsuredforis80%of$83,000,or$66,400.Theinsuranceiscalculatedat45¢perhundred,or$4.50perthousandofvalue.Sincethereare66.4thousandsofvalue,66.4×$4.50perthousandequalstheyearlypremiumof$298.80.

28.Thecorrectansweris(C).Thesizeofthejobcanbethoughtofthisway:18clerksworkingfor26daysdo18×26or468clerk-daysofwork.Todo468clerk-daysofworkinonly12dayswouldrequire468÷12=39clerks.

29.Thecorrectansweris(D).Thedigitsareallalike,soyoudonotneedtocalculate.Movethedecimalpointofthedivisortwoplacestotheright;dothesameforthedividend.

30.Thecorrectansweris(C).Solvethisproblemasyouwouldanymixture-valueproblem.Thenumbersofcarssoldareallrelatedtothenumberofthosesoldfor$4200.Callthenumberof$4200carssoldx.Then,thenumberof$5400carssoldis3x,andthenumberof$4800carsis2x.Thevalueof$4200carssoldis$4200•x.Thevalueof$4800carssoldis$4800•2x.Thevalueof$5400carssoldis$5400•3x.Thesumofthesevaluesequalsthetotalsales.

($4200•x)+($4800•2x)+(5400•3x)=$360,000$4200x+$9600x+$16,200x=$360,000$30,000x=$360,000x=$360,000÷$30,000x=12Sincex=12ofthe$4200cars,3x,or36,ofthe$5400modelweresold.

31.Thecorrectansweris(D).Thereisnotenoughinformationtoanswerthisproblem.Wemustknowhowmanyattendedthegametodeterminehowmanypaidfullprice.

32.Thecorrectansweris(A).Youmustborrowoneday’sworthofhoursandonehour’sworthofminutesandrewritetheproblemas:

6days26hr.80min.

–4days9hr.31min.

2days17hr.49min.

33.Thecorrectansweris(B).TheformulafortheareaofatriangleisA= bh.Pluginthe

numbers:

A=

A=84sq.ft.

34.Thecorrectansweris(C).Thisisatrickyquestion.Itdoesn’taskfor150%of72,butrathertoincrease72by150%.Since150%of72=108,weadd72and108forthecorrectanswer,180.

35.Thecorrectansweris(A).Choice(B)isread,“Fourtimesthesquareofacertainnumber,minus6,equals2.”Choice(C)isread,“Fourtimesthesquareofanumber,dividedby3,minus6,equals2.”Choice(D)isread,“Onefourthagivennumber,dividedby3,minus6,equals2.”

36.Thecorrectansweris(D).Writedownbothequationsandaddthemtogether.

14x–2y=32+x+2y=1315x=45x=3

37.Thecorrectansweris(B).Anordinarydiehassixsides,eachhavingadifferentnumber

ofdots.Thechanceofanyfacecomingupisthesame: .

38.Thecorrectansweris(A).First,pickthetwolongesttimes,thencomparethem.1440minutesand25hoursareobviouslythelongestperiods.25hourscontains1500minutes.

39.Thecorrectansweris(D).Thecarsaretravelingtowardeachother,sothedistancebetweenthemisbeingreducedat60+50=110milesperhour.Atarateof110mph,550mileswillbecoveredin5hours.Ifbothcarsleftat1p.m.,theyshouldmeetat6p.m.

40.Thecorrectansweris(B).Choice(A)is4930;choice(C)is483;choice(D)is133.

41.Thecorrectansweris(C).If10menearn$5400in12days,eachmanearns$540in12days,or$45perday.Therefore,6menworkingfor15daysat$45perdaywillearn$4050.

42.Thecorrectansweris(D).Themapdistanceis ,or or .Each =5

miles,andweknowtherearetwelve unitsin .Therefore,thetwelve units

correspondto60milesinactuality.

43.Thecorrectansweris(D).TheareaofacircleisfoundbyA=πr2.Theradiusishalfthediameter.Tofindthediameterwhentheareaisknown,dividetheareabyπtofindthesquareoftheradius.

78.5÷3.14=25

Sincethesquareoftheradiusis25,weknowtheradiusis5,andthediameteristwicetheradius,or10inches.

44.Thecorrectansweris(B).Ifa=6,ab+4=52becomes6b+4=52.If6b+4=526b=52–46b=48b=8

45.Thecorrectansweris(C).If ofthejarisfilledin1minute,then ofthejarisfilled

in minute.Sincethejaris full, remainstobefilled.Thejarwillbefullinanother

minute.

46.Thecorrectansweris(D).Firstconverttoa24-hourclock.3:30p.m.=15:30o’clock

15:30=14:90

–8:50=–8:50

6:40=6hours40minutes

Tosubtractalargernumberofminutesfromasmallernumberofminutes,borrow60minutesfromthehourtoenlargethesmallernumber.

47.Thecorrectansweris(D).

Assignarbitraryvaluestosolvethisproblem:

Asquare10ft.×10ft.=100sq.ft.

Arectangle9ft.×11ft.=99sq.ft.

100–99=1; =1%

Section5:Essay

Exampleofawell-writtenessay.

IfIcouldspendanafternoonwithanyauthor,IwouldhaveawonderfulconversationwithJulesVerne.IthinkofJulesVerneasthefatherofsciencefiction.Wewouldtalkabouthisbooksandwhytheymakesuchgoodreading.Iwouldtellhimhowmuchofhisfictionhasbecomefact.Thenwewouldprobablytalkaboutrecentsciencefictionandaboutthelatestscientificandtechnologicaladvances.Perhapswewouldpredictfuturedevelopments.

ThefirstbookIwouldmentionismyfavorite,TwentyThousandLeaguesUndertheSea.IwouldaskMr.VernehowhethoughtupthebookandwouldtellhimhowmuchIadmirehisworksandhowIrespecthisimagination.ThenIwouldtellhimaboutsubmarinesandsubmarinewarfareandwoulddescribeallthedeepseaexplorationsthatIknowabout.Itishardtopredictaconversationinadvance,butAroundtheWorldinEightyDayswouldcertainlybeagoodnexttopic,andwemightwellconsumetheremainderoftheafternoonwithdiscussionofmoderntravelandofallthecountriesandculturesthatcanbevisitedtoday.

NoconversationwithJulesVernecouldconcludewithoutmentionofmodernsciencefictionandofhowpredictiveitmightbe.IwonderwhatJulesVernewouldthinkofStarTrek.FinallyIwouldtellhimaboutspaceexploration,moonlandings,satellites,andalltheexcitingspaceworkthatisunfolding.

TheprospectofaconversationwithJulesVerneisveryappealing.EventhoughIknowitcannothappen,IamthinkingofmoreandmorethingsIwouldliketodiscusswithhim.Whatastimulatingafternoonitwouldbe.

SCOREYOURSELFScoresontheISEEaredeterminedbycomparingeachstudent’sresultsagainstallotherstudentsinhisorhergradelevelwhotookthatparticulartest.Ascaledscoreisthencalculated.Youcanusethefollowingcalculationstodeterminehowwellyoudidonthispracticetest,butkeepinmindthatwhenyoutaketheactualtest,yourscoremightvary.

Test RawScore ÷No.questions ×100= _____ %

Synonyms _____ ÷20 ×100= _____ %

SentenceCompletions _____ ÷20 ×100= _____ %

TotalVerbalAbility _____ ÷40 ×100= _____ %

Multiple-ChoiceQuantitative _____ ÷19 ×100= _____ %

QuantitativeComparisons _____ ÷18 ×100= _____ %

TotalQuantitativeAbility _____ ÷37 ×100= _____ %

ReadingComprehension _____ ÷36 ×100= _____ %

MathematicsAchievement _____ ÷47 ×100= _____ %

Highpercentagescoresshouldmakeyoufeelverygoodaboutyourself,butlowpercentagesdonotmeanthatyouareafailure.

Remember:•Scoresarenotreportedaspercentages.Alowpercentagemaytranslatetoarespectablescaledscore.

•Thesametestisgiventostudentsingrades8through12.Unlessyouhavefinishedhighschool,youhavenotbeentaughteverythingonthetest.Youarenotexpectedtoknowwhatyouhavenotbeentaught.

•Youwillbecomparedonlytostudentsinyourowngrade.

Useyourscorestoplanfurtherstudyifyouhavetime.

PracticeTest5:ISEE(UpperLevel)

SECTION1:VERBALREASONING40Questions•20Minutes

Directions:EachquestionismadeupofawordinCAPITALlettersfollowedbyfourchoices.ChoosetheonewordthatismostnearlythesameinmeaningasthewordinCAPITALletters.

1.IMPLIED

(A)acknowledged

(B)stated

(C)predicted

(D)hinted

2.FISCAL

(A)critical

(B)basic

(C)personal

(D)financial

3.STRINGENT

(A)demanding

(B)loud

(C)flexible

(D)clear

4.PERMEABLE

(A)penetrable

(B)durable

(C)unending

(D)allowable

5.SCRUPULOUS

(A)conscientious

(B)unprincipled

(C)intricate

(D)neurotic

6.STALEMATE

(A)pillar

(B)deadlock

(C)maneuver

(D)workslowdown

7.REDUNDANT

(A)concise

(B)reappearing

(C)superfluous

(D)lyingdown

8.SUPPLANT

(A)prune

(B)conquer

(C)uproot

(D)replace

9.COMMENSURATE

(A)identical

(B)ofthesameage

(C)proportionate

(D)measurable

10.ZENITH

(A)depths

(B)astronomicalsystem

(C)peak

(D)solarsystem

11.SUCCOR

(A)assistance

(B)nurse

(C)vitality

(D)distress

12.DISPATCH

(A)omitmentionof

(B)sendoutonanerrand

(C)tear

(D)dowithout

13.PORTABLE

(A)drinkable

(B)convenient

(C)havingwheels

(D)abletobecarried

14.VERBOSE

(A)vague

(B)brief

(C)wordy

(D)verbal

15.SUBVERSIVE

(A)secret

(B)foreign

(C)evasive

(D)destructive

16.MALLEABLE

(A)changeable

(B)equalizing

(C)decisive

(D)progressing

17.PETTY

(A)lengthy

(B)communal

(C)small

(D)miscellaneous

18.INTREPID

(A)willing

(B)fanciful

(C)cowardly

(D)fearless

19.NEGOTIATE

(A)argue

(B)think

(C)speak

(D)bargain

20.STERILE

(A)antique

(B)germ-free

(C)unclean

(D)perishable

Directions:Eachofthefollowingquestionsismadeupofasentencecontainingoneortwoblanks.Thesentenceswithoneblankindicatethatonewordismissing.Sentenceswithtwoblankshavetwomissingwords.Eachsentenceisfollowedbyfourchoices.Choosetheonewordorpairofwordsthatwillbestcompletethemeaningofthesentenceasawhole.

21.Undauntedbyhismanysetbacks,Joshua_____.

(A)crumpled

(B)drewback

(C)canceled

(D)persevered

22.Nationwide,collegeartsandsciencedepartmentsaretaking_____measurestoattractstudents.

(A)no

(B)puny

(C)innovative

(D)few

23.Withlesscapitalavailableandfewerdealsbeingdone,ithasclearlybecomea(n)_____market.

(A)heinous

(B)inflationary

(C)sellers’

(D)buyers’

24.Thepenaltyforviolatingthelawwould_____formultipleoffenses.

(A)accede

(B)nullify

(C)diminish

(D)escalate

25.Thehotelwasaworld-class_____propertyand,thankstorecentrefurbishingandclevermarketingefforts,itisexperiencinga_____.

(A)luxury...renaissance

(B)communal...withdrawal

(C)opulent...decline

(D)decadent...stalemate

26.Somecolleges,ratherthan_____studentstotakeartscourses,simplyforcethem.

(A)requiring

(B)enticing

(C)demanding

(D)allowing

27.Requiring_____bythecriminaltothevictimwouldbeafarbetterwayofdealingwithmanylawbreakersthan_____,sheargued.

(A)punishment...freedom

(B)imprisonment...pardon

(C)restitution...imprisonment

(D)meaculpa...negligence

28.Knowledgegainedfrombookswithoutthebenefitofpracticalexperienceisusuallynotsoprofitableineverydayworkastheopposite,_____without_____.

(A)culture...manners

(B)experiments...science

(C)experience...scholarship

(D)learning...knowing

29.To_____someofits_____overthehugeincreaseinstateinsurancepremiumsforemployees,theschooldistrictinvitedoneinsuranceexperttospeakatarecentboardmeeting.

(A)quell...anxiety

(B)dispel...myths

(C)aggravate...nervousness

(D)foment...trepidation

30.Whilemanyelderlyindeedare_____,povertyis_____amongthemillionsofolderAmericanswhorelysolelyonSocialSecurity.

(A)penurious...rampant

(B)invalid...abolished

(C)absolute...widespread

(D)comfortable...pervasive

31.Apoliceofficer’s_____jobistopreventcrime.

(A)primary

(B)only

(C)ostentatious

(D)ostensible

32.Thecandidate’sinabilitytoconnectwithmiddle-classvoterswashisgreatest_____.

(A)virtue

(B)extinction

(C)shortcoming

(D)performance

33.Despitereligiousdifferences,thefamily_____clashesbyrespectingeachother’svalues.

(A)denied

(B)averted

(C)condescended

(D)declined

34.Whilemarketingtohealth-consciousconsumerswill_____arestaurantchange,itwillalsohaveaneffectinsupermarkets.

(A)denigrate

(B)cancel

(C)encourage

(D)emit

35.Despitethepolitician’soverwhelmingloss,he_____hispopularitywithasmallcoreoffollowers.

(A)revoked

(B)maintained

(C)restrained

(D)encouraged

36.Thedecisiontoseektherapeutictreatmentisoftenprovokedbya(n)_____,suchasanarrestoradomesticdispute.

(A)dearth

(B)crisis

(C)enigma

(D)casualty

37.Knowingthatanyparticularnewbusinesscan_____,Joshuaavoidedinvestinginoneevenifthepotential_____washigh.

(A)succeed...downside

(B)reduce...profit

(C)dowell...monies

(D)fail...payoff

38.Becauseofthe_____causedbytheflood,livingconditionsintheareahave_____;manypeoplehavelostalloftheirbelongings.

(A)trepidation...augmented

(B)morass...careened

(C)devastation...deteriorated

(D)vertigo...ameliorated

39.Themanagementisprovidingallneededbuildingfacilitiestohelpthescientists_____theirresearchproject.

(A)magnify

(B)retard

(C)relinquish

(D)implement

40.Wecaneasilyforgoa_____wehaveneverhad,butonceobtaineditoftenislookeduponasbeing_____.

(A)requirement...unusual

(B)gift...useless

(C)luxury...essential

(D)bonus...unearned

SECTION2:QUANTITATIVEREASONING37Questions•35Minutes

Note:YoumayassumethatallfiguresaccompanyingQuantitativeReasoningquestionshavebeendrawnasaccuratelyaspossibleEXCEPTwhenitisspecificallystatedthataparticularfigureisnotdrawntoscale.Letterssuchasx,y,andnstandforrealnumbers.TheQuantitativeReasoningTestincludestwotypesofquestions.Thereareseparatedirectionsforeachtypeofquestion.

Directions:Forquestions1-19workeachprobleminyourheadoronscratchpaper.Choosetheletterofyouranswerchoice.

1.Ifthedecimalpointinanumberismovedoneplacetotheright,thenumberhasbeen

(A)dividedby10.

(B)multipliedby10.

(C)dividedby100.

(D)multipliedby100.

2.WhatisthetotalnumberofdegreesfoundinanglesAandCinthetrianglebelow?

(A)180°

(B)100°

(C)90°

(D)75°

3.Ifr+s=3s,thenwhichexpressionisequaltor?

(A)s

(B)2s

(C)4s

(D)3s2

4.Usingexponents,write329inexpandedform.

(A)(32×10)+(2×10)+9

(B)(3×102)+(2×10)+9

(C)(3×102)+(2×102)+9

(D)(3×103)+(2×10)+9

5.Findthecircumferenceofacirclewhoseradiusis21feet.

(A)65.94feet

(B)132feet

(C)153feet

(D)1769.4feet

6.Ifx>–4,andy<2,thenx∩yincludes

(A)–4,0,1,2

(B)–2,–1,1,2

(C)1,2,3,4

(D)–3,–2,–1,0,1

7.(6×2)+(7×3)=?

(A)(6×7)+(2×3)

(B)(7–6)+(3–2)

(C)(7×3)+(6×2)

(D)(7×3)×(6×2)

8.Whichofthefollowingwillsubstituteforxandmakethestatementbelowtrue?

56–(7–x)=53

(A)4

(B)3

(C)2

(D)1

9.Ananglethatisgreaterthan90°andlessthan180°isa(n)

(A)acuteangle.

(B)rightangle.

(C)reflexangle.

(D)obtuseangle.

10.Whatisthevalueoftheexpression

(A)21

(B)35

(C)44

(D)56

11. isgreaterthan

(A)

(B)

(C)

(D)

12.1centimeterequalswhatpartofameter?

(A)

(B)

(C)

(D)

13.Whatisthelowestcommondenominatorforthefractions ?

(A)24

(B)32

(C)36

(D)48

14.Thesetofcommonfactorsof36and64is

(A){1,2,4}

(B){1,2,3,4}

(C){1,2,4,6,18}

(D){1,2,3,4,6}

15.Ifoneangleofatrianglemeasures115°,whatisthesumoftheothertwoangles?

(A)245°

(B)195°

(C)75°

(D)65°

16.

=

(A)

(B)

(C)

(D)

17.Ifaplayingcardisdrawnfromastandarddeck,whatarethechancesitwillbea6?

(A)

(B)

(C)

(D)

18.Thescaleonamapis miles.If2townsare28milesapart,howmanyinches

willseparatethemonamap?

(A)

(B)

(C)

(D)

19.AcertainhighwayintersectionhashadAaccidentsovera10-yearperiod,resultinginBdeaths.Whatistheyearlyaveragedeathratefortheintersection?

(A)A+B–10

(B)

(C)

(D)

Directions:Comparethequantitiesinthetwocolumns,andchooseyour

answerasfollows:

(A)ifthequantityinColumnAisgreater

(B)ifthequantityinColumnBisgreater

(C)ifthequantitiesareequal

(D)iftherelationshipcannotbedeterminedfromtheinformationgiven

ColumnA ColumnB

20. x=–2

3x2+2x–1 x3+2x2+1

21.

AC BC

ColumnA ColumnB

22. (16÷4)+(8×2)–8 (3×4)+(10÷5)–3

23. Aradiopricedat$47.25includesa5%profit(basedoncost).

$44.89 Theoriginalcostbeforeprofit

24.a–b=–1–b–a=–3

b a

25. 25%ofthe300girlsintheschoolhaveblondehair.

Theratioofgirlswithblondehairtothosewithoutblondehair

26.

e+c=90°

b+a+d+f 270

ColumnA ColumnB

27.

ABCDisaparallelograminscribedincircleO.

a°+c° b°+d°

28. Differencebetween 0.5

29.Acandoajobalonein4days.Bcandoajobalonein3days.

ThenumberofdaysittakesAandBworkingtogethertodothejob 2days

30.Inacertaincollege,theratioofthenumberoffreshmentothenumberofseniorsis3:1.

Theratiobetweenthenumberofseniorsandthetotalenrollment

ColumnA ColumnB

31.

m n

32.a+b=xa–b=y

x y

33.

5 AB

34.

35.x>0.y<0

x–y x+y

36.Duringastoresale,a$43.50radiocan

bepurchasedata15%discount.

Thesellingpriceoftheradiowiththediscount $36

Duringastoresale,a$43.50radiocanbepurchasedata15%discount.

37.

SECTION3:READINGCOMPREHENSION36Questions•35Minutes

Directions:Eachreadingpassageisfollowedbyquestionsbasedonitscontent.Answerthequestionsonthebasisofwhatisstatedorimpliedinthepassage.

Comingintotherelaystationwitharush,thePonyExpressriderswungdownfromhisexhaustedmountandupontoafreshhorsewithhispreciousmochilla,thesaddlebagcontainingthemail.Hewasoffagainwithoutamoment’sdelay.Hewasexpectedtoreachthenextstation,andhedid,orhediedtrying.

Aridermightcomeintoastationatdawnonlytofindthatthestationhadbeenburned,thekeeperskilled,andthehorsesrunoffbyattackingIndians.Inthatcasehewouldcontinuetothenextstationwithoutfoodorrest.

“BuffaloBill,”aboyof18,madethelongestcontinuousruninthehistoryofthePonyExpress,384miles.Byriding280milesinjust22hours,JimMooreearnedthedistinctionofhavingmadethefastestrun.

Ninetyriderscoveredthetrailatalltimesofthedayandnight,oftenriskingtheirlivestogetthemailthroughwithintheten-daylimit.Mostmadeitineightdays.

Ontheaverage,theriderscouldtravel11milesanhour,aquickpaceoverterrainthatmightrequirethehorsetoswimriversorcat-footitswayalongnarrowclifftrails.

ThepaceofthemaildeliverybyPonyExpresswassnail-likebytoday’sstandards,butatthetimeofitscommencementin1860,itofferedthefastestcross-countrycommunicationthathadeverbeenachieved.AbrahamLincolnwaselectedpresidentduringthatyear,andthankstothePonyExpress,residentsofCaliforniareceivednewsofLincoln’svictoryinjustoveraweek.

ThePonyExpressriderscarriedthemailbetweenMissouriandCaliforniaforlessthantwoyears.Theystoppedridingin1861whenatelegraphlineofferedaswiftermeansofcommunication.TheeffortsoftheridersarememorializedatthePonyExpressNationalMuseum,establishedinSt.Joseph,Missouri.Accordingtotherecordsofthemuseum,thePonyExpresslostonlyasingledeliveryofmailduringtheentireperiodofitsoperation.

1.ThePonyExpressriderstoppedatastationto

(A)getafewhoursofsleep.

(B)getafreshmount.

(C)sortthemail.

(D)escapeNativeAmericanattacks.

2.Themochillareferstothe

(A)PonyExpressrider’ssaddlebags.

(B)PonyExpresshorses.

(C)stations.

(D)trails.

3.ThispassageimpliesthatmostofthePonyExpressriderswere

(A)sure-footed.

(B)faithfultotheirjobs.

(C)mountain-bred.

(D)killed.

4.ThosesendingmailbyPonyExpresscouldexpectthatitwouldreachitsdestinationwithin

(A)tendays.

(B)fivedays.

(C)amonth.

(D)beforedawn.

5.Thelongestcontinuousrunwas

(A)completedwithin22hours.

(B)280miles.

(C)madebytraveling11milesperhour.

(D)384miles.

6.Whichofthefollowingstatementsismostlikelytrue,basedonthepassage?

(A)ThePonyExpresswasreliableatdeliveringthemail.

(B)ThePonyExpresswasoperationalthroughoutthe1860s.

(C)MailwasdeliveredbyPonyExpressfromeastcoasttowestcoast.

(D)ThePonyExpresswaslessexpensivethanotherformsofmaildelivery.

Forgenerations,historiansandboatlovershavebeentryingtolearnmoreaboutthebraveshipthatbroughtthePilgrimstoAmerica.ThetaskisadifficultonebecauseMayflowerwassuchacommonnameforshipsbackinearlyseventeenth-centuryEnglandthattherewereatleasttwentyoftheminexistencewhenthePilgrimsleftfortheNewWorld.

AnexactduplicateoftheMayflowerhasbeenbuiltinEnglandandgiventothepeopleoftheUnitedStatesasasymbolofgoodwillandcommonancestrylinkingBritonsandAmericans.ThePilgrims’Mayflowerapparentlywasbuiltoriginallyasafishingvessel.Itseemstohavebeen90feetlongby22feetwide,displacing180tonsofwater.Theduplicatemeasures90feetby26feet,displaces183tons,andhasacrewof21,asdidtheoriginalvessel.ThenewMayflowerhasnomotorbuttravelsfasterthantheoldboat.

Whathappenedtothehistoricboat?Sofarascanbetold,theMayflowerwentbacktolesscolorfuljobsand,nottoomanyyearslater,wasscrapped.Whathappenedtothebeams,masts,andplankingisquestionable.IntheEnglishcityofAbingdon,thereisaCongregationalchurchthatcontainstwoheavywoodenpillars.SomesaythesepillarsaremastsfromtheMayflower.AbarnintheEnglishtownofJordansseemedtobebuiltofoldshiptimbers.Marineexpertssaidthesetimberswereimpregnatedwithsaltand,ifputtogether,wouldformavessel90feetby22feet.ThemanwhoownedthefarmwhenthepeculiarbarnwasbuiltwasarelativeofthemanwhoappraisedtheMayflowerwhenitwasscrapped.

SotheoriginalMayflowermaystillbedoingserviceashorewhileherduplicatesailstheseasagain.

7.AlongsearchwasmadeforthePilgrims’boatbecauseit

(A)containedvaluablematerials.

(B)mightstilldoseaservice.

(C)hashistoricalimportance.

(D)wouldlinkGreatBritainandAmerica.

8.IthasbeendifficulttodiscoverwhathappenedtotheoriginalMayflowerbecause

(A)manyshipsborethesamename.

(B)itwassuchasmallvessel.

(C)thesearchwasbeguntoolate.

(D)ithasbecomeimpregnatedwithsalt.

9.TheBritishrecentlyhadaduplicateoftheMayflowerbuiltbecause

(A)theoriginalcouldnotbelocated.

(B)theywantedtomakeagestureoffriendship.

(C)partsoftheoriginalcouldbeused.

(D)historiansrecommendedsuchastep.

10.ComparedwiththeoriginalMayflower,themodernduplicate

(A)islonger.

(B)isidentical.

(C)carriesalargercrew.

(D)issomewhatwider.

11.Thewordcolorfulmostnearlymeans

(A)humorous.

(B)decorative.

(C)bright.

(D)interesting.

12.Whentheauthorsaysthattheoriginalboatmaystillbedoingserviceashore,hemeansthat

(A)itmaybewholeandintactsomewhere.

(B)present-daybuildingsmayincludepartsofit.

(C)itmaybeinaboatlover’sprivatecollection.

(D)itmaybeintheserviceofpirates.

Fewanimalsareasdescriptivelynamedasthevaryinghare(Lepusamericanus),alsocommonlyknownasthesnowshoehare,whiterabbit,orsnowshoerabbit.Thevaryingharelivesintheforest,findingprotectionfrompredatorsinthethicketsofdenseunderbrush.Thespeciesderivesitsvariousnamesfromitsinterestingadaptationstotheseasonalchangesaffectingitshabitat.

Muchlikeachameleonchangesbodycolor,thehareisabletoadaptbychangingthecolorofitscoattoallowittobetterblendinwithitssurroundings.Thecolorchangesareeffectedbymeansofamolt,andaretimed(althoughthehareshavenovoluntarycontroloverthem)tocoincidewiththechangingappearancesofthebackground.

Theperiodsoftransition—fromwhitetobrowninthespring,andfrombrowntowhiteinthefall—requiremorethantwomonthsfromstarttocompletion,duringwhichtimetheharesareamottledbrownandwhite.Bywintertimeeachyear,theharesucceedsinturningcompletelywhiteoveritsentirebody,withtheexceptionofgreyatthetipsofitsearsandonitsfeet.Thewhitecoloringallowsittoeffectivelycamouflageitselfinthewintersnow.Inadditiontothechangesincolor,inthefallthesolesofthefeetdevelopaveryheavygrowthofhairthatfunctionsassnowshoes.

Thehares’dietconsistsprimarilyofbark,twigs,andstemsfromvarioustrees.Theymayalsoconsumecertainseasonalleavesandgrasses,dependingupontheiravailabilityinthesurroundinghabitat.

VaryingharesresidethroughoutNorthAmericaandhaveanextensiverange,fromtheeasternshoresofNewfoundland,Canada,toAlaskainthenorthwest.InNewYorkState,haresaremostabundantinandaroundtheAdirondackandCatskillMountains.Thrivingpopulations,withlessextensiveranges,arefoundinAllegany,Cattaraugus,Rensselaer,andChenangocounties.Smallercoloniesoflimitedrangearefoundinscatteredislands.

13.Whichtitlebestexpressesthemainideaofthisselection?

(A)“SeasonalChangesinBirds”

(B)“TheVaryingHare”

(C)“AnAmericanAnimal”

(D)“TheAbundanceofHares”

14.Termsusedtonametheserabbitsarerelatedtotheir

(A)abundanceinmanypartsofNewYorkState.

(B)sensitivitytoweatherconditionsthroughoutthestate.

(C)abilitytoadapttothechangeofseasons.

(D)thickwhitecoats.

15.Thewordmottledmostnearlymeans

(A)frightening.

(B)dedicated.

(C)speckled.

(D)relaxed.

16.Theserabbitshavebothbrownandwhitemarkingsin

(A)summerandwinter.

(B)springandfall.

(C)springandsummer.

(D)fallandwinter.

17.ThepartsofNewYorkStatewhererabbitpopulationsaremostplentifulare

(A)Allegany,Cattaraugus,Rensselaer,andChenangocounties.

(B)AdirondackandCatskillMountainregions.

(C)islandswithinthestate.

(D)snowyareasinthehills.

18.Whichstatementabouttheserabbitsistrueaccordingtotheselection?

(A)Theyarebecomingfewerinnumber.

(B)Theyarecapableofleapinggreatdistances.

(C)Theyaremoreplentifulinwinter.

(D)Theyhavenocontrolovertheircolorchanges.

LiketheUnitedStatestoday,Athenshadcourtswhereawrongmightberighted.Sinceanycitizenmightaccuseanotherofacrime,theAtheniancourtsoflawwereverybusy.Infact,unlessacitizenwasunusuallypeacefulorveryunimportant,hewouldbesuretofindhimselfinthecourtsatleastonceeveryfewyears.

Atatrial,boththeaccuserandthepersonaccusedwereallowedacertaintimetospeak.Thelengthoftimewasmarkedbyawaterclock.Freementestifiedunderoathastheydotoday,buttheoathofaslavewascountedasworthless.

Tojudgeatrial,ajurywaschosenfromthemembersoftheassemblywhohadreached30yearsofage.TheAthenianjurieswereverylarge,oftenconsistingof201;401;501;1,001;ormoremen,dependingupontheimportanceofthecasebeingtried.Byinvolvingalargenumberofjurors,theAtheniansystemhelpedeliminatethepossibilityofjurorsbeingbribedtovoteacertainwayinthetrial.Becauseitwouldbecostlytopaybribestohundredsofmen,thelargejurysystemhelpedthetrialtobedecidedinajustmanner.

Thejurymansworebythegodstolistencarefullytobothsidesofthequestionandtogivehishonestopinionofthecase.Eachjurymangavehisdecisionbydepositingawhiteorblackstoneinabox.UnlikeintheU.S.today,decisionsweremadewithoutdeliberationbythejury;jurymemberssimplycasttheirvotesindividually,basedontheirassessmentsofthecasespresentedbytheprosecutionandthedefense.Tokeepcitizensfrombeingtoocarelessinaccusingeachother,therewas

arulethatifthepersonaccuseddidnotreceiveacertainnumberofnegativevotes,theaccuserwascondemnedinstead.

19.Whichtitlebestexpressesthemainideaofthisselection?

(A)“AthensandtheUnitedStates”

(B)“JusticeinAncientAthens”

(C)“TestifyingUnderOath”

(D)“TheDutiesofJuries”

20.PeopleinAthenswerefrequentlyontrialinalawcourtbecause

(A)theylikedtoserveonjuries.

(B)ajurymanagreedtolistentobothsides.

(C)anypersonmightaccuseanotherofacrime.

(D)theslavesweretroublesome.

21.AnAthenianwaslikelytoavoidaccusinganotherwithoutagoodreasonbecause

(A)thejurymightcondemntheaccuserinsteadoftheaccused.

(B)thejurymightbeverylarge.

(C)caseswerejudgedbymenover30yearsold.

(D)therewasalimitonthetimeatrialcouldtake.

22.Whichstatementistrueaccordingtotheselection?

(A)Anaccusedpersonwasdeniedtheprivilegeoftellinghissideofthecase.

(B)Theimportanceofthecasedeterminedthenumberofjurors.

(C)Ajury’sdecisionwashandeddowninwriting.

(D)Acitizenhadtoappearincourteveryfewyears.

23.CasesweredecidedinancientAthensby

(A)individualvote.

(B)jurydeliberation.

(C)ajudge’sruling.

(D)briberyofjurors.

24.Inthecontextofthepassage,thephrasenegativevotesmostnearlymeansvotes

(A)forstrictpunishment

(B)againsttheaccused

(C)foralenientsentence

(D)againsttheaccuser

Thetemperatureoftheearth’supperatmosphereisoneofthemostrevealingpropertiesoftheearth’snearenvironment.Notonlydoesitvarywidelywithtimeandlocationbutitalsoreactsstronglytochangesinsolaractivity.Thevariationoftemperaturewithaltitudeandwithtimereflectsdirectlythedifferentenergysourcesthatinlargemeasuregovernthedynamicbehavioroftheupperatmosphere.Thetemperaturealsocontrolstherateofchangeofdensitywithaltitudethroughtherequirementofhydrostaticbalance.Inhydrostaticbalancetheatmosphericpressureatanyheightequalsthetotalweightoftheoverlyinggas,aconditionthatrequiresthatthepressureanddensityofthegasdecreaseexponentiallyatarateinverselyproportionaltothetemperature.

Thus,ifthealtitudeprofileofthetemperatureisknown,onecancalculatethealtitudeprofilesofpressureanddensityprovidedthemeanmolecularweightofthegasisalsoknown.Thisprovisoisnecessarybecausetherateofdecreaseofpressureanddensityisproportionaltothemeanmolecularweight.Sinceheavygases,suchasargonandcarbondioxide,aremoretightlyboundbytheearth’sgravitationalfield,theytendtoconcentrateatlowaltitudes,whilethedensityoflightconstituents,suchashydrogenandhelium,decreasesveryslowlywithheight.Ataltitudesbelowabout110kilometers,however,thistendencytowardgravitationalseparationoftheconstituentsisfullycounteractedbyturbulentmixingprocessessothemeanmolecularweightoftheatmospherevariesverylittlefromitssea-levelvalueof29atomicmassunits(amu).Athigheraltitudesthereislittlemixing.Theheavyconstituentsbecomeprogressivelymorerare,andthedominantatmosphericconstituentchanges,withincreasingaltitude,frommolecularnitrogen(28amu),toatomicoxygen(16amu),tohelium(4amu),and,atveryhighaltitudes,toatomichydrogen(1amu).

25.Allofthefollowingaffectthetemperatureoftheearth’supperatmosphereEXCEPT

(A)sun.

(B)altitude.

(C)time.

(D)weather.

26.Whichofthefollowingmoststronglyaffectstheupperatmosphere?

(A)Lackofoxygen

(B)Seasonalchanges

(C)Airpressure

(D)Heatofthesun

27.Whichofthefollowingaremostlikelytobetightlyboundbytheearth’sgravitationalfield?

(A)oxygenandhelium.

(B)hydrogenandhelium.

(C)argonandcarbondioxide.

(D)heliumandargon.

28.Themeanmolecularweightoftheatmosphereatsealevelisclosestto

(A)1atomicmassunit.

(B)4atomicmassunits.

(C)29atomicmassunits.

(D)36atomicmassunits.

29.Atthehighestaltitudeonewouldfind

(A)helium.

(B)oxygen.

(C)hydrogen.

(D)nitrogen.

30.Whichstatementbestreflectsthemainideaofthisarticle?

(A)Thereisagreatvariabilityinthetemperatureoftheearthfromplacetoplaceandfromtimetotime.

(B)Controloftheearth’supperatmospheredependsuponcontrolofitstemperature.

(C)Ourenvironmentiscontrolledbytheearth’stemperature.

(D)Highaltitudetemperaturetellsmuchabouttheearth’satmosphere.

OnMay8,1939,folksongcollectorandscholarHerbertHalpertarrivedinMississippitodocumentfolkloreandfolkmusicduringarecordingtouroftheSouthsponsoredbytheJointCommitteeontheArtsoftheWorksProgressAdministration(WPA).TheWPAwasoneofmanyNewDealagenciescreatedbyPresidentFranklinD.Roosevelttohelpboostthecountry’seconomyafterthesetbacksoftheGreatDepression.

Toconductthetour,HalpertdroveintoMississippiinanoldambulanceoutfittedwithcabinets,asmallcot,food,andclothes.Theambulancealsohadspeciallybuiltshelvesforthelatestinrecordingequipment—anacetatediscrecorderlentbytheArchiveofAmericanFolkSongattheLibraryofCongress.

TotakefulladvantageofHalpert’sshortvisit,localWPAworkersactedasintermediaries,precedingtherecordingtrucktomakearrangementswiththefolkmusicianshewouldvisitandgroupingartistsinconvenientplacestominimizetravelandmaximizerecordingtime.Followingtheirschedule,withafewsidetripstopursueacoupleofleadsofhisown,Halpertcut168recordsbetweenMay8andJune11,1939.

HalpertwasassistedbyAbbottFerriss,aMississippinative,wholaterwentontobecomeasociologistatColumbiaUniversity.FerrisswasemployedatthetimebyanotherNewDealprogramknownastheFederalWriters’Project.HeusedtheexperiencetogathersociologicalinformationaboutSouthernfolkmusiccommunitiesthatwouldeventuallyinfluencesomeofhisacademiccontributionstothefield.

Inadditiontohelpingwiththeactualrecording,Ferrisskeptfieldnotesonthetripandtookphotographsofthemusicians,theirfamilies,homes,andsurroundings.Attheproject’sconclusion,therecordingsbecamepartofthefolk-musiccollectionsattheLibraryofCongress.ThephotographsandmuchofthemanuscriptmaterialrelatedtotheprojectwereretainedinMississippi.

31.Accordingtothepassage,thepurposeofHalpert’sjourneytoMississippiwasto

(A)makearrangementsforthewritingoffolksongs.

(B)consultwithalocalnative.

(C)recordthefolkmusicofMississippiperformers.

(D)photographtheMississippilandscape.

32.Whichofthefollowingisnotstatedinthepassage?

(A)TheWPAsponsoredarecordingprojectintheSouth.

(B)LocalworkershelpedHalpertbysearchingformusicians.

(C)ThelocalworkerssoughttominimizeHalpert’straveltime.

(D)ThephotographsweresenttotheLibraryofCongress.

33.WhichofthefollowingcanbereasonablyinferredabouttheWPA?

(A)Itwasonlyinterestedinfolkmusic.

(B)IttookfulladvantageofshortvisitstotheLibraryofCongress.

(C)Itwasanationalorganizationwithlocaloffices.

(D)ItwassponsoredbytheJointCommitteeontheArts.

34.Accordingtothepassage,AbbottFerrisswasoriginallyfrom

(A)Columbia.

(B)Mississippi.

(C)NewYork.

(D)London.

35.DuringthetimeoftheMississippirecordingsin1939,AbbottFerrisswasemployedby

(A)theFederalWriters’Project.

(B)theLibraryofCongress.

(C)ColumbiaUniversity.

(D)folkmusiccommunities.

36.Thewordretainedmostnearlymeans

(A)lost

(B)repaired

(C)grown

(D)kept

SECTION4:MATHEMATICSACHIEVEMENT47Questions•40Minutes

Directions:Eachquestionisfollowedbyfouranswerchoices.Choosethecorrectanswertoeachquestion.

1.Arecipefor6quartsofpunchcallsfor cupofsugar.Howmuchsugarisneededfor9

quartsofpunch?

(A) cups

(B) cups

(C)1 cups

(D) cups

2.Howmanyyardsofribbonwillittaketomake45badgesifeachbadgeuses4inchesofribbon?

(A)5

(B)9

(C)11

(D)15

3.Asanemployeeataclothingstore,youareentitledtoa10%discountonallpurchases.Whenthestorehasasale,employeesarealsoentitledtoanadditional20%discountofferedtoallcustomers.Whatwouldyouhavetopayfora$60jacketboughtonasaleday?

(A)$6

(B)$10.80

(C)$36

(D)$43.20

4.Asectionofpavementthatis10feetlongand8feetwidecontainshowmanysquarefeet?

(A)18sq.ft.

(B)80sq.ft.

(C)92sq.ft.

(D)800sq.ft.

5.Thedecimalnumber0.375isequivalenttowhichofthefollowing?

(A)

(B)

(C)

(D)

6.In1972,approximately19,000fatalaccidentsweresustainedinindustry.Therewereapproximately130nonfatalinjuriestoeachfatalinjury.Thenumberofnonfatalaccidentsduring1972wasapproximately

(A)146,000

(B)190,000

(C)1,150,000

(D)2,500,000

7.Whatisthevalueofxwhen5x=5×4×2×0?

(A)6

(B)8

(C)1

(D)0

8.Aparticularstorehasa100%mark-upfromwholesaletoretailprices.Adressthatcosts$130retailwillcosthowmuchwholesale?

(A)$260

(B)$100

(C)$90

(D)$65

9.Thescaleusedonablueprintis =1foot.Ifaroomisactually17'×22'howlargewill

itbeonthedrawing?

(A)

(B)

(C)

(D)

10.Arollofcarpetingcontains90squarefeetofcarpet.Howmanyrollswillberequiredtocarpetaroom28'×20'?

(A)

(B)6

(C)

(D)

11.100–x=52.Whatisthevalueofx?

(A)75

(B)50

(C)25

(D)5

12.If72x=6y,andy=2,x=

(A)

(B)

(C)6

(D)12

13.

(A)54hr.44min.

(B)54hr.84min.

(C)55hr.44min.

(D)55hr.84min.

14.Whichofthefollowingrepresentsonehalfofacertainnumbersquared,minus6?

(A)

(B)

(C)

(D)

15.Amixturecontains20gallonsofwaterand5gallonsofnitricacid.If10moregallonsofwaterareadded,thepartthatiswateris

(A)

(B)

(C)

(D)

16.Whichisthelongesttime?

(A) ofaday

(B) hours

(C)100minutes

(D) ofamonth

17.Whatpercentageofacirclegraphwouldberepresentedbyaportionhavingarightangle?

(A)90%

(B)45%

(C)25%

(D)20%

18.Whichexpressionisequivalenttotheexpression(z+7)(z–7)?

(A)z2–49

(B)z2+49

(C)z2–14z–49

(D)z2+14z–49

19.Findtheperimeterofarectanglewiththedimensions115'×63'.

(A)7245'

(B)356'

(C)187'

(D)178'

20.6.28×1.003=

(A)0.629884

(B)6.29884

(C)62.9442

(D)629.884

21.Solveforx: =10.

(A)±5

(B)±10

(C)±20

(D)±25

22.IfMr.Greenborrowed$2000at12%simpleinterestfortwoyears,whatwouldthetotalinterestchargebe?

(A)$240

(B)$360

(C)$420

(D)$480

23.Thepuptentshownis3feetwideand2feethigh.Finditsvolumeifitis6feetlong.

(A)36sq.ft.

(B)18cu.ft.

(C)24cu.ft.

(D)36cu.ft.

24.Aboard30'longiscutintothreeunequalparts.Thefirstisthreetimesaslongasthesecond.Thethirdistwiceaslongasthefirst.Howlongisthelongestpiece?

(A)6'

(B)9'

(C)12'

(D)18'

25.Awinemerchanthas32gallonsofwineworth$1.50agallon.Ifhewishestoreducethepriceto$1.20agallon,howmanygallonsofwatermustheadd?

(A)10

(B)9

(C)8

(D)7

26.Sixisfourmorethan ofwhatnumber?

(A)1

(B)3

(C)4

(D)6

27.Thewinnerofaracereceived ofthetotalpurse.Thethird-placefinisherreceivedone

thirdofthewinner’sshare.Ifthewinner’ssharewas$2700,whatwasthetotalpurse?

(A)$8100

(B)$2700

(C)$1800

(D)$900

28.Twocarsstarttowardeachotheralongastraightroadbetweentwocitiesthatare450milesapart.Thespeedofthefirstcaris35mph,andthatofthesecondis48mph.Howmuchtimewillelapsebeforetheymeet?

(A)6.01hours

(B)5.42hours

(C)5.25hours

(D)4.98hours

29.Astockclerkhad600padsonhand.Hethenissued ofhissupplyofpadstoDivision

X, toDivisionY,and toDivisionZ.Thenumberofpadsremaininginstockis

(A)48

(B)125

(C)240

(D)475

30.Onemancanloadatruckin25minutes,asecondcanloaditin50minutes,andathirdcanloaditin10minutes.Howlongwouldittakethethreetogethertoloadthetruck?

(A) minutes

(B) minutes

(C) minutes

(D)10minutes

31.If4x–y=20,and2x+y=28,thenx=

(A)24

(B)16

(C)8

(D)6

32.If6+x+y=20,andx+y=k,then20–k=

(A)0

(B)6

(C)14

(D)20

33.

Inthefigureabove,m AOB=60°.IfOisthecenterofthecircle,thenminorarcABiswhatpartofthecircumferenceofthecircle?

(A)

(B)

(C)

(D)

34.IfallPareSandnoSareQ,itnecessarilyfollowsthat

(A)allQareS.

(B)allQareP.

(C)noPareQ.

(D)noSareP.

35.AisolderthanB.Withthepassageoftimethe

(A)ratiooftheagesofAandBremainsunchanged.

(B)ratiooftheagesofAandBincreases.

(C)ratiooftheagesofAandBdecreases.

(D)differenceintheiragesvaries.

36.Fromatemperatureof15°,adropof21°wouldresultinatemperatureof

(A)36°

(B)–6°

(C)–30°

(D)–36°

37.

Inthefigureabove,thesidesof ABCarerespectivelyparalleltothesidesof DEFIfthecomplementofAis40°,whatisthecomplementofD?

(A)20°

(B)40°

(C)50°

(D)60°

38.Alineofprintinamagazinearticlecontainsanaverageof6words.Thereare5linestotheinch.If8inchesareavailableforanarticlethatcontains270words,howmustthearticlebechanged?

(A)Add30words.

(B)Delete30words.

(C)Delete40words.

(D)Add60words.

39.

TheareaoftriangleRis3timestriangleS.TheareaoftriangleSis3timestriangleT.IftheareaoftriangleS=1,whatisthesumoftheareasofthethreetriangles?

(A)

(B)

(C)

(D)6

40.If5pintsofwaterareneededtowatereachsquarefootoflawn,theminimumgallonsofwaterneededforalawn8'by12'is

(A)5

(B)20

(C)40

(D)60

41.Intheformulal=p+prt,whatdoeslequalwhenp=500,r=20%,t=2?

(A)700

(B)8000

(C)10,000

(D)12,000

42.Acarownerfindsheneeds12gallonsofgasforeach120mileshedrives.Ifhehashiscarburetoradjusted,hewillneedonly80%asmuchgas.Howmanymileswill12gallonsofgasthenlasthim?

(A)90

(B)96

(C)150

(D)160

43.Whatisthemaximumnumberofbooks,each -inchthick,thatcanbeplacedstandingon

ashelfthatis4feetlong?

(A)16

(B)48

(C)96

(D)192

44.Inabagtherearered,green,black,andwhitemarbles.Ifthereare6red,8green,4black,and12white,andonemarbleistobeselectedatrandom,whatistheprobabilityitwillbewhite?

(A)

(B)

(C)

(D)

45.

Inthediagramabove, .IfCE=7andED=6,whatistheshortestdistancefromCtoD?

(A)6

(B)7

(C)

(D)

46.Ifa=3,thenaa•a=

(A)9

(B)18

(C)51

(D)81

47.(3+2)(6–2)(7+1)=(4+4)(x).Whatisthevalueofx?

(A)13+2

(B)14+4

(C)4+15

(D)8+12

SECTION5:ESSAY30Minutes

Directions:Writealegible,coherent,andcorrectessayonthefollowingtopic.

Topic:Tellaboutoneextracurricularactivityinwhichyouhopetoparticipateinhighschool.Givereasonswhyyouhavechosenthisactivity.

ANSWERSANDEXPLANATIONSSection1:VerbalReasoning

1.Thecorrectansweris(D).ToIMPLYistoindicateindirectly,tosuggest,ortohint.Byherstrangesmile,theMonaLisaimpliesthatsheknowsasecret.

2.Thecorrectansweris(D).ThatwhichisFISCALhastodowithmoney.Afiscalyearisthetwelve-monthperiodbetweensettlementofaccounts.

3.Thecorrectansweris(A).STRINGENTmeansrigidlycontrolled,strict,orsevere.Prospectivefirefightersmustpassastringentphysicalexamtoprovethattheycanmeetthedemandsofthejob.

4.Thecorrectansweris(A).ThatwhichisPERMEABLEcanbepenetrated,especiallybyfluids.Thewordmeaning“allowable”is“permissible”;thatmeaning“durable”or“unending”is“permanent.”Therainbarrelwascoveredwithapermeablecloththatstrainedoutsolidparticles.

5.Thecorrectansweris(A).SCRUPULOUSmeanscarefultodotheright,proper,orcorrectthingineverydetail.Thelegislatortookscrupulouscaretofillouttheethicsformaccurately.

6.Thecorrectansweris(B).ASTALEMATEisadeadlockorimpasse.Neithertheunionnormanagementwouldconcedeapoint,sothenegotiationswereatastalemate.

7.Thecorrectansweris(C).REDUNDANTmeansmorethannecessaryorsuperfluous.Theword“join”indicatesthatunitsareputtogether;therefore,intheexpression“jointogether”theword“together”isredundant.

8.Thecorrectansweris(D).ToSUPPLANTistosupersedeortoreplace.TheConstitutionsupplantedtheArticlesofConfederation.

9.Thecorrectansweris(C).COMMENSURATEmeansproportionate.The2-year-old’svocabularywascommensuratewithherage.

10.Thecorrectansweris(C).TheZENITHisthepointdirectlyoverheadorthehighestpoint.Thesunreachesitszenithatnoon.

11.Thecorrectansweris(A).SUCCORisaid,help,orrelief.Itistreasontogivesuccortotheenemybydistributingfalsepropaganda.

12.Thecorrectansweris(B).ToDISPATCHistosendoutquickly.Donotrelyonthemailservice;dispatchamessengertodeliverthepackage.

13.Thecorrectansweris(D).PORTABLEmeanseasilymovedorcarried.Thewordmeaning“drinkable”is“potable.”Atelevisionsetwithathree-inchscreenisaportableset.

14.Thecorrectansweris(C).VERBOSEmeanscontainingtoomanywordsorlong-winded.Thespeakerwassoverbosethatwethoughttheeveningwouldneverend.

15.Thecorrectansweris(D).SUBVERSIVEmeansseekingtooverthrowortodestroysomethingestablished.Subversiveelementsinthegovernmentareextremelydangeroustostability.

16.Thecorrectansweris(A).ThatwhichisMALLEABLEtendstobechangeable.Beforeitcools,hotmetalisquitemalleableandcanbetwistedintodifferentshapes.

17.Thecorrectansweris(C).PETTYmeanstrivial,narrow,orsmall.Thecrotchetyoldmanwasfullofpettycomplaints.

18.Thecorrectansweris(D).INTREPIDmeansbold,brave,andfearless.Theintrepidastronautswentontheirmissionasiftherehadneverbeenanaccident.

19.Thecorrectansweris(D).ToNEGOTIATEistomakearrangementsortobargain.Thebuyerandsellerofapropertymustnegotiatetoreachafairprice.

20.Thecorrectansweris(B).STERILEmeansextremelyclean,barren,orgerm-free.Forthesafetyofthepatient,thesurgeonmustworkinasterileenvironment.

21.Thecorrectansweris(D).SinceJoshuawasundaunted(notdiscouraged)byhisfailures,apositivewordisnecessary.PERSEVEREDistheonlypositiveword.

22.Thecorrectansweris(C).Inordertoattractstudents,INNOVATIVE(new,novel)methodsarenecessary.

23.Thecorrectansweris(D).Sincethereislessmoneyavailableandfewerbusiness

transactionsarebeingconducted,BUYERShavetheupperhandinhowtheyspendtheirmoney.

24.Thecorrectansweris(D).Multipleoffenseswouldrequireanincreasedpenalty;therefore,ESCALATEistheonlycorrectchoice.

25.Thecorrectansweris(A).Alltheadjectivesdescribingthehotelarepositive;therefore,apositivedescriptionisnecessary.

26.Thecorrectansweris(B).TheoppositeofforcingstudentstotakecoursesisENTICINGthemtodoso.

27.Thecorrectansweris(C).Aninnovativemethodofdealingwithcriminalsisbeingsuggestedincontrasttotheusualmethod,whichisIMPRISONMENT.

28.Thecorrectansweris(C).Thesentencementions“knowledgegainedwithoutexperience”andthenasksfortheoppositecondition,whichmustbeEXPERIENCEwithoutSCHOLARSHIP.

29.Thecorrectansweris(A).Byhavinganinsuranceexpertdiscussthechanges,theschoolsystemwasattemptingtoreduce,orQUELL,ANXIETYoverthoseincreasedfees.

30.Thecorrectansweris(D).Thefirsthalfofthesentencerequiresawordthatcontrastswithpoverty(COMFORTABLE).ThesecondwordmustshowthatpovertyiswidespreadamongolderAmericans.

31.Thecorrectansweris(A).Alittlecommonsenseshouldgiveyouthisanswerimmediately.Choice(C)makesnosenseatall.Apoliceofficer’sjobistopreventcrime,butthatisnottheofficer’sonlyjob,justthePRIMARYjob.

32.Thecorrectansweris(C).AninabilitytoappealtoacertainvotinggroupwouldbeaSHORTCOMINGforacandidate.

33.Thecorrectansweris(B).ThefactthatthefamilyrespectedoneanothermeansthattheyAVERTEDclashes.

34.Thecorrectansweris(C).Health-consciousconsumerswillENCOURAGEchange.

35.Thecorrectansweris(B).Althoughthepoliticianlosttheelection,heMAINTAINEDhispopularitywithsomefollowers.

36.Thecorrectansweris(B).AnarrestordomesticdisputeisconsideredaseriousCRISIS.

37.Thecorrectansweris(D).Anegativeword(FAIL)mustbefollowedbyapositiveword(PAYOFF).

38.Thecorrectansweris(C).Afloodthatdestroyspeople’sbelongingscausesdevastation.

Livingconditionsintheareacanbesaidtohavedeteriorated(worsened).

39.Thecorrectansweris(D).Theword“help”indicatestheneedforapositivewordtocompletethissentence.Therefore,youneedconsideronlychoices(A)and(D).Ofthesetwo,IMPLEMENT(meaningputintoaction)isabetterchoicethanmagnify(meaningtomakelarger).

40.Thecorrectansweris(C).Thewordsrequiredtocompletethethoughtmustbeopposites.ALUXURYissomethingwecaneasilydowithout,butoncewehavehadthatluxuryforawhilewecannolongerdowithoutitanditbecomesanecessity(anESSENTIAL).

Section2:QuantitativeReasoning

1.Thecorrectansweris(B).Itisusefultoknowthatyoumultiplyanddivideby10,100,1000,andsoon,bymovingthedecimalpoint.

2.Thecorrectansweris(C).Thesumoftheanglesofatriangleis180°.AngleBis90°.AnglesAandC,therefore,musttotal180°–90°,or90°.

3.Thecorrectansweris(B).Tofindthevalueofr,subtractsfrombothsidesoftheequation.Thisgivesusr=3s–s,orr=2s.

4.Thecorrectansweris(B).(3×102)+(2×10)+9.Choice(A)=929;choice(C)=300+200+9=509;andchoice(D)=3029.

5.Thecorrectansweris(B).Circumference=π•diameter;diameter=2•radius;π=

C=π•21•2

C=42•π=42• =132'

Youcanalsoestimatethisproblemandchoosetheclosestanswer.

6.Thecorrectansweris(D).Theset{x,y}includesallthosenumberslargerthan–4andsmallerthan2.Consideringonlywholenumbers,thissetincludes–3,–2,–1,0,1.

7.Thecorrectansweris(C).Theorderinwhichnumbersareaddeddoesnotaffectthesum.

8.Thecorrectansweris(A).Wewanttheamountintheparenthesestobeequalto3.Thevalueofxthatwillmaketheamountinparenthesesequalto3is4.

9.Thecorrectansweris(D).RefertotheGeometryReviewsectionifnecessary.

10.Thecorrectansweris(C).Usingtheorderofoperations(PEMDAS),performthe

calculationsinparenthesesfirst.

Next,performtheoperationswithexponents:

Next,performmultiplication,followedbyaddition:

11.Thecorrectansweris(B).Notethat isslightlylargerthan ,or .Choices(A),

(C),and(D)arecloserinvalueto1thanto .

12.Thecorrectansweris(B).100centimeters=1meter.Eachcentimeteris ofameter.

13.Thecorrectansweris(C).FindtheLCM(leastcommonmultiple)of4,9,6,and12,andthisbecomestheleastcommondenominator.

14.Thecorrectansweris(A).Thesetoffactorsof36is:{1,2,3,4,6,9,12,18,36}.Thesetoffactorsof64is:{1,2,4,8,16,32,64}.

Thesetofcommonfactorsis:

{1,2,4}.

15.Thecorrectansweris(D).Thesumoftheanglesofatriangleis180°.180°–115°=65°

16.Thecorrectansweris(D).Theunionofthetwoadjacentlinesegmentscreatesonecontinuouslinesegment.

17.Thecorrectansweris(C).The52playingcardsinadeckconsistof4suitsof13cards

each.Thereisone6ineachofthefoursuits,makingtheprobabilityofdrawinga6 ,or

.

18.Thecorrectansweris(A).Every8milesisrepresentedonthemapby inch.28÷8=

3 ,so3 half-inchunitsareneededtorepresent28miles.3 × = = =1 .

Youmaysolvetheproblemanotherway:If =8miles,1"=16miles.28÷16= ,

so arerequiredtorepresent28miles.

19.Thecorrectansweris(B).Thenumberofaccidentsisirrelevanttothequestion,soAhasnoplaceintheequation.

B(totaldeaths)÷10years= averagedeathsperyear.

20.Thecorrectansweris(A).

ColumnA>ColumnB

21.Thecorrectansweris(D).TherelationshipbetweenColumnAandColumnBcannotbedeterminedfromtheinformationgiven.

22.Thecorrectansweris(A).

ColumnA>ColumnB

23.Thecorrectansweris(B).

OriginalCost+Profit=SellingPrice

Letx=originalcostthenx+0.05(x)=$47.251.05x=$47.25x=$45

$45>$44.89

ColumnB>ColumnA

24.Thecorrectansweris(A).

ColumnA>ColumnB

25.Thecorrectansweris(C).0.25(300)=75(girlswithblondehair)

300–75=225(girlswithoutblondehair)

=

ColumnA=ColumnB

26.Thecorrectansweris(C).Thesumoftheanglesofatriangleequals180°.a+b+c=180andd+e+f=180a+b+c+d+e+f=360alsoe+c=90Therefore,a+b+d+f=270(subtraction)

ColumnA=ColumnB

27.Thecorrectansweris(C).Aparallelograminscribedinacircleisarectangle.Therefore,allanglesequal90°.

Hence,a+c=b+d

ColumnA=ColumnB

28.Thecorrectansweris(A).

0.525>0.5

ColumnA>ColumnB

29.Thecorrectansweris(B).Letx=thenumberofdaysAandBtakeworkingtogether.

Acandothejobin4days;thus,A’srateis .

Bcandothejobin3days;thus,B’srateis .

2days>1 days

ColumnB>ColumnA

30.Thecorrectansweris(A).Theratiobetweenseniorsandthetotalofseniorsandfreshmenis1:4.Theratiobetweenseniorsandthetotalenrollment(includingsophomoresandjuniors)wouldactuallydecrease.

ColumnA>ColumnB

31.Thecorrectansweris(C).Thesumofthethreeanglesofatriangleequals180°.Thus,x+x+60°=180°

2x=120°

x=60°

Therefore,thetriangleisequilateral.Hence,sidem=siden.

ColumnA=ColumnB

32.Thecorrectansweris(D).Therelationshipcannotbedeterminedfromtheinformationgiven.

33.Thecorrectansweris(B).Thesumoftheanglesofatriangleequals180°.Thus,x+2x+90°=180°3x=90°x=30°

Therefore ABCisa30°-60°-90°righttriangle.Ina30°-60°-90°righttrianglethehypotenuseisequaltotwicethesideoppositethe30°angle.

AB=8

8>5

ColumnB>ColumnA

34.Thecorrectansweris(C).

=5and

ColumnA=ColumnB

35.Thecorrectansweris(A).x>0,xispositivey<0,yisnegativeSubstitutesomearbitraryfiguresofyourchoosing,forexample:x=10;y=–2

x–y=10–(–2)=12 x+y=10+(–2)=8ColumnA>ColumnB

36.Thecorrectansweris(A).15%=0.15

0.15(43.50)=6.525

$43.50–6.53=36.97

$36.97>$36

ColumnA>ColumnB

37.Thecorrectansweris(C).

75%= 50%=

ColumnA=ColumnB

Section3:ReadingComprehension

1.Thecorrectansweris(B).Seethefirstsentence.

2.Thecorrectansweris(A).Thisdefinitionisgivenintheexplanatorystatementattheendofthefirstsentence.

3.Thecorrectansweris(B).TheentireselectionextolsthededicationofthePonyExpressridersinthefaceofthehazardstheymet.

4.Thecorrectansweris(A).Thefourthparagraphtellsusthattherewasaten-daylimitinwhichtheroutemustbecovered.

5.Thecorrectansweris(D).Seeparagraph3.BuffaloBillmadethelongestcontinuousrunof384miles;JimMooremadethefastestrun,280milesin22hours.

6.Thecorrectansweris(A).ThelastsentenceofthepassagestatesthatthePonyExpresslostonlyasingledeliveryofmailduringtheentireperiodofitsoperation.Thus,wecaninferthatmailsentbyPonyExpressgotthroughtoitsdestinationreliably.

7.Thecorrectansweris(C).Cluesmaybefoundinthefirstsentence,whichstatesthathistoriansaretryingtolearnmoreabouttheMayflower,andinthefirstsentenceofthethirdparagraph,whichdescribestheboatashistoric.

8.Thecorrectansweris(A).Ifyougotthiswrong,rereadthesecondsentenceofthefirstparagraph.

9.Thecorrectansweris(B).Seethefirstsentenceofthesecondparagraph.

10.Thecorrectansweris(D).TheoriginalMayflowerwas22feetwide;theduplicateis26feetwide.Thelengthsareboth90feet.

11.Thecorrectansweris(D).Tounderstandthemeaningofthisword,wemustlooktothecontextofthepassageforclues.Thesentenceitselftellsusthat“theMayflowerwentbacktolesscolorfuljobs”beforeitwasscrapped.ItsearliertaskhadbeentodeliverthePilgrimstoAmerica,whichwasajobofhistoricalsignificance.So,wearelookingfora

wordthatmeanssomethinglike“significant.”Thewordinterestingistheclosestfithere.Theotherchoicesdonotmakesensewhensubstitutedintothesentence.

12.Thecorrectansweris(B).YoucaninferthismeaningfromthefactthattheauthordescribestwobuildingsinEnglandthatmaycontainpartsoftheMayflower.

13.Thecorrectansweris(B).Theselectiondescribesthevaryinghare.

14.Thecorrectansweris(C).Asthenamesimply,therabbitsvarytheirappearancewiththeseasons.

15.Thecorrectansweris(C).Thesentencestatesthatwhiletheharesarechangingcolor,theircoatsareamottledbrownandwhite.Intherestoftheparagraph,welearnthathareschangefromallbrowntoallwhiteinthefall,andbacktoallbrowninthespring.Itmakessensetoexpectthatthehares’coatswouldbebothbrownandwhiteastheygothroughthecolorchange.Thewordspeckledconveysthatthecoatshaveflecksofbothcolors,sochoice(C)iscorrect.

16.Thecorrectansweris(B).Therabbitsaremottledbrownandwhitewhileinthemiddleofthemoltingprocessduringspringandfall.

17.Thecorrectansweris(B).Seethefirstsentenceofthelastparagraph.

18.Thecorrectansweris(D).Aparentheticalstatementinthesecondparagraphstatesthatthehareshavenovoluntarycontroloverthechangesintheirappearance.

19.Thecorrectansweris(B).TheentireselectionisaboutcourtpracticesinancientAthens.

20.Thecorrectansweris(C).Theanswerisinthesecondsentence.

21.Thecorrectansweris(A).Seethelastsentence.

22.Thecorrectansweris(B).Seelines9–11.Theaccusedwasallowedacertaintimetospeak;thejuryvotedbydepositingblackorwhitestonesinabox.

23.Thecorrectansweris(A).Inparagraphfour,welearnthattrialdecisionsweremadewithoutdeliberationbyjury,so(B)isincorrect.Thepassagestatesthatjurymemberscasttheirvotesindividually,whichconfirmsanswerchoice(A).

24.Thecorrectansweris(B).Thelastparagraphdescribesarulethatwasdesignedtopreventcitizensfrombeingtoocarelessinaccusingeachother.Therulestatedthatifthepersonaccuseddidnotreceiveacertainnumberofnegativevotes,theaccuserwouldbecondemnedinstead.Inotherwords,thepersonaccusedmustreceiveatleastaminimumnumberofvotescastagainsthim,toshowthatthecasewasvalid.

25.Thecorrectansweris(D).Thesecondsentencetellsusthatthetemperatureoftheearth’supperatmospherevarieswithtime,location(altitude),andsolaractivity(thesun).

Nomentionismadeoftheweather.

26.Thecorrectansweris(D).Thefirstparagraphmakesclearthatthetemperature(theheatofthesun)directlyaffectsmanyothermeasuresanddimensionsintheupperatmosphere.

27.Thecorrectansweris(B).Accordingtothepassage,heavygases,suchasargonandcarbondioxide,aremoretightlyboundbytheearth’sgravitationalfield.Lightgases,suchashydrogenandhelium,arelesstightlybound.

28.Thecorrectansweris(C).Thesecondparagraphofthepassageliststhemeanmolecularweightoftheatmosphereatsealevelas29atomicmassunits(amu).

29.Thecorrectansweris(C).Seethelastsentence.

30.Thecorrectansweris(D).Seethefirstsentence.

31.Thecorrectansweris(C).Seethefirstsentence.

32.Thecorrectansweris(D).ThelastsentencetellsusthatthephotographsremainedinMississippi.TherecordingsthemselvesweresenttotheLibraryofCongress.

33.Thecorrectansweris(C).ThesecondparagraphtellsoflocalWPAworkersandtheassistancetheygave.

34.Thecorrectansweris(B).Inparagraphfour,FerrissisdescribedasaMississippinative.ThismeansthathewasoriginallyfromMississippi,sochoice(B)iscorrect.

35.Thecorrectansweris(A).ThepassagementionsinparagraphfourthatFerrisswasemployedbytheFederalWriters’ProjectduringthetimethatheassistedHalpertin1939.

36.Thecorrectansweris(D).ThelastparagraphstatesthattherecordingswenttotheLibraryofCongress,andthephotographsandmanuscriptmaterialswereretainedinMississippi.Fromthecontext,wecaninferthateventhoughtherecordingswentelsewhere,thephotographsandmanuscriptmaterialsstayedinMississippi.Choice(D),kept,bestreflectsthismeaning.Nothinginthepassagesuggeststhatthephotographsandmanuscriptmaterialwerelost,sochoice(A)isincorrect.

Section4:MathematicsAchievement

1.Thecorrectansweris(C).Firstfindouthowmuchsugarisneededforonequartofpunch:

cup÷6=

For9quartsofpunchyouneed:

2.Thecorrectansweris(A).45badges×4incheseach=180inchesneeded.Thereare36inchesinoneyard:180inches÷36=5yardsofribbonneeded.

3.Thecorrectansweris(D).$60×0.10=$6(employeediscount)$60–$6=$54

$54×0.20=$10.80(salediscount)

$54–$10.80=$43.20

4.Thecorrectansweris(B).Areaequalslengthtimeswidth.A=l×wA=10ft.×8ft.

A=80sq.ft.

5.Thecorrectansweris(D).Thedecimalnumber0.375isequivalenttothefraction .

Thelineoverthedecimalnumberindicatesarepeatingnumberthatgoesonforever,so(A)and(B)areincorrect.

6.Thecorrectansweris(D).Foreachofthe19,000fatalaccidentstherewere130nonfatalinjuries.Tofindthetotalnumberofnonfatalaccidents,wemultiply19,000×130=2,470,000.Therewereapproximately2,500,000nonfatalaccidentsin1972.

7.Thecorrectansweris(D).Anynumbermultipliedby0equals0.Sinceonemultiplierononesideofthe=signis0,theproductonthatsideofthesignmustbe0.Thevalueontheothersideofthe=signmustalsobe0.

5x=5×4×2×05x=40×05x=0x=0

8.Thecorrectansweris(D).Anitemmarkedup100%hasaretailpricetwicethewholesaleprice.Thedressnowcosts$130,whichistwice$65.

9.Thecorrectansweris(B).Thewidthoftheroomwillbe17× ,or2 .Thelength

oftheroomwillbe22× ,or .

10.Thecorrectansweris(A).Theroomhasanareaof28'×20'=560sq.ft.Eachrollof

carpetcancover90sq.ft.Thenumberofrollsrequiredis560÷90=6 rolls.

11.Thecorrectansweris(A).Tosquareanumber,multiplyitbyitself.

100–x=52

100–x=5×5100–x=25100–25=xx=75

12.Thecorrectansweris(B).Ify=2,then72x=6(2)72x=12

x=

13.Thecorrectansweris(A).Borrow60minutesandrewriteas:

14.Thecorrectansweris(D).

15.Thecorrectansweris(D).Tenmoregallonsofwaterwouldbringthevolumeofthemixtureto30gallonsofwater+5gallonsofacid=35gallons.Thepartthatiswateris

,or .

16.Thecorrectansweris(D). ofamonthisaboutoneday.

17.Thecorrectansweris(C).Acirclegraphcontains360°,whilearightanglecontains

90°.Arightangle,therefore,contains ,or25%,ofthecircle.

18.Thecorrectansweris(A).Tofindthevalueofthisexpression,multiplythebinomialsusingtheFOILmethod.Multiplythefirst,outer,inner,andlastterms:

The–7zand+7zinthemiddlecanceleachotherout,andweareleftwithz2–49.

19.Thecorrectansweris(B).Theperimeteristhesumofthelengthsofthefoursides.Arectanglehastwopairsofsidesofequallength.Theperimeterthenis(2×115)+(2×63)=356feet.

115+63+115+63=356

Thisisagoodproblemtosolvebyestimation.Youcanreadilydiscardthreeofthepossibleanswerswithoutdoinganycalculation.

20.Thecorrectansweris(B).Don’tbothertocalculatehere!Noticethatyouranswerwillbeverycloseto6×1.Thereisonlyoneansweranywherenearthatestimate.

21.Thecorrectansweris(A).Thesquareofanumberdividedby2.5equals10.Thesquareofthenumber,then,equals10multipliedby2.5.

x2=10×2.5

x2=25x=±5

22.Thecorrectansweris(D).Eachyear,12%ischargedasinterest.On$2000,12%interestis$2000×0.12=$240.Fortwoyears,theamountis$240×2=$480.

23.Thecorrectansweris(B).Findthevolumebyfindingtheareaofthetriangularendand

multiplyingbythelength.A= bh.TheareaofthetriangularendisA= (3)(2)=3sq.

ft.Multiply3sq.ft.bythelengthtofindthevolume:3sq.ft.×6ft.=18cu.ft.

24.Thecorrectansweris(D).Notethatthesecondpieceistheshortest.Wedon’tknowitsexactlength,socallitxfeetlong.Thefirstpieceisthreetimesaslongasthesecond,or3xfeetlong.Thethirdistwiceaslongasthefirst,or6xfeetlong.Allthepiecestotalx+3x+6x=10xfeet.Theboardis30feetlong,andthepiecesare10xfeetlong.10x=30,sox=3,whichisthelengthoftheshortestpiece.Thelargestpieceissixtimestheshortest,or18feetlong.

25.Thecorrectansweris(C).Weassumethatthemerchantwantstohavethesametotalvalueofwineafterreducingtheprice.Henowhas(32•$1.50),or$48worthofwine.At$1.20pergallon,hewouldneed40gallonstohave$48worth.Therefore,hemustadd8gallonsofwater.

26.Thecorrectansweris(B).Ifsixisfourmorethan ofanumber,then6–4equals of

thenumber.Since6–4=2,weknowthat2is ofthenumber,sothenumberis3.

Anotherwaytosolvethisproblemistowriteanequation,andsolveforx:

x+4=6

x=2

x=3

27.Thecorrectansweris(A).Youhavetoreadonlythefirstandthirdsentencesoftheproblem.Theinformationinthesecondsentencecontainsinformationnotrelevanttothe

problem.Thewinnerreceived ofthetotal,or$2700.Thus,thetotalpursewas$2700×

3=$8100.

28.Thecorrectansweris(B).Thedistancebetweenthecarsisbeingreducedatarateequaltothesumoftheirspeeds.Theyarecomingclosertogetherat35+48=83milesperhour.Sincethedistancebetweenthemwas450miles,thetimerequiredfortravelingis450÷83=5.42hours.Rememberthatdistanceistheproductofrateandtime,ord=rt.

29.Thecorrectansweris(B).Ofhistotal,heissued ,sohehad pads

remaining.Histotalinstockwas600.600• =125padsremaining.

30.Thecorrectansweris(B).Thefirstmancanload ofthetruckin1minute.The

secondmancanload ofthetruckin1minute.Thethirdmancanload ofthetruck

in1minute.Togethertheycanload + + ofthetruckeachminute.

ofthetruckloadedperminute.Thewholejobthenrequires minutes,or6 minutes.

31.Thecorrectansweris(C).Thisisadifficultquestionunlessyouhavehadsomealgebra.Theequationsare“added.”They-termiseliminated,leavingxonly.

32.Thecorrectansweris(B).6+x+y=20x+y=14=k;nowsubstitute20–14=6

33.Thecorrectansweris(C).Acircleis360°;60°is of360°.

34.Thecorrectansweris(C).Diagramthisproblem:

35.Thecorrectansweris(C).Pickapairofagesandtryforyourself.Ais4;Bis2;theratiooftheiragesis4to2,or2to1.Intwoyears,Ais6andBis4.Theratiooftheiragesis6to4,or3to2.

36.Thecorrectansweris(B).15°–21°=–6°

37.Thecorrectansweris(B).Ifthesidesareparallel,theanglesarecongruent.

38.Thecorrectansweris(B).6wordsperline×5linesperinch=30wordsperinch.30wordsperinch×8inches=240words.

Ifthearticlehas270wordsandthereisspaceforonly240words,then30wordsmustbedeleted.

39.Thecorrectansweris(C).S=1;R=3×1;T=

1+(3×1)+ =4

40.Thecorrectansweris(D).Thelawnis8'×12'=96sq.ft.96×5=480pintsofwaterneeded

8pts.in1gal.;480÷8=60gallonsneeded

41.Thecorrectansweris(A).l=500+(500×0.20×2)l=500+200l=700

42.Thecorrectansweris(C).Rightnow,hegets120mi.÷12gal.=10mpg.With80%moreefficiency,hewillneed80%of12,or9.6gal.togo120miles.Hewillthenget120mi.÷9.6gal.=12.5mpg.

12gal.×12.5mpg=150mileson12gal.

43.Thecorrectansweris(D).4feet=48inches;48÷ =48×4=192books.

44.Thecorrectansweris(B).Thereare6+8+4+12=30marbles.12÷30=0.40=

45.Thecorrectansweris(C). isahypotenuse,sousethePythagoreanTheorem:

46.Thecorrectansweris(D).33×3=27×3=81

47.Thecorrectansweris(D).(3+2)(6–2)(7+1)=(4+4)(x)(5)(4)(8)=8x8x=160x=20=8+12

Section5:Essay

Exampleofawell-writtenessay.

WhenIenterhighschool,Iplantobecomeanactivememberofthedramaclub.Thedramacluboffersavarietyofactivitieswithinoneorganization.Inthecourseofasingleyear,amemberofthedramaclubcangetinvolvedinacting,setbuilding,lightingdesign,publicity,ticketsales,andmuchmore.Andbecauseofthevarietyofactivities,Iexpecttomakefriendswithclassmateswithvariedinterestsandabilities.

Peopleinvolvedwiththeaterappeartobehavingalotoffun.Whiletheremaybesomecompetitionamongstars,aproductionisgenerallyacooperativeeffort.Teamworkiskeytomakinganamateurproductionappeartobeprofessional.EveninkindergartenIgothighmarksfrommyteachersin“worksandplayswellwithothers,”andIwouldliketocarrythisaspectofmypersonalityintojoiningacastandcrewthatcreatestheater.

Anotherreasonforjoiningthedramaclubisthatitwillgivemeachancetoperform.Ihavealwaysbeenabitofashow-off.Beingonstagewillallowmetostrutaboutwithoutbeingcriticized.Ihopethatmyactingwillcontributetosuccessfulproductionsalongwithmyworkaspartofthebehind-the-scenescrew.

Finally,anycooperativeeffortmustbeasocialactivity.Thereshouldbelotsofgiveandtakeandconversationduringpreparationsandrehearsals.AndIdolookforwardtocastpartieswhentheshowcloses.Dramaclubseemsliketheperfectextracurricularactivity;Iwillhaveagoodtimewhiledoingsomethingworthwhile.

SCOREYOURSELFScoresontheISEEaredeterminedbycomparingeachstudent’sresultsagainstallotherstudentsinhisorhergradelevelwhotookthatparticulartest.Ascaledscoreisthencalculated.Youcanusethefollowingcalculationstodeterminehowwellyoudidonthispracticetest,butkeepinmindthatwhenyoutaketheactualtest,yourscoremightvary.

Test RawScore ÷No.questions ×100= _____ %

Synonyms _____ ÷20 ×100= _____ %

SentenceCompletions _____ ÷20 ×100= _____ %

TotalVerbalAbility _____ ÷40 ×100= _____ %

Multiple-ChoiceQuantitative _____ ÷19 ×100= _____ %

QuantitativeComparisons _____ ÷18 ×100= _____ %

TotalQuantitativeAbility _____ ÷37 ×100= _____ %

ReadingComprehension _____ ÷36 ×100= _____ %

MathematicsAchievement _____ ÷47 ×100= _____ %

Highpercentagescoresshouldmakeyoufeelverygoodaboutyourself,butlowpercentagesdonotmeanthatyouareafailure.

Remember:•Scoresarenotreportedaspercentages.Alowpercentagemaytranslatetoarespectablescaledscore.

•Thesametestisgiventostudentsingrades8through12.Unlessyouhavefinishedhighschool,youhavenotbeentaughteverythingonthetest.Youarenotexpectedtoknowwhatyouhavenotbeentaught.

•Youwillbecomparedonlytostudentsinyourowngrade.

Useyourscorestoplanfurtherstudyifyouhavetime.

APPENDIXES

AppendixA:ListofSynonyms

AppendixB:Parents’GuidetoPrivateSchools

AppendixC:PrivateSchoolsAt-a-Glance

AppendixA:ListofSynonyms

Aabbreviate—shorten,makeconciseabrasive—harsh,rough,irritatingabstain—refrain,giveup,holdbackacclaim—praise,approve,applaudaccumulate—acquire,gain,hoardaccuse—challenge,blame,incriminate(toattributeresponsibility)adaptable—flexible,changeableadept—skillful,proficient,competentadhere—attach,stick,follow,upholdadversary—enemy,opponent,foe,nemesis(anarchenemy)adverse—bad,negativeaggressive—combative,belligerent(hostile)agitate—irritate,anger,upset,stiragreeable—pleasant,likeable,delightfulambiguous—unclear,vagueambitious—determined,driven,motivatedambivalent—indecisive,wishy-washy,unsureamplify—expand,heighten,enlargeanimated—energetic,lively,spiritedanimosity—hostility,resentment,hatredanomalous—odd,inconsistent,irregular,unusualanonymous—nameless,unknown,unidentifiedantagonize—irritate,bother,annoyapathy—indifference,unconcern,disregard(lackofinterest)arbitrary—random,chance,inconsistentarid—dry,barren,parched(lackingwater)attentive—interested,observant,awareastute—quick-witted,intelligent,smartatypical—notnormal,uncommon,unnaturalaudacious—outrageous,bold,daring

audible—perceptible,discernible,distinct(abletobeheard)authentic—original,trustworthy,credibleautonomous—independent,self-governing

Bbaffle—confuse,stump,puzzlebanal—usual,common,ordinarybarren—lifeless,empty,unfruitful(notabletosupportlife)belated—late,overduebenevolent—kind,good-heartedbenign—mild,peaceable,harmlessberate—scold,criticize,reprimand(toscoldorblame)bleak—grim,hopeless,desolate(desertedandempty)boisterous—noisy,loud,rambunctious(uncontrolled)bombastic—boastful,ostentatious(showy),pompous(fullofoneself)buttress—bolster,reinforce,support

Ccajole—coax,persuade,wheedle(toconvincebyaskingnicely)camaraderie—friendship,companionship,togethernesscandid—truthful,straightforward,unrehearsedcandor—honesty,directness,veracity(truthfulness)capricious—willful,arbitrary,impulsive(actingwithoutthought)captivate—dazzle,enchant,fascinatecaustic—burning,hurtful,sarcastic(cuttingormocking)chaos—disorder,confusion,pandemonium(anuproarorhubbub)circumvent—goaround,avoid,elude(togetawayfrom)clandestine—secret,undercover,covert,surreptitious(hidden)cloying—sticky,sentimental,clingycoerce—force,bully,pressurecogent—powerful,logical,persuasivecognizant—aware,informed,sentient(conscious)coherent—understandable,clearcohesive—close-knit,unified,interconnectedcollaborate—worktogether,cooperate,joinforcescommend—praise,applaud,honorcompatible—harmonious,well-suited,congenial(friendly)

compel—force,require,pressurecompetent—skilled,qualified,proficient(goodat)complacent—self-satisfied,comfortable,smug(self-satisfied)comply—obey,conform,followcomprehensive—thorough,inclusive,completeconcise—short,tothepoint,succinct(brief)condescending—rude,snobbishconfident—sure,convinced,positiveconform—comply,submit,followconformity—compliance,submissioncongested—packed,jammed,blockedcongruent—alike,matching,harmonious(goeswelltogether)consensus—agreement,compromise,harmonyconsequential—important,major,meaningfulconservative—traditional,old-fashioned,conventionalconstant—steady,persistent,incessant(unceasing)constrain—holdback,restrict,inhibit(toslowdownorprevent)consummate—ideal,perfect,superlative(thebest)contaminate—pollute,spoil,taint(tospoilordamage)contemporary—modern,uptodate,newcontradict—disagree,oppose,challengeconventional—usual,established,typicalconverge—meet,cometogether,joinconvey—tell,express,communicateconvoluted—complex,difficultcorroborate—confirm,support,substantiate(tobackupwithevidence)corrupt—dishonest,shady,crookedcovert—secret,hidden,undergroundcriticize—disparage(tocutdown),denigrate(toputdown)culpable—responsible,guilty,atfaultcurtail—cutback,limit,shorten,restrict

Ddebased—corrupted,depraved(wicked)decisive—determined,conclusive,suredecorous—polite,proper,suitabledecry—devalue,disparage(tocriticize),demean(toputdown)

deficient—lacking,insufficient,notenoughdeficit—deficiency,loss,shortagedefinite—certain,explicit,indubitable(notquestionable)defy—disregard,flout(togoagainst)delete—remove,takeaway,expunge(togetridof)deleterious—damaging,hurtful,injurious(harmful)denigrate—malign,impugn,slander(toputdown)deplete—diminish,reduce,useupdeplore—despise,hate,undervaluedeprecate—ridicule,disparage(tocriticize),denigrate(toputdown)deprive—takeaway,rob,removedesire—want,longing,cravingdesolate—barren,lifeless,devoid(empty)destitute—poor,indigent(withoutmoney)destroy—eliminate,obliterate(towipeout),raze(tocompletelydestroy)detach—remove,segregate,separatedeter—stop,halt,hinderdetractor—critic,enemydetrimental—bad,harmful,unfavorabledevastate—destroy,wreck,annihilate(todemolish)devious—dishonest,evil,duplicitous(scheming)diffuse—spreadout,expanded,propagated(spreadout)digress—stray,ramble,deviate(togooffinanotherdirection)diligent—hard-working,earnest,persistentdiminish—decrease,dwindle,reducediminutive—petite,small,shortdire—critical,veryimportant,desperate,grave(serious)disagree—conflict,goagainst,dissent(todifferinopinion)discern—recognize,distinguish,perceivedisclose—tell,expose,reveal,makeknowndiscomfort—unpleasantness,irritation,pain,anguish(greatpain)disconcerting—disturbing,unbalancing,upsettingdiscord—disharmony,conflictdiscrepancy—variation,difference,incongruity(difference)dismal—bleak,sad,horribledismay—disappointment,discouragement,trepidation(anxietyorfear)

dismiss—sendaway,discard,pushasidedisparage—mock,criticize,belittle(toputdown)disparate—atvariance,contrasting,differentdisparity—imbalance,gap,inequitydispute—bicker,argue,contenddisruptive—disorderly,disturbingdisseminate—publicize,scatter,radiate,dispersedissipate—deplete,useup,squander(towaste)distinct—separate,clearlydefined,explicit(obvious)distorted—warped,bentoutofshape,perverteddiverge—separate,deviate(togooffcourse)diverse—dissimilar,different,varieddivide—splitup,disjoin,partition(tocutup)divulge—bringtolight,confess,telldominant—superior,controlling,maindominate—ruleover,influence,overshadowdubious—suspicious,doubtful,disputableduplicitous—two-faced,shady,dishonestdurable—rugged,tough,tenacious(persistent)duress—hardship,suffering,threat

Eendure—bear,withstand,suffer,tolerate,copewitheffusive—expressive,gushing,unrestrainedegotism—narcissism,self-absorption(focusingonlyonone’sselfandone’sowndesires)elaborate—ornate,refined,complicatedelated—thrilled,joyful,euphoric(extremelyhappy)elusive—mysterious,puzzling,baffling(difficulttounderstand)eminent—prestigious,well-known,illustrious(well-knowninapositivemanner)emulate—copy,mimic,actlikeencompass—include,circumscribe,encircleenervate—weaken,incapacitate,drainengage—dealwith,undertake,employengender—incite,provoke,rouse(tomovetoaction)enhance—heighten,improve,increaseenrich—improve,enhance,aggrandize(tomakebigger)enthrall—charm,captivate,mesmerize(tocapturetheattentionof)

eradicate—eliminate,destroy,getridoferratic—irregular,unpredictable,volatile(explosive)espouse—advocate,defend,supportessential—necessary,requisite(required),indispensible(verymuchneeded)euphoric—excited,thrilled,veryhappyexacerbate—embitter,intensify,irritateexasperate—provoke,rileup,infuriate(tomakeangry)exclude—keepout,omit,ostracize(tobansomeonefromagroup)exculpate—forgive,excuse,acquit(tofreefromguilt)exempt—notrequiredto,immuneexonerate—holdblameless,vindicate(tofreefromguilt),exculpate(tofreefromguilt)expand—enlarge,increase,swellexpansive—all-inclusive,broad,widespreadexpedite—quicken,hurry,hastenexpert—skilled,knowledgeable,experiencedextenuate—diminish,lessenextol—praise,exalt,acclaim(toraveabout)extraneous—extra,notneeded,unnecessary,irrelevantextricate—liberate,freeextrinsic—foreign,alien,external(outside)exuberant—cheerful,buoyant(high-spirited),ebullient(fullofpositiveenergy)

Ffabricate—manufacture,makeup,formulatefallacy—falsehood,lie,deceptionfanatic—lunatic,zealot,radical(extremist)fecund—propagating,fertile,fruitful(abletoreproduce)felicitous—appropriate,suitable,apropos(appropriate)feral—wild,savage,untamedfervent—sincere,impassionedflagrant—shameless,undisguised,brazen(brash)flamboyant—glamorous,over-the-top,pretentious(showy)forbid—prohibit,disallow,banformal—official,established,conventionalforthright—sincere,honest,candid(openanddirect)fortitude—courage,endurance,tenacity(persistence)foster—champion,support,nurture

fragile—breakable,weak,frailfrenetic—obsessive,overwrought(veryupset),maniacal(frenzied,likeamadman)frivolity—whimsicality,silliness,childishness,playfulnessfrugal—economical,penny-pinching,thriftyfurtive—secretive,clandestine(hidden),stealthy(doneinasneakyway)futile—pointless,purposeless,trifling(havingnovalue)

Ggargantuan—huge,gigantic,enormousgenerate—make,create,producegenial—cordial,amiable,kindly(likeable)grandiose—exaggerating,pompous(fullofoneself),ostentatious(actinglikea“show-off”)gratuitous—excessive,uncalledfor,unnecessarygreedy—gluttonous,insatiable(strongdesireforselfishgain)gregarious—sociable,outgoing,good-naturedguileless—truthful,honest,straightforwardgullible—simple,credulous(easilyfooled)

Hhackneyed—stale,common,trite(somethingoverdoneorconstantlyrepeated)haughty—snotty,narcissistic(focusedontheself),arrogant(feelingsuperiortoothers)heinous—wicked,repugnant(verybad),atrocious(awful)heretic—pagan,unbeliever,iconoclast(onewhogoesagainstabeliefsystem)hesitate—pause,defer(toputoff),balk(torefusetomoveforward)honorable—well-regarded,law-abiding,esteemed(well-respected)hypocrisy—phoniness,fraudulence(fakeness),duplicity(lying)hypothetical—supposed,presumed,guessed

Iimitate—copy,mimic(copy),impersonate(toactlikesomeone),emulate(tostrivetobelike)immature—childish,infantile(likeaninfant)impede—block,hinder,stymie(thwart)impersonate—mimic,copy,imitate(toactlike)impetuous—hasty,rash(actingwithlittlethought)impulsive—unpredictable,erratic(irregular),hasty(actingquickly,withoutthought)inarticulate—stammering,incomprehensible,tongue-tied(notwell-spoken)incisive—clever,acute,sharp(quick-witted)incompatible—opposite,clashing(conflicting),disparate(different)

incongruent—unlike,conflicting,inconsistentincontrovertible—irrefutable,unquestionable,sureincorporate—include,join,merge,mixincriminate—accuse,involve,blameindecision—ambivalence,hesitancy,tentativenessindependent—self-determining,free,self-sufficient,liberatedindeterminate—inexact,inconclusive,imprecise(notaccurate)indict—accuse,condemn,blameindifference—disinterest,apathy(lackofinterest)indistinct—poorlydefined,murky(unclear),ambiguous(notclearlymarkedorunderstood)induce—motivate,cause,instigate(tosetinmotion)inept—unskillful,clumsy,incompetent(notcapable),bungling(pronetomakingmistakes)inevitable—unavoidable,impending(happeningsoon),destined(boundbydestinytohappen)infamous—disreputable(havingabadreputation),notorious(well-knowninanegativeway)informal—casual,unofficial,unfussyingenious—gifted,intelligent,resourceful(clever)inherent—built-in,natural,innate(foundnaturallywithin)inhibit—constrain,suppress,restrain,preventinhibited—shy,subdued,reserved(quietortimidinmanner)initiate—start,begin,inaugurate(toimplement)initiative—drive,motivation,gumptioninnocuous—inoffensive,mild,harmlessinsipid—dull,tedious,boringinsolence—crudeness,disrespect,impertinence(lackofrespect)instigate—start,initiate(tobegin),foment(tostimulatetoaction)integrate—puttogether,merge,harmonizeintimidating—coercive,threatening,compellingintrepid—fearless,courageous,undaunted(unafraid)inundate—overload,flood,overwhelminvariable—unchanging,consistent,constant,steadyinvigorate—energize,stimulate,enliveninvincible—strong,unbeatable,indomitable(unabletobeconquered)irate—angry,furious,enragedirrational—illogical,nonsensicalirregular—variable,unsteady,inconsistent,unusual,variegated(fullofvariety)irresolute—uncertain,undecided,indecisive

irritate—annoy,upset,aggravate

Jjeopardize—endanger,threatenjovial—happy,upbeat,good-naturedjudicious—thoughtful,cautious,prudent(wise)

Kkeen—sharp,quick,astute(smart)

Llaborious—difficult,hard,demandinglackluster—dull,boring,uninterestinglament—mourn,regret,grievelanguid—slow,sluggish,weaklanguish—droop,decline,sufferlatent—unexpressed,inactive,hidden,undevelopedlavish—extravagant,posh,opulent(abundant)lax—relaxed,loose,permissive,lenient(notstrict)lazy—unmotivated,unenergetic,indolent(avoidingwork)legitimate—actual,real,verified(showntobetrue)lenient—allowing,permissive,forgiving,lax(notstrict)lethargic—slow,lazy,sluggish,inactiveliability—obligation,debt,weakness,disadvantagelinger—stay,loiter,delaylivid—angry,irate,furiouslofty—high,ambitious,pretentious(showy)loquacious—talkative,chatty,wordy,garrulous(talkative)lucid—clear,understandable,rational

Mmalevolence—hatred,illwill,malice(intenttoharm)malingering—lazy,shirking(avoidingduties)malignant—harmful,dangerousmalleable—changeable,bendable,pliable(easytobend)mature—full-grown,developed,ripemediocre—unimpressive,ordinary,average,so-somercurial—ever-changing,unpredictable,fickle(notloyal),capricious(quicktochange)

meticulous—neat,careful,detailed,precise(accurate)misconception—misunderstanding,misperception,falsebeliefmitigate—reduce,lessen,relievemoderate—mild,medium,averagemodicum—bit,tidbit,morsel(atinyamount)mollify—soothe,calm,pacify(tocalm)moribund—dying,ending,declining,terminal(attheend)morose—negative,dark,sullen(gloomy)mundane—ordinary,commonplace,everydaymunificent—generous,lavish,liberal(givingfreely)mutable—changeable,flexible,malleable(abletobechanged)

Nnaive—innocent,trusting,newcomernarcissistic—self-absorbed,conceited,selfishnebulous—vague,unclearneglect—forget,abandon,overlooknemesis—archenemy,opponent,adversarynocturnal—nighttime,vampirishnonchalant—casual,relaxed,laid-backnotorious—disreputable(havingabadreputation),infamous(well-knownforabadreason)novel—new,unique,imaginativenovice—beginner,trainee,neophyte(newlearner)nurture—carefor,foster,protect

Oobjective—fair,unbiased,open-mindedobliterate—demolish,eliminate,eradicate(wipeout)oblivious—unresponsive,unaware,forgettingobscure—unknown,minor,unseenobsolete—outdated,irrelevant,archaic(old)obstinate—stubborn,headstrong,obdurate(stubborn),tenacious(determined)obtrusive—obvious,prominent,blatant(highlynoticeable)occlude—block,obstruct,impede(toholdback)omnipotent—supreme,invincible(unstoppable)onerous—burdensome,tedious(repetitiveandboring),arduous(difficultanddemanding)opinionated—inflexible,unbending,dogmatic(havingrigidopinions)

opponent—rival,foe,challengeropportune—well-timed,advantageous(helpful)optimistic—hopeful,positive,sanguine(confident)opulent—lavish,luxurious,sumptuous(expensive)ordinary—common,usual,regular,normalornate—lavish,bejeweled,adorned(decorated)orthodox—conventional,mainstream,usualostentatious—flashy,flamboyant,pretentious(showy)ostracize—banish,ignore,castoutovert—open,unconcealed,blatant(obvious)

Pparched—dry,dehydrated,waterlesspariah—outcast,untouchable,exilepassion—enthusiasm,zeal,delightpassive—sluggish,lifeless,inert(inactive)pathetic—pitiful,wretched,lamepenitent—sorry,apologetic,contrite(sorry)perceptive—insightful,observantperish—die,passaway,expireperplexing—puzzling,bewildering,mystifyingpersevere—persist,continue,keeponperturb—annoy,disturb,botherpervasive—omnipresent,all-encompassing(foundeverywhere)pessimistic—gloomy,negativepious—religious,reverentplacate—soothe,pacify,appease(tocalmdown)placid—calm,peaceful,easygoingplausible—believable,possible,likelypopular—appealing,well-liked,admiredpotent—powerful,strong,effectivepractical—useful,sensible,no-nonsenseprecede—lead,gobeforepredatory—aggressive,rapacious(outtokill)predominant—major,principal,mostcommonpretentious—showy,conceited,self-importantprivilege—advantage,benefit

procrastinate—postpone,delayproficient—skilled,talented,capableproliferate—increase,flourish,spread,thrive(todowell)propagate—spread,transmit,publicizepropensity—tendency,inclination,penchant(tendency)proponent—supporter,advocate,fanpropriety—respectability,politenessprosaic—dull,ordinary,commonplaceprosperous—wealthy,affluent,abundantproximity—closeness,convenience,nearnessprudent—wise,cautious,practicalpunitive—penalizing,disciplinary,retaliatory(punishing)purify—cleanse,distill,filter,sanitize

Qquell—crush,defeat,conquer,suppressquerulous—difficult,irritable,argumentative,cantankerous(argumentative)quarrelsome—querulous,cranky,grouchy,bad-temperedquiescent—quiet,sluggish,passive(inactive),dormant(inactive)

Rrandom—accidental,haphazard,chance,casualrational—sane,normal,coherent(makessense)rebut—deny,disprove,invalidaterecede—ebb,diminish,drawbackreclusive—isolated,solitary,withdrawnreconcile—reunite,resolve,bringtogetherrecuperate—recover,getwell,improverefined—polished,developed,cultivatedremedy—cure,restore,fixremorse—guilt,sorrow,regret,shameremote—distant,isolated,farrenovate—renew,refresh,repairrenounce—reject,abandon,denyreplete—full,stuffed,plentifulreprehensible—criminal,wicked,disgracefulrepress—restrain,control,suppress,stifle(toholdback)

reputable—trustworthy,dependable,respectable,legitimateresilient—flexible,elastic,reboundingresplendent—dazzling,magnificent,glorious,stunningrestrain—control,confine,holdbackretain—keep,save,preserverevere—admire,respect,esteem(toregardhighly)robust—healthy,strong,vigorousroutine—usual,ordinary,normalrupture—break,burst,ripopen

Ssagacious—wise,shrewd,learned,perceptivesaturate—soak,flood,inundate(tooverwhelm)scrutinize—examine,inspect,analyzesecrete—conceal,hide,stashsedentary—inactive,immobile,lethargic(slow)sequential—inorder,chronological(sortedbytime)serene—calm,peaceful,tranquilskeptical—doubtful,unconvinced,disbelievingsolace—comfort,support,reliefsoporific—dull,sleep-inducing,monotonous(boring)sparse—limited,scarce,inadequate,scant(asmallamount)spontaneous—unplanned,spur-of-the-moment,impromptu(unplanned)squander—waste,spend,misusestagnant—still,inactive,inert(notactive)sterile—antiseptic,disinfected,sanitarystimulate—encourage,motivate,inspirestrenuous—taxing,straining,demanding,arduous(difficult)strict—firm,exacting,rigoroussubmissive—obedient,passive(inactive),compliant(willingtoobey)substantial—considerable,extensive,sizeable,significantsubstantiate—verify,prove,corroborate(toprove)subtle—slight,understated,delicatesullen—brooding,grim,gloomysummon—call,beckon,gathersuperficial—shallow,surfacesuperfluous—extra,surplus(morethanisneeded)

supply—provide,give,contributesupport—maintain,encourage,sustain(tokeepgoing)suppress—prevent,repress(toholddown),stifle(toholdback),constrain(tolimit)surge—rush,flow,pour,gushsurplus—extra,spare,leftoversurreptitious—secret,sneaky,stealthy(sneaky),covert(hidden)surrogate—substitute,replacement,stand-insustain—support,maintain,keepgoing

Ttame—domestic,friendly,docile(obedient)tardy—late,slow,delayedtedious—boring,dull,dreary,monotonous(boring)temperate—moderate,pleasant,mildtemperamental—unpredictable,moody,volatile(explosive)tenacious—stubborn,persistent,determinedtentative—cautious,hesitant,uncertaintenuous—weak,flimsy,fragiletense—worried,anxious,stressed,uptightterminate—end,finish,conclude,ceaseterse—abrupt,brief,concise(tothepoint),brusque(abrupt)therapeutic—healing,beneficial,helpfultolerant—broadminded,understanding,forbearing(patient)toxic—poisonous,deadly,lethal(deadly)tranquil—peaceful,calm,relaxing,serene(peaceful)transgression—wrongdoing,disobedience,offensetransient—temporary,brief,fleeting,short-livedtranslucent—clear,transparent,see-throughtreacherous—unsafe,dangerous,hazardous,perilous(dangerous)trepidation—fear,anxiety,apprehension(fear)trivial—minor,insignificant,petty,negligible(notimportant)truncate—shorten,abbreviate,trimturbulent—chaotic,confused,tumultuous(inturmoil)

Uunheralded—unannounced,unpredicted,unexpecteduniform—unchanging,unvarying,standardized,homogeneous(thesame)

unpalatable—unpleasant,distasteful,disagreeableunparalleled—matchless,unequaled,incomparable,supremeunstinting—generous,givinguntenable—indefensible,unreasonableutilitarian—useful,practical,functionalutopian—perfect,ideal

Vvague—unclear,hazy,indistinct(notclear)valid—legitimate,reasonable,sensiblevalidate—confirm,approve,certify,authorizevalid—authentic,legal,legitimate,officialverdant—green,lush,luxuriantversatile—adaptable,resourceful,multitalentedvex—annoy,pester,irritate,exasperate(frustrate)virtuoso—expert,master,ace,whizviscous—thick,sticky,glueyvitality—energy,liveliness,durabilityvital—essential,fundamental,crucialvivacious—lively,cheerful,spiritedvivid—bright,vibrant,colorfulvolatile—explosive,unpredictable,unstablevoluntary—unpaid,honorary,probono(donewithoutpay)vulnerable—unprotected,indanger,atrisk

Wwane—diminish,decline,fadewaver—hesitate,fluctuate,vacillate(tobeindecisive)whimsical—fanciful,quirky,eccentric(unusual)wordy—rambling,long-winded,verbose(fullofwords)

Yyouthful—young,vigorous,vital

Zzealous—eager,passionate,fervent(fullofpassion)zeal—enthusiasm,passion,eagerness

AppendixB:Parents’GuidetoPrivateSchools

Onthefollowingpagesyouwillfindarticlesonchoosingprivateschools,understandingadmissionapplicationforms,andpayingforprivateschools.Thearticlesarewrittenbyteachers,administrators,deans,andothereducationalprofessionals.

WhyChooseanIndependentSchool?

Dr.JohnE.ChubbPresidentoftheNationalAssociationofIndependentSchools(NAIS)

Morethanhalfamillionfamilieseachyearchooseindependentprivateschools.Eachindependentschoolhasauniquemissionthatguidesitsofferings.Atanindependentschool,youcanchooseaschoolwherethephilosophy,values,andteachingapproacharerightforyourchild.

Independentschoolsnurturenotjuststudents’intellectualabilityandcuriositybutalsotheirpersonalandsocialgrowthandcivicconscience.Opportunitiesextendwellbeyondtheclassroomtoathleticcompetitions,artisticpursuits,andschoolleadershipexperiences.

Ifyou’reinterestedinpersonalattentionforyourchild,independentprivateschoolshavelowstudent-teacherratios,whichhelpteacherstogettoknowstudentswell.MorethantwiceasmanystudentsattheschoolsbelongingtotheNationalAssociationofIndependentSchools(NAIS)reportedthattheyreceivedhelpfulfeedbackfromteachersonassignmentscomparedtostudentsatallothertypesofschools,accordingtoTheHighSchoolSurveyofStudentEngagement(http://ceep.indiana.edu/hssse/index.html).Inaddition,studentswhoattendedNAISschoolsweremorethantwiceaslikelyasstudentsatotherschoolstosaythattheschoolcontributed“verymuch”totheirgrowthincriticalthinking.

TheFreshmanSurveyTrendsReport,anannualstudyconductedbytheHigherEducationResearchInstitute,foundthatgraduatesofNAISschoolsfeltmorepreparedfortheacademicdemandsofcollegethantheirpublicschoolcounterparts.Ascollegefreshmen,NAISgraduatesreportedthattheyweremorelikelytoaskquestionsinclassandexploretopicsontheirown,eventhoughitwasnotrequiredforaclass.

Independentschoolsofferdynamiccommunitiesinwhichparentsareinvestedintheirchildren’seducationandparticipateactivelyinthelifeoftheschool.Theseschoolspromoteregularcommunicationamongstudents,parents,andteacherstoensureeveryoneisworkingtowardthesamegoalsforthestudent.

Visitthehttp://parents.nais.orgtodaytolearnmoreaboutindependentprivateschools.

AnotherOption:IndependentDaySchools

LilaLohr

Forthoseofuswhoarefortunateenoughtobeabletosendourchildrentoanindependentdayschool,itseemstoofferthebestofbothworlds.Ourchildrenareabletoreaptheenormousbenefitsofanindependentschooleducationandwe,asparents,areabletocontinuetoplayavital,dailyroleintheeducationofourchildren.Parentsenjoybeingseenaspartnerswithdayschoolsineducatingtheirchildren.

Asmoreandmoreindependentdayschoolshavesprungupincommunitiesacrossthecountry,moreandmoreparentsarechoosingtosendtheirchildrentothem,evenwhenitmightinvolvealengthydailycommute.Contrarytosomeoldstereotypes,parentsofindependentschoolstudentsarenotallcutfromthesamemold,livinginthesameneighborhoodwithidenticaldreamsandaspirationsfortheirchildren.Independentschoolparentsrepresentawiderangeofinterests,attitudes,andparentingstyles.

Theyalsohaveseveralthingsincommon.Mostparentssendtheirchildrentoindependentdayschoolsbecausetheythinktheirchildrenwillgetabettereducationinasafe,value-ladenenvironment.Manyparentsarewillingtopaysubstantialannualtuitionbecausetheybelievetheirchildrenwillbeheldtocertainstandards,challengedacademically,andthoroughlypreparedforcollege.

Thiswillingnesstomakewhatare,formany,substantialfinancialsacrificesreflectstherecognitionthatmuchofone’scharacterisformedinschool.Concernedparentswanttheirchildrentogotoschoolswherevaluesarediscussedandreinforced.Theyseekschoolsthathaveclearexpectationsandlimits.Thenonpublicstatusallowsindependentschoolstoestablishspecificstandardsofbehaviorandperformanceandtosuspendorexpelstudentswhodon’tconformtothoseexpectations.

Understandingthepowerofadolescentpeerpressure,parentsareeagertohavetheirchildrengotoschoolwithotherteenswhoareacademicallyambitiousandrequiredtobehave.Theyseekanenvironmentwhereitis“cool”tobesmart,toworkhard,andtobeinvolvedintheschoolcommunity.Inindependentdayschools,studentsspendtheireveningsdoinghomework,expecttobecalledoninclass,andparticipateinsportsorclubs.

Successfulindependentschools,whetherelementaryorhighschool,largeorsmall,single-sexorcoed,recognizetheimportanceofaschool-parentpartnershipineducatingeachchild.Experiencedfacultymembersandadministratorsreadilyacknowledgethat,whiletheyareexpertsoneducation,parentsaretheexpertsontheirownchildren.Gonearethedayswhen

parentssimplydroppedtheirchildrenoffinthemorning,pickedthemupattheendoftheday,andassumedtheschoolwoulddotheeducating.Clearly,childrenbenefitenormouslywhentheirparentsandteachersworktogether,sharingtheirobservationsandconcernsopenlyandfrequently.

Independentschoolsencouragethistwo-waygive-and-takeandarecommittedtotakingitwellbeyondthepublicschoolmodel.Annualback-to-schoolnightsareattendedbymorethan90percentofparents.Teacher-parentandstudent-teacher-parentconferences,extensivewrittencommentsaspartofthereportcards,andadvisersystemsthatencourageclosefaculty-studentrelationshipsareallstructuresthatfacilitatethisparent-schoolpartnership.Althoughmoreandmoreindependentschoolparentsworkfull-time,theymaketimeforthesecriticalopportunitiestositdownanddiscusstheirchildren’sprogress.

Mostindependentschoolswelcomeandencourageparentalinvolvementandsupport.Althoughtheindividualstructuresvaryfromschooltoschool,mostincludeopportunitiesbeyondmakingcookiesandchaperoningdances.Manyparentsenjoybeinginvolvedincommunityserviceprojects,workingonschoolfundraisers,participatinginadmissionactivities,sharingtheirexpertiseinappropriateacademicclasses,andevenofferingstudentinternships.Mostschoolshavemadeaconcertedefforttostructurespecificopportunitiesforworkingparentstoparticipateinthelifeoftheschool.

Independentdayschoolsrecognizethebenefitsofparentvolunteersandofextendingthemselvessothatparentsfeelthattheyareanimportantpartoftheschoolfamily.Buddysystemsthatpairnewparentswithfamilieswhohavebeenattheschoolforseveralyearshelpeasethetransitionforfamilieswhoarenewtotheindependentschoolsector.

Independentschoolshavealsorespondedtoincreasedparentalinterestinprogramsfocusingonparentingskills.Recognizingtheinherentdifficultiesofraisingchildren,independentdayschoolshaveprovidedforumsfordiscussingandlearningaboutdrugs,depression,stressmanagement,peerpressure,andthelike.Bookgroups,paneldiscussions,andworkshopsprovideimportantopportunitiesforparentstosharetheirconcernsandtogettoknowtheparentsoftheirchildren’sclassmates.Schoolsrecognizethatthisparent-to-parentcommunicationandnetworkingstrengthenstheentireschoolcommunity.

Manycurrentdayschoolparentswouldcontendthatwhenyouchooseanindependentdayschoolforyourchildyouarereallychoosingaschoolfortheentirefamily.Thestudentsbecomesoinvolvedintheiracademicandextracurricularactivitiesandtheparentsspendsomuchtimeatschoolsupportingthoseactivitiesthatitdoesbecometheentirefamily’sschool.

LilaLohrisaformerHeadofSchoolatPrincetonDaySchoolinPrinceton,NewJersey,andtheFriendsSchoolofBaltimoreinBaltimore,Maryland.Shehasbeenateacherandanadministratorinindependentdayschoolsformorethanthirtyyearsandisthemotherof3independentdayschoolgraduates.

WhyBoardingSchool:ReasonsIChoseThisPathandWhyYouMight,Too

JustinMuchnick

Myseventh-gradeliteratureclasschangedmylife.Tobemoreexact,Mr.Rogers,myseventh-gradeliteratureteacher,changedmylife.Onlyatemporarysubstituteteacher(theregularfacultymembertooktheyearoffformaternityleave),Mr.Rogersenteredtheclassroomonthefirstdayofschoolanddidsomethingremarkable:heaskedhisstudentstohaveaconversationaboutabookwehadreadoverthesummer.Myschool’straditional“raiseyourhandandwaittobecalledon”styleoflearninghadleftmecompletelyunpreparedforateacherwhowantedmetotalk.Nevertheless,Irelishedtheopportunitytolearninthisinterestinganddynamicclassroomenvironment,andIcertainlyjumpedatachancetovoicemyopinionswithoutfearofbeingchastisedforspeakingoutofturn.

Withinafewweeks,activeparticipationhadfullycementeditsreputationastheprimarywayoflearninginMr.Rogers’class.Insteadofemployingastandardrow-and-columnclassroomseatingpattern,Mr.Rogerspositionedourseatsinalargecircle.This“Harkness”methodencouragedfree-flowingconversationsinwhichMr.Rogerswouldserveonlyasamediatorandparticipant.Ratherthanwritingblandresponsestoevenblanderstudyquestions,myclasshoneditspublicspeakingskillsbyparticipatinginspur-of-the-momentdebatesaboutthesetopics.Mr.Rogers’classroom,reminiscentofasceneoutofDeadPoetsSociety,allowedtheactoflearningtointellectuallystretchandstimulateme.

AsMr.Rogersbecamemyfriendandmentoraswellasmyteacher,IlearnedthathehadpreviouslyworkedastheWriterinResidenceatPhillipsExeterAcademy,aboardingschoolinNewEngland.Atthattime,Ithoughtthatboardingschoolmeantaplacewheredisobedientchildrenweresenttoresolvetheirbehavioralissues.Mr.Rogers,though,spokeglowinglyoftheEastCoastboardingschoolsystemandtoldmethatschoolslikeExeterutilizeanactive,discussion-based,Mr.Rogers-esquestyleofteaching.Soon,IstarteddreamingofaschoolfullofteacherslikeMr.Rogers,fullofintellectuallycuriousstudents,andfullofeagerlearners.IdidsomeresearchaboutExeterandotherNewEnglandschools,andinthespringofmyseventh-gradeyear,Ipresentedtheboardingschoolideatomymomanddad.

Atfirst,myparentscommendedmeontheexcellentjokeIhadpulledonthem;however,theyquicklyrealizedthatIwasserious.TheybegantothinkthatIwantedtogetawayfromthemorthatIdislikedmyfamily.Butafterextensivenegotiations,Iwasabletoconvincethemthatthiswasnotthecase,andbymidwaythroughthesummerafterseventhgrade,theyfullybackedmydecision.OncemyfamilyagreedthatIwasinfactgoingtopursuetheboardingschoolidea,we

madeitourmissiontoselecttheschoolstowhichIwouldapply.IdevisedalistofqualitiesIwaslookingforinaboardingschool.Atthispointintheapplicationprocess,you,too,shouldcreatealistofcriteria—youcansaveyourselfthewastedtimeandeffortofapplyingtothe“wrong”schoolsbyknowingwhatkindsofschoolsbestsuityou.Thoughyourpersonallistmaybeverydifferent,minewasasfollows:

Size:Itriedtofindarelativelybigschool.Atmyprimaryschool,theaveragegradesizewasabout50students,and,astimeworeon,mydesiretoexpandbeyondmysmallgroupofclassmatesgrewstronger.Byapplyingtolargerschools,IfeltthatIcouldbothbroadenmysocialexperienceandavoidanother“smallschoolburnout.”

UniformPolicy:Myformerschoolenforcedastrictdresscode.SincecollaredshirtsandIneverreallyhitifoff,Ihaddefinitelywornauniformforlongenough.Thus,Ididnotwanttospendmyhighschoolyearswearingablazerandslacks.

Single-SexorCoed:Ididn’treallywanttospendmyhighschoolyearsatanall-boysschool.

Location:Forme,theEastCoastseemedlikethebestplacetofindaboardingschool.Afterall,that’swhatMr.Rogershadrecommended.

Academics:Ihaveanunquenchablethirstforknowledge.Ididnotwishto“dehydrate”myselfataschoolwithaless-than-excellentacademicreputation.

ProximitytoanAirport:Mymomthoughtofthisone.ShesuggestedthatIshouldapplytoschoolslocatednearamajorairportthatofferednonstopflightsfromLosAngeles.Cross-countryflightsaretoughenough;complicatingmatterswithaconnectingflightseemedunnecessary.

Herearesomeotherthingsthatyoumightwanttoconsider:

ReligiousAffiliation:Doyouwantreligiontoplayalargepartinyourhighschoolexperience,orwouldyourathergotoanondenominationalschool?

SpecialtySchools:Doyouwanttoapplytoschoolsthatfocusonaspecificaspectormethodoflearning?Areyouparticularlytalentedinacertainfield?Ifso,lookintoarts,mathandsciences,ormilitaryschools.

Athletics:Ifyouplayasport,youmighttrytolookforschoolswithstrongteamsandexceptionalathleticfacilities.Onegoodwaytodothisistocontactaschool’scoachorathleticdirector.Asbothasoccerplayerandawrestler,Italkedtomanycoachesfromanumberofdifferentboardingschoolstogetasenseofeachschool’sathleticprogram.

Cost/FinancialAidPolicy:Obviously,someschoolsaremoreexpensivethanothers.Inadditiontolookingatthetuition,youmaywanttofindoutwhichschoolsofferneed-basedfinancialaidoranymerit-basedscholarships.

Bycomparingvariousschoolstomypersonallistofattributes,Iwasabletofindfourschools

thatreallymatchedmyrequirements.IsentapplicationstoChoateRosemaryHall,TheLawrencevilleSchool,PhillipsExeterAcademy,andPhillipsAcademyAndover,andIwasfortunateenoughtohavebeenacceptedbyallfourschools.AftertakingtheSSAT®,writingapplications,andinterviewingwithadmissionofficers,littledidIrealizethatIwouldstillhaveonelast,equallysignificanthurdletojump.ThechoicethatIwasabouttomakewoulddirectlyimpactthenextfouryearsofmylife,somyparentsandIdideverythinginourpowertoensurethatmydecisionwasthecorrectone.Byobtainingcontactinformationfromadmissionofficesaswellasschoolcounselorsandfriendsoffriends,wesoughtoutcurrentstudentswholivedlocallyandattendedeachoftheschools.Wescheduledface-to-facemeetingswithasmanyofthemaswecould,duringwhichwe“grilled”themontheprosandconsoftheirschools.Wespenthoursreadingwebsites,blogs,andFacebookpagesinhopesofgettingstudents’unsolicitedperspectivesoftheirschools.Mostimportantly,however,weattendedtherevisitdaysforeachoftheschools.Forthoseatthisstageoftheboardingschoolprocess,Iwouldhighlyrecommendgoingtotheadmittedstudentseventsifyouhavethefinancialmeansandyourschedulepermits.Atanygivenrevisitday,IwastrulyabletogetafeelforthecampusandenvironmentandseeifIcouldenvisionmyselfasastudentatthatschoolnextyear—sometimesmygutinstinctwouldtellme“yes,”andothertimesitwouldtellme“no.”Whileoncampus,Ialsohadtheopportunitytoaskcountlesscurrentstudentsabouttheirhighschoolexperiences.Don’tbeafraidtoasktoughquestions.Bydoingso,youcanbetterunderstandthegeneralcampusvibe.Aftertherevisitdays,Iwasabletomakemydecisionwithconfidence.IwasgoingtoAndover!

YoumightchoosetoapplytoboardingschoolsfordifferentreasonsthanIdid.Notwocasesareexactlyalike,butmanyapplicantsfallintothesecategories.

ReadytoLeaveHome:Whetherit’sadesireforindependence,frictionwithinthehousehold,oranyotherreason,waitinguntilcollegetoliveonyourownisn’tthebestoptionforyou.

FamilyTradition:Asababy,youworeaboardingschoolbibaroundyourneck.Yourfavoriteshirtisoneemblazonedwithaparticularschool’semblem.Afewolderfamilymembershavepavedtheway,or,perhaps,generationsofrelativeshaveattended.Boardingschoolisinyourblood.

SearchingforDiversity:Youhavegrownupinahomogenouscommunity,oryouyearnfordifferenceinbothtraditionandmindset.Today’sboardingschoolsaffordyouinteractionwithpeersofallracial,financial,geographical,andreligiousbackgrounds—conducivetoamulticulturaleducationalexperience.

LoveofLearning:Isyournoseperpetuallystuckinabook?Doyoustayafterclasstodelvedeeperintoaconversationwithyourteacher?Isyourideaofafunweekendactivityreadinguponpoliticalaffairsorthelatestscientificbreakthrough?Doyoulovelearningforlearning’ssake?Ifso,boardingschoolisundoubtedlyworthexploring.

Forme,theprocessofapplyingtoandselectingboardingschoolswasfueledbymyinnatepassionforlearning:Isimplywantedtofindaplacewhereitwas“cooltobesmart.”WhenI

lookbackatthisinitialnotion,Irealizethatitwascertainlyidealisticandabitnaïve,butAndoverhascomeaboutascloseaspossible.Thoughbusyworkanduninterestingconversationsbogmedownfromtimetotime,theyaremorethanmadeupforbyengagingwritingprompts,stimulatingdiscussions,andinspirationalteachersthatwouldmakeevenMr.Rogersproud.

Tolearnmoreaboutboardingschools,lookforPeterson’sTheBoardingSchoolSurvivalGuide,byJustinMuchnick,availableinstores,online,andasanebook.

JustinMuchnickisastudentatPhillipsAcademyAndoverinMassachusetts.Heiscaptainofthevarsitywrestlingteam,playssoccer,participatesinthechessclub,andworksasacampustourguide.Apassionatelearner,Justinenjoysreading,writing,Latin,andmodernAmericanhistory.Heco-authoredStraight-AStudySkills(AdamsMedia,January2013).HeistheyoungestjournalistforTheBootleg®(http://stanford.scout.com),StanfordUniversity’ssportsnewswebsite,wherehewritesarticlesaboutcollegefootball.Whenheisnotatboardingschool,JustinresidesinNewportBeach,California,withhisparentsandthreeyoungersiblings.

WhyaTherapeuticorSpecialNeedsSchool?

DiederikvanRenesse

Familiescontactmewhenasonordaughterisexperiencingincreaseddifficultiesinschoolorhasshownarealchangeinattitudeathome.Uponfurtherdiscussion,parentsoftensharethefactthattheyhavespokenwiththeirchild’steachersandhaveheldmeetingstoestablishsupportsystemsintheschoolandathome.Evaluations,medications,therapists,andmotivationalcounselingarebutafewofthemultipleapproachesthatparentsandeducatorstake—yetinsomecases,thedownwardspiralcontinues.Anxietybuildsinthestudentandfamilymembers;schoolavoidanceandincreasedfamilyturmoilreachapointwherethesituationisintolerable,andalternativesmustbeexplored—beitaspecialneedsschool,atherapeuticschool,oracombinationofboth.

Butshouldthatschoolbeadayorresidentialschool,andhowdoparentsdecidewhichwillbestmeettheirchild’sneeds?ResourcessuchasPeterson’sPrivateSecondarySchoolsguide,theInternet,guidance/schoolcounselors,andtherapistsarevaluable;however,thesubtlenuancesinvolvedindeterminingtheenvironmentthatwillbestservethechildaredifficulttoascertain.Somefamiliesseekthehelpofanindependenteducationconsultanttoidentifythemostappropriatesetting.Manyindependenteducationconsultantsspecializeinworkingwithchildrenwhohavespecialneedssuchaslearningdifferences,anxietydisorders,emotionalissues,ADHD,opposition,defiance,schoolphobia,drugoralcoholabuse,AspergerSyndrome,autism,andmore.Consultantshavefrequentcontactwiththeschools,andtheyworkcloselywithparentsduringtheenrollmentprocess.

Giventhebroadspectrumofneedspresentedbyindividualstudents,manyparentsquestionwhetherthereisindeedadayschoolthatcanmeettheneedsoftheirchild.Theansweroftendependsonlocation,spaceavailability,willingnesstorelocate,andappropriatenessoftheoptions.WhiletherearemanydayschooloptionsthroughouttheUnitedStates,thereareevenmoreresidentialorboardingoptions.Clearlythedecisiontohaveyourchildattendaresidentialschoolisnotmadeeasily.Asafamilyyoumayfeelasthoughyoudonothaveachoice—butyoushouldundertakeathoroughassessmentofallthedayoptionsandhowtheymightmeetthemajorityofyourchild’sneeds.

Whentheprimaryconcernsarelearningdifferences,manylocaloptions(thoughoftensmallandissue-specific)areavailabletofamilies.LocalcounselorsareoftenvaluableresourcesasarelocalchaptersofnationalLDorganizations.Ifyoucomeupwithavarietyofoptions,carefullycomparethembyvisitingtheschoolsandmeetingwiththespecialistsateachschool—thoseindividualswhowillworkdirectlywithyourchild.

Withthedayoptions,itisimportanttokeepthefollowingfactorsinmind:programandstaff

credentials,transportationtimetoandfromtheschool,availabilityofadditionalresources(supportservices)inoroutsidetheschoolsetting,sportsandextracurricularofferings,facilitiesandaccessibility,andyourchild’spotentialpeergroup.Youwillalsoneedtoassessmanyofthesefactorswhenconsideringresidentialschools,althoughmostresidentialschoolsaremoreself-containedthandayschools.Alsosignificantiswhethertheschoolhasbeenapprovedbyandacceptsfundingfromitsstateand/orschooldistrict.

Forfamilieswhocannotavailthemselvesoflocaldayoptionsorwhosechildisbestservedinaresidentialsetting,anevengreaterspectrumofoptionsisavailable.Theserangefromtraditionalboardingschoolswithbuilt-inacademicsupportservicestotherapeuticboardingschools,wildernessoroutdoortherapeuticprograms,emotionalgrowthorbehaviormodificationschools,transitionalorindependentlivingprograms,andevenresidentialtreatmentcenters,hospitals,orotherhealthfacilities.

Giventhebreadthoftheresidentialschoolsorprograms,mostfamiliesarebestservedbyateamthatincludesnotonlytheparents(andattimesthestudent),butalsotheprofessionalswhohavetaught,counseled,andworkedcloselywiththechild.Together,theteamcanidentifythespecificneeds,deficits,orbehavioralissuesthatmustbeaddressed,andtheycanworktogethertomatchthosewiththeappropriateschools.Aswithdayschools,youshouldarrangetovisitthefacilitiessothatyouarewell-informedabouteachoptionandwillbecomfortablewithyourfinaldecision.Thesevisitsarenotonlyopportunitiesforyoutomeetthestaffandstudents,butalsoforyouandyourchildtobeginarelationshipthatwillcontinuewhenyourchildisenrolled.

Thereisnoquestionthatseekingalternativeoptions,whethertheyarespecialneedsortherapeutic,isadauntingtask.However,withthehelpofexpertresourcesandreliableprofessionals,therightschoolcanmakeasignificantandlastingimpactonyourchild’shealthandwell-being.

DiederikvanRenesseisaSeniorPartneratSteinbrecher&PartnersEducationalConsultingServicesinWestport,Connecticut.Aformerteacher,admissiondirector,andprivateschoolcounselor,henowspecializesinhelpingfamiliesthroughouttheUnitedStatesandabroadwithyoungsterswhorequirespecialneedsoralternativeschoolsorwhoneedinterventionsandtherapeuticsettings.

FindingthePerfectMatch

HeleneReynolds

Oneoftherealbenefitsofindependenteducationisthatitallowsyoutodeliberatelyseekoutandchooseaschoolcommunityforyourchild.Ifyouarelikemostparents,youwantyourchild’sschoolyearstoreflectanappropriatebalanceofacademicchallenge,socialdevelopment,andexplorationintoathleticsandthearts.Youhopethatthroughexposuretonewideasandsoundmentoringyourchildwilldevelopanawarenessofindividualsocialresponsibility,aswellasthestudyskillsandworkethictomakeacontributiontohisorherworld.Itiseveryparent’sfondestwishtohavetheschoolexperiencesparkthoseareasofcompetencethatcanbepursuedtowardexcellenceanddistinction.

Anincreasingnumberofparentsrealizethatthisidealeducationisfoundoutsidetheirpublicschoolsystem,thatshrinkingbudgets,divisiveschoolboards,andovercrowdedclassroomshaveresultedinschoolswhereotheragendasviewitheducationforattentionandmoney.Inthisenvironmentthereislesstimeandenergyforteacherstofocusonindividualneeds.

Thedecisiontochooseaprivateschoolcanbemadeforasmanydifferentreasonsastherearefamiliesmakingthechoice.Perhapsyourchildwouldbenefitfromsmallerclassesoracceleratedinstruction.Perhapsyourchildhasneedsorabilitiesthatcanbemoreappropriatelyaddressedinaspecializedenvironment.Perhapsyouareconcernedabouttheacademicqualityofyourlocalpublicschoolandtheimpactitmayhaveonyourchild’sacademicfuture.Orperhapsyoufeelthataprivateschooleducationisagiftyoucangiveyourchildtoguidehimorhertowardamoresuccessfulfuture.

Everychildisanindividual,andthismakesschoolchoiceaprocessuniquetoeachfamily.Thefactthatyourfatherattendedatop-flightEasternboardingschooltopreparefortheIvyLeaguedoesnotnecessarilymakethiseducationalcoursesuitableforallofhisgrandchildren.Inadditiontodeterminingtheschool’soverallquality,youmustexploretheappropriatenessofphilosophy,curriculum,levelofacademicdifficulty,andstylebeforemakingyourselection.Therightschoolistheschoolwhereyourchildwillthrive,andafamousnameandahallowedreputationarenotnecessarilythefactorsthatdefinetherightenvironment.Thechallengeisindiscoveringwhatthefactorsarethatmakethematchbetweenyourchildandhisorherschooltherightone.

Nomatterhowgooditsqualityandreputation,asingleschoolisunlikelytobeabletomeettheneedsofallchildren.Thequestionremains:Howdofamiliesbegintheirsearchwithconfidencesotheywillfindwhattheyarelookingfor?Howdotheymaketherightconnection?

Asaparent,thereareanumberofstepsyoucanfollowtoestablishareasonedandobjectivecourseofinformationgatheringthatwillleadtoasubjectivediscussionofthisinformationand

thewayitappliestothestudentinquestion.Thiscanonlyoccurifthefirststepisdonethoroughlyandinanorderlymanner.Ultimately,targetingasmallgroupofschools,anyofwhichcouldbeanexcellentchoice,isonlypossibleafterinformationgatheringanddiscussionhavetakenplace.Withworkandalittleluck,theresultofthisprocessisaschoolwithanacademicallysoundandchallengingprogrambasedonaneducationalphilosophythatisanextensionofthefamily’sviewsandwhichwillprovideanemotionallyandsociallysupportivemilieuforthechild.

STEP1:IDENTIFYSTUDENTNEEDSOftenthedecisiontochangeschoolsseemstocomeoutoftheblue,but,inretrospect,itcanbeseenasadecisionthefamilyhasbeenleadinguptoforsometime.Iwouldurgeparentstodecideontheirowngoalsforthesearchfirstandtomakesure,ifpossible,thattheycanworkinconcerttowardmeetingthesegoalsbeforeintroducingtheideatotheirchild.Thesegoalsareasdifferentastheparentswhoholdthem.Foroneparent,findingaschoolwithastate-of-the-artcomputerprogramisahighpriority.Foranother,findingaschoolwithafulldanceandmusicprogramisimportant.OtherswillbemostconcernedaboutfindingaschoolthathasthebestrecordofcollegeacceptancesandhighestSATorACTscores.

Onceyouhavedecidedyourowngoalsforthesearch,bringthechildintothediscussion.Ioftensaytoparentsthatthedecisiontoexploreisnotthedecisiontochangeschoolsbutonlythedecisiontogatherinformationandconsideroptions.Itisimportanttobeawarethateveryonehasanindividualstyleofdecisionmakingandthatthedecisiontomakeachangeisloadedwithconcerns,manyofwhichwillnotbediscovereduntiltheprocesshasbegun.

Ifyouhavealreadymadethedecisiontochangeyourchild’sschool,itisimportanttoletyourchildknowthatthisaspectofthedecisionisopentodiscussionbutnottonegotiation.Itisequallyimportantthatyouletyourchildknowthatheorshewillhaveresponsibilityinchoosingthespecificschool.Withoutthatknowledge,yoursonordaughtermayfeelthatheorshehasnocontroloverthecourseofhisorherownlife.

Somestudentsareresponsibleenoughtotaketheleadintheexploration;somearetooyoungtodoso.Butinallcases,childrenneedreassuranceabouttheirfutureandclarityaboutthereasonsforconsideringotherschoolsettings.Sometimesthesituationisfraughtwithdisparateopinionsthatcanturnschoolchoiceintoafamilybattleground,oneinwhichthechildistheultimatecasualty.Itisalwaysimportanttokeepinmindthatthewelfareofthechildistheprimarygoal.

Theknowledgethateachindividualhashisorherownagendaandwayofmakingdecisionsshouldbewarningenoughtopursuesomepreliminarydiscussionsothatyou,asparents,canavoidthepitfallofconflictinggoalsandmaintainaunitedfrontandareasonablydirectedcourseofaction.Thefamilydiscussionshouldbeenergetic,anddifferencesofopinionshouldbeencouragedashealthyandnecessaryandexpressedinaclimateoftrustandrespect.

Therearemanyreasonswhyyoumay,atthispoint,decidetoinvolveaprofessionaleducational

consultant.Oftenthischoiceismadetoprovideaneutralgroundwhereyouandyourchildcanbothspeakandbeheard.Anotherreasonistomakesurethatyouhaveestablishedasoundcourseofexplorationthattakesbothyourownandyourchild’sneedsintoconsideration.Consultantswhoareup-to-dateonschoolinformation,whohavevisitedeachcampus,andwhoarefamiliarwiththesituationsoftheirclientscanaddimmeasurablytotheprocess.Theycanprovidearealitycheck,reinforcementofpersonalimpressions,andexperience-basedinformationsupportforpeoplewhoaredoingasearchofthistypeforthefirsttime.Alltheresearchintheworldcannotreplacetheexperienceandindustryknowledgeofaseasonedprofessional.Inaddition,ifthespecificcircumstancesoftheplacementaredelicate,theeducationalconsultantisinapositiontoadvocateforyourchildduringtheplacementprocess.Therearealsosituationsinwhichafamilyincrisisdoesn’thavethetimeortheabilitytoapproachschoolchoiceinadeliberateandobjectivemanner.

Thesearesomeofthemanyreasonstoengagetheservicesofaconsultant,butitisthefamilyguidanceaspectthatmostfamiliesoverlookatthestartoftheprocessandvaluemosthighlyaftertheyhavecompletedit.Agoodconsultantprovidesneutralgroundandinformationbackupthatareinvaluable.

STEP2:EVALUATEYOURCHILD’SACADEMICPROFILEIfyourchild’sacademicprofileraisesquestionsabouthisorherability,learningstyle,oremotionalprofile,getaprofessionalevaluationtomakesurethatyourexpectationsforyourchildarecongruentwiththechild’sactualabilitiesandneeds.

Startgatheringinformationaboutyourchildfromthecurrentschool.Askguidancecounselorsandteachersfortheirobservations,andrequestaformalmeetingtoreviewthestandardizedtestingthatvirtuallyeveryschooladministers.Questiontheirviewsofyourchild’sbehavior,attentiveness,andareasofstrengthandweakness.Makesureyoufullyunderstandthereasonsbehindtheirrecommendations.Donotfeelshyaboutcallingbacktoaskquestionsatalaterdate,afteryouhavehadtimetothinkandconsiderthisimportantinformation.Yourchild’sfuturemaydependonthedecisionsyouaremaking;don’thesitatetokeepaskinguntilyouhavetheinformationyouneed.

Ifapictureofconcernemerges,asktheguidancecounselor,otherparents,oryourpediatricianforsuggestionsregardinglearningspecialistsorpsychologistsinthecommunitywhoworkwithchildrenandcanprovideanevaluationoftheiracademicability,academicachievement,andlearningstyle.Theevaluationshouldbereviewedin-depthwiththespecialist,whoshouldbeaskedaboutspecificrecommendationsforchangesintheyoungster’sschooling.

Remember,astheparent,itisultimatelyyourresponsibilitytoweightheideasofothersandtodecideifthedifficultylieswithyourchildortheenvironment,eitherofwhichcouldindicateaneedforachangeofschool.

STEP3:REVIEWTHEGOALSOFPLACEMENTDiscussyourdifferencesofopinionaboutmakingachange.Identifyalistofschoolsthatcreatesaballparkofeducationalpossibilities.(Aneducationalconsultantcanalsobehelpfulatthisstage.)

Itisimportantthatbothyouandyourchildtakethetimetoconsiderwhatcharacteristics,largeandsmall,youwouldlikeinthenewschoolandwhichyouwouldliketoavoid.Asyoueachmakelistsofprioritiesanddiscussthem,theprocessofschoolchoiceentersthesubjectivearena.Theimpersonaldescriptionsofschoolenvironmentstransformintoverypersonalvisualizationsofthewaysyouandyourchildviewthechildinanewsetting.

Achancetoplayicehockey,aseriesofcoursesinMandarinChinese,theopportunitytotakeprivateflutelessons,oradesiretomeetothersfromallovertheworldmaysoundlikeabizarremixofcriteria,butthedesiretoexploreandfindalloftheseoptionsinasingleenvironmentexpressestheexpansivenessofthestudent’smindandtheareasheorshewantstoperfect,tryout,orexplore.Don’texpectperfectlylogicalthinkingfromyourchildasheorsheconsidersoptions;don’ttakeeverythingheorshesaysliterallyortooseriously.Openandrespectfuldiscussionwillallowachildtoembraceanewpossibilityonedayandrejectitthenext—thisispartoftheprocessofdecisionmakingandaffirmationandpartofthefunofexploration.

STEP4:SETANITINERARYSetanitineraryforvisitsandinterviewssothatyouandyourchildcancomparecampusesandtestyourpreconceivedideasoftheschoolsyouhaveresearchedagainsttherealityofthecampuscommunity;forwardstandardizedtestingscoresandtranscriptstotheschoolspriortovisitssothattheadmissionofficehaspertinentinformationinadvanceofyourmeeting.

Inordertoallowyourchildthefreedomtoformopinionsabouttheschoolsyouvisit,youmaywanttokeepthesepointersinmind:

•Parentsshouldallowtheirchildtobefrontandcenterduringthevisitsandinterviews—allowyourchildtoanswerquestions,eveniftheyleaveoutdetailsyouthinkareimportant.

•Parentsshouldstayinthebackgroundandhaveconfidencethattheadmissionofficersknowhowtoengagekidsinconversation.

•Thismaybethefirsttimeyourchildhasbeentreatedbyaschoolasanindividualandresponsibleperson—enjoywatchinghimorheradjusttothisasanobserver,notasaprotectororparticipant.

•Don’tletyourownanxietyruinyourchild’sexperience.•Discussdressinadvancesoitdoesn’tbecometheissueandfocusofthetrip.

Keepyourideasandimpressionstoyourselfandallowyourchildfirstshotatverbalizing

opinions.Rememberthatimmediatereactionsarenotfinaldecisions;oftenthefirstresponseisonlyanattempttoprocesstheexperience.

STEP5:USETHEAPPLICATIONPROCESSFORPERSONALGUIDANCEMakesureyourchildusestheapplicationprocessnotonlytosatisfytheschool’sneedforinformationbutalsotocontinuethepersonalguidanceprocessofworkingthroughandtrulyunderstandinghisorhergoalsandexpectations.

Applicationquestionsdemandyourchild’spersonalinsightandexploration.Addressingquestionsaboutsignificantexperiences,peoplewhohaveinfluencedhisorherlife,orselectingfourwordsthatbestdescribehimorherarewaysofcomingtogripswithwhoyourchildisandwhatheorshewantstoaccomplishbothatthenewschoolandinlife.Althoughparentswanttheirchildrentocompleteseamlessandperfectapplications,itisimportanttorememberthattheapplicationmustbetheworkofthechildandthattheparenthasanexcellentopportunitytodiscussthequestionsandanswerstohelpguidethestudentinapositiveandobjectiveself-review.

Itismoreimportantthattheapplicationessaysaccuratelyreflectthepersonalityandvaluesofthestudentthanthattheybetechnicallyflawless.Sincetheschoolisbasingpartofitsacceptancedecisiononthecontentsoftheapplication,theschoolneedstomeettherealstudentintheapplication.Thechild’sowndeterminationofwhatitisimportantfortheschooltoknowaboutthemiscrucialtothisprocess.Thatbeingsaid,parentscanplayanimportantroleinhelpingthechildunderstandthedifferencebetweenunnecessarilybrutalhonestyandputtinghisorherbestfootforward.

STEP6:TRUSTYOUROBSERVATIONSAlthoughtheprocessofschoolexplorationdependsonobjectivity,itisrarethatafamilywillembraceaschoolsolelybecauseofitscomputerlabs,endowment,library,SATorACTscores,orfootballteam.Theseobjectivecriteriaframethesearch,butittendstobetheintangiblesthatdeterminethedecision.Itisthesubjective—instinctiveresponsestoeventsoncampus,peoplemet,qualityofinterview,unfathomablevibes—thatmakesthematch.

Itisimportanttoreviewwhataspectsoftheschoolenvironmentmadeyoufeelathome.Thesequestionsapplyequallytoparentandchild.Didyoulikethepeopleyoumetoncampus?Wasthetourinformationalbutinformal,withstudentsstoppingtogreetyouorthetourguide?Wasthetoneofthecampus(austereorhomey,modernortraditional)consistentwiththekindofeducationalatmosphereyouarelookingfor?Arethesportsfacilitiesbeyondyourwildestexpectation?Doesthecollege-sendingrecordgiveyouconfidencethatyourchildwillfindanintellectuallycomfortablepeergroup?Howlongdotheteacherstendtostaywiththeschool,

anddotheysendtheirownchildrenthere?Ifitisaboardingschool,doteachersliveoncampus?Howhomeyisthedormsetup?

Themostfundamentalquestionsare:Dopeopleintheschoolcommunitylikewheretheyare,trusteachother,haverespectforeachother,andfeelcomfortablethere?Isitafamilyyouwouldcaretojoin?Thesesubjectiveresponseswillhelpyourecognizewhichschoolswillmakeyourchildfeelheorsheispartofthecommunity,whereheorshewillfitinandberespectedforwhoheorsheisandwantstobecome.

HeleneReynoldsisaformereducationalconsultantfromPrinceton,NewJersey.

PlanaSuccessfulSchoolSearch

Applicationdeadlines,entranceexams,interviews,andacceptanceorrejectionletters—thesearesomeofthechallengesyoucanexpecttoencounterwhenapplyingtoprivateschools.Theschoolsearchmayseemdaunting,butitdoesn’thavetobe.Herearesometipstohelpgetyouonyourway.

Thefirststepistogatherinformation,preferablyinthespringbeforeyouplanonapplying.Peterson’sPrivateSecondarySchools,withvitalstatisticsonmorethan1,000leadingprivateschoolsintheUnitedStatesandabroad,canhelpyouevaluateschools,clarifyyourchoices,andhoneyoursearch.

Ifyou’reconsideringboardingschools,youmayalsowanttoobtainafreecopyoftheBoardingSchoolsDirectoryfromTheAssociationofBoardingSchools(TABS)bycalling828-258-5354orbygoingonlinetowww.tabs.org/Directory_Request.pdf.

VISITINGSCHOOLSThenextstepistostartalistofschoolsthatpiqueyouroryourchild’sinterest.You’llwanttocall,fax,e-mail,orwritetoadmissionofficesforcatalogsandapplications.Atthisstage,don’tletcostruleoutchoices.You’lllearnmoreabouttheschoollater—thefinancingresourcesitmakesavailabletostudentsanditspoliciesofawardingaid.

Withschoolbrochuresandcatalogsinhand,startplanningfallvisitsandinterviews.Reviewyourschoolcalendar,notingSaturdays,holidays,andvacations.Trytoplaninterviewsforthesedaysoff.Eachinterviewcouldlastabout3hours,ascampustoursandotheractivitiesareoftenincluded.

Onceyouhavedeterminedwhichschoolsyouwanttosee,wheretheyare,andinwhatorderyouwanttoseethem,calleachschooltosettheinterviewdateandtime.

Keepinmindthatthereisno‘‘magicnumber’’ofschoolstosee.Somestudentsinterviewatandapplytoonlyoneschool,feelingthatiftheyarenotaccepted,theywillstayattheircurrentschool.Somestudentsinterviewatmany,thinkingthatconsideringalargenumberandavarietyofschoolswillhelpthemfocusonrealneedsanddesires.

Afteryou’vemadeanappointmenttovisittheschool,rereadtheschool’scatalogand,ifpossible,itsdescriptioninthisguide,andcheckoutitswebsitesothatfactsabouttheschoolarefreshinyourmindwhenyouvisit.

THEAPPLICATIONPROCESS

Oncethefact-findingiscompleted,yourchildwillneedtoworkonapplications.MostschoolshaveJanuaryorFebruarydeadlines,soitpaystobeginfillingoutformsinNovember.

Applicationsmayaskforallorsomeofthefollowing:schoolrecords,referencesfromteachers,astudentstatement,awritingsampleoressay,anapplicationfee,andmedicalhistoryform.

Ifyouareworkingwithahard-copyform,makephotocopiesofallapplicationpagesbeforeyourchildbeginstocompletethem.Thatway,heorshewillhaveatleastonecopyforuseasaroughdraft.Alsomakecopiesofeachcompletedapplicationforyourrecords.

Referencesareusuallywrittenonspecificschoolformsandareconsideredconfidential.Toensureconfidentiality,peopleprovidingreferencesmailtheircommentsdirectlytotheschool.Aschoolmayrequirefourorfivereferences—threeacademicreferences,usuallyfromanEnglishteacher,amathteacher,andoneotherteacher,andoneortworeferencesfromotherevaluatorswhoknowyourchild’sstrengthsinareasotherthanacademics.Askthesepeopleinadvanceiftheywillwriteonyourchild’sbehalf.Givereference-writersappropriateformswithanyspecialinstructionsandstampedenvelopesaddressedtotheschool;besuretoprovideasmuchleadtimebeforethedeadlineaspossible.

Thestudentapplicationiscompletedonaspecialformandconsistsoffactualfamilyinformation,aswellassomelongorshortessayquestions.Astemptingasitmaybetohelp,letyourchilddothewriting.Theschoolsneedtoseethestudent’sstyle,mechanicalskills,andthewayheorshelooksatlifeandeducation.SomeschoolsrequireacorrectedwritingsamplefromanEnglishassignment.Inthiscase,haveyourchildaskhisorherEnglishteachertohelpchoosehisorherbestwork.

Foradditionalinformationonapplications,includingthecommonapplicationforms,checkoutthenextarticle,“UnderstandingtheAdmissionApplicationForm.”

Oncetheapplicationsaremailedorsubmittedonline,thehardpartisdone.Askadmissionofficerswhenyoucanexpecttoheartheirdecisions.MostschoolswillletyouknowinearlyMarch.Whileyouwait,youmaywanttoremindyoursonordaughterthatbeingturneddownbyaschoolisnotastatementabouthisorherworth.Schoolshavemanydifferentobjectivesinputtingaclasstogether.Andthat’salessonthatwillcomeinhandywhenyoufacethecollegeapplicationprocess.

UnderstandingtheAdmissionApplicationForm

GreggW.M.MalobertiFormerDeanofAdmissionTheLawrencevilleSchoolLawrenceville,NewJersey

Studentsapplyingtoindependentschoolsarepresentedwithamyriadofoptionswhenitcomestimetochoosethemethodofcompletingtheapplicationprocess.Whereonceeachschoolissuedandrequireditsownpaperapplication,manyschoolsnowacceptcommonapplicationssuchastheSecondarySchoolsApplicationfromSSAT(SecondarySchoolAdmissionTest),theAdmissionApplicationFormfromTABS(TheAssociationofBoardingSchools),orvariousotheronlineapplicationformssponsoredbyindividualschoolsandplacementprograms.Withsomanyoptions,manyapplicantsandparentsareperplexedastowhichmethodtoemploy,andothersworrythatthechoiceofonemethodoveranothermayhaveanegativeeffectontheirchancesofadmission.Understandingmoreaboutwhythesechangescameaboutandhowtheysaveapplicantsandschoolstimeandmoneymayhelpapplicantsandtheirparentsmakeaninformedchoiceaboutwhichmethodtouse.

Therecentdevelopmentsandinnovationsinindependentschoolapplicationsmirrorthechangesthathaveoccurredatthecollegelevel.TheCollegeBoard’sCommonApplicationisacceptedatover300collegesandisavailableonline.TheInternethasacceleratedtheinterestinonlineapplications.Atthesametime,studentsaremuchmoreaccustomedtowritingonacomputerthantheyoncewerewithpenandpaper.Concernsaboutthefinancialandenvironmentalcostsofapaper-basedapplicationthattravelsfromtheprintertotheschool,tothecandidate,tothecandidate’sschool,andbacktotheadmissionofficebymailorcouriercontributetotheideathatthetimeofanonlinecommonlyacceptedapplicationhascome.

TheStandardApplicationOnline(SAO)isavailableontheSSATwebsiteathttp://www.ssat.org/admission/the-sao.TheBoardingSchoolsAdmissionApplicationFormisavailableintheTABSBoardingSchoolsDirectoryandinelectronicformfromtheTABSwebsite:http://www.boardingschools.com/how-to-apply/application.aspx.

ThereareafewschoolsthatacceptonlytherecommendationformsfromtheAdmissionApplicationForm.It’sbesttocheckwitheachschooltofindoutwhichformsarepreferred.ThelistofschoolsacceptingtheSecondarySchoolsApplicationfromSSATisavailableatthisSSATwebsite:http://www.ssat.org/member-search.

COMMONAPPLICATIONSMAKESENSE

Anxiousparents’lingeringdoubtsabouttheuseofoneofthecommonapplicationformsarehardtoignore:Willthesubstitutionofthecommonapplicationfortheindividualschool’sapplicationcausetheadmissioncommitteetobeoffendedandcompromisemychild’schancesforadmission?Parentsshouldrestassuredthatschoolsagreeingtoacceptthecommonapplicationformsbelievethatafairandeffectiveadmissiondecisioncanbemadeonthebasisofthecommonformandthatitsuseinnowayerodesthequalityoftheirselectionprocess.

HOWDOESTHECOMMONAPPLICATIONDIFFER?Allapplicationsbeginwithabiographicalsketchofthecandidate:name,address,birthdate,mailingaddress,parents’names,andschoolsattended.Informationregardingsiblingorlegacyrelationships,interestinfinancialaid,citizenship,languagespoken,andevenracialandethnicdiversityiscollectedaswell.Exceptfortheorderinwhichthesequestionsappear,thereislittlevariationinthesequestiontypesfromoneschool’sapplicationtoanother.Thecommonapplicationformscertainlyrelievecandidatesoftheburdenofprovidingtheverysamebiographicalinformationoverandoveragain.

Thesecondsectionofanapplicationgenerallyrevealsacandidate’saccomplishmentsandambitions.Often,theapplicantsareaskedtocatalogtheirinterestsandactivitiesinlistornarrativeform.Schoolswanttoknowwhatthecandidatehasdone,forhowlong,withwhom,andtowhatdistinction,ifany.Inafewcases,someschoolsaskforaseriesofshortanswerstoacombinationofquestionsorlookfortheapplicanttocompleteasentence.Therearegenerallyno“right”answerstothesequestions—buthonestanswerscanhelptheschoolbegintocharacterizetheapplicant’scuriosity,maturity,ambition,andself-esteem.Hereagain,greatsimilarityexistsinthemannerandstylewithwhichthisinformationisgathered.Whilethecommonapplicationformsaskthesequestiontypesinamoredirectmanner,theyarenolesseffectivethantheindividualschool’sapplication,andtheiruseaffordsacandidateagenuinemeasureofefficiencywithoutcompromisingindividuality.

Schoolsthatadvocatetheuseoftheirownapplicationsoverthatofthecommonapplicationformsoftenbitterlydefendthethirdandfinalportionoftheirapplicationssinceitgenerallyincludesessayquestions.Withfewexceptions,thesequestions,whileoccasionallyposedinauniqueororiginalmanner,seektoprobemuchthesameterritorycoveredbythethreechoiceslistedintheessaysectionofthecommonapplicationforms:

Describeapersonyouadmireorwhohasinfluencedyouagreatdeal.Whatmakesyoutheinterestingpersonthatyouare?Explaintheimpactofaneventoractivitythathascreatedachangeinyourlifeorinyourwayofthinking.

Manyschoolsthatusethecommonapplicationsrequireasupplementthataffordsanopportunityforcandidatestoprovideinformationthatisnotrequestedbythecommonapplications.

Whilethecandidate’sabilitytowritewelliscertainlyunderreviewintheessayquestion,theexerciseinvestigatesacandidate’svaluesandexplorestheindividualexperiencesthathaveshapedhisorhercharacter.Thesequestionsgivecandidatesachancetorevealsuchqualitiesasindependence,self-reliance,creativity,originality,humility,generosity,curiosity,andgenius.Viewedinthislight,answeringthesequestionsbecomesatallorder.Thebestadvicemaybetojustanswerthem.Inaddition,candidatesshouldrecognizethatalthoughthecontentoftheiressaysisalwaysofinterest,grammar,spelling,punctuation,organization,andtheinclusionofevidenceorexamplesareofequalimportance.

Candidateswhocomefromdisadvantagedbackgroundsoftenfindthissectionoftheapplicationthemostchallengingandoccasionallyexclusionary.Someschoolsassumethatallapplicantshaveaccesstoopportunitiessuchassummercamps,musicinstruction,andperiodicalsandnewspapers.Whateverthecase,thecommonapplicationformsattempttobemoreinclusiveofabroadersetofexperiences.Infact,manyoutreachagencieswhoseektoidentifyandplacedisadvantagedstudentsinindependentschoolshaveeitherusedoneoftheexistingcommonapplicationformsorhavedevelopedtheirownapplicationsinlieuofindividualschoolapplicationforms.

Ifastudentfearsthatusingoneofthecommonapplicationswillsomehowfailtoconveyauniqueaspectofhisorherindividualityorthattheessayquestionanswerswillnotspeaktotheuniquequalitiesofwhyaparticularschoolmightbeagoodmatch,heorshemaywanttothinkaboutincludinganextraessay.Justbecauseacandidateusesacommonapplicationdoesnotmeanthatheorshemustuseacommonapproachtocompletingit.Imaginehowwelcomeasplashofcreativitymightbetoanindividualreaderorcommitteeofadmissionofficerswhomayreadhundredsoreventhousandsofapplicationseachadmissionseason.Anapplicationthatparrotsthelistofschoolcourses,sports,andactivitiesofferslittleinsightintothecandidate.Awell-writtenapplicationwillbeasuniqueastheindividualwhowroteit.

Applicantsandtheirparentsarenottheonlywinnerswhenacommonapplicationformisused.Theteacherswhodutifullycompletecountlessrecommendationformsenjoytheconvenienceofhavingtocompleteonlyoneformforeachoftheirstudentsapplyingtoindependentschools.Practicallyspeaking,ifthereiseveratimethatastudentwantstobeingoodfavorwithhisorherteacher,itisthemomentatwhichareferenceisbeinggiven.Usingacommonapplicationmakestheprocessofapplyingtomultipleschoolsamuchmoremanageableendeavor.Whenthereisonlyoneformtocomplete,mostteacherswillprovidelongerandmoreinformativeanswersthatarefarmorehelpfultoadmissionofficers.Commonapplicationsareagreatremedyforthefatigueandfrustrationenduredbyteacherswhohavebeenoverwhelmedbyabarrageofrecommendationforms.Currently,thereareevenmoreschoolsacceptingcommonrecommendationformsthanthereareschoolsacceptingtheentireSecondarySchoolApplicationortheAdmissionApplicationForm.Beforediscountingthebenefitsofacommonapplication,besuretoconsideratleasttheuseoftherecommendationforms.

COUNSELORSANDCONSULTANTSSPEAKOUT

LeeCarey,DirectorofAdmissionsandSecondarySchoolCounselingatShoreCountryDaySchoolinBeverly,Massachusetts,hasbeenadvisingeighth-gradersformanyyearsandfindstheworkloadassociatedwiththeapplicationprocessunreasonableformostofherstudents.“Itisinconceivabletoexpecta14-year-oldstudenttowriteupwardsofeightindividualessays,alloftopquality.Fromtakingtimeforschoolvisits,makingupmissedschoolwork,organizingforms,completingpaperwork,andpolishingwriting,theactofapplyingtosecondaryschoolsbecomesawholesecondjobforeighth-andninth-gradestudents.”Consideringthattheaverageapplicationincludesuptotendocuments,someofwhichmustpassbetweentheapplicant,thesendingschool,andbacktotheapplicantorthereceivingschool,aneighthgraderandhisorherparentsarenowlookingatcompletingmorethaneightydocuments!Ontopofthetestingprocessandapplyingforfinancialaid,thisamountstoanenormousadministrativechallenge.

KarlKoenigsbauer,DirectorofSecondarySchoolPlacement,EaglebrookSchoolinDeerfield,Massachusetts,agreesthatthecommonapplicationformsmaketheprocessmoreefficient,butheworriesabouthowtheymighterodetheprocessaswell.“Mygoalistohelpstudentsfindtheschoolthatwillbethebestmatchfortheirabilitiesandinterests.Theessayquestionsfromsomeschoolsreallyhelpthecandidatetounderstandmoreaboutwhatqualitiesofmindandspiritaschoolvalues.Whenacandidatecomestomeandsaysaparticularquestionistoodifficult,toosimplistic,orjustplainconfusing,itgivesmeanopportunitytohelphimorherseehowthatquestionrepresentstheidentityofthatparticularschoolandwhyitmayormaynotbeagoodmatch.IworrythatthecommonapplicationformswillhomogenizetheapplicationprocesstothepointwhereIlosethisopportunitytofine-tunetheplacementprocess.”

FaithHowland,anindependenteducationalconsultantinBoston,Massachusetts,andamemberoftheIndependentEducationalConsultantsAssociation(IECA),workswithfamiliestofindtherightschoolandisalsooftencontactedforhelpwhenastudent’sfirstroundofapplicationshasnotbeensuccessful.“Theapplicationprocesscanbenearoverwhelmingfor13-and14-year-olds.Towriteasmanyaseightdifferentapplications,eachwithdifferentessays,justwhenyouareexpectedtogetgreatgradesandcontinueyoursportscommitmentsandotherextracur-ricularactivities—nottomentionworkingtoprepareforentrancetests.Thisishighstress!Useofacommonapplicationformwouldbesupportivetostudentsandwouldbeextremelyhelpfulinstreamliningtheteacherrecommendations.Forthosekidswhoneedtosubmitasecondroundofapplications,thecommonapplicationformscouldbeinvaluable.Theseyoungstersarecopingwithdisappointmentwhileneedingtoresearchnewpossibilities.Ifschoolswerewillingtosharethecommonapplicationforms,it’sconceivablethatmanymorestudentswhomightsimplygiveupifnotsuccessfulontheirfirstapplicationscouldbeplaced.”

MANYSCHOOLS,ONEAPPLICATIONIncreasedacceptanceoftheSecondarySchoolsApplicationandtheAdmissionApplicationFormcouldleadtoamarkedincreaseinapplications.Commonapplicationsareespecially

helpfultothecandidatewhofailstoearnanyacceptancelettersattheendoftheapplicationprocess.Traditionally,ifacandidatewantstoapplytoanewlistofschools,heorshemuststartfromscratchandcompleteanewsetofforms.Commonapplicationscertainlyspeedupthisprocess,andinthecaseoftheSecondarySchoolApplicationfromSSAT,sendinganapplicationtoanadditionalschoolisaseasyassendingthetestscores.Candidatessimplysignintotheiraccountsandselectanotherschool.

Morethanhalfofthecandidateswhoapplytoindependentschoolscomefrompublicschoolsandmaynotenjoythebenefitofplacementcounselorsattheirschoolsnordotheyseektheadviceofindependentcounselors.Regardless,mostcandidatesarewellservedinusingoneofthecommonapplicationformswhenapplyingtomultipleschools.Onestrategymaybetocompleteafewindividualapplicationsandthensubmitoneofthecommonapplicationformstoafewotherschools—identifyingsomeadditionaloptionsandincreasingthelikelihoodofhavingmeaningfulchoicesafterthedecisionlettersaremailed.Manycandidatesfinditmucheasiertofigureoutwhichschooltheywantoncetheyknowwhichschoolwantsthem.

Fewschoolsrealizehowdifficulttheapplicationprocesscanbeforfamilieswhoareapplyingtomorethanoneschool.Commonapplicationformsmaketheprocessofapplyingtomultipleschoolsamuchmoremanageableendeavor.Theuseofacommonapplicationformaffordsfamiliesmuchmoretimeandenergytodevotetootheraspectsoftheapplicationandinterviewprocess.Byreducingtheduplicatedpaperworkofrecommendationsandtheneedtocompletesomanyessays,applicantsandtheirparentsaregrantedagreateropportunitytodiscusstherealissuessurroundingschoolselection,suchasthecompatibilityofcurriculum,styleofteaching,andprogramofferings.Ratherthancreatingfoldersforeachschoolandchasingdownmultiplelettersofrecommendation,applicantsandtheirparentscanfocusonjustafewessaysandremovethestressassociatedwithsortingandtrackingmultipledocuments.

Candidatesandtheirfamiliescanbeassuredoftheprofessionalismofadmissionofficersandfeelfreetouseoneofthecommonapplications.TheSecondarySchoolApplicationandtheAdmissionApplicationFormrepresenttheeffortsoftheverybestadmissionofficerswhohaveputtheinterestsoftheapplicantatthefore—shiftingthefocusawayfromtheschoolandbacktothecandidate.Candidatescanbeconfidentthatthecommonapplicationformwillmorethanadequatelyallowthemtomakeastrongcasefortheirownadmissionatanyschoolacceptingtheform.

AboutAdmissionTestsandtheSSAT

HeatherHoerle

ExecutiveDirectorSecondarySchoolAdmissionTestBoard(SSATB)

Mentiontheword“testing”toeventhemostcapablestudent,andheorsheislikelytobecomeanxious.It’snowonder,then,thattestingintheindependentschooladmissionprocesscausesnail-bitingamongstudentsandparentsalike.

Itisimportanttoremember,though,thatresultsofadmissiontesting,whileintegraltoanapplication,arejustoneofmanyfactorsconsideredbyadmissionofficerswhendeterminingifyourchildandtheirschoolsmakeagreatmatch.Thedegreeofemphasisplacedonscoresdependsontheschoolandonotherinformation,suchasthetranscriptandteacherrecommendations.Forthevastmajorityofschools,studentswithawiderangeofSSATscoresareadmitted.

Themostimportantthingtorememberaboutadmissiontestsisthattheyaredifferentfromotherkindsoftests.Admissiontests,while“standardized,”aredifferentfromaptitudeandachievementtests.Aclassroommathtest,forexample,isanachievementtest:Theteacherspecificallydesignedittoevaluatehowmuchyouknowaboutwhathasbeencoveredinclass.TheSSAT,ontheotherhand,isdesignedtomeasuretheverbal,quantitative,andreadingskillsyouhaveacquiredovertime,insteadoffocusingonyourmasteryofparticularcoursematerials.TheSSATprovidesindependentschooladmissionprofessionalswithmeaningfulinformationaboutthepossibleacademicsuccessofpotentialstudentslikeyouattheirinstitutions,regardlessofstudents’backgroundorexperience.

Admissiontestsarealsodifferentfromclassroomandachievementtests,becausetheyare“norm-referenced.”Thismeansthatyourchild’sscoreisinterpretedrelativetothegroupofstudentstakingthetest(thenormgroup).Forexample,ifyouareaboyinthesixthgrade,andyourpercentilerankontheSSATverbalsectionis70percent,itmeansthat70percentofalltheothersixthgradeboys’(whohavetakenthetestforthefirsttimeononeoftheStandardSaturdayorSundaySSATadministrationsintheUSAandCanadabetween2008and2011)scoresfallbelowyourscalescore.Therefore,thesamescalescoreontheSSATmayhaveadifferentpercentilerankfromyeartoyear.Incontrast,yourpercentcorrectfromaclassroommathtestis90percentbecauseyouanswered90percentofthequestionscorrectly.Yourscoreisnotreferencedtotheperformanceofanyoneelseinyourclass.

Finally,admissiontestsareconstructedsothatonly50percentoftheexamineeswillgettheaveragetestquestioncorrect.Thisissothatthetestwilleffectivelydifferentiateamongtest-

takers,whovaryintheirlevelofskills.Therefore,“average”ontheSSAT(50percent)isdifferentthan“average”onyourchild’smathtest(whichmaybesomethingmorelike80percent).Manyparentsexpressconcernthattheirchild’sSSATPercentileislowerthantheytypicallyscoreonothertestssuchasstandardizedachievementtestsandschoolexams.ItisimportanttorememberthatSSATtest-takersaremembersofasmallandhighlycompetitivegroupofstudentswhoplantoattendsomeoftheworld’sbestprivate/independentschools.Beinginthemiddleofthisgroupisstillimpressive!

TAKINGTHESSATTherearethreelevelsoftheSSATadministered.TheUpperLevelisadministeredtostudentsingrades8–11.TheMiddleLevelisadministeredingrades5–7,andtheElementaryLevelisadministeredingrades3and4.TheMiddleandUpperLevelexamsarebothapproximately3hoursinlength,whiletheElementarytestisjustshortof2hours.Eachtesthasverbal,reading,andquantitativesectionsthatareconstructedinamultiple-choiceformat,alongwithanunscoredwritingsample.

ThetwomathematicssectionsoftheMiddleandUpperLevelSSATmeasuresastudent’sknowledgeofalgebra,geometry,andotherquantitativeconceptsandconsistsoftwo25-questionsections.TheElementaryLevelSSATtestsmeasuresknowledgeofelementaryarithmetic,algebra,andgeometryinone30-minute,30-questionsection.

Thereadingcomprehensionsectionofthetestmeasuresastudent’sabilitytounderstandwhatheorshehasread.Afterreadingapassage,thestudentwillbeaskedquestionsaboutitscontentorabouttheauthor’sstyle,intent,orpointofview.Thegoalsaretodiscernthemainideaofthepiece,identifytheimportantdetailsthatmovethenarrativealongorcreateamoodortone,ortoidentifythedetailsthatsupportthewriter’sopinion.TheUpperandMiddleleveltestscontain40questions,whiletheElementarytestconsistsof28readingcomprehensionquestions.

Theverbalsectionasksstudentstoidentifysynonymsandinterpretanalogies.Thesynonymquestionstestthestrengthofthestudent’svocabulary,whiletheanalogyquestionsmeasureastudent’sabilitytorelateideastoeachotherlogically.TheUpperandMiddleLevelSSATsconsistof60verbalquestions,whiletheElementarytesthasjust30verbalsectionquestions.

Inaddition,theexamcontainsanunscoredwritingsamplethatisnotscoredbutisprovidedtoschoolstoofferasenseofthestudent’swritingskills.Thewritingsamplesectionvariesbasedontheleveloftest.TheUpperandMiddleLevelSSATsofferachoiceoftwowritingprompts.IntheUpperLevelSSAT,onepromptisacreativepromptandtheotheranessayprompt.OntheMiddleLevelexam,botharecreativepromptchoices.TheElementaryLevelexamoffersapicture,andthestudentmustwriteastoryaboutwhatishappeninginthepicture.

TherearetwotypesofSSATtestadministrations:StandardandFlex.TheStandardadministrationsaregiveneightSaturdaysduringtheacademicyear(October,November,December,January,February,March,April,andJune)atvarioustestlocationsthroughoutthe

UnitedStates,Canada,andotherlocationsaroundtheworld.Studentsalsohavetheoptionof“ondemand”Flextestingthroughselecteducationalconsultantsorplacementdirectorsatsomeindependentschools.StudentscantakeuptoalleightoftheStandardTests,buttheycanonlytaketheFlextestonce.

Test-takerscanarrangetohaveSSATscoressenttodifferentschoolsandhavetheoptiontoselectschoolscorerecipientseitherduringtestregistrationorafterreceivingtheirscores.Studentscanresearchschools,registerforthetest,findatestcenter,orderscoreservices,andfeesonlineatssat.orgorbycalling609-683-4440.Inaddition,studentscanordertheOfficialGuidetotheSSATfortheMiddleandUpperLeveltests.TheOfficialGuidetotheSSATistheonlystudyguidewrittenbytheSSATtestdevelopmentteam.TheOfficialGuidetotheElementaryLevelSSATisavailableforfreedownloadonssat.org.

SSAT.orgalsooffersavarietyofsourcesontheindependentschoolapplicationprocess;italsooffersfamilywebinarsonthetest,understandingscorereports,andmore.

HOWISTHESSATSCORED?TheSSATisaformula-scoredtest,meaningstudentsearnonepointforeverycorrectanswer,zeropointsforomittedquestions,andtheylose¼pointforeachincorrectanswer.Eachstudent’sscorereportincludesthenumberofcorrectandincorrectanswers,aswellasthenumberofquestionsomitted.

Understandingformulascoringiscriticaltoastudent’stest-takingstrategy.WhentakingtheSSAT,ifastudentcannoteliminateoneormoreoftheanswerchoicesaswrong,itisbesttoskipthequestiontomaintainahigherscore.

HOWIMPORTANTARETHETESTS?TheSSATnormgroupisahighlycompetitivegroup.Yourchildisbeingcomparedtoalltheotherstudents(samegrade/gender)whoaretakingthistestforadmissionintoindependentschools—someofwhichcanbethemostselectiveschoolsinthecountry.Icanassureyou,though,thatin100percentofindependentschools,thetestisjustonepartoftheselectionprocess.Thisisvitalforstudentsandfamiliestorememberasitrelievesthe“test-takingpressure”we’vereadsomuchaboutinthemediaandheardfromfamilies,teachers,andtheeducationalcommunity.Admissionofficersaretaskedwithfindingtherightstudentsthatnotonlyfittheacademicpieceoftheirschool,butthecultureandcommunityfounduniquetoeachindependentschoolenvironment.TheSSATisnotdesignedtomeasureothercharacteristicssuchasmotivation,persistence,orcreativitythatastudentmaycontributetoinaparticularschoolcommunityenvironment.

Thatsaid,parentsshouldbepartnersinthetest-preparationprocess,bothasatutorandreassuringvoiceforthestudent.Thissamerelationshipshouldextendtopartneringwithyour

studenttosubmitanapplicationthathighlightsastudent’sschoolworkportfoliosandotheracademicachievements,recommendations,andotherelementsthatarejustascriticaltotheselectionprocess.Remember,admissionofficersmustfindapplicantsthataretherightacademicANDsocial/culturalfitfortheirschool.TheSSATcanonlyassistindeterminingoneelementofthisprocess.

HeatherHoerle’scareerinindependentschoolsbeganasanadministrator,studentadvisor,andteacheratGeorgeSchool(PA)andWesttownSchool(PA).Shethenembarkedonasuccessful23-yeartenureinleadershiproleswithtwooftheworld’slargestnonprofitindependentschoolorganizations:firstasAssociateDirectorofTheAssociationofBoardingSchools(TABS),thenasDirectorofAdmissionandMarketingServicesforTheNationalAssociationofIndependentSchools(NAIS),leadingtohermostrecentpositionasNAIS’VicePresidentofMemberRelations.Heatheremphasizes,“Ibelievepassionatelyinthepowerofeducation(andespeciallyindependentprivateschools)toopenmindsandcreatebetterhumanbeings.Ihaveseenthroughmyownexperiencesasastudent,admissionofficer,teacher,trustee,andparentinindependentschools,thatourcommunityregularlytransformschildrenhelpingthemtobecomethoughtfulcontributorsinthisinterconnected,globalsociety.”HeatherholdsaB.A.inArtHistoryfromMountHolyokeCollegeandaM.Ed.fromHarvardUniversity.SheisaBoardMemberoftheWesttownSchoolandtheFriendsCouncilonEducation.HeathercurrentlyresidesinHopewell,NewJersey,withherhusbandanddaughter,andisanavidfanofthetheater.

PayingforaPrivateEducation

MarkJ.MitchellVicePresident,SchoolInformationServicesNationalAssociationofIndependentSchools(NAIS)

Imaginebeingabletobuya$23,000carfor$12,000becausethatisallyoucanafford.Whenyoubuyacar,youknowthatyouwillbepayingmorethanitcosttodesign,build,ship,andsellthecar.Thesalesstaffwillnotofferyouapricebasedonyourincome.Atbest,youmayreceivediscounts,rebates,orotherincentivesthatallowyoutopaythelowestpricethedealeriswillingtoaccept.Nomatterhowyoulookatit,youpaymorethanthecarcosttomake.

Tuitionatmanyprivateschoolscanapproachthecostofanewcar,butpayingforaprivateschooleducationisnotthesameasbuyingacar.OnedifferenceistheavailabilityoffinancialaidatthousandsofschoolsintheUnitedStatesandabroad.Financialaidhelpsoffsetthecostoftuition.Learningaboutthefinancingoptionsandproceduresavailablecanmakeprivateschoolarealityformanyfamilies.

NEED-BASEDFINANCIALAIDManyprivateschoolsofferassistancetofamilieswhodemonstratefinancialneed.Infact,forarecentacademicyear,schoolsthatbelongedtotheNationalAssociationofIndependentSchools(NAIS)providedmorethan$1.5trillioninneed-basedfinancialaidtonearly23percentoftheirstudents.For2012–13,themediangrantforboardingschoolstudentswas$22,053andthemediangrantfordayschoolstudentswas$10,574.Theseneed-basedgrantsdonotneedtoberepaidandareusedtooffsettheschool’stuition.Schoolsmakethissubstantialcommitmentasonewayofensuringasocioeconomicallydiversestudentbodyandtohelpensurethateverystudentqualifiedforadmissionhasthebestchancetoenroll,regardlessofhisorherfinancialcircumstances.

HOWFINANCIALNEEDISDETERMINEDManyschoolsuseaprocessofdeterminingfinancialneedthatrequiresthecompletionofapplicationsandthesubmissionoftaxformsandotherdocumentationtohelpthemdecidehowmuchhelpeachfamilyneeds.Currently,morethan2,100schoolsnationwideaskfamiliestocompleteTheSchoolandStudentServices(SSSbyNAIS)Parents’FinancialStatement(PFS)onlineathttp://SSSbyNAIS.orgtodetermineeligibilityforaid.TheParents’FinancialStatementgathersinformationaboutfamilysize,incomeandexpenses,parents’assetsandindebtedness,andthechild’sassets.Fromthisandotherinformation,schoolsareprovidedwithanestimateof

theamountofdiscretionaryincome(afterseveralallowancesaremadeforbasicnecessities)availableforeducationcosts.Schoolsrevieweachcaseindividuallyandusethisestimate,alongwithsuchsupportingdocumentationasmostrecentincometaxforms,tomakeafinaldecisiononafamily’sneedforafinancialaidgrant.Formoreinformation,pleasevisithttp://sssbynais.org.

Theamountofaneed-basedfinancialaidawardvariesfrompersontopersonandschooltoschool.Justasindividualshavedifferentfinancialresourcesandobligationsthatdictatetheirneedforassistance,schoolshavedifferentresourcesandpoliciesthatdictatetheirabilitytomeetyourfinancialneed.Tuitioncosts,endowmentincomes,andtheschool’sphilosophyaboutfinancialaidareafewofthethingsthatcanaffecthowmuchaidaschoolcanoffer.Ifyourdecisiontosendyourchildtoaprivateschooldependsheavilyongettingfinancialhelp,youwouldbenefitfromapplyingforaidatmorethanoneschool.

MERIT-BASEDAWARDSWhilethemajorityofaidofferedisbasedonafamily’sfinancialsituation,noteveryonewhoreceivesfinancialassistancemustdemonstratefinancialneed.Privateschoolsoffermillionsofdollarsinmerit-basedscholarshipstothousandsofstudents.Inthe2012–13academicyear,313NAIS-memberschoolsgrantedanaverageannualmeritawardworth$6,180tostudents,totalingmorethan$55.4million.Evenwiththislevelofcommitment,suchawardsarerare(just5.6percentofallenrolledstudentsreceivethistypeofaid)and,therefore,highlycompetitive.Theymayservetorewarddemonstratedtalentsorachievementsinareasrangingfromacademicstoathleticstothearts.

Someadditionalresourcesmaybeavailablefromorganizationsandagenciesinyourcommunity.Civicandreligiousgroups,foundations,andevenyouremployermaysponsorscholarshipsforstudentsatprivateschools.Unfortunately,theseoptionstendtobefewandfarbetween,andlimitedinnumberandsizeofaward.Besuretoaskafinancialaidofficerattheschool(s)inwhichyouareinterestedifheorsheisawareofsuchorganizationsandopportunities.Whetheritisofferedbytheschooloralocalorganization,understandingtherequirementsorconditionsonwhichamerit-basedscholarshipisbasediscritical.Askiftheawardisrenewableand,ifso,underwhatconditions.Often,certaincriteriamustbemet(suchasminimumGPA,communityservice,orparticipationinactivities)toensurerenewaloftheawardinsubsequentyears,andsomemeritawardsareavailableforjustoneyear.

TUITIONFINANCINGOPTIONSWhetherornotyouqualifyforgrantsorscholarships,anotherwaytogetfinancialhelpinvolvesfindingwaystomaketuitionpaymentseasieronyourfamily’smonthlybudget.Onecommonoptionisthetuitionpaymentplan.Theseplansallowyoutospreadtuitionpayments(lessanyformsoffinancialaidyoureceive)overaperiodofeighttotenmonths.Inmostcases,paymentsstartbeforetheschoolyearbegins,butthismethodcanbemorefeasiblethancomingupwithone

ortwolumpsumpaymentsbeforethebeginningoftheschoolyear.Paymentplansmaybeadministeredbytheschoolsthemselvesorbyaprivatecompanyapprovedbytheschool.Theydonotnormallyrequirecreditchecksorchargeinterest;however,theytypicallychargeanapplicationorservicefee,whichmayincludetuitioninsurance.AdditionalinformationabouttuitionpaymentplansisavailableontheNAISwebsiteathttp://sssbynais.org.

Sinceahigh-qualityeducationisoneofthebestinvestmentstheycanmakeintheirchild’sfuture,manyparentsfinancethecostjustastheywouldanyotherimportantexpense.Anumberofschools,banks,andotheragenciesoffertuitionloanprogramsspecificallyforelementaryandsecondaryschoolexpenses.Whilesuchloansaresubjecttocreditchecksandmustberepaidwithinterest,theytendtoofferratesandtermsthataremorefavorablethanthoseofotherconsumerloans.Itpaystocomparethedetailsofmorethanonetypeofloanprogramtofindthebestoneforyourneeds.Althoughtheyshouldalwaysberegardedasanoptionoflastresort,tuitionloanprogramscanbehelpful.Ofcourse,everyfamilymustconsiderboththeshort-andlong-termcostsofborrowingandmakeitsdecisionpartofalargerplanforeducationfinancing.

AFINALWORDAlthoughtheprimaryresponsibilitytopayforschoolcostsrestswiththefamily,thereareoptionsavailableifyouneedhelp.Asyoucansee,financingaprivateschooleducationcanresultinapartnershipbetweenthefamily,theschool,andsometimesoutsideagenciesorcompanies,witheachmakinganefforttoprovidewaystomeetthecosts.Thefinancialaidofficerattheschoolisthebestsourceofinformationaboutyouroptionsandiswillingtohelpyouineverywayheorshecan.Alwaysgotothefinancialaidofficerataschoolinwhichyouareinterestedwheneveryouhaveanyquestionsorconcernsaboutprogramsortheapplicationprocess.Understandingyourresponsibilities,meetingdeadlines,andlearningaboutthefullrangeofoptionsareyourbeststrategiesforobtainingassistance.Althoughtherearenoguarantees,withproperplanningandbyaskingtherightquestions,yourfamilyjustmightgetthehigh-qualityprivateeducationforless.

AppendixC:PrivateSchoolsAt-a-Glance

InthefollowingpagesyouwillfindvaluabledataonprivatesecondaryschoolsfromPeterson’sPrivateSecondarySchools2014–15.ThedatawerecollectedthroughPeterson’sAnnualSurveyofPrivateSecondarySchoolsduringsummerandfall2013.Alsoincludedwereschoolsthatsubmittedinformationforthe2012–13datacollectioneffortbutdidnotsubmitupdatesinthesummer/fall2013.Withminorexceptions,dataforthoseschoolsthatrespondedtotheonlinequestionnaireweresubmittedbyofficialsattheschoolsthemselves.Allusableinformationreceivedintimeforpublicationhasbeenincluded.

Thechartonthefollowingpageswillguideyoursearch,whetherit’sfocusedonaspecificgeographicregionoftheUnitedStatesoraroundtheworld.We’veprovidedquickanswerstokeyquestionsabouteachschool,suchas:

•Areitsstudentsboarding,day,orboth?•Isitcoeducational?•Whatgradesareofferedattheschool?•Howmanystudentsareenrolled?•Whatisthestudent/facultyratio?

ThechartalsoprovidesinformationaboutAdvancedPlacementsubjectareasandsports.

Onceyou’veusedthefollowingcharttohelpyouidentifyprospectiveschools,besuretocheckoutPeterson’sPrivateSecondarySchools,theonlycomprehensiveprivateschoolguideavailable.You’llfinddetailedprofilesonmorethan1,000accreditedprivateschoolsworldwide,aswellasvaluableadviceonplanningyoursearchandfinancingaprivateschooleducation.

STUDENTSACCEPTED

PRIVATESCHOOLSAT-A-GLANCE Boarding Day GRADES STUDENT/FACULTY STUDENTOFFERINGS

Boys Girls Boys Girls Lower Middle Upper Total Upper

Student/FacultyRatio

AdvancedPlacementPreparation Sports

UNITEDSTATESAlabama

BriarwoodChristianHighSchool,Birmingham X X K-6 7-8 9-12 1,971 591 23:1 X 18

TheDonohoSchool,Anniston X X PS-6 7-8 9-12 344 120 12:1 X 11

EdgewoodAcademy,Elmore X X PK-5 6-8 9-12 314 100 11:1 X 12

TheEllisAcademy,Toney X X X K-5 6-10 11-PG 9 2 5:1 X 18

JohnT.MorganAcademy,Selma X X PK-6 7-9 10-12 500 122 22:1 28

MadisonAcademy,Madison X PS-6 7-12 900 15:1

X - 450 9

MarsHillBibleSchool,Florence X X K4-4 5-8 9-12 540 198 14:1 X 11

McGill-ToolenCatholicHighSchool,Mobile X X - - 9-12 1,170 1,170 14:1 X 14

TheWestminsterSchoolatOakMountain,Birmingham K-6 - 7-12 508 178 6

Alaska

GraceChristianSchool,Anchorage X X K-6 7-8 9-12 617 205 15:1 X 8

Arizona

BlueprintEducation,Glendale - -

BourgadeCatholicHighSchool,Phoenix X X - - 9-12 401 401 20:1 X 16

BrophyCollegePreparatory,Phoenix X - 6-8 9-12 1,362 1,284 14:1 X 44

TheOrmeSchool,Mayer X X X X - - 8-PG 110 110 6:1 X 40

PhoenixChristianUnifiedSchools,Phoenix X X PS-5 6-8 9-12 424 231 20:1 X 16

PuschRidgeChristianAcademy,Tucson X X - 6-8 9-12 962 481 16:1 X 12

SaintMary’sHighSchool,Phoenix X X - - 9-12 503 503 15:1 X 17

SalpointeCatholicHighSchool,Tucson X X - - 9-12 1,041 1,041 15:1 X 23

ScottsdaleChristianAcademy,Phoenix X X PK-5 6-8 9-12 851 295 15:1 X 15

SetonCatholicHighSchool,Chandler X X - - 9-12 580 580 12:1 X 22

SouthwesternAcademy,Rimrock X X X X - - 9-PG 17 17 3:1 53

Tri-CityChristianAcademy,Chandler X X K4-6 7-8 9-12 304 92 16:1 4

XavierCollegePreparatory,Phoenix X - - 9-12 1,173 1,173 22:1 X 30

Arkansas

EpiscopalCollegiateSchool,LittleRock X X PK-5 6-8 9-12 780 224 10:1 X 16

SubiacoAcademy,Subiaco X X - 7-8 9-12 182 154 9:1 X 46

California

AcademyofOurLadyofPeace,SanDiego X - - 9-12 725 725 14:1 X 11

AlmaHeightsChristianHighSchool,Pacifica X X K-4 5-8 9-12 303 147 10:1 X 9

ArchbishopMittyHighSchool,SanJose X X - - 9-12 1,725 1,725 17:1 X 27

ArmyandNavyAcademy,Carlsbad X X 7-9 - 10-12 302 224 15:1 X 28

TheAthenianSchool,Danville X X X X - 6-8 9-12 476 311 10:1 X 15

TheBaySchoolofSanFrancisco,SanFrancisco X X - - 9-12 324 324 8:1 18

BesantHillSchool,Ojai X X X X - - 9-12 100 100 4:1 X 44

BishopMontgomeryHighSchool,Torrance X X - - 9-12 938 938 22:1 X 17

BishopO’DowdHighSchool,Oakland X X - - 9-12 1,151 1,151 15:1 X 35

BrentwoodSchool,LosAngeles X X K-6 7-8 9-12 996 467 7:1 X 41

BridgesAcademy,StudioCity X X - 5-8 9-12 138 74 8:1 3

CampbellHall(Episcopal),NorthHollywood X X K-6 7-8 9-12 1,117 547 8:1 X 20

CapistranoValleyChristianSchools,SanJuanCapistrano X X JK-6 7-8 9-12 396 162 13:1 X 16

CarondeletHighSchool,Concord X - - 9-12 800 800 15:1 X 22

CastillejaSchool,PaloAlto X - 6-8 9-12 445 258 6:1 X 14

CentralCatholicHighSchool,Modesto X X - - 9-12 385 385 14:1 X 15

ChadwickSchool,PalosVerdesPeninsula X X K-6 7-8 9-12 831 352 5:1 X 20

ChaminadeCollegePreparatory,WestHills X X - 6-8 9-12 2,011 1,328 16:1 X 25

Children’sCreativeandPerformingArtsAcademyofSanDiego,SanDiego X X X X K-5 6-8 9-12 262 100 18:1 X 31

ChineseChristianSchools,Alameda X X X X - - 9-12 220 220 8:1 X 17

CrossroadsSchoolforArts&Sciences,SantaMonica X X K-5 6-8 9-12 1,159 510 11:1 21

CrystalSpringsUplandsSchool,Hillsborough X X - 6-8 9-12 350 250 9:1 24

DamienHighSchool,LaVerne X - - 9-12 923 923 18:1 X 27

DeLaSalleHighSchool,Concord X - - 9-12 1,042 1,042 28:1 X 19

DelphiAcademyofLosAngeles,LakeViewTerrace X X K-3 4-8 9-12 144 35 15:1 20

EldoradoEmersonPrivateSchool,Orange X X K-6 - 7-12 150 80 18:1 X 7

FaithChristianHighSchool,YubaCity X X - - 9-12 100 100 12:1 X 8

FlintridgePreparatorySchool,LaCanadaFlintridge X X - 7-8 9-12 500 400 8:1 X 17

FresnoChristianSchools,Fresno X X K-6 7-8 9-12 465 188 11:1 X 16

TheFrostigSchool,Pasadena X X 1-5 6-8 9-12 90 52 6:1 4

GraceBrethrenSchool,SimiValley X X K-6 7-8 9-12 830 300 11:1 X 17

TheGrauerSchool,Encinitas X X - 6-8 9-12 161 100 6:1 X 54

Harvard-WestlakeSchool,StudioCity X X - 7-8 9-12 1,597 1,157 8:1 X 21

Head-RoyceSchool,Oakland X X K-5 6-8 9-12 876 370 9:1 X 20

HebrewAcademy,HuntingtonBeach X X X N-5 6-8 9-12 247 36 4:1 X 14

HeritageChristianSchool,NorthHills X X - 7-8 9-12 794 643 22:1 X 11

HighlandHallWaldorfSchool,Northridge X X N-6 7-8 9-12 241 86 6:1 6

ImmaculateHeartHighSchoolandMiddleSchool,LosAngeles X 6-8 9-12 520 520 15:1 X

InternationalHighSchool,SanFrancisco X X PK-5 6-8 9-12 1,046 339 10:1 24

JuniperoSerraHighSchool,SanMateo X - - 9-12 860 860 27:1 X 29

KingsChristianSchool,Lemoore X X PK-6 7-8 9-12 294 88 11:1 X 17

LaJollaCountryDaySchool,LaJolla X X N-4 5-8 9-12 1,131 472 16:1 X 31

Lick-WilmerdingHighSchool,SanFrancisco X X - - 9-12 460 460 9:1 X 18

LinfieldChristianSchool,Temecula X X JK-5 6-8 9-12 757 320 12:1 X 13

LodiAcademy,Lodi X X - - 9-12 90 90 9:1 X 6

LouisvilleHighSchool,WoodlandHills X - - 9-12 430 430 25:1 X 15

LyceeInternationaldeLosAngeles,Burbank X X PK-5 6-8 9-12 1,004 115 9:1 X 13

MaranathaHighSchool,Pasadena X X - - 9-12 670 670 14:1 X 20

MarinAcademy,SanRafael X X - - 9-12 406 406 9:1 X 35

MarymountHighSchool,LosAngeles X - - 9-12 374 374 7:1 X 20

MercyHighSchoolCollegePreparatory,SanFrancisco X - - 9-12 391 391 14:1 X 11

MesaGrandeSeventh-DayAcademy,Calimesa X X K-6 7-8 9-12 265 119 10:1 9

MidlandSchool,LosOlivos X X - - 9-12 77 77 5:1 X 13

Mid-PeninsulaHighSchool,MenloPark X X - - 9-12 117 117 8:1 7

ModestoChristianSchool,Modesto X X K-5 6-8 9-12 580 280 10:1 X 13

NotreDameAcademy,LosAngeles X - - 9-12 380 380 12:1 X 10

NotreDameHighSchool,SanJose X - - 9-12 628 628 16:1 X 12

OakGroveSchool,Ojai X X X X PK-6 7-8 9-12 209 53 7:1 X 15

OrindaAcademy,Orinda X X - 7-8 9-12 90 81 9:1 X 4

PalmaSchool,Salinas X - 7-8 9-12 534 402 15:1 X 22

PaloAltoPreparatorySchool,MountainView X X - - 8-12 65 65 8:1 X

ParadiseAdventistAcademy,Paradise X X K-4 5-8 9-12 180 76 8:1 5

ProvidenceHighSchool,Burbank X X - - 9-12 419 419 9:1 X 17

RedwoodChristianSchools,CastroValley X X K-5 6-8 9-12 692 299 10:1 X 8

RollingHillsPreparatorySchool,SanPedro X X - 6-8 9-12 225 130 9:1 X 21

SacramentoCountryDaySchool,Sacramento X X PK-5 6-8 9-12 481 115 9:1 X 15

SageHillSchool,NewportCoast X X - - 9-12 479 479 10:1 X 13

St.AugustineHighSchool,SanDiego X - - 9-12 700 700 25:1 X 44

St.Catherine’sAcademy,Anaheim X X K-6 7-8 157 14:1 24

SaintElizabethHighSchool,Oakland X X - - 9-12 143 143 15:1 X 7

SaintFrancisGirlsHighSchool,Sacramento X - - 9-12 1,078 1,078 15:1 X 15

SaintFrancisHighSchool,LaCanadaFlintridge X - - 9-12 678 678 15:1 X 10

SaintFrancisHighSchool,MountainView X X - - 9-12 1,753 1,753 29:1 X 29

St.Michael’sPreparatorySchooloftheNorbertineFathers,Silverado X 9-12 67 67 3:1 X 11

SanDiegoJewishAcademy,SanDiego X X K-5 6-8 9-12 485 181 18:1 X 26

SanDomenicoSchool,SanAnselmo X X X PK-5 6-8 9-12 598 163 5:1 X 12

SantaCatalinaSchool,Monterey X X - - 9-12 259 259 8:1 X 31

SonomaAcademy,SantaRosa X X - - 9-12 268 268 12:1 X 28

SouthwesternAcademy,SanMarino X X X X - - 124 6:1 X 34

SquawValleyAcademy,OlympicValley X X X X - - 9-12 100 100 8:1 X 68

StevensonSchool,PebbleBeach X X X X PK-5 6-8 9-12 747 520 10:1 X 34

SummerfieldWaldorfSchool,SantaRosa X X K-6 7-8 9-12 409 106 7:1 4

TheThacherSchool,Ojai X X X X - - 9-12 252 252 6:1 X 46

ValleyChristianHighSchool,SanJose X X K-5 6-8 9-12 2,475 1,415 17:1 X 20

VictorValleyChristianSchool,Victorville X X K-6 7-8 9-12 300 120 15:1 X 10

ViewpointSchool,Calabasas X X K-5 6-8 9-12 1,212 505 10:1 X 33

TheWebbSchools,Claremont X X X X - - 9-12 405 405 8:1 X 19

WestridgeSchool,Pasadena X 4-6 7-8 9-12 482 264 6:1 X 19

WestviewSchool,LosAngeles X X - 5-8 9-12 134 101 8:1 4

Colorado

DenverAcademy,Denver X X 1-6 7-8 9-12 331 175 8:1 28

DenverChristianHighSchool,Denver X X - - 9-12 142 142 19:1 12

FountainValleySchoolofColorado,ColoradoSprings X X X X - - 9-12 244 244 6:1 X 42

FrontRangeChristianHighSchool,Littleton X X PK-6 7-8 9-12 435 167 8:1 X 25

HumanexAcademy,Englewood X X - 6-8 9-12 30 18 8:1 21

KentDenverSchool,Englewood X X - 6-8 9-12 692 471 7:1 X 28

TheLowellWhitemanSchool,SteamboatSprings X X X X - - 9-12 5,360 53 7:1 X 70

TellurideMountainSchool,Telluride X X PK-4 5-8 9-12 88 14 5:1 27

TempleGrandinSchool,Boulder X X - 6-8 8-12 24 14 3:1

STUDENTSACCEPTED

PRIVATESCHOOLSAT-A-GLANCE Boarding Day GRADES STUDENT/FACULTY STUDENTOFFERINGS

Boys Girls Boys Girls Lower Middle Upper Total Upper

Student/FacultyRatio

AdvancedPlacementPreparation Sports

Connecticut

BrunswickSchool,Greenwich X PK-4 5-8 9-12 939 357 5:1 X 20

ChaseCollegiateSchool,Waterbury X X PK-5 6-8 9-12 402 185 7:1 X 32

ConventoftheSacredHeart,Greenwich X X PS-4 5-8 9-12 737 295 7:1 X 22

EagleHillSchool,Greenwich X X X X 1-6 - 6-9 255 120 4:1 33

FairfieldCollegePreparatorySchool,Fairfield X - - 9-12 901 901 18:1 X 31

TheGlenholmeSchool,DevereuxConnecticut,Washington X X X X 5-6 7-8 9-PG 95 79 10:1 44

TheGunnery,Washington X X X X - - 9-PG 298 298 6:1 X 22

HolyCrossHighSchool,Waterbury X X - - 9-12 630 630 15:1 X 24

HopkinsSchool,NewHaven X X - 7-8 9-12 711 560 6:1 X 37

KingLowHeywoodThomas,Stamford X X PK-5 6-8 9-12 692 326 8:1 X 21

Kingswood-OxfordSchool,WestHartford X X - 6-8 9-12 511 366 8:1 X 19

LauraltonHall,Milford X - - 9-12 477 477 12:1 18

MissPorter’sSchool,Farmington X X - - 9-12 311 311 7:1 X 45

NorthwestCatholicHighSchool,WestHartford X X - - 9-12 601 601 12:1 X 36

TheNorwichFreeAcademy,Norwich X X - - 9-12 2,300 22:1 X 36

TheRectorySchool,Pomfret X X X X 1-4 5-9 251 4:1 38

RumseyHallSchool,WashingtonDepot X X X X K-5 6-9 333 221 8:1 48

St.JosephHighSchool,Trumbull X X - - 9-12 830 830 20:1 X 17

TheTaftSchool,Watertown X X X X - - 9-PG 579 579 5:1 X 46

WatkinsonSchool,Hartford X X - 6-8 9-PG 266 192 6:1 30

WellspringFoundation,Bethlehem X X X X 1-6 - 7-12 51

WestminsterSchool,Simsbury X X X X - - 9-PG 390 390 6:1 X 34

WestoverSchool,Middlebury X X - - 9-12 204 204 7:1 X 48

TheWoodhallSchool,Bethlehem X X - - 9-PG 43 43 4:1 X 26

Delaware

TowerHillSchool,Wilmington X X PS-4 5-8 9-12 747 252 6:1 X 23

WilmingtonChristianSchool,Hockessin X X PK-5 6-8 9-12 517 231 15:1 X 12

WilmingtonFriendsSchool,Wilmington X X PS-5 6-8 9-12 765 270 10:1 X 15

DistrictofColumbia

GeorgetownDaySchool,Washington X X PK-5 6-8 9-12 1,075 500 7:1 X 9

GeorgetownVisitationPreparatorySchool,Washington X - - 9-12 490 490 11:1 X 20

GonzagaCollegeHighSchool,Washington X - - 9-12 958 958 15:1 X 33

TheLabSchoolofWashington,Washington X X 1-6 7-8 9-12 356 125 8:1 10

MaretSchool,Washington X X K-4 5-8 9-12 635 305 6:1 X 23

NationalCathedralSchool,Washington X 4-6 7-8 9-12 581 315 7:1 X 43

St.AlbansSchool,Washington X X 4-8 - 9-12 593 323 7:1 X 35

WashingtonInternationalSchool,Washington X X PK-5 6-8 9-12 899 270 7:1 10

Florida

AcademyattheLakes,LandO’Lakes X X PK-4 5-8 9-12 428 141 5:1 X 17

AmericanHeritageSchool,DelrayBeach X X PK-5 6-8 9-12 1,130 609 10:1 X 26

AmericanHeritageSchool,Plantation X X PK-6 - 7-12 2,395 1,686 15:1 X 22

BelenJesuitPreparatorySchool,Miami X - 6-8 9-12 1,488 890 13:1 X 19

TheBenjaminSchool,NorthPalmBeach X X PK-5 6-8 9-12 1,081 420 8:1 X 22

BerkeleyPreparatorySchool,Tampa X X PK-5 6-8 9-12 1,290 565 9:1 X 28

BishopJohnJ.SnyderHighSchool,Jacksonville X X - - 9-12 472 472 15:1 X 17

TheBollesSchool,Jacksonville X X X X PK-5 6-8 9-12 1,670 787 11:1 X 18

CanterburySchool,FortMyers X X PK-6 7-8 9-12 580 199 10:1 X 13

TheCanterburySchoolofFlorida,St.Petersburg X X PK-4 5-8 9-12 450 175 7:1 X 35

CardinalMooneyCatholicHighSchool,Sarasota X X - - 9-12 474 474 13:1 X 20

CardinalNewmanHighSchool,WestPalmBeach X X - - 9-12 540 540 25:1 X 18

Chaminade-MadonnaCollegePreparatory,Hollywood X X - - 9-12 552 552 19:1 X 16

ClearwaterCentralCatholicHighSchool,Clearwater X X - - 13:1 X 19

TheCommunitySchoolofNaples,Naples X X PK-5 6-8 9-12 706 286 8:1 X 17

TheCrenshawSchool,Gotha X X PK-5 6-8 9-12 102 50 10:1

FatherLopezHighSchool,DaytonaBeach X X - - 9-12 438 438 13:1 X 18

TheFirstAcademy,Orlando X X K4-6 7-8 9-12 990 410 17:1 X 22

ForestLakeAcademy,Apopka X X X X - - 9-12 357 357 12:1 13

FoundationAcademy,WinterGarden X X K-5 6-8 9-12 585 175 5:1 X 14

GladesDaySchool,BelleGlade X X PK-6 7-8 9-12 307 123 15:1 X 10

GulliverPreparatorySchool,Miami X X PK-4 5-8 9-12 1,834 754 8:1 X 38

Immaculata-LaSalleHighSchool,Miami X X - - 9-12 804 804 15:1 X 17

JesuitHighSchoolofTampa,Tampa X - - 9-12 758 758 13:1 X 19

LakeMaryPreparatorySchool,LakeMary X X X X PK-5 6-8 9-12 640 275 22:1 X 21

MontverdeAcademy,Montverde X X X X PK-5 6-8 9-PG 1,010 558 12:1 X 37

TheNorthBrowardPreparatoryUpperSchool,CoconutCreek X X X X PK-5 6-8 9-12 1,375 696 18:1 X 28

Out-Of-Door-Academy,Sarasota X X PK-5 6-8 9-12 680 247 7:1 X 20

PensacolaCatholicHighSchool,Pensacola X X - - 9-12 611 611 18:1 X 9

RansomEvergladesSchool,Miami X X - 6-8 9-12 1,064 608 10:1 X 22

SaddlebrookPreparatorySchool,WesleyChapel X X X X 3-5 6-8 9-12 80 63 8:1 1

SaintEdward’sSchool,VeroBeach X X PK-5 6-8 9-12 500 225 8:1 X 20

St.JosephAcademy,St.Augustine X X - - 9-12 260 260 11:1 X 17

St.ThomasAquinasHighSchool,FortLauderdale X X - - 9-12 2,194 2,194 18:1 X 30

TampaPreparatorySchool,Tampa X X - 6-8 9-12 576 403 10:1 X 32

TrinityPreparatorySchool,WinterPark X X - 6-8 9-12 865 520 10:1 X 21

UniversitySchoolofNovaSoutheasternUniversity,FortLauderdale X X PK-5 6-8 9-12 1,845 720 18:1 X 19

TheVanguardSchool,LakeWales X X X X - 5-8 9-PG 109 99 7:1 26

WestminsterChristianSchool,PalmettoBay X X PK-5 6-8 9-12 1,068 473 15:1 X 13

WindermerePreparatorySchool,Windermere X X X PK-5 6-8 9-12 1,193 432 11:1 X 22

Georgia

AthensAcademy,Athens X X N-4 5-8 9-12 949 287 8:1 X 11

AugustaChristianSchool,Martinez X X K-5 6-8 9-12 634 275 10:1 X 13

AugustaPreparatoryDaySchool,Martinez X X PS-4 5-8 9-12 552 212 9:1 15

BlessedTrinityHighSchool,Roswell X X - - 9-12 965 965 14:1 X 17

EatonAcademy,Roswell X X K-5 6-8 9-12 80 40 5:1 34

FirstPresbyterianDaySchool,Macon X X PK-5 6-8 9-12 1,008 355 12:1 X 17

TheGallowaySchool,Atlanta X X P3-4 5-8 9-12 769 270 11:1 X 11

TheHeritageSchool,Newnan X X PK-4 5-8 9-12 428 170 8:1 X 28

TheHowardSchool,Atlanta X X K-5 6-8 9-12 256 80 8:1 6

LandmarkChristianSchool,Fairburn X X K4-5 6-8 9-12 862 274 8:1 X 16

TheLovettSchool,Atlanta X X K-5 6-8 9-12 1,649 629 8:1 X 45

MaristSchool,Atlanta X X - - 7-12 1,085 1,085 12:1 X 17

Mt.DeSalesAcademy,Macon X X - 6-8 9-12 641 422 9:1 19

MountParanChristianSchool,Kennesaw X X K-5 6-8 9-12 1,105 406 X 19

MountVernonPresbyterianSchool,Atlanta X X K-4 5-8 9-12 891 242 10:1 19

ThePaideiaSchool,Atlanta X X PK-6 7-8 9-12 987 420 10:1 X 23

PiedmontAcademy,Monticello X X 1-5 6-8 9-12 277 109 13:1 10

PinecrestAcademy,Cumming X X PK-5 6-8 9-12 790 275 9:1 X 15

RabunGap-NacoocheeSchool,RabunGap X X X X - 5-8 9-12 404 306 10:1 X 37

St.PiusXCatholicHighSchool,Atlanta X X - - 9-12 1,100 1,100 12:1 X 23

WoodwardAcademy,CollegePark X X PK-6 7-8 9-12 2,738 1,101 X 20

Guam

AcademyofOurLadyofGuam,Agana X - - 9-12 413 413 25:1 X 10

Hawaii

HanalaniSchools,Mililani X X PK-6 - 7-12 700 337 16:1 X 25

HawaiiBaptistAcademy,Honolulu X X K-6 7-8 9-12 1,106 472 17:1 X 22

Hawai’iPreparatoryAcademy,Kamuela X X X X K-5 6-8 9-12 620 397 13:1 X 24

Ho’AlaSchool,Wahiawa K-6 7-8 9-12 73 15 5:1 X 4

LutheranHighSchoolofHawaii,Honolulu X X - 7-8 9-12 55 51 7:1 X 23

Mid-PacificInstitute,Honolulu X X K-5 6-8 9-12 1,550 840 20:1 X 31

PunahouSchool,Honolulu X X K-5 6-8 9-12 3,771 1,738 10:1 X 21

Idaho

BishopKellyHighSchool,Boise X X - - 9-12 723 723 15:1 X 22

ColeValleyChristianHighSchool,Meridian X X PK-6 7-8 9-12 813 260 10:1 X 10

RiverstoneInternationalSchool,Boise X X PS-5 6-8 9-12 314 107 6:1 29

Illinois

AcaciaAcademy,LagrangeHighlands X X 1-6 7-8 9-12 100 60 3:1 4

AllemanHighSchool,RockIsland - - 9-12 465 465

BenetAcademy,Lisle X X - - 9-12 1,357 1,357 18:1 X 23

BoylanCentralCatholicHighSchool,Rockford X X - - 9-12 1,025 1,025 12:1 X 27

BrehmPreparatorySchool,Carbondale X X X X - 6-8 9-PG 85 75 4:1 19

CarmelHighSchool,Mundelein X X - - 9-12 1,350 1,350 16:1 X 19

TheChicagoAcademyfortheArts,Chicago X X - - 9-12 143 143 10:1 X

ElginAcademy,Elgin X X PS-4 5-8 9-12 387 140 7:1 X 9

TheGovernorFrenchAcademy,Belleville X X X X K-8 - 9-12 157 51 7:1 X 6

ICCatholicPrep,Elmhurst X X - - 9-12 308 308 9:1 X 21

JosephinumAcademy,Chicago X - - 9-12 199 199 10:1 X 7

TheLatinSchoolofChicago,Chicago X X JK-4 5-8 9-12 1,124 444 8:1 X 25

MarianCentralCatholicHighSchool,Woodstock X X - - 9-12 716 716 17:1 X 16

MotherMcAuleyHighSchool,Chicago X - - 9-12 1,215 1,215 17:1 X 21

NazarethAcademy,LaGrangePark X X - - 9-12 802 802 18:1 X 16

NorthShoreCountryDaySchool,Winnetka X X PK-5 6-8 9-12 510 210 8:1 X 15

NotreDameCollegePrep,Niles X - - 9-12 834 834 17:1 X 27

SaintAnthonyHighSchool,Effingham X X - - 9-12 190 190 10:1 X 13

SaintJosephHighSchool,Westchester X X - - 9-12 476 476 16:1 X 20

SaintViatorHighSchool,ArlingtonHeights X X - - 9-12 990 990 13:1 18

TimothyChristianHighSchool,Elmhurst X X K-6 7-8 9-12 1,030 375 13:1 X 12

UniversityofChicagoLaboratorySchools,Chicago X X N-5 6-8 9-12 1,888 515 10:1 X 15

WaltherChristianAcademy,MelrosePark - - 12:1 X 13

WheatonAcademy,WestChicago X X - - 9-12 640 640 15:1 X 35

YeshivaHighSchool,Skokie X X - - 9-12 145 145 13:1 X 9

Indiana

ConcordiaLutheranHighSchool,FortWayne X X - - 9-12 708 708 16:1 X 21

TheCulverAcademies,Culver X X X X - - 9-PG 802 802 9:1 X 95

InternationalSchoolofIndiana,Indianapolis X X -5 6-8 9-12 64

OldenburgAcademy,Oldenburg X X - - 9-12 198 198 12:1 X 15

ShaweMemorialJunior/SeniorHighSchool,Madison X X K-6 7-8 9-12 342 111 8.5:1 X 12

STUDENTSACCEPTED

PRIVATESCHOOLSAT-A-GLANCE Boarding Day GRADES STUDENT/FACULTY STUDENTOFFERINGS

Boys Girls Boys Girls Lower Middle Upper Total Upper

Student/FacultyRatio

AdvancedPlacementPreparation Sports

Iowa

AlphaOmegaAcademy,RockRapids X X K-5 6-8 9-12 2,350 1,600

DowlingCatholicHighSchool,WestDesMoines X X - - 9-12 1,400 1,400 18:1 X 20

MaharishiSchooloftheAgeofEnlightenment,Fairfield X X X X PS-6 7-8 9-12 213 91 12:1 33

ReginaJunior-SeniorHighSchool,IowaCity PK-6 - 7-12 902 383 X 13

SaintAlbertJunior-SeniorHighSchool,CouncilBluffs PK-6 7-8 9-12 740 207 10:1 X 16

ScattergoodFriendsSchool,WestBranch X X X X - - 9-PG 40 40 2:1 22

Kansas

HymanBrandHebrewAcademyofGreaterKansasCity,OverlandPark X X K-5 6-8 9-12 224 39 5:1 X 4

IndependentSchool,Wichita X X PK-5 6-8 9-12 520 220 9:1 X 16

MaurHill-MountAcademy,Atchison X X X X - - 9-12 195 195 9:1 X 35

SaintThomasAquinasHighSchool,OverlandPark X X - - 9-12 937 937 15:1 X 20

Kentucky

BethHavenChristianSchool,Louisville X X K4-5 6-8 9-12 211 60 9:1 7

LandmarkChristianAcademy,Louisville X X K4-6 7-8 9-12 140 24 6:1 4

LexingtonCatholicHighSchool,Lexington X X - - 9-12 810 810 13:1 X 21

OneidaBaptistInstitute,Oneida X X X X - 6-8 9-12 275 210 11:1 X 10

St.FrancisHighSchool,Louisville X X - - 9-12 150 150 9:1 X 26

TrinityHighSchool,Louisville X - - 9-12 1,320 1,320 12:1 X 51

VillaMadonnaAcademy,Covington X X K-6 7-8 9-12 426 136 8:1 10

WhitefieldAcademy,Louisville X X PS-5 6-8 9-12 778 187 20:1 X 17

Louisiana

AcademyoftheSacredHeart,NewOrleans X 1-4 5-8 9-12 618 213 16:1 X 26

ArchbishopShawHighSchool,Marrero X - 8-9 10-12 655 415 24:1 X 14

HolyCrossSchool,NewOrleans X - 5-7 8-12 1,020 729 13:1 X 28

HolySaviorMenardCatholicHighSchool,Alexandria X X - 7-8 9-12 494 329 12:1 X 15

St.Joseph’sAcademy,BatonRouge X - - 9-12 1,048 1,048 15:1 X 25

St.Martin’sEpiscopalSchool,Metairie X X PK-5 6-8 9-12 483 205 6:1 X 16

SaintThomasMoreCatholicHighSchool,Lafayette X X - - 9-12 1,085 1,085 25:1 X 27

TeurlingsCatholicHighSchool,Lafayette X X - - 9-12 681 681 21:1 22

VandebiltCatholicHighSchool,Houma X X - - 8-12 921 921 25:1 18

WestminsterChristianAcademy,Opelousas X X PK-6 7-8 9-12 1,143 248 13:1 X 13

BangorChristianSchool,Bangor X X K4-5 6-8 9-12 310 80 10:1 10

FryeburgAcademy,Fryeburg X X X X - - 9-PG 599 599 10:1 X 63

GouldAcademy,Bethel X X X X - - 9-PG 232 232 6:1 X 12

LeeAcademy,Lee X X X X - - 9-PG 260 260 15:1 X 51

MaineCentralInstitute,Pittsfield X X X X - - 9-PG 455 455 14:1 X 29

SaintDominicAcademy,Auburn X X PK-6 7-8 9-12 571 203 12:1 X 15

ThorntonAcademy,Saco X X X X - 6-8 9-12 1,503 1,338 14:1 X 29

WaynfleteSchool,Portland X X PK-5 6-8 9-12 575 261 12:1 23

Maryland

AcademyoftheHolyCross,Kensington X - - 9-12 520 520 X 21

ArchbishopCurleyHighSchool,Baltimore X - - 9-12 526 526 14:1 X 20

TheBrynMawrSchoolforGirls,Baltimore X X K-5 6-8 9-12 685 289 7:1 X 41

CalvertHallCollegeHighSchool,Baltimore X - - 9-12 1,196 1,196 12:1 X 35

TheCatholicHighSchoolofBaltimore,Baltimore X - - 9-12 320 320 17:1 X 17

ChelseaSchool,Hyattsville X X - 5-8 9-12 61 44 8:1 X 4

DeMathaCatholicHighSchool,Hyattsville X - - 9-12 795 795 13:1 X 24

ElizabethSetonHighSchool,Bladensburg X - - 9-12 565 565 11:1 X 29

GarrisonForestSchool,OwingsMills X X X N-5 6-8 9-12 628 291 8:1 X 28

GeorgetownPreparatorySchool,NorthBethesda X X - - 9-12 490 490 8:1 X 51

GilmanSchool,Baltimore X K-5 6-8 9-12 1,033 461 8:1 X 26

TheHeightsSchool,PotomacX 3-5

6-89-12 460

2407:1

X 17

IndianCreekSchool,Crownsville - - X 15

InstituteofNotreDame,Baltimore X - - 9-12 367 367 11:1 X 32

TheKeySchool,Annapolis X X PK-4 5-8 9-12 652 209 7:1 X 27

LandonSchool,Bethesda X 3-5 6-8 9-12 675 330 6:1 X 25

Loyola-Blakefield,Baltimore X - 6-8 9-12 983 744 10:1 X 24

McDonoghSchool,OwingsMills X X X X K-4 5-8 9-12 1,299 585 9:1 X 28

TheNoraSchool,SilverSpring X X - - 9-12 60 60 5:1 27

OurLadyofMountCarmelHighSchool,Baltimore X X PK-5 6-8 9-12 16:1 X 12

TheParkSchoolofBaltimore,Baltimore X X PK-5 6-8 9-12 833 331 7:1 X 17

RolandParkCountrySchool,Baltimore X X PS-5 6-8 9-12 676 316 7:1 X 25

St.Andrew’sEpiscopalSchool,Potomac X X PS-3 4-8 9-12 506 248 7:1 X 17

SaintMary’sHighSchool,Annapolis X X - - 9-12 500 500 15:1 X 20

St.Timothy’sSchool,Stevenson X X - - 9-12 165 165 6:1 23

SandySpringFriendsSchool,SandySpring X X X X PK-5 6-8 9-12 572 263 8:1 X 43

WorcesterPreparatorySchool,Berlin X X PK-5 6-8 9-12 525 201 9:1 X 13

STUDENTSACCEPTED

PRIVATESCHOOLSAT-A-GLANCE Boarding Day GRADES STUDENT/FACULTY STUDENTOFFERINGS

Boys Girls Boys Girls Lower Middle Upper Total Upper

Student/FacultyRatio

AdvancedPlacementPreparation Sports

Massachusetts

TheAcademyatCharlemont,Charlemont X X - 7-8 9-PG 97 64 9:1 20

BerkshireSchool,Sheffield X X X X - - 9-PG 386 5:1 X 36

BishopConnollyHighSchool,FallRiver X X - - 9-12 280 280 16:1 X 17

BishopStangHighSchool,NorthDartmouth X X - - 9-12 644 644 13:1 X 30

BostonUniversityAcademy,Boston X X - - 9-12 163 163 8:1 13

BrimmerandMaySchool,ChestnutHill X X PK-5 6-8 9-12 400 135 6:1 X 20

TheBritishSchoolofBoston,Boston X X PK-5 6-8 9-12 417 59 3:1 20

BrooksSchool,NorthAndover X X X X - - 9-12 376 376 5:1 X 19

BuxtonSchool,Williamstown X X X X - - 9-12 89 89 4:1 25

CapeCodAcademy,Osterville X X PK-5 6-8 9-12 317 155 4:1 X 18

CentralCatholicHighSchool,Lawrence X X - - 9-12 1,327 1,327 24:1 X 32

CommonwealthSchool,Boston X X - - 9-12 145 145 4:1 X 16

ConcordAcademy,Concord X X X X - - 9-12 378 378 6:1 35

DeerfieldAcademy,Deerfield X X X X - - 9-PG 630 630 6:1 X 46

DexterSouthfield,Brookline X PK-5 6-8 9-12 424 136 7:1 X 16

DexterSouthfield,Brookline X PK-5 6-8 9-12 330 74 7:1 X 18

FalmouthAcademy,Falmouth X X - 7-8 9-12 195 116 4:1 X 3

FaySchool,Southborough X X X X PK-2 3-6 7-9 473 223 8:1 21

GannAcademy(TheNewJewishHighSchoolofGreaterBoston),Waltham X X 9-12 309 309 5:1 X 16

HillsideSchool,Marlborough X X - 5-9 135 7:1 48

TheJudgeRotenbergEducationalCenter,Canton X X - - 130 3

LandmarkSchool,PridesCrossing X X X X 1-5 6-8 9-12 449 301 3:1 19

LawrenceAcademy,Groton X X X X - - 9-12 399 399 5:1 X 30

MatignonHighSchool,Cambridge X X - - 9-12 458 458 18:1 X 25

MiddlesexSchool,Concord X X X X - - 9-12 378 378 6:1 X 23

MissHall’sSchool,Pittsfield X X - - 9-PG 206 206 6:1 X 33

NobleandGreenoughSchool,Dedham X X X X - 7-8 9-12 613 490 7:1 X 8

NorthfieldMountHermonSchool,MountHermon X X X X - - 9-PG 650 650 6:1 X 51

NotreDameAcademy,Hingham X - - 7-12 568 568 11:1 X 24

ThePingreeSchool,SouthHamilton X X - - 9-12 330 330 7:1 X 32

PioneerValleyChristianSchool,Springfield X X PS-5 6-8 9-12 270 71 4:1 X 10

TheRiversSchool,Weston X X - 6-8 9-12 479 359 6:1 X 18

TheRoxburyLatinSchool,WestRoxbury X - - 7-12 299 299 7:1 X 10

St.John’sPreparatorySchool,Danvers X - - 9-12 1,200 1,200 11:1 X 47

TheSudburyValleySchool,Framingham X X - - 160 16:1

ValleyViewSchool,NorthBrookfield X - 5-8 9-12 56 26 6:1 50

WaldorfHighSchoolofMassachusettsBay,Belmont X X - - 9-12 55 55 7:1 3

WhitinsvilleChristianSchool,Whitinsville X X K-5 6-8 9-12 499 180 8:1 X 10

Wilbraham&MonsonAcademy,Wilbraham X X X X - 6-8 9-PG 431 357 6:1 X 44

TheWillistonNorthamptonSchool,Easthampton X X X X - 7-8 9-PG 544 469 6:1 X 39

TheWoodwardSchool,Quincy X - 6-8 9-12 125 81 8:1 X 4

WorcesterAcademy,Worcester X X X X - 6-8 9-PG 625 510 8:1 X 20

Michigan

DetroitCountryDaySchool,BeverlyHills X X PK-5 6-8 9-12 1,600 698 8:1 X 32

GabrielRichardCatholicHighSchool,Riverview X X - - 9-12 289 289 17:1 X 17

GreenhillsSchool,AnnArbor X X - 6-8 9-12 569 330 8:1 X 15

JeanandSamuelFrankelJewishAcademyofMetropolitanDetroit,WestBloomfield X X - - 9-12 230 230 X

KalamazooChristianHighSchool,Kalamazoo X X K-4 5-8 9-12 773 149 12:1 X 17

TheLeelanauSchool,GlenArbor X X X X - - 9-12 50 50 5:1 X 49

LutheranHighSchoolNorthwest,RochesterHills X X - - 9-12 289 289 15:1 X 20

PowersCatholicHighSchool,Flint X X - - 9-12 545 545 18:1 X 29

SouthfieldChristianHighSchool,Southfield X X PK-5 6-8 9-12 577 209 20:1 X 12

UniversityLiggettSchool,GrossePointeWoods X X PK-5 6-8 9-12 616 282 8:1 16

TheValleySchool,SwartzCreek X X PK-4 5-8 9-12 58 14 8:1 8

Minnesota

CotterSchools,Winona X X X X - 7-8 9-12 343 259 11:1 X 30

Cretin-DerhamHall,SaintPaul X X - - 9-12 1,317 1,317 17:1 X 34

DeLaSalleHighSchool,Minneapolis X X - - 9-12 731 731 14:1 X 22

MarshallSchool,Duluth X X - 4-8 9-12 451 253 11:1 X 18

ProvidenceAcademy,Plymouth X X PK-5 6-8 9-12 867 335 11:1 X 16

St.CroixSchools,WestSt.Paul X X X X - 6-8 9-12 500 440 15:1 X 38

SaintJohn’sPreparatorySchool,Collegeville X X X X 6-6 7-8 9-PG 295 220 10:1 X 50

St.PaulAcademyandSummitSchool,St.Paul X X K-5 6-8 9-12 901 390 7:1 25

St.PaulPreparatorySchool,SaintPaul X X - - 9-12 203 203 12:1 X 2

Shattuck-St.Mary’sSchool,Faribault X X X X - 6-9 9-PG 443 338 9:1 X 27

BrookhavenAcademy,Brookhaven X X -6 - 7-12 506 209 13:1 13

St.StanislausCollege,BaySt.Louis X X - 7-8 9-12 352 250 12:1 X 34

VicksburgCatholicSchool,Vicksburg X X PK-6 - 7-12 577 270 11:1 X 13

Missouri

JohnBurroughsSchool,St.Louis X X - 7-8 9-12 600 409 7:1 X 27

KansasCityAcademy,KansasCity X X - 6-8 9-12 45 32 6:1

LutheranHighSchoolNorth,St.Louis X X - - 9-12 310 310 10:1 X 13

MaryInstituteandSt.LouisCountryDaySchool(MICDS),St.Louis X X JK-4 5-8 9-12 1,247 628 8:1 X 27

MissouriMilitaryAcademy,Mexico X X - 6-8 9-PG 210 178 11:1 X 42

MUHighSchool,Columbia X X - - X

NerinxHall,WebsterGroves X - - 9-12 593 593 10:1 X 13

NewCovenantAcademy,Springfield X X JK-6 7-8 9-12 397 82 6

RockhurstHighSchool,KansasCity X - - 9-12 1,100 12:1 X 23

SaintTeresa’sAcademy,KansasCity X - - 9-12 607 607 12:1 X 25

ThomasJeffersonSchool,St.Louis X X X X - 7-8 9-PG 91 68 6:1 X 9

ValleCatholicHighSchool,Ste.Genevieve X X - - 9-12 152 152 9:1 14

VisitationAcademy,St.Louis X X PK-6 - 7-12 642 478 9:1 X 15

WhitfieldSchool,St.Louis X X - - 6-12 379 379 8:1 X 14

STUDENTSACCEPTED

PRIVATESCHOOLSAT-A-GLANCE Boarding Day GRADES STUDENT/FACULTYSTUDENTOFFERINGS

Boys Girls Boys Girls Lower Middle Upper Total Upper

Student/FacultyRatio

AdvancedPlacementPreparation Sports

Montana

ButteCentralCatholicHighSchool,Butte X X X - - 9-12 141 141 10:1 X 11

LustreChristianHighSchool,Lustre X X X X - - 9-12 40 40 4:1 4

ManhattanChristianHighSchool,Manhattan X X PK-5 6-8 9-12 266 65 8:1 X 12

SummitPreparatorySchool,Kalispell X X - - 9-12 46 46 5:1 X 61

Nebraska

MountMichaelBenedictineSchool,Elkhorn X X - - 9-12 221 221 8:1 X 22

NebraskaChristianSchools,CentralCity X X X X K-6 7-8 9-12 204 121 10:1 6

ScotusCentralCatholicHighSchool,Columbus X X - 7-8 9-12 396 257 14:1 X 13

NevadaTheDr.MiriamandSheldonG.AdelsonEducationalCampus,TheAdelsonUpperSchool,LasVegas X X PS-4 5-8 9-12 618 132 10:1 X 15

FaithLutheranHighSchool,LasVegas X X - 6-8 9-12 1,390 749 17:1 X 22

TheMeadowsSchool,LasVegas X X PK-5 6-8 9-12 895 273 11:1 X 14

NewHampshire

BishopBradyHighSchool,Concord X X - - 9-12 344 344 16:1 X 30

BishopGuertinHighSchool,Nashua X X - - 9-12 900 900 X 35

CardiganMountainSchool,Canaan X X - 6-9 215 192 4:1 41

TheDerryfieldSchool,Manchester X X - 6-8 9-12 387 271 8:1 X 26

HampshireCountrySchool,Rindge X 3-6 - 7-12 21 16 2:1 23

HighMowingSchool,Wilton X X X X - - 9-12 109 109 5:1 X 41

KimballUnionAcademy,Meriden X X X X - - 9-PG 338 338 6:1 X 38

PortsmouthChristianAcademy,Dover X X PK-5 6-8 9-12 617 181 13:1 X 16

ProctorAcademy,Andover X X X X - - 9-12 360 360 5:1 X 58

St.ThomasAquinasHighSchool,Dover X X - - 9-12 585 585 14:1 X 18

TiltonSchool,Tilton X X X X - - 9-PG 222 222 5:1 X 28

TrinityHighSchool,Manchester X X - - 9-12 442 442 12:1 X 22

TheWhiteMountainSchool,Bethlehem X X X X - - 9-PG 112 112 5:1 X 45

NewJersey

TheAmericanBoychoirSchool,Princeton X X 4-5 6-8 51 5:1 2

BarnstableAcademy,Oakland X X - 5-8 9-12 120 90 8:1 X 16

ChristianBrothersAcademy,Lincroft X - - 9-12 1,007 1,007 14:1 X 16

DelbartonSchool,Morristown X - 7-8 9-12 558 491 13:1 X 28

DePaulCatholicHighSchool,Wayne X X - - 650 19:1 X 38

HawthorneChristianAcademy,Hawthorne X X PS-5 6-8 9-12 496 147 7:1 X 9

TheHudsonSchool,Hoboken X X - 5-8 9-12 185 85 4:1 X 15

TheHunSchoolofPrinceton,Princeton X X X X - 6-8 9-PG 630 535 8:1 X 35

KentPlaceSchool,Summit X X N-5 6-8 9-12 636 277 7:1 X 17

TheLawrencevilleSchool,Lawrenceville X X X X - - 9-PG 819 819 8:1 51

MoorestownFriendsSchool,Moorestown X X PS-4 5-8 9-12 722 303 7:1 X 18

NewarkAcademy,Livingston X X - 6-8 9-12 582 406 12:1 X 32

OurLadyofMercyAcademy,Newfield X - - 9-12 150 150 11:1 17

ThePenningtonSchool,Pennington X X X X - 6-8 9-12 487 406 8:1 X 24

ThePingrySchool,BaskingRidge X X K-5 6-8 9-12 1,084 558 8:1 X 25

PopeJohnXXIIIRegionalHighSchool,Sparta X X - - 8-12 943 943 13:1 X 19

RanneySchool,TintonFalls X X N-5 6-8 9-12 820 303 9:1 X 25

SaintAugustinePreparatorySchool,Richland X - - 9-12 680 680 12:1 X 25

VillaVictoriaAcademy,Ewing X PK-6 7-8 9-12 168 55 8:1 X 10

VillaWalshAcademy,Morristown X - 7-8 9-12 252 222 8:1 X 11

NewMexico

AlbuquerqueAcademy,Albuquerque X X - 6-7 8-12 1,110 813 9:1 X 28

SandiaPreparatorySchool,Albuquerque X X - 6-8 9-12 612 348 7:1 33

NewYork

AllendaleColumbiaSchool,Rochester X X N-5 6-8 9-12 375 141 X 11

AllHallowsHighSchool,Bronx X - - 9-12 640 640 15:1 14

BayRidgePreparatorySchool,Brooklyn X X K-5 6-8 9-12 7:1 X 13

TheBeekmanSchool,NewYork X X - - 9-PG 80 80 8:1 X

BerkeleyCarrollSchool,Brooklyn X X N-4 5-8 9-12 900 270 8:1 15

TheBirchWathenLenoxSchool,NewYork X X K-5 6-8 9-12 570 170 15:1 X 26

CascadillaSchool,Ithaca X X X X - - 9-PG 50 6:1 X 51

CathedralHighSchool,NewYork X - - 9-12 550 550 17:1 X 6

CatholicCentralHighSchool,Troy X X - 7-8 9-12 520 415 15:1 X 17

ChristianBrothersAcademy,Syracuse X X - - 7-12 750 750 X 18

ChristianCentralAcademy,Williamsville X X K-5 6-8 9-12 355 110 10:1 X 12

CollegiateSchool,NewYork X K-4 5-8 9-12 650 227 4:1 X 13

ConventoftheSacredHeart,NewYork X PK-4 5-8 9-12 700 210 10:1 X 23

DarrowSchool,NewLebanon X X X X - - 9-12 120 120 4:1 X 27

DominicanAcademy,NewYork X - - 9-12 208 208 8:1 X 9

FordhamPreparatorySchool,Bronx X - - 9-12 936 936 11:1 X 23

French-AmericanSchoolofNewYork,Mamaroneck X X N-5 6-8 9-12 842 208 6:1 X 8

FriendsAcademy,LocustValley X X N-5 6-8 9-12 780 376 8:1 X 18

TheHarleySchool,Rochester X X N-4 5-8 9-12 537 186 8:1 X 15

HebrewAcademyoftheFiveTowns&Rockaway,Cedarhurst X X 9-12 388 388 X 7

HoughtonAcademy,Houghton X X X X - 6-8 9-PG 130 107 8:1 X 15

TheKew-ForestSchool,ForestHills X X N-6 7-8 9-12 238 100 8:1 X 17

KildonanSchool,Amenia X X X X 2-6 7-8 9-PG 91 50 2:1 39

LaScuolaD’ItaliaGuglielmoMarconi,NewYork X X PK-5 6-8 9-12 261 44 X 4

LittleRedSchoolHouseandElisabethIrwinHighSchool,NewYork X X N-4 5-8 9-12 650 240 7:1 28

LongIslandLutheranMiddleandHighSchool,Brookville X X - 6-8 9-12 590 413 9:1 X 23

LoyolaSchool,NewYork X X - - 9-12 202 202 9:1 15

MaplebrookSchool,Amenia X X X X - - 75 62 8:1 41

TheMaryLouisAcademy,JamaicaEstates X - - 9-12 855 855 13:1 X 27

MillbrookSchool,Millbrook X X X X - - 9-12 283 283 5:1 X 30

NormanHowardSchool,Rochester X X 4-4 5-8 9-12 123 82 3:1 7

TheParkSchoolofBuffalo,Snyder X X X X PK-4 5-8 9-12 290 133 8:1 X 31

PoughkeepsieDaySchool,Poughkeepsie X X PK-5 6-8 9-12 280 115 7:1 X 27

PrestonHighSchool,Bronx X - - 9-12 600 600 25:1 X 7

ProfessionalChildren’sSchool,NewYork X X - 6-8 9-12 195 160 8:1

RegisHighSchool,NewYork X - - 9-12 530 530 15:1 X 8

RiverdaleCountrySchool,Bronx X X PK-5 6-8 9-12 1,125 500 8:1 23

RossSchool,EastHampton X X X X N-6 - 7-12 521 370 7:1 X 21

SmithSchool,NewYork X X - 7-8 9-12 57 42 4:1 7

SoundviewPreparatorySchool,YorktownHeights X X - 6-8 9-12 68 56 5:1 X 4

TheSpenceSchool,NewYork X K-4 5-8 9-12 725 234 7:1 X 13

TheStormKingSchool,Cornwall-on-Hudson X X X X - 8-8 9-12 139 132 6:1 X 65

TrevorDaySchool,NewYork X X N-5 6-8 9-12 800 248 6:1 X 14

Trinity-PawlingSchool,Pawling X X - 7-8 9-PG 283 260 8:1 X 40

TheWaldorfSchoolofGardenCity,GardenCity X X N-5 6-8 9-12 350 105 5:1 14

TheWaldorfSchoolofSaratogaSprings,SaratogaSprings X X PK-8 - 9-12 241 45 13:1 15

TheWindsorSchool,Flushing X X - 6-8 9-13 180 170 14:1 X 11

WinstonPreparatorySchool,NewYork X X - 4-8 9-12 180 124 3:1 14

YorkPreparatorySchool,NewYork X X - 6-8 9-12 358 262 6:1 X 26

NorthCarolina

ArthurMorganSchool,Burnsville X X X X - 7-9 27 2:1 26

CapeFearAcademy,Wilmington X X PK-5 6-8 9-12 573 222 7:1 X 13

CardinalGibbonsHighSchool,Raleigh X X - - 9-12 1,294 1,294 14:1 X 34

CharlotteCountryDaySchool,Charlotte X X PK-4 5-8 9-12 1,631 506 12:1 21

CharlotteLatinSchool,Charlotte X X K-5 6-8 9-12 1,401 500 8:1 X 20

DurhamAcademy,Durham X X PK-4 5-8 9-12 1,165 411 8:1 X 25

FayettevilleAcademy,Fayetteville X X PK-5 6-8 9-12 370 140 14:1 X 13

GastonDaySchool,Gastonia X X PS-4 5-8 9-12 512 158 7:1 X 11

HarrellsChristianAcademy,Harrells X X K-5 6-8 9-12 412 163 11:1 X 9

TheO’NealSchool,SouthernPines X X PK-5 6-8 9-12 415 158 12:1 X 10

ProvidenceDaySchool,Charlotte X X PK-5 6-8 9-12 1,580 570 9:1 X 24

SalemAcademy,Winston-Salem X X - - 9-12 160 160 7:1 X 22

WestchesterCountryDaySchool,HighPoint X X PK-5 6-8 9-12 355 126 6:1 X 15

NorthDakota

ShanleyHighSchool,Fargo X X - - 9-12 313 313 18:1 X 11

Ohio

ArchbishopAlterHighSchool,Kettering X X - - 9-12 673 673 12:1 X 20

ArchbishopHobanHighSchool,Akron X X - - 9-12 861 861 15:1 X 22

TheColumbusAcademy,Gahanna X X PK-4 5-8 9-12 1,085 372 8:1 X 15

ColumbusSchoolforGirls,Columbus X PK-5 6-8 9-12 561 195 9:1 X 16

TheGrandRiverAcademy,Austinburg X - - 9-12 111 111 6:1 X 62

LawrenceSchool,SagamoreHills X X K-6 7-8 9-12 280 136 11:1 11

PaduaFranciscanHighSchool,Parma X X - - 9-12 755 755 19:1 X 43

St.FrancisdeSalesHighSchool,Toledo X - - 9-12 612 612 14:1 X 17

SaintUrsulaAcademy,Toledo X - 7-8 9-12 549 522 12:1 X 31

TheSevenHillsSchool,Cincinnati X X PK-5 6-8 9-12 1,028 315 9:1 X 13

TheSummitCountryDaySchool,Cincinnati X X PK-4 5-8 9-12 1,020 369 9:1 X 19

TrinityHighSchool,GarfieldHeights X X - - 9-12 352 352 12:1 X 15

VillaAngela-St.JosephHighSchool,Cleveland X X - - 9-12 375 375 12:1 X 21

BishopMcGuinnessCatholicHighSchool,OklahomaCity X X 9-10 - 11-12 701 341 13:1 X 18

Oregon

BlanchetSchool,Salem X X - 6-8 9-12 357 231 17:1 X 15

TheCatlinGabelSchool,Portland X X PS-5 6-8 9-12 753 309 8:1 50

DeLaSalleNorthCatholicHighSchool,Portland X X - - 9-12 322 322 17:1 X 7

NorthwestAcademy,Portland X X - 6-8 9-12 182 80 4:1 10

OregonEpiscopalSchool,Portland X X X X PK-5 6-8 9-12 858 316 7:1 X 21

PacificCrestCommunitySchool,Portland X X - 6-8 9-12 85 55 9:1 12

St.Mary’sSchool,Medford X X X X - 6-8 9-12 461 312 11:1 X 30

SalemAcademy,Salem X X K-5 6-8 9-12 631 258 9:1 X 12

WellspringsFriendsSchool,Eugene X X - - 9-12 45 45 8:1 7

STUDENTSACCEPTED

PRIVATESCHOOLSAT-A-GLANCE Boarding Day GRADES STUDENT/FACULTYSTUDENTOFFERINGS

Boys Girls Boys Girls Lower Middle Upper Total Upper

Student/FacultyRatio

AdvancedPlacementPreparation Sports

Pennsylvania

AcademyoftheNewChurchBoys’School,BrynAthyn X X - - 9-12 124 124 8:1 X 6

AcademyoftheNewChurchGirls’School,BrynAthyn X X - - 9-12 101 101 8:1 X 9

TheAgnesIrwinSchool,Rosemont X PK-4 5-8 9-12 703 289 5:1 X 23

BishopShanahanHighSchool,Downingtown X X - - 9-12 1,125 1,125 X 16

CamphillSpecialSchool,Glenmoore X X X X K-5 6-8 9-13 118 59 5:1

CardinalO’HaraHighSchool,Springfield X X - - 9-12 1,289 21:1 X 21

ChristopherDockMennoniteHighSchool,Lansdale X X X X - - 9-12 351 351 11:1 X 12

TheConceptSchool,Westtown X X - 5-8 9-12 24 16 5:1 13

CountryDaySchooloftheSacredHeart,BrynMawr X PK-4 5-8 9-12 314 165 8:1 X 11

DevonPreparatorySchool,Devon X - 6-8 9-12 253 199 10:1 X 10

TheEpiscopalAcademy,NewtownSquare X X PK-5 6-8 9-12 1,216 528 7:1 X 37

FatherJudgeHighSchool,Philadelphia X - - 9-12 986 986 28:1 X 24

Friends’CentralSchool,Wynnewood X X N-5 5-8 9-12 812 377 8:1 25

GeorgeSchool,Newtown X X X X - - 9-12 545 545 7:1 X 25

GermantownFriendsSchool,Philadelphia X X K-5 6-8 9-12 861 353 9:1 17

GirardCollege,Philadelphia X X 1-6 - 7-12 331 201 16:1 X 32

GrierSchool,Tyrone X X - 7-8 9-PG 306 262 7:1 X 46

TheHillTopPreparatorySchool,Rosemont X X - 5-9 10-12 73 34 6:1 27

LancasterCountryDaySchool,Lancaster X X PS-5 6-8 9-12 615 213 6:1 X 20

LancasterMennoniteHighSchool,Lancaster X X X X PK-5 6-8 9-12 1,412 638 15:1 X 14

LansdaleCatholicHighSchool,Lansdale X X - - 9-12 763 763 21:1 X 25

LehighValleyChristianHighSchool,Catasauqua X X - - 9-12 122 122 9:1 X 8

MarianCatholicHighSchool,Tamaqua X X - - 9-12 378 378 11:1 X 8

MercersburgAcademy,Mercersburg X X X X - - 9-PG 430 430 5:1 X 58

MMIPreparatorySchool,Freeland X X - 6-8 9-12 247 157 11:1 X 11

MoravianAcademy,Bethlehem X X PK-5 6-8 9-12 750 300 7:1 X 14

NazarethAcademyHighSchoolforGirls,Philadelphia X - - 9-12 378 378 9:1 X 14

NotreDameJunior/SeniorHighSchool,EastStroudsburg X X - - 7-12 270 270 15:1 X 14

TheOaklandSchool,Pittsburgh X X - - 8-12 40 40 6:1 32

ThePathwaySchool,Norristown X X - - 120 46 6:1 5

ThePhelpsSchool,Malvern X X - - 7-PG 110 110 4:1 X 24

Philadelphia-MontgomeryChristianAcademy,Erdenheim X X K-5 6-8 9-12 309 140 10:1 X 8

St.Joseph’sPreparatorySchool,Philadelphia X - - 9-12 968 968 16:1 X 32

SewickleyAcademy,Sewickley X X PK-5 6-8 9-12 708 289 7:1 X 15

ShadySideAcademy,Pittsburgh X X X X PK-5 6-8 9-12 935 492 8:1 X 25

TheShipleySchool,BrynMawr X X PK-5 6-8 9-12 823 352 7:1 X 25

SpringsideChestnutHillAcademy,Philadelphia X X PK-4 5-8 9-12 1,102 472 7:1 X 18

VillaJosephMarieHighSchool,Holland X - - 9-12 363 363 9:1 X 15

WesttownSchool,WestChester X X X X PK-5 6-8 9-12 632 381 8:1 X 36

WilliamPennCharterSchool,Philadelphia X X PK-5 6-8 9-12 966 458 9:1 X 19

WoodlyndeSchool,Strafford X X K-5 6-8 9-12 262 100 5:1 X 11

BaldwinSchoolofPuertoRico,Bayamon X X PK-5 6-8 9-12 812 216 8:1 X 14

ColegioSanJose,SanJuan X - 7-8 9-12 515 351 X 13

FowlersAcademy,Guaynabo X X - 7-8 9-12 64 44 15:1 8

GuamaniPrivateSchool,Guayama X X 1-6 7-8 9-12 606 164 13:1 X 6

WesleyanAcademy,Guaynabo X X PK-6 7-8 9-12 959 161 23:1 X 8

RhodeIsland

BishopHendrickenHighSchool,Warwick X - - 9-12 948 948 14:1 X 34

MountSaintCharlesAcademy,Woonsocket X X - - 7-12 699 699 14:1 X 27

ProvidenceCountryDaySchool,EastProvidence X X - 6-8 9-12 200 160 7:1 X 19

St.Andrew’sSchool,Barrington X X X X - 6-8 9-PG 205 175 5:1 X 32

St.George’sSchool,Middletown X X X X - - 9-12 365 365 6:1 X 22

TheWheelerSchool,Providence X X N-5 6-8 9-12 817 350 7:1 X 15

SouthCarolina

AshleyHall,Charleston X X PS-4 5-6 7-12 677 299 8:1 X 41

ChristChurchEpiscopalSchool,Greenville X X K-4 5-8 9-12 1,151 409 10:1 16

HankHaneyInternationalJuniorGolfAcademy,HiltonHeadIsland X X X X 5-PG 140 140 10:1 X 1

Porter-GaudSchool,Charleston X X 1-5 6-8 9-12 886 359 12:1 X 26

St.Joseph’sCatholicSchool,Greenville X X - 6-8 9-12 669 405 13:1 X 18

WilsonHall,Sumter X X PS-5 6-8 9-12 778 242 13:1 X 34

SouthDakota

FreemanAcademy,Freeman X X X X 1-4 5-8 9-12 68 34 5:1 7

SunshineBibleAcademy,Miller X X X X K-5 6-8 9-12 94 65 9:1 12

Tennessee

BachmanAcademy,McDonald X X X X - 6-8 9-PG 39 28 3:1 40

BattleGroundAcademy,Franklin X X K-4 5-8 9-12 809 324 8:1 X 30

BriarcrestChristianHighSchool,Eads X X PK-5 6-8 9-12 1,640 535 12:1 X 20

ChattanoogaChristianSchool,Chattanooga X X K-5 6-8 9-12 1,179 449 17:1 X 18

ClarksvilleAcademy,Clarksville X X PK-5 6-8 9-12 576 183 12:1 X 12

ColumbiaAcademy,Columbia X X K-6 - 7-12 765 326 10:1 X 15

CurreyIngramAcademy,Brentwood X X K-4 5-8 9-12 301 77 4:1 8

DavidsonAcademy,Nashville X X PK-6 7-8 9-12 750 17:1 X 16

DonelsonChristianAcademy,Nashville X X K4-5 6-8 9-12 701 219 16:1 X 19

Ezell-HardingChristianSchool,Antioch X X PK-4 5-8 9-12 543 198 14:1 X 14

FatherRyanHighSchool,Nashville X X - - 9-12 948 948 12:1 X 30

FranklinRoadAcademy,Nashville X X PK-4 5-8 9-12 720 264 8:1 X 23

GirlsPreparatorySchool,Chattanooga X - 6-8 9-12 584 347 8:1 X 45

GraceBaptistAcademy,Chattanooga X X K4-5 6-8 9-12 572 161 12:1 X 16

TheHarpethHallSchool,Nashville X - 5-8 9-12 671 400 8:1 X 38

LipscombAcademy,Nashville X X PK-4 5-8 9-12 1,318 523 15:1 X 14

MiddleTennesseeChristianSchool,Murfreesboro X X PK-6 7-8 9-12 665 200 18:1 X 13

MontgomeryBellAcademy,Nashville X - 7-8 9-12 718 495 7:1 X 38

St.BenedictatAuburndale,Cordova X X - - 9-12 945 945 16:1 X 24

St.George’sIndependentSchool,Collierville X X PK-5 6-8 9-12 1,213 394 7:1 X 16

St.Mary’sEpiscopalSchool,Memphis X PK-4 5-8 9-12 849 246 13:1 X 12

TrinityChristianAcademy,Jackson X X PS-5 6-8 9-12 772 241 7:1 13

UniversitySchoolofJackson,Jackson X X PK-5 6-8 9-12 1,150 315 13:1 X 16

WebbSchoolofKnoxville,Knoxville X X K-5 6-8 9-12 1,034 471 10:1 X 19

Texas

TheBrookHillSchool,Bullard X X X X PK-5 6-8 9-12 519 191 9:1 X 24

CarrolltonChristianAcademy,Carrollton X X PK-6 7-8 9-12 252 14

CistercianPreparatorySchool,Irving X - 5-8 9-12 352 178 5:1 X 14

DuchesneAcademyoftheSacredHeart,Houston X PK-4 5-8 9-12 680 236 7:1 X 14

TheEmery/WeinerSchool,Houston X X - 6-8 9-12 7:1 X 15

EpiscopalHighSchool,Bellaire X X - - 9-12 676 676 9:1 X 23

FirstBaptistAcademy,Dallas X X PK-5 6-8 9-12 207 89 8:1 X 14

FortWorthChristianSchool,NorthRichlandHills X X PK-5 6-8 9-12 850 356 14:1 X 16

GatewaySchool,Arlington X X - 5-8 9-12 5:1 6

GreenhillSchool,Addison X X PK-4 5-8 9-12 1,276 476 7:1 X 29

TheHockadaySchool,Dallas X X PK-4 5-8 9-12 1,086 478 15:1 X 42

Huntington-SurreySchool,Austin X X - - 9-12 48 48 4:1 1

HydeParkBaptistSchool,Austin X X PK-5 6-8 9-12 599 286 12:1 X 14

JesuitCollegePreparatorySchool,Dallas X - - 9-12 1,095 1,095 11:1 X 28

KeystoneSchool,SanAntonio X X K-4 5-8 9-12 431 128 10:1 X 8

LakehillPreparatorySchool,Dallas X X K-4 5-8 9-12 400 115 10:1 X 17

MarineMilitaryAcademy,Harlingen X - - 8-12 250 250 13:1 X 40

NorthCentralTexasAcademy,Granbury X X X X K-5 6-8 9-12 194 70 7:1 17

ParishEpiscopalSchool,Dallas X X PK-4 5-8 9-12 1,100 22

PresbyterianPanAmericanSchool,Kingsville X X X X - - 9-12 160 160 10:1 19

ProvidenceCatholicSchool,TheCollegePreparatorySchoolforGirlsGrades6-12,SanAntonio X 6-8 9-12 315 193 11:1 X 20

St.AgnesAcademy,Houston X - - 9-12 899 899 12:1 X 20

St.Mark’sSchoolofTexas,Dallas X 1-4 5-8 9-12 851 364 8:1 X 41

SaintMary’sHall,SanAntonio X X PK-5 6-8 9-PG 1,008 397 6:1 X 21

St.PiusXHighSchool,Houston X X - - 9-12 670 670 12:1 X 17

St.Stephen’sEpiscopalSchool,Austin X X X X - 6-8 9-12 689 487 8:1 X 39

SanMarcosBaptistAcademy,SanMarcos X X X X - 7-8 9-12 286 237 7:1 X 24

SheltonSchoolandEvaluationCenter,Dallas X X PS-4 5-8 9-12 827 262 8:1 12

TheTenneySchool,Houston X X - 6-8 9-12 57 37 2:1 X

TMI-TheEpiscopalSchoolofTexas,SanAntonio X X X X - 6-8 9-12 470 340 10:1 X 23

TrinitySchoolofTexas,Longview X X PK-5 6-8 9-12 275 60 8:1 X 12

TrinityValleySchool,FortWorth X X K-4 5-8 9-12 970 348 5:1 X 12

TylerStreetChristianAcademy,Dallas X X P3-6 7-8 9-12 198 47 13:1 X 16

TheUrsulineAcademyofDallas,Dallas X - - 9-12 800 800 10:1 X 14

WestburyChristianSchool,Houston X X PK-4 5-8 9-12 480 225 10:1 X 13

TheWinstonSchool,Dallas X X 1-6 7-8 9-12 166 80 5:1 21

TheWinstonSchoolSanAntonio,SanAntonio X X K-6 7-8 9-12 206 95 8:1 14

STUDENTSACCEPTED

PRIVATESCHOOLSAT-A-GLANCE Boarding Day GRADES STUDENT/FACULTY STUDENTOFFERINGS

Boys Girls Boys Girls Lower Middle Upper Total Upper

Student/FacultyRatio

AdvancedPlacementPreparation Sports

Utah

IntermountainChristianSchool,SaltLakeCity X X PK-5 6-8 9-12 285 59 9:1 X 5

RealmsofInquiry,Murray X X - 6-8 9-12 14 10 7:1 62

RowlandHall,SaltLakeCity X X PK-5 6-8 9-12 927 300 7:1 X 30

WasatchAcademy,Mt.Pleasant X X X X - 7-8 9-PG 304 290 10:1 X 65

TheWaterfordSchool,Sandy X X PK-5 6-8 9-12 875 251 4:1 X 33

Vermont

BurrandBurtonAcademy,Manchester X X X X - - 9-12 667 667 12:1 X 26

LongTrailSchool,Dorset X X - 6-8 9-12 155 93 8:1 26

TheSharonAcademy,Sharon X X - 7-8 9-12 165 129 16:1

VirginIslands

GoodHopeCountryDaySchool,Kingshill X X N-6 7-8 9-12 440 150 12:1 X 12

Virginia

BishopDenisJ.O’ConnellHighSchool,Arlington X X - - 1,221 1,221 12:1 X 24

BishopIretonHighSchool,Alexandria X X - - 9-12 785 785 14:1 X 25

CarlisleSchool,Axton X X X X PK-5 6-8 9-12 453 138 8:1 X 17

EasternMennoniteHighSchool,Harrisonburg X X K-5 6-8 9-12 385 194 12:1 X 11

EpiscopalHighSchool,Alexandria X X - - 9-12 439 439 6:1 X 44

FishburneMilitarySchool,Waynesboro X X - 7-8 9-12 170 155 9:1 X 12

FoxcroftSchool,Middleburg X X - - 9-12 165 165 7:1 X 34

FredericksburgAcademy,Fredericksburg X X PK-5 6-8 9-12 552 135 6:1 X 11

FuquaSchool,Farmville X X PK-5 6-8 9-12 382 143 7:1 X 13

HamptonRoadsAcademy,NewportNews X X PK-4 5-8 9-12 588 248 10:1 X 32

HargraveMilitaryAcademy,Chatham X X - 7-9 10-PG 310 260 12:1 X 49

LittleKeswickSchool,Keswick X - - 34 3:1 13

TheMadeiraSchool,McLean X X - - 9-12 310 310 10:1 X 18

NorfolkAcademy,Norfolk X X 1-6 7-9 10-12 1,248 379 10:1 X 22

OaklandSchool,Keswick X X X X - - 5:1 43

ThePotomacSchool,McLean X X K-3 4-8 9-12 1,024 423 6:1 X 29

Randolph-MaconAcademy,FrontRoyal X X X X - 6-8 9-PG 330 264 14:1 X 27

SaintGertrudeHighSchool,Richmond X - - 9-12 266 266 9:1 X 12

St.Stephen’s&St.AgnesSchool,Alexandria X X JK-5 6-8 9-12 1,144 456 9:1 X 28

WakefieldSchool,ThePlains X X PS-5 6-8 9-12 397 152 11:1 X 17

Washington

AnnieWrightSchool,Tacoma X X X PS-5 6-8 9-12 467 180 5:1 8

ArchbishopThomasJ.MurphyHighSchool,Everett X X - - 14:1 X 14

ChrysalisSchool,Woodinville X X K-6 7-8 9-12 165 108 3:1

ExplorationsAcademy,Bellingham X X - - 41 41 7:1 X

HolyNamesAcademy,Seattle X - - 9-12 690 690

TheOverlakeSchool,Redmond X X - 5-8 9-12 534 304 9:1 X 36

SeattleAcademyofArtsandSciences,Seattle X X - 6-8 9-12 720 458 9:1 26

ShorelineChristian,Shoreline X X PS-6 7-8 9-12 212 76 7:1 5

WestSoundAcademy,Poulsbo X X X X - 6-8 9-12 95 61 7:1 X 15

WestVirginia

TheLinslySchool,Wheeling X X X X 5-8 - 9-12 443 298 10:1 X 47

Wisconsin

CatholicCentralHighSchool,Burlington X X - - 9-12 180 180 9:1 X 23

MadisonCountryDaySchool,Waunakee X X PK-4 5-8 9-12 366 62 6:1 5

St.LawrenceSeminaryHighSchool,MountCalvary X - - 9-12 195 195 9:1 26

UniversitySchoolofMilwaukee,Milwaukee X X PK-4 5-8 9-12 1,120 376 9:1 X 16

CANADATheAcademyforGiftedChildren(PACE),RichmondHill,ON X X 1-3 4-7 8-12 306 113 15:1 X 43

BalmoralHallSchool,Winnipeg,MB X X N-5 6-8 9-12 474 154 7:1 X 67

BranksomeHall,Toronto,ON X X JK-6 7-8 9-12 885 445 9:1 42

CentralAlbertaChristianHighSchool,Lacombe,AB X X - - 10-12 103 103 13:1 22

ColumbiaInternationalCollegeofCanada,Hamilton,ON X X X X - 7-9 10-12 1,804 1,600 20:1 X 50

CommunityHebrewAcademy,Toronto,ON X X - - 9-12 1,300 1,300 8:1 17

TheCountryDaySchool,KingCity,ON X X JK-6 7-8 9-12 700 300 9:1 X 32

CovenantCanadianReformedSchool,Neerlandia,AB X X K-6 7-9 10-12 221 28 10:1 17

CrawfordAdventistAcademy,Willowdale,ON X X JK-6 7-8 9-12 354 151 16:1 X 13

DeLaSalleCollege,Toronto,ON X X 5-6 7-8 9-12 627 434 15:1 X 21

GlenlyonNorfolkSchool,Victoria,BC X X JK-5 6-8 9-12 685 280 8:1 27

GreatLakesChristianHighSchool,Beamsville,ON X X X X - - 9-12 94 94 19

HeritageChristianSchool,Jordan,ON X X K-8 - 9-12 630 181 15:1 5

HighroadAcademy,Chilliwack,BC X X K-6 7-8 9-12 390 114 10:1

HolyTrinitySchool,RichmondHill,ON X X JK-6 7-8 9-12 750 400 17:1 X 15

King’s-EdgehillSchool,Windsor,NS X X X X - 6-9 10-12 290 200 10:1 33

KingswayCollege,Oshawa,ON X X X X - - 9-12 187 187 12:1 17

LakefieldCollegeSchool,Lakefield,ON X X X X - - 9-12 366 366 7:1 X 43

TheLaureateAcademy,Winnipeg,MB X X 1-5 6-8 9-12 80 30 6:1 36

LutherCollegeHighSchool,Regina,SK X X X X - - 9-12 430 430 17:1 21

MeadowridgeSchool,MapleRidge,BC X X JK-5 - 6-12 540 290 12:1 12

MennoniteCollegiateInstitute,Gretna,MB X X X X - 7-8 9-12 140 122 13:1 X 14

MissEdgar’sandMissCramp’sSchool,Montreal,QC X K-5 6-8 9-11 335 115 9:1 X 26

Newton’sGroveSchool,Toronto,ON X X JK-6 7-8 9-12 255 120 18:1 27

NiagaraChristianCommunityofSchools,FortErie,ON X X X X - 6-8 9-12 178 165 16:1 18

NorthTorontoChristianSchool,Toronto,ON X X JK-6 7-8 9-12 438 182 15:1 27

QueenMargaret’sSchool,Duncan,BC X X X PS-3 4-8 9-12 317 128 8:1 X 63

RedeemerChristianHighSchool,Ottawa,ON X X - - 9-12 167 167 9:1

RidleyCollege,St.Catharines,ON X X X X JK-8 - 9-PG 651 470 7:1 78

RosseauLakeCollege,Rosseau,ON X X X X - 7-8 9-12 84 74 6:1 82

RoyalCanadianCollege,Vancouver,BC X X - 9-10 11-12 110 91 20:1 7

SacredHeartSchoolofHalifax,Halifax,NS X X K-6 - 7-12 461 258 10:1 X 17

St.Andrew’sCollege,Aurora,ON X X - 5-8 9-12 613 446 8:1 X 62

St.Andrew’sRegionalHighSchool,Victoria,BC X X - 8-9 10-12 14:1 8

St.George’sSchool,Vancouver,BC X X 1-7 - 8-12 1,157 778 8:1 X 33

St.George’sSchoolofMontreal,Montreal,QC X X K-6 - 7-11 426 236 15:1 X 41

St.Michael’sCollegeSchool,Toronto,ON X - 7-8 9-12 1,067 870 16:1 X 32

St.Patrick’sRegionalSecondary,Vancouver,BC X X - - 8-12 500 500 X 6

St.Paul’sHighSchool,Winnipeg,MB X - - 9-12 596 596 14:1 X 22

ShawniganLakeSchool,ShawniganLake,BC X X X X - - 8-12 457 457 8:1 X 52

SignetChristianSchool,NorthYork,ON X X JK-8 - 9-12 37 16 5:1 9

TheStudy,Westmount,QC X K-3 4-6 7-11 316 125 8:1 X 30

TrafalgarCastleSchool,Whitby,ON X X - 5-8 9-12 191 134 9:1 X 29

TrinityCollegeSchool,PortHope,ON X X X X - 5-8 9-12 554 450 8:1 X 37

UnitedMennoniteEducationalInstitute,Leamington,ON X X - - 9-12 48 48 15:1 18

UpperCanadaCollege,Toronto,ON X X K-7 8-9 9-12 1,190 650 8:1 59

VentaPreparatorySchool,Ottawa,ON X X X X JK-7 - 8-10 74 20 6:1 14

TheYorkSchool,Toronto,ON X X JK-5 6-8 9-12 600 235 16:1 26

INTERNATIONALColombia

ColegioBolivar,Cali X X PK-5 6-8 9-12 1,258 365 9:1 X 13

Greece

AmericanCommunitySchoolsofAthens,Athens X X JK-5 6-8 9-12 859 342 20:1 X 11

Italy

St.Stephen’sSchool,Rome,Rome X X X X - - 9-PG 278 278 7:1 X 7

Japan

SaintMaurInternationalSchool,Yokohama X X PK-5 6-8 9-12 436 128 4:1 X 6

SeisenInternationalSchool,Tokyo X X K-6 7-8 9-12 634 198 6:1 12

Malaysia

Mont’KiaraInternationalSchool,KualaLumpur X X 1-5 6-8 9-12 1,005 250 10:1 18

Mexico

TheAmericanSchoolFoundation,MexicoCity,D.F. X X 1-5 6-8 9-12 2,571 743 11:1 X 7

InternationalSchoolManila,1634TaguigCity X X PK-4 5-8 9-12 2,200 780 9:1 X 18

Spain

TheAmericanSchoolofMadrid,Madrid X X PK-5 6-8 9-12 868 284 8:1 8

Switzerland

InstitutMonteRosa,Montreux X X X X - - 5:1 X 39

Thailand

InternationalSchoolBangkok,Pakkret X X PK-5 6-8 9-12 1,896 709 10:1 X 15

Turkey

IstanbulInternationalCommunitySchool,Istanbul X X 1-6 - 7-12 578 269 9:1 12

UnitedKingdom

MerchistonCastleSchool,Edinburgh X X - - 174 9:1 46